Летопись МИФИ

Торжественное схождение с ума


ЕГЭ-2024
Тесты ЕГЭ Онлайн
Задачи ЕГЭ по математике
Решения ЕГЭ по математике

Вступительные экзамены и специальности
Фишки для Корума:
а б в г д е ё ж з и й к л м н о п р с т у ф х ц ч ш щ ъ ы ь э ю я
a b c d e f g h i j k l m n o p q r s t u v w x y z

Рейтинг пользователей Корума
Настроение • Модераторы
Темы • Картина дня • Realtime
Прочие фишки:
Нецензурная брань
Народная орфография
Морзянка онлайн • Калькулятор
Анаграммы • Игра в города

Загрузка календаря

Новые записи

20.05Задача про фермера и его кредит
26.01Актуализация сервисов ЕГЭ по математике 2014 года
05.11Поломалось
28.08Смена парадигмы
18.07Как вести себя в приличном обществе, предварительно обмочив штаны
оглавление »

Лучшие записи

1.Математическое порно1563
2.Ответы ко всем задачам ЕГЭ по математике 2010 года793
3.Тесты ЕГЭ Онлайн515
4.Результаты ЕГЭ по математике368
5.Результаты ЕГЭ по русскому языку268

О чем тут?

NX VBAB Webometrics igjhs А1-08 Абитуриенты Бачинский ВКонтакте Ващенифтему Волга Диплом Дрессировка преподов Дума ЕГЭ Жизнь Забабахал Инновации История Кафедра 26 Кларк Корум Лженаука МИФИ МИФИсты Морзянка НИЯУ Нанотехнологии Наука Образование Омоймоск ПЦ Поздравляю Поиск Президент Преподы Приколы Программное обеспечение Рейтинги Русский язык Сессия Смерть Статистика Стихи Сувениринг Тест Учеба Учебные материалы ФЯУ Физтех Фотки Ядерщики матанализ

Комментарии

День памяти
  20 мая 2023 (мифи умер)

Задача про фермера и его кредит
  20 мая 2023 (Алекс)

Математическое порно
  22 марта 2023 (Angleton)

Российский Союз ректоров
  19 февраля 2023 (Hellen Paul )

В помощь юному радисту: Морзянка 1.0
  13 ноября 2022 (Сергей)

Знахари и шаманы в МГТУ имени Баумана
  5 ноября 2021 (монах из кельи)

Зачет по инженерной графике
  24 августа 2020 (Инженерная графика)

Пасынки Вселенной
  18 февраля 2020 (Max Brown)

Финансовая пирамида за 10 рублей
  7 февраля 2020 (Флора Миллс)

База решений задач ЕГЭ по математике
  26 декабря 2019 (Мария)

$kib@t®onЪ
Сейчас на скибатроне
Шедевры
12 октября 2009, 15:46:55

Ответы ко всем задачам ЕГЭ по математике 2010 года

На днях Федеральная служба по надзору в сфере образования начала выполнять свое давнишнее обещание обнародовать все задания ЕГЭ по математике 2009 года.

7 октября начал свою работу сайт «Открытый банк задач ЕГЭ по математике», где уже размещены почти две тысячи задач B1–B12 (первой части ЕГЭ по математике, на которые нужно дать краткий ответ). По заверениям авторов проекта, банк продолжает пополняться.

Всё хорошо в этом банке, кроме одного: ответы к заданиям там не публикуются. Это, кстати, укрепляет мою веру в то, что задания эти — действительно те самые, которые будут включены в единый экзамен в следующем году.

Я бы с удовольствием прорешал для вас, дорогие мои, все 1876 задач, плюс те, которые добавятся позже, но какой я после этого джедай коленописных веб-приложений? Пусть лучше они решатся коллективным сознанием тысяч молодых энергичных людей.

С целью осуществления своего коварного плана я позволил себе потратить пару часиков на слизывалку всех заданий в свою базу данных, а сегодня — еще часика четыре на интерфейс ввода ответов. Теперь каждый может стать частью этого самого коллективного сознания, зайдя в наш местный

Открытый банк
задач ЕГЭ по математике 2010

Нас много, друзья. В системе «Тесты ЕГЭ Онлайн» сейчас зарегистрировано почти 50 тысяч человек. Если каждый потратит несколько минут и решит хотя бы пару-тройку из задач банка, моя система с легкостью определит единственно правильный ответ к каждой из них.

Дело в том, что даже если задача очень сложная, правильный ответ в общей куче предположений все равно встречается намного чаще, чем любой неправильный. Достаточно лишь набрать статистику.

Есть реальный шанс собрать все ответы к ЕГЭ по математике 2010 года, чем черт не шутит.

Пока готов только сам модуль отображения заданий и ввода ответов. Думаю, логично запустить его уже в таком виде, а пока Коллективный Разум работает, я допишу все остальные плюшки — рейтинг решателей, показометр всего процесса выполнения Плана и, собственно, сам отображатель реальных заданий ЕГЭ по математике с ответами.

P.S. Ссылки на этот пост категорически приветствуются. Чем коллективнее будет разум, тем быстрее и точнее будут данные.

Update (03.09.2010). Сейчас в базе более 35 тысяч реальных задач Открытого банка ЕГЭ по математике 2011 года.

Update (03.10.2011). Сейчас в базе более 80 тысяч реальных задач Открытого банка ЕГЭ по математике 2012 года (обновлённый стандарт, 14 заданий в части B).

Update (02.05.2013). Сейчас в базе более 90 тысяч реальных задач Открытого банка ЕГЭ по математике 2013 года (добавлено 3982 задания).
FaceBook

ранее:
И еще кучка онлайновых тестов
далее:
Ассоциативное мышление

Уже 4548 комментариев

вверхвниз
1. Илья, 12 октября 2009, 20:19:45
Хочу сдать егэ хорошо!!!

2. Марика ебала я ЕГЭ!!!, 13 октября 2009, 08:32:27
уфф прикольная идейка...)))

3. Вероника, 13 октября 2009, 11:25:13
егэ(

4. Санек, 13 октября 2009, 16:25:12
Весьма занятная вещица. Затягивает как хорошая игра. Поскольку очень полезно на практике все это применять. А когда хотя бы относительный рейтинг результатов будет?

5. Lexxus, 13 октября 2009, 17:53:26
А когда хотя бы относительный рейтинг результатов будет?

Постараюсь закончить завтра.
Клятвенно обещаю до конца недели =)

6. ITF5000, 16 октября 2009, 11:51:52
Небольшой [:|||||:]

Пластина «Авто» устанавливается под номерной знак транспортного средства. Она действует по двум направлениям: стабилизирует энергетическое поле и оптимизирует информационное поля автомобиля.

http://nanocomfort.ru/catalog/auto/

7. Lexxus, 16 октября 2009, 12:11:21
Пластина «Авто»...

Чуть не удалил как спам=)

Ты бы хоть подходящий пост выбрал для коммента. Вот этот, например.

8. Sash, 16 октября 2009, 15:48:43
В корне - идея распределенных вычислений.
Алексей, видимо тебе это навеяло со студенческих времен,
когда вся группа прорешивает все варианты заранее, а на контрольную идет с готовыми бомбами.

9. Леона, 16 октября 2009, 20:36:49
ЕГЭ ЭТО ЖУТКО

10. ......, 17 октября 2009, 09:08:07
не навижу егэ!!!

11. belkins, 17 октября 2009, 16:24:55
почему этот блог так поехал на егэ? хочу больше информации про мифи, заметок из жизни. егэ можно было бы отправить на отдельную площадку...

12. Lexxus, 17 октября 2009, 19:18:51
заметок из жизни.

Купил зимние шины и диски. Теперь нищий и готовый к зиме...

больше информации про мифи

... и это произошло совсем недалеко от МИФИ %)))

13. .kkursor, 20 октября 2009, 01:51:47
»не навижу егэ!!!
Ещё бы Вы его любили... писать бы грамотно научились, для начала.

PS: как человек, сдававший и успешно его сдавший, могу сказать, что оно ничуть не страшно. Если Ваш интеллектуальный уровень хоть чуточку выше, чем у кроссовка.

14. olga666, 21 октября 2009, 10:11:13
Страшно до ужаса(((Не смотря,на то что наш год самый маленький,т.е. меньше выпускников чем раньше, это не факт,что ты поступишь. Приморский край вообще отстой(Ничего ещё толком не налаженно.Все только и хотят,что сорвать с тебя побольше бабла.Макулатуры дома уже столько!А то что надо того и нет(В нете совсем немного сайтов на которых могут дать дельные советы и нужную информацию.Если знаете такой киньте мне))Пожалуйста.

15. ucheba.pro, 22 октября 2009, 01:11:52
http://forum.albega.ru/viewtopic.php?f=15&t=460
Решения некоторых заданий из всех книжек Ященко

16. vladik2, 22 октября 2009, 18:22:09
Привет всем! Здравствуйте, уважаемый Lexxus!
Я не понимаю условие в задаче:
Задание B10 (№ 6113)
В боковой стенке цилиндрического бака вблизи дна закреплён кран. После его открытия вода начинает вытекать из бака, при этом высота столба воды в нём меняется по закону H(t)=7,2-1,92t+0,128t^2 , где t — время в минутах. В течение какого времени вода будет вытекать из бака?
Так вот, про кран.Что значит-"вблизи дна".
По-моему, не зная точной высоты на которой находится кран, задачу нельзя решить.
Прав ли я?
И еще один вопрос: в банкезадач есть фишка "в корзину".
Не понятно ее назначение.


17. Lexxus, 22 октября 2009, 19:52:25
Так вот, про кран.Что значит-"вблизи дна".
По-моему, не зная точной высоты на которой находится кран, задачу нельзя решить.
Прав ли я?

Ты, конечно, в принципе, прав.
Но в данном случае это значит, что тебе предложено пренебречь высотой, на которой находится кран, и считать, что он расположен прямо на уровне дна.

И еще один вопрос: в банкезадач есть фишка "в корзину".
Не понятно ее назначение.

Мне тоже :)

18. Жень-шень, 23 октября 2009, 08:17:39
что означает фраза "найдите точку минимума функции"? Смущает ответ в виде целого числа или десятичной дроби.
Что писать - абциссу или ординату?

19. Lexxus, 23 октября 2009, 09:44:01
что означает фраза "найдите точку минимума функции"?

"Точка минимума/максимума" - значит, абсцисса.
"Минимальное/максимальное значение" или просто "минимум/максимум" - значит, ордината.

20. vladik2, 23 октября 2009, 13:28:26
Уважаемый Lexxus!
Я все по поводу крана.
А как в условиях экзамена, когда нервы на пределе, додуматься, что авторы задачи имели в виду , что кран на уровне дна?
Если Вы имеете связь с составителями КИМов, то может попросите их уточнить это?

21. Lexxus, 23 октября 2009, 15:15:50
А как в условиях экзамена, когда нервы на пределе, додуматься, что авторы задачи имели в виду , что кран на уровне дна?

У многих преподавателей (в том числе, составителей задач) есть одна общая проблема: они слишком многое считают очевидным не только для себя, но и для всех окружающих. Я в свое время это назвал "преподавательским синдромом".
Совет один: решай больше задач, "набивай руку".

Если Вы имеете связь с составителями КИМов, то может попросите их уточнить это?

Увы, таких связей у меня нет.

22. vladik2, 23 октября 2009, 16:45:43
Привет всем!
Уважаемый Lexxus!
По поводу составителей задач.
Вот задача В10:
Коэффициент полезного действия некоторого двигателя определяется формулой n=(T1-T2)/T1. При каких значениях температуры нагревателя КПД этого двигателя будет больше 30%, если температура холодильника 350 град. ?
Ну, я составил неравенство (Т1-350)/Т1>0,3
Решил его.
Получаю ответ:Т1>500
И вот вопрос: а что же записывать в ответ?
500? 501?
Что хотели те преподы, которые составляли эту задачу?
А еще я среди этих задач В10 нашел задачу о камнеметательной машитне.Там вообще полный атас!
Если кто ее решит выложите решение здесь, пожалуйста.Очень хочется узнать как она решается.
Там вся трудность тоже в записи ответа.Нужно догадаться, что преподы имели в виду.



23. Lexxus, 23 октября 2009, 17:10:08
Получаю ответ:Т1>500
И вот вопрос: а что же записывать в ответ?
500? 501?

Ты, конечно, прав: задача поставлена некорректно.
Но я уверен, что в подобных случаях нужно понимать "более" как "не менее", а "менее" - как "не более". То есть, правильный ответ - 500.

24. Lexxus, 23 октября 2009, 17:33:30
Модель камнеметательной машины, выстреливает камни под определенным углом к горизонту с фиксированной начальной скоростью. Её конструкция такова, что траектория полета камня описывается формулой y=ax^2+bx, где a=-1/10000 1/м, b=1/10 — постоянные параметры. На каком наибольшем расстоянии (в метрах) от крепостной стены высоты 9 м нужно расположить машину, чтобы камни перелетали через неё?

Эта задача? Сложно объяснять, не рисуя, но я попробую.

У нас есть уравнение траектории полета камня:
y=-x^2/10000+x/10
Это перевернутая парабола с "началом" в точке (0,0). Нам нужно найти такое значение x (это и есть расстояние до стены, нарисуй график - увидишь), при котором камень перелетит через девятиметровую стену, то есть y>=9 (на самом деле, и тут та же фигня: неравенство по идее должно быть строгим, но мы условились не заморачиваться по этому поводу).

Получается квадратное неравенство:
-x^2/10000+x/10>=9
А поскольку нам нужно максимальное расстояние, то нам остается найти только больший корень уравнения
-x^2/10000+x/10=9

Корни там нормальные получаются.

25. vladik2, 23 октября 2009, 19:28:36
Спасибо!
Я думаю, что еще не все учителя математики и 11-классники проснулись, думаю, что скоро поднимется "шухер" и эти задачи заставят переделать, чтобы на экзамене не пришлось заморачиваться.
Ведь в этой задаче при ответе 900 камень по идее не перелетит, а врежется в стену на высоте 9м или пойдет по касательной, что кстати тоже не удовлетворяет условию задачи.
В пн озадачу свою уч.математики.Что-то она скажет?
Она у нас молодец, раз в неделю на уроке разбираем задачи группы С и также есть факультстив по этим задачам.Не боится их.Иногда не все получается, но мы ей все прощаем.

26. vladik2, 25 октября 2009, 10:47:02
Что-то тут случилось.Мне не говорят,"здравствуй, vladik2".
Снова зарегестрировался, под № 3.

27. vladik3, 25 октября 2009, 10:51:38
Мне и "здравствуй,vladik3" не говорят.
Обидно.

28. vladik2, 26 октября 2009, 23:31:21
Обидно!Меня мой комп(а может и не комп) подводит.
Меня сайт не видит и не засчитывает мне решенные задачи.
Хоть за эти задачи премии и не будет но все равно с рейтингом интереснее.
Сегодня посмотрел- 22 балла.
Решил 3 задачки, результат тот же!
Обидно.
Кстати, сегодня в школе на уроке решали задачу на наибольшее значение.
Там получаются три ответа: 23, корень из трех +что-то еще, и третий ответ типа двух пи + что-то еще.
Так здесь и дураку ясно нада брать 23 и не заморачиваться больше ни с чем.
Это конечно хорошо.
Но с другой стороны составители-они что не смогли что-то другое придумать?

29. vladik2, 28 октября 2009, 21:05:56
Я написал письмо по поводу задач про кран, КПД и камнеметательную машину на адрес сайта "Банк задач по матем. для ЕГЭ".
Ну и мне написали, что,да, действительно в этих задачах ошибки и идет работа над исправлением этих ошибок.

30. Lexxus, 28 октября 2009, 22:11:37
Я написал письмо по поводу задач про кран, КПД и камнеметательную машину на адрес сайта "Банк задач по матем. для ЕГЭ".
Ну и мне написали, что,да, действительно в этих задачах ошибки и идет работа над исправлением этих ошибок.

Да, я тоже писал. Тоже ответили, что, мол, исправляют и дорабатывают.
Но о задачах части С так ничего и не ответили.

31. Викуля, 28 октября 2009, 23:32:15
уЖАСНО СТРАШНО СДАВАТЬ егэ, занимаюсь, но задания С пугают, как их написать?

32. Екатерина, 29 октября 2009, 09:40:44
Да,согласна,ужасно страшно....
Зачем убрали часть А????Они подумали о детях???Козлы...как сдавать..
Я ненавижу алгебру..

33. тосик, 29 октября 2009, 15:56:35
Я очень боюсь ЕГЭ

34. леонид, 31 октября 2009, 13:25:25
почему большинство аналитических заданий В11 имеют неоднозначные ответы? Как их вносить в бланк? Или что-то будет меняться? Разумна ли затея с проектом в период кризиса? Это же огромные деньги. В графических заданиях есть задания, когда нет решения! Что с ними?

35. леонид, 3 ноября 2009, 19:14:42
Каким образом определяется уровень сложности? В11 задания уровня сложности С1 и С2 егэ пред. лет. Базовый ли уровень задан вами, когда надо найти производную сложной функции, содержащей степенную под логарифмом или с показательной (4055,3869, 4079 и другие) и почти совсем простые (3453 , но оценочные). Как будет формироваться вариант по уровню сложности заданий? Нужны ли абыо-кадатры с корнями и пи в условиях - ловить, видит ли экзаменующися постоянную, коэффициент ? Больше писанины, чем математики!

36. леонид, 3 ноября 2009, 19:19:12
абры - кадабры

37. оля, 8 ноября 2009, 09:02:44
Смысл включения заданий на прямоугольный треугольник и определения триг функций, критикуя насыщенность заданий егэ до проекта лишь- только на память. И тет никакой тригонометрии. А в продаваемых сборниках (Глазков и проект) В4 образца предю егэ. На что ориентироваться?


Где увидеть Ваши ответы?

38. настенька, 9 ноября 2009, 14:23:41
скоро пробник, я в ужасе от того что часть А убрали!!!!!!!!!!!! ВЕРНИТЕ ЧАСТЬ А!!!!!!

39. Nelli, 9 ноября 2009, 14:55:54
написала ЕГЭ на 2!!!!!((( в среду опять пробное...(((

40. Melli, 9 ноября 2009, 14:59:06
напишите кто занимается по книжке ДЛЯ ПОДГОТОВКИ К ЕГЭ автора ЛЫСЕНКО!!!!

41. oleg), 10 ноября 2009, 10:06:11
ну что такое!!!?? почему вы не отвечаете на вопросы людей??
что делать если ты не такой умный,как парень,написавший задачу о камне...... машине???как сдавать ЕГЭ без части А,если только А и мог решить раньше???

42. Lexxus, 10 ноября 2009, 12:22:53
ну что такое!!!?? почему вы не отвечаете на вопросы людей??
что делать если ты не такой умный...

Я не знаю, что делать тем, кто "не такой умный". Задания не я придумывал, знаете ли. И часть А тоже не я убрал.

43. Lili, 10 ноября 2009, 22:17:52
Боже люди!!!! вообще о чем мы все!!!! ЕГЭ вообще какая то сплошная диверсия иностранных спецслужб!!!!!! У меня такое ощущение что они только и хотят, что бы половина наших выпускников окончило школу со справками!!!!! А это же ужасно!!! На этот проект угрохано такое количество денег, что даже представить страшно!!! А все те, кто придумал это, наверное, все таки понимают что то, что они намутили- полный бред!!! А деваться уже некуда... как говорится.."За все уплочено!!!!!" Но я все же надеюсь, что наши министры одумаются и нашим младшим братьям и сестрам не придется столкнуться с такой ужасной и всеразрушающей машиной как ЕГЭ!!!!!!!!!!!!!!!! а нам пока только и остается, грызть гранит алгебры и геометрии и вспоминать нехорошими словами организаторов ЕГЭ!!!

44. леонид, 12 ноября 2009, 06:45:27
в заданиях В4 очень разный уровень сложности - от решений в 1-2 шага до 4 шагов, кроме того встречаются задания с ответами бесконечной десятичной дроби. что по этому поводу? Округлять (это плохие тесты)? Оставлять Обыкновенную дробь - как в бланк её вписывать?

Ваши слова "останутся на тысячалетия" раздражают. Воспринимаются как издевательство.

45. Lexxus, 12 ноября 2009, 09:21:17
в заданиях В4 очень разный уровень сложности - от решений в 1-2 шага до 4 шагов, кроме того встречаются задания с ответами бесконечной десятичной дроби.

Я не замечал ни одного задания с ответом в виде бесконечной дроби. Номера не припомнишь?

Ваши слова "останутся на тысячалетия" раздражают. Воспринимаются как издевательство.

На вкус и цвет все фломастеры разные.

46. Ирина, 12 ноября 2009, 14:59:37
еге это ужасно!какие предурки вообще его придумали?всю жизнь испортили!

47. Ирина, 12 ноября 2009, 15:04:05
А вообще реально найти на каких нибудь сайтах ответы на егэ на 2010г.?кто знает пдскажите!!!!!!!!!

48. Izolda, 12 ноября 2009, 19:06:28
Lexxus, а можно поинтересоваться:
Каков критерий определения правильного ответа. Ну то есть сколько ответов должно быть получено на задачу и сколько должно совпасть, чтобы ответ был засчитан в итоге.
И ещё. Когда я решаю задачи, они мне выдаются в произвольном порядке, или все же те, к которым нет ещё определенных ответов. Подозреваю, что все же произвольно. И если так, то можно ли как-нибудь решать именно те задачи, к которым ответы еще не найдены.

49. Izolda, 12 ноября 2009, 19:15:02
И еще. Мне тут два раза почти подряд попалась одна и та же задача. я не то, чтобы против, но если я ее неправильно решила, то и второй раз могу дать неправильный ответ. А в случае, когда нужно определить правильный ответ, это ухудшает статистику.

50. vladik2, 12 ноября 2009, 21:02:40
Ирина! Ты пишешь:"еге это ужасно!какие предурки вообще его придумали?всю жизнь испортили! "
Даже учитывая, что ты возмущена и не контролируешь себя то позволь тебя поправить. Слово "предурки" пишется через "и".
Извини!
И еще.На сайте, где выложены задачи сказано, что на троечку надо сделать 5 заданий.
Вот задача." Сколько пирожков по цене 13 рублей можно купить на 42 рубля?"
Ирина, разве это сложная задача? Ее может решить даже ученик начальной школы.
А вот еще одна легкая задача: найти площадь фигуры,нарисованной на бумаге в клетку. Да тут просто подсчитать количество кв.см и все.
Это тоже может решить ученик начальной школы.
А вот еще задача на касательную-там по клеточкам просто подсчитать tg угла и все!!!
Ирочка! Не бойся! К маю ты будешь решать эти "задачки" устно.И все у тебя будет хорошо.



51. Ирина, 13 ноября 2009, 08:35:34
спасибо за подсказку.....

52. Lexxus, 13 ноября 2009, 12:11:17
Каков критерий определения правильного ответа. Ну то есть сколько ответов должно быть получено на задачу и сколько должно совпасть, чтобы ответ был засчитан в итоге.

Я пока не могу сказать совсем точно, но с очень большой вероятностью если самый популярный ответ не менее чем втрое более популярный, чем следующий за ним, то он правильный.
То есть, скажем, если ответ А дали 6 человек, а ответ B и остальные варианты - не более 2 человек, то почти наверняка ответ A - правильный.

Когда я решаю задачи, они мне выдаются в произвольном порядке, или все же те, к которым нет ещё определенных ответов. Подозреваю, что все же произвольно. И если так, то можно ли как-нибудь решать именно те задачи, к которым ответы еще не найдены.

Сейчас задачи выдаются совсем произвольно. Но мне уже кое-кто говорил, что неплохо бы сделать возможность выдачи только заданий с пока неизвестными ответами. Я сделаю.

Мне тут два раза почти подряд попалась одна и та же задача. я не то, чтобы против, но если я ее неправильно решила, то и второй раз могу дать неправильный ответ. А в случае, когда нужно определить правильный ответ, это ухудшает статистику.

Пути генератора случайных чисел неисповедимы :)
Ничего страшного, такие случаи слишком редки, чтобы заметно портить статистику.

53. Леонид, 13 ноября 2009, 18:14:23
по поводу бесконечной десятичной дроби в ответе: В4-4657 и В4 - 4665

54. леонид, 14 ноября 2009, 17:06:35
задания В8 № 6045 и 6051
производная функции 3х2 + 2х + 8 = 8 - угловой коэффициент касательной.
х(3х + 2) = 0, х = 0 и х = 2/3. Два ответа и деск. дес. дробь.
то же самое и во многих других заданиях этого типа. Или я , что-то не понимаю?


55. Элина, 14 ноября 2009, 19:20:10
Мне вот интересно, почему на уроках алгебры мы решаем одно, а вот сейчас решаю ЕГЭ, и здесь совершенно другое, не по теме... Можно узнать, по какому принципу составлены задачи к ЕГЭ? И сколько нужно решить заданий на четверку?

56. леонид, 14 ноября 2009, 22:45:12
Уважаемая Элина, такие задания в учебниках встречаются, кроме графических. Посоветуйте учителю не урок по теме брать задания с этого сайта и увидете - всё станет на свои места. Многое подзабыли Вы: диаграммы и графики за 5 класс.
Советую, задачи на клеточки - клеточки не считайте, приём палетки подведет. Используйте формулу площади фигуры. Ещё: значение производной в точке, угловой коэффициент касательной, тангес наклона касательной - это одно и тоже. И т.д.

57. Lexxus, 15 ноября 2009, 05:47:44
производная функции 3х2 + 2х + 8 = 8 - угловой коэффициент касательной.
х(3х + 2) = 0, х = 0 и х = 2/3

Ну, всё правильно. Но там же сказано: "Прямая y=8x-9 ЯВЛЯЕТСЯ касательной". Не параллельна касательной, а является. То есть, помимо равенства производной угловому коэффициенту касательной, у нас есть еще одно условие - равенство значений самой функции и касательной, правильно?

x^3+x^2+8x-9 = 8x-9

Сюда из найденных тобой корней подходит только один - x = 0.
Всё.

58. леонид, 15 ноября 2009, 09:40:52
большое спасибо за ответ. Читать надо внимательно и вдумчиво

59. леонид, 15 ноября 2009, 11:20:14
наши ответы на В8 №6416:2 и 6411:-2.
как быть с концами отрезков в этих заданиях? Как сравнить значения функции на концах отрезков со значением в точке экстремума?

60. vladik2, 15 ноября 2009, 16:47:20
Леонид! По поводу задачи о касательной. Ответ должен быть целым числом или дес. дробью.И если получилось 2/3 то не надо заморачиваться.Просто берешь другое целое число(в даном случае 0) и все.

61. леонид, 15 ноября 2009, 18:12:11
Владик, Вы не правы! Ответ на мой запрос выше. Допусти, встретится ситуация 2 значения целые или десятичные дроби, а брать следует одно из них. Какое? Внимательное чтение задачи - прямая является касательной к графику функции и касательная к графику параллельная заданной прямой ,- разные вещи.

Есть ещё вопрос: брошюра ЕГЭ 2010 новая версия под обшей редакцией Семёнова и Ященко. Как к ней относиться? В банке заданий в В8 такие задания не предлагаются. А в брошюре нет заданий типа В8 из открытого банка заданий.

62. 65607 (Санек), 16 ноября 2009, 15:56:17
Вот решаю, решаю для себя, и как-то решил посмотреть на таблицу "Коллективный разум". И ужаснулся. Я практически в 2 раза больше ошибок делаю, нежели чем соседи. И это при том, что неизвестные мне логарифмы и часть заданий на нахождение наибольшего/наименьшего значения я просто пролистываю, оставляя поле для ответа пустым. Lexxus, если несложно, то выложите общие формулы, которыми надо руководствоваться при решении задач каждого типа (там где формулы вообще есть).
По отмене части "А" могу сказать одно: в новой части "В" 3-4 задания требуют больше общего умения мыслить, а не конкретных знаний. В качестве примера приведу задание В 11, которое встретил:
"Найдите минимальное значение функции f=(16-х)е^16-х"
Пусть знающие люди поправят, но рассуждал я так: Поскольку е - некоторое число(а точнее экспонента), то возводя его в степень положительную или отрицательную, мы в итоге придем к числу. Причем положительному, потому что число положительно, а отрицательная степень вычислятся: е^-х=е^1/х
А поскольку чтобы произведение было равно нулю, необходимо, чтобы один из множителей был равен нулю, достичь этого возможно только приравняв первый множитель 16-х; х=16 (второй множитель никак не равен нулю, так как x^0=1)

63. Lexxus, 16 ноября 2009, 18:06:06
"Найдите минимальное значение функции f=(16-х)е^16-х"

Во-первых, не "минимальное значение", а "точку минимума". Это разные вещи.

Причем положительному, потому что число положительно, а отрицательная степень вычислятся: е^-х=е^1/х

Во-вторых, e^(-x) = 1/(e^x), а не то, что ты написал ;)

А поскольку чтобы произведение было равно нулю

В-третьих, кто тебе сказал, что тебе нужно найти точку, в которой произведение должно обратиться в ноль? Множитель (16-x) может принимать и отрицательные значения, а значит, и всё произведение - тоже.
Подставь x = 17. Получится -1e^(-1) = -1/e. Вполне себе отрицательное число.

Чтобы решить эту задачу, нужно взять производную функции f(x):

df(x)/dx = (d(16-x)/dx)*e^(16-x) + (16-x)*(d(e^(16-x))/dx) = -1*e^(16-x) + (16-x)(-e^(16-x)) = (-1-16+x)e^(16-x) = (x-17)*e^(16-x)

И вот её уже нужно приравнять к нулю. Экспонента, как ты правильно сказал, в ноль не обращается, так что ответ - x = 17.


64. 65607 (Санек), 16 ноября 2009, 18:33:54
[quote]e^(-x) = 1/(e^x), а не то, что ты написал ;)
Только сейчас понял, какую ахинею привнес в тему...
что тебе нужно найти точку, в которой произведение должно обратиться в ноль? Множитель (16-x) может принимать и отрицательные значения, а значит, и всё произведение - тоже

Но тогда и экспонента будет отрицательна. А произведение отрицательных чисел положительно.
нужно взять производную функции f(x):

К стыду своему, не вспомнил про алгоритм решения сразу... Плохо быть двоечником...

65. vladik2, 16 ноября 2009, 23:22:20
Леонид! Конечно ты прав! Лучше знать как решается задача.
Но если получаются 2 ответа, один из которых не впишешь в бланк ответов, и , если ты уверен в ходе решения задачи, то не надо дальше проверять то, что второй ("плохой")ответ не удовлетворяет условию задачи!
Еще пример на эту тему.
Надо найти наибольшее значение функции на отрезке.В ходе решения ты получил три значения:"пи", 2 корня из "пи" и 4.
Тут можно смело писать в бланк ответов 4.
По той простой причине, что два других - не целые числа и недесятичные дроби.
Тут просто экономится время для задач С.

66. леонид, 17 ноября 2009, 03:22:05
Владик, вами приведённые числа - пример неудачный, 4 - явно наибольшее. А наименьшее, вдруг попросят среди них, что в ответе? По Вшему 4.

67. 65607 (Санек), 17 ноября 2009, 09:13:30
Попробую дальше продемонстрировать возможность решения задач без каких-либо особых знаний.
Прямоугольный параллелепипед описан около единичной сферы. Найдите его площадь поверхности. Там даже чертежик был пририсован. Последовательность решения: включить мозги, отключить функцию всплывающих идиотизмов и начать рассуждать. Поскольку сфера единичная, то значит ее радиус - 1, причем она касается всех сторон параллелепипеда, в который вписана. Охота сделать вывод, что сам параллелепипед не то квадрат, не то еще что-то, но воздержусь. Поскольку центр сферы(точка О) по совместительству и.о. точки пересечения диагоналей, то можно сказать точно, что кратчайшее растояние до любой из сторон равно перпендикуляру из О, а он равен 1, то два таких перпендикуляра равны 2. Поскольку параллелепипед прямоугольный, то и сторона его равна 2. Аналогичные размышления провожу над второй и третьей стороной, и прихожу к выводу, что и высота, и ширина, и длина этого параллелепипеда равна 2. Отсюда начинаю считать площадь, которая выходит 2*2*6=24. Надеюсь, что в этот раз все посчитал нормально... Если что не так, поправьте пожалуйста.

68. Lexxus, 17 ноября 2009, 09:45:47
Отсюда начинаю считать площадь, которая выходит 2*2*6=24. Надеюсь, что в этот раз все посчитал нормально... Если что не так, поправьте пожалуйста.

Всё правильно.

69. 65607 (Санек), 18 ноября 2009, 17:43:36
Далее по просторам заданий шагаю...
Найдите корень уравнения: sqrt(4+3х)=-x
Если уравнение имеем более одного корня, укажите меньший из них.

Только сегодня повторяли свойства четных корней, вроде решений на этот случай жизни нет вовсе... Или я снова не учел чего-то, или задание немного некорректно...

70. Lexxus, 18 ноября 2009, 18:05:15
Найдите корень уравнения: sqrt(4+3х)=-x
...
вроде решений на этот случай жизни нет вовсе... Или я снова не учел чего-то, или задание немного некорректно...

А что тебя смущает? Процедура стандартная: возводишь обе части в квадрат, решаешь квадратное уравнение, убираешь то, при котором под корнем получается отрицательное число. Останется нужное решение.

Есть и джедайский способ:
"-x" - строго убывает, "sqrt(4+3x)" - строго возрастает, значит, у уравнения максимум один корень. Корень также всюду положителен, поэтому ищем ответ среди отрицательных чисел. Кроме того, корень - штука обычно иррациональная, поэтому ищем ответ среди чисел вида (-точный_квадрат) - -1,-4,-9...
Подставляем -1, улыбаемся и больше ничего не делаем.
(это только написано многословно, а на самом деле - значительно быстрее, чем решить квадратное уравнение).

У этого уравнения ровно один корень, приписка "если... то укажите меньший" - стандартная фраза для такого рода заданий.

71. vkadik2, 18 ноября 2009, 18:06:36
Санек!
Возводим в квадрат, получаем ур. х^2 -3х-4=0
Корни:4 и -1.
Делаем проверку.
Имеем только один корень: -1.
И записывй это в бланк ответов.
А насчет 2-х корней не заморачивайся.
Сказано:решите уравнение.
Мы его решили. Получили один единств. корень.
А вот если бы получили 2 корня или три корня то в бланк ответов записали бы меньший.

72. Леонид, 19 ноября 2009, 22:03:05
Lexxus, задания графические стали чётче. Молодцы! Но сколько их ещё будет.

И какой всё жу будет формат ЕГЭ? Где официальный документ по этому поводу?

73. Ирина, 21 ноября 2009, 14:28:42
Привет,vkadik2,представляешь у нас в школе был пробный егэ и я сдала на 2.А ты говоришь легкий.4 из 12 написала правильно.А ты вообще как с математикой дружишь?

74. kochkov_i, 21 ноября 2009, 17:20:57
проверьте задачу B1 № 20909
У меня получается бесконечная дробь.
А что в ответ?

75. vladik2, 22 ноября 2009, 01:10:12
Ирина! Что такое математика? Это лучший предмет на фоне химии и физики.
У нас было уже три пробника, уж не знаю, то ли директор сам устраивает, то ли ему указания из департамента шлют, но у нас в классе двоек нет, чесслово.
Но и двенадцать примеров решают стабильно только трое ботаников-математиков.
Впереди полгода, не паникуй!

76. алла, 22 ноября 2009, 20:25:53
Банк пополнился новыми заданиями. Меня поставил в тупик вопрос в задаче 19617. В треугольнике АВС АС=ВС=5, АВ=2sqt(21). Найдите синус внешнего угла при вершине 21. Что имели в виду? В математике нет такого понятия: угол при вершине 21? Подскажите, что спрашивают авторы?

77. Леонид, 22 ноября 2009, 21:09:19
Алла, продли сторону за вершину за вершину получишь внешний углд
К = 180-В, SinK = Sin(180 - B). SinK = SinB ответ готов

78. Ирина, 22 ноября 2009, 21:44:46
Привет,vladik2!А у нас в классе писало 24 человека из них получили 3 только 3 человека,а все остальные написали на 2 в том числе и я!А ты с какого города?

79. алла, 22 ноября 2009, 22:08:11
Леонид, спасибо за ответ. Но ты не понял сути вопроса. За какую вершину? Угол внешний при какой вершине? Что такое угол при вершине 21?

80. Алла, 23 ноября 2009, 20:21:05
Жду помощи. Кто-нибудь ответьте, пожалуйста, на мой вопрос.

81. Ирина, 23 ноября 2009, 22:11:52
Привет,Алла!Если честно я сама впервые это слышу!Мне тоже стало интересно.......

82. vladik2, 23 ноября 2009, 23:57:43
Алла!В равнобедренном тр-ке АСВ ( АС=ВС) проведи высоту СD. По теореме Пифагора находим ее :CD=sqrt(25-21)=2.
Далее находим sinB=2:5=0,4
Ну а sinB равен синусу смежного с ним угла СВК,например.
Ответ 0,4.
А насчет условия не заморачивайся. Не может быть угла при вершине 21.
Тут только угол при вершине В или А или С.
Просто синус угла при вершине С ищется чуть сложнее.
Тут в этой задаче составители что-то напутали.
Думаю, на ЕГЭ такого не будет.
Я даже написал составителям ( адрес на портале единого банка задач) об ошибках. И они ответили, что да, есть ошибки и мы их исправляем.
Я общался с ребятами из др. школ Москвы - результаты где как!
Где хорошие, где плохие.
Не представляю, что бы я делал если бы ввели обязательный ЕГЭ по химии.
Тут в нашем классе были бы одни двойки.

83. Леонид, 24 ноября 2009, 20:02:33
Алла, в 19617 (я нашёл его) вообще ерунда написана, выбросьте из головы и не ломайте её. Ещё пригодится. ПРИВЕТ...

84. Алла, 24 ноября 2009, 20:12:59
Я бы выбросила, но мне эту проблему надо решить! А в задачах 19613, 19619, 19621, 19627, 19631, 19633, 19635 вообще предлагается треугольник, который не существует, это уж совсем не профессионально. Я ждала от авторов открытого банка большей внимательности.

85. vladik2, 24 ноября 2009, 23:51:09
Здравствуйте, уважаемый Lexxus!
Хочу спросить: а как подсчитываются баллы ? Я сейчас решил 43 задачи и на 76 месте. А кто-то решил меньше и он выше?
Премии , конечно не будет, но все же...
Спасибо.

86. Lexxus, 25 ноября 2009, 09:36:32
Хочу спросить: а как подсчитываются баллы ? Я сейчас решил 43 задачи и на 76 месте. А кто-то решил меньше и он выше?
Премии , конечно не будет, но все же...

За любой ответ (не имеет значения, правильный он, или нет) прибавляется один балл. Если этот ответ оказывается правильным (даже если на момент твоего ответа правильный еще неизвестен, а определился потом), то еще 10 баллов.

87. Леонид, 26 ноября 2009, 08:07:35
С какой целью в В7 множитель записан 18,0

88. Леонид, 26 ноября 2009, 08:26:54
Алла, в задачах В7 № 196(31,33,35 и ещё 37) - понимаете мои скобки - тоже глупости: вершины 1,6,21 не могут быть так записаны. Должна быть буква. Молодец! Дерзаете!

89. Алла, 26 ноября 2009, 16:55:44
Спасибо, Леонид! Множитель 18,0 записан в таком виде по правилу записи чисел округляемых до десятых. Видимо, придумывает задачу человек, а меняет параметры компьютер, которому задали условие, что число должно содержать десятые доли.

90. Izolda, 29 ноября 2009, 01:14:14
О, теперь можно выбирать нерешенные и "безответные" задачи.
Спасибо за это!

91. аня, 30 ноября 2009, 09:00:40
Коментарий скрыт (грубость — 19%)показать

92. Ксюша, 30 ноября 2009, 12:44:07
математика ум в порядок приводит.

93. vladik2, 30 ноября 2009, 18:01:02
Некоторые тут начинают выражаться. Это не есть хорошо.
Пользуясь тем, что здесь нет товарищей , которые сначала просматривают, а потом уже помещают наши перлы, они что-то дают себе волю не стесняться в выражениях. И то, что позволительно в шк. курилке( туалете), непозволительно здесь.Так мне кажется.
А насчет математики...Химия и физика -вот это полный абзац.
А математика... Именно на уроках математики мы можем делать десятки своих пусть маленьких открытий. И этим она интересна.
И у меня вопрос ко всем: в инете есть сайт по р.яз., похожий на МИФИстский?

94. Lexxus, 30 ноября 2009, 18:20:33
И у меня вопрос ко всем: в инете есть сайт по р.яз., похожий на МИФИстский?

А что бы ты хотел увидеть на "сайте по русскому языку"?
Здесь вот тоже не по-китайски пишут, ага.

А математика... Именно на уроках математики мы можем делать десятки своих пусть маленьких открытий. И этим она интересна.

Вот только не надо тут делать фетиш из м-м-м... Математики. Да и из любой другой дисциплины.
В нежном школьном возрасте это прямой путь в дурдом.

95. Леонид, 30 ноября 2009, 18:23:48
Химия правая рука физики, а математика её глаз - если не перепутал - Владик, вы толковый парень.

96. vladik2, 30 ноября 2009, 20:13:09
Уважаемый Lexxus!
По поводу сайта по русскому.
Вот скажите мне пожалуйста, что такое тропа? Не в лесу и не поле, а в русском языке? Эту тропу искал по всему инету,нашел, но до конца не понял, что же это такое.
А вопрос нашел из книжки для ЕГЭ.
И может кто-то создал сайт, где можно тренироваться по таким вопросам.
Ну, в общем, я бы потренировался.И может там есть такой же умный филолог, как Вы,Lexxus, математик.
Я пытался пройти тест на грамотность, но, уважаемый Lexxus, уж очень мало времени дается на его прохождение, не успеваешь печатать.Может добавите время? Ну хотя бы на 10 секунд на каждое слово?



97. Леонид, 2 декабря 2009, 15:07:07
Уважаемые, В8 № 6059 не решается у нас. производная функции равна угловому коэффициенту прямой - получается квадратное уравнение с отрицательным Д. Просим ответа.

98. Lexxus, 2 декабря 2009, 16:32:46
Уважаемые, В8 № 6059 не решается у нас. производная функции равна угловому коэффициенту прямой - получается квадратное уравнение с отрицательным Д. Просим ответа.

Неправда ваша. Всё там чудесненько получается.

Какое квадратное уравнение получилось?

99. vladik2, 2 декабря 2009, 17:17:32
Леонид!
Твоя задача:"Прямая у=-6х-2 является касательной к графику функции у=x3-5x2+x -5 . Найдите абсциссу точки касания."
Производная равна 3х^2-10x+1.
Эту производную надо приравнять к угл. коэф. прямой(касат.).
Получим кв. ур., 3х^2-10x+7=0 , корни которого 1и 7/3.
Ну и все . Можно записывать ответ.Какой? Рассуждаем так: в бланк ответов 7/3 не запишешь.След-но, ответом явл. число 1.
Ну а если бы были не такие числа, н-р, 1 и 2.Тут уже надо знать, что прямая явл. касательной к графику функции в точке хо, если выполняютя 2(два!!!!) условия: f штрих в точке хо равен угл. коэф. касат.и значение функции в точке касания равно значению касательной в точке касания.
1) у(Хо)=f(Xo)
2) f"(Хо)=К

100. Леонид, 3 декабря 2009, 09:50:05
Владик, Вы прав! Произошдо недоразумение в вычислениях. Всё потом получилось. Поторопился обратиться. Но Ваше изложение пойдёт на пользу другим (кто читает). Спасибо за Ваши активность и соучастие. Если не снкрет: "Вы выпускник, и какого города?"

101. vladik2, 3 декабря 2009, 17:40:26
Леонид! Есть в Москве район Алексеевский(ВДНХ).
Так я оттуда.
Хотя это интернет и я могу быть отовсюду.Даже из Владивостока.
Наша математичка сказала, что с группой В пока закончим и до марта ее трогать не будем. И все силы надо бросить на С.
Она пообещала что С1,С2и С3 мы будем решать. Может не так уверенно как В,
но будем.
Жалко, что в инете нет сайта где так же можно вопросы задать насчет непонятки с группой С.
Надо искать репетитора хорошего Один был, но он дороговато брал, хотя объяснял толково и требовал много. Однажды я пришел без дз , он и заниматься не стал.
Иди и делай.Потом придешь.

102. Леонид, 4 декабря 2009, 06:37:40
Привет. Владик, что требовал- это неплохо. Вы в заданиях ориентируетесь хорошо. Но нужна и натренированность. Как результат в мониторинге 19.11?
А за репетиторство что берут? Нормально что стоит час? Математик наверное выбрала стратегию такую из-за хорошей подготовки ребят.

103. кто-то, 4 декабря 2009, 09:31:50
http://ucheba.pro/
вот неплохой сайт , где разбираются все задания егэ по математике, и не только ... там помогают с решениями заданий, этот сайт очень сильно мне помогает готовиться к егэ , рекомендую его всем

104. 65607 (Санек), 4 декабря 2009, 16:44:45
Lexxus, от дши благодарю тебя... Порешав задания, выложенные тобой, я намалевал лучше всех в классе пробный ЕГЭ(37%). Во вторник будет очередное испытание, надеюсь получится не хуже... ПО части С никаких новостей не появляется?

105. Lexxus, 4 декабря 2009, 16:52:11
Lexxus, от дши благодарю тебя... Порешав задания, выложенные тобой, я намалевал лучше всех в классе пробный ЕГЭ(37%).

Спасибо за добрые слова. А, кстати, твои 37% - это сколько решенных заданий (и какие конкретно)?

ПО части С никаких новостей не появляется?

К сожалению, нет. Да и если они появятся, мне придется крепко подумать, как сделать систему их "коллективного решения". Хотя, кое-какие задумки на этот счет имеются.

106. vladik2, 4 декабря 2009, 21:04:21
Побывал я на сайте http://ucheba.pro/
Должен сказать: задумано хорошо, но все сделано не очень.
Думал найти там десятки пробников по р.яз. Ничего подобного.Их там нет. разбирается один со словами"Дипломат, дипломант". Так это мы еще в 9(10) классе видели на ур. р.яз.
В общем пробников по р.яз. НЕТ.
Задачи разбираются, но с не очень понятной кодировкой.
Я бы на месте организаторов разбил этот сайт на несколько: по матем.,русск., литер., ну и тд.
А то всех собрали и ничего путного.
НО!!! Если хочешь получить решение какой-то задачи С (серия В там вообще не разбирается) то в принципе тебе ее решат.
Леонид!
Всем в нашем классе надоели задачи В и наша препод это просто почуствовала.Последний пробник ( ими уже просто зободали) написали все-
процент и баллы не подсчитывали, просто оценки.Двоек нет.Наша сказала, что два поставит если не будет решено 6 бэшных задач.
А %, она наверное не умеет считать.


107. Алла, 4 декабря 2009, 22:32:46
Хорошая шутка - учитель математики не умеет считать процент. Просто написали наверное хорошо, вот вас и нехотят расслаблять перед экзаменами слишком хорошим результатом

108. Леонид, 5 декабря 2009, 08:19:36
Привет Влад! Мне тоже что-то не осень предложенный сайт.
Ваша будет включать, наверное, бешные задания по мере прохождения программного материала. В них тоже встречаются вещи не для каждого. Банк пополняется усложнёнными задачами.
Если сравнить мониторинговые С, то в брошюрах Якушева ... задания сложнее. Это тоже о чём-то говорит.

109. 65607 (Санек), 5 декабря 2009, 14:48:04
Мои 37% - это вся часть В за исключением 4 и 7 заданий, а также половина С2. Подробнее -по 3% за ответ в части В и по 7 - за балл в части С (С1-1 балл мах, С2, - 2, С3 и С4 -3, С5 и С6 - 4.
По поводу учебы.ру - как-то сумбурно там, непонятно. Чтобы решать С6 достаточно просто выучить до 3 курса программу ВУЗа))) Задания на mathege и впраду усложняются, приходится напрягать мозга... Но ничего, уже скоро решим все, и будем учить ответы))

110. vladik2, 5 декабря 2009, 18:05:40
Леонид! Привет!
Мы с нашей математичкой по алгебре почти все изучили (остались интегралы первообразные и площади, как она сказала) еще в 10 классе. По геометрии осталось тоже немного.Теперь у нас мин. теории, забыл когда уже теорему с доказательством учил. Только задачи. И нам все это очень нравится.

Санек! Если вам объясняли как будет подсчитан итоговый балл по математике на ЕГЭ объясни еще раз, чего то я не понял. В1-В12 могут дать 36 баллов.Так?
И я не понял, н-р я решу С1,С2,С3. Сколько я за них получу? По твоим объяснениям 6 баллов. Но это же неверно.В общем если нетрудно...

111. катрин, 5 декабря 2009, 22:17:41
Капец как достали эти тесты эти егэ.

112. ксюха, 6 декабря 2009, 07:01:24
катрин, а ты не смотри на них. Занимайся тем, что не достаёт.

113. 65607 (Санек), 6 декабря 2009, 10:31:15
To Vladik2: Может я не совсем понятно объяснил, повторю еще раз...
В1-12 по 1 баллу
С1 - 1 балл.
С2 - 2 балла
С3,4 - 3 балла максимум за каждое
С5,6 -4 балла за каждое.
Вычисления я производил на основе таблицы, где все учащиеся в параллели были запианы в алфавитном порядке... Кому интересно могу эту таблицу экспроприировать и сфотографировать....

114. Лёша, 6 декабря 2009, 13:08:21
С1 и 2 по 2 балла возможных, С3 и 4 по 3 возможных и С5 и 6 по 4 в0зможных. Это может быть от 0 до возможныхю.- первичных.

115. Леонид, 7 декабря 2009, 09:39:41
Уважаемые Lexxus, я уже обращался по поводу В4 №№ 4657 и 4665. Посмотрите - ответы бесконечные десятичные дроби.

116. Lexxus, 7 декабря 2009, 10:00:13
Уважаемые Lexxus, я уже обращался по поводу В4 №№ 4657 и 4665. Посмотрите - ответы бесконечные десятичные дроби.

Действительно, косяк. Спасибо, будем чистить.

117. 65607 (Санек), 7 декабря 2009, 11:52:31
Вопрос ко всем: как оформлять решение С1?
Приведу cистему и ответ к ней:
sin y=x-6
cos y=x-7
при у=0 х=6
Пожалуйста, кого не затруднит, хоть в общем виде, хоть конкретно к этой системе распишите...

118. Lexxus, 7 декабря 2009, 12:47:14
Приведу cистему и ответ к ней:
sin y=x-6
cos y=x-7
при у=0 х=6

Начнем с того, что ответ у тебя неправильный. Правильный ответ, насколько я соображаю,
{x=6, y=пи+2пи*n}, {x=7, y=пи/2+2пи*n}

А решение - например, таким образом:

Мы знаем основное тригонометрическое тождество: sin^2(y)+cos^2(y) = 1.
Возводим оба уравнения в квадрат и получаем:

(x-6)^2+(x-7)^2 = 1

Решаем это уравнение и получаем два корня - x=6 и x=7.

Поставляем в исходную систему x=6:
{sin y = 0, cos y = -1}, отсюда y = пи+2пи*n

Поставляем в исходную систему x=7:
{sin y = 1, cos y = 0}, отсюда y = пи/2+2пи*n

119. 65607 (Санек), 7 декабря 2009, 13:22:17
Lexxus, very thank you!
Надеюсь что завтра смогу решить аналогичное уравнение по предложенному тобой алгоритму.

120. Леонид, 7 декабря 2009, 21:58:16
Уважаемые, Lexxus! Дайте ответ по поводу заданий В4, где вместо обозначений буквенных написаны числа - речь о внешних углах. Запросы, вопросы и переписку вы смотрели смотрели с 22.11.09 (Алла) и мои подтверждения. Там указано ряд номеров задач.

Ждём Ваш отклик.

121. Lexxus, 7 декабря 2009, 22:29:52
Задачи 19613, 19615, 19617, ..., 19637 удалены. В них, действительно, какой-то бред.

122. Леонид, 8 декабря 2009, 07:23:47
Алла, я же Вам говорил, не ломайте голову, она ещё пригодится. Вы довольны - читайте выше ?

123. Алла, 8 декабря 2009, 15:42:03
Да, Леонид, я довольна! А еще больше довольна тем, что нас читают и прислушиваются, т.к. в банке стали исчезать некорректные задания. Призываю всех, если видите проблемные задачи, пишите коментарии к ним, к нам прислушиваются!

124. Я, 8 декабря 2009, 20:48:02
Говорят, что в нэт хотят выкинуть и часть C, это реально?

125. Леонид, 8 декабря 2009, 21:18:41
Привет, дружище Владик!
Что - то не даёшь о себе знать? Есть вопрос: Вы прорешивали задания С2 "Тематическая рабочая тетрадь" авт. Ященко, Шестаков, Захаров?
Что - то некоторые мои ответы не сходятся с авторскими. Там некоторые задачи в части построения приличной сложности (линейного угла двугранного угла, например ДР 9, с хорошими заморочками), ДР5 несложная - ответ не идёт, тоже ДР8. Несложно, но интересно указать расстояние между скрещивающимися прямымы в ДР6. ДР - Диагностическая работа.
Не находите ли Вы задания типа С, как и В где-то в инете? Подскажите.

"Я", что такое "нэт" и от куда эти слухи?

126. Lexxus, 8 декабря 2009, 21:37:18
Говорят, что в нэт хотят выкинуть и часть C, это реально?

Собственно, не менее реально, чем часть B)

"Я", что такое "нэт" и от куда эти слухи?

Я полагаю, имеется в виду малоизвестная разработка родом из суверенного Пендостана под странным названием "Internet" %)


127. Леонид, 8 декабря 2009, 23:32:49
Друзья? Lexxus! С какой целью в появившихся заданиях В6 - многих - параллелограмм, ромб, прямоугольник, квадрат вы именуете трапецией? Корректно ли это? Если уж ставите проблему узнавания фигуры, не лучше ли в этих номерах заданий употребить понятие "четырёхугольник"?
Вы можете и оспаривать. Но стоит ли это делать?

128. vladik2, 8 декабря 2009, 23:57:17
Леониид!
Видел я эту книжку у наших ботаников.На уроках ее не рассматривали.А самому- что то пока нет настроения.
В данный момент ищу пробники по русскому.
Нашел пока 2 на сайте ctege.org
У меня это идея фикс.
И еще насчет группы С.
В интернете думаю этих задач не будет.
Еще мне кажется , что задач С похожих на те , что в демо выложены, не будет.
Будут другие, совершенно не похожие на эти, для решения которых надо знать другие алгоритмы решения. А задача С6 в демо-я даже ее решения не понимаю.
Это я повторяю слова нашей математички.
Она объяснила порядок наших действий: разобрать все уравнения и неравенства ВУЗовского уровня. И тогда есть шанс решить 2 задачи С.
На факультативе будет разбирать параметры. Ну и на геометрии тоже все эти задачи , вроде тех ,что ты перечислил.
Я вот сижу и думаю: нужны мне эти параметры или нет?А скр. прямые и углы между ними?
Ходить на факультатив или нет?
И насчет баллов.
В1-В12 по баллу. С1,С2-по 2 балла, С3,С4-по 3 балла, С5,С6-по 4 балла.
За всю работу можно получить 30 баллов.
100% : на 30 баллов. Получаем, что 1 балл=3,33%
Я,н-р, решу 12 бэшных и С1, С2. Набираю16 баллов.
Умножаю на 3,33. Получаем мой сертиф. балл:53,28.
Очень мало. Для нрмального эконом.института очень мало.
И это очень печально.

129. Леонид, 9 декабря 2009, 07:08:56
Привет Влад! Нет, баллы считают не по пропрциональному принципу. У них есть какая-то формула расчёта. Вы парень толковый и надо ставить цель, как минимум до С4 включительно. Часть 2 выгодна тем, что она прписывается. При наличии верных мыслей, пусть будет с ошибочкой, но баллы будут. С2 считаю, нужна натренированность в построении угла и расстояния между скр. прямыми, расст. м/у парал. плоскостями, линейного угла двугранного, расст. от (.) до пл. А решения несложны, почти на уровне В4- только увидеть надо.
До С2, как пишите Вы, - это будет в пределах 70 сертификатных.
Дело и в том, что с появлением новых бэшек, идёт рост усложнения. А это время, некоторые задания вычислительные, громоздки, даже,если владеешь приёмами вычисления. Мне кажется, охмырнётся Россия двойками, по сравнению с прошлым годом. Пишите - интересно общаться по делу.
Что там за слухи пишет "Я" выше о "нэт" - москвичи должны знать.

130. Алла, 9 декабря 2009, 16:13:17
Владу и Леониду! Попробуйте при решении С2 использовать координатный метод, вам понравится, задача сразу станет простой!

131. Леонид, 9 декабря 2009, 19:05:45
А что, Алла, - это идея! Не готов сразу на окончательный вывод, но на прямоугольный параллелепипед (куб), наверное хорошо пойдёт. Спасибо за идею, надо опрбировать. Достаточно там будет школьной теории? В любом ли случае это удобно? Фигуру - то всё равно надо изображать - дело всё в этом, а решения несложны.

132. vladik2, 9 декабря 2009, 19:41:26
Леонид!
Насчет названия четырехугольников.
Это не админ Lexxus задачи составляет.
Это господа составители КИМов называют квадрат трапецией.
Кстати, по ф-ле площади трапеции вполне можно посчитать и пл. квадрата( ха-ха). Просто на эти "ашипки" не нужно обращать внимание.
Спрашивал сегодня в школе , что такое "нет". Никто не слышал.
Сказали :"А ты забей в поисковике"(ха-ха).
И еще : формулу подсчета баллов где нибудь видел? Или это государственная тайна?
Если бэшки и С1,С2 будут весить 70 баллов то это уже неплохо.
( Я на ты, не пиши здесь Вы, все мы одного возраста, наверное один Lexxus здесь постарше)

133. Леонид, 9 декабря 2009, 20:31:18
Влад!
Формулу подсчёта не открывают, так было до этого года, видимо и сохранится.
Когда собирают результаты ЕГЭ на проверку, устанавливают решимость каждого задания задания по России (уровень сложности) - от сюда и расчёт баллов. Например, по России с В1 справились 92% выпускников , а В2 - 84% и т.д. Всё это в неоткрытую формулу - и результат.
Но читая тебя, ещё раз - цель бери не ниже С4, у тебя получится. И будет окей.
На "ашипки" - шутка, я не согласен. Там пишут найти площадь фигуры ограниченной линиями. А здесь геометрическая трактовка - в В6.
Да, можно. Но, нужно - ли? Смысл в смекалке на ЕГЭ

134. Егор, 10 декабря 2009, 17:58:46
Люди, скажите пожалуйста, если я набрал 16 первичных баллов из 30, то сколько это будет в 100-бальной системе? Хотя бы примерно...

135. Алла, 10 декабря 2009, 18:11:46
Это будет зависить только от того как сдадут экзамены остальные выпускники. Сначала собирают результаты по стране, а потом выстраивается шкала оценивания. И до экзамена никто не сможет сказать сколько "стоит" каждое задание. Да и какая разница сейчас сколько вы набрали баллов? Чем больше, тем лучше!!!!!!!!!!! Вперед и с песней!!!!!!!!!!!!

136. ОNV, 10 декабря 2009, 18:54:18
Посмотрите здесь:ЕГЭ-тренер, есть видеоуроки и С2-С4 http://egetrener.ru/view_zadachi.php?zadacha=C4

137. Я, 10 декабря 2009, 18:58:16
Леонид "Нэт"-это интеренет))

Lexxus хех, это хорошо)))

138. Леонид, 11 декабря 2009, 15:49:23
Lexxus! К записке от 8.12. Атанасян... Просвещение, 2008, стр. 103 "Трапецией называется чет-к, у которого две стороны параллельны, а две другие стороны не параллельны". "Параллельные стороны трапеции называются её основаниями, а две другие стороны - боковыми сторонаи".

По идее, по формуле площади трапеции можно находмть и площадь треуголтника : 1/2(а + 0)h ....

139. Lexxus, 11 декабря 2009, 23:24:50
Lexxus! К записке от 8.12. Атанасян... Просвещение, 2008, стр. 103 "Трапецией называется чет-к, у которого две стороны параллельны, а две другие стороны не параллельны".

Мне трудно спорить с авторитетом Атанасяна, но мне лично кажется, что это определение убого, поскольку требование непараллельности двух других сторон - бесполезное ограничение общности понятия.

Но в любом случае на решение задач это никак не влияет :)

140. Леонид, 12 декабря 2009, 06:24:51
Неслишком ли высокомерно об убогости определения?

141. 65607 (Санек), 12 декабря 2009, 10:34:19
Lexxus, опять к Вам с вопросом обращаюсь. Если несложно, nо просветите меня, а заодно и всех остальных, как решается С5 и С6.
С5
Найдите все значения а, при каждом из которых решение неравенства |3x-a|+2=(больше либо равно)<|x-4| образуют отрезок длины 1
С6
Найдите все пары целых чисел (х,у), удовлетворяющие системе
2х^2+2y^2+24x-28y+167<0
x+2y<15/2

142. vladik2, 12 декабря 2009, 15:10:00
Санек!
1-я твоя задача непонятно написана.
А решение второй по моему такое.
После преобразований получим, что первое неравенство будет таким:
(х+6)^2 + (y-7)^2<1,5
И тогда можно нарисовать все эти точки координаты которых будут удовлетворять этому неравенству-это будут точки внутри окружности с центром (-6, 7) и радиусом r=корень из 1,5.
Внутри этой окружности есть всего 5 точек с целыми координатами:(-5,7),
(-6,8), (-6,7), (-6,6) и (-5,7)- нарисуй картинку -увидишь.
И теперь среди этих точек нужно отобрать те которые удовлетворяют 2-му неравенству. Делается это простым подбором.Такая точка только одна-(-6,6).
Ответ:(-6,6)

143. Алла, 12 декабря 2009, 15:36:09
Санек,
если я не ошибаюсь, решение задачи С5, написанной тобой, есть в видиоролике egetrener.ru/ решение классное, посмотри

144. Lexxus, 12 декабря 2009, 16:18:42
С5
Найдите все значения а, при каждом из которых решение неравенства |3x-a|+2=(больше либо равно)<|x-4| образуют отрезок длины 1

Во-первых, все-таки, "меньше либо равно". Если "больше либо равно", то таких значений параметра вообще нет.
А во-вторых,
Решение C5

145. Леонид, 12 декабря 2009, 16:33:27
При решении тестов из раздела "нерешённые" выдаои задание В8 (без номера) "определить промежутки убывания и указать длину наибольшего из них" - два отрезка и оба длиной по 4 ???

146. Lexxus, 12 декабря 2009, 16:37:07
При решении тестов из раздела "нерешённые" выдаои задание В8 (без номера) "определить промежутки убывания и указать длину наибольшего из них" - два отрезка и оба длиной по 4 ???

У многих B8 вообще проблемы с однозначностью ответа. Ответь "4" и посмотри его номер.

147. Леонид, 12 декабря 2009, 17:09:28
По В8 графические о длине наибольшего нашёл в просмотре, кроме того задания - №№ 8427 и 8439. Может быть есть ещё?

148. Леонид, 12 декабря 2009, 17:32:53
Lexxus! Почитайте и формулировку (график) В8 №8503 - о промежутках возрастания. Как отвечать на вопрос? Один промежуток закрыт, другой не закрыт - внешне меньшей длины, но фактически её не имеющий. Что делать?

149. vladik2, 12 декабря 2009, 18:30:35
Леонид! Привет!
По поводу №8503. Там два промежутка убывания:[-7;-4] и [8;9). На этих проомежутках ф. и убывает.А первый имеет наибольшую длину=3. А второй... ну тут ясно, что у него длина меньше. В ответе пишем 3.
А вот если бы вопрос стоял о поиске длины наименьшего промежутка то это и была бы "ашипка" составителей.

150. Леонид, 12 декабря 2009, 19:51:06
Привет! Влад!
Посмотри В8 №№ 8513 и 8361 ???
И ещё, то ли нечёткость графика, то ли берём В8: 8259 х=4, 8273 х= 2, 8251 х=2?
Как успехи в мониторинге или вы не участвуете?

151. Леонид, 12 декабря 2009, 21:23:04
Lexxus! Извиняюсь за назойливость. Выше уже обращался по В8 №№ 8427 и 8439. В первом два отрезка по 4. Указать наибольший, в другом 2 по 3 - указать длину наибольшего. В разделе "решённые" вы высвечиваете ответы 4 и 3 соответственно!!!!. Считаете это нормальным, коль в ответах?
Такое может случиться и на ЕГЭ?

152. Алла, 12 декабря 2009, 21:46:13
Привет, Леонид! Что тебя смущает? Если два промежутка имеют одинаковую длину, значит они оба будут считаться наибольшими (конечно если больших промежутков нет)

153. Леонид, 12 декабря 2009, 22:15:25
Рад встретиться, Алла! Что значит "смушает"? Смысл в таких заданиях? Дело в том, что есть, правда с открытой точкой. Есть понятие валидности и грамотности тестов. А это загадки для ВЕЧЕРИНКИ - моё мнение. Безграмотны!

154. vladik2, 12 декабря 2009, 23:20:03
Леонид!
В этих задачах-ошибки.
Да,действительно, в них получаются по два промежутка одинаковой длины.
Но я бы не стал заморачиваться.
Написал бы в ответе 4 и 3.
Пробниками замучили. У нас уже как-то и интереса нет. Я бэшки все решил.И сдал.

155. Леонид, 12 декабря 2009, 23:41:14
Ну м хорошо, брось их пока, но пиши здесь. Тренируйся на С2, С3 и С4 для уверенности. Ну, не могу я согласиться, что из 4 и 4 наименьшее - зто 4. Пока, у нас уже 3 ночи - в Сибири. Завтра выходной.

156. Lexxus, 13 декабря 2009, 03:49:03
Lexxus! Извиняюсь за назойливость. Выше уже обращался по В8 №№ 8427 и 8439. В первом два отрезка по 4. Указать наибольший, в другом 2 по 3 - указать длину наибольшего. В разделе "решённые" вы высвечиваете ответы 4 и 3 соответственно!!!!. Считаете это нормальным, коль в ответах?
Такое может случиться и на ЕГЭ?

Во-первых, я еще раз напомню, что к составлению задач я никакого отношения не имею. Но догадываюсь, что происходит это так: сначала человек составляет шаблон, а потом компьютер по этому шаблону генерирует задания - подставляет разные числа, графики, меняет слова "возрастание" и "убывание".
Не совсем грамотное составление таких генераторов заданий приводит к тому, что некоторые задания получаются неоднозначными или вовсе не имеющими решений. Я, честно говоря, боялся, что такое будет встречаться даже гораздо чаще, чем оказалось на самом деле.

Что касается заданий 8427, 8439 и подобных, то тут и вовсе всё просто, несмотря на действительно кривоватую формулировку. Если у нас два одинаковых отрезка, а нас просят указать самый длинный, то понятно, что указав в ответе длину обоих отрезков, мы точно не ошибемся - хотя бы потому, что больше никаких вариантов быть не может.

157. Леонид, 13 декабря 2009, 09:01:32
Спасибо, Lexxus! Уберите "кривоватые" формулировки и все прблемы. Слишком не чисто получается: параллелограмм, треугольник, квадрат, прямоугольник, ромб - это трапеция; среди равных чисел выбрать наибольшее или наименьшее. Что значит указать длину обоих отрезков? Пусть авторы проверяют своих генераторов заданий и "плохие" отбрасывают. Не нравятся такие допущения и обобщения. Это ЕГЭ - эта часть рассчитывается на каждого выпускника. Должно быть всё как положено. Ваше согласие успокаивает. Добейтесь, пожжалуйста, чтобы авторы чистили. Не надо ибижаться, Вы же просили номера.

158. Я, 13 декабря 2009, 21:18:19
А кто-нибудь может мне помочь решить кое-что не связанное с этим разделом?..

159. Lexxus, 13 декабря 2009, 22:00:33
А кто-нибудь может мне помочь решить кое-что не связанное с этим разделом?..

Вот разберемся.
"Кто-нибудь" - под этим разумно понимать любую особь нашего вида. Если верить во множественность обитаемых миров с разумной жизнью на борту, то любое разумное существо.

"кое-что не связанное с этим разделом" - очевидно, что вопросов, не связанных с этим разделом - бесконечное множество. Хотя и, полагаю, счетное.
Также очевидно, что множество решенных вопросов (иначе говоря, тех, ответы на которые известны хотя бы одному разумному существу) принципиально конечное.

Значит, не зная и не предполагая ничего о сути твоего вопроса или стоящей перед тобой проблемы, можно положить вероятность того, что найдется хоть одно существо, способное тебе помочь, равной нулю.

Так что ответ, к сожалению, - нет.

С другой стороны, почему бы не попытаться? :)

160. Slam, 13 декабря 2009, 23:17:23
найдите наибольшее значение функции y=12x-2sinx+3 на отрезке [-пи/2;0]
задание b11 (3473) Объясните пожалуйста как это решать!=)))))

161. Lexxus, 13 декабря 2009, 23:34:16
найдите наибольшее значение функции y=12x-2sinx+3 на отрезке [-пи/2;0]
задание b11 (3473) Объясните пожалуйста как это решать!=)))))

Сначала нужно взять производную:

12-2cos(x) - она всегда больше нуля, а значит, функция y(x) - строго возрастающая. А это значит, что на отрезке [-пи/2;0] она достигает максимального значения в правой границе этого отрезка, в точке 0.

y(0) = 12*0-2*sin(0)+3 = 3

А вот если оказывается, что производная имеет нули на заданном отрезке, то нужно вычислять значения функции на обоих границах отрезка и в точке (или точках) экстремума, а потом брать наибольшее из получившихся.

162. vladik2, 14 декабря 2009, 01:41:29
Уважаемый Lexxus! Объясните , что такое бесконечное счетное множество?
Разве бесконечное мн-во можно сосчитать?
Вот Вам и вопрос не связанный с этим разделом!!!

163. Lexxus, 14 декабря 2009, 09:32:59
Уважаемый Lexxus! Объясните , что такое бесконечное счетное множество?

Если поступишь в нормальный технический ВУЗ, тебе по-любому придется это узнать :)

164. Я, 14 декабря 2009, 11:14:37
Lexxus и кто же готов попытаться?)))

165. Алла, 14 декабря 2009, 11:21:35
Привет! Lexxus!
Прочитала реплику-ответ на вопрос "я" от 13 декабря.
Браво!!!!!!!!!!!!!

166. Slam, 14 декабря 2009, 19:26:59
Уважаемый Lexxus идругие участики этого сайта! Объясните и решите пожалуйста вот эту задачу:
С1 СИСТЕМА!
(2*x^2-5*x-3)*под корнем cos y=0
sin y=x Это СИСТЕМА!


P.S *-умножение!!!!!!!!

167. Slam, 14 декабря 2009, 19:49:57
Lexxus спасибо за предыдущий ответ=))))))!

168. vladik2, 14 декабря 2009, 23:38:18
Slam! Очень трудно написать решение, но попробую.
Система разбивается на 2 системы.
1-я.
2х^2-5x-3=0
siny=x
2-я
Корень cosy =0
siny=х

В 1-ой системе корни уравнения:3 и -0,5
Но siny не =3. Значит sinу=-0,5
Получается пара (-0,5; минус 1 в степени n+1 умноженное на пи/6 + пиэн.)
Вроде все хорошо, но косинус стоит в системе под корнем и поэтому он должен быть > или = 0.
С помощью круга отбираем нужные числа: минус 1 в степени n+1 умноженное на пи/6 + 2пиэн.
Итак первая пара :(-0,5; минус 1 в степени n+1 умноженное на пи/6 + 2пиэн.)
Вторая система:
cosy=0, y=пи на 2 + 2пиэн
Если cosy=0, то siny=+ - 1.
Сл-но, х=+-1.
Вторая пара:(1, пи на 2 +пиэн)
Третья пара: (-1, пи на 2 + пиэн)
Очень трудно все это так расписать.
Надеюсь ошибок нет


169. vladik2, 14 декабря 2009, 23:47:15
Бесконечное множество называется счетным, если его числа можно пересчитать,т.е. можно найти такой закон(порядок, алгоритм,шаблон, правило...) что каждому числу бесконечного множества можно поставить в соответствие натуральное число.
Простейший пример- это само мн-во нат. чисел.
Lexxus! Это мне объяснил один студент.
У меня вопрос: он правильно рассказал?

170. Lexxus, 15 декабря 2009, 01:06:07
Lexxus! Это мне объяснил один студент.
У меня вопрос: он правильно рассказал?

Вполне.

Slam! Очень трудно написать решение, но попробую.
Система разбивается на 2 системы.

...

Очень трудно все это так расписать.
Надеюсь ошибок нет

Кажись, правильно. Позже гляну пристальнее.

171. Леонид, 15 декабря 2009, 07:35:34
Привет, Влад! Всё хорошо, на мой взгляд. Первая пара, думаю -0,5 и
-пи на 6 + 2пи n -из 4-ой четверти

172. Slam, 16 декабря 2009, 08:52:59
такой легкий пример но не могу решить=(((((( по каким формулам это решается?
(28sin72гр*cos72гр)/sin144гр
гр-градус
№17301

173. Леонид, 16 декабря 2009, 10:26:38
сверни в числите сверни по формуле 2sin a . cos a = sin 2a запишем
14.2si 144 : sin 144 = 14

174. Slam, 16 декабря 2009, 15:47:46
спасибо большое Леонид!

175. Сергей, 17 декабря 2009, 22:15:07
Помогите плз.
У меня большинство B10 задач отображаются не нормальными формулами, а не понятными символами. В чем может быть дело??
Вот пример:
http://dl.dropbox.com/u/20076/111.png

176. Slam, 18 декабря 2009, 18:21:59
скажите пожалуйста как быстро решать задания B7, там где иррациональные выражения. По какой формуле они решаются?
Ну вот например задание №16923

177. Lexxus, 18 декабря 2009, 18:35:16
скажите пожалуйста как быстро решать задания B7, там где иррациональные выражения. По какой формуле они решаются?
Ну вот например задание №16923

А они никак быстро не решаются, разве что с калькулятором.
Это так называемые "дебильные задания".
Нужно применить формулу разности квадратов, перемножить, получить шестизначное число, а потом тупо пытаться прикинуть, чего это число квадрат.

178. Slam, 18 декабря 2009, 19:14:01
ниче себе =), спасибо за ответ!

179. Алла, 18 декабря 2009, 19:17:04
Привет, Lexxus!
"Дебильность задания" исчезнет, если решать его рационально!!!!!!!!!!!!!!!!!! Да надо применить формулу разности квадратов, но для чего перемножать получившиеся числа, для того, чтобы помучить себя любимого? Принято числа раскладывать на простые множители, а уж затем извлекать корень и тогда задание из "дебильного" превращается почти в устное!!!!!!!!!!!!!!!!!!

180. Алла, 18 декабря 2009, 19:23:38
Slam!
Читай технологию в предыдущем ответе (хотя совсем не обязательно раскладывать числа на простые множители)
Например № 16925
показываю работу с подкоренным выражением:
(458-120)(458+120)
338*578
2*169*2*289
Извлекаем корень: 2*13*17=442
Успехов!

181. карина, 18 декабря 2009, 20:58:25
извините, но не чего сказать не могу.....

182. Леонид, 18 декабря 2009, 22:38:34
Slam и Санёк, интересующие Вас задания (системы и др. С1-С6) - перед "ПИШИ" на этой станице кликните "ОТКРЫТОГО БАНКА ...), откроется страница, смотрите раздел "ПРОВЕРОЧНЫЕ РАБОТЫ" , там варианты и ответы, в ответах решения этих самых заданий. УСПЕХОВ! Нашли?

183. 390840(Slam), 19 декабря 2009, 21:38:07
#6422 как решать такого типа задачи? Объясните пожалуйста! Это B-8!

184. 390840(Slam, 19 декабря 2009, 21:42:02
Не нашел, Леонид ! Где этот раздел "Проверочные работы"?!

185. Lexxus, 19 декабря 2009, 22:27:54
Привет, Lexxus!
"Дебильность задания" исчезнет, если решать его рационально!!!!!!!!!!!!!!!!!! Да надо применить формулу разности квадратов, но для чего перемножать получившиеся числа, для того, чтобы помучить себя любимого? Принято числа раскладывать на простые множители, а уж затем извлекать корень и тогда задание из "дебильного" превращается почти в устное!!!!!!!!!!!!!!!!!!

М-да, об этом способе я как-то не подумал)))
Вот что значит всегда иметь под рукой комп или хотя бы телефон с калькулятором на борту - способность к устному счету стремительно атрофируется:(

186. Леонид , 19 декабря 2009, 23:40:31
Slam, привет! Когда зайдёшь сюда, вернись к началу этой страницы, щёлкни на выделенные слова "ОТКРЫТЫЙ БАНК ЗАДАЧ ЕГЭ ПО МАТЕМАТИКЕ". Затем опустись в конец страницы (до пиши), но щёлкни на выделенные слова перед "пиши .... ОТКРЫТОГО БАНКА ЗАДАЧ ЕГЭ ПО МАТЕМАТИКЕ", жди, откроется страница, в левой колонке раздел "ТРЕНИРОВОЧНЫЕ РАБОТЫ" (кликни, что нада или варианты, или ответы - лучше ответы, там все С с условием и решением). Пробуй.

187. Леонид, 20 декабря 2009, 00:00:11
Slam, D8-6422- производная положительна на промежутках возрастания функции (там график функции). Целые точки толька одна х = 2.

188. 390840(Slam), 20 декабря 2009, 11:27:33
нашел, спсб=)

189. илья, 20 декабря 2009, 17:19:32
столько ошибок в ответах. особенно где графики производной

190. Lexxus, 20 декабря 2009, 18:20:19
столько ошибок в ответах. особенно где графики производной

А потому что там часто вообще неоднозначные ответы.

А еще очень часто забывают, что там везде функция определена на ИНТЕРВАЛЕ (крайние точки не включены в область определения). И когда задание, скажем, - "указать количество целых точек", - как правило, самый "популярный" ответ - на единицу больше, чем правильный.

191. Алиса, 20 декабря 2009, 21:51:21
Научите решать задачи про рабочих(((

192. Леонид , 20 декабря 2009, 22:55:45
Алиса, также как и на движение:
путь s - вся работа A (объём, работа), скорость v= s:t - производительность p=A:t(работа в единицу времени),время в пути t = t время затраченное на выполнение работы. Уравнение

193. vladik2, 21 декабря 2009, 01:11:31
Задача про рабочих.
Один раб. может выполнить работу на пять часов раньше другого, а вместе они ее сделают за 3 часа.ск. час надо каждому, чтобы выполнить работу?
Схема решения.
1 раб за х ч, тогда второй за х+5 час.
За 1 час они могут сделать 1:х и 1:(х+5) частей работы- это дрю словами их производительность.
находим общую производительность: 1:х +1:(х+5)
и теперь ее умножая на 3 часа получаем всю работу, которую принимаем за 1.
Ур. 3( 1:х+1:(х+5))=1
И все!!!!

194. vladik2, 21 декабря 2009, 01:15:02
А вообще-то , кто-то обещал что-то придумать с задачами группы С.
Или спасение утопающих- дело самих утопающих?

195. Таня, 23 декабря 2009, 20:52:43
Помогите решить В12 №5753, очень нужно прямо сейчас.

196. Lexxus, 23 декабря 2009, 23:27:12
Помогите решить В12 №5753, очень нужно прямо сейчас.

Не знаю, прошло ли уже "прямо сейчас", но тем не менее.

Вообще, практически все задачи B12 можно даже не решать, а просто угадать ответ. В этих задачах с теплоходом или двумя велосипедистами всегда
v1*t1 = v2*t2 = S,
причем v1 в км/ч равно t2 в часах, а v2 в км/ч равно t1 в часах.

Так и с этим теплоходом - по течению он шел 21 час со скоростью 29 км/ч, а против течения - 29 часов со скоростью 21 км/ч. Что 21*29, что 29*21, получается 609. А заметив это, уже нетрудно понять, что
29 = 25+4, а 21 = 25-4, то есть ответ - 4.

То есть такие задачи можно "решать", просто раскладывая на множители значение пути и проверяя выполнение остальных условий - это достаточно быстро.

А если делать честно, то нужно решать систему из трех уравнений:
Нам известно, что путь туда равен пути обратно и равен 609 км, что скорость теплохода равна 25 км/ч, и что общее время в пути - 50 часов (58 минус время стоянки).
Итак, обозначим t1 - время в пути по течению, t2 - время против течения, vp - скорость течения, которую надо найти.
Получаем систему:

(25+vp)*t1 = 609 (путь по течению)
(25-vp)*t2 = 609 (путь против течения)
t1+t2 = 50 (общее время в пути)

Тут удобно выразить t1 через t2 в третьем уравнении и подставить в первое, а второе пока не трогать:

(25+vp)*(50-t2) = 609
(25-vp)*t2 = 609

Теперь вычтем из первого уравнения второе, аккуратно приведем подобные и получим

t2 = vp + 25

Теперь подставим это во второе уравнение:

(25+vp)*(25-vp) = 609

Обычное квадратное уравнение, корни которого: 4 и -4. Скорость течения отрицательной быть не может, так что наш ответ - 4 км/ч.


197. vladik2, 24 декабря 2009, 00:18:19
Таня!
Предлагаю другое решение.
1. Скорость теч. реки обычно 2,3,4,5 км/ч.
2.Ответ нужно искать среди этих чисел.
3. х-ск. теч.
25+х это ск. по теч.
25-х это ск. против теч.
4.Составляем уравнение:
609/(25+х) + 609/(25-х) =50
50, потому что он стоял 8 час.
5.Самое главное: не надо заморачиваться и решать это ур.
Разложим 609 на множители: 609=3х7х29
Значит в знаменателе очевидно должно быть 29и 21
Таким образом х=4


198. АЛЁНА, 24 декабря 2009, 17:06:12
обьясните, пожалуйста,решение предыдущей задачи. как можно угадать ответ? если в произведении получилось 609, как перейти к следующему выводу:
29=25+4 и 21=25-4???

199. vladik2, 24 декабря 2009, 18:19:54
Алена!
609 должно делиться на 2 каких то числа и в ответе получится 50.
Теперь найдем на что делится 609.
Оно делится на 29 и 21.
А дальше простейший подбор.

200. алёна, 24 декабря 2009, 18:26:19
я опятьничего не поняла:)) если не сложно, то можно еще раз попонятней?и никак не могу найти оригинальное условие задачи.

201. Леонид, 24 декабря 2009, 19:34:44
разложение на прстые множители: 609:3=203 ( 203 на 5 не делится, пробуем на 7). 203:7=29. Вот и числа: 3Х7 =21 и 29. В сумме 21 + 29 = 50. Понятно?

Влад, Привет!

202. vladik2, 24 декабря 2009, 19:46:02
Алена!
Зайди по этому адресу http://mathege.ru:8080/or/ege/Mainсу:
Введи слева внизу № задачи5753. И все! Она тебе откроется.Не забудь , нажать на левую кнопочку мышки наведя на стрелочку, там увидишь ее(стрелочку).
Ну, а проще так:надо найти на что делится 609 и чтобы потом получилось 50.
Если так не получается, то нужно решать это уравнение, от которого зубы могут заболеть станд. способом.
Леонид! привет!С наступающим!

203. алёна, 24 декабря 2009, 20:06:54
Ребят, спасибо большое!!!!! Я себя в вашем обществе идиотом ощущаю:)) если знате, как еще просто что- нибудь решить, напишите, пожалуйста, буду просвещаться:)))))

204. ЛАНА, 25 декабря 2009, 09:04:01
ОБЪЯСНИТЕ ПОЖАЛУЙСТА, ПОЧЕМУ В ЗАДАНИИ В2(№1995) ответ 7? У меня почему то 5 получается?!

205. vladik2, 25 декабря 2009, 15:02:51
Лана! Там ответ вообще-то 6.

206. 390840(Slam), 25 декабря 2009, 15:49:21
vladik2, А почему 6 , там же вроде 7 должно быть!

207. Lexxus, 25 декабря 2009, 16:20:49
У меня почему то 5 получается?!
Там ответ вообще-то 6.
А почему 6 , там же вроде 7 должно быть!

Так, прекратить истерику.
Считаем вместе. НЕ МЕНЕЕ 12 градусов, судя по графику (загибаем пальцы),
11-го, 12-го, 13-го, 14-го, 15-го, 16-го и 17-го числа.
Сколько пальцев загнулись? Семь.

208. Леонид, 25 декабря 2009, 16:39:14
прекрасно рассчитант: Ребяты! Вычитанием здесь (18-12=6) из-зя! Перечисление Да! Выже находите не длину отрезка....

Загадка:
Как поместить жерафа в холодильник?
Как поместить слона в холодильник?
Лев собрал всех зверей и живность на собрание, а одного не было. Кого?
Надо переплысть реку, в которой много- много крокодилов водится?

209. Леонид, 25 декабря 2009, 16:47:33
Алёна, не надо себя чувствовать себя тем, как ты выразилась... Пиши, обращайся - конкретно! Колективный разум поможет,м.б. и справимся.

210. vladik2, 25 декабря 2009, 19:11:07
Точно! В этой задаче ответ 7 дней.
Надо же ошибся.Кошмар!

211. Леонид, 25 декабря 2009, 19:37:31
Привет, Влад! Никакого кошмара - это невнимательность. Правда на ЕГЭ - досада, непоправимая. Я тоже как торопыжка, но локоть, хотя и близко, попробуй укуси его. А на загадку твой отве? Или не понрвилась?

212. vladik2, 26 декабря 2009, 00:06:37
Леонид,привет!
Что то я слышал про жирафа и слона но не помню.
Надо открыть хол-к, поместить туда слона и закрыть хол-к
Что то типа этого.

213. vladik2, 26 декабря 2009, 00:26:52
Даю примеры С1,С2,С3 , которые были в московской тренировочной работе 21 декабря. Может кого заинтересует.Жаль только , что это все нормально не напишешь.Матем. символы из адобаридера что то не коприруются и не вставляются.Но желающие могут проверить свои знания.

C1 Решите систему уравнений:
cos(x + y) = - 0,5
sin x + sin y =sqrt 3 .
C2 В основании прямой призмы ABCA1B1C1лежит прямоугольный
треугольник АВС, у которого угол С равен 90°, угол А равен 30°,
АС = 10sqrt3. Диагональ боковой грани B1C составляет угол 30° с
плоскостью AA1B1. Найдите высоту призмы.
С3 Решите неравенство: .

(3^((х-2):2))sqrt(3^x - 10sqrt3^x +9) >=0

214. Леонид, 26 декабря 2009, 08:10:17
Да, Влад! Не было слона - он в холодильнике. Плыви не бойся - все крокодилы на собрании.
Влад, добавь ещё и свою С4. Расшифруй в с1 правую часть и придумай понятнее с3 - словами прпиши. С Наступающим Годом Новым!!! Тигринным...

215. vladik2, 26 декабря 2009, 11:07:07
Леонид!
sqrt-кв. корень
С4 не было(С5С6 тоже)
С3:
3 в степени х-2 деленное на 2 умножить на кв. корень из 3 в степнеи х минус 10 корней из 3 плюс 9 больше или равно нуля
С наступающим! Он наступит и 2011 мы все будем встречать студентами!!!
И будем всем говорить :ой,да ЕГЭ это ерунда, ничего страшного.

216. Леонид, 26 декабря 2009, 11:50:46
Спасибо, Влад!

217. vladik2, 26 декабря 2009, 14:58:02
Леонид! А вот еще.Это ноябр.тренир. моск. ЕГЭ
C1 ) Решите систему уравнений
16cos x &#8722; 10 . 4 cos x + 16 = 0,
2sin x+&#8730;y = 0.

C2) В правильной шестиугольной призме ABCDEF A1B1C1D1E1F1
все ребра которой равны 1, найдите косинус угла между
прямыми AB1 и BC1

C3) Решите неравенство log x(log9(3x &#8722; 9)) < 1.(x и 9- основания)

C4 )Точки D и E – основания высот непрямоугольного треугольника
ABC, проведённых из вершин A и C соответственно. Известно, что DE/ AC= k
BC = a , AB = b
Найдите сторону AC.

C5) Найти все значения a, при каждом из которых уравнение
cos&#8730;( a2 &#8722; x2 ) = 1 имеет ровно восемь различных решений.(а в кв., х в кв., корень над всей разн.)

C6) Решите в натуральных числах уравнение n ! + 5n + 13 = k2(к в кв.)
где n!=1•2•…•n – произведение всех натуральных чисел от 1 до n.

218. vladik2, 26 декабря 2009, 15:07:43
Исправления:
С1
16 в степени косинус х - 10умноженное на 4 в степени косинус х +16 =0
С3
лог по осн х от лог по осн 9 от 3 в степени х -9 меньше 1
С5
косинус от корня (а в кв. - х в кв.)=1

219. Леонид, 26 декабря 2009, 16:30:41
Спасибо. Скопировал, что не пойму, позже спрошу.

220. vladik2, 26 декабря 2009, 17:24:53
ИСПРАВЛЕНИЕ
В С1 2синус х плюс корень из у равно нулю.

221. Леонид , 26 декабря 2009, 18:52:28
Всё восстановил! Как полугодие?

222. Леонид , 26 декабря 2009, 20:37:46
Влад, в С4 ДЕ/АС=к - это отношение равно к или пересекаются в точке К?

223. vladik2, 26 декабря 2009, 22:29:58
Равно

224. xeqrt, 28 декабря 2009, 06:04:28
вы все бараны над самому думать а не ответы скачивать

225. Леонид, 28 декабря 2009, 07:52:33
Вы! Выше! Неужели непонятно, что народ не ищет ответы, а учится. Интересно, как ты общаешся с людьми - здесь взял и всех оскорбил.

Ребята, осторожно с задачами В12 - с устным подбором. Это чистая случайность. Такой подход возможен, если одинаковы числа (только числа) по условию, когда разница по скорости и времени совпадают. Например, скорость одного на 4 меньше (больше) и затраченное время на 4 меньше (больше). То есть и там, и там участвует число 4. Ибо произведение скорости и времени - это не о чём.

Надёжнее: составить уравнение и решить.

226. Лёша, 29 декабря 2009, 14:24:26
Что-то неверно учитывается результат решённых заданий - число баллов совпадает с числом выполненных заданий.

227. Леонид, 29 декабря 2009, 14:30:19
Уважаемые! В мониторинговых работах, в вариантах без производной В8 предлагают логарифмические уравнения. Не нехожу таковых в открытом банке заданий ЕГЭ 2010 по математике. Почему?

228. Леонид, 31 декабря 2009, 11:08:18
Lexxus! Влад! Да и все остальные нашегго сообщества. Пусть Дед Мороз Багряный нос в сбербанке сделает вам взнос! Пусть Снегурочка тайком, весь год поит вас коньяком! Пусть превратит весёлыёй гном ваш холодильник в гастроном!!! С Новым Годом, ДРУЗЬЯ!

229. 390840(Slam), 2 января 2010, 17:01:32
Всех с Новым ГОдом=)))))) Объясните пожалуйста как найти ответ в задаче №5301 !!!!!!!!

230. Леонид, 2 января 2010, 18:12:10
Смотри Влада от 24.12. в 19.46

231. 390840(Slam), 2 января 2010, 18:20:38
Леонид, это воопще другая задача, я написал про b6 где надо найти полощадь половины окружности и ответ выразить в s/pi

232. Леонид, 2 января 2010, 19:54:10
Площадь круга Пи R кв. Бери ту часть, которая заштрихована, снимая радиус по клеткам. Например R = 2. Площадь круга 4пи, половины круга 2пи. Делим на пи, получаем 2 - ответ. Деление на пи сделано, чтобы вписывался ответ - целое число или конечная десятичная дробь.
Чтобы посмотреть, заходи как я тебе описывл. Ниже есть окно со стрлкой, вписывай номер , на стрелку и жди

233. 390840(Slam), 3 января 2010, 14:47:15
Че-то долго мы эти 10000 задач решаем =))))), пока решили только 10%, из всех.

234. алёна, 3 января 2010, 21:54:21
помогите,пожалуйста, найти наибольшее значение функции:
Y= 4x-4tgx +п -9
на отрезке От -п4 до п4 включительно

235. алёна, 3 января 2010, 21:57:26
на отрезке от -п делить на 4, до п делить на четыре:))

236. Леонид, 3 января 2010, 22:27:53
Слам! Многие из них отличаются только числами. Набивай руку, главное не наляпать вычислительных ошибок. Внимательно и неспеша.

Алёна! Бери производную 4 - 4/cos кв. х. Приравнивай 0. cosx = 1, cosx = -1. Ни -пи/2 , ни пи/2 в данный промежуток не входят, другие тем более. Находи значения функции при х=-пи/4 и х=пи/4. Выбирай что просят: наибольшее или наименьшее.
А если заметила, что производная в данном случае положительна, то наименьшее в точке х=пи/4, наибольшее при х=пи/4. Подставляй в функцию и считай. Здесь трудно печатать решение полное.

237. Lexxus, 3 января 2010, 22:28:55
помогите,пожалуйста, найти наибольшее значение функции:
Y= 4x-4tgx +п -9
на отрезке От -п/4 до п/4 включительно

Как решать такие задания, я уже писал тут в комментах 13 декабря 2009 в 23:34:16.

Здесь трудность состоит разве что в том, что производная здесь
4-4/(cos(x))^2, и в точке x=0 она обращается в ноль. Но и справа, и слева от этой точки она отрицательна. Так что наша функция на заданном отрезке все-таки убывающая, а вернее - невозрастающая.
Поэтому максимальное возможное значение она принимает в левой границе отрезка, в точке x=-pi/4

y(-pi/4) = 4*(-pi/4)-4*tg(-pi/4)+pi-9 = -pi-4*(-1)+pi-9 = 4-9 = -5

238. Lexxus, 3 января 2010, 22:33:47
Приравнивай 0. cosx = 1, cosx = -1. Ни -пи/2 , ни пи/2 в данный промежуток не входят, другие тем более.

ай-яй-яй.

239. алёна, 3 января 2010, 22:51:26
Лёня!!!! я тебя не понимаю.впрочем всех остальных тоже. Давайте снизойдем до моего уровня( это чуть выше нуля) и объясним чуть проще? буду оч благодарна:)

240. Леонид, 3 января 2010, 22:52:46
Что ай-яй-яй? Производная равна 0, если косинус = -1 или 1.

241. Lexxus, 3 января 2010, 22:56:41
Лёня!!!! я тебя не понимаю.впрочем всех остальных тоже. Давайте снизойдем до моего уровня( это чуть выше нуля) и объясним чуть проще? буду оч благодарна:)

Да нивапрос.
Считай значение функции во всех точках, на которых можешь его посчитать в пределах отрезка [-pi/4; pi/4]. Выбирай наибольшее.

242. Lexxus, 3 января 2010, 22:59:02
Что ай-яй-яй? Производная равна 0, если косинус = -1 или 1.

Верно. Только косинус равен 1 в точках 0+2*pi*n, а не pi/2+2*pi*n ;)

243. Леонид, 3 января 2010, 22:59:26
Производная равна 0, если косинус = -1 или 1. Да, ляп - это о , пи, - пи. Но здесь видно 1 - 1 на что-то (положительное) делённая , величина положительная или равная 0. Немного, Алёна, поспешил и насмешил.

244. алёна, 4 января 2010, 16:26:23
в10. никак не сходится с ответом:
Высота, на коей находился камень, брошенный с земли вертикально вверх, меняется по закону h(t)=2+14t-5t^2(м). сколько секунд камень будет находиться на высоте более 10 метров?
уже всю голову сломала, получается кв. уравнение, из которого 2 корня: 0,8 и 2 . это все ,что я смогла придумать:))

245. vladik2, 4 января 2010, 16:51:44
Алёна, твой камень через 0,8 сек был на высоте 10 м,далее он летел вверх, потом начал падать, и через 2 сек он опять был на высоте 10м.
Значит, на высоте большей 10 м он был 1,2с (2-0,8=1,2)

246. алена, 4 января 2010, 16:58:28
Спасибо большое, до меня(слава Господу) дошло.. еще с заданием не поможите?:
объем цилиндра равен 1 куб.см. Радиус основания уменьшили в 2 раза, высоту увел. в 3 раза. найдите новый объем

247. Леонид, 4 января 2010, 17:35:02
Объм цилидра V = pi R кв. Н. После уменьшения радиуса и увеличения высоты будем иметь pi (R/2) кв. (3Н) =
= pi R кв./4 * 3Н = pi R кв. Н * 0,75 = о,75 , т.к. pi R кв. Н по условию 1. 0,75 это 3/4

248. алёна, 4 января 2010, 18:11:07
спасибо!!!!!! я бы еще системку подкинула, но мне уже стыдно:)))

249. Алла, 4 января 2010, 18:43:06
Алена! Стыдно не пытаться и не учиться, а учиться и спрашивать никогда не стыдно! Дерзай и у тебя все получится

250. Артём, 5 января 2010, 16:59:32
Можете написать алгоритм как делать задания B-8?
Объясните пожалуйста как это задание делать! 6423 !

251. Алла, 5 января 2010, 19:31:30
Артем! На рисунке график функции, если производная функции отрицательна, то функция убывает. Значит, остается посчитать количество целых значений переменной х, при которых график идет сверху вниз (естественно: слева направо). Ответ в задаче 6423 8 (промежутков убывания на рисунке два, считаешь на обоих промежутках)

252. Артём, 5 января 2010, 20:13:30
Спасибо Алла!

253. Леонид , 5 января 2010, 20:35:11
Алёна, где твоя система?

254. Артём, 5 января 2010, 20:39:21
Если решать задание 5301 по формуле площади круга, то получается 8 , а вот если смотреть по рисунку, то там площадь половины круга больше 20!
Это рисунок не правильный, или я что-то эту задачу не правильно понимаю?
задание 5301

255. алёна, 5 января 2010, 20:51:15
Я, как похоже самый понятливый здесь человек, сново слезно прошу вашей помощи:
система:
1 уравнение: 16^(cosx) - 10*4^(cosx) +16 =0
2 ур: корень кв. из у + 2 sinx=0

по выше указанной причине, объяснения, содержащие оч умные слова можно пропустить и начать сразу "для чайников":))

256. алена, 5 января 2010, 20:54:33
во 2 ур. Корень кв. только из у.

257. Леонид, 5 января 2010, 21:33:35
Артём! На рисунке круг радиуса 4. Так? Площадь круга Пи* 4 в кв., т.е.
16 пи. Площадь половины круга 8пи. Так? Да, разделив на пи, 8 - ответ.

258. Алла, 5 января 2010, 21:41:14
Артем и Леонид! Не загружайтесь по поводу задачи 5301, ее убрали из открытого банка, значит на экзамене ее не будет

259. Алла, 5 января 2010, 21:53:53
Алена!
Для первого уравнения выполняешь замену 4^cosx=t
t^2-10t+16=0
t=2 или t=8
Получаем, что cosx=0,5 или cosx=1,5
Второе уравнение не имеет решения
Если проанализируешь второе уравнение системы, то увидишь, что синус принимает только отрицательные значения или ноль.
Косинус положительный, а синус отрицательный только в 4 четверти
Значит x=-пи/3 +2пик
sinx=-корень из 3/2
подставляешь значение синуса во второе уравнение системы и получаешь, что y=3
Если трудно разобраться напиши свой электронный адрес, пришлю решение подробное (здесь очень трудно записывать решение части С)

260. Леонид, 5 января 2010, 21:54:44
Алёна. Запиши 1-ое уравнение 4 в ст.(2 cos x) -10* 4 в ст. (cos x) + 16 = 0.
Пусть 4 в ст. (cos x) = a. Имеем а в кв. -10*а+16=0. а=8 или а = 2.
4 в ст.(cos x) = 8 или 4 в ст. (cos x)= 2. Отсюда cos x = 1,5 нет решений;
cos x = 1/2. х = пи/3 + 2пиК или х = -пи/3 + 2пиК.
Ко 2-ому уравнению: корень из у = -2sin(пи/3 + 2пиК),
корень из у = - корень из 3 - нет решений.
И корень из у = -2sin(-пи/3 + 2пиК), корень из у = корню из 3. то у = 9.
ответ: (-пи/3 + 2пиК;9)

сложно печатать математику. Думаю, поймёшь.

261. алёна, 5 января 2010, 22:41:32
Спасибо всем!!!! я разобралась!!!!!!

262. vladik2, 5 января 2010, 22:48:39
По этой системе имею честь сказать след.
Ответ у меня здесь такой:

Решите систему уравнений
16^cos x − 10 ⋅ 4^cos x + 16 = 0,
√y + 2sin x = 0.
Ответ:
x = − π/3+ 2πn, n ∈ Z
y = 3

Так, что если кого то это все заинтересовало и вы пытаетесь решить и у вас др. ответ , то он может быть таким.

263. vladik2, 5 января 2010, 22:52:40
Плохо здесь объяснять решения. Получается абракадабра

х равен минус пи на 3 плюс 2пиэн,
а y = 3


264. Lexxus, 5 января 2010, 23:21:19
Плохо здесь объяснять решения. Получается абракадабра

А ты поменьше специальных символов используй.
Чем тебя, например, не устраивают дефисы в качестве минусов и звездочки в роли знаков умножения?

265. vladik2, 6 января 2010, 00:13:20
Lexxus!
Научите как это Вы так ловко пи вставили?
У меня не получается.Напесатал в ворде, вроде все хорошо и понятно , потом всавил сюда и получается абракадабра.
И спасибо за исправления.
У меня предки сейчас спорят какой телик купить:ЖК или плазму?
Решили, что для нашей малогабаритки лучше ЖК тк не будет шума от вентиляторов.
А вы же наверняка физик. Вот и Ваш совет сгодился бы.ЖК или плазма?

266. Lexxus, 6 января 2010, 01:48:59
Научите как это Вы так ловко пи вставили?
У меня не получается.Напесатал в ворде, вроде все хорошо и понятно , потом всавил сюда и получается абракадабра.
И спасибо за исправления.

Я это сделал через админку.
Пользователь этого сделать принципиально не может. Во всяком случае, пока я не внесу изменения в функцию обработки полученного комментария.

У меня предки сейчас спорят какой телик купить:ЖК или плазму?
Решили, что для нашей малогабаритки лучше ЖК тк не будет шума от вентиляторов.
А вы же наверняка физик. Вот и Ваш совет сгодился бы.ЖК или плазма?

Ну, уж если пошел такой оффтоп... :)

Шум от вентиляторов - вещь, надо сказать, сильно преувеличенная, тем более что в современных плазмах, я слышал, от них уже научились избавляться.
Но плазма - штука такая... Если хоть сколько-нибудь важны циферки в счете за электричество, то нужно помнить, что плазма ест в разы больше электроэнергии, чем ЖК. Но зато плазма может использоваться как дополнительный комнатный нагреватель - она заметно теплая, когда работает. Особенно когда показывает что-то яркое :)

Кроме того, у плазмы есть такой косяк, как "выгорание" - если ты постоянно смотришь, скажем, первый канал, то достаточно скоро его фирменная единичка будет заметна на экране постоянно, при просмотре любых других каналов и всяких дивидей.

Зато у плазмы значительно лучше контрастность изображения и глубина черного цвета - включенная плазма с чернотой на экране на глаз не отличается от выключенной, а ЖК - конкретно светлее.

Еще у плазмы практически нулевое время отклика (это плюс, если что :) - например, если что-то яркое быстро перемещается по темному фону, то "хвоста" от этого изображения на плазме видно не будет, а на ЖК - в принципе, будет. Хотя сейчас и на ЖК-телевизорах этот эффект для неискушенного глаза почти не заметен.

В общем, когда передо мной стояла такая же проблема, то я выбрал ЖК.

267. Леонид, 6 января 2010, 07:44:11
Алла, просматривая страницы В6, №5301 присутствует, именно то, которым Артём интересуется. С наступившим 2 десятым! И с Рождеством грядущим!

268. Lexxus, 6 января 2010, 10:18:04
Алла, просматривая страницы В6, №5301 присутствует, именно то, которым Артём интересуется.

Алла имеет в виду, что эту задачу убрали с сайта mathege.ru. Но, вообще говоря, это не обязательно значит, что задач такого типа точно не будет во вариантах ЕГЭ.

269. Алла, 6 января 2010, 11:18:43
Lexxus!
Если вы внимательно читали документы,связанные с изменением структуры ЕГЭ по математике, то прочитали, что обещают в варианты ЕГЭ включать задания только из открытого банка (речь идет о части В) единственное, что могут сделать - изменить числовые параметры.

270. Леонид, 6 января 2010, 15:06:59
Алла! Какой официальный докуент об этом говорит? Ведь до сих пор звучит ПРОЕКТ. Или что-то пропустил, или у нас не знают?

271. vladik2, 6 января 2010, 16:58:08
Lexxus!
Спасибо за совет!

272. Алла, 6 января 2010, 17:29:59
Леонид!
Смотри, например, пояснительную записку на сайте mathege. Я слушала Семенова в onlin, который это преобразование начал. Ну и вообще планируется переход к открытости экзамена, т.е. создание сборника, из которого формируются задания экзамена (сборник электронный - это и есть открытый банк заданий, который планировалось пополнять до декабря, что и происходило, а сейчас происходит коррекция банка и если вы заметили он перестал пополнятся, а начал уменьшаться за счет того, что из него убирают некорректные задания). Если те кто "стоят у руля" нас не обманывают, то открытый банк стоит прорешать, что быть готовыми к экзамену

273. Леонид , 6 января 2010, 18:20:09
Да, это всё так. Открытость Банка заданий - здорово, если так будет.

274. 390840(Slam), 6 января 2010, 18:24:43
будем надеяться что не обманывают=)

275. med326, 7 января 2010, 01:28:44
я своим ученикам посоветовал этот сайтю они решают и некоторые давольно успешно . но мне хотелось бы побольше статистики: скорлько каждого задания решено , сколько правильно и сколько неправильно. чтобы работать индивидуально.по каждому разделу В. может это уже есть - тогда извените. но я не нашел. тогда и для ученика и для учителя картина будет видно сразу. можно даже диаграммой: два столба зеленный - решенные правильно , ну а красный -.. отличный чайт.спасибо.

276. КLASS, 7 января 2010, 09:54:51
Люди вот я вам завидую в хорошем смысле этого слова. МОлодцы!!! я еле 10 правильных решаю из части В а из части С максимум баллов 3 набираю. Ладно, а где же ответы на банк заданий ? очень нужно скажите плиз

277. Л.М., 7 января 2010, 10:02:19
Привет коллега! C Рождеством! Я это делаю не по заданиям, а тематически. Но все задания отсюда, распределив их по видам,типам и приёмам решения. Пишите, делитесь опытом.

278. Л.М., 7 января 2010, 10:08:59
Молодец "КЛАСС"! Уже хорошо. Время есть, совершенствуй свои знания, а главнон - навыки. Математика хорошо поддаётся "стальному заду"

279. med326, 7 января 2010, 10:28:04
Л.М. привет. у меня есть подборка заданий в вордовском варианте. могу поделиться - пиши.кстати мои ученики под именами @@@xxx@@@. просьба такой ник не занимать. а сам решаю только нерешённые чтоб у ниих ответы были правильные.

280. KLASS, 7 января 2010, 11:14:04
Друзья, я хоть и хвалиться не люблю, но! я по своей сути гуманитарий и поступать буду в лингвистический, изучать 4 иностраных языка, и за эти решенные задачи мне по праву необходимо наградить медалью! (шутка! :-)
Кстати, это правда что проходной балл всего пять правильно решенных задач из части В??

281. алена, 7 января 2010, 14:14:27
С праздником светлого Рождества Христова!!!!!
помогите решить,пожалуйста, задачу:
два автомобиля отправляются в 420-км. пробег. Первый автомобиль едет со скоростью на 10 км/c большей, чем второй, и прибывает к финишу на 1 час раньше второго.. Найти скорость автом., пришедшего к финишу вторым.

из сего получилось:
х+10=420х/420 -х. и еще кучу всего разного, неправильного. ответ должен быть равен 60, но у меня раз пять получалось 410, может я баран???

282. 390840(Slam), 7 января 2010, 15:43:36
алена,
1)420/(x+10)-420/x=1
2)пропроцией перемножаешь и получается: 420X-420(X+10)=(x+10)x
3) Далее раскрываешь скобки и у тебя получается квадратное ур-е
4) Решаешь Дискриминантом
5)Ответы: x1=60; x2=-70- этот ответ не опдходит т.к скорость не может быть с минусом , и значит окончательный ответ 60 км/ч

283. Л.М., 7 января 2010, 15:44:17
Х-скорость второго (пришедшего вторым). То (х+10)-скорость первого.
Время второго 420/х. Время первого 420/(х+10). По условию время второго на 1 час больше. Уравнение 420/х - 420/(х+10) = 1. Всё получится.
Кстати, приём подбора, который раньше рекламировали, здесь не подойдёт - числа времени и разности скоростей разные.

Коллега, у мення есть тоже в ворде по каждому типу заданий В около 50-100 упражнений, на основании которых и планирую по способам и приёмам решения. (из этого банка заданий).

284. 390840(Slam), 7 января 2010, 15:50:38
med326, Л.М., можете сказать,
что нужно знать и каков алгоритм решения заданий B-8?

285. Л.М., 7 января 2010, 18:53:24
Первое-внимательно читать и смотреть, что дано: график функции или грфик производной.
Второе - уметь читать по графику свойства. По грфику функции - функция возрастает, то производная неотрицательна;функция убывает , то производная неположительна на отрезках [;].
И наоборот. По графику производной: производная положительная - функция возрастает, производная отрицательная -функция убывает.
Экстремумы по графику производной там, где меняется её знак при пересечении с осью ОХ. По графику функции - экстремумы в вершинах, т.к. в них производная равна нулю. Например, ск. касательных параллельных прямой у=5. Столько сколько вершин у графика функции (вершин в верху и внизу). Касательных параллельных у =2х-3 по графику производной - столько, в скольких точках эта прямая пересекает график производной, проведя прямую у = 2. И т.д. Необходима хорошая оринтация в понятиях: монотонность, экстремумы, знаки функции и производной и видеть их по графикам.
Почему такой вопрос? Что на уроках? Обратись к учителю! Это несложно, необходимо внимание,и, конечно, знания!!! Друг, СЛАМ!

286. Alexxxa1, 7 января 2010, 20:54:58
кто может подсказать алгоритм решения заданий части С6??заранее спасибо.

287. К.Е.Н., 7 января 2010, 21:02:13
Решила задачу, но сомневаюсь в ответе.

"Моторная лодка прошла против течения 24 км/час и вернулась обратно, затратив на обратный путь на 20 минут меньше, чем при движении против течения. Найдите скорость лодки в неподвижной воде, если скорость течения 3 км/час."

У меня получился ответ 21 км/час.
Где можно сверить ответ? Или может вы подскажете?

288. Л.М., 7 января 2010, 21:59:29
к.е.н.! Верно. 21.

289. Lexxus, 7 января 2010, 22:20:37
я своим ученикам посоветовал этот сайтю они решают и некоторые давольно успешно . но мне хотелось бы побольше статистики: скорлько каждого задания решено , сколько правильно и сколько неправильно. чтобы работать индивидуально.по каждому разделу В. может это уже есть - тогда извените. но я не нашел. тогда и для ученика и для учителя картина будет видно сразу. можно даже диаграммой: два столба зеленный - решенные правильно , ну а красный -...

Весьма польщен.
med326, если не сложно, напишите поподробнее, какая статистика и в каком виде интересна. И лучше на почту: lexxus@mephist.ru

кстати мои ученики под именами @@@xxx@@@. просьба такой ник не занимать. а сам решаю только нерешённые чтоб у ниих ответы были правильные.

Ух ты, а я все думаю, откуда столько таких одинаковых ников :)

290. 390840(Slam), 8 января 2010, 15:37:42
Л.М., спасибо за ответ=)

291. Артём, 8 января 2010, 19:10:37
посмотрите пожалуйста задание № 27448 У меня получился ответ 2, это правильный ответ?

292. Lexxus, 8 января 2010, 19:30:09
посмотрите пожалуйста задание № 27448 У меня получился ответ 2, это правильный ответ?

Задания с таким номером нет. С таким номером есть прототип на сайте mathege.ru. Если ты его имеешь в виду, то да, правильный ответ - 2.
Хотя угол по клеточкам определять - это извращение.

293. Артём, 8 января 2010, 19:37:34
да, прототип имею ввиду=)? а можешь сказать как углы определять, если уж не по клеточкам=), и еще что значит "Прототип задания"?. и почему на сайте этот прототип не пишут просто задание №xxxx.....?

294. Lexxus, 8 января 2010, 19:46:17
а можешь сказать как углы определять, если уж не по клеточкам=)

В данном случае - никак. Это не к тебе претензия, а к тем, кто такие задания сочиняет.

и еще что значит "Прототип задания"?. и почему на сайте этот прототип не пишут просто задание №xxxx.....?

По идее, "прототип" - это такая фигня, из которой получаются собственно задания при изменении в нем некоторых чисел и/или рисунка.
Из одного прототипа можно сделать кучу заданий.

А почему нет заданий по этому прототипу - не знаю. Может, они там тоже понимают, что нехорошо определять угол по клеточкам? =)

295. 390840(Slam), 8 января 2010, 20:05:12
Прикольно получилось после того как начали давать за одно задание максимум 10 баллов, а то некоторые я думаю на одно и тоже задание отвечали по 40, 50..... раз. это я говорю потому что некоторые стоявшие на 4-5 месте сразу же скатились на 20-30 места:)

296. Lexxus, 8 января 2010, 20:11:40
Прикольно получилось после того как начали давать за одно задание максимум 10 баллов, а то некоторые я думаю на одно и тоже задание отвечали по 40, 50..... раз. это я говорю потому что некоторые стоявшие на 4-5 месте сразу же скатились на 20-30 места:)

Да я тут как раз кое-что фигачил и случайно обнаружил одного красавчика, который решил одно и то же задание 358 раз :)
Вот и сделал быстренько, чтобы рейтинг поднимался только когда отвечаешь в первый раз.

297. 390840(Slam), 8 января 2010, 20:25:48
ппц, и че ему этот рейтинг дал интересно:). С новыми правилами решать лучше:)

298. Артём, 8 января 2010, 23:35:56
№5853, Можете сказать как правильное уравнение составить к этому заданию!

299. vladik2, 9 января 2010, 00:17:01
#5853
На изготовление 16 деталей первый рабочий затрачивает на 6 часов меньше, чем второй рабочий на изготовление 40 таких же деталей. Известно, что первый рабочий за час делает на 3 детали больше, чем второй. Сколько деталей в час делает второй рабочий?
1-х+3 дет.
2-х дет
1-16/х+3 часов
2-40/х час, это на 6 больше
УРАВНЕНИЕ
40/х - 16/х+3 =6
Вот и все.

300. Алла, 9 января 2010, 00:31:18
Lexxus!
Иногда меня удивляет ваше отношение к некоторым заданиям (извините конечно) Я говорю о задаче 27448. Это вовсе не "фигня по клеточкам", а применение определения тригонометрических функций острого угла прямоугольного треугольника, достаточно провести перпендикуляр (чтобы получить прямоугольный треугольник) и решить прямоугольный треугольник, зная длину его двух катетов по клеточкам. Задача очень простая и плохо, что таким приемам не учат на уроках, хотя должны!!!!!!!!!!!!!!!!1

301. Л.М., 9 января 2010, 00:37:38
Слам, тебе помогло изложение о решении заданий В8 (графические)? На производную? Конечно, показать бы тебе.

302. Леонид, 9 января 2010, 00:49:50
Алла, что за задание о поиске угла по клеточкам? Не встречал. Это ни те, где надо найти производную в точке касания по крафику функции и касательной. Так, они нормальные. Там выделены 2 точки, приняв за гипотенузу и на них прямоугольный треугольник. И по определению тангенса...

303. Елена, 9 января 2010, 06:25:55
Скажите пожалуйста как скачать ответы к ЕГЭ по математите

304. Леонид, 9 января 2010, 09:02:59
Алла, нашёл эту штуку, мне понравилась даже больше, чем сами В4. Вас поддерживаю полностью.

305. 390840(Slam), 9 января 2010, 11:17:09
Л.М., половниу заданий b-8 (а может больше) я понял , а вот такие типы заданий я вооще не понимаю как делать
№6419
№6414
№6035, можете объяснить если не трудно?
если что номер ICQ 413509549

306. Алла, 9 января 2010, 12:14:00
Slam!
№ 6035 если опустить всю теоретическую подоплеку, то можно просто приравнять производные заданных двух функций. (если прямые параллельны, значит равны их угловые коэффициенты, угловой коэффициент касательной - это и есть производная данной функции)
№ 6419
касательная к графику функции параллельна или совпадает с графиком прямой y=-3x-11
-3 - это угловой коэффициент касательной или значение производной в точке касания, т.е. производная равна (-3), на рисунке график производной, осталось посчитать сколько раз прямая у=-3 пересекает график (ответ: 4)

307. Л.М., 9 января 2010, 12:36:53
Слам! 6419 и 6435 - одного типа. Прямая у=-3х-11. Угловой коэффициент её (-3 - при х) - бери на оси ОУ (-3) и проводи прямую параллельную оси ОХ. Она пересечёт график производной (он на рисунке) в четырёх точках. Ответ 4. (угловой коэффициент и произвоная одно и тоже, значения производной
- 3 по её графику в четырёх точках)! Понятно? Другой пробуй сам и хвались!

6414! На оси ОХ выдели отрезок [-4;0]. На нём видно, что график производной ниже оси ОХ - это говорит, что производная здесь отрицательная, а значит функция - убывающая. Наименьшее её значение на данном отрезке в точке х=0. Ответ 0. Всё понял, дружище?

Что значит указанный тобой номер (а если что)? Удачи.

308. алёна, 9 января 2010, 12:57:46
К е н , Л. М.!!!! Какое верное уравнение должно получиться в задаче от 7. 01?
t1-t2 =20
а t1=24/x+3 t2= 24/x?
только так у меня совсем не получается.
и еще зачачи со сплавами и смесями тоже не получаются. помогите, пожалуйста, разобраться:


Смешав 70-% и 60-% растворы кислот и добавив 2 кг чистой воды, получили 50-% раствор кислоты. Если бы вместо 2 кг врды добавили 2 кг 90-% раствора той же кислоты, то получили бы 70-% раствор кислоты. сколько кг 70-%-го раствора ислользовали для получения смеси.

309. vladik2, 9 января 2010, 13:03:34
Я хочу сказать о задаче 27448.
Кратко об этой задаче: нарисован угол в 45 градусов и просят найти его синус да еще потом умножить на 2 корня из двух.
Только самый посл. двоечник не знает чему равен син. 45 град.
Это 1 дел. на корень из двух. Умножаем и получаем ответ.
И не надо там высчитывать клеточки.
Мое мнение: эту задачу составлял двоечник.
Составители у совсем нас за идиотов держат.

310. Артём, 9 января 2010, 13:15:47
vladik2, есть еще немного другие задания, например 27455, а там как угол найти например?!

311. Л.М., 9 января 2010, 13:25:38
24/х - 24/(х+3) = 1/3, 1/3 часа - это 20 мин.

312. Lexxus, 9 января 2010, 14:23:48
vladik2, есть еще немного другие задания, например 27455, а там как угол найти например?!

А он тоже 45 градусов.
Положим, что точка О имеет координаты (0,0). Соединим точки с координатами (1,3) и (2,1).
У нас получился треугольник со сторонами sqrt(5), sqrt(5) и sqrt(10)
("sqrt" - это квадратный корень, если кто не в курсе).
Совсем нетрудно доказать, что он не только равнобедренный, но и прямоугольный. А у такого треугольника углы при основании равны 45 градусов.

Так что и тут ответ - 1/sqrt(2)*2*sqrt(2) = 2

313. vladik2, 9 января 2010, 14:42:12
А я думаю проще надо.
Итак мое решение( ведь составители совсем дурацкие задачи составляют).
Этот угол, Артем, может быть 45 или 30 градусов( ну и еще 60, но это навряд ли).
Но...., 30 градусов не подходит, т.к. корень из трех деленноое на 2 плохо умножается на корень из двух деленное на два.
Ответ то должен быть целым.
Кстати 60 град. тоже не может быть.Сам подумай почему.
Поэтому опять не надо ничего считать и cos45 умножить на это число и записать ответ.
И все. И финита ля комедия.
Конечно, если останется время, можно все рассчитать, но получишь тот же результат.

314. Леонид, 9 января 2010, 16:21:55
Lexxus! Зачем доказывать прямоугольность треугольника. Рвнобедренный сказали, высота чере середину. И косинус по определению сразу. И ещё - обойти координаты,как у Вас. Стороны легко находятся по теореме Пифагора в клеточных условиях. Всё просто и всем известно.


Влад! Привет! Как житьё - бытьё? У нас морозы под сорок!!! В этих задачах, ка ты советушь,мне кажется может случиться недаразумение (без заданного домножения). Ведь надо найти син. или кос. - а вдруг о, 25 - не от "хороших" углов. . Ведь от составителей что угодно можно ожидать.

315. vladik2, 9 января 2010, 16:36:01
Леонид! Привет!
Жалко каникулы кончаются.
У нас таких морозов нет, а то бы школу отменили.
А по поводу синусов- не будет "нехороших" углов.
И вообще надо уже думать серьезно о группе С.
А бэшные задачи я уже и не смотрю- это для гуманитариев они сложные.

316. 390840(Slam), 9 января 2010, 17:11:12
Л.М.,понял=) вот решал и ответы правильные пока что получаются, сейчас дальше продолжу решать.....
Тот номер который я дал номер ICQ(разг.-"аська"), это такая программа по которой можно переписываться текстовыми сообщенияvми(бесплатно), тот кто регистрируется на сайте icq.com получает номер ( мой например 413509549). подробнее можете узнать об этой программе набрав в поисковике(google.ru , yandex.ru и т.п)-icq. Вот так вот=)

317. Леонид, 9 января 2010, 17:11:39
Здорова ещё раз! С - их что-то мало,кроме как в книжицах да в вариантах проверочных-мониторинговых. Там несложно. Да и очень похожи, только цифрами отличаются. Явно на ЕГЭ будет не так. А где их брать? У тебя, скажу В просто получаются, ты ориентируешься в материале. Потому что знаешь его. Другим сложно. Это видно по товарищам.
Да, Влад, одна девушка обращалась с задачей на смеси - это самые сложные задачи. Откуда она - задача (м.б. глупый вопрос), я их здесь не встречал.

318. 390840(Slam), 9 января 2010, 18:00:53
Леонид, это задача из "типовые тестовые задания егэ 2010, разработано МИОО..."
под редакцией Семенова, ященко

319. Алла, 9 января 2010, 18:51:42
Здраствйуте Леонид и Slam!
Если у вас проблема с набором задач типа С, советую посмотреть книги Ф.Ф.Лысенко из серии "Подготовка к ЕГЭ", у него достаточно сложные и разнообразные задачи.
Удачи!

320. Л.М., 10 января 2010, 12:58:39
Алёна, на Вашу просьбу! Пусть х кг 70% кислоты, в нём 0,7х кг кислоты. А у кг 60% раствора, кислоты в нём 0,6у кг. Всего кислоты 0,7х +0,6у.
Раствора после добавления 2л воды получилось х+у+2. По условию получили 50% раствор кислоты. Значит 0,7х+0,6у = 0,5(х+у+2).Упрощаем: 2х+у=10.

Если добавить 2кг 90% раствора кислоты, то добавится 2*0,9 =1,8кг кислоты. Получим уравнение 0,7х+0,6у+1,8=0,7(х+у+2). Т.к. по условию получили 70% р-р, имеем 0,7х=0,6у=1,8=0,7х+0,7у+1,4; 0,1у=0,4, то у=4.
Система 2х+у =10 и у=4. Подставив у в первое уравнение,получим х=3.

321. анютка, 10 января 2010, 13:36:12
Сделайте,пожалуйста,ЕГЭ 2010 по биологии,очень прошу!!!!

322. алена, 10 января 2010, 16:31:39
Спасибо за задачу!!!!!!

323. Алексия, 11 января 2010, 10:30:25
Я лично не по теме, но хочу спросить: "Почему я не могу пройти тестирование???" Я уже зарегистрировалась, выбираю "Тесты ЕГЭ Онлайн", я перехожу на следующую страницу, и мне пишет, что мне надо зарегистрироваться?! КАК?! Ещё раз?! Объясните мне пожалуйста в чем у меня ошибка (хотя на вряд ли ошибка у меня)??????

324. Lexxus, 11 января 2010, 10:50:21
Почему я не могу пройти тестирование???

А каким ты пользуешься браузером?
Думаю, что причина в том, что у тебя отключены cookies.
хотя на вряд ли ошибка у меня

А вот в этом я сомневаюсь: почти 70 тысяч человек пользуются и не жалуются ;)

325. 390840(Slam), 11 января 2010, 19:25:01
Помогите пожалуйста с задачей по физике:
При равноускоренном движении из состояния покоя тело проходит за пятую секунду 0.9м. Какой путь тело пройдет за седьмую секунду?

326. 390840(Slam), 11 января 2010, 20:02:34
а все уже не надо,решил=),ответ получился 1.3м и ускорение 0.2м=)

327. Лёша, 13 января 2010, 06:46:59
Ребята! Предложите ваши рассуждения по задвче В1 6369. Что в ответе писать "получить?"

328. Lexxus, 13 января 2010, 08:53:24
Ребята! Предложите ваши рассуждения по задвче В1 6369. Что в ответе писать "получить?"

Всё просто. До акции на 200 рублей можно было купить 20 шоколадок, а теперь за каждую седьмую купленную шоколадку дают еще одну.
Это значит, что мы получим по еще одной шоколадке за 7-ю и 14-ю купленные. То есть еще две.
Ответ - 22.

329. Лёша, 13 января 2010, 09:10:26
Большое спасибо, Lexxus! Вселил уверенность в заданиях такого вида.

330. Альбина, 15 января 2010, 11:17:02
Люди помогите решить задачу №4887 ..ход решения...Тяжело даются такие задания !!

331. Lexxus, 15 января 2010, 11:31:23
Люди помогите решить задачу №4887 ..ход решения...Тяжело даются такие задания !!

Из всех прямоугольных параллелепипедов сферу можно вписать только в куб, ребро которого равно диаметру сферы. Значит, ребро куба равно 7.5*2 = 15.

Объем куба = длина ребра в кубе.
15^3 = 3375.

332. Алла, 15 января 2010, 18:38:15
Альбина!
В задачах, о которых ты спрашиваешь, самое главное связать элементы тел, участвующих в комбинации.
Например: радиус шара и ребра многогранника; радиус и высоту цилиндра с ребрами многогранника и т.д. Если ты не знаешь как связаны эти элементы можно об этом догадаться, если внимательно посмотришь на рисунок.
Удачи!

333. Леонид, 16 января 2010, 08:18:01
Друзья! Что - то пыл под остыл? Интерес пропадает что ли? Всё хорошо получается? Давайте советоваться о методах и приёмах решения. Тогда останется только внимательно првести вычиления.

334. 390840(Slam, 16 января 2010, 19:25:34
Lexxus, а че за "Доверие" в Коллективном разуме? Это что значит?

335. Lexxus, 16 января 2010, 20:44:15
Lexxus, а че за "Доверие" в Коллективном разуме? Это что значит?

Это численное выражение степени доверия к тебе моей программы. Чем ближе эта величина к 100%, тем больше она верит в то, что данные тобой ответы правильные.

Философский аспект освещен в одной из свежих моих заметок %)

336. 390840(Slam), 17 января 2010, 09:54:46
Скажите пожалуйста. №6171 как без калькулятора решить по быстрому?

337. Леонид, 17 января 2010, 22:57:36
Lexxus! На правильные ответы выскакивает "И этот ответ неправильный".

338. Lexxus, 18 января 2010, 02:04:27
На правильные ответы выскакивает "И этот ответ неправильный".

А номера заданий?

339. @@@Девяткин@@@, 18 января 2010, 11:13:58
ВЫДАЕТ ОДНИ И ТЕ ЖЕ ЗАДАНИЯ B11 ПОЧЕМУ?

340. Lexxus, 18 января 2010, 11:20:51
ВЫДАЕТ ОДНИ И ТЕ ЖЕ ЗАДАНИЯ B11 ПОЧЕМУ?

Если решаешь "решенные", то не удивительно: их всего-то 28 штук пока. Показываются они в случайном порядке, вероятность повтора очень большая.

341. Леонид, 18 января 2010, 12:31:23
Видимо, в этом причина и моих ответов. Решал "решённые". В коллективном разуме" учёт вроде нормальный.

342. Артём, 18 января 2010, 16:56:03
# 4517 по какой формуле решать?

343. Леонид, 18 января 2010, 23:19:44
Артём, напиши задачу.

344. Артём, 19 января 2010, 14:49:55
8^log64 4 (восемь в степени логарифм по основанию 64 из числа 4)

345. Леонид, 19 января 2010, 15:41:57
Можно по разному, Артём. log(осн.64) 4 по определению 1/з, т.к. 64 в ст. 1/3 равен 4. После этого имеем 8 в ст. 1/3 равно 2. Да?

Можно к основному логарифмическому тождеству 64 это 8 в ст. 2. Выносим перед логарифмом 1/2 (из основания логарифма). запишем (8 в ст. log(осн. 8) 4) всё это в ст. 1/2. По тождеству 4, а в ст. 1/2 есть 2. Да? Всё, Артём, понял?

346. Сергей, 19 января 2010, 16:12:04
напомните как находиться тангес угла в равнобедренном треугольнике))))))или решите задачку:

в треугольнике ABC AC=BC=корень из 5 ,AB=4 найдите тангес угла А


заранее спасибо

347. Lexxus, 19 января 2010, 17:20:53
напомните как находиться тангес угла в равнобедренном треугольнике))))))или решите задачку:

в треугольнике ABC AC=BC=корень из 5 ,AB=4 найдите тангес угла А

Проводишь высоту CH к основанию треугольника. Получаешь прямоугольный треугольник с гипотенузой AC и одним катетом AH = AB/2.
Находишь длину высоты (еще одного катета) по теореме Пифагора.

Тангенс - это отношение противолежащего катета к прилежащему, CH/AH.


348. Леонид, 19 января 2010, 20:45:35
Ну, Артём,З разобрался? Чего молчишь?

349. Леонид, 19 января 2010, 21:12:46
Lexxus! Попробовал решение тестов на "решённых", на все ответы получил -"не правильно". Не уверен:№2967 (0,5), № 15253 (-198), №8705 (4), №21295(7) - в скобках мои ответы. Загляните,пожалуйста, если не затруднит.

350. Артём, 19 января 2010, 21:25:31
Леонид, разобрался. спасибо. Теперь такие примеры буду делать по 2 способу!

351. Леонид, 19 января 2010, 21:40:32
Смотри, Артём, как тебе надёжнее! Здесь дело вот в чём(к основному лог. тождеству): число под логарифмом в виде степени, показатель выносится как множитель перед логарифмом; если основание логарифма в виде степени, то множитель выносится перед логарифмом обратный показателю. Далее записываешь степеь в степени и ОКЕЙ! Вот, улови, главное про запись множителя перед логарифмом в том и другом случае. Вынуди свою учительницу это чётко растолковать. ПРИВЕД, рад, что оказал помощь.

352. Марина, 19 января 2010, 21:53:38
Всем привет! объясните мне как это решается: найдите точку максимума функции y=(x+3)*e в степени 3-х . Я так поняла, что это через производную, но не могу ее найти

353. Lexxus, 19 января 2010, 22:41:16
Lexxus! Попробовал решение тестов на "решённых", на все ответы получил -"не правильно". Не уверен:№2967 (0,5), № 15253 (-198), №8705 (4), №21295(7) - в скобках мои ответы. Загляните,пожалуйста, если не затруднит.

Из решенных тобой сегодня заданий 10 из 11 решены правильно, одно (5229) - неправильно.

354. Леонид, 19 января 2010, 22:53:02
Спасибо! На одном ткнул поторопился, но было поздно, там трапеция на клетках.

355. Lexxus, 19 января 2010, 22:53:16
y=(x+3)*e в степени 3-х


Производная произведения двух функций: (f(x)*g(x))' = f'(x)*g(x)+g'(x)*f(x)
В твоем примере:
f(x) = x+3, f'(x) = 1
g(x) = e^(3-x), g'(x) = -e^(3-x) (не забываем про минус перед экспонентой, который появляется при дифференцировании сложной функции).

Производная нашей функции:

1*e^(3-x)+(x+3)*(-e^(3-x)) = e^(3-x)-(x+3)*e^(3-x) =
= (x+2)*e^(3-x)

356. Lexxus, 19 января 2010, 22:55:48
Леонид, ты уверен, что тебе сегодня больше одного раза программа говорила, что ты ошибся?

357. Леонид, 19 января 2010, 23:17:01
Да! Нажав на ответ, выскакивало "И это неправильно" во всех случаях. Где допустил промах, там как-то по другому было сказано. Я специально решал "решённые", так как и раньше были такие прблемы. Уверен, что решил верно, а получал такую запись, хотя в разжеле "коллективный разум" баллы шли и вроде более одного за верный ответ.

358. Lexxus, 19 января 2010, 23:31:57
Да! Нажав на ответ, выскакивало "И это неправильно" во всех случаях. Где допустил промах, там как-то по другому было сказано.

Этого вообще ни при каких обстоятельствах не может быть.
Если программа считает ответ правильным, то выскакивает зелёная строчка "И он правильный!", если нет - красная "Это неправильный ответ, но все равно спасибо".
Если ты решаешь задание с известным ответом, которого раньше не решал (а в твоем случае это именно так), других вариантов в принципе не бывает.

Так что либо ты что-то путаешь, либо случилось то, чего я боялся: программа стала слишком умной и теперь готовит план порабощения человечества.

359. Леонид, 19 января 2010, 23:36:31
Попробовал ещё, все верно получилось. Даже с одной умышленной как бы ошибкой. Спасибо.

360. Lexxus, 20 января 2010, 12:44:35
Попробовал ещё, все верно получилось. Даже с одной умышленной как бы ошибкой. Спасибо.

Тебе спасибо. Выдохнули, человечество пока может быть спокойно.

361. Александр, 20 января 2010, 18:47:30
Первая труба наполняет бак объемом 600 литров, а вторая труба - бак объемом 900 литров. Известно, что одна из труб пропускает в минуту на 3 л воды больше, чем другая.
не получается(

362. Александр, 20 января 2010, 18:49:59
забыл)
Сколько литров воды в минуту воды в минуту пропускает вторая труба,если баки были наполнены за одно и тоже время?

363. Леонид, 20 января 2010, 19:56:38
Какая, первая или вторая труба, пропускает в бак на 3л воды больше? Т.е., что - то не то с условием. Это важно, тобы составить уравнение. "одна из туб" фраза не идёт. Если первая на 3л меньше, пусть х - вторая.
Уравнение: 600/(х-3) = 900/х - их времена. УТОЧНИ УСЛОВИЕ.

364. Александр, 20 января 2010, 22:55:13
еси б была указана труба у которая в 3 раза больше пропускает я б и сам решил)а тут именно так...

365. Александра, 21 января 2010, 07:27:39
Здравствуйте! решала задачу : камень брошен вверх. пока камень не упал, высота, на которой он находится, описывается формулой h(t)=-5 t ^2 (в кадрате) +18t. h-высота в метрах, t-время в секундах, прошедшее с момента броска. найти сколько сек камень находился на высоте не менее 9 метров.

366. Александра, 21 января 2010, 07:32:04
получилось 2 ответа (если правильно решила) 3 и 0.6. какой ответ правильный?

367. ЛЕНОЧКА, 21 января 2010, 17:51:46
СКАЖИ ПОЖАЛУЙСТА КАК ПРАВИЛЬНО СКАЧАТЬ ОТВЕТЫ К ЕГЭ ПО МАТЕМАТИКЕ!!!

368. Алла, 21 января 2010, 20:59:16
Леночка!
Зачем тебе ответы? Если для экзамена, то это утопия (и числа будут другие, и номера заданий как и номер варианта своей работы ты знать не будешь) Не трать силы зря, лучше попробуй научиться решать, это вполне под силу каждому, кто умеет трудиться!
Опыт лет проведения ЕГЭ уже доказал, что если кто-то приходит на экзамен с ответами скаченными из Интернета, не может ими воспользоваться, потому что этих ответов в Интернете просто нет (вариантов для каждого региона от 30-90, и в разных регионах - разные варианты, а теперь прикинь сколько регионов по России и оцени реальность своих усилий по скачиванию ответов)
Удачи тебе на экзамене!

369. Алла, 21 января 2010, 22:24:03
Александра!
Если у тебя получилось два ответа, то наверное ты решала уравнение, а надо было решать неравенство (-5t^2+18t больше или равно 9), решением которого получается промежуток от 0,6 до 3.
Так как спрашивают сколько секунд..., то ответ получаешь так: 3-0,6=2,4
Ответ: 2,4

370. надя, 23 января 2010, 18:58:33
ммммм блин помоги пожалуйста сдать егэ по матем 2010

371. @лено4ка, 23 января 2010, 19:08:40
почему в ответах к заданию № 2789 (В3) дается число 8 ??? Это же неправильный ответ!!!

372. Lexxus, 23 января 2010, 20:06:50
почему в ответах к заданию № 2789 (В3) дается число 8 ??? Это же неправильный ответ!!!

А какой же тогда правильный? O_o

373. марина, 24 января 2010, 13:22:01
Группа туристов прошла первую половину маршрута со скоростью на 30% больше запланированной. Определите максимальное число процентов, на которое туристы могут сбавить скорость на второй половине маршрута, чтобы закончить поход к запланированному времени. Помогите решить, пожалуйста

374. Los Angeles, 24 января 2010, 15:09:23
о, боже мой...
как сдавать эту математику, вообще не представляю =D
на пробных на 0 баллов сдала.. ахаах. пистец =|

375. Сашуля, 24 января 2010, 16:48:25
Помогите решить задачи:
1)Моторная лодка прошла против течения 16 км и вернулась обратно, затратив на обратный путь на 1 час меньше, чем при движении против течения. Найдите скорость (в км/ч) лодки в неподвижной воде, если скорость течения равна 4 км/ч.

2) 60/ 6 log по основанию 6 из 5

376. Леонид, 24 января 2010, 18:17:44
Сашуля! 1. х - собственная скорость лодки - в стоячей воде, по течению х+4, против течения х-4. Время по течению 16/(х+4), против течения 16/(х-4). Против течения на 1ч дольше, уравнение 16/(х-4) - 16/(х+4) = 1.

2. 60: 5 = 12, так как по основному лог. тождеству 6 в ст. лог 5 по оснн. 6. Если я правильнр понял твою запись!

Марина,откуда такая задача?

377. Асус, 24 января 2010, 20:31:16
У меня вопрос! Как вообще будет работать база с ответами на егэ 2010 по математике,в форме прилажения? Если да то когда это дело будет готово,примерно? Думаю всем интересно.

378. Гарипова Лилия, 25 января 2010, 02:17:48
Привет!Почему нельзя давать ответы на те задания,где помечено "верный ответ еще не определен"?я решила,а ответ куда писать-то?

379. Гарипова Лилия, 25 января 2010, 02:29:50
Отбой,я поняла.

380. Гульназ, 25 января 2010, 15:20:24
Завтра пробный ЕГЭ поматематике,боюсь.!!!Что делать,а?((

381. Леонид, 25 января 2010, 15:30:37
Чиркни некоторые задания. Что там предполагают. Можно будет ориентироваться. Насколько они соответствуют открытому банку заданий по математике ЕГЭ 2010.

382. Марина , 25 января 2010, 18:57:12
Леонид, нам на курсах дали дома решить. Это из теста, что-то вроде демонстрационного варианта, задание В12.

383. Гульназ, 25 января 2010, 19:46:48
Найдите все натуральные числа, не представимые в виде суммы двух взаимно простых чисел, отличных от 1.Помогите,пожалуйста решить эту задачку...)

384. Леонид, 25 января 2010, 20:11:53
Путь весь 1. х - планируемая скорость. Могут сбавить на у - 0,01у%. 1-ая половина - скорость 1,3х. 2-ая половина пути скорость (х-0,01ху).
Уравнение (1/2):1,3х + (1/2):(х-0,01ху) = 1:х.
Решая, получим 0,016ху+1,3х-2,6х+0,026ху=0; х(0,016у-0,3)=0; 0,016у=0,3; у=18,75. Марине.

В банке данных задач такой сложности я не встречал.

385. Марина, 25 января 2010, 20:25:09
Леонид, почему могут сбавить на у - 0,01у%? не понимаю

386. Марина, 25 января 2010, 20:35:35
Ааа, все поняла сама, спасибо большое

387. Lexxus, 25 января 2010, 20:43:57
Найдите все натуральные числа, не представимые в виде суммы двух взаимно простых чисел, отличных от 1.Помогите,пожалуйста решить эту задачку...)


Каждое натуральное число может быть либо четным (2*k), либо нечетным (2*k+1).

1. Если число нечетное:
n = 2*k+1 = (k)+(k+1). Числа k и k+1 всегда взаимно простые

(если есть некоторое число d, являющееся делителем x и y, то число |x-y| тоже должно делиться на d. (k+1)-(k) = 1, то есть 1 должно делиться на d, то есть d=1, а это и есть доказательство взаимной простоты)

То есть мы доказали, что все нечетные числа могут быть представлены в виде суммы двух взаимно простых.
Исключением по условию будут являться числа 1 и 3, поскольку 1 вообще нельзя представить в виде суммы натуральных, а 3 = 2+1 и никак иначе, а единица в качестве слагаемого не подходит по условию.

2. Если число четное:
n = 2*k
Тут придется рассмотреть два случая:

2.1. k - четное, т.е. представимое в виде k = 2*m.
Тогда n = 4*m = (2*m+1)+(2*m-1).
Числа (2*m+1) и (2*m-1) могут иметь общий делитель только такой (см. выше), на который делится число (2*m+1)-(2*m-1) = 2. 2 делится на 1 и 2.
Но если делитель равен 2, то получается, что нечетное число 2*m+1 должно делиться на 2. Этого не может быть, поэтому остается только 1.

Так мы доказали, что все числа вида 4*m (то есть кратные 4) тоже могут быть представлены в виде суммы двух взаимно простых.
Тут исключение - число 4 (m=1), которое хотя и может быть представлено в виде 1+3, но единица в качестве слагаемого нам по-прежнему не подходит.

2.1. k - нечетное, т.е. представимое в виде k = 2*m-1.
Тогда n = 2*(2*m-1) = 4*m-2 = (2*m-3)+(2*m+1)
Числа (2*m-3) и (2*m+1) могут иметь общий делитель, на который делится число 4. То есть либо 1, либо 2, либо 4. Но ни 2, ни 4 не годятся, поскольку (2*m+1) - число нечетное, и ни на 2, ни на 4 делиться не может.

Так мы доказали, что все числа вида 4*m-2 (то есть все кратные 2, но не кратные 4) тоже могут быть представлены в виде суммы двух взаимно простых.
Тут исключения - числа 2 (m=1) и 6 (m=2), у которых одно из слагаемых в разложении на пару взаимно простых равно единице.

Ответ: 1,2,3,4,6

388. Леонид, 25 января 2010, 21:00:09
Марина, могут сбавить на у, а в % на 0,01у% - там речь о %

389. ЛёШкА, 25 января 2010, 21:07:02
6^{sqrt{6}+5} cdot 6^{-1 - sqrt{6}}.

подскажите как вот этот бред решать?!?!?!?!?!?!

390. Lexxus, 25 января 2010, 21:10:27
6^{sqrt{6}+5} cdot 6^{-1 - sqrt{6}}.


6^(sqrt(6)+5)*6^(-1-sqrt(6)) = 6^(sqrt(6)+5-1-sqrt(6)) = 6^4 = 36^2 = 1296

391. бабушка, 26 января 2010, 10:05:35
Уважаемые умники и умницы, возникли затруднения в решении задачи " на работу" : Маша и Настя моют окно за 20 минут, Настя и Лена - за 15 минут, Маша и Лена - за 12. За сколько минут они вымоют окно втроем.
Помогите с решением, и вообще, каков алгоритм решения таких задач?

392. Леонид, 26 января 2010, 10:50:00
ЛёШкА! При умножении степеней показатели сскладываются, основание остаётся тем же: корни уйдут, 5-1=4, 6 в 4-ой степени 1296.

393. Agony, 26 января 2010, 11:01:08
На pV- диаграмме показано, как изменялось давление идеального газа в зависимости от объема. Каково отношение работ газа на участках 1-2 и 1-4?

График выложил тут (не судите строго, рисовал в паинте): http://ifolder.ru/16111664

В чем, собственно, затруднение: в результате расчетов, работа газа на участке 1-4 = 0,т.к. V не изменяется... но ведь на ноль делить нельзя? Я что то не так делаю, или задача неправильная? Помогите плз..

394. каролина, 26 января 2010, 15:01:00
егэ-это тот случай, когда из хорошистки ты мнгновенно превращаешься в двоечницу!несправедливо оценивают!у нас в классе очень много людей которые хорошо знают алгебру,но в результате:у нас 20 двоек по пробному егэ!!!!учитель в шоке и мы тоже!!!как это понимать!подчёркиваю:задания были ну, очень лёгкие!!!!!!а в чём причина?!!!!

395. Алла, 26 января 2010, 22:25:10
Каролина!
Причина , видимо, в том, что вам и вашему учителю только кажеться, что вы знаете алгебру. Независимая экспертиза, как правило, вскрывает проблемы. Надо убрать состояние шока, проанализировать результат, и начать работать, время еще есть. Даже двоечника за это время можно "натаскать" на "4" (я не говорю о хорошистах), если этому двоечнику показать траекторию развития, а не убеждать его в том, что он "знает алгебру". Учителю надо собраться с силами и начать готовить вас решать задания из открытого банка (большинства этих заданий, к сожалению, в школьных учебниках нет)
Удачи на экзамене.

396. Леонид, 27 января 2010, 07:30:42
Ltxxus! Есть вопрос. Ваша консультация нужна. Задача, например В5 №5459: как понимать запись 0,15 в кв. - площадь каждого стекла? это 0,15 м. в кв. или 0,15*0,15=0,0225? Покажите пожалуйста расчёт, хотя бы по одной строке.

397. Леонид, 27 января 2010, 07:45:44
Для БАБУШКИ от дедушки! за х - Маша, за у -Настя, к - Лена (за минут мыли стекло). Производительность, успешность, скорость мытья соответсттвенно 1/х,1/у,1/к. Три уравнения: 1) 1/х + 1/у = 1/20; 2) 1/у + 1/к = 1/15;
3) 1/х + 1/к = 1/12 -производителььность в парах. Решив систему (долго и неудобно писать), ответ 10 мин.

Вопрос. От куда задача? В банке ЕГЭ 2010 таких нет - они проще В12. Это видимо из ЕГЭ предыдущих лет или олимпиадная для 7-8 класса.

398. Lexxus, 27 января 2010, 09:38:35
как понимать запись 0,15 в кв. - площадь каждого стекла? это 0,15 м. в кв.

Да, это 0.15 квадратных метров. Там просто пропущена буква "м".

399. Саша:), 27 января 2010, 10:33:47
Здравствуйте! помогите решить текстовую задачу, хотя бы алгоритм. задача: "даны 3 месторождения руды. объем добычи на них соответсвуют отношению 7:6:14. затем объем добычи на первом месторождении уменьшили на 14%, на втором тоже уменьшили на 14%. на сколькол нужно изменить объем добычи руды на третьем месторождении, если объем добычи руды на 3-х месторождениях не изменится"

400. Lexxus, 27 января 2010, 10:42:18
даны 3 месторождения руды. объем добычи на них соответсвуют отношению 7:6:14. затем объем добычи на первом месторождении уменьшили на 14%, на втором тоже уменьшили на 14%. на сколькол нужно изменить объем добычи руды на третьем месторождении, если объем добычи руды на 3-х месторождениях не изменится

Требуется, чтобы и до, и после изменения общий объем добычи, равный 7+6+14, сохранился. Составляем уравнение:
7+6+14 = 7*(1-14/100)+6*(1-14/100)+14*(1+x/100),
где x - это как раз на сколько процентов нужно увеличить объем добычи на третьем месторождении.
Решаешь, получаешь в ответе 13.

401. Леонид, 27 января 2010, 13:43:38
Спасибо,Lxxus? По В5 - всё понятно. И это важно, видимо будут сделаны поправки. По крайней мере, такого не встретитсч в настоящем ЕГЭ.

402. Саша:), 27 января 2010, 14:17:10
Lexxus, Спасибо:)

403. Саша:(, 27 января 2010, 14:29:51
простите за глупости, а можно узнать почему (1-14)/100?

404. Lexxus, 27 января 2010, 14:39:11
простите за глупости, а можно узнать почему (1-14)/100?

Не (1-14)/100, а (1-14/100). Большая разница, между прочим.

14% от чего-нибудь = 14/100 = 0.14 чего-нибудь.
Если что-то уменьшилось на 14%, то осталось 1-14/100 = 1-0.14 = 0.86 чего-нибудь = 86%.

405. Саша:), 27 января 2010, 14:45:23
Благодарю:)))))

406. бабушка, 27 января 2010, 15:10:57
Леониду спасибо за помощь, решала также, , но, видимо, запуталась в вычислениях, поэтому и не получился ответ - 10 минут
Задача из сборника "ЕГЭ 2010. Сборник тренировочных работ по математике" под редакцией Семенова и Ященко, тренировочная работа №1. задача В12.
Там еще интересная задача В1 в тренировочной работе №3, отличается от "автобусных билетов" и "сырков"

407. ангел, 27 января 2010, 16:38:48
Привет всем. помогите решить задачу.пожалуйста.
оля и аня занимаются оформлением документов за 504 минуты,оля и лена-за 420 минут,а лена с аней -за 840 мин.сколько девочкам понадобиться времени,если они втроем будут заниматься одним делом!
эта задача была на егэ 2009!

спасибо заранее!

408. Lexxus, 27 января 2010, 17:03:37
оля и аня занимаются оформлением документов за 504 минуты,оля и лена-за 420 минут,а лена с аней -за 840 мин.сколько девочкам понадобиться времени,если они втроем будут заниматься одним делом!

Так и написано - "одним делом"? O_o :))))

А если серьёзно, то тут мыслить удобно так. У каждой девочки есть своя скорость выполнения этой работы (величина, обратная времени, за которое она сделает работу в одиночестве, 1/t).
Обозначим эти скорости как v1, v2 и v3 (оля, аня, лена соответственно).

Получим систему из трех уравнений:
v1+v2 = 1/504
v1+v3 = 1/420
v2+v3 = 1/840

В принципе, можно честно решить систему, найти скорость выполнения работы каждой девочки в отдельности, а потом вычислить искомое время, которое будет равно 1/(v1+v2+v3).

Но проще - заметить, что если мы сложим все три уравнения, то получим
2*(v1+v2+v3) = 1/504+1/420+1/840

В левой части мы получили как раз удвоенную суммарную скорость всех троих - полшага до ответа:

2/T = 1/504+1/420+1/840

T = 2/(1/504+1/420+1/840)

Без калькулятора, конечно, это тяжко, но в ответе должно получиться 360.

409. Леонид, 27 января 2010, 18:55:44
Lexxus и всем, кто активно решает. Я снова по В5 №5475. В результате расчётов получилось две равных наименьших затрат по 9875 руб. Перерешивал, ошибки в вычислениях (чем чёрт не шутит) не должно быть. Может ли быть в предложении о выборе из трёх вариантов "ну самого дешёвого"?. Или как в функции о выборе отрезка большей (меньшей) длины из двух равных? (была у нас полемика). Дайте своё резюме! А может быть внести поправку для 2 случая (например) не 60, а 55 оплату за резку стекла. Посмотрите, пожалуйста, на это задание.

По пред. задаче был ответ для БАБУШКИ.

410. ангел, 27 января 2010, 20:29:25
Lexxus,ты молодчина! Преподаватели уверяют,что ответ 180! Хотя я не верю:)
у меня тоже получилось 360.

411. Ксения, 28 января 2010, 01:02:50
Здравствуйте!!! научите пожалуйста решать B 10!!
например, вот эту: В розетку электросети подключены приборы, общее сопротивление которых составляет Ом. Параллельно с ними в розетку предполагается подключить электрообогреватель. Определите (в омах) наименьшее возможное сопротивление этого электрообогревателя, если известно, что при параллельном соединении двух проводников с сопротивлениями и их общее сопротивление даётся формулой , а для нормального функционирования электросети, общее сопротивление в ней должно быть не меньше 30 Ом.

очень нужна ваша помощь!=)
Заранее спасибо

412. бабушка, 28 января 2010, 07:26:59
Lexxus! Огромное спасибо за выложенное решение задачи. Я поняла, в чем там "фишка", решение красивое. И хотя на экзамене нужно будет только поставить цифру ответа в клеточку и не писать ход решения, такой прием позволит сэкономить время и не наделать вычислительных ошибок.
Тут уже высказывалось мнение, что задания могут быть и не из банка данных, согласна с этим, поэтому и нужно прорешать несколько иные задачи.

413. @лено4ка, 28 января 2010, 16:57:01
подскажите, пожалуйста, ход решения задания В8 № 6045

414. Леонид, 28 января 2010, 17:37:02
Александр, от 20.01. Как с задачей о трубах? Есть такое, в книжице нашёл. Там на 3. Ты же писал то на 3, то в 3.

415. ЧеРт-)), 30 января 2010, 11:58:19
Все вы ботаны!!! занимаетесь ерундой!!!!!

416. надежда 2010, 30 января 2010, 13:24:00
не врубаюсь в задачу№19931, В-4,проверила ответ: 14,375 - это никак невяжется с условием: в треугольнике АВС АВ=23, АС=ВС, cosA=0.8, найтиАС.У меня получилось 18.4.Как это?

417. Леонид, 30 января 2010, 13:25:49
Лено4ка. 6045. к - угл. коэфф. касательно (прямой) к = 8 - это производная функции ( У штрих = 3 х (кв) + 2х + 8)) = 8, будет 2 корня x=0 и x==-2/3. Коль прямая является касательной, надо решить уравнение - их общие абсциссы, приравнять:
8х - 9 = х^3 + x^2 + 8x -9, x=0 x=-1. Ответ х=0. Разберёшься?

418. Karolina, 30 января 2010, 18:06:15
Отличная идея!!! Все должны помогать друг другу!
А ,кстати,могу ли я сдать ЕГЭ на территории РФ,будучи гражданкой Узбекистана? Кто-нибудь,подскажите,please!

419. Леонид , 30 января 2010, 18:54:51
Надежда! Проведи высоту СН в равнобедренном треугольнике. Тогда в прямоугольном треуг. по определению косинуса 0,8 = АН:АС (Н- середина АВ) и АС = 11,5 : 0,8. АС = 14,375.


Karolina - МОЖЕТЕ.

420. надежда 2010, 30 января 2010, 19:54:20
Спасибо!!! Это я лопухнулась: не дочитала условие, по инерции решала с прямоугольным треугольником АВС, как предыдущие задачи. Вывод сделала: надо читать условие до конца и внимательно.

421. Приветики, 30 января 2010, 20:50:41
помогите решить пожалуйста задачу
Из пункта А в пункт В, расстояние между которыми 6 км, отправился пешеход, а через 30 мин вслед за ним выехал велосипедист, скорость которого в 2,4
раза больше скорости пешехода. В пункт В велосипедист приехал на 12 мин раньше пешехода. Найдите скорость велосипедиста.

422. 390840(Slam), 30 января 2010, 21:10:44
Lexxus! не могу зайти под своим ником на "Задачи ЕГЭ по математике"! Ввожу свой ник на тестирование онлайн , все нормально заходит, но потом перехожу на "здешний" банк задач и выходит что вы не зарегистрированы...... ЧТо делать? Как зайти?

423. 390840(Slam), 30 января 2010, 21:18:00
ну все , зашел=)

424. Леонид , 30 января 2010, 23:16:59
НА Приветики! х км в мин - скорость велосипедиста, чтобы не забыть что писать в ответ. Тогда скорость пешехода х:2,4 км в мин.
Время велосипд. 6/х, пешехода 6/(х:2,4). (42мин. = 30 мин + 12 мин).
Уравнение: 6/(х:2,4) - 6/х = 42 мин. (6*2,4)/х - 6/х = 42.
14,4/х - 6/х = 42. 8,4 = 42 х, х = 0,2 км в мин или 12 км в час. В каких ед. просят ответ?

425. Ксения, 30 января 2010, 23:27:14
Ну помогите мне, пожалуууууйста!!!!!

426. Жанна, 31 января 2010, 10:08:35
Спасибо за банк данных! Мне уже за 50.Моему сыну сдавать ЕГЭ.Живу в таком месте,что совета спросить не у кого.Математик постоянно на больничном.Решаю сама.Потом объясняю сыну то,в чем он не может разобраться.Прорешали задания В 1-9 из Тематической тетради Ященко,кимы.А сейчас открыли Ваш банк данных и тренируемся решать задани В9.Задания из раздела геометрии.Вчера решили задачу,где нужно было сообразить и приравнять объемы параллелепипедов.Радовались вместе с сыном.Планирую этой теме посвятить еще 7 занятий,а потом перейти к следующим заданиям.Что не будет получаться ,буду обращаться е Вам! У Д А Ч И !

427. 390840(Slam), 31 января 2010, 17:33:22
Только что зашел в "Прогресс" и увидел, что на 27% заданий получено более 5 ответов! А у нас "решенные" только 12%. Я уже прорешал более 700 задач и мне довольно-таки часто начали попадаться мною уже решенные задачи.
Lexxus, может поставишь в решенные все 27% задач?!

428. Бобринева Т. М., 31 января 2010, 18:03:33

У меня нет изображений: графиков в просмотре задач, рисунков. А были. что я сделала не так?

429. Julie, 31 января 2010, 19:47:32
идея хорошая))

430. ninelechk@, 1 февраля 2010, 08:09:59
Lexxus!Спасибо тебе огромное за твои труды, и, дабы помочь тебе, хочу обратить твое внимание на задание B7 (4491). Ответ там дают 2, хотя по свойству логарифмов там выходит 9. Прошу, посмотри пожалуйста =)))))))))))))

431. Lexxus, 1 февраля 2010, 09:34:48
Ответ там дают 2, хотя по свойству логарифмов там выходит 9.

Классический случай. Радостно вспомнив про то, что сумма логарифмов равна логарифму произведения, ты начисто забываешь про сам логарифм.
Действительно, 1.8*5 = 9. А вот логарифм по основанию 3 от этой девятки как раз равен двум.

432. Леонид , 1 февраля 2010, 10:47:39
Что по двум одинаковым ответам в В5 из трёх вариантов. Обращался 27.01

433. Lexxus, 1 февраля 2010, 10:53:15
Что по двум одинаковым ответам в В5 из трёх вариантов. Обращался 27.01

Тебе не кажется, что твой вопрос, мягко говоря, несколько странен? Ты что, в таком случае что-то другое в ответе напишешь?

434. Леонид , 1 февраля 2010, 11:13:30
Не кажется. Грамотность задачи под сомнением.

435. Алла, 1 февраля 2010, 15:12:39
Здравствуй, Леонид!
Не понимаю в чем ты видишь проблему. В задании нужно указать сумму, которую нужно заплатить за самый выгодный заказ, а не фирму в которой будешь этот заказ оформлять. В двух фирмах наименьшая (одинаковая) сумма 9875 рублей, даешь ответ 9875 и выбираешь ту фирму, которая ближе к дому (если говорить о практичеком применении этого задания)

436. Леонид, 1 февраля 2010, 16:52:18
Алла, абсолютно нив чём не вижу проблемы. Тесты д.б. грамотными. речь идёт о базовом математическом уровне знаний. Не слишком ли много допущений в расчёте на каждого ученика. То отрезок найти наибольшей длины среди равных, то одинаковые ответы в вариативных заданиях (и дело совсем не фирмах) , то параллелограм - прямоугольник - кваждрат- ромб - это трапеции (по этой формуле и площадь треугольника "с нулевым основании трапеции" вычисляется). И таких допущений в математике можно уйму придумать. Чистить задания надо, коль выходят на этот "проект". Должно быть всё как положено. Без всяких смущаемых ситуаций побочных. Помнить, ученик в условиях экзамена, стрессовой ситуации. Зачем здесь такие вкщи, выбивающие из калеи. Тогда давайте все задания в вещи такие обратим. Почему один будет получать нормальное задание, а другой с побочными загадками. Ваше мнение, проблем нет. Моё- тоже, нет проблем,но считаю, надо навести порядок!

437. Lexxus, 1 февраля 2010, 17:07:49
Леонид, следуя твоей логике, все варианты экзамена нужно вообще сделать одинаковыми. Ну, чтобы все были уж совсем в одинаковых условиях. Иначе не получится.

Кто-то, может, умеет только до тысячи считать. А в тех же B5 встречаются задания, где все вычисления производятся как в пределах тысячи, так и выше. Условия не равные, не порядок.

438. Леонид , 1 февраля 2010, 18:01:02
Уважаемый Lexxus! Я ведь совсем не о том. Причём та одинаковость. Там тысячи, да, но без побочных "загадок". Загадок, не подходящих для массовой экзаменационной ситуации. Я думаю меня поймут правилльно. Автоматические шаблоны проблема здесь, которые, выдимо, недосуг поправить. Простите, но если взять выпуклый четырёхугольник с перпендикулярными диагоналями, его тоже можно "признать" квадратом, ромбом или наоборот. Площадь находится одинаково - полупроизведение диагоналей. Но не для массового ученика. В заданиях (базовых) и без того есть над чем трудиться. Больше в такие полемики не вступаю. Бесполезно, хотя, считаю вопрос очень, серьёзным. Но и легко решаемым, теми кто работает над тестами.

439. Eva, 1 февраля 2010, 21:28:44
В магазине "Четверочка" проходит рекламная акция: тем, кто покупает 5 шоколадок, дают 6-ую в подарок. До проведения акции, чтобы купить 8 шоколадок, нужно было иметь не менее 200 рублей. Сколько шоколадок можно получить на 200 рублей во время акции??

Что означает НЕ МЕНЕЕ? Т.е. можно купить на 210,220,230?? или все таки на 200??вроде понятно, что ответ 9, но как то сформулировано непонятно. Объясните пожалуйста.

440. Lefi, 1 февраля 2010, 21:50:17
Кому не трудно, объясните пожалуйста как быстро решать В10 с коренью. Некоторые не очень трудные. Но есть относительно сложные. Не знаю метод решения. Прошу объяснить на примере "корень 291(в квадрате) - 216(в квадрате), корень закрывается". Заранее спасибо.

441. Lexxus, 1 февраля 2010, 21:53:32
Что означает НЕ МЕНЕЕ?

То и означает. 8 шоколадок стоят ровно 200 рублей. И даже если ты ИМЕЕШЬ сто тыщ мильёнов, ты все равно же можешь купить 8 шоколадок, логично?
"Иметь" же не значит "заплатить", правда? :)

442. @лено4ка, 1 февраля 2010, 22:35:10
Леонид, 30 января 2010, 13:25:49

спасибо большое.

443. MSever, 1 февраля 2010, 23:00:59
В магазине "Четверочка" проходит рекламная акция: тем, кто покупает 5 шоколадок, дают 6-ую в подарок. До проведения акции, чтобы купить 8 шоколадок, нужно было иметь не менее 200 рублей. Сколько шоколадок можно получить на 200 рублей во время акции??

Эти задачи убрали из открытого банка.

444. MSever, 1 февраля 2010, 23:56:51
Хочу узнать мнение специалистов по поводу решения задачи В8 №6421 (условие поместить сюда не могу, т.к. там график).

445. Милолика, 2 февраля 2010, 00:13:37
9 чтоли

446. Лорд Гэлеон, 2 февраля 2010, 02:14:27
Задание http://live.mephist.ru/show/mathege2010/view/id/2559/ 3 получится, а не 4.

447. Лорд Гэлеон, 2 февраля 2010, 02:18:10
Всё понял. Забыл, что нужно округлить до целого.

448. Леонид , 2 февраля 2010, 09:13:26
MSever! Очень высоко разнообразие задач В8. В школе, к учителю: задачам уделить, внимание. Они не сложные, необходимо владеть понятиями функция - производная- касательная- тангенс угла наклона-монотонность и т. д. Что касается данной задачи: на рисунке график функции! касательные параллельны у=6 (сравни у=кх+в), т.е. к=0, тангенс угла наклона касательной равен 0. угол равен 0 - прямая параллельна оси ОХ. Коль tga=0, т.е. производная функции равна 0 - это вершинки графика! функции (вверху и внизу). Только в них касательные параллельны ОХ.


2)Кто сказал, что задания о шоколадках убрали?

449. MSever, 2 февраля 2010, 12:09:21
Леонид!
За столь подробные рекомендации спасибо, но вот школы, а тем более учителя рядом нет. Да, честно говоря, я и сам их хорошо решаю.
В этой же задаче меня интересует точка х = 0.
Это явно точка перегиба. Первая производная может быть в ней равна нулю (вспомним хотя бы функцию у = х^3).
Почему же Вы считаете, что в ней производная не равна нулю?

2) ну раз я написал, то получается, что я и сказал)
А если Вас что -то смущает, то возьму на себя смелость и порекомендую заглянуть в открытый банк.

450. Леонид , 2 февраля 2010, 12:41:59
Там дан график функции, при х=0. у=0. Где гарантия, что и её производная равна нулю. А когда будем иметь у=х^3, по ней и будем говорить. У неё нет касательных парал. ОХ - нет экстремума.


Спасибо. Открытый банк заданий постоянно просматриваю.

451. Ксюша, 2 февраля 2010, 13:20:10
Лорд. ответ 4, а не 3. Не округляем, берём "целое + 1", так как 3 упаковок таблеток не хватит.

452. MSever, 2 февраля 2010, 13:40:43
Леонид. "Где гарантия, что и её производная равна нулю."

А где гарантия, что она там не равна нулю?

"А когда будем иметь у=х^3, по ней и будем говорить. У неё нет касательных парал. ОХ - нет экстремума.*

Ну здесь я в корне с Вами не согласен. Касательная параллельная оси Х может быть не только в точке экстремума, например, как раз у функции у = x^3 в точке х= 0 уравнение касательной имеет вид у = 0, а экстремума в этой точке у функции нет. А у функции у= модуль х точка х = 0 - точка минимума, а касательной в этой точке нет.

453. Алла, 2 февраля 2010, 18:06:05
Здравствуй, Леонид!
Возмущение по поводу содержания задач (1.02.2010) полностью поддерживаю. Банк задач "чистить " надо и думаю, что чистить будут. На ЕГЭ таких "ляп" еще не было ( я думаю, что варианты заданий ЕГЭ прорешиваются и перепроверяются ни один раз и разными людьми). Надеюсь проблемы, которые возникают в задачах открытого банка, на экзамене не возникнут, их просто не пропустят.

454. Lexxus, 2 февраля 2010, 20:00:38
Касательная параллельная оси Х может быть не только в точке экстремума, например, как раз у функции у = x^3 в точке х= 0 уравнение касательной имеет вид у = 0, а экстремума в этой точке у функции нет. А у функции у= модуль х точка х = 0 - точка минимума, а касательной в этой точке нет.

Таки да.

Леонид, поосторожнее: ты здесь уже приобрел определенный авторитет, а значит, всё больше становится тех, кто тебе верит, и кому могут навредить твои ошибки.

Это явно точка перегиба. Первая производная может быть в ней равна нулю (вспомним хотя бы функцию у = х^3).
Почему же Вы считаете, что в ней производная не равна нулю?

А здесь уже не прав ты, MSever.
Да, там, похоже, точка перегиба. А точка перегиба бывает там, где обращается в ноль не первая, а вторая производная.

Если провести касательную к графику в задаче 6421 в точке (0,0), то сразу станет понятно, что ее угловой коеффициент точно не равен нулю, а значит, и производная в этой точке в ноль не обращается.

455. MSever, 2 февраля 2010, 20:41:48
"А здесь уже не прав ты, MSever"

А разве я утверждал, что она там равна нулю? Я кажется спрашивал почему Вы считаете, что она не равна нулю?

"А точка перегиба бывает там, где обращается в ноль не первая, а вторая производная."

Это конечно так. А если в точке и первая и вторая производная обращается в ноль? Тогда как быть?

"Если провести касательную к графику в задаче 6421 в точке (0,0), то сразу станет понятно, что ее угловой коеффициент точно не равен нулю, а значит, и производная в этой точке в ноль не обращается."

Вот это уже ближе к тому о чем я спрашивал!
Т.е. Вы делаете вывод только из визуальных соображений?
Хотя к словам "сразу станет понятно" я бы относился осторожно))
Для меня, например, сей факт не кажется столь очевидным.

456. Людмила, 2 февраля 2010, 20:46:00
Задача В10 из диагностической работы вариант 101. В открытом банке задач аналогичные №28015-28025

При температуре 0 град. ж/д рельс имеет длину &#8467;0=15м. Зазор между рельсами 6,3мм. При тепловом расширении зазор уменьшается. Длина рельса меняется по закону &#8467;=&#8467;0 (1+аt), а=1,2*10^-5. При какой температуре зазор исчезнет?

Так как нагреваются оба соседних рельса, то &#8710; &#8467; нужно разделить на 2
6,3&#247;2=3,15
Имеем: &#8467; - &#8467;0 &#8804;&#8710; &#8467;
15-15&#9679;(1+1,2&#9679;10-5t) &#8804;0,00315
t=17,5
В ответе 35. Кто прав?

457. Людмила, 2 февраля 2010, 20:57:13
Правка

При температуре 0 град. ж/д рельс имеет длину l0=15м. Зазор между рельсами 6,3мм. При тепловом расширении зазор уменьшается. Длина рельса меняется по закону l=l0 (1+аt), а=1,2*10^-5. При какой температуре зазор исчезнет?

Так как нагреваются оба соседних рельса, то 6,3 нужно разделить на 2
6,3:2=3,15
Имеем: l - l0 =0,00315
15-15*(1+1,2*10^-5t) =0,00315
t=17,5
В ответе 35. Кто прав?

458. Леонид , 2 февраля 2010, 20:59:14
Lexxus, спасибо. Буду иметь в виду. Я не стал акцентировать внимание на второй производной (согласен). Она в школе, практически, не рассматривается. Только из этих соображений. И там же не конкретная функция. Вернее так, "конкретный" общий график, где не надо вести об этом речь - о точке перегиба.


Алла!!! И Вам выражаю благодарность за поддержку чистоты математических понятий и тестов. Надеюсь, что так и будет!!!

459. Lexxus, 2 февраля 2010, 21:39:54
Так как нагреваются оба соседних рельса, то 6,3 нужно разделить на 2
6,3:2=3,15
Имеем: l - l0 =0,00315
15-15*(1+1,2*10^-5t) =0,00315
t=17,5
В ответе 35. Кто прав?

Да, нагреваются оба рельса. Но и в обе стороны: зазоры ведь исчезают с обоих концов каждого рельса.
Поэтому делить пополам зазор не нужно, и прав таки ответ.

460. Леонид , 3 февраля 2010, 17:03:45
Lexxus! Помогите 27379163 разложить на просттые множители. Выделил 7*17*230077, дальше не могу. М.б. 230077 - число простое? Ну, очень необходимо. к ЕГЭ это не относится. Заранее БЛАГОДАРЕН.

461. Lexxus, 3 февраля 2010, 17:19:16
М.б. 230077 - число простое?

Да, простое.
Заданьице, однако, то ещё. Факторизация чисел - задача не для бумажки и ручки.

462. RuS92, 3 февраля 2010, 17:42:48
Всем привет!Вот допустим мы решаем "нерешенные задачи", к ним ведь баллы не добавляются в рейтинг.А когда эти задачи перейдут в раздел "решенные" добавится ли 10 очков?

463. Lexxus, 3 февраля 2010, 17:44:04
А когда эти задачи перейдут в раздел "решенные" добавится ли 10 очков?

Да, добавится.

464. Леонид , 3 февраля 2010, 18:28:21
Lexxus! Большое спасибо! Мне такого ответа достаточно,

465. >>>>ИЛЮХА<<<<, 5 февраля 2010, 14:26:10
быстрейбы уж расепаться с етой хренью!!!!!!!!!!у мя уж башка раскалывается!!!!!!!от етого ЕГЭ!!!!!!!!!!!!!

466. Леонид , 5 февраля 2010, 14:33:15
Для "БАБУШКИ" к задаче от 26.01 (10:15), ответ 27.01 (07.45). Решив систему, ответ 10.

Сложив все 3 уравнения, будем иметь 2(1/х + 1/у + 1/к) = 1/20 + 1/15 + 1/12;
2(1/х + 1/у + 1/к) = 12/60 = 1/5; то 1/х + 1/у + 1/к =1/5 = 1/10 (по рекомендации Lexxus быстро считается). Ответ 10.

"Оживляю этот раздел" данный

467. RuS92, 6 февраля 2010, 14:27:10
А сколько должно быть заданий в "нерешенном" разделе,чтобы оно добавилось в решенные!

468. 390840(Slam), 6 февраля 2010, 16:37:17
Lexxus, пополни пожалуйста все задачи в в открытый банк задач, а то всего, B-9 на сайте http://mathege.ru 306, а в здешнем банке только 87! Также обстоят дела и с остальными заданиями B!

469. Жанна, 7 февраля 2010, 18:53:11
Уважаемые помогите решить задание В11.Найти наименьшее значение функции на отрезке 0;пи/2. y=13х-9sinх+9.Сначала я нашла производную.y=13-9cosx.А дальше надо производную приравнять к 0 или нет?И как понять эта производная положительная или отрицательная?Вот тут до меня никак не доходит.Потом если производная отрицательная,то функция убывает.И значение будем брать слева,т.е. 0.А если производная положительная,то и функция будет возрастать.И значение берем в правом углу.Точку пи/2.Не могу понять как определить производную.Какая она?

470. Леонид , 7 февраля 2010, 19:56:20
Жанна. 13-9cosx присмотрись, выражение всегда положительно, косинус от - 1 до 1. От 13 отнимаем 9 при наибольшем косинусе. Какой РЕЗУЛЬТАТ? Итак, производная положительна, функция значит возрастающая, то наименьшее значение при х=0. 13-9кос.0+9 = 13-9*1+9=13.

471. Lexxus, 8 февраля 2010, 09:46:35
Lexxus, пополни пожалуйста все задачи в в открытый банк задач, а то всего, B-9 на сайте http://mathege.ru 306, а в здешнем банке только 87! Также обстоят дела и с остальными заданиями B!

Снова просканировал mathege.ru, все новые задачи (1389 штук) добавил.

Попозже сделаю, чтобы тут у меня помечались задания, которые оттуда уже убраны. Удалять их я особого смысла не вижу, но пометить стоит.

472. Жанна, 8 февраля 2010, 15:30:02
Спасибо Леонид! Правда sin0=0 и ответ будет 9.Будем разбирать далее задания В11. 11 будет пробный ЕГЭ.

473. Леонид , 8 февраля 2010, 15:55:56
Жанна, немного ещё поделюсь по В11- тригонометрия- наиболшее (наименьшее). Если нет точек на отрезке, приравняв производную к 0. Да - находи функцию на концах отрезка, выбирай - что просят. Если посмотреть в этих случаях производная либо -, либо + И берёшь один конец отрезка - что просят. Получается легко и быстро. Может оказаться, что кроме концов отрезка, есть ещё точки попадающие в него - находи функцию в них и на концах - выбирай нужное. Будут вопросы обращайся. На тригонометрию в банке заданий я установил возможных разновидности - случая по 4 виду функций и заданного отрезка. Очень хорошо помогает в них единичная окружность, чтобы не перебирать К для определения корней производной, попадающих в отрезок.

474. Yura)), 8 февраля 2010, 16:18:20
Мдя с егэ запар полный... У меня знакомый доктор наук ехал с Москвы в Питер и за 4 часа не смог решить... а это доктор наук по математике... пипец ваще государству нужен рабочий клас...

475. Леонид , 8 февраля 2010, 21:59:39
Lexxus! Есть вопросы по В9. Вы пополнинили перечень задач из Открытого банка.1) Что-то непонятно с чертежами задач, где вершина куба или середина ребра (ребро видимо дано) куба является центром шара данного радиуса найти площадь поерхности шара, лежащей внутри куба...- страница чистая, в левом внешнем углу мелкий кусок чертежа. Или моя техника ерундит ? Например, № 25841. Другие задачи скопировались хорошо, делал в одно время. И фраза в условии "Середина ребра куба со стороной..."

2) Нет ли опечатки в задачах №№ 2586(1,3,5):"Найти объём треугольной пирамиды АВСВ1"? Если нет, то каков смысл и идея предложенного на чертеже сечения параллелепипеда через три вершины, нележащие в одной грани - АД1С? Ибо есть точно такие же формулировики и с такими же данными без сечения АД1С, но с сечением АВ1С - эти задачи понятны.

476. Леонид , 8 февраля 2010, 22:32:51
Сравните (задачи) - задачи 25865 и 25855 из В 9. Если можно, то решение,пожалйста, обех задач.

477. Lexxus, 8 февраля 2010, 23:12:30
Например, № 25841.

М-да, хороший чертежик... Эта задача тоже нормально скопировалась, на mathege.ru точно такая же хреновина.
Но тут, на самом деле, всё просто. Центр шара радиусом 0.95 лежит на середине ребра куба, а длина этого ребра равна 1.9. То есть, внутри куба находится ровно четверть шара. Ну, и четверть сферы, соответственно.
Значит, в ответе надо указать площадь сферы (4*pi*R^2), деленную на 4, и деленную на pi.

Нет ли опечатки в задачах №№ 2586(1,3,5):"Найти объём треугольной пирамиды АВСВ1"? Если нет, то каков смысл и идея предложенного на чертеже сечения параллелепипеда через три вершины, нележащие в одной грани - АД1С?

Да нет, в условии всё нормально. Просто сделали универсальный чертёж на несколько типов задач.

Сравните (задачи) - задачи 25865 и 25855 из В 9. Если можно, то решение,пожалйста, обех задач.

Они абсолютно одинаковые.
И решение одно и то же - объем пирамиды равен трети произведения площади основания на высоту. Или трети объема описанной вокруг неё призмы.
Если описать вокруг нашей пирамиды призму (с основаниями ABC и A1B1C1), то она займет как раз половину параллелепипеда.
То есть в ответе будет объем параллелепипеда, деленный пополам и еще на три.

478. Леонид , 8 февраля 2010, 23:53:51
Спасибо, Lexxus! Всё понятно, если не появится "Найти объём пирамиды АД1СВ1" там, где то самое сечение. Я правильно понял: в любом случае находить объём пирамиды АВСВ1?. А её объём шестая часть объёма параллелепипеда. И по шарику всё понятно.

479. Жанна, 9 февраля 2010, 06:02:01
Уважаемый Леонид! огромное спасибо за Ваши советы.Возник еще один вопрос по заданию В11. Из банка данных это примеры нр 3597 и нр 3693.Вот один из них. Найти наибольшее значение функции на отрезке -пи4 и 0. y=5tgx-5x+4 . Получаем производную y=5*1/cos^2x-5 .Если cosx положительный и равен 1 или -1,то производная получается равна 0. как рассуждать далее?

480. Леонид , 9 февраля 2010, 09:59:14
Да, Жанна, это один из видов,о которых я говорил. Рассуждаю так. Заметь, коэффициенты х и тангенса равные. Вынеси 5 и приведи к общ. знам., имеем
(1-кос.кв. Х)/кос.кв.Х = 0. Скобка здесь всегда больше 0 - кос. от -1 до 1, а кос.кв. Х от 0 до 1. (ДА?), (кроме тех о которых ты пишешь 1 и -1). В данном задании это (нули производной) неважны: производная, имея нули, проходя их, знака не меняет. Положительна производная - функция возрастает - наибольшее значение принимает в правом конце отрезка.
Если будет в задании наоборот, например 4Х - 4Тг - производная будет всегда отрицательной, не меняя знака при переходе своих нулей - функция убывает - далее конец отрезка, в зависимости от вопроса наибольшее (наименьшее). В других заданиях коэффициенты Тг и Х в 2 раза - там нули производной входят в прмежуток (какие, зови ед. окр.) и т.д. Извини. что не переходил на англ.яз. ДОЛГО. Думаю поймёш! Если что, ПИШИ.

481. RuS92, 9 февраля 2010, 14:31:12
В задаче B8 (7347) http://live.mephist.ru/show/mathege2010/view/id/7347/ правильный ответ пишет что -11, а я думаю правильный ответ -5

482. Lexxus, 9 февраля 2010, 16:10:14
В задаче B8 (7347) http://live.mephist.ru/show/mathege2010/view/id/7347/ правильный ответ пишет что -11, а я думаю правильный ответ -5

Там написано, что на графике изображен график ФУНКЦИИ.
Вот если бы там был график производной, то да, было бы -5.

483. Жанна, 9 февраля 2010, 18:59:02
Спасибо Леонид!Ваши советы очень полезны для меня.С тангенсами все понятно.Прорешала задания типа нр 3535.В этих функциях встречается пи.Но вот нр 3427 никак не получается. Производная 2пи это 0 ? Задание такое :найти наименьшее значение функции на отрезке 0;пи/2 y=6+2пи-8х-8корень квадратный из 2-х умножить на cosx.Далее пошли функции типа нр 4101.Найти точку минимума y=(x-2)^2*е^x-5.Какую формулу надо применить для нахождения производной? С уважением !

484. Леонид , 9 февраля 2010, 23:16:43
Жанна! № 4101. Здесь производная произведения:
у штр. = 2(х-2)*е в ст. (х-5) + (х-2) в кв.* е в ст.(х-5) = ((х-2)*е в ст.(х-5)). Имеем у штр. = (х-2)*е в ст. (2 +(х-2)) = х(х-2)*е в ст. (х-5), приравняв О, получим 2 точки :х = 0 и х = 2. На числовую ось их, методом интервалов знаки производной. Это "+" "-" "+". Ф-я возрю, убыв. возр., то минимум в точке х=2. Вроде не ошибся - внимательно смотри. Да. здесь участвовала и показательная ф-я с основанием "е"- производная её равна самой функции, но имей ввиду: в этих заданиях участвует и производная сложной функции. Произв. (х-2) в кв. = 2(х-2)*(х-2)штрих = 2(х-2), т.к. коэфф. при х 1. Может быть и е в ст (5-х), её производная будет (-е в ст.(х-5))! Производная произведения повтори - с логарифмами и показательной функцией - идёт часто! И производная сложной функции- функции, содержащей линейную - в учебнике есть это свойство. Другие позже, не понял условия.

485. Леонид , 9 февраля 2010, 23:50:32
Жана! Ну и что, встречается пи в 3535! Производную взяла, а 24/пи - это больше чем даже 7 (прикинь). Видно, производная 6 кос. Х - (больше, чем 6) отрицательная. Тебя пи напугало. Это задание из самых просты.
№ 3427 Это, когда млгутыть точки на отрезке, кроме его концов. Производная: -8 - 8 корней из 2 *(-син. Х) = 0, син. Х = корень из 2/2.. Не пиши по формуле. На ед.окр. на ОУ отложи корень из 2/2, проведи параллельную ОХ и увидешь х = пи/4 попадает в отрезок 0;пи/2. Находи значение функции у(0), у(пи/4), у(пи/2). выбирай наименьшее из того, что получится (просят). Всё не считай. Бери "хороший ответ". Это у(пи4) = -2 - проверь.

486. ирина, 11 февраля 2010, 18:17:31
мы готовимся к Егэ по математике

487. жанна, 11 февраля 2010, 18:39:02
Огромное спасибо за Ваши советы,Леонид!С этими все понятно.А вот еще одна функция найти наибольшее значение на отрезке (5пи/4;17пи/12) функция у=4cos(x-пи/12). С уважением.

488. RuS92, 11 февраля 2010, 18:51:02
А почему баллы не добавляются?(когда из нерешенных задач перешел в решенные).

489. Lexxus, 11 февраля 2010, 19:07:52
А почему баллы не добавляются?(когда из нерешенных задач перешел в решенные).

Значит, неправильно решил.

490. Леонид , 11 февраля 2010, 20:33:01
Жанна, не понял задания. Производная: -4 син.(х - пи/12) =0. х-пи/12 = пи*К, х = пи/12 +пиК .Промежуток в градусах от 225 до 255. х = (переберая К) = 15,195,375, -175 - вроде нет (проверь тщательнее). Находи на концах, выбирай. Попадающие точки в отрезоки на концах, выбирайнаибольшее.

491. Lexxus, 11 февраля 2010, 22:39:03
найти наибольшее значение на отрезке (5пи/4;17пи/12) функция у=4cos(x-пи/12)

Находим производную:

y'(x) = -4sin(x-пи/12)

x-пи/12 лежит между (14пи/12 и 16пи/12), то есть (7пи/6 и 4пи/3). Это нижняя правая четверть единичной окружности, там везде синус отрицательный, а значит, наша производная строго положительна.
И исходная функция строго возрастает.
Вычисляешь ее значение в правой границе отрезка, получаешь ответ:

4*cos(4пи/3) = 4*(-1/2) = -2

492. АРТЕМ, 13 февраля 2010, 13:45:53
vyt ye;ys jndtns yf dfhbfyn 220

493. Артём, 13 февраля 2010, 21:22:51
№6422, объясните пожалуйста как делать!

494. Леонид , 13 февраля 2010, 23:18:08
Твою заачу не открывают. Пиши о чём заача. По наомерам искать долкго перехоить туда-сюда.

495. Lexxus, 13 февраля 2010, 23:39:11
6422: изображен график функции, требуется найти количество целых точек, в которых производная функции положительна.

Производная положительна = функция возрастает.

На графике видно, что функция возрастает на участках (-5, примерно -4.8), (примерно 1.5, примерно 2.5) и (примерно 4.3, 5).
В эти интервалы входит всего одна целая точка (2). Значит, правильный ответ - 1.

496. RuS92, 14 февраля 2010, 12:05:06
Не могу решить задачу,помогите пожалуйста составить уравнение:

На изготовление 16 деталей первый рабочий затрачивает на 6 часов меньше, чем второй рабочий на изготовление 40 таких же деталей. Известно, что первый рабочий за час делает на 3 детали больше, чем второй. Сколько деталей в час делает второй рабочий?

497. Леонид , 14 февраля 2010, 14:29:29
х - елает в час второй. (х+3) - первый в час. 16/(х+3) - время первого, время второго 40/х. Уравнение, время второго на 6 больше: 40/х - 16/(х+3) = 6.
20(х+3) - 8х = 3х(х+3). х = 5.

498. RuS92, 14 февраля 2010, 17:11:18
Cпасибо большое!

499. RuS92, 14 февраля 2010, 17:36:14
На каком сайте можно скачать задания по математике ЕГЭ 2010 часть С.!заранее спасибо!(с решениями)

500. Анжела, 14 февраля 2010, 17:49:07
Помоги пожалуйста решить задачу В4:
длины двух сторон треугольника 4 и 6. Найдите наибольшее целое значение длины третьей стороны.

501. Lexxus, 14 февраля 2010, 18:46:02
Помоги пожалуйста решить задачу В4:
длины двух сторон треугольника 4 и 6. Найдите наибольшее целое значение длины третьей стороны.

Третья сторона треугольника должна быть меньше суммы двух других. То есть меньше 10.
Значит, 9.

502. надежда 2010, 14 февраля 2010, 18:52:16
Попалось задание В-3 №3069:решить уравнение: корень квадратный из 2Х-51=9, решаю, ответ: Х=66, но в просмотре указывается Х=15, значит в условии должно быть : корень квадратный из 2Х+ 51= 9? Посмотрите, пожалуйста.

503. Леонид , 14 февраля 2010, 19:03:38
Мониторинговые в ноябре и декабре- заходи в "Открытый банк заач по математике ЕГЭ 2010", Там есть разел "Проверочные работы", кликни и жди. Открывай то,что нужно. Если интересуешься банком заданий С, мне бы тоже хотелось его увидеть. По-моему такого ещё нет. Есть - подскажите где!!!!

504. Леонид , 14 февраля 2010, 20:24:55
Надежда. В открытом банке под корнем 2х+51

505. Леонид , 15 февраля 2010, 09:40:46
Lexxus! Что то не идёт с задачей В12 №5875, она в решённых с ответом 31. Посмотрите, пожалуйста, перепроверял несколько раз - дискрименант отрицателный. Не пойму в чём дело.

506. Алла, 15 февраля 2010, 11:11:30
Леонид!
Задача № 5875:
x - производительность первого рабочего
837/(x-4) -713/x=8
837x-713x+713*4=8x(x-4)
2x^2-39x-713=0
положительный корень 31

507. Леонид , 15 февраля 2010, 12:55:21
Спасибо, Алла. Всё так, проскакивал вычислительную ош., думая, что там её не могло быть. ВНИМАТЕЛЬНОСТЬ!

508. Elli, 15 февраля 2010, 17:17:58
Послезавтра пробный ЕГЭ. Помогите решить задачу, плиз. В прямом круговом цилиндре диаметр нижнего основанияАВ равен 6, точка С- середина дуги АВ, высота цилиндра АД равна 6. Найти угол между прямой АД и плоскостью ДВС. Мне попалась на тестировании, смогла найти косинус. получилась дробь с корнями.2 корня из 26/13. поэтому угол найти не смогла. может ошибка в вычислениях?

509. Леонид , 16 февраля 2010, 09:23:04
Точный текст заачи напишите. Точно, угол надо найти. Это, похоже, задача типа С2. Коль точка С сереина дуги, то треуг. АВС - равнобедренный прямоугольный, т.е. ВС перпеникуляр ДС по т. о 3-х перпенд. Легко находится АВ и ДС и треуг. АДС - прямоугольный. Угол межу прямой и плоскостью-угол межу этой прямой и её проекцией на плоскость. Пл. СДВ перпендикулярна пл. АДС, ну и всё.



510. Леонид , 16 февраля 2010, 09:32:51
Lexxus! Неужели, ничего нельзя сделать с "Каким будет ЕГЭ..."? Это же ужас.

511. Гаппазов Риколай, 16 февраля 2010, 10:58:42
помогите на демонстрационном варианте по математике 2010 КИМ

дайте ответ по егэ

513. ГаппазовНиколай, 16 февраля 2010, 11:02:58
помогите на демонстрационном варианте по математике 2010 КИМ

514. Оля, 16 февраля 2010, 14:15:22
Он же полностью решён. Достань, там всё есть.

515. Elli, 16 февраля 2010, 15:08:24
Леонид! Спасибо, кажется разобралась. Надо найти угол АДС получается.Тогда и ДС искать не обязательно. найти тангенс угла АС/АД=корень из 2 попалам. А как правильно записать ответ в бланке?

516. Леонид , 16 февраля 2010, 16:30:44
Эта задача, видимо, С2 - пишется решение, ответ может быть каким угодно. Он в клеточки не пишется. Может быть таким, как у тебя. Элли,где взяла задачу и продублируй её точнее. Хорошо?

517. Наташа, 16 февраля 2010, 16:37:17
Помогите пожалуйста!!!
Из пункта А в В одновременно выехали два автомобилиста. Первый проехал с постоянной скоростью весь путь. Второй проехал первую половину пути со скоростью 24км/ч, а вторую половину пути-со скоростью, на 16км/ч большей скорости первого, в результате чего прибыл в В одновременно с первым. Найдите скорость первого.

518. Леонид , 16 февраля 2010, 16:58:17
Наташка! х - скорость 1-го. Его время 1/х. Время второго о,5/24 + 0,5/(х+16) - по половине пути. Время оно (оновременно) 1х = 0,5/24 + 0,5/(х+16). 1 - весь путь, 0,5 - половина пути.

519. Ангелина, 16 февраля 2010, 20:36:03
Очень хороший сайт)))Спасибо-очень помогает подготовится))...ещё бы С часть и вообще -самый лучший сайт подготовки к егэ))...а кто может подсказать какой средний бал нужно набрать для поступления на бюджет?

520. Татьяна, 16 февраля 2010, 22:00:15
№5641 Велосипедист выехал с постоянной скоростью из города А в город В, расстояние между которыми равно 154 км. На следующий день он отправился обратно в А со скоростью на 3 км/ч больше прежней. По дороге он сделал остановку на 3 ч. В результате велосипедист затратил на обратный путь столько же времени, сколько на путь из А в В. Найдите скорость велосипедиста на пути из В в А. Ответ дайте в км/ч.

Ответ: 14 - даётся на сайте

Хоть убейте, правильный ответ 11!!!
уверена на 100%
проверьте пожалуйста!
154:х=154:(х+3)+3
положительный корень получаем 11!!!

521. Леонид , 16 февраля 2010, 23:30:59
Может быть и правильно 11, но... Это (по твоему уравнению скорость из А в В), а в ответе просят из В в А- здесь кажнтся загвоздка. Таня, обозначай чаще всего за х то, что надо найти, тогда не влетишь с ответом. Или будь преельно внимательна! Ага? Присмотрись, я не прорешивал. Ответ м.б. 14?
Это приводит к тому,умница,но пролёт непоправимый!

522. Elli, 17 февраля 2010, 00:51:36
Леонид! Задача и правда С2. Попалась мне на пробнике(платном). Условие точное,как в листе(нам их отдали домой). "В прямом круговом цилиндре диаметр нижнего основанияАВ равен 6, точка С- середина дуги АВ, высота цилиндра АД равна 6. Найти угол между прямой АД и плоскостью ДВС."
А вот ещё С4, если интересно. Втреугольнике АВС на стороне АС отмечена точка К так,что АК/КС=2/3, на стороне ВС точка L так,что BL/LC=3/1. оТРЕЗКИ AL и BK пересекаются в точке М. Найти площадь треугольника АВС, если площадь треугольника АВМ=3

523. Леонид , 17 февраля 2010, 06:18:42
Большое спасибо, Элли. Для тренировки задачи эти нужны.

524. Леонид , 17 февраля 2010, 06:26:23
Элли, а С 1 и С 2 , не получится напечатать? Попробуй.

525. Леонид , 17 февраля 2010, 12:41:01
С1 и С3

526. АйГиз, 17 февраля 2010, 16:11:18
nothing

527. Lera, 17 февраля 2010, 17:06:12
Нерод,помогите если можете!(мне завтра сдать нужно три задачи а я считать тока до 6 умею(
по-любому не успею подготовиться(от этого зависит оставят меня на второй год или нет(
Хелп!((((((((((((

528. RuS92 , 17 февраля 2010, 18:35:02
Помогите решить эту задачу http://live.mephist.ru/show/mathege2010/view/id/26999/
В7 26999

529. Лёша, 17 февраля 2010, 18:57:56
Что там, долго лазать, ждать.

530. YYYrra, 17 февраля 2010, 20:51:09
длина морской волеы равна 2 м.Какое количество колебаний за 10 с совершит на ней поплавок,если скорость распространения волны равна 6м/с?

531. Леонид , 17 февраля 2010, 21:05:18
Передовику. р(х) = х-10 по условию. Находим р(2х) = 2х-10. Находим р(х+5)=х+5-10=х-5. Да? Подставляем 5((2х-10) -2(х-5))= 5(2х-10-2х+10)=5*0=0. Посмотри внимательно, вдруг поторопился! Идея: функция р(х), находи её значения, подставляя вместо х 2х и х+5. И все дела.


УУУ, что такое волеы?

532. YYYrra, 17 февраля 2010, 21:22:59
волны*
опечатка...
спасибо)))

533. 390840(Slam), 17 февраля 2010, 21:43:46
Леонид , имелось ввиду "ВОЛНЫ"

534. Леонид , 18 февраля 2010, 07:36:31
Слам, УУУрра. Я так рассуждал: длина волны (лямда -л, скорость - В, частота - НЮ - н, чтобы не прыгать на латиницу). л=вТ (Т-период) или л = В/н, то н=В/л=6/2=3 - частота колебаний в 1 сек. То за 10 сек 30 колебаний.
Разберётесь?
Что-то "ЛЕКСУС" молчит? Может быть ответит, поправит.
Математически. За 10 секунд волна распространится на 60 метров. Коль её длина 2м, то 60:2 = 30 столько раз пройдёт волна, значит и столько раз совершит колебаний поплавок. 30. Я думаю так.

535. Lexxus, 18 февраля 2010, 12:47:29
Что-то "ЛЕКСУС" молчит? Может быть ответит, поправит.

Был шибко занят.

За 10 секунд волна распространится на 60 метров. Коль её длина 2м, то 60:2 = 30 столько раз пройдёт волна, значит и столько раз совершит колебаний поплавок. 30. Я думаю так.

Прав.

Lexxus! Неужели, ничего нельзя сделать с "Каким будет ЕГЭ..."? Это же ужас.

Я пытаюсь чистить. Но в дни особого наплыва хронических дебилов не всегда успеваю. Их много, а я один.

536. Леонид , 18 февраля 2010, 12:58:01
Спасибо. МОЛОДЦОМ! Там они уже заикнулись, что это мешает их "тёпло - интересному" общению.

537. Татьяна, 18 февраля 2010, 14:43:25
Спасибо, Леонид! Надо же быть такой невнимательной! Действительно "найдите скорость велосипедиста на пути из В в А" ! Последую твоему совету, чтобы "не влететь с ответом впредь"!
Сайт очень полезный, советую его посетить всем знакомым, спасибо!

538. Алёна, 18 февраля 2010, 20:01:25
Привет всем, не могли бы мне помочь решить одну задачу. А именно, во сколько раз изменится площадь поверхности пирамиды, если её рёбра увеличить в 2 раза???)))

539. Леонид , 19 февраля 2010, 03:16:49
Высота основания и сторона основания по в 2 раза - площаь в 4 раз. Боковое ребро, а значит и аповема в по 2 раза - площаь в 4 раза. Сумма площаей боковой и основания - в целом в 4 раза. Площади подобных фигур относятся, как квадраты сходственных сторон. Я думаю, в 4 раза. А объём в 8 раз.

540. Elli, 19 февраля 2010, 11:41:15
Леонид! Решил ли ты мою задачку? Втреугольнике АВС на стороне АС отмечена точка К так,что АК/КС=2/3, на стороне ВС точка L так,что BL/LC=3/1. оТРЕЗКИ AL и BK пересекаются в точке М. Найти площадь треугольника АВС, если площадь треугольника АВМ=3 .оч. хочется узнать решение.

541. Elli, 19 февраля 2010, 11:48:28
задание с1 было ерундовое, так же как и с3.Попробую написать, но не знаю, как это будет выглядеть на компе. С1. решить систему уравнений. 2sinx*кв.корень из2+у=0, укв+3у+2кв корня из(укв+3у-6)=14

542. Леонид , 19 февраля 2010, 13:48:59
Элли, спасибо. Задачу о площадях подобную решал, встречалась, там манипуляции с прощадями треугольников. Они выражаются друг через друга. Например: площадь треугольника АВК = 2/5 площади АВС, КВС = 3/5 АВС, АВЛ = 3/4 площади АВС, АЛС = 1/4 АВС. Крутить, выражая через данное АВМ = 3 и получится. Подготовлю р/п попробую напечатать, если есть интерес. Здесь сложно печать.
С1 понял ( в первом ур. 2+у под одни корнем кв.?). А - С4?.

543. Elli, 19 февраля 2010, 14:58:04
Леонид! нет в первом уравнении (2 sinx*корень из 2)+у=0
с4 решить неравенство дробь в числителе(кв корень из 64(хкуб)-1) -1,знаменатель 4х+1. вот эта дробь больше или равно 4х. не знаю, как ещё написать. вот так получилось.
"Подготовлю р/п попробую напечатать, если есть интерес." интерес есть,а что такое р/п? вообще, можно здесь как-то помещать отсканированные решения?

544. Elli, 19 февраля 2010, 15:02:49
Леонид!почему не просишь с5 и с6? с6 тоже не сложная оказалась. решить в целых числах уравнение кв корень из (2х+1)+кв.корень из (2у+1)=4

545. Леонид , 19 февраля 2010, 16:09:55
Да, несложно. Спасибо Элли. Печатать же трудно. Если не трудно, буду рад.

546. Леонид , 19 февраля 2010, 21:18:41
Элли, р/п - рукописный, чтоб соображать как здесь печатать. И опять к 1-му ур. системы. Коэффициент 2 перед синусом Х, и ещё множитель для них же чтоли корень кв. из 2. Если я правильно понял. Тогда смысл такой записи? Это 2 кв. корень из 2 умн. на син.Х и ко всему + у, да?
С4 д.б. планиметрическая задача, а это С3- про неравенство.

547. Леонид , 19 февраля 2010, 23:46:41
Элли! Пытаюсь. Чтобы не прыгать в латиницу, Т пл. АВС. АМК=2/5Т-3, ВМЛ = 3/4Т-3. КМС=1,5АМК=1,5(2/5Т-3)=3/5Т-4,5. МЛС =АЛС-АМК-КМС=3/5Т-(3/5Т-4,5)-(3/4Т-3)=3/5Т-3/5Т+4,5-3/4Т+3=7,5-3/4Т. МЛС=7,5-3/4Т
С др. стороны МЛС= 1/3МВЛ=1/3(3/4Т-3)=1/4Т-1. ТО 7,5-3/4Т=1/4Т-1 и Т=8,5.
Проверь, может быть,где-то соврал или опечатался. М.б решение есть короче. Поймёшь: КМС, например-это имеется в виду площадь КМС. Идея - одно и то же выразить с двух сторон, приравнять, введя "вспомогательное" неизвестное. У меня Т. Л- наша вместо латинской созвучной.

548. 19931215, 20 февраля 2010, 15:34:01
а по биологии тестов не будет?

549. Леонид , 20 февраля 2010, 15:41:41
Дорогой, здесь только математика и немного физики.

550. RuS92, 20 февраля 2010, 17:23:20
Какие сайты егэ существуют?напишите пожалуйста все!!!!!!!!!!!!!!!!!

551. Elli, 20 февраля 2010, 17:25:28
Леонид!ты правильно понял: Это 2 кв. корень из 2 умн. на син.Х и ко всему + у=0. А неравенство-это точно с3, а с4 - это как раз про площадь. У меня тоже получилось 8.5, но решение длиной в 2 страницы. Сейчас перепишу твоё и буду разбираться.Пасибки!

552. Elli, 20 февраля 2010, 19:50:38
Леонид! Разобралась с решением. Оказалось всё гораздо проще. я соединяла точкиК и Л,а надо былоМ и С. только не понятно, почему МЛС =АЛС-АМК-КМС=3/5Т-(3/5Т-4,5)-(3/4Т-3. у меня МЛС =АЛС-АМК-КМС=1/4Т-2/5Т+3-3/5Т+9/2.Хотя результат тот же. Вот ещё из пробника задание С4.
Из вершин треугольника АВС проведены высоты к противоположным сторонам ВК, СЕ, АМ. Найти все углы треугольника АСВ, если в треугольнике КЕМ углы равны 30, 60 и 90 градусов. жду решения. я не смогла.

553. Леонид , 20 февраля 2010, 20:54:37
Элли, всё нормально, я же писал, что задача на "манипуляции" с треугольниками и их площадями - надо смотреть как я шёл МЛС = АЛС - (АМК+КМС) - писал сразу без скобок и что ранее. Ты просто с другой стороны шла. По чертежу проще было бы объяснить. . Рад, что разобралась и ответ тот же. Писал с чертежа по условию, но нашими буквами, мог напороть - предупреждал о внимательности.
С4 интересная, спасибо, поморокую, напишу. Пока идея, не знаю полезность, испробуй - точки Е и М лежат на окружности диаметра АС (все треугольники вписанные в окружность и опирающиеся на диамет, обязательно прямоугольные, а там прямые углы - высоты). Пробуй. И я смотрю. Пока.


554. клопик, 21 февраля 2010, 08:44:24
Помогите решить задачку "Брюки дороже рубашки на 20 процентов и дешевле пиджака на 46 процентов. На сколько процентов рубашка дешевле пиджака?"

555. Slam, 21 февраля 2010, 13:59:45
Первая труба пропускает на 1 литр воды в минуту меньше, чем вторая. Сколько литров воды в минуту пропускает вторая труба, если резервуар объемом 783 литра она заполняет на 2 минуты быстрее, чем первая труба заполняет резервуар объемом 812 литров?
ответ 28 или 29?

556. Lexxus, 21 февраля 2010, 14:26:28
Первая труба пропускает на 1 литр воды в минуту меньше, чем вторая. Сколько литров воды в минуту пропускает вторая труба, если резервуар объемом 783 литра она заполняет на 2 минуты быстрее, чем первая труба заполняет резервуар объемом 812 литров?
ответ 28 или 29?

За x надо принимать то, что тебе нужно найти. А именно - пропускную способность второй трубы.
Тогда уравнение будет выглядеть так:
783/x + 2 = 812/(x-1), и x будет равен 29. Это и есть правильный ответ.

557. Slam, 21 февраля 2010, 21:12:05
Lexxus, спасибо!
Объясните плиз как №6045 делать!

558. Леонид , 21 февраля 2010, 22:15:00
Слам. 1) Находи производную функции с Х в кубе и приравнивай к угловому коэффициету (у=кх + в, к) прямой. Решив кв. уравнения, получишь 2 корня. Выбрать в ответ тот, который удовлетворяет уравнению, приравняв правые части прямой и функции с Х в кубе ("хороший" - что в клеточки бланка впишется). Почему? Коль прямая является касательной, то она имеет общую точку с гр. функции.

559. Оля, 22 февраля 2010, 00:10:55
КЛОПИК. в твоей задаче нет ни каких требований к ответу - об округлении?

560. Slam, 22 февраля 2010, 10:02:20
Леонид , 3x^2+2x+8=8
3x^2+2x=
x(3x+2)=0
x=0 или x=2/3
Хороший ответ: 0, но вот если оба корня будут хорошими, как мне правильный выбрать? " Выбрать в ответ тот, который удовлетворяет уравнению, приравняв правые части прямой и функции с Х в кубе ("хороший" - что в клеточки бланка впишется). Почему? Коль прямая является касательной, то она имеет общую точку с гр. функции." - вот эти строчки ,я вообще не понял=(((((

561. Леонид , 22 февраля 2010, 12:38:07
МОЛОДЕЦ! Всё верно, подставляй в уравнение оба корня - подойдёт только один (точка касания она). Например (не из этого заания, из головы) :
у =2х+6 - прямая, у = х в кубе + 3 х в кв. - 6х + 4. Приравниваем:
2х+6=х в кубе + 3х в кв. - 6х +4, всё в одну часть, подобные, получим кубическое уравнение. В него и подставляй найденные корни, бери тот, при котором равенство верное.
"Коль прямая является касательной, то она имеет общую точку с гр. функции." - поэтому и приравниваем.

Если не понял, пиши задание. Их в В8 2 вида: когда является касательной и кога параллельна касательной.

562. клопик, 22 февраля 2010, 16:17:02
ОЛЯ, в задаче должен быть "круглый " ответ. Об округлении нет речи.

563. Slam, 22 февраля 2010, 19:06:58
Леонид , понял, спасибо!

564. Алинка, 22 февраля 2010, 21:42:59
задачи- моя вечная проблема. подскажите как решать задание В12 тест №134190

565. Леонид , 22 февраля 2010, 22:10:55
Ты что пишешь, Алинка, такого номера нет. Чтобы не прыгать по разделам, пиши задачу.

566. Алинка, 23 февраля 2010, 11:04:44
блин, значит я что то перепутала.вот задача:
Из А в В одновременно выехали два автомобилиста. Первый проехал с постоянной скоростью весь путь. Второй проехал первую половину пути со скоростью 33 км/ч, а вторую половину пути — со скоростью, на 22 км/ч большей скорости первого, в результате чего прибыл в В одновременно с первым автомобилистом. Найдите скорость первого автомобилиста. Ответ дайте в км/ч

наверное она очень легкая, но я в этой фигне вообще ниче не понимаю. всё под В правильно,а задача никак(((((((((((((((((((

567. Леонид , 23 февраля 2010, 12:08:51
Мужчины, юноши и пацаны! С Днём Защитника Отечечтва! Благополучия! И вас женщины и девочки! Без надёжного тыла и любви, армия не может быть сильной. Здоровья! Мирного неба!

568. Леонид , 23 февраля 2010, 12:30:37
Алинка, все задачи В12 решаются по одной технологии. За Х бери то, что просят в ответ. Пусть Х - здесь скорость первого. Путь не указан, бери за 1.
Время первого 1/х. Время второго на первой половине пути 0,5/33, на второй половине пути 0,5/(х+22).
Время одинаковое, уравнение: 1/х = 0,5/33 + 0,5/(х+22).
Можно без дробей 0,5, если путь взять 2. Уравнение 2/х = 1/33 + 1/(х+22). Решай. Всё в ону часть, к обшему знаменателю и т.д.

569. клопик, 23 февраля 2010, 17:26:20
Друзья, как же быть с задачей на проценты про брюки, рубашку, пиджак? Это задача В12. ПОМОГИТЕ, пробую решить - никак. Надеюсь на Вашу помощь.

570. Леонид , 23 февраля 2010, 17:56:47
Ну, что попало позывной, какой-то "клопик". Смени. Пиджак, рубаха и штаны.
Хр. - стоимость рубахи, тога стимость брюк 1,2Хр. Стоимость пиджака 1,2Х*1,46 = 1,752Хр. В процентное соотношение перехоим:
Хр. - 100%
1,752Хр. - у%. у% = (1,752Хр.*100)/Х = 175,2% - стоимость пиджака в % по отношению к рубахе. 175,2% - 100% = 75,2%. Рубашка дешевле пиджака на 72,5%. Думаю, что так.

Пиши, какие задания были на пробнике С1 - С6.

571. Slam, 23 февраля 2010, 18:00:04
и ответ будет 44 =)

572. Slam, 23 февраля 2010, 18:08:58
Леонид! Пробник C2:
в прямом круговом цилиндре диаметр нижнего основания AB равен 8 , точка C-середина дуги AB. Найдите высоту цилиндра AD, если угол между прямой AD и плоскостью DBC равен 30 градусов

573. Slam, 23 февраля 2010, 18:10:06
Помоги пожалуйста решить!

574. Леонид , 23 февраля 2010, 18:39:23
Slam. Так как С на дуге АВ, то угол АСВ как вписанный 90 град. А ещё С -середина дуги АВ, то треугольник АСВ прямоугольный равнобедренный, где АС=СВ (центр описанной окружности около прямоугольного треуг. обязательно на середине гипотенузы). По т. ПИФ. Хкв.+Хкв.=64. (Х это АС и СВ) х=4 крня кв. из двух. Тогда ДС гипотенуза 2АС (катет против угла 30 град. в прямоуг. треуг. АДС) 8корней кв. из 2. По т. ПИФ. АД кв = ДСкв.-АСкв. ; АДкв.= 128 - 32 = 96. АД = 4 корня кв. из 6. Проверь!!!

Что за ответ 44?

575. Инна, 23 февраля 2010, 23:15:00
Помогите решить задачу: Для получения на экране увеличенного изображения лампочки в лаборатории используется собирающая линза с главным фокусным расстоянием f=45 см. Расстояние d1 от линзы до лампочки может изменяться в пределах от 50 до 70 см, а расстояние d2 от линзы до экрана - в пределах от 200 до 270 см. Изображение на экране будет четким, если выполнено соотношение 1/d1+1/d2=1/f. Укажите, на каком наименьшем расстоянии от линзы можно поместить лампочку, чтобы ее изображение на экране было четким.Ответ дайте в см.

576. Украинский Станислав, 24 февраля 2010, 14:22:10
а где тут ответы то???это тупо форум чтоль??

577. Леонид , 24 февраля 2010, 14:39:53
Кто бы для тебя их готовил, выдавал? Решай сам. "Тупо форум" - Что это?

578. Украинский Станислав, 24 февраля 2010, 14:41:57
ну да впринципе,совсем даже не тупо)

579. Украинский Станислав, 24 февраля 2010, 14:46:49
а сколько проходной балл по алгебре,ну минимальный?и знает ли кто нибуть такую тему,что в сахалине пишут быстрее чем тут, ближе к европе,и очень рано утром есть шанс найти решение некоторых вариантов)у меня в лицее так пара человек смогли сделать,кто нибудб пробовал так?и реально это вообще?

580. Леонид , 24 февраля 2010, 15:16:43
Нет. Займись делом. Ты же из лицея.

581. katya.16ru, 24 февраля 2010, 16:23:44
чем меньше расстояние от линзы до лампочки, то есть д1, тем больше будет расстояние от линзы до экрана, то есть д2. значит берем 270. ф известно. подставляем все данные в уравнение. у меня получилось 54. смотрим удовлетворяет ли условию от 50 до 70. удовлетворяет. значит ,ответ 54.

582. ADdis, 24 февраля 2010, 16:38:53
ппц...мужики...заканчиваю 11 класс...сдаю физику и математику....всё учу!!! НО не знаю почему когда гачинаю деать ЕГЭ всё забываю(((

583. katya.16ru, 24 февраля 2010, 17:00:06
почему только мужики?...

584. бабушка, 25 февраля 2010, 11:39:47
Помогите с решением задачи В12, текстовой : поезд, двигаясь с постоянной скоростью 60 км/час, проезжает мимо платформы длиной 300 м за 30 сек. Найти длину поезда в метрах. У меня получается 500 метров, но это неправильный ответ.

585. Lexxus, 25 февраля 2010, 12:21:06
поезд, двигаясь с постоянной скоростью 60 км/час, проезжает мимо платформы длиной 300 м за 30 сек. Найти длину поезда в метрах.

Поезд проехал мимо платформы за 30 секунд.
Это значит, что он поравнялся мордой (М) с началом платформы, а через 30 секунд поравнялся задом (З) с её концом.
(В - это вагоны)

ЗВВВВВВМ->
========ПЛАТФОРМА======

================ЗВВВВВВМ->
========ПЛАТФОРМА======

То есть, поезд прошел путь (300 метров + x), где x - его собственная длина, которую нам и надо найти.
Значит, надо решить уравнение:
(300 метров + x) = (60 км/час)*(30 секунд)

Переводим всё в километры и часы (300 м = 0.3 км, 30 сек = 1/2 мин = 1/120 ч):
0.3 + x = 60*(1/120)
0.3 + x = 0.5
x = 0.2

Ответ - 200 метров.

586. бабушка, 25 февраля 2010, 13:27:53
Lexxus, спасибо, поняла, где моя ошибка - не отняла 300 метров, длину платформы.

587. Леонид , 25 февраля 2010, 14:51:20
Lexxus! НАРОД! "SOS" !!! Прошу консилиума по заданию В7 № 26983 "Найти а/в, если (2а+5в)/(5в+2а)=7. Решается легко, что я и сделал, не обратив внимания - невнимание подводит, по правилу пропорции 2а+5в=35в+14а из условия. -12а= 30в, а/в = -2,5. ТАК? Решил для интереса проверить:
а=-2,5в , подставив в знаменатель (5в-5в=0). На нуль делить нельзя!!!
Потом увиел - в условии числитель и знаменатель - то равные. То есть, сократив, имеем 1 = 7. Вобщем, какой-то СОФИЗМ у меня получется. И ответ есть и писать его нельзя. Или я чего-то не то?

588. Мария, 25 февраля 2010, 19:17:56
Помогите пожалуйста решить задание В11!
Найдите наименьшее значение функции у=8tgX - 8X - 2П(пи) + 5 на отрезке
-П/4; П/4

589. Леонид , 25 февраля 2010, 21:46:18
Мария, таких упражнений ранее здесь много показывалось. Бери производную у штрих = 8/кос. кв. Х - 8 = 8(1-кос.кв.Х)/кос.кв.Х - неотрицательная. Функция возрастающая. Следовательно наименьшее значение принимает при х=-пи/4-подставляй в данную функцию
у=-8 +2пи-2пи+5 = -3.



НАРОД! Что с моим "СОФИЗМОМ" ? выше на 1 шаг

590. katya.16ru, 25 февраля 2010, 21:59:00
у меня тоже также получилось. так получается что решений нет

591. Леонид , 25 февраля 2010, 22:07:44
Lеxxus. Красивая модель-рисунок к задаче для бабушки. Твоё мнение на мой "СОФИЗМ"?

592. Lexxus, 25 февраля 2010, 22:18:12
НАРОД! Что с моим "СОФИЗМОМ" ? выше на 1 шаг

Всё правильно, эта задача не имеет решения.

593. ангелинка, 26 февраля 2010, 16:27:42
люди помогите, в части "с" вторая задача. формулировка второй задачи такова: "к диогонали куба провели перпендикуляры из остальных вершин куба. на сколько частей и в каком отношении основания этих перпендикуляров разделили диогональ. почему в школе не дают таких заданий решать? дадут 20 аданий одинаковых, только с разными цыфрами!!!!!!!!!!!!!!!!!!!!!!!!!!!!!!!

594. Gunka, 26 февраля 2010, 16:42:32
Народ, я думаю всем итак ясно, что настоящий ЕГЭ по математике полный провал.Я написала две диагностические работы, результат разный.Каждый раз новые задания придумывают...По мне лучше бы оставили прошлогодний вариант,но наши министры ничего не понимают,вот посадить бы их за варианты егэ,поймут какую чушь они ввели.И на истории творится то же самое.Говорят через пару годков должный убрать эту систему,но я очень сильно сомневаюсь,поэтому по 6 часов в неделю убиваю время на занятия по математике,сами учителя порой не понимают смысла задач.Поэтому не раскисайте))))Просто так это безобразие остаться не может))

595. Леонид , 26 февраля 2010, 17:30:48
Ох, и неправду,Зинка,гутаришь, что учителя сами не понимают. Там ведь нечего понимать. Результаты разные - нет, дева, натренированности, системы знаний. Новые задания каждый раз - мир задач настоль велик, что двух одинаковых в природе не встречаются. Учиться надо- вишь, даже 6-часовое репититорство не помогает. Запущенность аховая. Но не раскисай, трудись. Математика поддаётся "стальному заду". Понимаешь? .

596. Леонид , 26 февраля 2010, 17:47:44
Вообще-то, Зинка, здесь об этом не пишут. Переходи на другие разделы.
Алинка, кажется, твоя задача из демоверсии, скопируй, там есть её решение.

597. Lexxus, 26 февраля 2010, 18:25:54
к диогонали куба провели перпендикуляры из остальных вершин куба. на сколько частей и в каком отношении основания этих перпендикуляров разделили диогональ

На самом деле, тут всё не так уж и страшно.
Если у нас куб ABCDA1B1C1D1, а его диагональ, скажем, A1C, то рассмотрим прямоугольник A1B1CD.
Его стороны равны 1 (ребро куба) и (корень из 2) (диагональ грани куба).
Диагональ этого прямоугольника (которая заодно и диагональ нашего куба) равна (корень из 3) (Пифагор).


Диагональ куба

Проведем из вершин B1 и D перпендикуляры к A1C.

Для треугольников A1H2B1 и CH2B1 пишем теорему всё того же Пифагора.
Получаем систему из двух уравнений с двумя неизвестными - A1H2 и B1H2.
Решаем её и получаем, что A1H2 = (корень из 3)/3, то есть ровно треть диагонали.
Там всё симметрично, так что СH1 = A1H2 = (корень из 3)/3.

То есть перпендикуляры B1H2 и DH1 делят диагональ куба на три равные части.

А это значит, что и перпендикуляры из всех оставшихся вершин куба разделят диагональ на те же самые три равные части.
Ответ - 1:1:1

598. Slam, 26 февраля 2010, 19:23:57
katya.16ru, ты из Набережных Челнов?
Gunka, я вот например из банка заданий очень много задач решил, и на пробном ЕГЭ 17 февраля из B части набрал 12 баллов , из C 2 балла , в сумме 14 баллов. А Ведь я в ноябре, когда первый раз демонстрационный решил , оттуда смог решить только 6 "Бешных" задач -правильно. Леонид верно говорит, что математика поддается "стальному заду". Так что трудись, до егэ конечно 3 месяца всего лишь осталось , но все же, натренироваться еще можно=)

599. ангелинка, 26 февраля 2010, 20:16:30
леонид я не алинка,а ангелинка, научитесь читать ок? ненавижу когда другими именами кличат((((((((((((((

600. ангелинка, 26 февраля 2010, 20:23:36
Slam безусловно натренероваться можно, но дело в том, что и в школе, и у репита тя тренеруют по одним и тем же заданиям, а вдруг нашему министру образования Фурсенко еще какая-нибудь ерунда в голову придет и че тогда...........

601. ангелинка, 26 февраля 2010, 20:26:05
спс Lexxus. я эту задачу решила сама, ток по другому чуть

602. Леонид , 26 февраля 2010, 22:24:22
Ангелинка. "навидишь", аль "ненавидишь" - это никакой роли не играет. Но читать учиться буду.
Слам - тебя с успехом! Это уже хорошая "4". Как с задачей, что просил? Постарайся напечатать свои С1,С3,С4. Да, Слам, ты из какой территории - области, края?

603. elli_93, 27 февраля 2010, 14:10:26
Lexxus! Леонид! Люди!!!!!!!! Ну помогите же мне решить задачу c4! Бьюсь над ней уже неделю, ничего не получается! И бросить не могу, чем-то она меня зацепила. Киньте хоть идею.
Из вершин треугольника АВС проведены высоты к противоположным сторонам ВК, СЕ, АМ. Найти все углы треугольника АСВ, если в треугольнике КЕМ углы равны 30, 60 и 90 градусов. жду решения.

604. Lexxus, 27 февраля 2010, 15:17:19
Из вершин треугольника АВС проведены высоты к противоположным сторонам ВК, СЕ, АМ. Найти все углы треугольника АСВ, если в треугольнике КЕМ углы равны 30, 60 и 90 градусов. жду решения.

Треугольник KEM называется ортоцентрическим треугольником, или просто ортотреугольником. Высоты треугольника ABC являются его биссектрисами.
Это свойство ортотреугольника. Я, правда, не знаю, надо ли это доказывать.

А дальше всё просто - раз CE - биссектриса угла KEM, то угол AEK равен углу BEM, и оба они равны (180 градусов - угол KEM)/2. Точно также находим углы BME, CMK, CKM и AKE.
А дальше находим углы A, B и C из треугольников AEK, BME и CMK соответственно.

605. вичи, 27 февраля 2010, 16:22:05
На диаграмме показана среднемесячная температура воздуха в Екатеринбурге (Свердловске) за каждый месяц 1973 года. По горизонтали указываются месяцы, по вертикали - температура в градусах Цельсия. Определите по диаграмме наименьшую среднемесячную температуру в период с мая по декабрь 1973 года включительно.

606. Алинка, 27 февраля 2010, 17:34:55
Ааааааааааааа,спасибо большое Леонид. теперь главное, чтобы на ЕГЭ такой тип попался))))))))))))))))))))конечно это мало вероятно((((((( ну что ж будем пробовать другие!

607. Маримелька, 27 февраля 2010, 18:38:29
у меня мурашки по коже от этого ЕГЭ=(

608. Маримелька, 27 февраля 2010, 18:41:06
556608556 моя Ася добавляйтесь=)

609. Леонид , 27 февраля 2010, 18:51:39
МАримелька перейди в лругие раздеы "какими будут тесты..." и др., тпм любят ныть, плакать в тряпочку... А здесь ребята учатся решать, советуются

610. katya.16ru, 27 февраля 2010, 19:25:56
slam, я не из набережных челнов,но тоже из татарстана

611. Slam, 27 февраля 2010, 20:13:07
Леонид , я не смог сфотографировать КИМ, потому что были очень! строгие 2 тетеньки=), которые сидели у нас в аудитории. С2 я тебе уже написал, просто я эту задачу на руку себе переписал. И еще один момент, в B4, B10,B11,B12, задачи были не из открытого банка заданий! Это ппц просто подстава, и я не смог решить b4((((((.
P.S. Мне в C1 попался очень легкий пример.
Я из республики Татарстан, город Набережные Челны-у нас делают КамАЗы=)))))

612. Леонид , 27 февраля 2010, 20:41:14
Lexxus, А нельзя зесть обойти понятие ортотреугольника? Просто доказать, что высоты треуг. АВС - это биссектрисы треуг. МЕК. Только как попроще? И ещё, треугольник АВС м.б. тупоугольным (???), остроугольным (45, 60,75), прямоугольным (отрезок высота из прямого угла, вместо треуг. ЕКМ, что говорить зесь?) - С4 большинство предполагает рассмотрение случаев.

613. Леонид , 27 февраля 2010, 20:54:31
Slam! Понятно. Я из Кузбасса (Кемеровской обл.). Спросил вот к чему: разные (наши) временные пояса, предугадать сложно, но обмен заданиями С был бы полезен (были пробы 19 ноября, 8 декабря и вот сейчас 17-го, в марте пробный). Не исключено, предлагаемые "С" у вас могут попасть к нам и наоборот. Понял? - на сам егэ. А тексты вам должны были отдать, или можно их заполучить, они в вашей школе есть точно, обращайтесь.

614. Lexxus, 27 февраля 2010, 21:04:07
Просто доказать, что высоты треуг. АВС - это биссектрисы треуг. МЕК. Только как попроще?

А вот не знаю, как попроще. Там не то чтобы сложное, но с подвыподвертом доказательство :(

И ещё, треугольник АВС м.б. тупоугольным (???), остроугольным (45, 60,75), прямоугольным (отрезок высота из прямого угла, вместо треуг. ЕКМ, что говорить зесь?) - С4 большинство предполагает рассмотрение случаев.

Может быть и остроугольным, и тупоугольным. Всё равно высоты ABC являются биссектрисами KEM.
А вот прямоугольным он быть никак не может, поскольку там две высоты в одну точку сходятся. Кстати, у задачи, насколько я понимаю, сразу четыре разных решения - для остроугольного треугольника и аж для трех разных тупоугольных.

615. Леонид , 27 февраля 2010, 21:31:10
А если построить рассуждения так. Н - точка пересечения высот треугольника АВС. Она же точка пересечения либо высот, либо медиан, либо биссектрис треугольника КЕМ. Пусть в точке Е угол прямой (тр. КЕМ), опустив перпендикуляры из точки Н на ЕК и ЕМ по равентству прямоугольных треугольничков она (Н) равноудалена от ЕМ и ЕК, т.е. лежит на биссектрисе, ЕС - биссектриса. Она же равноудалена от ЕК и ВК, от МК и ЕМ (пары равных прямоугольных треугольников) ---- Н - пересечение биссектрис и далее Ваше (Lеxxus) решение. Не накрутил я ? .

616. katya.16ru, 27 февраля 2010, 21:52:39
Катер в 10:00 вышел из пункта А в пункт В, расположенный в 30 км от А. Пробыв в пункте В 2 часа 30 минут, катер отправился назад и вернулся в пункт А в 18:00. Определите (в км/час) скорость течения реки, если известно, что собственная скорость катера равна 11 км/

помогите,что-то не получается.

617. Lexxus, 27 февраля 2010, 22:00:01
Она же точка пересечения либо высот, либо медиан, либо биссектрис треугольника КЕМ.

А с чего это вдруг? Нет, это неправильно. Вернее, так рассуждать неправильно.

Короче, я придумал нормальное решение. Напишу покрасивше и выложу.

618. Леонид , 27 февраля 2010, 22:44:04
Ну да, верно, нет исходного для моего предположения. Жду вашего решения. Спасибо.

619. Леонид , 27 февраля 2010, 22:55:40
Задача о катере. Всего в поездке 18:00 - 10:00 = 8ч. В движении (без времени стоянки) 8 - 2,5= 5,5. Х скорость течения, (11-х) скорость катера против течения, (11+х) - по течению.
Уравнение: плыл 5,5 = 30/(11-х) + 30/(11+х)- всего времени по течению и против течения.

620. katya.16ru, 27 февраля 2010, 23:15:18
у меня тоже такое уравнение, но из дискриминанта число не выносится(

621. Lexxus, 27 февраля 2010, 23:19:44
Итак,
Из вершин треугольника АВС проведены высоты к противоположным сторонам ВК, СЕ, АМ. Найти все углы треугольника АСВ, если в треугольнике КЕМ углы равны 30, 60 и 90 градусов.


1. Треугольник АВС - остроугольный:

Решение С4 про ортотреугольник. Случай 1

Кстати, тут мы между делом и доказываем, что высоты треугольника являются биссектрисами ортотреугольника.

2. Треугольник АВС - тупоугольный:

Решение С4 про ортотреугольник. Случай 2

622. Slam, 27 февраля 2010, 23:21:30
Леонид , у нас был платный пробный от ФГУ (Федеральный центр тестирования). Задания они непременно отдадут, но только еще результаты не пришли, результаты должны сказать , а также отдать нам наши задания примерно 4-6 марта.Напишу все задания , как ответы раздадут.
P.S Вспомнил C4: Даны 2 окружности, первая окружность имеет радиус 25, вторая 16, через эти две окружности проведена касательная. Найдите радиус третьей окружности которая касается касательной и обеих окружностей.
Записал условие задачи только по памяти , вроде бы ничего упомянуть не забыл

623. katya.16ru, 27 февраля 2010, 23:23:09
все получилось)))

624. Slam, 27 февраля 2010, 23:32:17
katya.16ru,
Там получается неполное квадратное уравнение, и никакой дискриминант не нужен ответ: 1 км/ч

625. Slam, 27 февраля 2010, 23:49:44
Леонид , решение мне не надо? т.к. я нацелен только на всю часть B, C1 и C2

626. Lexxus, 27 февраля 2010, 23:53:58
Даны 2 окружности, первая окружность имеет радиус 25, вторая 16, через эти две окружности проведена касательная. Найдите радиус третьей окружности которая касается касательной и обеих окружностей.
Записал условие задачи только по памяти , вроде бы ничего упомянуть не забыл

Забыл упомянуть, что две первые окружности касаются друг друга, причем снаружи, а касательная проведена не через точку касания этих окружностей.
Иначе ответ любой подойдет.

Позже напишу решение.

627. Slam, 28 февраля 2010, 00:34:22
"Забыл упомянуть, что две первые окружности касаются друг друга, причем снаружи"-да, точно это тоже в условии было!
"а касательная проведена не через точку касания этих окружностей."-а вот насчет этого условия не помню, может быть и такое было в условии=)

628. Леонид , 28 февраля 2010, 07:02:08
Спасибо Lexxus за задачу о треугольниках. ХОРОШО!


Slam, а почему такая цель? Больше решить, не помешает!!! У тебя же хорошее продвижение. И время ещё есть.

629. бабушка, 28 февраля 2010, 08:16:42
Это опять я, любительница текстовых задач. Вот умом понимаю задачу, а записать математически не могу. Помогите с решением.

Между стартом и финишем горнолыжного спуска круглосуточно действует подвесная канатная дорога. кабинки сверху и снизу отправлятся одновременно каждые 3 минуты. время движения в одну сторону составляет 14 минут. на старте и финише какбинка стоит 1 минуту, вклчая режим проскальзывания каната. определить количество кабинок, двигащихся вниз, которые встречатся горнолыжнику при подъеме

630. Slam, 28 февраля 2010, 09:59:02
Леонид , 17 февраля в B части я не допустил ни одной ошибки только благодаря натаскиванию себя примерами. В этом только сайте я решил 925 примеров, а на самом деле наверное 1.5 тысячи. На пробном, который организован ФГУ где-то 3-4 задания были не из открытого банка, и я,по моим расчетам, допустил в B 2 ошибки. так что, мне лучше отработать некоторые из B и C1 С2. Еще у меня есть физика и русский язык=)
P.S. В B-11 допустил ошибку из-за своей невнимательности(((((((((

631. Мила, 28 февраля 2010, 11:36:04
Народ напишите пожалуйста полное решение какого нибудь примера С1.

632. elli_93, 28 февраля 2010, 12:32:09
Lexxus,СПАСИБО!!!!!!!!!!!!!!! избавил от мучений!

633. elli_93, 28 февраля 2010, 18:24:01
Оказывается, очень легко решается задача о треугольниках. Если бы знать заранее про ортотреугольник! ни в одном моём справочнике ничего похожего нет. Подозревала, конечно, что надо искать подобные, но эта моя самая нелюбимая тема! Спасибо за решение!!!!!!!!

634. Анастасия Юрьевна, 28 февраля 2010, 22:29:29
Хочу сдать ЕГе, но очень боюсь Этого, средне специальное образование есть. Хочу высшее, но другую специальность ((. У нас в Дагестане как вы сами знаете с обучением не так строго, как в других реегионах, необходимо либо немалое колличество денег, либо отличные знания, которые нам не даны полностью((((((.

635. дюймовочка, 1 марта 2010, 16:53:13
В понедельник одна акция строительной компании «N» стоила 60 руб. В среду стоимость акций компании упала на 20%. Какое наибольшее количество акций компании можно купить в среду на 3000 руб.?

636. дюймовочка, 1 марта 2010, 16:55:00
помогите пожалуйста решить задачу.В стеклянный сосуд, имеющий форму прямоугольного параллелепипеда, с основанием 20см х 20 см и высотой 30 см налита вода до высоты 15 см. В сосуд бросили 5 металлических кубиков с ребром 4 см. На сколько повысился уровень воды. Ответ выразите в см.

637. Леонид , 1 марта 2010, 18:14:03
1). Стоимость акций упала или акции? Если одной акции и стоимость упала на 60%, то стала стоить 60р. * 0.8 = 48р. Купить можно 3000р.: 48р. =62,5 шт. Купить наибольшее число акций можно 62.

2) Объём воды первоначального уровня 20*20*15 и + 5*(4*4*4) объём 5 кубиков, т.е. 20*20*15 + 5*4*4*4 = 20*20* Н -стал объём (Н-новый уровень). Считаем: (не перемножал специально), Делим на 20 и 4, имеем
5*15+4=5Н, Н= 15,8см. (здессь работает формула Vпараллелеп.=а*в*с - прямоугольного, размеры основания не меняются, меняется высота при увеличении первоначальног объёма на 5 кубиков 6000+320 = 400Н) - как вам удобнее. Проверьте вычисления.

638. katya.16ru, 1 марта 2010, 18:39:54
в цилиндр вписана призма , основанием которой служит прямоугольный треугольник, катет которого равен 2а, а прилежащий угол равен 60 градусов. диагональ большей стороны призмы составляет с плоскостью ее основания угол 45 градусов. найти объем цилиндра.
ПОМОГИТЕ, ПОЖАЛУЙСТА!!!

639. katya.16ru, 1 марта 2010, 18:49:37
дюймовочка,сначала надо найти объем воды налитой в сосуд. 20*20*15= 6000. находим объем одного кубика 4*4*4=64, их пять ,значит 64*5=320. кубики опустили в воду,значит 6000+320=6320.
V=S*h, 6320=20*20*h, отсюда h=15.8
проверь ,может где то ошибка в вычислении,но смысл решения такой.

640. Леонид , 1 марта 2010, 19:27:41
Основание призмы- это вписанный прямоугольный треугольник в основание цилиндра - в окружность, значит центр окружности в середине гипотенузы.
Угол, противолежащий угол катету 2а - есть 30 градусов, то гипотенуза 4а - это и диаметр, тогда радиус 2а.
Наверное диагональ большей грани, а не стороны призмы? Коль с плоскостью основания 45 градусов, то высота цилиндра и диаметр основания по 4а. Объём пи*Rкв.*Н = пи (2а)кв.* 4а = 16пи*а куб.
Это С2, в какой территории? Когда проводили? Что было С1,С3,С4 -попрбуй напечатать.

641. katya.16ru, 1 марта 2010, 19:36:02
я из татарстана . но этого не было на пробном егэ. там геометрические задачи легкие были, и их уже разобрали здесь,кажется. а эту задачу нам сказали дома решить, тока у меня ниче не получалось . за решение спасибо)))))))

642. Slam, 1 марта 2010, 21:40:08
Lexxus, почему Вы не ставите на "Решенные" все 52%, на которые даны более 5 ответов, ведь В10 и B11 например есть довольно сложные задачи, а у нас в "решеных" их всего лишь около 20!

643. Slam, 1 марта 2010, 22:29:12
и еще в mathege.ru задания B10 348, а тут 487!

644. Lexxus, 1 марта 2010, 23:35:09
Lexxus, почему Вы не ставите на "Решенные" все 52%, на которые даны более 5 ответов, ведь В10 и B11 например есть довольно сложные задачи, а у нас в "решеных" их всего лишь около 20!

К сожалению, автоматическая утверждалка верных ответов всё ещё не готова к самостоятельному плаванию. Пока это делается вручную, а значит, не слишком шустро. Но я работаю над этим.

и еще в mathege.ru задания B10 348, а тут 487!

Из банка убрано много заданий из тех, что были там раньше. Лично я считаю, что чем больше заданий будет, тем лучше.

645. Slam, 2 марта 2010, 14:54:35
Lexxus, сделайте пожалуйста это , т.к. до ЕГЭ меньше трех месяцев осталось!
И еще, я думаю, что если будет хотя бы 50% в решеных, то посещаемость сайта увеличится еще больше, а это наверное вам+. Ну это так, к слову=)

646. Natalie, 2 марта 2010, 17:15:12
Здравствуйте!
Помогите решить задачу: дан равнобедренный треугольник АВС, АС - основание. Высота ВН-16 см, косинус угла А равен 3/5. Нужно найти АС. А насчет русского, то действительно хороший сайт найти трудно. Vladic2, вы спрашивали про тропы. Троп - художественно-стилистический прием. Видами тропов являются начиная от простых метафор, олицетворений и заканчивая синекдохой, метонимией, эвфемизмом и перифразой. Ну и еще пара-тройка названий.

647. Леонид , 2 марта 2010, 17:38:08
синус А = 4/5, то тангенс А = 4/5 : 3/5 = 4/3. 4/3 = 16/АН по опре. тангенса. АН = 12, АС = 24.

648. Оля, 2 марта 2010, 18:45:33
Подскажите пожалуйста кто-нибудь как решаются задания В7 и В8?
34sin146 * cos146/sin292 -не получается никак((

649. Леонид , 2 марта 2010, 18:54:41
ОЛЯ. 17*(2* синус146*сосинус 146) : синус 292 - в скобках сверни по формуле в синус двойного угла, сократи, останется 17, и все дела. Проще не бывает. УСПЕХОВ!

650. Оля, 2 марта 2010, 19:02:00
Cпасибо большое!

651. Викуся, 2 марта 2010, 20:57:40
В треугольнике АВС угол А= 45 градусам
АВ=8, АС=12.
Найти остальные элементы треугольника

652. Леонид , 2 марта 2010, 21:12:36
Викуся, нахои по теореме косинусов ВСкв. = АВкв. + АСкв. - 2АВ*АС*косинус 45 град. ВСкв.= 64+144-(2*8*12* корень кв. из 2/2)= 208-96корней кв. из 2. По той же теореме косинусов, например для АС найдёшь уголВ.Затем и угол С. Какие ещё элементы? Откуда эта задача?

653. Леонид , 2 марта 2010, 22:33:23
Lexxus, что за адреса выдали (умничка) на "каким будет ЕГЭ...", есть смысл туда заходить? Невредно? Чтобы не вляпаться.

654. Lexxus, 2 марта 2010, 22:54:55
Lexxus, что за адреса выдали (умничка) на "каким будет ЕГЭ...", есть смысл туда заходить? Невредно? Чтобы не вляпаться.

Я бы не советовал. По ссылкам - исполняемые файлы (*.EXE), а я не слишком доверяю системе безопасности на своем компе. Завтра попытаюсь разобраться.

655. Леонид , 2 марта 2010, 23:09:02
Ясненько. СПАСИБО.

656. elli_93, 3 марта 2010, 01:49:01
Нужна помощь в решении вот такой задачи.
Боковое ребро правильной четырехугольной пирамиды SABCD равно , а площадь сечения пирамиды плоскостью W, проведенной через центр основания ABCD параллельно боковой грани SBC, равна 63. Найти расстояние от вершины A до плоскости W.

не понимаю даже какой угол требуется найти.(((

657. Леонид , 3 марта 2010, 06:43:12
Элли, почитай ещё разок своё условие. Что всётаки с боковым ребром? П причём здесь "не понимаю даже какой угол требуется найти", и найти расстояние от А до сечения? Ага? И что за задача? к ЕГЭ её отношение?

658. Natalie, 3 марта 2010, 11:30:17
Спасибо большое! А как решаются задания В8? не совсем понятно(

659. бабушка, 3 марта 2010, 11:49:30
Lexxus! Леонид! Что-то Вы обошли вниманием мою задачу от 28 февраля.
Может быть все-таки посмотрите, поможете. На две предыдущие задачи
Lexxus очень "красиво" ответил.

660. DARINDA, 3 марта 2010, 12:38:12
Здравствуйте...Я не думала,что когда-либо прибегну к тому,что буду просить помощи в решение(до этого момента как-то сама справлялась),но видно этот момент настал))) Я понимаю,что эти 2 задания проще простого, но сомневаюсь в решение, помогите,пожалуйста,всем,чем сможете))))))
1. Через среднюю линию основания треугольной призмы, объем которой равен 32, проведена плоскость, параллельная боковому ребру. Найдите объем отсеченной треугольной призмы.
2. Во сколько раз увеличится объем правильного тетраэдра, если все его ребра увеличились в 2 раза
Заранее спасибо. И...напишите ответ,пожалуйста, побыстрее,а то скоро ночь,и моя учительница,если я не принесу решение завтра, меня зарежет)))))



661. DARINDA, 3 марта 2010, 14:05:43
Хны-хны...Ну,пожалуйста,напишите решение,вы же все такие умные... Мне уже скоро баю-бай, а я без решения задач...

662. Natalie, 3 марта 2010, 14:47:34
Добрый день!
Lexxus!Я конечно понимаю, что задачу, о которой я спрошу очень простая.Но, к сожалению, я отношусь к числу неисправимых гуманитариев((((
Два велосипедиста одновременно отправились в 104 км пробег. Первый ехал со скоростью на 5км/ч больше, чем скорость второго и прибыл к финишу на 5 часов раньше второго. Нужно найти скорость второго велосипедиста.
Заранее спасибо!!!!

663. Леонид , 3 марта 2010, 15:20:24
DARINDA, не хныч. Раз боишья, что учительница "ЗАРЕЖЕТ" - это она правильно сделает, значит толк будет. Не знаю как получится без чертежа, но попробую:
1) объём данной призмы S осн*Н = 32. Сечение, о котором ты пишешь -тоже треугольник со стороной в 2 раза меньше (там средняя линия). А площадь значит будет меньше в 4 раза, чем у данной. Высота призмы остаётся той же. Значит S/4*Н = (SH)/4 = 32:4=8.
2) "Во сколько раз увеличится объем правильного тетраэдра, если все его ребра увеличились в 2 раза". Если линейная величина увеличиваетс в 2 раза, то при нахождении объём увеличится в 8 раз. В 4 раза площадь основания (как в 1-ой задаче) и в 2 раза высота - всего в 8 раз (ответ)

664. DARINDA, 3 марта 2010, 15:26:30
Спокойной всем ночи и спасибо за "помощь"...((( А я,пожалуй,возьмусь за веревку и мыло, лучше своими руками покончить с собой,чем со мной покончит учитель...((( Или... я еще подожду 8ч(как раз высплюсь) и перед школой еще РАЗОК посмотрю, нет ли решения...Ну,уж если решения не будет,придется попрощаться с родней....

665. Леонид , 3 марта 2010, 15:35:18
НАТАШКА. х-скорость второго (её найти надо). Тогда скорост первого (х+5). Время второго 104/х, время первого 104/(х+5). Первый прибыл на 5ч. раньше (меньше на 5ч. времени затратил). Уравнение: 104/х - 104/(х+5) = 5.

104(х+5) - 104х = 5х(х+5) - привели к общему знаменателю. раскрывай скрбки, всё в одну часть, подобные, кв. ур-е 5хкв.+25х-520=0 или хкв. +5х-104=0. Находи корни.Берём 8 (проверь, делал по ходу, почти устно)

666. DARINDA, 3 марта 2010, 15:35:28
Леванид! У меня небольшие проблемы с компом, т.ч изначально ваше решение не увидела))),следующее мое сообщение не в счет(с собой я и так ничего не собиралась делать,думала пожалеете и быстрее напишите решение)))) Думаю,я вас не разочарую,дело в том,что я решила эти задачи,но не была уверена в ответе.т.к изначально решила, но забыла,что речь идет об объеме(решила через S полной поверхности. Старею,зрение меня подводит))))Так что огромное спасибо,теперь я уж точно уверена в своем решение))))Эх,каких умных людей нам страна понарожала!!!

667. elli_93, 3 марта 2010, 15:53:47
Леонид! Писала условие в 2 часа ночи, заморочилась просто, написала ерунду. на самом деле вот так оно выглядит.
Боковое ребро правильной четырехугольной пирамиды SABCD равно 5 кв. корней из 7 , а площадь сечения пирамиды плоскостью W, проведенной через центр основания ABCD параллельно боковой грани SBC, равна 63. Найти расстояние от вершины A до плоскости W.
я имела ввиду, что даже не понимаю, какой треугольник надо рассматриать,чтобы найти расстояние. Ведь это перпендикуляр от А до плоскости W? Можно заменить расстоянием от середины АД до W. или от ценра основания до SBC -это ведь будут всё одинаковые расстояния? Мне вё равно не хватает данных, чтобы решить. или ума?(((

668. Леонид , 3 марта 2010, 15:58:35
Юмористка - "старушечка" - зрение имею ввиду - до старости учишься. Ясно, что шутки. С ДВ?

669. дашуля, 3 марта 2010, 21:54:32
фурсенко казнить!!!!!!!!!!!!!!!!!!!!!!

670. Natalie, 4 марта 2010, 03:30:19
Леонид, СПАСИБО БОЛЬШОЕ!!!
Здорово конечно, когда такие задачки устно решаются. А ты вообще до какого задания решаешь? С5/С6 получается?

671. Леонид , 4 марта 2010, 06:38:09
Дашуля,
перейди в другое место (каким будет ЕГЭ..., Тесты онлайн....), здесь народ это мокрушество не пишет, а делом занимается - не читешь? А там тебе понравится.

Элли,
через некоторое время. Геометрию здесь очень трудно - без чертежа.


Наташка,
да решаю помаленьку. И до 6 разные по-разному от сложности. Да как быстро идея блеснёт и не погаснет. Пиши свои С, я их собираю. Что решу напишу. Не получится Lexxus попросим, там кто-то толковый одминистратор, постоянно откликается.

672. DARINDA, 4 марта 2010, 08:43:36
Здравствуйте, Леонид! ОГРОМЕДНОЕ спасибо за решение,сегодня получила пятерку по геом)))Да, я с ДВ,точнее сказать с о.Сахалина,вы не думайте,мы не дикари,просто оторваны от человечества водным пространством,но благодаря СМИ и НЕТу в курсе всех событий))).А можно спросить у мозга этого проекта,тобишь у вас..откуда вы? Дело в том,что с Москвой разница во времени у нас на 7ч и мне бы хотелось знать во сколько вы обычно выходите сюда,для того,чтобы разъяснить решение(думаю,больше вас обременять с решением не буду,а вот на счет идеи решения я бы могла проконсультироваться,решать люблю сама,просто не всегда могу увидеть в решение очевидные вещи...)...И еще один крохотяшный вопросик...надеюсь,вы такой же смертный как и мы(т.е обыкновенный ученик)?никакая-нибудь учительница с ДВ?Просто,если я не ошибаюсь,здесь зависает моя учительница, и она под другим именем(под каким неизвестно,но сказала,что может быть всякое)))))

673. elli_93, 4 марта 2010, 10:42:08
БАБУШКА! Ваша задача меня заинтересовала. Красиво не получилось решить. Уравнение не составила. Но мне кажется она легко решается с помощью рассуждений.
Между стартом и финишем горнолыжного спуска круглосуточно действует подвесная канатная дорога. кабинки сверху и снизу отправлятся одновременно каждые 3 минуты. время движения в одну сторону составляет 14 минут. на старте и финише какбинка стоит 1 минуту, вклчая режим проскальзывания каната. определить количество кабинок, двигащихся вниз, которые встречатся горнолыжнику при подъеме
кабинка совершает полный оборот за 30 минут.(14+1+14+1). значит, всего курсирует 10 кабинок (30/3). Когда 1 начинает подьём, 10ой предстоит ещё 2 минуты спуска(30-3,1 минута стоит), т.е они встретятся. Когда 1ая прошла 12 минут, 2ая кабинка начала своё движение вниз, т.е они тоже встретятся. Таким образом, 1ая кабинка встретится со всеми остальными 9 кабинками.

674. Владислав, 4 марта 2010, 10:49:32
Интересно) сегодня вечером гляну местный банк задач, ибо качать все это с реального сайта лень-матушка не позволяет.

675. elli_93, 4 марта 2010, 11:09:20
Леонид! Ты писал Даринде: Если линейная величина увеличиваетс в 2 раза, то при нахождении объём увеличится в 8 раз. В 4 раза площадь основания (как в 1-ой задаче) и в 2 раза высота - всего в 8 раз (ответ)
Это всегда так бывает? Или тоько с тетраэдрами?

676. Леонид , 4 марта 2010, 12:52:21
Да. Элли. Только внимательно вчитывайся в условие, многие задачи из банка так решаются. Я называю их "игрой с формулами": бери формулу того, что дано, в неё же вноси предлагаемые изменения (увеличили , уменьшили в ? раз), выделяй в новом данную формулу - ответ готов. Геометрическую (про пирамиду) ещё не смотрел, по чертежу всё ясно, но не прорешивал (остаточность данных). Кстати, что это за задача.

Даринда, я из Кузбасса. Знаешь такой? Так что не боись своей училки. Мы побеим. Но, видно, что она стремится, чтобы её чады что-то знали. Поэтому, подведёшь, точно "ЗАРЕЖЕТ".

Почему поинтересовался РЕГИОНОМ? Так что не боись своей училки. Мы победим.

Девчата! Наши разные пояса во времени. Надо делиться заданиями С1-6. Вполне взможно , предлагаемые у вас в ноябре, декабре и феврале могут оказаться у нас на самом ЕГЭ. Правда, поясов временных у нашей прекрасной Роины Матушки России много.

677. DARINDA, 4 марта 2010, 15:01:10
О великий мозг, не могли бы вы подсказать идею решения вот такого чудненького и столь кратенького уравнения: 3^n-2^m=1
Я так понимаю, верно будет при n=1 и m=1 (3-2=1)
n=2 и m=3 (9-8=1) замечательный метод подбора =). Затем, я пыталась найти хоть какую-нить закономерность для того,чтобы выяснить,есть ли еще какие-нить варианты решения или убедиться в том,что решений нет))))).Перепробовала все(мне так кажется))),но не нашла...Графически получилась белибердень...
Что я упустила?Ну, или подскажите с чего можно начать...И,пожалуйста,скажите мне,что решение очень краткое и быстрое))))

678. Natalie, 4 марта 2010, 15:06:02
Леонид! Вот например задачка С3:
В прямоугольном параллелепипеде ABCDA1B1C1D1 у которогоАВ=4, ВС=6, СС1=4. нужно найти тангенс угла между плоскостью АВС и прямой EF, проходящей через середины ребер АА1 и С1D1.
Я кстати тоже из Кузбасса, а ты откуда именно?

679. RiBk@, 4 марта 2010, 15:23:48
как успехи у вас в подготовке к ЕГЭ?

680. elli_93, 4 марта 2010, 18:14:15
Даринда. я не поняла записи 3^n-2^m=1.
это что ли 3 в степени n - 2 в степени m=1 или 3 в степени (n-2) в степени m=1 или 3 в степени(n-2) умножается на m.?

681. katya.16ru, 4 марта 2010, 18:35:53
помогите разобраться с задачей в12.
бабушке для вышивания картины требовалось ниток теплых тонов в 2раза больше ,чем холодных. После того ,как она заменила некоторые нитки по своему вкусу, ниток теплых тонов оказалось в полтора раза больше,чем холодных. во сколько раз бабушка использовала ниток холодных тонов больше ,чем предполагалось изначально?

682. DARINDA, 5 марта 2010, 02:04:18
Здравствуйте,Элли.Когда я писала уравнение,то не подумала,что у кого-то могут возникнуть вопросы по записи примера(^-знак степени,и первая ваша трактовка уравнения верна), но после вашего послания я снова обратилась к записи уже моего послания и поняла в чем проблемы.Запись цифр(шрифт,написание) выбраны не совсем удачно,тройка не пропорциональна двойке,а именно больше двойки (убедитесь сами 3 и 2),именно это и вызвало у вас подозрения того,что данный пример можно трактовать по-разному.Думаю,если бы я хотела написать что-то другое, я бы поставила скобки,именно так как это сделали вы...

683. DARINDA, 5 марта 2010, 02:09:46
Кстати,кто решил извратиться над написанием? Нельзя было поставить Times New Roman, так как это в Word(е), тогда бы с цифрами проблем не было и вопросов по записи примеров не возникало?

684. Леонид , 5 марта 2010, 10:46:12
ДАРИНДА, не бузи и не применяй слов (четвёртое) Почитай так аль нет: (2 в ст. Н) - (3 в ст. М) = 1 (попытался извернуться). Оно? И ещё: какое условие к нему? "решить ур. в целых числах"? М.б. "в натуральных"? Я бы рассуждал так: две ф-ции у=2в ст. Н и у = 3 в ст. М +1. В одной сист. координат., обе монотонно возрастающие - показательные. Графики их могут пересечься справа от ОХ и только в одной (пока сомневаюсь) точке. Подбором х= 1. Ещё подумаю!

Элли! Смотри.

685. DARINDA, 5 марта 2010, 11:52:13
Дорогой Леонид, я вас тоже попрошу не применять в мой адрес таких слов как второе с частицей не(немного на блатняк смахивает или на феню)!И почему вы переделали мой пример, я его не так записала(посмотрите выше запись)...и решить надо в натуральных...А теперь прошу прощения за свой высокомерный тон и сарказм,вы меня задели(хотя в литературном значении это слово под собой не подразумевает того,о чем вы могли подумать!обозначает исказить,испортить...)Надеюсь,вы меня простите и простите мой вспыльчивый нрав :-)А так же поможете дорешать уравнение :-) I'm sorry

686. Леонид , 5 марта 2010, 15:43:39
ДАРИНА, я не переделывал твой пример. Записал по-своему, что-ли неверно записал уравнение? Я читал, что написано выше. И нет никаких обид. Здесь печатать сложно. Поэтому и обратился. Какой категории сложности, какое С? И откуда взято? Пойми-это важно знать!

687. Алена, 5 марта 2010, 16:22:57
уф... читаю читаю... глазки уже болят) решила сдавать егэ во второй раз (школу закончила 2 года назад), причем решила неделю назад. Зарегистрировалась, взяла демо варианты и ужаснулась) части А нет, В и С совершенно незнакомые. Интегралов и производных практически нет, а их в универе хорошо подучила. Школьную программу забыла ужас просто. Как подготовиться самой за три месяца? может кто книжки хоть подскажет хорошие)))

688. Леонид , 5 марта 2010, 18:09:55
Как же, Алёна, В 8 и 11, всё на производной. С в1 по в7, для Вас вполне подсилу. Уже неплохо. Вполне возможно.

689. DARINDA, 6 марта 2010, 07:57:25
Здравствуйте,Леонид. Задание С6,это задание было на одном из пробников. Если следовать вашей трактовке уравнения,то действительно корень=1.Моя же запись выглядит следующим образом 3^n-2^m=1(именно поэтому корни следующие n=1 и m=1,n=2 и m=3-метод подбора,но этот метод не математическое решение.Вопрос: есть ли еще корни?или как доказать что больше корней нет?Ну,и как решить математически?)

690. Алена, 6 марта 2010, 12:04:08
Да конечно, часть В я решаю практически устно, а вот за С пока не бралась, смотреть страшно)) единственная загвоздка с В10( про собирающую линзу) почитала тут у вас как надо решать но все равно бред получается. Если f=30 d1=30-50 d2=150-180. Как решить? что то не получается у меня. Найти надо наименьшее растояние d1...

691. Артём, 6 марта 2010, 18:10:29
Найдите точку максимума функции y=(x-7)^2*e^(x-2)
Производную я нашел:
y`=e^(x-2)*(x-5)(x-7)
А вот дальше что делать?, подскажите пожалуйста!

692. Леонид , 6 марта 2010, 18:33:17
Приравнивай производную к 0, х=5 и х=7. Находи знак производной на промежутках до 5, от 5 до7, от 7 .Точка, проходя которую, знак произвоной меняется с + на -, в ней и будет максимум.

693. Артём, 6 марта 2010, 19:04:22
Задания типа B8 почти полностью научился решать, но вот эти задачи вызвали затруднения:
8271
7825
8247
8253
объясните пожалуйста как они решаются!

694. Артём, 6 марта 2010, 19:11:39
Леонид ,А почему после того как приравниваешь производную к нулю, то получается, x=5 и x=7?

695. Леонид , 6 марта 2010, 19:52:21
Артём, я по твоей записи. Чтобы установить точки минимума (максимума) следует найти производную(ты её нашёл и вроде верно), производную приравниваем к о и решаем уравнение е в ст. (х-2) * (х-5)*(х-7) = о. Произведение равно нулю, когда? Да? е в ст. (любой) никога не = 0, более того всегда положительн. Ну и всё: х-5=0 или х-7=0 и твои х=5 и х=7. Далее написал. Темка, имей в виду, в этих заданиях некоторых надо быть внимательным. Производная, например е в ст. (5-х) - это -е в ст.(х-5).
Артём, долго прыгать, искать номера. Чиркни.

696. Леонид , 6 марта 2010, 20:26:41
Артём, нашёл твои номера, описать сложно без картинки.
Пробую. №8271 -дан график ПРОИЗВОДНОЙ, функция возрастает на промежутках, где производная положительна - это промежутки (-4;-2,...] и [4,...;8) (на них график производной выше оси ОХ). Целые точки, входящие в эти промежутки - складываем (-3+5+6+7 = 15), ответ 15.
№ 8253 аналогичен: в промежутках (-4,...;-1,...] точки (-3 и -2), на [2,...;4,...] точки (3 и 4), их сумма (-3-2+3+4=2).

№7825- Число максимумов по графику производно: максимумы (минимумы) там, где график ПРОИЗВОДНОЙ пересекает ось ОХ (производная равна 0). Максимум там, где произвоная меняет знак с + на -, проходя эти точки. Их, 2 в указанном условием промежутке(в точках -13 и -3). Ответ:2

Понял?

697. Slam, 6 марта 2010, 21:19:22
ТОлько что решал задачи из B10 и увидел одну задачу №28359:
"не менее (B) километров?"- Это сколько километров????????

698. Леонид , 6 марта 2010, 22:03:44
И задачи №№ 28355, 28357. На консультацию Lexxus надеемся.

699. Леонид , 6 марта 2010, 22:15:15
АРТЁМ. № 8247. График производной вункции на [-4,...; -0,...] выше оси ОХ, производная здесь положительная, а функция значит возрастающая - целые точки (абсциссы: -3-2-1 = -6).

700. Lexxus, 6 марта 2010, 22:59:26
И задачи №№ 28355, 28357. На консультацию Lexxus надеемся.

В задачах 28355, 28357, 28359, 28361 и 28363 теперь нормальные условия и нормальные ответы. Спасибо за сигнал=)

701. Артём, 7 марта 2010, 10:30:58
Спасибо! Только вот в задаче 7825 там надо найти минимумы, и ответ будет 3
С заданиями B8 вроде бы все понятно, а вот B11 № 4099 никак не понимаю(((((

702. Вика, 7 марта 2010, 11:10:04
4479 как решить этот пример!!!!!!!

703. Slam, 7 марта 2010, 13:54:41
В некоторых задачах(например №6561) Вместо формул выходит непонятно что.
Может это только у меня так? Проверьте пожулауйста

704. марина, 7 марта 2010, 14:09:11
люди!!!!!!!!! Это сайт самовыражения своей точки зрения по поводу егэ а вы что тут устроили "басплатное репетиторство"?

705. Леонид , 7 марта 2010, 14:20:12
Артём, Да в 7825 будет 3 точки минимума. Я тебе написал 2 максимума. Ну ведь и в других твоих номерах теже идеи,только по готовому графику - сопоставь условия вдумчиво.
№4099. Ты верно нашёл производную? Я писал по твоей. Итак 1)найдём призводную (производная произведения. ДА?). у штрих= 2(х-7)*е в ст.(х-2) + (х-2)в кв. * е в ст.(х-2) - за скобки = (х-7)*е в ст.(х-2) * (2 +х-7) = е в ст.(х-2)* (х-7)(х-5) - ну далее приравнивай её к нулю, т.к. функция (здесь) максимумы или минимумы может иметь там, где производная равна нулю. Это х=5 и х=7: 3 промежутка, методом интервалов (далее я подробно описал).

Артём, а что на уроках, учитель, друзья? Ты из какого региона. Напиши свои С.


Вика! - это ЛОГ 8 в ст. 1/2 по основанию 8 в ст. (-1) = (1/2):(-1) = - 0,5

706. Лёша, 7 марта 2010, 14:31:49
Слам, 6561 у меня неоткрылась в банке.

707. Елизавета, 7 марта 2010, 16:19:35
кто-нибудь скажет какое максимальное количество баллов за весь ЕГЭ по математике?

708. Вика, 7 марта 2010, 22:29:45
Спасибо за предыдущий ответ , скажите пожалуйста как №4403 решить?

709. Леонид , 7 марта 2010, 23:01:44
Считай изнутри здесь: ЛОГ 81 по осн. 9 = 2, тогда ЛОГ 2 по осн. 4 = 0,5 и все дела - Вика, примеры устные!!!!!!

710. Natalie, 9 марта 2010, 11:54:21
Всем привет!!!
Помогите пожалуйста решить задачку:
Объем 1-ого цилиндра 12 м.У 2-ого цилиндра высота в 3 раза больше,чем у первого,а радиус основания в 2 раза меньше.Нужно найти объем 2-ого.
Я рассуждала так:формула для объема выглядит так:V=п*r в квадрате*h.
Тогда объем 2-ого по формуле: V=п*3h*r кв/2/..... а дальше не знаю(((((

711. DARINDA, 9 марта 2010, 13:21:08
Здравствуйте, Натали. Я наткнулась на вашу задачу и решила попробовать ее решить)))...У меня получилось 9. Рассуждала следующим образом...Вы почти все записали верно, вот только r уменьшился в 2 раза, так и запишем r/2 (или 1/2*r), а теперь обратимся к формуле: у нас r в квадрате-возведем в квадрат,получим r в квадрате разделить на 2 в квадрате или (1/4*r в квадрате)
V=п*r в квадрате*h первый цилиндр = 12м
V=п*3h*1/4*r в квадрате,
поудобнее запишем вторую формулу: V=п*r в квадрате*h *3/4 (V=п*r в квадрате*h=12), то V= 12*3/4, V=9
Ответ: 9
Натали,старалась объяснить,не знаю получилось ли....))))

712. Ольга, 9 марта 2010, 14:49:26
основании прямой призмы АВСА1В1С1 лежит прямоугольный треугольник АВС ( угол С=90 градусов). Через сторону ВС и вершину А1 проведена плоскость: угол ВА1С=30 градусов , А1В=10, Ас=5 Найти площадь боковой поверхности призмы/ помогите решить(((((((((((((часа 4 мучаюсь

713. таша, 9 марта 2010, 16:33:28
для Ольги
у тебя получался ответ 50+50корень из 2

714. Леонид , 9 марта 2010, 18:20:59
ВС = 5, против угла 30 град.в прямоуг. треугольнике СА1В. В треуг. АВС (прямоугольный) по тю Пиф. АВ = 5 крней из 2. Из треуг. АА1С (прямоугольный) АА1 = 10, т.к. из треуг. СА1В по т. Пифагора А1С = 5 корень из 3. Боковая пов. - периметр основания на высоту (5+5+ 5 корень из 2)* 10 = 10 +50 корней из 2.

715. Леонид , 9 марта 2010, 18:26:37
Не пешите сюда свои Д/3.

716. Ника, 9 марта 2010, 19:12:40
А сколько макс. можно получить за часть Б и сколько за часть С?как баллы то распределяются?

717. katya.16ru, 9 марта 2010, 21:06:24
велосипедист выехал одновременно с мотоциклистом в другой город. причем с каждой минутой велосипедист ехал на 800 м меньше чем мотоциклист. на путь в 30 км мотоц-т затратил на 2 часа меньше чем вел-т. сколько ехал км в час мотоц-т?
нашла эту задачу в части В за2009год. помогите пожалуйста

718. Леонид , 9 марта 2010, 23:31:02
Поправка к задаче выше, ошибочка, ответ 100 + 50 кв. кор. из 2.

719. Эля, 10 марта 2010, 11:44:27
Как это решается y=4tgx-4x-x-П

720. Ника, 10 марта 2010, 14:52:48
А сколько макс. можно получить за часть Б и сколько за часть С?как баллы то распределяются?,ну пожскажите пожалуйста)

721. Леонид , 10 марта 2010, 15:11:55
что и на каком промежутке, Эля? И ещё, выше такие упражнения уже показывались. Полистай.
Не ищите за 2009, задач достаточно в банке заданий.

722. alfa20, 10 марта 2010, 18:37:03
Хотела посмотреть рейтинг (alfa20), а там:
"Ошибка
Пользователя с заданным номером (124384) в рейтинге не найдено.
Быть может, он существует, но не решил ни одного задания."
А в рейтинге пользователей - №7, как и был. Ничего не пойму... чё-то не ту кнопочку, видимо, нажала...Lexxus, помоги, пожалуйста!!!!

723. ka, 10 марта 2010, 19:01:11
я просто хотела узнать как такие задачи решаются

724. Carol, 10 марта 2010, 23:06:55
Помогите пожалуйста,вот сижу и туплю. Не знаю как решить,казалось бы,простую задачу:
Камень брошен вертикально вверх. Пока камень не упал, высота на которой он находился, описывается формулой h(t) = -5t2 + 18t ( h - высота в метрах, t - время в секундах, прошедшее с момента броска). Найдите, сколько секунд камень находился на высоте не менее 9 метров.

Подскажите пожалуйста ход решения.
Зарание огромное спасибо!

725. таша, 11 марта 2010, 09:52:49
для Carol
решаем квадратичное неравенство -5t2 + 18t больше или равно (т.к. по условию не менее) 9

726. ростислава, 11 марта 2010, 16:03:56
Что можно неписать про шестиугольную призму?Какие у нее боковые грани, ребра, основание, вершина.

727. лидия, 12 марта 2010, 12:47:35
Уменя проблемы с подготовкой к ЕГЭ по математике

728. Natalie, 12 марта 2010, 17:11:44
DARINDA!огромное спасибо!!!!Мне надо только еще чуть-чуть подумать и тоже бы получилось)))Но не могу решить еще одну задачу:
Двое рабочих вместе выполняют работу за 12 дней.За сколько дней первый рабочий выполнит этот же объем работы, если за 2 дня он делает столько же, сколько второй за 3 дня?
Помогите пожалуйста решить

729. Леонид , 12 марта 2010, 19:47:03
за х дней выполнит всю работу первый рабочий или за 1 день 1/х всей работы, тогда за 2 дня выполнит 2/х всей работы. Второй рабочий выполнит всю работу за у дней - за 1 день 1/у всей работы, за 3 дня 3/у. Составляем уравнения: 1) 2/х = 3/у, 2) 1/х + 1/у = 1/12 (всю работу вместе выполняют за 12 дней сказано). Из 1-го у= 1,5х, подсталяем во 2-е 1/х + 1/1,5х = 1/12 - всё в одну часть, к общему знаменателю, х=20.

730. Анастасия, 14 марта 2010, 15:51:05
какого хрена у меня уже в двух заданиях подряд получается два варианта ответа, оба правильные и оба удв. ОДЗ ??????????????????????????????

731. Леонид , 14 марта 2010, 16:31:38
Это где так? Только без овощей и приправ.

732. Андрей, 14 марта 2010, 17:54:46
ДАВАЙТЕ С6 обсуждать?

733. Маргарита, 15 марта 2010, 14:15:59
Я Рита.
От одного дома до другого 136,2 км. На встречу к друг другу выехали 2 трактора один двигался со скоростью 9кмчас а второй не извесно, но они встретились через 12 мин. С какой скоростью двигался 2 трактор?

734. Маргарита, 15 марта 2010, 14:17:45
енгшщ енгш как произошла планета?

735. Леонид , 15 марта 2010, 15:07:43
что-то км. и минуты не вяжутся, Рита.

736. регишка, 15 марта 2010, 18:20:33
если не сдам егэ,я повешусь...я не шучу......

737. Леонид , 15 марта 2010, 19:53:48
Lexxus!!! М.Б. уже и писал, В4 № 4661 не решается:угол С=90, Син.А=12/37, АС = 12, ВС-?. Моё решение: Кос. А = 35/37, Танг.А = 12/35. ВС = 144/35 - ответ Б.Д.дробь. (извиняюсь, не переходил в латиницу).

738. Леонид , 15 марта 2010, 20:12:50
Lexxus!!! В открытом банке задач под этим номером другая задача: АС=2, Син.А = (5 корней из 41)/41 - ответ 2,5. Интересно, много таких несовпадения задач под одним номером?

739. Lexxus, 15 марта 2010, 20:39:33
№ 4661 не решается

Действительно, не решается.

Интересно, много таких несовпадения задач под одним номером?

Думаю, что немного, но проверить сложно.

740. Леонид , 15 марта 2010, 21:23:49
СПАСИБО !

741. Муся, 16 марта 2010, 09:14:25
В8 прямая у=15х+94 является касательной к графику функции у=х(в кубе)+8х(в квадрате)-20х-200. Найти абсциссу точки касания. Брала производную, приравняла 15. Но корни у уравнения какие-то дикие. Как такое решается?.

742. Леонид , 16 марта 2010, 12:15:23
Муся, начало верное (корни -7 и 5/3 по твоим записям?). Теперь линейную и кубическую функции приравняй (коль является касательной - имеют общую точку), получишь уравнение, подставляй найденные корни, бери потом то, при котором равенство верное. Явно 5/3 в ответ не вписывается, ответ -7.

743. Аинька)))), 16 марта 2010, 19:34:23
Муся)я тоже сейчас решала,у меня тоже получились такие же корни как у Леонида))

744. Муся, 17 марта 2010, 07:12:34
Спасибо ребята. Не знаю, почему я этих корней испугалась, теперь буду, как советует Леонид, делать проверку. А скажите, почему здесь нет рисунков, и нельзя посоветоваться по заданиям С2 и С4 ?

745. саяна, 17 марта 2010, 07:38:41
блин, вообще! не провалить бы! если провалю, то конец всем моим планам! и дальше даже сама незнаю!

746. Леонид , 17 марта 2010, 09:27:12
Муся, пиши свои задачи из части С.

747. Нася, 17 марта 2010, 13:30:49
Всем привет!! у меня куча вопросов!!!например, я не могу понять, как решать В4!!!(((( вот такая задачка: дан равнобедренный треугольник АВС ( АВ=ВС, А=2, АВ=20) найти СН????

748. Муся, 17 марта 2010, 13:38:24
Пишу, Леонид, но как же мы будем понимать друг друга без русунка?

К диагонали A1C куба АВСD A1D1C1D1 провели:

1) перпендикуляры из середины ребер АВ и АD
Найти угол меду этими перпендикулярами

2) перпендикуляры из вершин А и В
Найти угол меду этими перпендикулярами



749. Леонид , 17 марта 2010, 15:20:23
Lexxus! Посмотрите, пожалуйста, в решениях вармантов 2,4,6,8 С1 -то ли опечатка, то ли ошибка. Д.б. sin x =-1/4 (там 1/4) в мониторинговой работе от 17.03.10 "Открытый банк заданий..."

750. Леонид , 17 марта 2010, 15:29:29
Нася, помотри своё условие, что значит А=2? Пиши.

Муся, - это две разных задачи ( угол м/у скрещивающимися прямыми) ? Я правильно понял? Откуда они? Пиши. Посмотрю - чиркну.

751. Lexxus, 17 марта 2010, 15:59:24
К диагонали A1C куба АВСD A1D1C1D1 провели:

1) перпендикуляры из середины ребер АВ и АD
Найти угол меду этими перпендикулярами

2) перпендикуляры из вершин А и В
Найти угол меду этими перпендикулярами

Так. Рисунки рисовать лень, так что включаем воображение.

1) Пусть K - середина AD, M - середина AB. Рассмотрим треугольники A1CK и A1CM.
A1C = корень из 3, A1K = CK = A1M = CM = (корень из 5)/2.
То есть, они равнобедренные и равны между собой. Значит, перпендикуляры из точек K и M к A1C сходятся в одну точку (Ну, например, N), делящую нашу диагональ ровно пополам.
Смотрим на треугольник MNK.
Из треугольника AMK: MK = (корень из 2)/2,
из треугольника A1NK = A1NM: MN = KN = (корень из 2)
А нужен нам угол MNK, который, если я ничего не напутал, будет равен
2arcsin(1/4).

2) А вот тут, конечно, можно и с таким условием посчитать, но там заковыристо.
Из соображений симметрии мне очень сильно кажется, что в условии были перпендикуляры не из A и B, а из A и B1.
Правильно?

752. Evan, 17 марта 2010, 17:00:49
1) Два велосипедиста стартовали одновременно и движутся в одном направлении с постоянной скоростью по трассе.В момент старта второй велосипедист находился перед первым на расстоянии 146 метров.Первый велосипедист догнал второго через одну минуту, пройдя расстояние равное 1/120 от общей протяженности дистанции и финишировал на 30 минут раньше второго.Определите скорость первого велосипедиста.

2)Из пункта А ушел первый поезд.Через 2 часа из пункта А в том же направлении вышел второй поезд и через 10 часов догнал первый поезд.Найдите среднюю скорость второго поезда, если сумма средних скоростей поездов равна 110 км/ч.

753. Evan, 17 марта 2010, 17:02:08
помогите решить выше выложенные задачи В12

754. Леонид , 17 марта 2010, 18:11:55
Иван, под какими номерами эти задачи в банке заданий ЕГЭ 2010?

755. Муся, 17 марта 2010, 18:34:23

1) Пусть K - середина AD, M - середина AB. Рассмотрим треугольники A1CK и A1CM.
A1C = корень из 3, A1K = CK = A1M = CM = (корень из 5)/2.
То есть, они равнобедренные и равны между собой. Значит, перпендикуляры из точек K и M к A1C сходятся в одну точку (Ну, например, N), делящую нашу диагональ ровно пополам.
Смотрим на треугольник MNK.
Из треугольника AMK: MK = (корень из 2)/2,

Lexxus, до этого момента у меня также, а вос потом нет.
Согласна, что искомый угол - это угол MNK. Его искала по теореме косинусов в треугольнике MNK: МК=(корень из2)/2, KN=MN=(корень из 2)/2.Их нашла, рассматривая прямоугольный треугольник А1MN. Получилось, что треуг. MNK -равносторнний, значит все углы 60 градусов и искомый угол тоже 60 градусов.

Второе условие записано верно, после долгих мучений искомый угол получился тоже 60 градусов. Ответов на эти задания нет, хотелось бы узнать - правильно или нет решено. Задачи из сборника тренировочных работ под редакцией Ященко. А все-таки почему сюда нельзя вставить рисунки?

756. Lexxus, 17 марта 2010, 18:52:49
Согласна, что искомый угол - это угол MNK. Его искала по теореме косинусов в треугольнике MNK: МК=(корень из2)/2, KN=MN=(корень из 2)/2.Их нашла, рассматривая прямоугольный треугольник А1MN. Получилось, что треуг. MNK -равносторнний, значит все углы 60 градусов и искомый угол тоже 60 градусов.

Всё правильно, это я ошибся. Извиняюсь.

757. Evan, 17 марта 2010, 18:58:35
Леонид, не знаю мне и знакомой на пробнике попались такие

758. Lexxus, 17 марта 2010, 19:07:13
Второе условие записано верно, после долгих мучений искомый угол получился тоже 60 градусов.

И это тоже правильно.
Этот случай даже проще, я просто не въехал поначалу))
Перпендикуляр к диагонали из А параллелен перпендикуляру из C1, а тот с перпендикуляром из B сходится в одну точку и образует с ним угол 120 градусов.
Значит, правильный ответ - 60, верно.

А все-таки почему сюда нельзя вставить рисунки?

Я подумаю, может, и сделаю такую возможность, раз так надо)

759. Леонид , 17 марта 2010, 19:09:05
Муся, я согласен с твоим решением - 60 град - 1 задача. И зачем т. косинусов, если получили все стороны равными? 2-ю не смотрел. А что в ответах автора брошюры?

760. Леонид , 17 марта 2010, 20:12:00
ВК - перпендикуляр к А1С в треуг.А1ВС (по ф. площадей 1*корень из 2 = ВК*корень из 3, ВК= корень из (2/3), аналогично и С1К из треуг. А1С1С. по т. косинусов: кос. К = -1/2, 120 град. Искомый ему смежный 60.

Пока набирал Ваше появилось.

761. Леонид , 18 марта 2010, 07:49:19
ИВАНУ. Задача 2. х-ск. второго, то ск. первого (110-х). Путь первого
12(110-х), путь второго 10х. Расстояние одно и то же - уравнение:
12(110-х)=10х. Ответ 60.

Условие задачи 1 проверь. Например: в каких ед. давать ответ? и др. данные.

762. Муся, 18 марта 2010, 08:17:32
Evan, решала твои задачи. Вторая легкая - х и у скорости поездов, далее легкая система 12х =10у и (х+у)/2=110.
А вот в первой, что-то не вяжется с цифрами, уравнение составила, но решить не могу, очень напрягает 1/120. Ты все точно написал?

763. Муся, 18 марта 2010, 08:33:23
.--------146+х--I-------------I
146 .---Х---I------------ II
момент
встречи через минуту.
то есть в ед. времени ( 1 минуту ) I прошел путь (146+х) и это будет его скорость, аналогично для II, в то же время (146+х) -1/120 дистанции.
Теперь все известно для составления уравнения, но по приведенным цифрам оно у меня не рашается.

Правильно или я ошибаюсь в рассуждениях?

764. Леонид , 18 марта 2010, 09:52:12
за х- что просят - ск.1-го. За 1 мин 1/120 всей истанции, значит на дистанцию 2ч. 2-ой затратит времени 2,5ч (150 мин). Скорость сближения 146, то ск. 2-го (х-146). Уравнение: 120х = 150(х-146) + 146 - дистанция одинакова. 30х=146*149 - плохой ответ. Либо напутал, либо что-то с условием.

765. Evan, 18 марта 2010, 10:36:19
Леонид, спасибо! Если решать (другую задачу из поста 752) так, это реально.
Пусть х м/мин скорость первого, а S м -длина всей трассы или путь первого.
По условию х = 1/120*S. значит S = 120х м и первый пройдет всю трассу за 120 мин
Скорость второго (х-146) м, его время до финиша 150мин, т. е. его путь 150(х-146) м и он меньше пути первого на 146 м.
Составляем уравнение: 120х=150(х-146), х = 730 м/мин = 43,8 км/час

766. Evan, 18 марта 2010, 10:51:20
Леонид, тогда так?

120х=150(х-146)-146, х = 725 м/мин = 43,5 км/час

767. Evan, 18 марта 2010, 10:56:46
Ошибка в записи

120х=150(х-146)+146, х = 725 м/мин = 43,5 км/час

768. Kama, 18 марта 2010, 12:42:32
Привет всем!!! люди можете помочь решить егэ ким 103???

769. Муся, 18 марта 2010, 13:26:52
Ребята, с задачами Ивана вы разобрались лучше, я пошла по более сложному пути и запуталась. А мне с задачкой поможете? Она из того же задачника.

Велосипедист от дома до места работы едет со средней скоростью 10 км/час, а обратно - со средней скоростью 15 км/час, поскольку дорога идет немного под уклон. Найти среднюю скорость движения велосипедиста на всем пути от дома до работы и обратно.
Простым сложением и делением пополам не решается, так как ответ - 12.

770. Леонид , 18 марта 2010, 16:00:41
Муси - для! х - время от дома до работы, у - от работы до дома (под уклон).
Путь туда 10х, обратно 15у - он один: 10х=15, х=1,5у.
Весь путь (туда и обратно) 10х+15у = 1,5*10у +15у =30у.
Время х+у 1,5у+у=30у. Скорость (30у):2,5у = 12. Проследи, при печатании ничего не пропустил?

771. Evan, 18 марта 2010, 17:07:36
Муся-посмотрите еще вариант: Пусть расстояние от дома до пункта назначения х, тогда время затраченное на работу х/10, а время затраченное с работы х/15. Так как для того чтобы рассчитать среднюю скорость нужно общее расстояние разделить на общее время получаем отношение Vср.=Sобщ./tобщ. , т.е. Vср.=2x/(x/10+x/15)=12.

772. Муся, 18 марта 2010, 18:27:19
Ребята, вы такие молодчинки, оба варианта решения простые и понятные, хотя и решены с разным подходом. Спасибо. У нас на 23 марта назначен очередной тренировочный экзамен, вот и тренируюсь. Самым сложным для меня в группе В - это задачи В8 . Не все я там понимаю. Но здесь без рисунка графика не проконсультируешься. Еще раз спасибо, будут затруднения, напишу.

773. Паша, 18 марта 2010, 20:14:35
В заданиях B 3 в открытом банке 3329 , а у вас всего 2887!

774. Леонид , 18 марта 2010, 20:35:38
Муся, умно подходишь к геометрии, В8 затруняешься, не верю. Читай их внимательно: график чего дан! Монотонность функции - знак производной, знак производной - монотонность; экстремумы - смена знака производной; производная - угловой коэффициент одно и тоже. После 23 пиши задания С - они больше интересуют. Вопрос по В8 - называй номер задания из банка-постараемя помочь. Ещё раз - они требуют пристального внимания и ничего сложного.

775. Айрат, 19 марта 2010, 08:37:17
Из пункта А в пункт В, расстояние между которыми 50 км, одновременно выехали автомобилист и велосипедист. Известно, что в час автомобилист проезжает на 40 км больше, чем велосипедист. Определите скорость велосипедиста, если известно, что он прибыл в пункт В на 4 часа позже автомобилиста. Ответ дайте в км/ч.

776. Леонид , 19 марта 2010, 09:17:13
х- ск. велосипе., то (х+4) - ск. авто. Время и уравнение: 50/х - 50/(х+4)=4

777. sirina, 19 марта 2010, 12:29:57
Первый рабочий за час делает на 4 детали больше, чем второй рабочий, и заканчивает работу над заказом, состоящим из 713 деталей, на 8 часов раньше, чем второй рабочий выполняет заказ, состоящий из 837 таких же деталей. Сколько деталей делает в час первый рабочий?

778. Леонид , 19 марта 2010, 13:48:44
для 777. х - делает в час 1-й рабочий (заметь, - это надо найти), тогда 2-й делает в час (х-4) детали. 1-му на всю работу потребуется 713/х час.,
2-му 837/(х-4).
2-ой времени затратит на 8 час. больше - уравнение: 837/(х-4) - 713/х = 8.

779. Муся, 19 марта 2010, 15:28:48
Леонид, вот такой вопрос по В8
Функция определена на промежутке. Изображен график производной этой функции. График 4 раза пересекает ось ОХ. Надо найти точку минимума самой функции.
Я уже четко уяснила, что когда график производной функции пересекает ось ОХ, то в этих точках - экстремумы функции, причем, если переход с - на +, то это точка min, переход с + на - , то это max.
А вот на этом графике точек минимума 2 (две), так какую надо брать?

780. Леонид , 19 марта 2010, 16:06:19
Муся, ну, всё верно. В каком номере? Всё ли ты указала здесь из условия? Посмотри внимательно.

781. Александра, 19 марта 2010, 17:03:48
B12
Из пункта А в пункт Б выехали одновременно два автомобилиста. первый ехал весь путь не меняя скорости. второй же первую половину пути ехал на 16 кмч меньше первогоба вторую половину пути-96 кмч.нужно узнать скорость первого если известно что она больше 54 кмч и в пункт Б они приехали одновременно.

B9
правильная треугольная призма.уровень воды достигает в ней 16 см. нужно узнать какой станет высота, если сторону основания увеличить в 4 раза.

782. Муся, 19 марта 2010, 17:08:25
Ой. Леонид, правильно, я так нервничаю. когда вижу эти функции и графики, что пропустила важное: точку минимума надо определить на определенном интервале, а в нем только один переход с - на +. Значит и одна точка. Извиняюсь за беспокойство.

783. Леонид , 19 марта 2010, 18:10:50
МУСЯ, совет. Именно в этих упражнениях проявляй максимум внимания, собранности и спакойствия, главное без нервов,ну как Зоя Космодемьянская, и всё получится - уверен, судя по изложению мыслей в предписьме.

784. Леонид , 19 марта 2010, 18:30:39
САША. В12.
х ск. 1-го (её надо найти). Путь одинаков- я бы взял 2, чтобы не было дробей в числителе при составлении уравнения (увидешь ниже). Время 1-го 2/х.
(х-16) - ск. 2-го на первой половине пути, то время его зесь 1/(х-16), его же время на 2-ой половие пути 1/96. Пришли одновременно, уравнение:
1/(х-16) + 1/96 = 2/х. В ответ бери х, что больше 54.

В9. Обьём призмы (воды) не меняется, её просто перелили. Формула объёма призмы (Пл. осн.*Н). Первоначально (Пл. осн.)*16 = 16*(Пл. осн.)* Х - перелили (сторона увеличена в 4 раза, то площаь в 16 раз). (Пл.осн.) - одно и тоже до переливания и после. Уравнение: (Пл.осн.)*16=16(Пл.осн.)* Х - дели на (Пл.осн.) и на 16. Ответ 1. Внимательно,м.б. есть опечатки (набирал без записи на бумаге).

785. Александра, 19 марта 2010, 19:56:06
ой спасибки Леонид))) только я не очень поняла почему в 12 задаче мы не 1х а 2х

786. Леонид , 19 марта 2010, 20:20:35
Саша,если путь взять 1, то надо 0,5/(х-16) + 0,5/96 = 1/х , тебе хочется лишний раз с дробями возиться? Вот здесь умножь обе части ур-я на 2 и получишь то, что писал. 0,5 половина пути. я взял путь 2, то половина 1.

787. таша, 20 марта 2010, 13:16:42
в кубе АВСДА1В1С1Д1 найдите косинус угла между прямыми АВ и СА1. Решение: АС- проекция А1С на плоскость (АВС)-в которой лежит АВ. АВСД-квадрат, АС-диагональ его, значит косинус угла между прямыми АВ и СА1=косинусу угла ВАС=косинус 45 градусов =корень из двух на два. Решение правильное или нет? это С4

788. elli_93, 20 марта 2010, 16:18:32
таша. думаю, это неправильно.Углом между двумя скрещивающимися прямыми называется угол между двумя пересекающимися прямыми, соответственно параллельными данным скрещивающимся прямым.
так что искомый уголА1СД, тк при параллельном переносе АВ перейдёт в СД. пусть сторона куба=а. тогда в прямоуг треугольникеА1СД сторона А1Д=а корней из 2. соs=а/а корней из 2=1/2.
Леонид, как думаешь?

789. elli_93, 20 марта 2010, 16:24:59
что-то лёгкая задача с4. мне вот такая попалась. помогите решить.
в правильную призму, основание которой выпуклый четырёзугольник, вписана сфера. найти площадь сферы, если известно, что площадь основания четырёхугольника=9, сумма квадратов его сторон= 36^2, наибольшая диагональ призмы=q.

790. Муся, 20 марта 2010, 18:03:09
Для elli, согласна с твоим решением задачи Таши, только окончание смущает, мне кажется, что найден котангенс угла между А1С и АВ, так как в прямоугольном треугольнике А1СD угол D - прямой и отношение а к а*(корень из 2) это есть отношение прилежащего катета DC к противолежащему катету A1D. А искомый косинус - это отношение DC к А1С - а/а*(корень из 3).
Я права? А в следующей задаче непонятно, чему равно q.

791. elli_93, 20 марта 2010, 19:46:33
Муся! ты права. я была невнимательна. действительно 1/ корень из 3- вот наш косинус. А q просто q. ничего про неё не было сказано. видимо, в ответе получается выражение, содержащее q

792. elli_93, 20 марта 2010, 19:47:51
в правильную призму, основание которой выпуклый четырёхугольник, вписана сфера. найти площадь сферы, если известно, что площадь основания четырёхугольника=9, сумма квадратов его сторон= 36^2, наибольшая диагональ призмы=q.

793. Асель, 20 марта 2010, 21:21:39
как найти прямоугольный параллепипед наименьшей поверхности при данном объеме V. и это все, что написано в задаче,пожалйуста помогите)))буду очень признательна)))

794. Алиюш, 21 марта 2010, 08:26:26
К ванне подведены два крана. Через один она наполняется за 12 минут, а через второй в 1,5 раза быстрее. За сколько минут наполнится 5/6 ванны при двух открытых кранах?

795. Алиюш, 21 марта 2010, 08:31:19
помогите решить пожалуйста.К ванне подведены два крана. Через один она наполняется за 12 минут, а через второй в 1,5 раза быстрее. За сколько минут наполнится 5/6 ванны при двух открытых кранах?

796. Леонид , 21 марта 2010, 09:19:32
Девчёнки, откуда эти геометрические задачи, вами именуемые С4, С4 - задачи из курса плпниметрии! Не входил - что то глючило.

Елли, в твоей задаче (посмотрю позе, там наверное больше дел с алгебраическими преобразованиями). Для начала: четырёхугольник описан-равны суммы противоположных сторон. S=1/2ar+1/2br+1/2cr+1/2dr=9; (a и с -проттивоположные), то (а+с)r=9 и (b+d)r=9, 1/2r(a+b+c+d)=9. Н=2r. Пока остановлюсь, смотреть надо (сумма кв. сторон и - это q).

Алиюш. производительность (ск. наполнения 1-й) 1/12, 2-й 1,5*(1/12) = 1/8.
За 1 мин производительность вместе 5/24 ванны. 5/6 = (5/24)*Т. Т= 4 мин.

797. Алиюш, 21 марта 2010, 09:54:50
Леонид спасибо огромное, выручил... С утра думать не охота было...

798. Алиюш, 21 марта 2010, 10:25:31
не понимаю как решать такие задачки...((( Вот например: На изготовление 837 деталей первый рабочий затрачивает на 4 часа меньше, чем второй рабочий на изготовление 899 таких же деталей. Известно, что первый рабочий за час делает на 2 детали больше, чем второй. Сколько деталей в час делает первый рабочий?

799. Алиюш, 21 марта 2010, 11:09:13
На изготовление 837 деталей первый рабочий затрачивает на 4 часа меньше, чем второй рабочий на изготовление 899 таких же деталей. Известно, что первый рабочий за час делает на 2 детали больше, чем второй. Сколько деталей в час делает первый рабочий?

800. Умида, 21 марта 2010, 12:00:59
помогите пожалуйста......скажите сколько баллов надо набрать,а точней сколько заданий надо решить по математике ,чтобы получить оценку 3????а то мне математика совершенно не нужна,а сдавать ее надо!!!!

801. Артём, 21 марта 2010, 12:37:41
как решить? №26968

802. Леонид , 21 марта 2010, 15:26:30
Если думать не охота, толку не будет.

803. Леонид , 21 марта 2010, 15:51:31
Артём, что после записи 705 -здесь? Где 100г. и пирожок?

Нежелающая думать, смотри запись 778 - здесь.

"Скромность ваша, вас и красит!

804. Леонид , 22 марта 2010, 06:09:13
№26968.(Артём). Начинать можно по-разному. Запишем:
7sin a + 13 cos a =3(5sin a - 17cos a) (3=3/1 и по првилу пропорции). 7sin a + 13cos a - 15sin a +21cos a = 0 (раскрыли скобки и в одну часть, подобные) ,
-8 sin a - 8 cos a=0 (обе части поделили на косинус), 8tg a + 8=0, 8tg a =-8, tg a = -1.

805. Артём, 22 марта 2010, 17:14:46
Я из Казани , Татарстан. С тот пример "705" научился решать, спасибо!

806. $KUDG, 23 марта 2010, 11:58:46
ПРИВЕТ ВСЕМ Я ИЗ ПЕРЕСЛАВЛЯ!!!!!!:))))))

807. Гузель, 23 марта 2010, 12:03:42
эго это просто издевательвсто и просто тупизм

808. Гузель, 23 марта 2010, 12:05:15
ЕГЭ- это просто издевательвсто и просто тупизм

809. таша, 23 марта 2010, 12:57:59
Элли и Муся! Спасибо, что поправили, что-то тупить начала, не ту прямую спроецировала. Если хотите, то могу написать остальные задания ЕГЭ от 20 марта мои

810. Алексей, 23 марта 2010, 13:05:43
Объясните Пожалуйста как решить задачу...
Задание B10 (28547)
на странице http://live.mephist.ru/show/mathege2010/view/B10/notsolved/260/

811. elli_93, 23 марта 2010, 14:03:00
Таша! пиши все задания С. Мы их собираем. У нас завтра пробник. Напишу потом свои. Кто объяснит задачку Асель? как найти прямоугольный параллепипед наименьшей поверхности при данном объеме V. Я тоже не знаю, как такие решать!

812. Ксюша, 23 марта 2010, 17:04:48
Мне думается, что это куб, поверхность которого 6 корней кубических из кварата объёма.
Если прямоугольный параллелепипе (не куб), думается, д.б. ещё данное, а так 3 переменных- высота,ширина,длина.

813. таша, 23 марта 2010, 22:40:28
Элли! Для тебя! Писать задания буду так, как они читаются, если что-то будет непонятно, напишите. задания попались не сложные, до сих пор не могу понять как могла спутать прямые... торопилась, тк работы собрали меньше чем через три часа от начала - "все сдали работы и ты сдавай".
И ещё, очень хочется свериться ответами, так что жду...
С1. система двух уравнений: первое уравнение:два синус квадрат икс минус семь синус икс плюс три равно нулю; второе уравнение: шесть синус икс плюс пять игрек равно тринадцать.
С2 уже писала - случайно выдав его за С4 (про куб).
С3 Решить неравенство: дробь меньше одной второй. В числителе дроби стоит логарифм от скобка открывается корень из три икс плюс один далее от корня отнять три скобка закрывается по основанию одна третья и в знаменателе тоже самое только к корню прибавить три.
С4 две окружности радиусов 1 и 4 касаются внешним образом и находятся по одну сторону от касательной. найти радиус третьей окружности которая касается этих двух окружностей и указанной касательной.
С5 вспомню поточнее напишу. но пока кажется так : найти все значения а, прикоторых функция имеет не менее трё точек пересечений с осью абсцисс f(x)= x^2 -| x^2- 4x -5 |-a
С6 найти корни уравнения f(x) = x^2 + px +q если больший корень и значение функции при х=5 простые числа, а f(5) корни данногоуравнения натуральные числа
Срешением Ксюши согласна я тоже так думала

814. Леонид , 24 марта 2010, 04:37:10
Ташуля! спасибо!

815. таша, 24 марта 2010, 07:27:27
С5 с поправкой Найти все значения а, при каждом из которых график функции пересекает ось абсцисс более чем в двух точках

816. Леонид , 24 марта 2010, 07:41:16
С5 (Таша) - хорошая поправка (пробовал 3-х точек не получается), С2 у нас решено - девочки выше писали. С1 у меня простая, если в выч. не ошибся:
(-1) в ст. к арксин1/4 +2пиК;2,3). Другие ещё не смотрел. Таша, ты из какого региона?

817. Фи, 24 марта 2010, 20:26:06
Объясните плиз решение

Задание B9 (4979)
В основании прямой призмы лежит квадрат со стороной 5. Боковые ребра равны . Найдите объем цилиндра, описанного около этой призмы.
Ответ: 25

818. Леонид , 24 марта 2010, 22:18:08
ФИ. Цилинр описан около призмы - в осн. квадрат, значит круг - осн. цилиндра описан около квадрата (черти) АВСД - диагональ его из треуг. АСД по т.Пиф. АС= 2 кв. корень из 5, то радиус ОС = кв. корень из 5. По формуле объёма цилинра пи*R кв.*Н = пи*5*Н (боковое ребро ты не указал, но судя по ответу)*5/пи = 25. Всё понятно? (* - умножить).


ТАША. С4 Ответ (8/3;35/3)

819. таша, 25 марта 2010, 11:12:18
Леонид! А у меня получились ответы в С4 0.25 и 4
и в С1 тоже, кажется другие ответы, может ты задание не правильно понял? там все просто

820. Леонид , 25 марта 2010, 15:00:15
Это ответ ни в С4, а в С3 - решить неравенство (я допустил опечатку).
С1 - в первом уравнении Д=25,корни 1/4 и 3/2(не подх.). син. Х = 1/4.
2-е ур.: 6*(1/4)+5у=13, у=2,3. Что не так?

821. Фи, 25 марта 2010, 15:09:01
ЛЕОНИД
изивните, я пропустила, боковые ребра равны 2/пи

822. Леонид , 25 марта 2010, 17:50:24
Фи, тогда ответ 10.

823. Леонид , 25 марта 2010, 18:07:01
ЗАПИСЬ 818. ФИ, и я извиняюсь. Поднапорол с вычислением радиуса. "Цилинр описан около призмы - в осн. квадрат, значит круг - осн. цилиндра описан около квадрата (черти) АВСД - диагональ его из треуг. АСД по т.Пиф. АС= 2 кв. корень из 5, то радиус ОС = кв. корень из 5. По формуле объёма цилинра пи*R кв.*Н = пи*5*Н (боковое ребро ты не указал, но судя по ответу)*5/пи = 25. Всё понятно? (* - умножить)."

НОВОЕ. Решение такое же, только диаметр 5 корень кв. из 2.
Радиус (5/2)*корень из 2.Объём = пи * на квадрат ((5/2)корней из 2) *2/пи = =пи* (25/4)*2*(2/пи) = 25 Теперерь ОК.


824. леся, 26 марта 2010, 09:13:54
я очень хочу чтобы еге прошло быстро

825. маня, 26 марта 2010, 10:16:19
Помогите решить задачу В12: четыре рубашки дешевле куртки на 20%. на сколько процентов шесть рубашек дороже куртки?

826. Леонид , 26 марта 2010, 11:30:34
Маня, номер этой заачи в банке заданий?

827. таша, 26 марта 2010, 12:35:26
Деонид! при Д=25, корни 3 и 0,5, там коэффициенты квадратного уравнения 2t^2-7t+3=0 откуда корни 0,25 и 1,5?

828. таша, 26 марта 2010, 12:43:46
Для Мани! Если 4 рубашки дешевле куртки на 20%, то 4 рубашки составляют 80%, отсюда 1 рубашка составляет 20%. 6рубашек -120%, значит 6 рубашекдороже куртки на 20%. Я думаю так.

829. elli_93, 26 марта 2010, 12:55:11
Таша! у меня в с3 получилось так же, как у Леонида.Ответ (8/3;35/3)
в с1 у=2, х=пи/6+2 пиn, х= 5/6пи+2пиn,в с 4 у меня 2 ответа 3/4 или4/5, а с параметрами я не сильна. если будет не трудно, напиши,как решала

830. elli_93, 26 марта 2010, 12:57:29
Таша, а у тебя какой ответ в с3?

831. таша, 26 марта 2010, 14:28:39
В С1 ответы х=(-1)^n*П/6+Пn; у=2
С3 ответ такой же
С4 я уже писала R=1/4 или 4
С5 с помощью графика - нарисовать не могу

832. Леонид , 26 марта 2010, 14:47:22
Таша, elli 93, dы правы с С1 - делил, почему - то на 8, а не на 4, когда находил корни.

833. elli_93, 26 марта 2010, 17:17:07
Таша. поправка с 4 у меня 2 ответа 4 или 4/9

834. elli_93, 26 марта 2010, 17:40:40
Таша! я вот по этой формуле считала.1/корень из b + 1/корень из a = 1/корень из c. ,где с- радиус окружности, заключённой между двумя крайними с радиусами а и в. А ты как искала?

835. Леонид , 26 марта 2010, 19:04:24
elli_93. 4/9 - да (здесь маленькая окружность между двумя данными и касательной). Опишите второй случай чертежа. Откуда предложенная формула - сама вывела?

836. Фи, 26 марта 2010, 19:17:56
ЛЕОНИД , спасибо!


объясните плиз решение вот этой задачи

Два ребра прямоугольного параллелепипеда, выходящие из одной вершины, равны 3 и 4. Площадь поверхности этого параллелепипеда равна 94. Найдите третье ребро, выходящее из той же вершины.

ответ:5

837. elli_93, 26 марта 2010, 19:55:11
Леонид! чертёж тот же. просто представь, что тебе дан радиус маленькой окр(которая между двумя большими) и радиус одной большой. а надо найти радиус второй большой. Формулу взяла из задачи, которую когда-то решала. и она работает! но можно вывести и самой. только без рисунка сложновато объяснить. Попробую

838. elli_93, 26 марта 2010, 20:24:01
Рисуй две окр с центрами А (радиус а) и В(радиус в). третья (маленькая между ними) с центром С радус с. Проекции В,С,А на касательную обозначим В1,С1,А1.проекцию точки С на прямую ВВ1 обозначим Св, проекцию точки С на прямую АА1 обозначим Са, проекцию точки А на прямую ВВ1 обозначим Ав, Рассматриваем прямоуг. треугольники и по т. Пифагора.ААв^2=АВ^2-ВАв^2=(в+а)^2-(в-а)^2=4ва=В1А1^2. Тогда В1А1= корень из 4ва. Аналогично ССв^2=СВ^2-ВСв^2=(в+с)^2-(в-с)^2=4вс=В1С1^2 иВ1С1=корень из 4вс. из треуг САСа находим аналог. С1А1=корень из 4ас. Но В1А1=В1С1+С1А1. ПОДСТАВЛЯМ найденные значения и получаем корень из ав= корень из ас+ корень из вс. (4 вынесли из под корня и сократили). дальше арифметика. получается 1/корень из с=1/корень из b + 1/корень из a . Надо только помнить, что с- радиус окружности, которая между двумя большими.))) Подставляй 4 и 1 сначала вместо а и в, а 2 случай подставляй 4 и 1 вместо а и с

839. elli_93, 26 марта 2010, 20:32:22
ФИ! Подставляй данные в формулу 2(св+са+ав) =Sполн.
2(4а+3а+3*4)=94
7а=47-12
а=5

840. Леонид , 26 марта 2010, 21:10:41
elli_93. Спасибо. Я не подумал, что может оказаться даны радиусы соседних окружностей и достроить справа или слева, радиус которой слеует найти. Ну а решение моё на 4/9 выглядит так (хотя суть таже):по твоим обозначениям (если где-то опечатался - ты поймёшь). Теже треугольники - прямой через центр маленькой, параллельно касательной и линии между центрами с данными радиусами.
САа = корень из((1+с)кв. - (1-с)кв.) и ((4+с)кв. -(4-с)кв.)= СВв. Их сумма 4 (между точками касания на прямой). Под корнями всё хорошо получается и 2корень из с + 4корень из с = 4,
6 корень из с =4, 3корень из с = 2, 9с=4, с=4/9.
Ну надо же, посчитал по условию - даны радиусы крайних окружносте...
Ещё раз спаибо, надоумила, не вчиталься в условие. А решение аналогично бует.

841. Леонид , 26 марта 2010, 21:31:01
elli_93. По С5 соображения. Помоульный кв. трёхчлен имеет корни (-1) и 5. От моуля по опреелению:
1)если х меньше -1 и больше 5, имеем:
Ф(х) = х кв. - х кв. +4х+5 - а = 4х+5 -а - прямая, она не может иметь двух точек пересечения с другой прямой.
2) если -1 меньше или равно х меньше или равно 5, имеем:
Ф(х)= х кв. + х кв. -4х - 5 - а. Ф(х)= 2х кв. - 4х -(5+а) - будет иметь неменее 2 точек пересечения (ветви видно вверх), если Д больше 0.
16+40+8а больше 0. а больше - 7. Как ты считаешь?
Можно показать и графически.

842. elli_93, 26 марта 2010, 23:00:34
Леонид! я не умею решать с параметрами. Рассуждения вроде правильные, но если я строю график(опять же не уверена, что правильный), то получается при а=0 - три пересечения, а вот уже при -1 или 1 только два(((. может, попросим помощи у Lexxus?
Lexxus! помоги!найти все значения а, при каждом из которых график функции пересекает ось абсцисс более чем в двух точках
f(x)= x^2 -| x^2- 4x -5 |-a

843. Леонид , 26 марта 2010, 23:52:57
Да и а меньше или равно 1, 4х +5 - а = 0 - ещё 2 точки при х = -1, -4+5-а=0, а=1 (от -7 до 1 с =)

844. таша, 27 марта 2010, 09:56:23
С4 Я решала так.
Две окружности радиусов 1 (маленькая, центр О1) и 4 (большая, центр О2), лежат по одну сторону от касательной ВС (точка В принадлежит маленькой окружности, О1В=1; а точка С принадлежит большой окружности, О2С=4). Точка А - точка касания двух окружностей ( О1А+АО2=1+4=5). Рисунок понятен? Далее. Получили прямоугольную трапецию О2СВО1.В ней опускаем из точки О1 на сторону О2С перпендикуляр О1Д.Получили прямоугольный треугольник О1ДО2 и прямоугольник ДСВО1. ВО1=СД=1, отсюда ДО2=3, значит катет ДО1=4.
1случай, когда третья окружность- самая маленькая находится между данными коружностями. Пусть О3-центр третьей окружности, ЕО3-радиус её, ЕО3=х, точка е принадлежит Касательной ВС. Опять прямоугольная трапеция О1ВЕО3, где из точки О3 на сторону О1В опускаем перпендикуляр О3F. Снова прямоугольный треугольник О3FО1, где О3О1=1+х и О3Е=х, BF=х, О1F=1-х, по теореме Пифагора, О3F= 2корень из х. Теперь рассматриваем подобие прямоугольных треугольников О1FО3 и О2ДО1. О1О2 относится к О1О3 также как О1Д относится к О3F, то есть 5/1+x = 4/2корень из х, в итоге квадратное уравнение имеет вид 4х^2-17x+4=0, Д=225, х1=4, х2=1/4.
2 случай Третья окружность (центр О4) строится с другой стороны от окружности радиуса =1,получаем, что её радиус такой же как у окружности радиуса =4.
Может быть долго и нудно, но я думаю что так. Или где-то снова ошиблась?

845. таша, 27 марта 2010, 10:03:00
Элли, а где же твои задачи? Обещала же написать....

846. MeGusta, 27 марта 2010, 14:22:31
a po istorii zdes net testov?(ili ia, kak obichno ishy nepravilno?)

i eche: vse voprosi po resheniu zadach zadavat zdes? a vi mojete kak-nibyd tak sdelat, chtobi mojno bilo prosmotret pravilnoe reshenie? prosto ia 90% po matematike reshau nepravilno, i mne sprashivat stolko stidno./

847. elli_93, 27 марта 2010, 15:24:06
Таша! Обязательно напишу свои после каникул. Все черновики забрали, а точно не помню условия. Спасибо, что написала свои задачи, я перед пробником повторила окружности, и они попались в моём варианте. Позже напишу условия. Насчёт твоего решения.У меня вызывает большое сомнение подобие прямоугольных треугольников О1FО3 и О2ДО1, на которых основано твоё решение. Объясни.

848. Леонид , 27 марта 2010, 16:30:14
elli_93, таша. (Кузбасс, от 27.03 - пробный, вариант 223)
С1. Система : 16 в ст. кос.х - 10*4 в ст. кос.х +16 =0, 2син.х + корень из у=0.
С2. Точка М - середина ребра АД куба АВСДА1В1С1Д1. Найдите угол между прямыми С1М и В1С.
С3. Неравенство (ЛОГ числа(корень кв. из (7-2х) по осн.2)*(ЛОГ 2 по осн.х) меньше или равно 1.
С4.Две окружности радиусов корень из 5 и корень из 17 имеют общую хорду АВ, длина которой равна 12. Через точку В проведён диамнтр ВМ большей окружности, причём прямая ВМ вторично пересекает меньшую окружность в точке С. Найдите площадь треугольника АСМ. - жду вашего решения этой задачи. (девчёнки посмотрите мое решение С4 - таша, там без особых заморочек).
С5. Найти все значения пар. а, при каждом из которых во мн-ве решений н-ва 21х+ мод.(3х - а) меньше или равно 4а-х нет целых положительных чисел.
С6. Решите уравнение в натуральных числах n кв. + a кв.(от n)=2000, где сигма (от n)- сумма десятичных цифр числа n.

Какой ваш регион?


849. elli_93, 27 марта 2010, 17:36:15
Леонид! уточни в с4 длину хорды.Она не может быть длиннее радиуса. Я из Подмосковья))

850. Леонид , 27 марта 2010, 18:03:22
elli_93, Точно опечатка, АВ = 2. Регион - спасибо, надо вести обмены "С", понимаешь почему?

851. Леонид , 27 марта 2010, 18:16:23
Рассуждения по В6 (таша). f(x) = x^2 +px +q. Пусть больший корень x1-простое и f(5)- простое. f(5)= 25+5p+q - простое. Подбором находим p=-13, q=42. x^2-13x+42=0, x1=7- простое, x2=6. f(5)=25+5(-13)+42=
=25-65+42= 2-простое. Корни уравнения 7 и 6 натуральные числа.

852. Таша, 27 марта 2010, 21:14:17
Леонид! Твои рассуждения мне нравятся у меня такая же пара 7и6, но если корнями уравнения будут ещё одна пара чисел 7и4? р=-11, q= 28. Интересно, а как решать такие задания, если не подбором? Я из Казани. Твои задания посмотрю позднее.
Элли_93! А попробуй начертить с помощью циркуля в реальных единицах, у меня реально радиус третьей окружности 1/4.

853. таша, 27 марта 2010, 21:58:46
elli_93! Хотя ладно, не пробуй чертить там всё таки ближе к половине малого радиуса.
Леонид! С1. косинус х - 10 это в степени числа 16, что то недопоняла.

854. elli_93, 27 марта 2010, 22:21:21
Таша! Не обижайся! Но они, действительно не подобны ни по одному признаку подобия. У треугольников только один угол совпадает-прямой. угол FO3O1=углуO3O1D, а нам для подобия по двум углам надо, чтоб он был равен углу DO1O2(((. ПРи построении у меня получился радиус маленькой окр приблизительно=1/2. что, вобщем-то, недалеко от4/9.

855. Леонид , 27 марта 2010, 22:26:44
Нет. -10 начинается Вх как в кв. ур.таша. А в той задаче радиус 4, думаю.

856. таша, 27 марта 2010, 22:40:08
Леонид! С1 у=2,х=плюс минус П/3+2Пn Ответы такие?

857. Таша, 27 марта 2010, 23:10:47
Леонид! С5
1) х меньше равно а/5
2) х меньше равно 3а/19
значит а принимает все значения от 1 до 4.
Сам то сколько заданий решил?

858. Леонид , 28 марта 2010, 08:15:06
Таша, (запись 856) Да, х такие, но походит ли пи/3 +2 пи К, ведь( корень из у)=-2(коней из 3/2). и ещё, почему у=2?..... корень из у =2(корень из 3/2), у=3.

С6 не брался, С5 не до конца.

859. таша, 28 марта 2010, 10:01:27
у=3 опечатка была

860. Даша, 28 марта 2010, 13:09:28
Задание B3 (12763)

ответ 8 неправильный!
сказано, написать меньший корень уравнения, а 8 > -9

861. Эл, 28 марта 2010, 13:52:45
Задание B9 (5069)

Стороны основания правильной четырехугольной пирамиды равны 10, боковые ребра равны 13. Найдите площадь поверхности этой пирамиды.



Ответ: 340





Задание B9 (5021)

Объем конуса равен 16. Через середину высоты параллельно основанию конуса проведено сечение, которое является основанием меньшего конуса с той же вершиной. Найдите объем меньшего конуса.


Ответ: 2




скажи пожл, как они решаются

862. Леонид , 28 марта 2010, 14:07:12
Эл. Основ квадрат, его площадь 100. Боковые грани равные равноб. треуг.- его высота - апофема 169-25=144, т.е. 12. Площадь грани 1/2*10*12=60, бок. поверх. 60*4 = 240, добавив пл. осн.: 240+100 = 340.

Радиус уменьшили в 2 раза, пл. уменьшится в 4 раза. Уменьшени и высота в 2 раза. Значит объём уменьшится в 8 раз. 16:8 = 2.

863. Леонид , 28 марта 2010, 14:22:00
Даша, а в чём дело? -9 меньший х - не подходит (подставь, арифметическое значение корня 9 =-9) , остаётся 8 - это и есть ответ.

864. elli_93, 28 марта 2010, 15:19:08
Леонид! Начала решать твой вариант. Мне он кажется сложным. с1 получилось у=3, х=4/3пи+2пин,х=-пи/3+2пин. с 2- непонятно, получается прямой угол там? с4 ты решил? тоже у меня загвоздка. остальные не бралась ещё

865. Леонид , 28 марта 2010, 16:56:36
elli_93, С1 -да, С2 (корень из 2)/6, Нет черновиков-попозже. Да вариант сложный. В ноябре и екабре были проще.
В С2. С1К параллельно В1С - треуг. МС1К. С1К = корень из 2.МС1 (сначала МС=корень из 1+1/4=5/4) = 3/2. МК (сложнее, сделай КР парал. ДС и провеи ДР, то треуг МКР, где МК =3/2, КР = 1)=(корень из 13)/4. Т. косинусов
13/4=9/4+2-2*3/2*(корень из 2)КОС. МС1К. Не знаю, понятно ли сотворил? Отвечай.
С3 крученое, хотя не сложно, чере часок поелюсь.

866. elli_93, 28 марта 2010, 18:49:54
Леонид! Решая твой вариант, я поняла, что абсолютная тупица! твоё объяснение поняла только только до того места,где МК начинается. а дальше очень лихо ты закрутил, плохо у меня с пространственным воображением(((. Я немного по-другому решала. Смотри при рараллельном переносе СВ переходит в ДА1, а теперь перенесём её вдоль ДА.Тогда МА2=1/2ДА1. РАССМОТРИМ треугол С1МА2.-равнобедренный он. С1А2=С1М=3/2, МА2=КОРЕНЬ ИЗ 2 ПОПАЛАМ. по теореме кос получаем кос=корень из 2, делённый на 6. Такой же у тебя ответ? но ведь нас просят угол найти ,как ты угол записываешь?через арккосинус? как ответ твой выглядит? с4 тож не могу решить. Дай подсказку, в чём секрет там?

867. Леонид , 28 марта 2010, 19:50:57
elli_93. С2 попробую по другому сказать. Продлил заднюю грань куба в право и С1К параллельно В1С. Продлил в право нижнюю граннь (Р нижняя правая её вершина). Я привык делать "вынесенные" чертежи-мне это очень помогает - в натуральном виде что получилось в нижней плоскости. Виднл хорошо -легко находится МК и МС, далее треуг. МС1К.
Ответ в таких случаях записывается арккос а - это угол.

С3 Решаю по этапно - надёжнее: 1-й этап - ОДЗ: х больше нуля, х не равно 1 - требования к основанию, х меньше 3,5 - подкорнем и под логарифмом.
2-й этап - РЕШЕНИЕ: к первому множителю (ЛОГ) примени формулу перехода к новому основанию (х), второй множитель (ЛОГ х с осн.2) и знаменатель (ЛОГ х с осн.2) сокращаются. Далее важный момент, распадается на два случая:а) осн. от 0 до 1, б) осн. больше 1. Заменив 1 справа как (ЛОГ х по осн. х) с учётом монотонности (случаи а и б) освобождаемя от логарифмов. Далее все просто.
3-й этап- выбор ответе - Это ПЕРЕСЕЧЕНИЕ множеств ОДЗ и решения - ОТВЕТ. Считаю так надёжнее.
Не люблю в одной записи пользовться и фигкрными и [ скобками.

868. elli_93, 28 марта 2010, 20:09:39
Леонид! Поняла с2 как ты делал. ПРосто чертёж по=другому расположила. к себе лицом повернула граньАА1Д1Д. и стало видно. ВСЁ-ТАКИ , в геометрии- сделать правильный чертёж=почти решить задачу.)) а что с4.? мне никак не удаётс найти расстояние от С до ЦЕНТРА окр. без этого не могу найти площадь

869. ксюююююююююю, 28 марта 2010, 20:10:07
мне бы хоть на 3 сдать алгебру

870. боюсь экзаменов, 28 марта 2010, 20:11:54
блин.. люди..подскажите как сдать егэ по алгебре на 3!?

871. ОТВЕТИК, 28 марта 2010, 20:41:58
Выучи ее на 3.5 =)

872. Леонид , 28 марта 2010, 20:47:21
elli_93, красиво сказано "к себе лицом повернула граньАА1Д1Д. и стало видно". Да, чертёж, почти всё! А что о С3 молчишь, я же старался, жаль что здесь плохо печатать. С4 позже. Не забудь свои задачи С - чиркнуть.

Что - то другие молчат. Где -то был Слам. Кто-то бы с Востока откликнулся, Курган, Омск, Иркутск.

873. Леонид , 28 марта 2010, 22:13:07
elli_93, С4. В треугольнике АВМ всё известно, в том числе син.В и кос.А.
1) Треуг. АВС: по формулам площади через радиус описанной окружности= 1/2АВ*ВС*син.В находится АС.
2) треуг. САМ по т. косинусов для АС кв. находим СМ. Станут известыми все стороны. Формула Герона - площадь треуг. Но преобразования и вычисления - чёрт ногу сломит. Может быть ругие идеи буут легче?

874. elli_93, 29 марта 2010, 02:19:06
Леонид! что-то я ступила, не догадалась найти СМ из т косинусов (не зная угол МАС). Тогда всё понятно. И Герон уже не нужен. Утебя же есть СМ, МА и синус В.(половина произведения сторон на синус между ними) Получилось 320/17 у меня. А у тебя?всё равно вычисления жуткие, особенно для СМ. Специально они что ли дают такие числа, чтобы заодно проверить, как мы умеем считать без калькулятора? Если бы мне такой вариант достался, мне просто не хватило бы времени. У меня был легче. Но с5 и с6 нам вообще не давали. После каникул напишу свои(черновики все забрали((. с 3 ещё не смотрела твою. завтра))

875. Леонид , 29 марта 2010, 09:20:20
С3 у меня (0;1) и (1/2 (-1+корень из 13);3)

876. Besnyshka, 29 марта 2010, 15:01:40
а мне тут нравица))помогает очень)

877. вова, 29 марта 2010, 17:15:35
Мне ваще плохо все эти эге УЖАС!!!

878. elli_93, 29 марта 2010, 22:58:41
Леонид!
С3 у меня (0;1) и (-1+2корень из 2);3.5)
с4 я напутала с вычислениями, получилось у меня при пересчёте 64/17. А у тебя?

879. elli_93, 29 марта 2010, 23:00:57
Откуда ты это взял? 1/2 (-1+корень из 13);3) ???
там получается ур-ие x^2+2x-7 или я опять напутала?

880. Леонид , 30 марта 2010, 05:38:40
elli_93, Доброе утро. Что же так в Москве?
Прошу прощения, ты ничего не напутала. Я гононул, ответ другого задания написал. Твой - С3 ответ верный.
Можешь потренироватся, вот оно (Лог. корня из (3-х) с осн.2)*(Лог. 2 с осн. х) меньше или равно 1 - решение аналогично.
С4 - ещё не восстановил.

881. Besnyshka, 30 марта 2010, 07:58:23
приветик)очень трудно даются задания С4,С5 и С6.не могли бы вы помочь?)
не могу решить такую задачку:Окружности рдиусом 2 и4 касаются в точке В.Через т.В проведена прямая, пересекающая второй раз меньшую окружность в т.А, а большую - в т.С. Известно, что АС=3корень из 2.Найти ВС.
это С4

882. таша, 30 марта 2010, 08:40:18
Подобные треугольники ВО1А и СО2В (О1 -радиус менишей окр-ти, О2- большей),пусть ВС=х, АВ = 3корень из 2 - х, тогда О1В/О2В = АВ/О2С, отсюда х=ВС=2корень из 2

883. Леонид , 30 марта 2010, 08:45:24
Besnyshka, и второй случай, окружности касаются внутренним образом - проще отв. 3 корень из 2.
Откуда эта задача? Смотри запись 848 с пробного. Если и твоя с пробного (кога) и напиши регион

884. katya.16ru, 30 марта 2010, 10:13:29
привет.скажите как решаются такие задания.
решить уравнение в натуральных числах mn+25=4m

885. chestime, 30 марта 2010, 11:02:10
Как сделать вот эти задания № 27214, № 27215.
http://mathege.ru:8080/or/ege/ShowProblems?offset=170&posMask=256&showProto=true

886. Таша, 30 марта 2010, 12:18:39
Для Katya.16ru
Я думаю так:
4m-mn=25
m(4-n)=25
методом подбора только числа m=25, n=3

887. katya.16ru, 30 марта 2010, 12:24:20
спасибо)

888. RuS92, 30 марта 2010, 18:41:56
Решите задачу Камень брошен вниз с высоты 8 м.Высота h, на которой находится камень во время падения, зависит от времени t: h(t)=7-6t-t^2.Сколько секунд камень будет падать?

889. Леонид , 30 марта 2010, 20:56:20
elli_93, по моей С4. Результаты по этапам решения.
1) КОС. М=корень из(6/7) - это и Син. АВС. АМ = 2 корня из 6.
2) Из равенства площади треуг.АВС (через радиус описанной и 1/2 произвед. АВ на ВС на СИН. м/у ними), АС= 2 корня из (30/7).
3) т. КОС. в треуг. АМС: Для СМ квадратное уравнение получилось, из которого СМ= (12 корней из 7 + - 4корня из 42)/7.
4) Площадь = 1/2*2корня из 6*(12корней из 7 + - 4корня из 42)/7 * 1/корень из 7(СИН. М).
Ответ: 12/7(корень из 6 +-2) - оба идут, от расположения АВ. Проверь, что не так? Жду твоих задач. Сравни, чтопросила (запись 881) и эта задача.

Как получила 64/17. Что во втором случае?

890. elli_93, 30 марта 2010, 22:15:29
Леонид! Может ты сделал опечатку в условии?Две окружности радиусов корень из 5 и корень из 17 имеют общую хорду АВ, длина которой равна 2. Через точку В проведён диамнтр ВМ большей окружности, причём прямая ВМ вторично пересекает меньшую окружность в точке С. Найдите площадь треугольника АСМ. ПОтому что cos м= 4/корень из 17(АМ по т Пифагора=корень из (2корня из 17)^2-2^2=8. кос М =8/2корня из 17)

891. Леонид , 30 марта 2010, 23:10:50
elli_93,Да по памяти, сегодня увидел текст корень из 5 и корень из 17, в др. варианте корень из 10 и корень из 5. Твоих текстов ещё нет?

892. elli_93, 30 марта 2010, 23:40:59
Леонид! Каникулы до 1 апреля)). Второй случай, когда окружности пересекаются таким образом, чтоСМ=СВ+ВМ. т.е в случае, который мы рассматривали меньшая окружность почти лежит в первой и обе они по одну сторону от АВ. а во втором случае треугольники АВС и АВМ лежат по разные стороны то АВ.
Да, я же говорю, у тебя сложный очень вариант, не сравнить с 881!!! У меня тоже не очень сложный был, не помню на память, как отшибло(((

893. Леонид , 31 марта 2010, 00:00:07
elli_93,благодарю за общение, у нас уже около трёх ночи. С чертежом согласен, решение пересмотрю теперь

894. elli_93, 31 марта 2010, 00:23:29
Леонид! Я поняла, ты брал корень из5 и корень из 7. я прорешала. с твоим ответом (889 сообщение) частично согласна.Ответ: 12/7(корень из 6 +-2) - оба идут, от расположения АВ. я думаю, что с + ответ не подойдёт, тк площадь АСМ окажется больше ВАМ. и что ты имел ввиду "от расположения АВ."?

895. elli_93, 31 марта 2010, 01:37:58
Леонид! Я прорешала второй вариант расположения окружностей и с данными корень из 17 и корень из 5 получилась площадь=280/17. что у тебя?

896. Настя, 31 марта 2010, 08:15:38
у меня такая задачка, помогите решить!:) Брюки дороже рубашки на 20% и дешевле пиджака на 46%.На сколько % рубашка дешевле пиджака? если можно, подробно!!!:) заранее спасибо!!!!

897. Настя, 31 марта 2010, 08:39:43
еще вопросик! задание С2: в кубе АВСDA1D1C1D1 все ребра равны 1. найдите расстояние от точки С до прямой BD1.

898. elli_93, 31 марта 2010, 13:27:58
Настя! Думаю так: Пусть рубашка стоит Х, пиджак У, брюки Z.
Брюки на 20% дороже рубашки, т.е. стоят 120% от стоимости рубашки или
Z=1,2 *X. Эти же брюки дешевле пиджака на 46%, т.е. их стоимость соcтавит 100%-46%=54% от стоимости пиджака, или Z=0,54*Y. Приравняем оотношения для Z, получим 1,2*Х = 0,54*У,откуда Х/У = 0,54/1,2 = 0,45.
Это означает, что стоимость рубашки составляет 45% от стоимости пиджака, или, другими словами, что рубашка на 100%-45%=55% дешевле, чем пиджак.
Может кто попроще объяснит))

899. elli_93, 31 марта 2010, 13:37:50
Настя! Рассмотрим треуг. ВСД1-прямоугольный. ВС=1,ВД1=корень из3,Д1С=кореньиз2. h-высота из С к ВД1.Площадь=1/2ВС*СД1=корень из 2/2. Площадь=1/2*ВД1*h Подставляем в последнюю формулу то.что получили корень из 2/2=(корень из 3/2)*h. отсюда h=корень из (2/3). по-моему так

900. U, 31 марта 2010, 13:40:26
было бы удобней если бы был поиск задач по номерам, если они конечно соответствуют. скажем я решаю задание В3, там несколько видов уравнений, и на одно иррациональное - целых 6 прототипов, я хочу про решать в отдельности все и сверить ответы. что б наверняка. Было бы очень удобно!

901. elli_93, 31 марта 2010, 13:48:26
Настя! ответ можно записать ещё как (корень из 6)/3.

902. Настя, 31 марта 2010, 14:31:30
Во сколько раз увеличится площадь поверхности пирамиды,если все ее ребра увеличить в 2 раза?

903. Леонид , 31 марта 2010, 20:08:02
878. elli_93, 29 марта 2010, 22:58:41.
Леонид!
"с4 я напутала с вычислениями, получилось у меня при пересчёте 64/17. А у тебя?"

Мои ответы: 64/17 и 192/17.


904. elli_93, 31 марта 2010, 21:54:05
Леонид! Я прорешала второй вариант расположения окружностей и с данными корень из 17 и корень из 5 получилась площадь=280/17. как получил 192/17?

905. Леонид , 31 марта 2010, 22:57:56
elli_93. Как бы объяснить без чертежа? Применяя формулы пл. АВС через радиус описанной окр. и полупроизведения сторон на син. м/у ними. В треуг. АСМ - т. кос. и формулу площади - стороны др. через др. не выражаются (нет в этом необхоимости. Син. при вер. В один и тотже углов В и 180 - В. Уравнение для СМ одно и тоже. Правильно ты говоришь (треуг. по одну сторону от АВ и по разные). В первом случае один не подх, в другом - второй. М.б. я что-то не усматриваю. Но как 280/17 не усматриваю.

906. Mrs.AFI, 1 апреля 2010, 17:07:58
В треугольнике АВС угол С равен 90 градусам, угол В равен 60 градусам, АВ=6. Найдите ВС. Помогите решить.

907. Леонид , 1 апреля 2010, 18:18:03
В=60.град. значит угол А = 30 град. ВС - против угла 30, значит половина гипотен. АВ, ВС=3.

908. Polina_7, 2 апреля 2010, 07:47:04
Как решить задачу?С2 Основание прямой четырехугольной призмы АВСДА1В1С1Д1-прямоугольник АВСД,в котором АВ=5,АД=корень из 33.Найдите тангенс угла между плоскостью грани АА1Д1Д призмы и плоскостью,проходящей через середину ребра СД перпендикулярно прямой В1Д,если расстояние прямыми А1С1 и ВД равно корень из 3.

909. Lexxus, 2 апреля 2010, 14:31:33
С2 Основание прямой четырехугольной призмы АВСДА1В1С1Д1-прямоугольник АВСД,в котором АВ=5,АД=корень из 33.Найдите тангенс угла между плоскостью грани АА1Д1Д призмы и плоскостью,проходящей через середину ребра СД перпендикулярно прямой В1Д,если расстояние прямыми А1С1 и ВД равно корень из 3.

Призма прямая, в основании прямоугольник. Значит, она еще и прямоугольный параллелепипед.
Это значит, что расстояние между A1C1 и BD (диагоналями оснований призмы) равно длине боковых ребер - sqrt(3).
Нам нужно найти тангенс угла между боковой гранью AA1D1D и плоскостью, перпендикулярной диагонали B1D параллелепипеда (неважно, что эта плоскость проходит через середину ребра CD - это лишняя информация).

В сущности, нам достаточно найти угол между плоскостью грани AA1D1D и самой диагональю B1D, а искомый угол будет равен (пи - найденный угол).

Поскольку мы имеем дело с прямоугольным параллелепипедом, то этот угол легко найти из прямоугольного треугольника B1DA1. Угол B1DA1 - и есть угол между гранью и диагональю.

Катет А1B1 = AB = 5
Катет A1D (по теореме Пифагора из треугольника AA1D) = sqrt(sqrt(3)^2+sqrt(33)^2) = 6

Значит, КОТАНГЕНС угла B1DA1 равен 6/5.
Ну а нужный нам тангенс (пи - B1DA1) как раз и равен котангенсу B1DA1
(одна из формул приведения: tg(п-а) = ctg(а)).

То есть, ответ - 6/5.

910. gala, 2 апреля 2010, 18:32:03
Помогите разобраться с задачей
Между стартом и финишем горнолыжного спуска круглосуточно действует подвесная канатная дорога. Кабинки сверху и снизу отправляются одновременно каждые 3 минуты. Время движения в одну сторону составляет 14 минут. На старте и финише кабинка стоит 1 минуту, включая режим проскальзывания каната. Определите количество кабинок, двигающихся вниз, которые встречаются горнолыжнику при подъеме.
Спасибо.

911. катя, 3 апреля 2010, 22:17:14
{&#9608;(х+у+&#8730;(хх-уу)=1)@х^3 &#8730;(хх-уу)=0)&#9508;

как решать такую систему?

912. катя, 3 апреля 2010, 22:29:09
что то не очень понятно получилось((
первое уравнение х+у+ ( под квадратным корнем х в квадрате - у в квадрате)=1
второе уравнение х в кубе умножить на ( под квадратным корнем х в квадрате - у в квадрате)=0

913. Леонид , 3 апреля 2010, 22:47:08
2-м ур. х=0, подставив в 1- е - решений нет.
2-м ур. х кв. - у кв. = 0.
Два случая:а) х = у, у+у=1, у = 1/2 и х = 1/2 решение (1/2;12).
б) х=-у, подстави в 1-е уравнение, получаем 0=1 нет реш. Ответ (1/2;1/2)

914. катя, 3 апреля 2010, 23:06:03
спасибо большое)))

915. gala, 4 апреля 2010, 01:55:57
Что же с моей задачей? Говорят, что она может быть на ЕГЭ в В12...Пожалуйста, помогите с решением. Спасибо.

916. inga, 4 апреля 2010, 09:17:35
Как решать задачи В12.

Моторная лодка прошла против течения реки 195 км и вернулась в пункт отправления, затратив на обратный путь на 2 часа меньше. Найдите скорость лодки в неподвижной воде, если скорость течения равна 1 км/ч. Ответ дайте в км/ч.

917. inga, 4 апреля 2010, 09:23:07
Помогите решить.

На верфи инженеры проектируют новый аппарат для погружения на небольшие глубины. Конструкция имеет форму сферы, а значит, дейcтвующая на аппарат выталкивающая (архимедова) сила, выражаемая в ньютонах, будет определяться по формуле: , где — постоянная, r — радиус аппарата в метрах, — плотность воды, а g — ускорение свободного падения (считайте Н/кг). Каков может быть максимальный радиус аппарата, чтобы выталкивающая сила при погружении была не больше, чем 141750 Н? Ответ выразите в метрах.

918. как решить??????, 4 апреля 2010, 14:22:48
sin^2x-2cosx/2=-2
(синус квадрат икс минус два косинуса икс деленного на два равно минус два). ПОМОГИТЕ ПОЖАЛУЙСТА

919. chestime, 4 апреля 2010, 16:45:23
Lexxus, как в здешнем банке задач искать задачи по номеру? Я например прорешал на официальном банке все прототипы в В12. Как мне помочь этому сайту? Т.е. вставить мои ответы (и заодно сверить) сюда на те задачи?

920. Айгуль, 5 апреля 2010, 11:48:23
Как записать ответы к задачам В1 типа: Когда заканчивается урок в начальной школе, т.к. ответы в часах и минутах, не переводимых в десятичную дробь

921. Lexxus, 5 апреля 2010, 12:34:17
sin^2x-2cosx/2=-2
(синус квадрат икс минус два косинуса икс деленного на два равно минус два). ПОМОГИТЕ ПОЖАЛУЙСТА

Что, не решается уравнение четвертой степени?)) Всё гораздо проще.

sin^2(x) может принимать значения от 0 до 1.
2cos(x/2) может принимать значения от -2 до 2.

То есть, sin^2(x)-2cos(x/2) может равняться (-2) только тогда, когда

sin^2(x) = 0 и одновременно 2cos(x/2) = 2.

То есть,

{sin(x) = 0, cos(x/2) = 1}
{x=пи*n, x/2 = 2*пи*n}

Ответ: x = 4*пи*n, n = 0, +/-1, +/-2...

922. Lexxus, 5 апреля 2010, 13:01:45
Lexxus, как в здешнем банке задач искать задачи по номеру? Я например прорешал на официальном банке все прототипы в В12. Как мне помочь этому сайту? Т.е. вставить мои ответы (и заодно сверить) сюда на те задачи?

Я специально сделал невозможным решать конкретные задачи с известным номером - так меньше вероятность всевозможных злоупотреблений и вандализма.

Но если ты запостишь сюда (или выложишь куда-нибудь, с указанием ссылки) свои результаты, то я их с удовольствием вставлю.

923. chestime, 5 апреля 2010, 18:54:29
Здесь указаны номера прототипов, через тире ответы:
26578-32
26579-52
26580-10
26581-10
26582-7
26583-16
26584-8
26585-3
26586-16
26587-11
26588-5
26589-16
26590-20
26591-11
26592-10
26593-13
26594-25
26595-10
26596-20
26597-10
26598-11
26599-10
26600-25
26610-2
27482-10
Прототипы взяты из банка, что-то он сейчас не открывается, ссылку дать не могу... Короче в банке->каталог по заданиям->просмотреть прототипы В12...как-то так, вроде... Не знаю сможешь вставить или нет мои результаты (в моих все-таки решения прототипов, а не задач отдельных), но все равно прошу всех, кто решал, сверить свои результаты с моими. Не совпадает - пишите.

924. даша, 5 апреля 2010, 19:35:01
из пункта А в пункт В выехал мотоциклист и одновременно из В в А выехал автомобилист. мотоциклист прибыл в В через 4 часа после встречи,а автомобилист в А через 1 час после встречи.сколько часов был в пути мотоциклист?

925. toyzy, 5 апреля 2010, 22:46:49
Ребят, прошу, помогите решить задание вот такого типа
sqrt{818^2 - 240^2}

я уже всю голову сломала, а вспомнить как такое решается не могу(

926. Леонид , 5 апреля 2010, 23:11:17
всё под корнем(818-240)(818+240)=всё под корнем578*1058=всё под корнем 2*289*2*529. Извлекаем 2*17*23. Ответ 782. Начинали с разности квадратов по корнем.

927. Razor, 6 апреля 2010, 12:39:38
Задание B1 (21145)

Шариковая ручка стоит 20 рублей. Какое наибольшее число таких ручек можно будет купить на 500 рублей после повышения цены на 10%?
Ответ: 22

Я вот хз, получается 27!!! Докажу 20 умножить на 90 и разделить на 100 получается 18 а потом 500 делим на 18 в итоге 27 с остатком, округляем получается 27, а ответ "22" не будьте пофигистами, помогите, подскажите решение!!!!

928. Razor, 6 апреля 2010, 12:46:03
аааа я втупил цена же повысилась значит надо 20 умножать на 110, и делить на 100 а потом 500 делить на 22 и все фуф блин, а я голову ломал)

929. Lexxus, 6 апреля 2010, 12:46:21
20 умножить на 90 и разделить на 100

... то получится не повышение, а понижение цены на 10% ;)

930. Леонид , 6 апреля 2010, 13:08:51
Lexxus. Будьте добры, хотябы кратенько Ваше решение. Крутили-крутили, берут сомнения. Там с корнями вычисления. Неплохо бы с чертежом, если возможно. Мои ответы:64 корень из 17 и 192 корень их 17.
Пожалуйста.
С4.Две окружности радиусов корень из 5 и корень из 17 имеют общую хорду АВ, длина которой равна 12. Через точку В проведён диамнтр ВМ большей окружности, причём прямая ВМ вторично пересекает меньшую окружность в точке С. Найдите площадь треугольника АСМ.

931. Lexxus, 6 апреля 2010, 14:44:09
С4.Две окружности радиусов корень из 5 и корень из 17 имеют общую хорду АВ, длина которой равна 12. Через точку В проведён диамнтр ВМ большей окружности, причём прямая ВМ вторично пересекает меньшую окружность в точке С. Найдите площадь треугольника АСМ.

Мои ответы:64 корень из 17 и 192 корень их 17.

Что, сразу два ответа получилось? Условие о вторичном пересечении меньшей окружности - как раз для того, чтобы можно было выбрать единственный (скорее всего, меньший из полученных) ответ.

Впрочем, я посмотрю позже, сейчас занят.

932. elli_93, 6 апреля 2010, 15:00:01
Леонид! Точно! Мы невнимательно читали условие! Ведь сказано, что ВМ- диаметр большей окружности, а значит второй случай-не годится. Хотя дальше вроде говорится, что прямая ВМ, а раз прямая- значит можно продолить во все стороны??? Некорректное какое-то условие. если бы говорилось, что именно диаметр большей окр. пересекает меньшую - тогда действительно только один ответ 64/17 . А вот если прямая ВМ- тогда и второй случай подходит(окр по разные стороны от АВ)Площадь получается 192/17. Я в прошлый раз напутала с вычислениями 280/17-это ошибка))

933. toyzy, 6 апреля 2010, 15:00:35
Леонид, огромное спасибо! =)

934. Lexxus, 6 апреля 2010, 15:01:30
Две окружности радиусов корень из 5 и корень из 17 имеют общую хорду АВ, длина которой равна 12.

Эу, стоп. А как окружность радиусом sqrt(5) (чуть больше двух) может иметь хорду длиной 12? Да и у большей окружности радиус маловат, чтобы такую хорду иметь O_o

935. elli_93, 6 апреля 2010, 15:05:48
Lexxus! Это опечатка. хорда =2

936. Lexxus, 6 апреля 2010, 15:08:42
А, уже увидел. Длина хорды - 2. Порешаем.

937. Леонид , 6 апреля 2010, 15:17:47
Lexxus! elli_93. Спасибо, проблема отпала. Нет, в одном 64 /на корень из 17. во втором 192 / корень из 17. Но, что интересно - они дубрируются, выбрарываемые по одному в каждом случае.

938. elli_93, 6 апреля 2010, 15:25:15
не поняла! почему делишь на корень из 17? и что же там в условии? см 932

939. elli_93, 6 апреля 2010, 15:28:21
у меня не было интернэта. сечас напишу свои задания от 24 марта

940. elli_93, 6 апреля 2010, 16:11:14
пробник от 24 03 Моск. обл.
с1 решить систему.(3^x)+2sin y=0 и (4cos^2 y)-(4cos y)-3=0
c2 в правильной треугольной призме АВСА1В1С1 высота =1, а ребро основания=2. Найти расстояние от А1 до прямой ВС1.
с3 решить неравенство дробь, числитель которй 1-корень кв из (1- 8 log в кубе по осн 2 числа х),знаменатель 2 log по осн 2 числа х, <1
с4 две окружности пересекаются в точках А и В. через т.А проведены диаметры АС и АД. найти расстояния между центрами окружностей, если ВС=7, ВД =3
С5 И С6-не было у нас

941. Lexxus, 6 апреля 2010, 16:57:08
С4.Две окружности радиусов корень из 5 и корень из 17 имеют общую хорду АВ, длина которой равна 2. Через точку В проведён диаметр ВМ большей окружности, причём прямая ВМ вторично пересекает меньшую окружность в точке С. Найдите площадь треугольника АСМ.

Я решал через уравнение окружности:
(x-x0)^2+(y-y0)^2 = R^2, где x0,y0 - координаты центра, а R - радиус.

Предположим, центр большой окружности совпадает с началом координат. Тогда её уравнение
x^2+y^2 = 17


Ещё предположим, что точка B лежит на оси абсцисс, то есть её координаты (sqrt(17);0). В треугольнике ABM (он прямоугольный, как и любой треугольник, вписанный в окружность так, что одна из его сторон является её диаметром) находим сначала AM, а потом AH - высоту, опущенную к BM из точки А.
Для этого придется решить некрасивую систему уравнений, из которой станет известна не только длина AH (8/sqrt(17)), но и координаты точки A - (sqrt(17)-2/sqrt(17); 8/sqrt(17)).

Теперь у нас есть координаты двух точек малой окружности, и мы можем вычислить координаты её центра. Для этого составляем систему из двух уравнений окружности, в одно из которых вместо (x,y) подставляем координаты точки B, а во второе - соответственно, A:

(sqrt(17)-x0)^2+(-y0)^2 = 5
(sqrt(17)-2/sqrt(17)-x0)^2+(8/sqrt(17)-y0)^2 = 5

Если мы чудом её решим и не ошибёмся, то получим две пары значений координат центра второй окружности:

1. (8/sqrt(17); 2/sqrt(17)) (на рисунке - красная окружность)
2. (24/sqrt(17); 6/sqrt(17)) (на рисунке - синяя окружность).

Зная координаты центра для каждого случая, находим координату x точки С Она у нас лежит на оси абсцисс, поэтому чтобы это сделать, достаточно в уравнении окружности приравнять y к нулю и решить его относительно x:

1. (x-8/sqrt(17))^2+(2/sqrt(17))^2=5
2. (x-24/sqrt(17))^2+(6/sqrt(17))^2=5

Решая эти два уравнения, получаем по два корня. В обоих случаях один из них будет sqrt(17) (координата точки B), а другой -

1. x = -1/sqrt(17)
2. x = 31/sqrt(17)

Осталось только для обоих случаев найти MC и подставить в формулу площади треугольника MC*AH/2:

1. S = 64/17
2. S = 192/17

942. Леонид , 6 апреля 2010, 17:00:45
elli_93, Большое спасибо. Разобрался с условием. С маху - в С4 варианты АВ между центрами и вне центров. Далее треугольники вписаны - прямоугольные, АВ из них по т. П. и приравнять и т.д.

Да, вариант проще нашего.

943. Леонид , 6 апреля 2010, 17:17:42
Спасибо, Lexxus. Но решение попроще, если на чердеже ориентироваться от прямо через центры и середину отрезка АВ. Там площадь меншего треуг. через раиус описанной окр. и 1/2 сторон на син. м/у ними, Теорема кос. и его пл. через 1/2 сторон и син.м/у ними. Син и кос. легко опрееляются в прямоуг. треуг. АВМ - впис. в окр.

Ваше решение красивое




944. Марина, 6 апреля 2010, 18:20:36
Помогите решить по алгебре к вечеру пожалуйста...с решением пожалуйста

Найти точки минимума функции у=2корень из3 Cosx + 2sinx - 2x +1

заранее спасибо...

945. Леонид , 7 апреля 2010, 18:56:19
всё верно?

946. Света, 7 апреля 2010, 19:55:33
Помогите, пожалуйста, решить

Середина ребра куба со стороной 0,6 является центром шара радиусом 0,3.
Найдите площадь S части поверхности шара, лежащей внутри куба.

947. Иринка, 7 апреля 2010, 19:58:11
А у меня вопрос, почему больше половины заданий не решены? Кто их вообще решает?

948. Марина, 7 апреля 2010, 21:38:05
Ребят пожалуйста решите ..... я уже давала это вчера ..


Помогите решить по алгебре к вечеру пожалуйста...с решением пожалуйста

Найти точки минимума функции у=2корень из 3 Cosx + 2sinx - 2x +1

заранее спасибо...

949. Lexxus, 7 апреля 2010, 22:06:12
Найти точки минимума функции у=2корень из 3 Cosx + 2sinx - 2x +1

Нифига не понятно пишешь. Косинус под корнем, или только 3?

Середина ребра куба со стороной 0,6 является центром шара радиусом 0,3.
Найдите площадь S части поверхности шара, лежащей внутри куба.

Ребро куба является диаметром шара. Значит, внутри куба лежит ровно четверть шара.
Площадь поверхности шара - 4*пи*R^2.
Значит, ответ - пи*R^2 = 0.09*пи.

950. катя, 7 апреля 2010, 22:31:57
в окружности две хорды ав=3, ас=5,причем длина дуги ас в 2 раза больше длины дуги ав. найти радиус окружности.

951. Lexxus, 7 апреля 2010, 23:13:17
в окружности две хорды ав=3, ас=5,причем длина дуги ас в 2 раза больше длины дуги ав. найти радиус окружности.

Длина хорды равна 2R*sin(альфа/2), где R - радиус окружности, а альфа - угол с вершиной в центре окружности, опирающийся на хорду.
Длина дуги равна альфа*R.

Так что тут у нас система из трех уравнений с тремя неизвестными:

2R*sin(бета/2) = 3
2R*sin(гамма/2) = 5
гамма*R = 2*бета*R

(бета и альфа - углы с вершинами в центре окружности, опирающиеся на АВ и АС соответственно)

Ну и решаем:
2R*sin(бета/2) = 3
2R*sin(бета) = 5

Из первого: бета/2 = arcsin(3/2R)
А второе, по формуле синуса двойного угла:
4R*sin(бета/2)*cos(бета/2) = 5

Подставляем:
4R*3/2R*sqrt(1-(3/2R)^2) = 5
Считаем, получаем R = 9/sqrt(11) (9 поделить на корень из 11)

952. Леонид , 8 апреля 2010, 08:21:14
940. elli_93, твой ответ в С4? Мой (0;1) и (1; корень из 2)- на своей ОДЗ. Если всё так, неравенство интересное очевидностями на ОДЗ, оценивая выражения по ходу решения.
Считаю задание логически умненьким от использования монотонности ЛОГ с осн.2, чтобы не решать традиционно 2 системы, установления знака функции, рассмотривая 2 случая - знаков (ЛОГ с осн.2 числа х):от 0 до 1 и больше 1. После этого вывод о числителе.
Иррациональное неравенство - делать выводы, что обе части положительны и возвоить в квадрат. Сравнивать выражение с 1 от х в кубическом уравнении "плохо" решаемым. Решал разными приёмами. Как ты его решала?

953. домашова анна, 8 апреля 2010, 11:45:53
Из пункта А в пункт в выехал мотоциклист и одновременно из В в А выехал автомобилист. Мотоциклист прибыл в В через 3 часа после встречи, а автомобилист в А через 45 минут после встречи. Сколько часов в пути был мотоциклист

954. elli_93, 8 апреля 2010, 12:06:38
Леонид! Ты имел ввиду С3? У МЕНЯ (1; корень кубический из 2), а в с4- рассматриваю средние линии треугольников. там 2 ответа 5 и 2

955. Леонид , 8 апреля 2010, 12:29:07
elli_93. Почему корень куб?Оон уходит с 1/8 и с кубом логарифма. Далее 1/2 =ЛОГ.корня кв.из 2 с осно.2. И - промежутки!
С4 - да. В этих задачах (тип): точка пересечения АВ и линии центров м/у центрами или вне!!!
Как тебе решение моей задачи (красивое, но длиновато - через векторы)?

956. Марина, 8 апреля 2010, 15:07:29
Спасибо что помогли!!!!! Зачем создавать такой сайт, если вы не решаете то, что вас просят помочь!

957. Марина, 8 апреля 2010, 15:09:01
Найти точки минимума функции у=2корень из 3 Cosx + 2sinx - 2x +1

под корнем только 3

958. Lexxus, 8 апреля 2010, 16:32:28
Найти точки минимума функции у=2корень из 3 Cosx + 2sinx - 2x +1

под корнем только 3

Так, значит, у нас функция:
y = 2*sqrt(3)*cos(x)+2*sin(x)-2x+1

Для начала, мы кае-чего с ней сделаем:
y = 4*(cos(x)*sqrt(3)/2+sin(x)*1/2)-2x+1
sqrt(3)/2 и 1/2 - это косинус и синус пи/6 соответственно:
y = 4*(cos(x)*cos(пи/6)+sin(x)*sin(пи/6))-2x+1
В скобках у нас получилась готовая формула косинуса разности:
y = 4*cos(x-пи/6)-2x+1

Вот теперь можно и минимумы искать. Берем производную:
y' = -4*sin(x-пи/6)-2
Приравниваем её к нулю:
-4sin(x-пи/6)=2
sin(x-пи/6)=-1/2

x-пи/6 = ((-1)^n)*(-пи/6)+пи*n
x = пи/6+((-1)^n)*(-пи/6)+пи*n, n = 0,+/-1,+/-2

Это все экстремумы - и минимумы, и максимумы. Теперь прикидываем, как выглядит график производной (похож на "минус синус", только сдвинутый вправо на пи/6) и понимаем, что при четных n у нас будут максимумы, а при нечетных - минимумы.

Для нечетных n выражение для x можно записать так:

x = -2*пи/3+2*пи*k , k = 0,+/-1,+/-2 и т.д.

959. Serdzeedka, 8 апреля 2010, 17:48:59
Пожалуйста,помогите решить задачу!а задачка вот такая:
"Из пункта А в пункт В выехал мотоциклист и одновременно из В в А выехал автомобилист.Мотоциклист прибыл в В через 2 часа после встречи,а автомобилист в А через 30 минут после встречи.Сколько часов был в пути автомобилист?

960. Serdzeedka, 8 апреля 2010, 18:52:04
Люди,пожалуйста прошу выручите меня!!!!!!!!!!решите задачку,которую я написала выше,а то мне завтра от училки капец будет...(((((((((((

961. Радик МС, 8 апреля 2010, 19:10:35
в основании прямой призмы ABCA1B1C1 лежит прямоугольный треугольник ACB (C=90), AC=5, BC=12. Через сторону BC и вершину A1 проведена плоскость; A1BC=60. Найдите площадь боковой поверхности призмы

962. Радик МС, 8 апреля 2010, 19:12:16
Мне нужно до 11 часов. Помогите!!!

963. Марина, 8 апреля 2010, 19:12:20
Огромное спасибо=)

964. Радик МС, 8 апреля 2010, 19:14:54
Просто забыл пару вещей добавить. В основании прямой призмы ABCA1B1C1 лежит прямоугольный треугольник ACB (угол C=90), AC=5, BC=12. Через сторону BC и вершину A1 проведена плоскость; угол A1BC=60. Найдите площадь боковой поверхности призмы. Помогите до 11 часов вечера!!!

965. Lexxus, 8 апреля 2010, 20:17:40
Из пункта А в пункт В выехал мотоциклист и одновременно из В в А выехал автомобилист.Мотоциклист прибыл в В через 2 часа после встречи,а автомобилист в А через 30 минут после встречи.Сколько часов был в пути автомобилист?

Допустим, x - это часть пути, пройденная мотоциклистом (М) до встречи, а заодно и часть его полного времени в пути до момента встречи.
Тогда (1-x) - часть пути (и времени), пройденная автомобилистом (А) до встречи.
За tm и ta обозначим время в пути мотоциклиста и автомобилиста соответственно.

Мы знаем:
tm*x = ta*(1-x) (они выехали одновременно)
tm*(1-x) = 2 (М после встречи ехал еще два часа)
ta*x = 0.5 (А после встречи ехал полчаса)
Система из трех уравнений с тремя неизвестными.

Аккуратно выражаем из второго уравнения tm, из третьего - x, подставляем всё в первое и решаем его относительно ta.

Получаем ta = 3/2 (автомобилист был в дороге полтора часа)

А мотоциклист, кстати - ровно в два раза больше, 3 часа.

966. Lexxus, 8 апреля 2010, 20:43:38
В основании прямой призмы ABCA1B1C1 лежит прямоугольный треугольник ACB (угол C=90), AC=5, BC=12. Через сторону BC и вершину A1 проведена плоскость; угол A1BC=60. Найдите площадь боковой поверхности призмы.

Для начала, найдем AB по теореме Пифагора. Выходит 13.

Дальше едем. Раз призма прямая, а в основании прямоугольный треугольник, то ребро BC перпендикулярно плоскости грани AA1C1C. А значит, и любой прямой, принадлежащей этой плоскости.
То есть BC перпендикулярно A1C, а значит, треугольник BCA1 - прямоугольный.
Его гипотенуза A1B равна BC/cos(60) = 24.

Дальше берем треугольник BAA1, который тоже прямоугольный (раз призма прямая), а значит, AA1 можно найти по теореме упомянутого выше Пифагора:

AA1 = sqrt(24^2-13^2) = sqrt(407)
Некрасивое какое-то число.

Ну, а площадь боковой поверхности будет равна периметру основания, умноженному на высоту:
(5+12+13)*sqrt(407) = 30 корней из 407


967. Serdzeedka, 8 апреля 2010, 22:06:33
Lexxus,спасибо огромное!!!!!!!!!!!!!!очень выручил))))))))

968. elli_93, 8 апреля 2010, 22:08:07
Леонид ,может ты неправильно понял задание?здесь сложно написать именно так, как оно выглядит реально.8 log куб основание 2 числа х выглядит так, что 3 написана прямо над 2, а потом справа х)). поэтому 8 log куб основание 2 числа х я преобразовала как 8/3 log по основанию 2 числа х. в одной части осталось корень из (1-8/3 log по основанию 2 числа х)> 1-2 logпо осн 2 числа х. обе части в квадрат получается log по осн 2 числа х>0 и log по осн 2 х<1/3. отсюда 1<х<корень куб из 2. остальное не подходит по ОДЗ.

969. elli_93, 8 апреля 2010, 23:36:07
Леонид, может это и неправильное решение, результаты пробника ещё не сказали. А ты сколько набрал на пробнике? Решение твоей задачи Lexxusом мне не очень понравилось, оно, безусловно , красиво, но, на мой взгляд, сложновато, и вычисления сложные. На ЕГЭ на это просто не хватит времени.

970. Леонид , 9 апреля 2010, 00:24:04
elli_93, результаты не пришли, но думаю В1 - С4, много времени ушло на С4.
Если бы 3 была над 2 в основании, то было бы так, как ты пишешь (8/3ЛОГ. ...). Куб самого логарифма никуда не вынесишь. Найди ОДЗ: 1-8ЛОГ.куб х по осн. 2 больше-равно 0. -8Лог.куб Х по осн.2 больше- равно
(-1), 8ЛОГ.куб х осн.2 меньше-равно 1.
ЛОГ.куб Х осн.2 меньше-равно 1/8. Без кубов ЛОГ. Х осн. 2 меньше 1/2.
1/2 как ЛОГ. с осн. 2 - это числа 2 в степени 1/2.
Функция возр. (осн. ЛОГ.2 больше 1) и пол-ся х меньше-равно корня кв. из2. Ты бы имела право записать 8/3, если основание в 3 степени, ты же указываешь, что сам ЛОГ. в кубе. И т.д.

971. Радик МС, 9 апреля 2010, 07:03:03
Спасибо!!!

972. Саблезуб, 9 апреля 2010, 07:10:09
В прямом параллелепипеде АВСДА1В1С1Д1 основанием служит ромб со стороной, равной а, угол АВС=120. Через сторону ВС и вершину А1 проведена плоскость, составляющая с плоскостью основания угол 45. Найдите длину бокового ребра и площадь сечения. Срочно!!! Спасайте!!!

973. Lexxus, 9 апреля 2010, 12:03:41
В прямом параллелепипеде АВСДА1В1С1Д1 основанием служит ромб со стороной, равной а, угол АВС=120. Через сторону ВС и вершину А1 проведена плоскость, составляющая с плоскостью основания угол 45. Найдите длину бокового ребра и площадь сечения.



Опустим из точки B перпендикуляры - BH к AD и BH1 к A1D1. Треугольник H1BH - прямоугольный (так как параллелепипед прямой), угол H1BH по условию равен 45 градусов.
Значит, HH1 = BH*tg(45) = BH.
BH найдем из прямоугольного треугольника AHB. Угол ABH = 120-90 = 30 градусов.
Значит, BH = HH1 = боковое ребро = a*cos(30) = a*sqrt(3)/2.

Теперь сечение. Оно у нас параллелограмм, основание BC которого мы знаем, а высота = BH1.
Из треугольника H1BH: BH1 = BH*sqrt(2) = a*sqrt(2)*sqrt(3)/2 = a*sqrt(6)/2

Значит, площадь сечения = BH1*BC = a^2*sqrt(6)/2

Итак, ответ:
Боковое ребро = а * корень из трех пополам
Площадь сечения = а квадрат * корень из шести пополам

974. uvan, 9 апреля 2010, 12:16:03
билет на автобус стоит 15 рублей.какое наибольшее число билетов можно купить на 100 рублей после повышения цены билета на 20%

975. elli_93, 9 апреля 2010, 12:26:36
Lexxus! найти все значения а, при каждом из которых график функции пересекает ось абсцисс более чем в двух точках
f(x)= x^2 -| x^2- 4x -5 |-a


976. elli_93, 9 апреля 2010, 12:31:42
Леонид! Спасиб, что надоумил! там действительно логарифм в кубе, а не основание! это я перепутала все свойства логарифмов(((. НО тогда кубическое ур0ие не решается у меня, а решение неравенства- область определения.

977. Леонид , 9 апреля 2010, 13:32:20
elli_93, Понял, по позже поелюсь. К степенеи логарифма некоторый совет в осторожности. Пример (ЛОГ. в ст. 3 (числа х в кв.) осн.2). выносим не кв., а 2 в кубе. - часто на этм рузья влетают. Лучше записать (ЛОГ. х кв. с осн. 2) всё это в кубе, селай запись в скобках (2ЛОГ. х с осн.2) всё в кубе =
= 2 в кубе* (ЛОГ. х с осн.2) в кубе = 8(ЛОГ.Х с осн. 2) в кубе.

978. Лика, 9 апреля 2010, 18:57:25

Задача С2 вариант 101 башкирия. Помогите пожалуйста срочно решить

В основании четырехугольной пирамиды SАВСД лежит квадрат АВСД со стороной (3умножить на корень из 10):5Длина всех боковых ребер 3 точка М-середина ребра AS.Через прямую ВМ паралельную диагонали АС проведена плоскость.Определите величину угла между этой плоскостью и SAC.

979. Леонид , 9 апреля 2010, 22:00:44
Лика. В плоскости АSC проводи МК параллельно АС (К - сереина SC). Плоскость ВМК и бует параллельна АС по признаку параллельности пр. и пл.
Далее ВТ (Т - сереина МК) перпендикуляр ВК (треуг. ВМК равнобедренный). ТО (О - центр основания пирамиы) - высота трапеции АМКС, т.е. угол ВТО - линейный угол, величину которого и просят найти. Это в треуг.ВТО, у которого ВО половина диагонали основания (ВД по т. Пиф. в треуг. АВД) , ТО - половина высоты пирамиы (SO в треуг. SOC по т. Пиф.). Танген искомого угла отношение ВО к ТО. Решение предложено сходу без листа. Пробуй. Кстати, ВО после вычисления получается равной ТО, т.е. угол 45 град.
Пиши С1, С3, С4. Обязательно. Твоя задача отличается от задач других регионов.

980. Леонид , 9 апреля 2010, 22:12:47
Ребята клавиша Д борохлит, имейте ввиду.

981. Леонид , 9 апреля 2010, 22:32:00
973. Lexxus, 9 апреля 2010, 12:03:41. Высоту ромба основания скорее найти из произведения сторон на минус угла между ними = произведению основания на высоту.

982. elli_93, 10 апреля 2010, 00:46:30
Леонид! Перерешала своё задание с3 решить неравенство дробь, числитель которй 1-корень кв из (1- 8 log в кубе по осн 2 числа х),знаменатель 2 log по осн 2 числа х, <1 с твоими рекомендациями. После преобразований получилось log x(log x -1/2)(log x+1)<0 (все логар по осн.2) И тогда выплывает ещё 1/2. ответ(0,1/2)(1,корень из 2). как думаешь?
у тебя был ответ (0;1) и (1; корень из 2)

983. Леонид , 10 апреля 2010, 02:20:03
elli_93, было некогда довести до конца, там получается кубическое уравнение как ты писала. Посмотрю завтра и сообщу. Ты видимо его разложила на множители. что-то, так просто читая, 1/2 берёт сомнение, но не утверждаю.
Два случая рассматривала? Перенести сначала, к общему зн. Знаменатель прос: либо плюс, лобо минус , причём в соответствии с ОДЗ. Ну ладно,до завтра. У нас уже 5 утра.

984. Леонид , 10 апреля 2010, 14:58:10
982. elli_93. log x(log x -1/2)(log x+1)<0 - раскрой скобки


У меня что-то с комп. или вредители - тунеягодки напакостили, которым нет желания развивать мозки. Ответте, кто-нибудь.

985. elli_93, 10 апреля 2010, 15:17:40
У меня тоже что-то не так с компом. Зачем раскрывать скобки?

986. elli_93, 10 апреля 2010, 16:15:13
Леонид! Я специально разложила на множители, чтоб решить методом интервалов. НА прямой отметила точки ОДЗ и "пробные" точки из приведённого неравенства. это получились точки 0, 1/2,1 и корень из 2.

987. Леонид , 10 апреля 2010, 18:20:47
989. elli_93,ПРоверить себя раскрытием скобок. У тебя нормально в компе здесь на странице. И LEXXUS по поводу пакостей.

988. Di, 10 апреля 2010, 18:43:52
Проверьте пожалуйста Задание B2 (1995) и Задание B2 (1971).
У меня получаются ответы 6 и 10 соответственно...или я ошибаюсь?

989. Леонид , 11 апреля 2010, 09:57:20
989. elli_93, 10 апреля 2010, 16:15:13
"Леонид! Я специально разложила на множители, чтоб решить методом интервалов. НА прямой отметила точки ОДЗ и "пробные" точки из приведённого неравенства. это получились точки 0, 1/2,1 и корень из 2."

Как умудрилась? Пусть 2ЛОГ. х осн.2 = У большем 0, то Укуб+Укв-1 Б. 0. С помощью производной - освой приём, часто помогает. У штр.=3Укв.+2У. Метод интервалов берём У больше 0 - по ОДЗ и производная здесь положительня, наша ф-я возрастает. Выразим У из нашего обозначения Х=2 в ст.(У/2). Используем ОДЗ 1меньше Х меньше-равно корень из Х, подставим сюда Х=2в ст.(У/2). Все части к осн. 2, показ ф-я возраст.,
получим 0 меньше У меньше - равно 1 (можно было сразу селать вывод по ОДЗ. Ф-я знака на промежутке не меняет. Поэтмо я и далтакой ответ.


LEXXUS, убери пожплуйста хулиганство, ликвидируй их регистрацию и заблокируй, да и плату ещё с них возьми.

990. Леонид , 11 апреля 2010, 18:12:03
Самое интересное, что вы подчеркнули удобной систему оплаты за ваши никому ненужные решения каких-то вариантов. Вариантов - добытых ещё в январе, которых нет до сих пор. Идёт толька обкатка заданий пробником и мониторингом, чистится банк задиний, ибо там есть ещё много недочётов. Интересно, кто клюнет на вашу наживку и сколько вам удастся собрать "копейков".


Эй, ты, под № 1002 Lёxxus такой охрененной безграмотностью не страдает.

991. chestime, 11 апреля 2010, 20:51:30
посмотрите здесь: http://mathege.ru:8080/or/ege/ShowProblems?posMask=128&showProto=true
На № 27487 получилось у меня 4. Это правильно?

992. Леонид , 11 апреля 2010, 21:02:56
ДА

993. Lexxus, 12 апреля 2010, 13:52:53
С этим нельзя покончит?

С этим так или иначе придется покончить. Я, кажется, весь мусор вычистил, но он обязательно появится снова, если ничего не сделать ((

994. Леонид , 12 апреля 2010, 14:54:36
998. Lexxus. МОЛОДЦОМ, а то невозможно было общаться с толковыми ребятами. Надо сделать, надеемся всё сможешь, чтобы не появилось!

995. Леонид , 12 апреля 2010, 15:04:00
988. Di, 10 апреля 2010, 18:43:52
Проверьте пожалуйста Задание B2 (1995) и Задание B2 (1971).
У меня получаются ответы 6 и 10 соответственно...или я ошибаюсь?

Таких номеров нет в банке заданий. Откуда их вытащил?

996. Олег, 12 апреля 2010, 17:28:18
пОМОГИТЕ РЕШИТЬ!!!
нАЙДИТЕ ЗНАЧЕНИЕ ВЫРАЖЕНИЯ:
1) 2sin 30 - cos 150 + tg 120
2) sin(-(pi/4)) + tg(pi/6) - cos(-(pi/3))

Упростите выражение
cos^4*a( a - это альфа) - cos^2*a + sin^2*a

Докажите тождество
-ctg(-a) - (sin(-a)/1+cos(-a)) = 1/sin*a

Упростите:
sin((pi/2) + (2a)) + cos((3pi/2) + (2a)) * tg(pi + a)

Помогите!!!

997. Леонид , 12 апреля 2010, 20:16:21
1001. Олег. Эти все задания на формулы привеения (90 и 270 функция меняется, 180 и 360 не меняется) - выделяй их. И знач. триг.функций углов(табличка). Зедесь сложно и долго печатать - а примеры очень простые.
90 это пи/2, 180 это пи, 270 это 3пи/2, 360 это 2пи.

Олег, у тебя какой регион?. Напиши свои задани С с пробного.

998. ОЛЕГ, 12 апреля 2010, 20:26:58
Леонид Кемеровская область

999. Леонид , 12 апреля 2010, 23:08:52
Олежка, земляк! Даже не знаю как тебе с тригонометрией помочь? А хочется земляка поддержать. Давай какой-нибудь посложнее для тебя, попробую написать.

1000. Ольгуня, 13 апреля 2010, 07:10:54
помогите пожалуйста решить с3. log(4+7x-2x в квадрате) по основанию /x+2/ все это меньше или равно 2.
я решила это выражение у меня получились промежутки: 1) случае..(минус бесконечности до "о"), а 2)случае..(от "1" до плюс бесконечности),НО не один из этих промежутков не попадает в ОДЗ..=((((ПОмогите пожалуйста, заранее спасибо)))

1001. Олег, 13 апреля 2010, 07:53:18
Ну сделай мне хотя бы "Докажите тождество" и Упростите!!! Помоги мне пожалуйста!!! Спасай!!!

1002. Олег, 13 апреля 2010, 07:56:18
Нам учительница выдала листки и сказала, ч то это с ЕГЭ.

1003. sima4ok, 13 апреля 2010, 08:54:46
в задачах В8 и одной из зада4 В6 есть опечатки где написано что найти точку эксремума, а их 7!и написано что найти площадь трапеции, а изображён палаллелограм

1004. Леонид , 13 апреля 2010, 10:29:14
1008. sima4ok, - конкретные номера какие? И внимательно читал, график чего в задании (функции или производной функции)-В8? По В6 - выше уже была полемика- считай площадь параллелограммы (идёт и формула трапеции - полусумма оснований трапеции и будет одно основание параллелограмма).

ОЛЕГ. Докажите тождество
-ctg(-a) - (sin(-a)/1+cos(-a)) = 1/sin*a, используя чётность, запишем ctg a +(sin a/1+cos a)= cos a/sin a + sin a/1+cos a = к общ. знам. = (cos a +cos ^2 a + sin^2 a)/sin a(1+cos a) =(cos a+1)/sin a(1+coss a) = 1/sin a -но это задание не из банка ЕГЭ.

Упростите:
sin((pi/2) + (2a)) + cos((3pi/2) + (2a)) * tg(pi + a). Поформулам приведения
cos2a+cos2a*tg a и далее. Здесь в задании всё верно? Зачем их решаете, выберите в банке ЕГЭ и решайте - руку набивайте, там хорошие задания В7.

1005. chestime, 13 апреля 2010, 17:08:47
Сегодня был пробный по математике... Задачи жесть... В части "В" мало заданий с банка задач. Это наверное не Москва составляла, а регион, потому что задания ну совсем разные... Часть С тоже. В С1 какое-то странное уравнение с корнями. Такого в С1 вообще ни разу не видел. В С2 тоже сложно. Я только С3 сделал. Более менее уверен, что правильно. И С6 просто попробовал. Вроде тоже нормально. За С6 может быть что-то дадут.

1006. chestime, 13 апреля 2010, 17:11:16
Олег, это не с ЕГЭ.

1007. chestime, 13 апреля 2010, 17:14:12
Параллелограмм - частный случай трапеции.

1008. Юля, 13 апреля 2010, 17:17:36
Где мне найти полные работы для распечатки, чтобы на уроках заниматься пришлите хоть ссылку.

1009. Рудик, 13 апреля 2010, 19:25:54
cos^4a - 1 нельзя дальше разложить?

Найти значение выражения:
1) 2sin 30 - cos 150 + tg 120
2) sin(-(pi/4)) + tg(pi/6) - cos(-(pi/3))

Упростите выражение
cos^4*a( a - это альфа) - cos^2*a + sin^2*a - что-то я их не могу решить, все время мне говорят об ошибке.

Найти значение выражения:
1) получилось 1/корень из 3
2) (-(((крень из 6) + 2) / (2*корень 3))

А в упростите - вообще путанница!!! Помогите!!!

1010. екатерина, 13 апреля 2010, 20:03:07
кто-нибудь может сказать на реальном егэ будут такие же бредовые задачи абсолютно непонятного происхождения или же как к каким готовились весь год из банка задач?

1011. Леонид , 13 апреля 2010, 20:45:44
cos^2a-sin^2a-1=-(1-cos^2a)-sin^2a=-sin^2a-sin^2a=-2sin^a. Рудник выучи формулы триг. ф-й двойного угла и правило формул приведения и всё получится. Ребята, зачем вы гоните сюда свои Д/З? Почему не решаете из банка ЕГЭ туже тригокометрию В7? Там хорошие задания, на эти же формулы. Скачайте их своей учительнице и занимайтесь делом, готовьтесь к ЕГЭ, вы что оттуда всё умеете.

1)1+ cos30-tg60=1+1/2 корней из 3 - корень из 3= 1-корень из 3.

2)-sin45+tg30-cos60= -1/2 корень из 2 +1/3 корень из 3 - 1/2= Да, пустая трата времни.

1012. валя, 14 апреля 2010, 17:37:05
Товарищи!!!! Зачем решать весь банк заданий!!!!!! надо решать только прототипы задач. Это в разы меньше. Давайте решать только протипы в том же банке задач. Облегчим себе жизнь!!!!!!

1013. chestime, 14 апреля 2010, 19:57:15
Екатерина, у меня точно такой же вопрос... Только к кому? Кто на него сможет ответить?

1014. Krived, 14 апреля 2010, 22:26:12
В треугольнике ABC угол C равен 90градусов, AB=5, cos A=0,8 Найдите BC

1015. Леонид , 14 апреля 2010, 23:51:07
1014. Krived. По определению косинуса: 0,8 = АС:АВ, 0,8=АС:5, АС= 4, тогда ВС=3 -либо по т. Пифагора, или, если знаешь египетский треугольник:3,4,5.

Девочки с пртотипами больше запутаетесь - это (прототип) как формула типов задач.

1016. elli_93, 15 апреля 2010, 10:52:54
Леонид! я всё со своим заданием с3. Ты писал, что не усматриваешь 1/2,но если подставить её в первоначальное неравенство, не получается оно верным. Посмотри его, пожалуйста, ещё разок.)) на пробнике потеряла на нём 2 балла, остальные-правильно решила)) Как твои результат?
см(982. elli_93, 10 апреля 2010, 00:46:30 Леонид! Перерешала своё задание с3 решить неравенство дробь, числитель которй 1-корень кв из (1- 8 log в кубе по осн 2 числа х),знаменатель 2 log по осн 2 числа х, <1 с твоими рекомендациями. После преобразований получилось log x(log x -1/2)(log x+1)<0 (все логар по осн.2) И тогда выплывает ещё 1/2. ответ(0,1/2)(1,корень из 2). как думаешь?
у тебя был ответ (0;1) и (1; корень из 2) )

1017. Лена, 15 апреля 2010, 11:39:30
Помогите решить пож!!B4 В треугольнике ABC AB = BC, AC = 10, высота CH равна 5. Найдите синус
угла ACB.


1018. Елена, 15 апреля 2010, 11:59:05
Треуг АВС равнобед, уголА=углуС. Значит и синусы этих углов равны.Найдем синус углаА:НС разделить наАС. Получим 0,5.Вот и все

1019. дима, 15 апреля 2010, 11:59:06
Объем шара равен 36 pi . Найдите площадь его поверхности, деленную на pi .

1020. Лена, 15 апреля 2010, 12:34:06
большое спасибо...!моя невнимательность....=(

1021. дима, 15 апреля 2010, 13:04:26
Два ребра прямоугольного параллелепипеда, выходящие из одной вершины, равны 3 и 4. Площадь поверхности этого параллелепипеда равна 52. Найдите третье ребро, выходящее из той же вершины. Ответ:2

1022. Леонид , 15 апреля 2010, 15:55:52
1016. elli_93! Прости, я решил твоё неравенство (записал задание) в числителе только корень из(1 - ЛОГ. куб х осн.2) и думаю, ну в чём же дело.
Решение - ответ: (ты правильно решила) (0;1) (1;1/2)(1/2; корень из 2).
Ну, да, ладно - потренируйся на моём варианте, там выбор ответа интересный. И закрепишь мои советы, ведь вполне может быть такое задание или чем-то похожее. Не внимательно уловил условие сначала, нет бы перечитать. Ну пока! А, а что было в вариантах других?


КЛАСС уважаемый друг, только в В7 заданий более 2000...............

1023. брюнетка, 15 апреля 2010, 16:04:51
объясните плиз одно из следующих заданий, из части В8


Задание B8 (№ 27490) На рисунке изображен график функции , определенной на интервале . Найдите сумму точек экстремума функции .


1024. Леонид , 15 апреля 2010, 17:56:23
Точки экстремумов функции по ГРАФИКУ ФУНКЦИИ - это абсциссы (х) вершин графика 1,2,4,7,9,11 - сложи их. В этих точках производная равна 0 (посмотри не пропустил ли вершики крафика, а то ещё раз переходить долго).

1025. blac381, 15 апреля 2010, 22:20:02
Дурацкий сайт. Задания вообще легкотня. Даже скучно. На мой взгляд расчианы на 9-ти классников двоечников. Админ пожалуйста добавь номальные и интересные задачки.

1026. Леонид , 16 апреля 2010, 01:07:50
1026. blac381! МОЛОДЕЦ! Хвост пистолетом "доржишь"! Только, почему сайт то дурацкий? Всё знаешь и умеешь, скучно, не заходи. Вот, мне думается, что ты в 80% зааний допустишь ошибки в В8 на графиках производной и функции. Как пить дать.

1027. вася, 16 апреля 2010, 11:29:12
Объясните как делать ? Б9 27186.

1028. Леонид , 16 апреля 2010, 11:55:47
Ты, друг, Васька,извини,следующий раз пиши задание -а то долго прыгать туда сюда. Объём шара 4/3 пи Rкуб = 36 пи, R куб=27 (всё разделили на 4/3 пи), то R= 3. Поверхност 4 пи R кв. = 4 пи*9 (да разделим на пи)= 36. Всё, дружище, понял. Василий, напиши свои С с пробного в марте.

1029. Алексей, 16 апреля 2010, 12:38:45
Здравствуйте, меня смутила на вид элементарная задачка B3 (10093). найти корень уравнения (5/6)x=-20(5/6) я дал ответ -20, но правильный 25. объясните почему, а то я что то туплю)

1030. Алексей, 16 апреля 2010, 12:39:33
Ошибочка, правильный ответ -25

1031. Lexxus, 16 апреля 2010, 12:51:25
Здравствуйте, меня смутила на вид элементарная задачка B3 (10093). найти корень уравнения (5/6)x=-20(5/6) я дал ответ -20, но правильный 25. объясните почему, а то я что то туплю)

Потому что "двадцать целых пять шестых" и "двадцать умножить на пять шестых" - это не одно и то же.

1032. Алексей, 16 апреля 2010, 12:59:06
оу, точно) мда, перезанимался) Но все же спасибо за помощь)

1033. дима, 16 апреля 2010, 13:54:12
Два ребра прямоугольного параллелепипеда, выходящие из одной вершины, равны 2 и 6. Объем параллелепипеда равен 48. Найдите третье ребро параллелепипеда, выходящее из той же вершины.Ответ:1,5 проверти пожалуйста

1034. Леонид , 16 апреля 2010, 16:46:41
1034. Димыч. Извини, но ай-яй-яй !!!! Объём прямоуголгьного параллелепипеда - произведение длины на ширину и на высоту - это как раз 3 ребра из одной вершины. То есть а*в*х = 48. 2*6*х =48, 12*х = 48, х=4.
Откуда такая заача? О в честь чего ответ 1,5. Или ты что-то не то написал.

1035. дима, 16 апреля 2010, 18:59:46
Ещё один вопрос.Три ребра прямоугольного параллелепипеда, выходящие из одной вершины, равны 4, 6, 9. Найдите ребро равновеликого ему куба.Объясните

1036. Леонид , 16 апреля 2010, 21:10:27
Димка, где 100 грамм и пирожок за пред. задачу? Или когда будет, паря? Объём параллелепипеда твоего опять-таки 4*6*9, сколько будет? Правильно, Дима, 216!!! Куб равновеликий ему, значит такого же объёма. Объём куба равен ребру в кубе: а^3 = 216. а = 6, т.к. 6*6*6=216. Дима!!1 - сто грамм и порожок. Не забудь.

1037. Юля , 17 апреля 2010, 12:12:35
Помогите решить пожалуйста х в кубе минус 4 умноженное на х квадрат плюс х плюс 6 =0

1038. Леночка)), 17 апреля 2010, 16:46:16
chestime,я решила половину прототипов В12...и с твоими ответами пока сходиться!а вот я застряла на прототипе №26592...не объяснишь как решал???твои ответы действительно мне очень помогли))))быть может ты и прототипы других номеров решаешь???если да,то хотела бы свериться)))заранее огромное спасибо))))

1039. Rika, 17 апреля 2010, 20:17:35
Мда... Странно, что народ не справляется с В-частью. Или это просто нас на "10 математиках" в неделю так натаскали. В общем, у меня вопрос по С5 будет))) Уж очень хочется решить.
Итак, собственно говоря:
Найдите все значения параметра А, при которых уравнение cos sqrt(A^2-X^2) = 1
имеет ровно 10 решений.
Меня очень смущает именно вот это 10.

1040. Леонид , 17 апреля 2010, 22:05:46
Почему 10 смущает? В тригонометрических уравнениях БМК - выбирай. Чего народ упрекаешь, если у тебя 10ч математики? Откуда вытащила это задание? Каков результатик на пробнике?

1041. Леночка)), 18 апреля 2010, 11:07:41
Помогите пожалуйста решить....всегда решала подобные задачи..и считала их элементарными,ведь они действительно легкие...а тут застряла..может где-то в решении запуталась(((

Баржа в 10:00 вышла из пункта А в пункт В, расположенный в 15 км от А. Пробыв в пункте В 1 час 20 минут, баржа отправилась назад и вернулась в пункт А в 16:00. Определите (в км/час) скорость течения реки, если известно, что собственная скорость баржи равна 7 км/ч.

1042. Гриха, 18 апреля 2010, 14:45:59
Нужно решение.1)Боковые ребра треугольной пирамиды взаимно перпендикулярны, каждое из них равно 3. Найдите объем пирамиды.

1043. Rika, 18 апреля 2010, 16:58:23
Леонид, я-то не упрекаю. У нас тоже куча вопросов вначале возникала. И про расположение крана в баке, и про экстремумы и прочая, прочая.
Задание из книги по подготовке.
А пробник через неделю. Полноценные четыре часа вместо двух, даваемых ранее. За два часа, если не туплю, делаю B1 - C3-4

1044. Леонид , 18 апреля 2010, 17:20:36
1044. Rika. Это всё ладно,что решаешь хорошо, молодец. Но почему из каких-то изаний, непонятно. У нас такого нет. Напиши свои С. Не поленись, пожалуйста.

1045. муся, 18 апреля 2010, 19:51:23
решите пожалуйста.Лыжник пробежал 1-ую половину пути со скор-ю 14 км/ч,а 2-ую -18 км/ч.Какова средняя скорость на дистанции?


1046. муся, 18 апреля 2010, 20:19:18
Cкороcть колеблющегоcя на пружине груза меняетcя по закону v(t) = 3sinfrac{pi t}{4} (cм/c), где t — время в cекундах. Какую долю времени из первой cекунды cкороcть движения превышала 1,5 cм/c? Ответ выразите деcятичной дробью, еcли нужно, округлите до cотых.

1047. Алексей, 19 апреля 2010, 09:59:08
в правильной треугольной призме ABCA1B1C1 Высота равна 1, а ребро основания равно 2.Найдите расстояние от точки A1 до прямой BC1

1048. Алексей, 19 апреля 2010, 10:00:05
Пожалуйста помогите, решите...

1049. Lexxus, 19 апреля 2010, 10:47:06
в правильной треугольной призме ABCA1B1C1 Высота равна 1, а ребро основания равно 2.Найдите расстояние от точки A1 до прямой BC1

Рассмотрим треугольник A1BC1.
A1C1 по условию равно 2, A1B и С1B находим по теореме Пифагора (поскольку боковые грани правильной призмы - прямоугольники):
sqrt(1^2+2^2) = sqrt(5).

Нам нужно найти расстояние от A1 до BC1, то есть длину высоты A1H, опущенной из A1 к BC1. Все стороны треугольника A1BC1 нам известны, поэтому можно найти угол C1BA1 (точнее, его косинус) по теореме косинусов.
Получится 0.6.
А синус этого угла (именно он нам сейчас понадобится) по основному тригонометрическому тождеству будет равен sqrt(1-0.6^2) = 0.8.
Дальше из прямоугольного треугольника BA1H:
A1H = A1B*sin(найденного угла) = sqrt(5)*0.8

Или, что то же самое, 4 поделить на корень из пяти.

1050. Azer, 19 апреля 2010, 18:11:06
В12. Теплоход проходит по течению реки до пункта назначения 234 км и после стоянки возвращается в пункт отправления.Найдите скорость течения реки,если скорость теплохода в неподвижной воде равна 22км.ч,стоянка длится 4 часа и в пункт отправления он возвращается через 26 часов после отплытия из него.

Помогите, плиз!!!!!!!!!!!!!

1051. Katerinka!!, 19 апреля 2010, 19:37:37
Здраствуйте!!!Помогите решить пожалуйста задачу «Решить задачу В треугольнике АВС проведены биссектрисы АД и ВК.Найти длину отрезка КДбАВ=10бАК=5,DL=10/3»

1052. Ромка!!! =), 19 апреля 2010, 20:47:38
Не могу доделать:
2cos(x/4) - корень из 3 = 0 решить уравнение.

Решить уравнение:

(1 - 3 cosX)(1 + 5cosX) = 0

cos5x*cos3x + sin5x*sin3x = -1

Просто реально не могу сделать где-то брожу до правильного ответа, а путаюсь. Очень сильно помогите!!!

1053. Алексей, 20 апреля 2010, 11:30:27
Lexxus,
Рассмотрим треугольник A1BC1.
A1C1 по условию равно 2, A1B и С1B находим по теореме Пифагора (поскольку боковые грани правильной призмы - прямоугольники):
sqrt(1^2+2^2) = sqrt(5).

Нам нужно найти расстояние от A1 до BC1, то есть длину высоты A1H, опущенной из A1 к BC1. Все стороны треугольника A1BC1 нам известны, поэтому можно найти угол C1BA1 (точнее, его косинус) по теореме косинусов.
Получится 0.6.
А синус этого угла (именно он нам сейчас понадобится) по основному тригонометрическому тождеству будет равен sqrt(1-0.6^2) = 0.8.
Дальше из прямоугольного треугольника BA1H:
A1H = A1B*sin(найденного угла) = sqrt(5)*0.8

Или, что то же самое, 4 поделить на корень из пяти.


Напиши пожалуйста ток решение)))))
Я в геометри проктически не гу-гу....

1054. Леонид , 20 апреля 2010, 12:45:55
Lexxus, ответьте пожалуйста - это новое появившееся, полезное? Стоит ли туда заходить - деньги просят?

"Решение задач по любым предметам в сжатые сроки. 1 контрольная от 500 руб.!
kursovaja.ru
Тесты ЕГЭ
Спецификации ЕГЭ 2010 г. Тесты, проверка, консультации. Бесплатные варианты
college.ru" - ПРОСЯТ ДАЖЕ РЕГИСТРАЦИЮ?

1055. Lexxus, 20 апреля 2010, 14:39:21
Lexxus, ответьте пожалуйста - это новое появившееся, полезное? Стоит ли туда заходить - деньги просят?

Это просто рекламный блок. Я понятия не имею, какие там объявления показываются разным пользователям (это зависит от того, что они недавно искали на Яндексе).
Но раз люди заказывают рекламу в Яндекс.Директе, то они предлагают платные услуги. А уж какого они качества - это совсем другой вопрос.

1056. диман, 20 апреля 2010, 18:34:34
Распишите пожалуйста решение.В правильной четырехугольной пирамиде высота равна 6, боковое ребро равно 10. Найдите ее объем.

1057. Lexxus, 20 апреля 2010, 19:30:15
Распишите пожалуйста решение.В правильной четырехугольной пирамиде высота равна 6, боковое ребро равно 10. Найдите ее объем.

Так. Пусть у нас пирамида ABCDE, ABCD - основание (квадрат, кстати).
EH - высота. Поскольку пирамида правильная точка H находится точно в центре квадрата.
Глядим на треугольник AEH. Он у нас, само собой, прямоугольный, AE = 10, EH = 6.
По теореме Пифагора находим оставшийся катет AH = sqrt(10^2-6^2) = 8.
Как мы уже поняли, точка H делит диагонали квадрата пополам. Поэтому диагональ равна 8*2 = 16.
Площадь квадрата (который тоже ромб) равна половине произведения диагоналей = (16^2)/2 = 128.

Объем пирамиды равен трети произведения площади основания на высоту = 6*128/3 = 256.

1058. Вася, 20 апреля 2010, 19:35:53
Помогите решить=))))))))
1-sqrt(3)/2

1059. Мария, 20 апреля 2010, 20:00:34
Один мастер может выполнить заказ за 3 часа, другой-за 6 часов. За сколько часов выполнят заказ оба мастера, работая вместе? Пожалуйста,напишите алгоритм решения. Заранее спасибо.

1060. SANCHES, 20 апреля 2010, 21:12:50
пусть х - то что нужно найти. 1 - вся работа. 1/6- выполняет один за час. 1/3-другой. (1/6+1/3)*х=1

1061. SANCHES, 20 апреля 2010, 22:44:50
Расстояния между тремя точками сферы равны 6,8 и 10, а расстояние от проходящей через них плоскости до центра сферы равно 12. найдите диаметр сферы.
решил. получилось не целое число,но если округлить то с ответом сходится! разве так бывает это то в задании В9 ???
подскажите пожалуйста

1062. Lexxus, 20 апреля 2010, 23:20:08
Расстояния между тремя точками сферы равны 6,8 и 10, а расстояние от проходящей через них плоскости до центра сферы равно 12. найдите диаметр сферы.
решил. получилось не целое число,но если округлить то с ответом сходится! разве так бывает это то в задании В9 ???
подскажите пожалуйста

Я не видел заданий такого типа (откуда оно, кстати?), но ты решил неправильно. В ответе ровно 26 должно получиться.

Треугольник (скажем, ABC) со сторонами 6,8 и 10 (скажем, АВ = 10) - прямоугольный (6^2+8^2 = 10^2).
Все точки пересечения плоскости со сферой образуют окружность, в которую этот треугольник вписан.
Поскольку он прямоугольный, то его гипотенуза (AB) совпадает с диаметром этой окружности, и прямо в середину этого отрезка упирается высота OH, опущенная из центра сферы (O) на нашу плоскость (она у нас равна 12).

Теперь рассматриваем треугольник AOH. Он прямоугольный, катеты OH = 12, AH = 5. Значит, радиус сферы AO = sqrt(12^2+5^2) = 13, а диаметр сферы - ровно вдвое больше, 26.

1063. Леонид , 20 апреля 2010, 23:40:24
1056. Lexxus. Спасибо. Чего народу не хватает, здесь же всё есть. И ШЕФ откликается на все вопросы. Даже на решение задач не из формата ЕГЭ. Кому можно написать о благодарности за работу ШЕФУ - Lexxus .

1064. ирина, 21 апреля 2010, 19:08:09
Объясните пожалуйста.Боковые ребра треугольной пирамиды взаимно перпендикулярны, каждое из них равно 3. Найдите объем пирамиды.

1065. страйфи, 21 апреля 2010, 19:46:44
класссссс

1066. Дария, 21 апреля 2010, 22:16:36
в стеклянный сосуд, имеющий форму прямоугольного параллелепипеда, с основанием 10 см *20 см и высотой 20 см налита вода до высоты 10 см. В сосуд бросили 2 металлических кубика с ребрами 2 см.на сколько повысится уровень воды.

не могу решить, мозг кипит...

1067. Лия, 22 апреля 2010, 11:11:55
Задание B8 (6432) Я не могу разобраться с этим вопросом. Помогите

1068. Лия, 22 апреля 2010, 11:13:42
Задание B8 (7643) не могу понять почему с моими ответами не совпадают ваши

1069. депутат, 22 апреля 2010, 13:57:14
вы все тупые и решать не умеете бараны. у вас нм грамма мазгов нет. идите домой

1070. депутат, 22 апреля 2010, 13:59:42
я могу вас всех перестрелять рогаткой. за то что ерундой занимаетесь. идиоты

1071. SANCHES, 22 апреля 2010, 14:28:19
это задание я взял из комплекта егэ ,который мы всем классом закупали в супермаркете и решали как пробный егэ

1072. Юлия, 22 апреля 2010, 16:30:14
Из пункта А в пункт Б выехал мотоциклист и одновременно из Б в А - автомобилист. Мотоциклист прибыл в Б через 2 часа после встречи, а автомобилист в А через 30минут после встречи. Сколько часов был в пути автомобилист?
Помогите,пожалуйста,кто знает не могу сообразить....

1073. Игорь, 22 апреля 2010, 18:23:00
Ответ Юлии.
Так как, мотоциклист до встречи проехал расстояние, которое автомобилист проежает за 0,5 часа , а мотоцтклист тратит на проезд оставшегося расстояния в 4 раза больше времени , то скорость автомобилиста в 4 раза больше, чем у мотоциклиста . Следовательно предыдущий участок ,до встречи, автомобилист проехал за время в четыре раза меньшее ( 0,5 часа) и был в пути 1 час.

1074. Игорь, 22 апреля 2010, 18:56:21
В12. Теплоход проходит по течению реки до пункта назначения 234 км и после стоянки возвращается в пункт отправления.Найдите скорость течения реки,если скорость теплохода в неподвижной воде равна 22км.ч,стоянка длится 4 часа и в пункт отправления он возвращается через 26 часов после отплытия из него.

Помогите, плиз!!!!!!!!!!!!!

Запомните, В12 составленны таким образом что при попытке решать их через уравнение получится квадратное уравнение с трудно вычислимым дискриминантом ( нужен калькулятор, чего не ЕГЭ не будет). Гораздо проще решать вот так:

Путь -234км
время в пути 26-4=22
Пусть х - скорость по течению, а
у- против течения
получаем
(х+у)/2 = 22-собственная скорость теплохода
234/х +234/у=22
теперь найдем делители числа 234 которые в сумме дают 22 и между этими числами число 22 ( собственная скорость теплохода)
проще всего искать числа путем ражложения 234 на прстые множинтели и их последующей комбинацией, в нашем случае это ряд 2*3*3*13 или путем комбинирования 9*26 и 18*13 так как сумма 9+13 дает 22 а ( 26+18)/2=22 то ясно, что х=26 ( по течению ) а у= 18 (против течения) , соответственно скорость течения 4 км/ч .
можно попоробовать решить влоб
пусть х- скорость течения , тогда ( 234/( 22+х)) +(234/( 22-х))- 22 =0
Сгоните все под один знаменатель , затем попробуйте решить квадратное уравнение , только честно без калькулятора, и сравните методику.

1075. Леонид , 22 апреля 2010, 19:34:49
1075. Игорь. Да, некоторые задачи решаются так даже без составления уравнения (и без переменных) - делители пути, если время и скорость отличаются одним числом - здесь 4. Другие задачи так не пойдут, только уравнение надо состалять. Тк что панации на все случаи жизни нет!!!

1076. SANCHES, 22 апреля 2010, 19:58:24
Игорь! я решил задачу :
Юлия, 22 апреля 2010, 16:30:14
Из пункта А в пункт Б выехал мотоциклист и одновременно из Б в А - автомобилист. Мотоциклист прибыл в Б через 2 часа после встречи, а автомобилист в А через 30минут после встречи. Сколько часов был в пути автомобилист?
.... с Х и получил другой ответ (1,5)!!! мне кажется твой аналитический способ неправильный

1077. katya.16ru, 22 апреля 2010, 20:13:03
в основании правильной пирамиды SABCD лежит четырехугольник ABCD . объём пирамиды равен 36 корень из 3.через точки В и S проходит сфера ,которая пересекает ребра SС и ВС соответственно в точках М и N,так,что SС :МС=5:2 и ВС : NС =2:1. найдите апофему пирамиды.

1078. Игорь, 22 апреля 2010, 21:27:04
Мда лоханулся, извиняйте, действительно 1,5 часа.
Не плохо было бы на ваше решение взглянуть

1079. Женёк, 22 апреля 2010, 21:54:16
Первая труба пропускает на пять литров воды в минуту меньше, чем вторая труба. Сколько литров воды в минуту пропускает вторая труба, если резервуар объемом триста семдесят пять литров она заполняет на десять минут раньше, чем первая труба заполняет резервуар объемом пятьсот литров?


1080. Женёк, 22 апреля 2010, 21:55:45
момогите пож

1081. Игорь, 22 апреля 2010, 22:31:03
Женёк, ну посмотри выше задачу на теплоход, пойми и потренируйся на прототипах.
Решение:
Пусть х- пропускная способность( л/мин) второй трубы ( за х берем то что спрашивают), а
у- первой, тогда,
у-х=5
375/х- время заполнения второй трубой, 500/у - первой , получаем
(500/х) - (375/х)= 10
ищем делители 500 и 375 чтобы они отличались на 5 и выполнялось условие (500/х) - (375/х)= 10 , разложим 500 и 375 на множители 500= 5*5* 5*2*2, 375= 5*5*5*3, или 500=25*20, а 375= 25*15 следовательно нам нужны в качестве делителей числа 20 и 25 , чтобы 500/20 - 375/25 = 25-15=10, кроме того выполнилось условие у-х =5 25-20 =5 , ответ 20.

1082. Игорь, 22 апреля 2010, 22:45:31
Извиняюсь, немного напутал с обозначениями, хотя я просто решаю, а потом смотрю чего спрашиваю - большую или меньшую в ответ писать.

Пусть х- пропускная способность( л/мин) второй трубы ( за х берем то что спрашивают), а
у- первой, тогда,
х-у=5
375/х- время заполнения второй трубой, 500/у - первой , получаем
(500/у) - (375/х)= 10
ищем делители 500 и 375 чтобы они отличались на 5 и выполнялось условие (500/у) - (375/х)= 10 , разложим 500 и 375 на множители 500= 5*5* 5*2*2, 375= 5*5*5*3, или 500=25*20, а 375= 25*15 следовательно нам нужны в качестве делителей числа 20 и 25 , чтобы 500/20 - 375/25 = 25-15=10, кроме того выполнилось условие х-у =5 25-20 =5 , ответ 25.

1083. Леонид , 22 апреля 2010, 22:52:02
1067. Дария, 21 апреля 2010, 22:16:36. Первоначально вода занимала объйм 10*20*10 = 2000. Два кубика с ребром по 2 - их объём 2^3 + 2^3 = 8+8=16.
Кинули их в воду, объём стал 2000+16=2016. Основание сосуда тоже самое, его площаь 10*20 = 200. Тогда уровень будет 2016:200 = 10,08 см.

Единицы в условии правильные?

1084. SANCHES, 22 апреля 2010, 22:59:24
до встречи каждый проехал по X часов. 1- все расстояние. Х+0,5- был в пути авто. Х+2-был в пути мото. 1/(Х+0,5)-скор. авто. 1/(Х+2)-скор. мото. (1/(Х+0,5))*0,5-проехал авто.после встречи. (1/(Х+2))*2-проехал мото после встречи. а вместе после встречи они проехали все расст.,т.е. 1.
(1/(Х+0,5))*0,5+(1/(Х+2))*2=1 .находим Х=1 -проехал авто до встречи .а был в пути получается Х+0,5=1+0,5=1,5

1085. Леонид , 22 апреля 2010, 23:06:25
1069. Лия, 22 апреля 2010, 11:13:42

№7643. Дан ГРАФИК ПРОИЗВОНОЙ функции. На отрезке [-3;2] линия графика ниже оси ОХ, т.е. производная отрицательная, а функция значит убывающая - наибольшее значение принимеет в точке х = -3.
№6432 в банке такого нет.

1086. Женёк, 22 апреля 2010, 23:25:24
спасибо Игорь)

1087. toyzy, 23 апреля 2010, 02:27:14
В треугольнике ABC AC=BC=3 sqrt{13}, AB=12. Найдите tg A.

Ребят, помогите решить задачу такого типа((

1088. Игорь, 23 апреля 2010, 06:48:53
To toyzy?, Треугольник равнобедренный, с вершиной С и основанием АВ, из С опускаем высоту, которая будет так же и медианой , получим два равных прямоугольных треугольника с гипотенузой АС или ВС и катетом при угле А катет равен половине АВ по теореме Пифагора находим второй катет, и имея два катета находим tg a, вычисления не привожу.

1089. Ольга, 23 апреля 2010, 16:57:10
Объясните решение.Основанием пирамиды служит прямоугольник, одна боковая грань перпендикулярна плоскости основания, а три другие боковые грани наклонены к плоскости основания под углом 60^circ. Высота пирамиды равна 6. Найдите объем пирамиды.

1090. Вася, 23 апреля 2010, 17:42:49
sqrt(2)/2-1
как решить этот пример?
Напишите пожалуйста!!!!!!!!!!!!!!!!!!!!!!

1091. Lexxus, 23 апреля 2010, 17:43:31
Основанием пирамиды служит прямоугольник, одна боковая грань перпендикулярна плоскости основания, а три другие боковые грани наклонены к плоскости основания под углом 60^circ. Высота пирамиды равна 6. Найдите объем пирамиды.

Пирамида ABCDE, ABCD - основание, AED - грань, перпендикулярная плоскости основания.
Проведем высоту EK к ребру AD. Она у нас по условию равна 6.
Ещё проведем высоту EM к грани BC.

Поскольку плоскость AED перпендикулярна плоскости основания, а все остальные грани наклонены к ней под одинаковым углом, то углы EDA=EAD=EMK = 60 градусов, и прямоугольные треугольники AEK, DEK и MEK равны.

Из этих треугольников найдем сразу всё, чего нам не хватает:
KM = KD = KA = EK/tg(60гр) = 6/sqrt(3)

Площадь ABCD = KM*(AK+KD) = 2*(6/sqrt(3))^2 = 24

Объем пирамиды равен 1/3*24*6 = 48

1092. Александра, 23 апреля 2010, 17:44:23
Прямоугольный параллелепипед описан около цилиндра,
радиус основания которого равен 3. Объем параллелепипеда равен 18.

ответ должен быть 0,5. что-то не могу решить=(

1093. марик, 23 апреля 2010, 19:11:17
я не понял условие задачи б 10 : b = - 0,4 К/ мин^2 - это значит что надо минуты в квадрат возводить? кстати, это было выделено жирным шрифтом почему -то....

1094. Вася, 23 апреля 2010, 19:13:26
ну напишите уж решение,пример же нетрудный)))))))

1095. Вася, 23 апреля 2010, 19:23:14
Александра, сторона праллепипеда будет равна 2 радиуса =6, стороны параллепипеда равны т.е е стороны по 6, чтобы найти высоту надо 18/36=0.5
Формула V=abc, c=v/ab=18/36=0.5
ответ третья сторона =0.5

1096. Лия, 23 апреля 2010, 21:29:36
6416 почему ответ 2?
Потому что 2-это точка минимума?

1097. Лия, 23 апреля 2010, 21:32:12
Задание B8 (6418) как тут решать?

1098. Lexxus, 23 апреля 2010, 21:40:27
6416 почему ответ 2?
Потому что 2-это точка минимума?

Грубо говоря, да. На самом деле, не только поэтому, но это лишняя информация, она только запутает.

1099. Lexxus, 23 апреля 2010, 21:52:08
Задание B8 (6418) как тут решать?

Функция параллельна прямой k*x+{неважно} в тех точках, в которых её производная равна k.

1100. Сергей, 23 апреля 2010, 23:33:43
Камнеметательная машина выстреливает камни под некоторым острым углом к горизонту. Траектория полeта камня описывается формулой y = ax^2 + bx, где a = - 1/600 м^(-1), b=4/15 — постоянные параметры, x (м) — смещение камня по горизонтали, y (м) — высота камня над землeй. На каком наибольшем расстоянии (в метрах) от крепостной стены высотой 9 м нужно расположить машину, чтобы камни пролетали над стеной на высоте не менее 1 метра?

1101. Сергей, 23 апреля 2010, 23:35:43
можете подсказать решение этой задачи???? если да то если можно на мыло вышлите сообщение. заранее спасибо!!!

1102. Сергей, 23 апреля 2010, 23:38:03
serega-leito@mail.ru вот мыло))))))))

1103. Игорь, 24 апреля 2010, 06:57:30
Сергею, подставь в формулу значения а и b , а у = 9+1=10 , получишь квадратное уравнение , реши его, наибольший корень и будет ответом.

1104. Patriot, 24 апреля 2010, 08:05:58
НОРОД, БЕНК ЗАДАНИЙ СИЛЬНО ИЗМЕНИЛСЯ!!!

МНОГО НОВЫХ ЗАДАНИЙ В ЧАСТИ В6 ДА И В4 ЕСТЬ!!!

МОЖНО ИХ ТОЖЕ КАК - НИБУДЬ ЗАСУНУТЬ В НАШУ БАЗУ С ОТВЕТАМИ?

1105. никитос, 24 апреля 2010, 08:10:27
народ кто уже написал пробный егэ по математике сёдня скажите ответы хотябы примерно или какие задания были

1106. зафар, 24 апреля 2010, 10:03:51
Объясните Диаметр основания конуса равен 6, а угол при вершине осевого сечения равен 90°. Вычислите объем конуса, деленный на pi .

1107. Леонид , 24 апреля 2010, 12:08:14
В осевом сечении конуса прямоугольтый треугольник. Пусть образующя конуса, уравнение x^2 + x^2 = 36. X^2 = 18. Находи высоту конуса по т. Пмфагора 18 - 9 (кв. радиуса) = 9. Н = 3 корня из 2. Объём 1/3 пи *9*(3 корня из 2). Вычисляем и делим на пи = 9 корней из 2 .

1108. Лёша, 24 апреля 2010, 12:18:24
"Бесплатные тесты по русскому языку, математике, физике и другим предметам
testege10.ru" - это предложения с рекламы (стр. здесь) . ХА-ХА. Сунься - первое - это платные услуги. Во БРЕШУТ.

1109. Даша, 24 апреля 2010, 13:35:14
В треугольнике ABC AC = BC = 5 , AB = 4. Найдите tgA.

1110. Александра, 24 апреля 2010, 15:08:44
спасибо Василий)
сегодня снова писали егэ и опять не смогла решить б10!
Прямоугольный параллелипипед описан около сферы радиуса 7. найти его объем.

1111. Александра, 24 апреля 2010, 15:11:10
Спасибо Василий =) сегодня снова писали егэ, и опять не смогла решить б10((
Прямоугольный параллелепипед описан около сферы радиуса 7. найти его объем)

1112. Лия, 24 апреля 2010, 16:08:21
что такое прототипы?

1113. Лия, 24 апреля 2010, 16:18:51
Сижу тут неделю, не могу понять где сами тесты(((
Простите за не понятливость

1114. игорь, 24 апреля 2010, 17:11:02
Даша, смотри сообщение 1088- аналогично

1115. Лия, 24 апреля 2010, 17:18:32
1098. Lexxus, 23 апреля 2010, 21:40:27
6416 почему ответ 2?
Потому что 2-это точка минимума?

Грубо говоря, да. На самом деле, не только поэтому, но это лишняя информация, она только запутает.


Мне нужно все понять)))
Может потому что если производная отрицательная, то функция убывает. Если положительная то возрастает. Тут получается наименьшее значение по функции точка 2?

1116. Леонид , 24 апреля 2010, 17:32:01
1111. Александра, 24 апреля 2010, 15:11:10
Спасибо Василий =) сегодня снова писали егэ, и опять не смогла решить б10((
Прямоугольный параллелепипед описан около сферы радиуса 7. найти его объем)

Прямоугольный паралл-д описанный около сферы - это куб с ребром равным диаметру сферы - ребро значит 14. Объём куба 14*1*14 = считай

1117. Михаил, 24 апреля 2010, 18:41:47
Привет, сегодня писал пробный ЕГЭ. Одна задачка, прям так и засела, никак не могу разобраться. Мож кто подскажет решение.

В12. Длина пути 240 км. Два велосипедиста выехали одновременно, первый ехал на 1 км/ч быстрее второго и финишировал на один час раньше второго. Найдите скорость в км/ч первого велосипедиста.


1118. Irina, 24 апреля 2010, 18:48:16
У меня где то 5-6 мая будут готовые ответы на все егэ, но к сожалению только часть А. Кого интересует пишите мне в асю. 604083126

1119. вася, 24 апреля 2010, 18:58:48
Нужно решение.Длина окружности основания цилиндра равна 3, высота равна 2. Найдите площадь боковой поверхности цилиндра.

1120. Леонид , 24 апреля 2010, 19:19:27
С=2 пи. Р(радиус) = 3, то Р =3/(2 пи.). Боков. поверх. ц. = пи. Р в кв.*Н =
=пи. (9/4 пи.кв.)*3 = 27/(4пи.).

Вася - Василёк, ты где берёшь эти задачи? Таких нет в банке ЕГЭ. Вам уже нечего от туда решать?

1118. Irinaх!!! Тргуешь помаленьку?

1121. Леонид , 24 апреля 2010, 19:28:21
1117 - МИШКА. В12. Длина пути 240 км. Два велосипедиста выехали одновременно, первый ехал на 1 км/ч быстрее второго и финишировал на один час раньше второго. Найдите скорость в км/ч первого велосипедиста.


За х - что просят, скорость 1-го. То скорость второго (х-1).
Время 1-го 240/х, время 2-го 240/(х-1). 1-ый затратил на 1ч. меньше, уравнение 240/(х-1) - 240/х = 1. К общему знаменателю - кв. ур-е - ответ.

1122. Михаил, 24 апреля 2010, 19:47:50
всё, всё, разобрался))) оказывается легкотня, просто давно не решал
там, как ты говорил в начале, время первого, равно скорости второго, так и получилось у одного 16 км в час, а у другого 15 км в час)
спасибо)

1123. Михаил, 24 апреля 2010, 19:51:26
1118. Irina, это на все на все предметы?) которые только предстоит решать?

1124. Оксана, 24 апреля 2010, 20:01:12
немогу решить задучу!помогите ((

Из пункта А в пункт В, расстояние между которыми 6 км, отправился пешеход, а через 30 мин вслед за ним выехал велосипедист, скорость которого в 2,4. раза больше скорости пешехода. В пункт В велосипедист приехал на 12 мин раньше пешехода. Найдите скорость велосипедиста.

1125. Леонид , 24 апреля 2010, 20:02:00
МИХАИЛ. Ты, что совсем пацан, веришь? А что давно не брался, плхо. По 2-3 задачи в день, натренируешься как в спорте и все проблемы. Голова у человек, чтобы нетолько думать где и за сколько, но и суметь справиться своими силами.

1126. Даша, 24 апреля 2010, 20:03:52
кто сдавал пробный егэ по математике сегодня????????скажите ответы плиз))

1127. Даша, 24 апреля 2010, 20:18:02
Леонид!ты сдавал пробный?привет))))

1128. elli_93, 25 апреля 2010, 09:00:36
1120. Леонид , 24 апреля 2010, 19:19:27
С=2 пи. Р(радиус) = 3, то Р =3/(2 пи.). Боков. поверх. ц. = пи. Р в кв.*Н =
=пи. (9/4 пи.кв.)*3 = 27/(4пи.).
Ну и где ты взял эту формулу? сам подумай, площадь боковой поверхности цилиндра- это прямоугольник со сторонами С- длина окружности и Н- высота цилиндра. Задача в одно действие 3*2=6.))

1129. elli_93, 25 апреля 2010, 09:12:50
Кто-нибудь сдавал досрочно математику? Хочется узнать, намного ли отличается демоверсия от экзамена)) Какие задачи части С? Хотя бы приблизительно))

1130. funduk, 25 апреля 2010, 09:48:53
Народ !!Оч. важно.
Кому не трудно выложите изображения как В8 делать.
Как решать я знаю,но решить не могу=(.

1131. Леонид , 25 апреля 2010, 09:56:53
Какие В8, аналитические или графические? Если графические, рисовать может только Lexxus.

1132. funduk, 25 апреля 2010, 10:25:43
B8: 6404,6427,6422,6412,6423,8439
Вроде все разные.Если есть ещё какие-то,то и их не могу решать.
Пока только экстремумы нахожу,но задания где надо сложить решаю не верно.

1133. Лия, 25 апреля 2010, 14:20:08
я задала несколько вопросов.Никто не ответил((

1134. funduk, 25 апреля 2010, 14:38:54
Почему через оперу-мини нельзя нажать кнопку "Ответить" в заданиях и загружает http://live.mephist.ru/?mid=1255348015 не доконца.
Просьба исправить.

1135. Анна, 25 апреля 2010, 15:29:48
Во сколько раз увеличится площадь боковой поверхности конуса, если его образующую увеличить в 3 раза?

1136. Лия, 25 апреля 2010, 16:12:33
Во сколько раз увеличится площадь боковой поверхности конуса, если его образующую увеличить в 3 раза?

Ответ : в 3 раза

1137. рустам, 25 апреля 2010, 17:22:35
можете выложить решения задач B8? А не только ответы!!!

1138. Сергей, 25 апреля 2010, 18:08:02
Прямая y=6x+9 параллельна касательной к графику функции y=x^2+7x-6. Найдите абсциссу точки касания.

Можно увидеть решение этой задачи??? Спасибо заранее

1139. Сергей, 25 апреля 2010, 18:12:44
Груз маccой 0,8 кг колеблетcя на пружине cо cкороcтью, меняющейcя по закону v(t)=0,3sin pi t, где t — время в cекундах. Кинетичеcкая энергия груза, измеряемая в джоулях, вычиcляетcя по формуле E = mv^2 / 2, где m — маccа груза (в кг), v — cкороcть груза (в м/c). Определите, какую долю времени из первой cекунды поcле начала движения кинетичеcкая энергия груза будет не менее 27*10^(-3) Дж. Ответ выразите деcятичной дробью, еcли нужно, округлите до cотых.

И решение этой задачи напишите плиз.

1140. funduk, 25 апреля 2010, 19:07:15
Кто нибудь отправьте изображения с решением заданий В8!!

1141. Наталья, 25 апреля 2010, 21:26:05
1138. Сергей
Прямая параллельна касательной, значит угловые коэффициенты равны. K=6. Угловой коэффициент касательной равен значению производной в точке касания, т.е. y'(X0)=2X0+7=6, отсюда X0=-0,5.

1142. Наталья, 25 апреля 2010, 23:35:43
Задание B10 (№ 28405) из открытого банка заданий.
Для поддержания навеса планируется использовать цилиндрическую колонну. Давление P (в паскалях), оказываемое навесом и колонной на опору, определяется по формуле P = (4mg)/(pi*D^2), где m = 600 кг — общая масса навеса и колонны, D — диаметр колонны (в метрах). Считая ускорение свободного падения g=10 м/с^2, а pi = 3, определите наименьший возможный диаметр колонны, если давление, оказываемое на опору, не должно быть больше 500000 Па. Ответ выразите в метрах.

Получилось, что необходимо взять корень из числа 0,016. Кто-нибудь, проверьте пожалуйста

1143. Lexxus, 26 апреля 2010, 09:54:38
Задание B10 (№ 28405) из открытого банка заданий.
Для поддержания навеса планируется использовать цилиндрическую колонну. Давление P (в паскалях), оказываемое навесом и колонной на опору, определяется по формуле P = (4mg)/(pi*D^2), где m = 600 кг — общая масса навеса и колонны, D — диаметр колонны (в метрах). Считая ускорение свободного падения g=10 м/с^2, а pi = 3, определите наименьший возможный диаметр колонны, если давление, оказываемое на опору, не должно быть больше 500000 Па. Ответ выразите в метрах.

Получилось, что необходимо взять корень из числа 0,016. Кто-нибудь, проверьте пожалуйста

И правда, нолик там в значении наибольшего давления либо один лишний, либо одного не хватает. Убрал один нолик, теперь корень нужно извлекать из 0.16.

1144. Муся, 26 апреля 2010, 12:48:44
Для Сергея 1139
Все имеющиеся данные подставить в формулу для Е, после приведения к общему знаменателю получится:
2 х 27 х 10(в степени -3)=0,8 х (0,3 sinпиt )в квадрате.
После возведения в квадрат 54 х 10( в степени -3)= 0,8 х 0,09 sin(в квадрате)пиt. Далее после преобразований 54= 72 sin(в квадрате)пиt.
sin(в квадрате)пиt. =3/4, а sinпиt =( корень из 3)/2. То есть пиt = пи/3, а t =1/3 или в сотых 0,33

1145. Вадим, 26 апреля 2010, 15:24:36
C1 из пробника ,прошедешего в Казани:
81^(tgx)-8*9^(tgx)-9=0
{
sqrt(y-6)+12 cosx=0
У меня получился ответ:
x=5pi/4+2pin; n принадл z
y=78
КТо-нибудь напишите свои ответы , хочу свериться

1146. Azer, 26 апреля 2010, 15:51:24
Помогите решить задание В9
Объем данного правильного тетраэдра равен 64 см3. Найдите
объем правильного тетраэдра, ребро которого в 2 раза меньше ребра
данного тетраэдра. Ответ дайте в см3.

1147. Игорь, 26 апреля 2010, 17:54:27
Админ! Пополни пожалуйста здешний банк задачами из mathege.ru .
В mathege.ru задач типа B12 700+, а тут всего лишь 198. Так же и с другими "B"шками.

1148. Леонид , 26 апреля 2010, 17:55:55
ДЛЯ 1146. Ответ: 8. Здесь просто - ребро уменьшается в 2 раза, то объём уменьшится в 2^3 раз, то есть 8 раз (64:8=8).


1145. Вадим, 26 апреля 2010, 15:24:36. Молодец, что написал! Пиши другие С своего КИМ. Это решу напишу, что получается.

1149. Катюшка, 26 апреля 2010, 18:33:54
помогите пожалуйста решить задачу... очень-очень надо+)
точка М лежит на стороне АВ треугольника АВС, ВМ=7, АМ=3, угол ВСМ=углу ВАС, косинус угла В=корень из трёх десятых. Найдите площадь треугольника СВМ

1150. Lexxus, 26 апреля 2010, 18:48:48
C1 из пробника ,прошедешего в Казани:
81^(tgx)-8*9^(tgx)-9=0
{
sqrt(y-6)+12 cosx=0
У меня получился ответ:
x=5pi/4+2pin; n принадл z
y=78

Всё правильно, ничего не забыл и ничего лишнего.

1151. KLASS, 26 апреля 2010, 20:22:26
Ребят большое спасибо за удаление моего коммента!)))))))))))) Есть кто нить кто хочет в ПГЛУ поступать?

1152. Я, 26 апреля 2010, 20:32:47
Я думаю, нужно срочно сделать поиск по заданиям. и чтобы комментарии выводились не все сразу, очень неудобно, особенно, если заходишь с телефона. Уходит очень много трафика.

1153. Анастасия, 26 апреля 2010, 20:45:18
Здраствуйте, помогите пожалуйста решить задачу...вы конечно похожую решили....но ход решения не совсем понятен....
.Две окружности радиусов корень из 10 и корень из 5 имеют общую хорду АВ, длина которой равна 2. Через точку В проведён диаметр ВМ большей окружности, причём прямая ВМ вторично пересекает меньшую окружность в точке С. Найдите площадь треугольника АСМ. .....вот.......я из 9 класса.....так что если можно то очень подробно....=).заранее спасибо...за мной глупый вопрос.

1154. М, 26 апреля 2010, 21:06:22
Кто-нибудь знает, как решать 6422 б8?

1155. funduk, 26 апреля 2010, 21:29:19
Если надо,то скажите куда выложить задания,которые были на пробном.
Вроде составлял край, но мб тупо скачали откуда-нибудь.

1156. Леонид , 26 апреля 2010, 23:46:21
1155. funduk. Составляет эти вещи ни какой не край,а центр. Напиши свои С сюда, пожалуйста. Какой регион у тебя?

1157. Наталья, 27 апреля 2010, 01:05:50
1154. М, Задание 6422, в8 на сайте пишет:
Указанная задача не существует или недоступна

1158. Ваня, 27 апреля 2010, 10:22:55
Напишите пожалуйста решение:
Цена на электрический чайник была повышена на 21% и составила 3025 рублей. Сколько рублей стоил товар до повышения цены?

1159. М, 27 апреля 2010, 14:16:19
Наталья, 6422 http://live.mephist.ru/show/mathege2010/view/id/6422/

1160. RuS92, 27 апреля 2010, 15:22:53
Почему на этом сайте не все задания из Открытого Банка?Вот допусти вот этого задания нет:
Прототип задания B5 (№ 26687)
Для того, чтобы связать свитер, хозяйке нужно 400 граммов шерсти синего цвета. Можно купить синюю пряжу по цене 60 руб. за 50 г, а можно купить белую пряжу по цене 50 руб. за 50 г и окрасить ее. Один пакетик краски стоит 10 руб. и рассчитан на окраску 200 г белой пряжи. Какой вариант покупки дешевле? В ответ напишите, сколько рублей будет стоить эта покупка.
http://www.mathege.ru:8080/or/ege/ShowProblems?offset=14&posMask=16&showProto=true

1161. Наталья, 27 апреля 2010, 15:23:04
Ваня, x- цена до повышения. 21% от х это 0,21*х - на столько повысилась и стала составлять х+0,21х=1,21х = 3025, х=2500.

М, производная больше нуля на некоторых промежутках, это значит, что функция на этих промежутках возрастает. Дан график функции, смотрим, где она возрастает. Края этих промежутков считать не нужно, а единственная целая точка, которая в них попала - это х=2. Но нам надо посчитать количество таких точек, т.е. одна. Если непонятно, напиши, я еще подробнее распишу

1162. Наталья, 27 апреля 2010, 15:25:54
М.
Но, кстати, задачу эту из официального открытого банка удалили

1163. RuS92, 27 апреля 2010, 15:27:52
Да нет не удалили!!!!!!!!!!!
http://www.mathege.ru:8080/or/ege/ShowProblems?offset=14&posMask=16&showProto=true

1164. funduk, 27 апреля 2010, 15:34:09
Системы:
I
1 2sqrt(2)*sin(x)+y=0
2 sqr(y)+3y+2*sqrt(sqr[y]+3y-6)=14
II
1 sin(x)*cos(y)=1/4
2 sin(y)*(cos(x)=-1/4
III
1 (6*sin(sqr[x])+7*sinx-5)*sqrt(-cos[x])=0
2 sin(sqr[x])=cos(sqr[y])

Решение мне не надо.

1165. Вадим, 27 апреля 2010, 16:02:21
Согласен, здесь есть задачи которые не из отвкрытого банка, а точнее те которые раншье там были , а теперь их нет.+ не хватает некоторых заданий напирмер в B12 их 700+, а тут всего лишь 198, такие же дела и с другими задачами

1166. Lexxus, 27 апреля 2010, 16:03:20
НОРОД, БЕНК ЗАДАНИЙ СИЛЬНО ИЗМЕНИЛСЯ!!!

МНОГО НОВЫХ ЗАДАНИЙ В ЧАСТИ В6 ДА И В4 ЕСТЬ!!!

МОЖНО ИХ ТОЖЕ КАК - НИБУДЬ ЗАСУНУТЬ В НАШУ БАЗУ С ОТВЕТАМИ?

Запустил выдиралку заданий из банка. Через несколько часов, надеюсь, получу все новые задания и залью в систему.
Спасибо за сигнал.

1167. Azer, 27 апреля 2010, 16:37:01
Задание С1 в пробном егэ в Астрахани
I x+y I= 1 I--I - модуль
sinx+siny=2П/3
Вроде так!!

1168. брюнетка, 27 апреля 2010, 16:41:16
блин народ тут большинство заданий (ответы) не верные. неужели все верят этим ответам?))

1169. Lexxus, 27 апреля 2010, 16:44:44
Коментарий скрыт (грубость — 6%)показать

1170. Наталья, 27 апреля 2010, 16:51:24
RuS92, да, задач в открытом банке ощутимо больше, это я не про вашу задачу писала, а для М.


funduk, sqr - как в программировании, возведение в квадрат?

1171. Михаил, 27 апреля 2010, 17:07:15


2^{10} х 11^{8} : 22^{6}.

ответ: 22^10,, так или нет??????? если так, то это невозможно отобразить, слишком большое число(
а если не так, то как???

1172. Михаил, 27 апреля 2010, 17:08:45
P.S: х - это умножение)

1173. Lexxus, 27 апреля 2010, 17:12:26
2^{10} х 11^{8} : 22^{6}.

ответ: 22^10,, так или нет???????

Хотел бы я знать, какие размышления тебя привели к такому ответу. Нет, не так.
(2^10)*(11^8)/(22^6) = (2^10)*(11^8)/(2^6)/(11^6) = (2^4)*(11^2) = 16*121

1174. Михаил, 27 апреля 2010, 17:44:13
1173. Lexxus,
спасибо, а я блин, со степенями велосипед изобретал)

(2^10)*(11^8)/(22^6) = (2^5*2)*(11^2*4)/(22^6)= (2^2^5)*(11^2^4)/(22^6)=
= ((22)^2)^9/(22^6) ........... бред, короче))))




1175. Lexxus, 27 апреля 2010, 18:23:40
В банк добавилось больше тысячи новых задач!

1176. Леонид , 27 апреля 2010, 19:06:03
1164. funduk, 27 апреля 2010, 15:34:09! СПАСИБО. А С 2-4, хотя бы. И какой регион.

1177. funduk, 27 апреля 2010, 20:33:42
1176.Леонид, пожалуйста=).
Регион- Ставропольский край.
Через некоторое время ещё выложу остальные.Просто сейчас варианты у учителя, а это переписал из тетради.

Кто-нибудь нормально может показать как делать В8,а не просто ответ.

1178. funduk, 27 апреля 2010, 20:38:06
Можно не объяснять,но хотя бы сказать,как эти задания называются не на школьному уровне,а остальное я найду.

1179. Михаил, 27 апреля 2010, 21:30:32
Моторная лодка прошла против течения реки 80 км и вернулась в пункт отправления, затратив на обратный путь на 2 часов меньше, чем на путь против течения. Найдите скорость течения, если скорость лодки в неподвижной воде равна 9 км/ч. Ответ дайте в км/ч.

друзья, подскажите, как решать? у меня ответ не получается нормальным, мож я не так решаю(

1180. Lexxus, 27 апреля 2010, 21:51:27
Кто-нибудь нормально может показать как делать В8,а не просто ответ.

Там несколько типов заданий. Но вот тот, который вызывает больше всего затруднений:



Изображен график производной, а нам нужно определить, в каких точках касательная к графику самой функции параллельна или совпадает с прямой a*x+b.
Две прямые параллельны, когда у них равные углы наклона касательных, то есть равные производные. Значит, нам достаточно определить, в каких точках график производной f(x) пересекается с графиком производной (a*x+b).

(a*x+b)' = a,
b ни на что не влияет и нас не интересует.

Так что на рисунке надо провести горизонтальную линию y=a и определить, в скольких точках она пересекает график производной.

1181. Таша, 27 апреля 2010, 21:52:31
Двое рабочих совместно могут выполнить заданную работу за 12 дней. Если первый рабочий сделает половину работы, а затем второ - вторую половину, то вся работа будет закончена за 25 дней. Во сколько раз один из рабочих работет быстрее другого?

1182. таша, 27 апреля 2010, 21:56:12
помогите плиз. с задачей.

1183. elli_93, 27 апреля 2010, 22:06:58
Леонид! И ВСЕ, КОМУ ИНТЕРЕСНО! Вот задача с4 с досрочного. не могу решить. У кого какие идеи?
в окружность радиуса( корень из 61)/2 вписана трапеция с основаниями 5 и 7. Найти расстояние от центра окружности до т. пересечения диагоналей трапеции.

1184. elli_93, 27 апреля 2010, 22:22:40
Таша! Пусть 1- вся работа., х-производительность первого, у- произ второго. Тогда система 1/12=х+у и 0,5/х+0,5/у=25. путём несложных вычислений получаем х=1/30 и у=1/20. теперь 1/20(он быстрее работает)/ на 1/30. получам 1,5. те в полтора раза один работает быстрее другого. Вроде бы так))

1185. Леонид , 27 апреля 2010, 22:30:06
1183. elli_93, 27 апреля 2010, 22:06:58! У нас 2-ой час ночи, задачу записал, завтра что-то "выдам". Но первое, спасибо, что не забыла про меня. А другие С? Могу сходу отметить: трапеция равнобедренная, расстояние искомое на высоте трапеции через середины оснований и т.д. Смотрю.

1186. Леонид , 27 апреля 2010, 22:43:32
1179. Михаил, 27 апреля 2010, 21:30:32

х-ск. теч. реки. ТО (9-х) - ск. лодки против теч., (9+х) - ск. по теч.
Время против теч. 80/(9-х). Время по теч. 80/(9+х). Явно, против теч. времени затрачено больше, уравнение: 80/(9-х) - 80/(9+х) = 2.

Ты, находя время,то от того отнимал? Не допустил ошибки (х-9) в ск. лодки против теч.?

1187. Таша, 27 апреля 2010, 22:51:21
скорость моторной лодки по течению реки больше 21 км/ч и меньше 23 км/ч, а скорость пртив течения больше 19 км/ч и меньше 21 км/ч. В каком промежутке будет собственная скорость лодки (в стояче воде)?

1188. Таша, 27 апреля 2010, 22:56:52
решите плиз

1189. Игорь, 27 апреля 2010, 23:00:53
1179. Михаил, 27 апреля 2010, 21:30:32
Моторная лодка прошла против течения реки 80 км и вернулась в пункт отправления, затратив на обратный путь на 2 часов меньше, чем на путь против течения. Найдите скорость течения, если скорость лодки в неподвижной воде равна 9 км/ч. Ответ дайте в км/ч.

пусть х скрость по течению
пусть у скорость против течения
(х+у)/2=9- собственная скорость лодки
(80/у) - ( 80/х) =2
ясно, что прсто просится в качестве у= 8 а х= 10
проверяем (8+10)/2=9 -сходится
(80/8)-(80/10) =2 -сходится
если по течению 10 км/ч, а против 8 км / ч, и собственная скорость 9 км/ч, то скорость течения =1 км/ч

или по другому
пусть скорость течения х
тогда
(80/ (9-х)) -( 80/ (9+х))= 2
дальше получается дробь ( х^2 + 80х -81 ) / (81 - х^2) =0
попробуй без калькулятора ( на ЕГЭ он не предусмотрен) решить
х^2 + 80х -81 =0

1190. таша, 27 апреля 2010, 23:07:27
помогите решить задачу.
пожалуста.

1191. Игорь, 27 апреля 2010, 23:08:07
скорость моторной лодки по течению реки больше 21 км/ч и меньше 23 км/ч, а скорость пртив течения больше 19 км/ч и меньше 21 км/ч. В каком промежутке будет собственная скорость лодки (в стояче воде)?
Пиши систему неравенств
21<x<23
19<x<21
теперь слохи столбиком получишь
40<2x<44
теперь раздели все на два
20<x<22

1192. таша, 27 апреля 2010, 23:11:17
спасибо большое

1193. Леонид , 27 апреля 2010, 23:29:04
1183. elli_93.
Посмотрел, продолжаю: АВСД - трап. (равнобедр), ВН - высота, МК - тоже высота через середины оснований - М на ВС(верхнее 5), К-на АД (нижнее 7).
Р - пересечение диагоналей, О - центр окр. внутри трапеции на МК.
Т. Пифагора: Треуг. ВМО - находим МО, Треуг ОКД - находим ОК, то находится высота трап. МО+ОК+МК = ВН.
Два подобных треуг. ВНД и РКД - НД (6):КД(3,5) = ВН (нашли):РК - искомое. ОР = РК-ОК. Не вычислял - корни там печатать муторно и пояснения без чертежа, но ты поймёшь. Это 1-й случ.
2-й сл. центр окр. вне трапеции - высота трапеции = разности ОМ и ОК в тех же треуг.

1194. tolerop, 28 апреля 2010, 00:53:32
мне кажется, что в задании 12801 ответ надо исправить с 7 на -3. Или я что-то недопонял...

1195. Леонид , 28 апреля 2010, 01:13:53
1194. tolerop. Подставь (-3) и что, + = - твоя ошибка в понимании арифметического корня: перед кв. корень в левой части знак +. В этом маленькая хитрость этих заданий.

1196. tolerop, 28 апреля 2010, 01:52:47
но там же в ответе нужно указать меньший корень. Их получилось 2: 7 и -3. Значит меньший корень -3.

1197. Леонид , 28 апреля 2010, 07:27:31
1196. tolerop. Ты ничего не понял. (-3) корнем не является.

1198. Funduk, 28 апреля 2010, 08:30:31
1180.Lexxus, спс большое!

1199. Михаил, 28 апреля 2010, 11:49:55
Леонид и Игорь, спасибо за решение, я со знаками путался(
а теперь разобрался)

1200. Михаил, 28 апреля 2010, 12:49:18
В цилиндрический сосуд налили 1800{см}^3 воды. Уровень воды при этом достигает высоты 12 см. В жидкость полностью погрузили деталь. При этом уровень жидкости в сосуде поднялся на 2 см. Чему равен объем детали? Ответ выразите в {см}^3.

я вот не помню, надо ли при решении этой задачи, кубы переводить в обычные см, при работе с ними. Я могу напрямую разделить 1800 см^3 на 12 см???

1201. Наталья, 28 апреля 2010, 13:05:38
Михаил, и там и там сантиметры, можно делить

1202. Михаил, 28 апреля 2010, 13:19:40
Наталья, спасибо)))

1203. tolerop, 28 апреля 2010, 13:33:42
Спасибо Леонид! Разобрался и понял теперь.

1204. Наталья, 28 апреля 2010, 14:51:21
Что-то говорят, что добавили задания, а в заданиях B6 не вижу векторов, в В4 окружностей, что добавилось -то?

1205. Михаил, 28 апреля 2010, 14:57:22
В равнобедренном треугольнике ABC с основанием AC боковая сторона AB равна 8, а cos A=(7^1/2)/4. Найдите высоту, проведенную к основанию.

Друзья подскажите, здесь косинус в синус надо переводить? Или есть другое решение?

1206. Иван, 28 апреля 2010, 14:58:54
Длина окружности основания конуса равна 3, образующая равна 2. Найдите площадь боковой поверхности конуса.Проверти ответ 6

1207. Наталья, 28 апреля 2010, 18:34:56
Михаил, cosА=АН/8, (Н - точка, в которую падает высота ВН), отсюда найти АН, а потом ВН по теореме Пифагора как корень из АВ^2-AH^2

1208. Наталья, 28 апреля 2010, 18:37:11
Иван, правильно, шесть

1209. Lexxus, 28 апреля 2010, 18:49:32
Что-то говорят, что добавили задания, а в заданиях B6 не вижу векторов, в В4 окружностей, что добавилось -то?

Только что добавил сюда все прототипы с mathege.ru

Есть и про векторы, и про окружности.

1210. Михаил, 28 апреля 2010, 19:20:06
Из А в В одновременно выехали два автомобиля. Первый проехал с постоянной скоростью весь путь. Второй проехал первую половину пути со скоростью, меньшей скорости первого на 4 км/ч, а вторую половину пути — со скоростью 45 км/ч, в результате чего прибыл в В одновременно с первым автомобилем. Найдите скорость первого автомобиля, если известно, что она меньше 30 км/ч. Ответ дайте в км/ч.

Друзья, помогите, по какой это формуле решается, что-то у меня не одна не подходит?

1211. Евгения, 28 апреля 2010, 20:09:11
Найдите корень уравнения: корень из 28 - 3x= x Если уравнение имеем более одного корня, укажите меньший из них.

Ответ: -7

но ведь ответ 7
x не может быть равен отрицательному числу

1212. Михаил, 28 апреля 2010, 20:17:00
Евгения, уравнение имеет всего один корень и это 7 и он наименьший!


1213. Иван, 28 апреля 2010, 20:23:03
Во сколько раз уменьшится площадь боковой поверхности конуса, если его радиус его основания уменьшить в 1,5 раза?

1214. Александра, 28 апреля 2010, 20:35:36
1.прямоугольный параллелепипед описан около сферы радиуса 7. найти его объём)


2.прямоугольный параллелепипед описан около цилиндра, радиус и высота =1. найти объем параллелепипеда)

1215. Анастасия, 28 апреля 2010, 21:01:47
Было бы здорово, если бы именно эти задания были на ЕГЭ.Думаю, что мне повезет... Всем желаю удачи!!!

1216. Леонид , 28 апреля 2010, 21:28:53
1210. Михаил, 28 апреля 2010, 19:20:06

х -ск. 1-го, (х-4)-ск. 2-го на первой половине пути. Пусть весь путь 2 (так прще - без дробей). Тогда время 1-го 2/х. Время второго на 1-ой половине пути 1/(х-4), на второй половине пути его время 1/45. Прибыли оновременно, равно их время, уравнение 2/х = 1/(х-4) + 1/45. К общему знамен., в одну часть, решаем уравнение. Поятно пояснил, Михайло?

1206. Иван. Про конус, да 6.
1213. Иван. уменьшится в 2,25 раза. Уменьшение радиуса в квадрате ( в формуле раиус в кв.)

1214 Александра. 1) Это куб с ребром равным диаметру шара:14*14*14.
2)Параллелепипед со стороной основания равной двум радиусам, значит 2. Объём 2*2*1 = 4.

1217. Игорь, 28 апреля 2010, 22:54:47
Леониду и Михаилу на сообщение 1210-
Очень замечательно , что попалась такая задача , так как я прорешал прототипы такого типа :
"Из А в В одновременно выехали два автомобиля. Первый проехал с постоянной скоростью весь путь. Второй проехал первую половину пути со скоростью, меньшей скорости первого на 17 км/ч, а вторую половину пути - со скоростью 102 км/ч, в результате чего прибыл в В одновременно с первым автомобилем. Найдите скорость первого автомобиля, если известно, что она больше 60 км/ч. Ответ дайте в км/ч."
Она решается без квадратного уравнения путем рассуждения - если вы отставали на х км/ч на первой половине пути , то чтобы догнать к финишу нужно вторую половину ехать со скоростью +2х от скорости движущегося с постоянной скоростью Итого имеем отставание было 17 и после увеличения на 2*17 стала 102 значит скорость постояннодвижущегося 102-34= 68
Если решать по схеме Леонида то выходим на уравнение -х^2+119х-3468=0
как его решить без калькулятора я не представляю хотя корни будут 51 и 68

1218. Леонид , 28 апреля 2010, 23:24:57
Игорёк, я твои рассуждения не думаю оспаривать, но не каждому под силу такие логические рассуждения. Надёжнее, если человек научится алгоритмическому подходу в решении задач. Все они решаются по одной схеме. Поверь, часто допускают ошибку при записи ответа даже с правильным решением.
Ещё раз советую брать за х то, что просят в задаче., спокойно составить уравнение и решить его. Неслучайно эти простые задачки вызывают затруднения ну у очень многих. А твои рассуждения мы уже указывали здесь в октябре- ноябре, полистай. А решать без калькулятора просто: не спеши перемножать числа-уравнение будет делиться на одно и тоже число. Чтобы без проблем извлечь корень, разложи число на простые множители (5кл.), как в заданиях В7 с разностью квадратов под корнем, это часто приходится елать и в В 4. И т. д.
Сделай доброе дело, напиши свои пробные С и какой регион.

1219. Наталья, 28 апреля 2010, 23:41:26
Игорь, а как быть тогда с такой задачей?
Задание B12 (39045)
Из А в В одновременно выехали два автомобиля. Первый проехал с постоянной скоростью весь путь. Второй проехал первую половину пути со скоростью 90 км/ч, а вторую половину пути — со скоростью, на 11 км/ч большей скорости первого, в результате чего прибыл в В одновременно с первым автомобилем. Найдите скорость первого автомобиля. Ответ дайте в км/ч.
Какие будут рассуждения? Ответ тут 99 км/ч

1220. Liy, 29 апреля 2010, 13:30:54
я не могу тесты под№ В6 решать. Вместо ответов пишет
Верный ответ пока не определен.
Что делать?

1221. коза, 29 апреля 2010, 14:52:02
идиотка

1222. "Анастасия", 29 апреля 2010, 15:03:06
ЕГЭ действительно сложное! Особенно сложно будет сдать ЕГЭ тем , кто живёт в деревнях и посёлках, потому что нет возможности подготовиться на отлично. У нас некоторые задачи даже учителя прорешать не могут, что уж говорить об учениках! Я начала готовится с начала года и всё же не могу решить задания из части "С"- это просто WAY задания! Я чесами сижу за учебниками, даже на своё 18-летия я решала ЕГЭ! Я очень боюсь подвести себя, а прежде всего своих родителей!

1223. валя, 29 апреля 2010, 17:13:15
Боковые ребра треугольной пирамиды взаимно перпендикулярны, каждое из них равно 3. Найдите объем пирамиды.

1224. Михаил, 29 апреля 2010, 18:24:37
На рисунке изображен график производной функции f(x), определенной на интервале (-9;8). В какой точке отрезка [-7;-3] f(x) принимает наибольшее значение.

понятно, что это - 7, но где это на графике смотреть?

Р.S/ я не имею ввиду именно это задание, а такого плана все задания.

кто подскажет?

1225. Lexxus, 29 апреля 2010, 18:40:16
На рисунке изображен график производной функции f(x), определенной на интервале (-9;8). В какой точке отрезка [-7;-3] f(x) принимает наибольшее значение.

Во всех таких заданиях на указанном отрезке производная либо везде отрицательная, либо везде положительная.

Если на отрезке производная положительная, то на нем везде функция возрастает, и наименьшее значение она принимает на левой границе отрезка, а наибольшее - на правой.

Если же производная отрицательная, то функция на отрезке убывает, и всё наоборот - наибольшее значение она принимает на левой границе, а наименьшее - на правой.

1226. Леонид , 29 апреля 2010, 18:40:40
Смотри, возрастает или убывает (по графику графику произвоной на этом промежутки): произвоная + где её гафик выше ОХ, (-) где график ниже оси
ОХ. Возрастает- точка левая, убывает-точка правая, имеется в виду заданный [ -7;-3 ]/

1227. Михаил, 29 апреля 2010, 19:28:43
Леонид и Lexxus, с этим разобрался, спасибо)

а как найти количество точек экстремума функции f(x) на отрезке?

я вроде бы понимаю и не понимаю, это тоже самое что локальный мин. и мах?


1228. Игорь, 29 апреля 2010, 19:52:25
Михаилу на 1227, в экстремумах ( локальных мин или мах) функция их возрастаущей переходит в убфваущюю или наоборот, следовательно производная меняет знак, следовательно по графику нужно смотреть где пересекает ОХ

1229. Михаил, 29 апреля 2010, 20:18:32
Определите количество целых точек, в которых производная функции f(x) отрицательна.

ф-я отрицательная, когда она убывает... а что есть целые точки?


1230. Михаил, 29 апреля 2010, 20:36:11
целые точки - это целые числа...

кто-н. может показать дураку где в Задание "B8 (8239)" 23 целые точки???


1231. Lexxus, 29 апреля 2010, 20:46:48
кто-н. может показать дураку где в Задание "B8 (8239)" 23 целые точки???

Целых точек, где функция убывает (т.е. производная отрицательна), там всего пять: -1, 3, 4, 8 и 9.
Но в задании просят найти сумму целых точек, а не их количество.
-1+3+4+8+9 = 23

1232. Pogodar, 29 апреля 2010, 21:03:09
Ребят,слегка сумасшедший вопрос)))телефон на экзамен есть возможность принести и им воспользоваться?

1233. Михаил, 29 апреля 2010, 21:07:14
я всё где-то рядом и так не могу догнать, а в задание "Задание B8 (6423)", где там 8 точек?
Lexxus - можешь на графики их изобразить?
Заранее спасибо)

1234. Михаил, 29 апреля 2010, 21:16:10

А почему в "Задание B8 (6424) " - функция положительная?
мож я не так полярность определяю...

1235. Lexxus, 29 апреля 2010, 21:29:55
Миша, внимательно читай задания. Ты, судя по всему, никак не можешь заметить, где изображен график самой функции, а где - график производной пункции.

1236. mt, 29 апреля 2010, 22:16:23
C1-C5, Москва, май 2008 г., см.
http://i258.photobucket.com/albums/hh254/m962/1555.jpg

1237. Игорь, 29 апреля 2010, 22:18:06
1219. Наталья, 28 апреля 2010, 23:41:26
Игорь, а как быть тогда с такой задачей?
Задание B12 (39045)
Из А в В одновременно выехали два автомобиля. Первый проехал с постоянной скоростью весь путь. Второй проехал первую половину пути со скоростью 90 км/ч, а вторую половину пути — со скоростью, на 11 км/ч большей скорости первого, в результате чего прибыл в В одновременно с первым автомобилем. Найдите скорость первого автомобиля. Ответ дайте в км/ч.
Какие будут рассуждения? Ответ тут 99 км/ч
Убедили, все таки алгоритм Леонида гораздо универсальнее.

1238. Михаил, 30 апреля 2010, 10:26:24
"1235. Lexxus, 29 апреля 2010, 21:29:55
Миша, внимательно читай задания. Ты, судя по всему, никак не можешь заметить, где изображен график самой функции, а где - график производной пункции. "

а чем они отличаются? я вот никак не могу вникнуть в эти целые точки?
как их находить? подскажите пожалуйста)

1239. Александра, 30 апреля 2010, 13:42:55
спасибо, Леонид=)
вот в онлайне решаю егэ, кто-нить знает как решить задания такого плана...
1)Б7 Найдите значение выражения: 39·26:65.
2)Вопрос B9 из 12


Прямоугольный параллелепипед описан около цилиндра,
радиус основания которого равен 3. Объем параллелепипеда равен 18.
(формулу плз напишите ) если я правильно поняла это будет пr^2 * h? и тогда ответ 2?

3)Вопрос B10 из 12
Для определения эффективной температуры звёзд используют закон Стефана–Больцмана, согласно которому мощность излучения нагретого тела прямо пропорциональна площади его поверхности и четвёртой степени температуры: P = &#231;ST4, где &#231; = 5,7·10-8 — постоянная, площадь S измеряется в квадратных метрах, температура T — в градусах Кельвина, а мощность P — в ваттах.

Известно, что некоторая звезда имеет площадь S = (1/228)·1020 м2, а излучаемая ею мощность P не менее 1,5625·1025 Вт. Определите наименьшую возможную температуру этой звезды. Ответ дайте в градусах Кельвина.
доведите плиз до формулы нахождения Т ну а там я сама уже посчитаю =)

1240. Александра, 30 апреля 2010, 13:45:16
оу.. и кому не трудно вот это задание попробывать решить... не люблю я задания с этим "е"
Вопрос B11 из 12
Найдите наименьшее значение функции y = (x–7)e в степени x-6 на отрезке [5;7].

1241. Александра, 30 апреля 2010, 13:48:37
ахахах прочитала свои записи.. мда.. фиг что поймешь((( вообщем в В7 там 3 в степени 9 , 2 в 6 и 6 в 5.
а в Б10 там вместо всей этой символики P= cST^4 где с=5,7·10-8

1242. Леонид , 30 апреля 2010, 14:15:33
1167. Azer, 27 апреля 2010, 16:37:01
Задание С1 в пробном егэ в Астрахани
I x+y I= 1 I--I - модуль
sinx+siny=2П/3
Вроде так!!

Вроде НЕ так. Посмотри, 2-е ур-е решений не имеет! Не может быть, в крайнем случае, больше или равно 2 (2пи/3).

1243. Лена, 30 апреля 2010, 15:30:47
Объем куба равен 27. Найдите площадь его поверхности.Объясните решение

1244. Артём, 30 апреля 2010, 17:10:32
Так как объём вычисляется перемножением длины, ширины и высоты, которые у куба равны, то нужно извлечь из 27-ми корень 3-й степени и умножить его на количество сторон.

1245. Наталья, 30 апреля 2010, 17:16:18
Лена, объем куба = а в кубе, где а - ребро. Тогда а^3= 27, а=3. Полная поверхность складывается из шести одинаковых квадратов площадью а^2 каждый. Значит площадь поверхности куба = 6* а^2 = 54

1246. Вадим, 30 апреля 2010, 18:00:59
Посмотрите вот этот пример:
13157 , по-моему там должен быть ответ: -0,5
если я не прав, напишите пожалуйста решение кто-нибудь!

1247. Иван, 30 апреля 2010, 18:27:47
№ 27111.Боковые ребра треугольной пирамиды взаимно перпендикулярны, каждое из них равно 3. Найдите объем пирамиды. Объясните

1248. Наталья, 30 апреля 2010, 18:58:01
Вадим, у меня тоже в номере 13157 ответ получился -0,5. Кстати, и по статистике ответов он тоже на первом месте, непонятно почему в итоге ответом признан -5,5

1249. Lexxus, 30 апреля 2010, 19:10:45
Вадим, у меня тоже в номере 13157 ответ получился -0,5. Кстати, и по статистике ответов он тоже на первом месте, непонятно почему в итоге ответом признан -5,5

Косяк исправлен, спасибо.

1250. Игорь, 30 апреля 2010, 22:51:19
Кто может помочь разобраться с С5 пример - http://www.mathege.ru:8080/diagnostics/ansege11maRF17022010_C.pdf
Понимаю как решается, не понимаю почему исключается промежуток из ответа( а за не исключение снимут два бала) вчем тут суть?
Разве на исключенном промежутке решение не отрезок????

1251. Вадим, 1 мая 2010, 11:01:42
Lexxus! убери пожалуйста те задачи из здешнего банка, которых теперь уже нет в MATHEGE.RU
Досрочный ЕГЭ(тот который сдавали спортсмены и т.п) показал , что все задания части B из открытого банка, а тут например в B4 заданий примерно 1200, а на Mathege.ru всего лишь около 600! Нет смысла тратить время на задачи , которые не будут на экзамене
P.S. помоги плиз не тратить время впустую

1252. EVANASEN, 1 мая 2010, 20:42:00
Проверьте,пожалуйста, B8 (№ 27490)
1+2+4+7+9+10+11=44

1253. мухаа, 1 мая 2010, 21:13:40
скажите пожалуйста, действительно ли на егэ будут встречаться задания из открытого банка задач?или будут не такие же, а только типовые?Оо

1254. Леонид , 1 мая 2010, 22:21:22
1252. EVANASEN, 1 мая 2010, 20:42:00. ДА! ВСЁ чётко!

1255. EVANASEN, 2 мая 2010, 10:49:01
1254. Леонид , 1 мая 2010, 22:21:22
Спасибо.

1256. tolerop, 2 мая 2010, 14:09:45
Пожалуйста, объясните, как решаются задачи такого типа: Дан график производной. Необходимо найти точки max или min функции на определенном отрезке. Когда на этом отрезке график производной пересекает ось Х, т.е. производная равна 0, мне всё понятно. Но есть задачи, где нужно найти экстремум на отрезке, где график производной НЕ пересекает ось Х. Ну никак не соображу...

1257. Леонид , 2 мая 2010, 15:55:50
1256. tolerop, 2 мая 2010, 14:09:45 "Дан график производной" - смотри, выше или ниже оси ОХ он расположен на указанном отрезке.
Ниже - произвдоная отрицательна, то функция убывающая: точка макс. в левой границе отрезка, миним.-в правой.
График производной выше ОХ-функция вшзрастающая, то миним. в левом конце отрезка, макс. - в правой.


1258. tolerop, 2 мая 2010, 17:43:17
Спасибо, Леонид! Всё ясно.

1259. Леонид , 2 мая 2010, 19:57:48
1164. funduk, 27 апреля 2010, 15:34:09
Системы:
I
1 2sqrt(2)*sin(x)+y=0
2 sqr(y)+3y+2*sqrt(sqr[y]+3y-6)=14
II
1 sin(x)*cos(y)=1/4
2 sin(y)*(cos(x)=-1/4
III
1 (6*sin(sqr[x])+7*sinx-5)*sqrt(-cos[x])=0
2 sin(sqr[x])=cos(sqr[y])

Друг, посмотри, всё ли верно напмсано? Что значит [x]? Это же не целая часть числа?. В третьей системе - 1-е ур. sin корень из х и sin х? Навереное (6*корень из СИН.х +7 СИН.х-5)* корень из(-КОС.х) =0. Жду! И др. С?

1260. Влад, 3 мая 2010, 12:55:18
Длина окружности основания конуса равна 3, образующая равна 2. Найдите площадь боковой поверхности конуса.Ответ 3 или 6.

1261. Ришка), 4 мая 2010, 16:13:04
Помогите пожалуйста (по теме комплексные числа)

изобразите на комплексной плоскости(график):
а) середину отрезка соединяющие точки 1+2i и 3+2i
б) множество точек Z удовлетворяющих условие - argZ=П/4
в) множество точек Z , удовлетворябщее условие |Z|<=3

пожалуйста помогите) очень надо....заранее спасибо...

1262. Lexxus, 4 мая 2010, 16:29:36
Помогите пожалуйста (по теме комплексные числа)

Как-то это слегка не по теме. Ну, ладно:

а) середину отрезка соединяющие точки 1+2i и 3+2i

Это горизонтальный отрезок длины 2. Его середина - точка 2+2i

б) множество точек Z удовлетворяющих условие - argZ=П/4

Все комплексные числа с одинаковым аргументом образуют на комплексной плоскости луч из начала координат с "выколотым" нулем (для комплексного нуля нет понятия аргумента).
В данном случае этот луч будет под углом 45 градусов (п/4) к горизонтальной оси (положительному направлению действительных чисел).

в) множество точек Z , удовлетворябщее условие |Z|<=3

Все комплексные числа с одинаковым модулем образуют окружность с центром в точке 0+0i и радиусом, равным данному модулю. Все точки с модулем, меньшим или равным данному, образуют круг, ограниченный такой окружностью.

1263. Ришка), 4 мая 2010, 18:25:53
Lexxus, спасибо, что помогли=))))

1264. Леонид , 4 мая 2010, 18:41:06
Боковая поверхность конуса - формула: pi*R*l = 1,5*2 = 3! Почему 6 то? Как в правильной пирамиде бковая поверхность равна половине произвеения периметра основания на апофему. ... длине ПОЛУОКРУЖНОСТИ...

1265. Lexxus, 4 мая 2010, 21:57:31
Боковая поверхность конуса - формула: pi*R*l = 1,5*2 = 3! Почему 6 то? Как в правильной пирамиде бковая поверхность равна половине произвеения периметра основания на апофему. ... длине ПОЛУОКРУЖНОСТИ...

М-да, что-то я не о том в том момент подумал O_o
Пользуясь служебным положением, спешно удаляю свой позоръ))

1266. oxotnik589, 5 мая 2010, 08:12:33
С2.. не могу решить задачу=(((
В кубе АВСД, А1 В1 С1 Д1, все рёбра равны 1. Найдите расстояние от точки С до прямой ВД1....
Я всё понял и кажеться решил почти.. но вот до прямой ВД1.. это на середину провести или как? решите плиз с объяснением.. и лучше на Электронный адрес [ kilroma@yandex.ru ] или в аську [351910331] P.S. мне 15... прошу помочь неучу) спс, за ранее...

1267. Lexxus, 5 мая 2010, 08:59:45
В кубе АВСД, А1 В1 С1 Д1, все рёбра равны 1. Найдите расстояние от точки С до прямой ВД1....

Вот в этом комментарии я решал практически такую же задачу, в точности до обозначений.
Там я находил, в каком отношении перпендикуляры, опущенные из вершин куба к его диагонали делят эту диагональ, а тебе нужно найти длину этих перпендикуляров (из тех же самых прямоугольных треугольников).
В ответе должно получиться sqrt(2)/sqrt(3).

1268. Михаил, 5 мая 2010, 11:58:47
Я слава богу разобрался с функциями, но некоторые задания меня всё равно тревожат....

Например: почему в "Задание B8 (8193) ", ответ: -9? у меня вышел -3, а -9 никак не выходит.

1269. Леонид , 5 мая 2010, 12:42:54
1268. Михаил. По графику произвоной - функция возрастает, где график производной строго выше ОХ. Считай: -6-3-2-1+3 целые точки. Итого (-9).

1270. николай, 5 мая 2010, 12:45:39
Для транспортировки 36 тонн груза на 500 км. можно использовать одного из трех перевозчиков. Причем у каждого из них своя грузоподъемность используемых автомобилей. Сколько рублей придется заплатить за самую дешевую перевозку за один рейс?
Перевозчик Стоимость перевозки одним автомобилем (р. на 100км)
Грузоподъемность автомобилей (тонн)
А 3200 р. 3,5 т
Б 4100 р. 5т
В 9500 р. 12 т

Ответ: 142500
Почему в задании спрашивается сколько нужно заплатить за самую дешевую перевозку за один рейс. а в ответах стоит полная стоимость самой дешевой перевозки?

1271. Михаил, 5 мая 2010, 13:06:52
Леонид, спасибо)

а вы бы не могли бы мне ещё помочь с нахождением минимум точек и максимум точек? как они определяются правильно?277

1272. Александр, 5 мая 2010, 15:44:13
ЕГЭ 2010. Помощь в сдаче экзаменов. Скачать вопросы и ответы.
Сайт - egego.tk

1273. Леонид , 5 мая 2010, 16:17:31
1271. Михаил, что за № 277 ?

1274. Леонид , 5 мая 2010, 16:23:51
1272. Александр! Ты, что из этой конторы, что деньги за брехню дерёт. Если нет, здесь достаточно заданий - тренируйся без рекламирования и рекомендаций ссылок на сомнительных материалов.

1275. юля, 5 мая 2010, 19:27:24
Обьясните как можно написать свой ответ к какому-либо заданию

1276. Михаил, 5 мая 2010, 19:50:56
Леонид, это просто опечатка)

1277. Михаил, 5 мая 2010, 19:55:19
Ребят, кто-н. знает, можно ли из формулы суммы ариф. прогрессии

Sn=[2a1 + d (n-1)]n/2.

Найти n, если известно а1 , d и Sn????

Как будет выглядеть формула нахождения n???


1278. ДИМА, 5 мая 2010, 20:03:52
На какой угол повернется отраженный от плоского зеркала луч, если зеркало повернуть на угол &#966;? Направление падающего луча остается неизменным.

решите плиз

1279. ксеня, 5 мая 2010, 20:35:33
здравствуйте, помогите пожалуйста такое решить Logx&#8331;&#8321; (X+2)&#8804;0.

1280. Леонид , 5 мая 2010, 21:18:22
1279. ксеня. ИЗОБРАЗИ ПОПРОЩЕ.


1277. Михаил. ДА. Подставляй данные в ту формулу, что записал а1, д и сумму, найёшь "н"

1281. ксеня, 5 мая 2010, 21:26:05
не знаю, как нормально написать) логарифм Х+2 по основанию Х-1 меньше или равно нулю... вот

1282. Михаил, 5 мая 2010, 22:42:41
1280. Леонид , 5 мая 2010, 21:18:22

Не получается у меня найти n, я уж всё перепробовал, ну никак. Причём я знаю. чему равно n, но оно никак не выходит. А как формула будет выглядеть?

1283. Леонид , 6 мая 2010, 07:54:44
1277. Михаил. Где твои данные? Напиши полное условие. Как показать решение, если написана только формула и сказано о том, что дано. По твоим записям подставляй данные и все дела

1284. Михаил, 6 мая 2010, 10:56:52
всё Леонид, спасибо, я нашёл. Сегодня с утра встал и так легко решил, видно просто вчера перегрелся)))



1285. Михаил, 6 мая 2010, 11:25:38
Ребят, а как можно без калькулятора оперативно решить вот такого рода кв. уравнения:

15x2 +13x - 1630 = 0

там D=97969^12



1286. Михаил, 6 мая 2010, 12:19:28
* D=97969^1/2

1287. Игорь, 6 мая 2010, 12:30:57
По идее нужно воспользоваться обратной теоремой виета
ах^2+вх+с=0
х1+х2= -в/а
х1*х2=с/а
таким образом
х1+х2= -(13/15)
х1*х2 = -1630/15 = -108,66666- тут только один возможный делитель без дробной части 10 и с дробной -10,8666 ,
а откуда такое хитрое уравнение получилось?

1288. Михаил, 6 мая 2010, 13:13:21
Игорь, это из С6. Тема: ариф. прогрессия.

а обратная теорема Виета, какая-то мудрённая)))

а как понять, что здесь надо воспользоваться именно ей?

1289. Михаил, 6 мая 2010, 13:18:15
о, а ещё ведь это сложно просчитать х1 и х2, зная левую часть, здесь так же надо решать кв. ура. исходя из системы. ил можно по другому?

1290. регишаааа), 6 мая 2010, 13:30:17
я ваще не понимаю зачем придумали ЕГЭ!!!!вот я возьму и не пойду его сдавать!!!и ВАМ советую!!!

1291. Леонид , 6 мая 2010, 13:53:48
1289. Михаил. Напиши всё задание. КВ. ур-е: считай Д, подкоренное выражение разложи на простые множители и будет видно, что можно вынести из под корня. В ответе будет (видимо) запись с корнями. Предложение о т. Виета в данном случае твоей проблемы не решает. Возьми за правило задание писать полностью, а не выписки из своего решения. Вдруг, тобой была сделана ошибка.

1292. Михаил, 6 мая 2010, 14:15:01
Хорошо, Леонид, я буду писать задания полностью. Это просто громоздкое, а кв. уравнение лишь малая часть. Я его решил, но с помощью калькулятора. Но ведь калькулятором нельзя пользоваться на ЕГЭ, поэтому и спрашиваю, если попадётся такого рода уравнения, то как решать. Сейчас сижу, пытаюсь разложить число 97969 на множители, но это тоже не просто.

А задание, такое С6

"Последние члены двух конечных арифметических прогрессий
а1=5, а2=8, ..., aN и в1=9, в2=14, ..., вM
совпадают, а сумма всех совпадающих (взятых по одному разу) членов этих прогрессий равна 815. Найди число членов в каждой прогрессии".

Я это задание 2 дня решал)))





1293. Михаил, 6 мая 2010, 15:10:05
число 97969= 313 х 313... попробуй тут подобрать, свихнёшься(((

1294. вика, 6 мая 2010, 17:23:12
помогите плиз решить

Сколько целых чисел входит в решение системы неравенств
(x+2)(2-x)<(x+3)(4-x)
{
3+x/4 + 1-2x/6 > или равно 1

1295. витя, 6 мая 2010, 18:01:20
В треугольнике ABC угол C равен 90^circ, AB = 4 sqrt{15}, sin A = 0,25. Найдите высоту CH. Объясните решение.

1296. Алина, 6 мая 2010, 21:57:41
ПОМОГИТЕ РЕШИТЬ С2!!умоляю!
Диагональ A1C куба АВСВDA1B1C1D1 служит ребром двугранного угла,грани которого проходят через вершины B и D. Найдите величину этого угла.

1297. мария, 6 мая 2010, 22:16:52
Сколько целых чисел находится на числовой оси на расстоянии, не превосходящем 17 от точки, соответствующей числу 4,2?

1298. Алексей, 7 мая 2010, 13:53:48
Решите пожалуйста:
кореньCOSy корень6x-x в квадрате-8=0
Система
кореньSiNx корень2-y-y в квадрате=0

_______________________________________-
В кубе ABCDA1B1C1D1 все рёбра равны 1.Найдите расстояние от точки С до прямой BD1


С ув уч. 11 класса...

1299. Lexxus, 7 мая 2010, 14:13:09
Сколько целых чисел входит в решение системы неравенств
(x+2)(2-x)<(x+3)(4-x)
{
3+x/4 + 1-2x/6 > или равно 1

Насколько я понял, второе уравнение должно выглядеть вот так:
(3+x)/4 + (1-2x)/6 >= 1
а то как-то неуклюже.

Раскрываем скобки в первом неравенстве, приводим подобные, остается
x > -8
Во втором неравенстве умножаем обе части на 12, приводим подобные, остается
x <= -1

И тому, и другому, удовлетворяют целые числа -7,-6,-5,-4,-3,-2 и -1.
Всего семь.

1300. Lexxus, 7 мая 2010, 14:57:58
Диагональ A1C куба АВСВDA1B1C1D1 служит ребром двугранного угла,грани которого проходят через вершины B и D. Найдите величину этого угла.

Для начала, предположим, что ребро куба равно 1. Диагональ куба тогда равна sqrt(3).
Рассмотрим треугольники A1CB и A1CD. Они равны по трем сторонам.
Опустим перпендикуляры к A1C из точек B и D. Раз треугольники равны, перпендикуляры тоже равны и попадают в одну точку H.



Таким образом, нам нужно найти величину угла BHD.

Для этого найдем длины BH и DH.
В прямоугольном треугольнике CHB CH = A1C/3 = sqrt(3)/3 (вот здесь это доказывается), CB = 1.
Значит, BD = sqrt(2)/sqrt(3).

Теперь в треугольнике BHD мы знаем все стороны и можем найти угол BHD по теореме косинусов.
Получится cos(BHD) = -1/2, т.е. BHD = 120 градусов.

1301. Михаил, 7 мая 2010, 15:44:28
в Задание B6 (22679) сказано,

Найдите площадь трапеции, вершины которой имеют координаты (1;2), (10;2), (10;7), (1;7). И изображён рисунок прямоугольника. Разницы нет, какой формулой нахождения S пользоваться, в обоих случаях будет одинаковое число. Но ведь прямоугольник, не является трапецией. Трапеция, это то, где одна из пар противолежащих сторон ||, а одна нет. Почему здесь сказана именно про трапецию?

1302. KLASS, 7 мая 2010, 16:10:16
1290. регишаааа), 6 мая 2010, 13:30:17
я ваще не понимаю зачем придумали ЕГЭ!!!!вот я возьму и не пойду его сдавать!!!и ВАМ советую!!!


ну ты и приколистка хахахахахахаха насильно тебя поведут

1303. toyzy, 7 мая 2010, 16:25:01
На рисунке изображен график функции y=f(x), определенной на интервале (-10; 2). Найдите количество точек, в которых касательная к графику функции параллельна прямой y=20.

я не очень понимаю смысл задания, объясните пожалуйста

1304. KLASS, 7 мая 2010, 16:33:32
посмотри запись № 1180 Lexxus объяснил на графике сразу понятно все станет

1305. Леонид , 7 мая 2010, 16:34:44
1301. Михаил, 7 мая 2010, 15:44:28. Ты прав. Но по формуле площади трапеции можно находить и площадь треугольника - как трапеции с одним из оснований равным 0 (имей ввиду, вдруг пригодится). Вообще, считаю это безобразием, такие задания на общую массу выпускников.


1298. Алексей, 7 мая 2010, 13:53:48
Решите пожалуйста:
кореньCOSy корень6x-x в квадрате-8=0
Система
кореньSiNx корень2-y-y в квадрате=0

Леша, что между корнями, что под корнями? Продублируй лучше.Это задание С1. Селай. Помогу. Откуда пример и твой регион? ПОЖАЛУЙСТА!

1306. vika-sitnikova, 7 мая 2010, 18:18:52
Здравствуйте, Lexxus.
Имеются ли у вас задания части С?
Если нет, не могли бы вы порекомендовать какой-нибудь сайт?

1307. EVANASEN, 7 мая 2010, 19:01:08
Помогите пожалуйста с решением B7 (№ 26840)

1308. Леонид , 7 мая 2010, 19:04:37
1306. vika-sitnikova, 7 мая 2010, 18:18:52

Листай здесь. Задач С много. Напиши свои С с пробнрго. И регион ваш.

1309. Надюшка, 8 мая 2010, 21:05:34
этот сайт очень полезный)

1310. Надюшка, 8 мая 2010, 21:14:22
помогите, пожалуйста, с С1(решить систему):3sinx +5cosy=5 и 4sinx-cosy =-1!!! C3 (решить неравенство)под кв.корнем 16-x^2*(4x^2+5x+2)>больше - либо равно нулю!

1311. Аня, 8 мая 2010, 22:49:55
умоляю ПОМОЧЬ РЕШИТЬ! в правильной четырехугольной пирамиде сторона основания равна высоте и равна 4. найти расстояние (р)от вершины основания до плоскости диагонального сечения,не проходящего через эту вершину. в ответе запишите р&#8730;2/2

1312. Аня, 8 мая 2010, 22:51:27
умоляю ПОМОЧЬ РЕШИТЬ! в правильной четырехугольной пирамиде сторона основания равна высоте и равна 4. найти расстояние (р)от вершины основания до плоскости диагонального сечения,не проходящего через эту вершину. в ответе запишите р корень из/2

1313. Аня, 8 мая 2010, 22:52:57
в ответе запишите р корень из 2/2

1314. Леонид , 8 мая 2010, 22:56:28
1310. НАДЮШКА ! 8 мая 2010, 21:14:22

Откуда эти задания? Какой регион (простые)? Через полчасика помогу, подумав как лучше написать! ДА! В С3 квадратный трёхчлен под корнем или корень умножен на него?

1315. Надюшка, 8 мая 2010, 23:59:15
Воронежский, это задания из ЕГЭ, которые учительница нам теперь даёт как контрольную, под корнем только 16-x^2, который умножен на скобку(4x^2+5x+2)и т.д.

1316. Леонид , 9 мая 2010, 09:16:21
1310. Надюшка, Для ЕГЭ очень просты.
1)Решай как систему линейных уравнений в 7 классе. Например, обечасти второго уравнения умножь на 5 и сложи с 1-м ур-ем, получим: 23 СИН.х =0, СИН.х=0, х=пи.К. Далее можно обе части 1-го у. * на (-4), 2-го на 3 и сложим полученные уравнения, получим: -23КОС.х=-23, КОС.х=1, х=2пи.К (ответ: пи.К;2пи.К)
2)Корень неотрицательны на своей ОДЗ: 16-х в кв. больше или = 0. х в кв. меньше или равно 16,т.е. хпринадлежит [-4;4].
Произведение неотрицательно,если 4x^2+5x+2 больше или =0. Д=25-32 больше нуля-корней нет, ветви параболы вверх - кв. трёхчлен положительный при всех Х. Ответ: [-4;4]. Напиши, что было на пробном С1-С4.

1317. Алексей, 9 мая 2010, 09:22:58
Решите пожалуйста:
|COSy| |6x-x в квадрате-8|=0
Система
|SiNx| |2-y-y в квадрате|=0

_______________________________________-
В кубе ABCDA1B1C1D1 все рёбра равны 1.Найдите расстояние от точки С до прямой BD1


С ув уч. 11 класса...

|-нач корня |-конец корня


1305. Леонид Я живу в Белгороде задание дола учительница но я успел скатать и откуда оно(оно было с пробника):динамические работы по математике 17 февраля 2010г 5 В

1318. Алексей, 9 мая 2010, 09:23:55
Дала*

1319. Вадим, 9 мая 2010, 11:34:54
6955 ответ неверный исправьте пожалуйста!

1320. Леонид , 9 мая 2010, 13:01:33
1317. Алексей.

2) ВД1 - диагональ куба, равна корню из 3, по . в прямоугольном параллелепипеде кв. диагонали равен сумме кв. его измерений
ВД1 в кв.=1+1+1 . Расстояние от С до ВД1 - это высота в прямоугольном треугольнике ВД1С из вершины прямого угла - это СН - её и надо найти. Используй, например формулы площаи 1/2 ВД1 *СН = 1/2 ВС*СД1, поставля и все дела (СД1= корню из 2): СН = (ВС*СД1):ВД1 = (1*корень из 2): на корень из 3 = корень из (2/3). И всё.

1) ЭТО произведения корней, я правиьно понял?

1321. Леонид , 9 мая 2010, 14:50:14
1319. Вадим. Какой твой ответ? Ответ: 3

1322. лиля, 9 мая 2010, 16:21:58
Здравствуйте, умные люди, решите пожалуйста эту задач!!! в треугольнике АВС уголА= альфа, сторона ВС=a, Н-точка пересечения высот. найдите радиус окружности, описанной около треугольника ВНС.

1323. Одиночка, 9 мая 2010, 17:39:55
Приветствую всех!
Есть мелочный вопрос:
задание В7 из математики: корень из(116^2-80^2)=
там ответ вычисляется только методом подбора чисел (нужно извлечь корень из 7056) или есть метод разложения?
Заранее спасибо!

1324. Леонид , 9 мая 2010, 19:10:07
Одиночка, распиши по корнем (116-80)(116+80)= 36*196. Извлекаем: 6*14=84. Так решаются все примеры такого типа из банка ЕГЭ. А ты панику развоишь! Будешь подбирать,сколько надо времени? Вы, что в школе этого не делаете?

1325. nelya, 9 мая 2010, 23:35:30
Из пункта А в пункт В, расстояние между которыми 60 км, одновременно выехали автомобилист и велосипедист. Известно, что в час автомобилист проезжает на 110 км больше, чем велосипедист. Определите скорость велосипедиста, если известно, что он прибыл в пункт В на 5,5 часов позже автомобилиста.

Помогите,пожалуйста решить!

В цилиндре диогональ осевого сечения образует с основанием угол 63 град. , образующая 15 см. найти площадь сечения и площадь основание

1327. Екатерина, 10 мая 2010, 11:52:21
Увеличив среднюю скорость с 250 до 300 м/мин, спортсменка стала пробегать дистанцию на 1 мин быстрее. Какова длина дистанции?

1328. вадим, 10 мая 2010, 16:09:08
Боковые ребра треугольной пирамиды взаимно перпендикулярны, каждое из них равно 3. Найдите объем пирамиды. Нужно решение

1329. Алексей, 10 мая 2010, 16:40:31
1320. Леонид

да произведение корней

1330. Елена, 10 мая 2010, 17:29:04
Задание B9 (№ 27140)
Площадь поверхности куба равна 8. Найдите его диагональ.

1331. MSever, 10 мая 2010, 18:47:11
1330. Елена
В кубе d^2= 3a^2
S поверхности = 6a^2=8
значит, d^2=3a^2=4, т. е. d=2.

1332. bel_julia, 10 мая 2010, 19:28:52
Задание B9 (25649) - ответ неверный!
В задании - найти площадь поверхности, а ответ найден для объема. Исправьте, пожалуйста!

1333. cxem, 10 мая 2010, 20:42:00
Добавьте ещё тесты 2010 года!))

1334. bel_julia, 10 мая 2010, 20:53:39
Задание B9 (4969) - ответ неверный! Забыли поделить найденную гипотенузу на 2, чтобы получить радиус. В результате получится в четыре раза меньший объем. Интересно, что большинство ответило правильно, а ответ все-равно указан неверный.

1335. аня, 10 мая 2010, 21:04:02
умоляю ПОМОЧЬ РЕШИТЬ! в правильной четырехугольной пирамиде сторона основания равна высоте и равна 4. найти расстояние (р)от вершины основания до плоскости диагонального сечения,не проходящего через эту вершину. в ответе запишите р корень из2/2

1336. Леонид , 10 мая 2010, 21:15:07
1334. bel_julia. Не знаю какой у тебя ответ. Почему не написала? ОТВЕТ:7,5

1335. аня, Это половина диагонали основания (квадрата), т.к. диагональ квадрата перпендикулярна другой его диагонали и высоте пирамиды (т.е.) твоему сечению. ОТВЕТ: 2*(корень из 2).

1337. Марина, 10 мая 2010, 22:34:24
биссектриса угла а прямоугольника АВСД пересекает сторону ВС в точке М,а биссектриса угла С пересекает сторону АД в точке К.Докажите,что АВМ равнобедренный.АВМ=СДК

1338. юлька, 11 мая 2010, 14:26:07
дайте ответы по рус

1339. юлька, 11 мая 2010, 14:27:57
какие тесты лучше прорешать у меня переводной

1340. Сергей, 11 мая 2010, 17:48:31
помогаю БЕСПЛАТНО решить ЕГЭ
пишите на мыло eofjapan@rambler.ru

1341. Безымянный, 11 мая 2010, 21:56:18
Можно было бы писать комментарии к заданиям чтоле, обсуждали бы решение заданий, а то некоторые сложные все таки попадаются и ответа для понятия решения недостаточно

1342. drugoy10, 11 мая 2010, 22:07:09
Задание B4 (4673) - по-моему, не правильный ответ. Найдена сторона ВС, а спрашивают про АС

1343. Леонид , 12 мая 2010, 07:52:09
Ты пиши что олучил без кажется. Ответ здесь 32

1344. Вадим, 12 мая 2010, 08:24:42
25649
Неправильный ответ! Там никак не может быть 16!

1345. Lexxus, 12 мая 2010, 08:49:24
25649
Неправильный ответ! Там никак не может быть 16!

А сколько же там, по твоему, может быть?

1346. Wo3G, 12 мая 2010, 11:16:59
54 там, потому что 16-объем, а вопрос - площадь поверхности

1347. Lexxus, 12 мая 2010, 11:32:56
54 там, потому что 16-объем, а вопрос - площадь поверхности

Да, действительно.
Вот так, на элементарщине, можно потерять ценные баллы.

1348. Михаил, 12 мая 2010, 11:41:13
Ребят, не могу никак решить одно задание.
Дан конус у которого V=144, этот конус рассекает сечение || основанию, найти объём конуса, у которого основанием является то самое сечение.
Я пытался из 144 вычесть V ус. конуса, но у меня ничего не получилось.
кто-н. знает как решить?

1349. Марина, 12 мая 2010, 16:22:43
Помогите с одним заданием....(до вечера пожалуйста)

Найдите множество значений функции у=arctg(корень из(2) + sinX-cosX/2*корень из(2))..

1350. Катеринка, 12 мая 2010, 18:44:24
О, да......... ЕГЭ можно обсуждать бесконечно.................

соглашусь, что очень много задач вообще, СОВЕРШЕННО некорректны, как ни печально..........
они даже не знания проверяют, порой, а внимательность. Из-за этого столько ошибок иногда делаю, ТУПЕЙШИХ на самом деле, что аж обидно до слез........


Ну ничего, ТОВАРИЩИ))))) ЕГЭ - это еще не конец жизни.......... надеюсь)))))))
Всем: ни пуха, ни пера!!!!!)

1351. дима, 12 мая 2010, 18:57:34
26858 какой ответ ?

1352. Leisira, 12 мая 2010, 20:03:37
к чертуууу:))*****

1353. funduk, 12 мая 2010, 20:13:00
Кто-нибудь знает,изменения есть какие-либо в структуре ЕГЭ по матиматике или вокруг просто так орут??

1354. funduk, 12 мая 2010, 20:48:21
Задание В8(8129) почему ответ 14 должен быть??Целые точки - 4,5,6 -, их сумма равна 15.

1355. Леонид , 12 мая 2010, 21:15:10
1164. funduk, 27 апреля 2010, 15:34:09
Системы:
I
1 2sqrt(2)*sin(x)+y=0
2 sqr(y)+3y+2*sqrt(sqr[y]+3y-6)=14
II
1 sin(x)*cos(y)=1/4
2 sin(y)*(cos(x)=-1/4
III
1 (6*sin(sqr[x])+7*sinx-5)*sqrt(-cos[x])=0
2 sin(sqr[x])=cos(sqr[y])

Друг, посмотри, всё ли верно напмсано? Что значит [x]? Это же не целая часть числа?. В третьей системе - 1-е ур. sin корень из х и sin х? Наверное (6*корень из СИН.х +7 СИН.х-5)* корень из(-КОС.х) =0. Жду! И др. С?

Я просил тебя - что молчишь? Давай! А я тебе смотрю по записи № 1354

1356. Стеничева Анна, 12 мая 2010, 22:06:09
!

1357. Леонид , 13 мая 2010, 07:05:07
lexxus, почемуто 2-ой день не открывается открытый банк задач...

1358. Николай, 13 мая 2010, 09:26:28
Найдите наименьшее значение функции y=(1/3)в степени -1-хквадрат

1359. ЯрусскиЙ, 13 мая 2010, 10:35:50
Первая труба пропускает на 3 литра воды в минуту меньше,чем вторая. Сколько литров воды в минуту пропускает первая труба,если резервуар объемом 270 литров заполняет на 3 минуты дольше,чем вторая труба

1360. dimonez, 13 мая 2010, 12:03:06
Найти наименьшее значение функции y= 8tgx -8x-2Пи+5 на отрезке [-Пи/4;П/]
скажите ответ , пожалуста,срочно!!!!!

1361. funduk.funduk@ya.ru, 13 мая 2010, 16:28:24
Это я скобки поставил чтобы не перепутать с двойными=).Просто привык уже так вводить.

1362. Леонид , 13 мая 2010, 16:57:46
"1354. funduk, 12 мая 2010, 20:48:21
Задание В8(8129) почему ответ 14 должен быть??Целые точки - 4,5,6 -, их сумма равна 15".

Точки: -1,4,5,6 в сумме 14. Ты левую целую точку не увидел - график производной в ней тоже ниже ХО. Ведь так?

А третью систему попробуй поновой записать. Что значит "с двойными =)" не перепутать.

1363. катя, 13 мая 2010, 18:54:53
В треугольнике ABC угол C равен , , . Найдите косинус внешнего угла при вершине A.

1364. катя, 13 мая 2010, 19:02:05
В треугольнике ABC угол C равен90 градусов. аб=15,вс=9.Найдите косинус внешнего угла при вершине A.

1365. Leisira, 13 мая 2010, 19:06:31
dimonez_у мя вот такой ответ на в11.ответ:13:))

1366. funduk, 13 мая 2010, 19:40:32
Опа! 3 система не точно переписал.Сори=)
I. 6*sin(в кв.)х+7*sinx-5)*sqrt(-cosx)=0
II. sin(в кв.)x=cos(в кв.)х

1367. Leisira, 13 мая 2010, 19:42:07
катя у мя в в4 ответ 1.3 получилось)))

1368. катя, 13 мая 2010, 21:06:40
объясните пожалуйста как находить косинус внешнего угла в треугольнике

1369. Леонид , 13 мая 2010, 22:20:10
1) 1366. FUNDIK, 13 мая 2010, 19:40:32
Опа! 3 система не точно переписал.Сори=)
I. 6*sin(в кв.)х+7*sinx-5)*sqrt(-cosx)=0
II. sin(в кв.)x=cos(в кв.)х

Молодец! Спасибо. Один вопросик: Перед 6 круглая скобка? А в В8 том, уловил? (-1)?

2) 1364. КАТЯ, 13 мая 2010, 19:02:05
В треугольнике ABC угол C равен90 градусов. аб=15,вс=9.Найдите косинус внешнего угла при вершине A.

Найди по т. Пифагора АС= 12, КОС. уг.А = 12/15. Внешний КОС.(180-А)=-КОС А = -12/15 = -4/5=-0,8.

1370. Леонид , 13 мая 2010, 23:00:32
1) 1366. FUNDIK, 13 мая 2010, 19:40:32
Опа! 3 система не точно переписал.Сори=)
I. 6*sin(в кв.)х+7*sinx-5)*sqrt(-cosx)=0
II. sin(в кв.)x=cos(в кв.)х


перехвалил тебя, друг. ты почему такой несерьёзный? Смотри, а "У" где?

1371. Вадим, 14 мая 2010, 08:53:20
C6) Решите в натуральных числах уравнение n ! + 5n + 13 = k2(к в кв.)
где n!=1•2•…•n – произведение всех натуральных чисел от 1 до n.
помогите пожалуйста с решением попроще и по понятнее

1372. funduk, 14 мая 2010, 11:48:44
В8 я понял.Спс.
во второй системе
sin(в кв.)x=cos(в кв.)y

1373. Леонид , 14 мая 2010, 14:31:11
1372. funduk, - это окончательно, надеюсь? Ты видимо парень хороший, но что такой невнимательно-скоростной. Это тебе мешает. Мужик д.б. степеным, надёжным. Ну давай, теперь молодец. А что в ставрополье с С2 и С4? У вас теплотища. Учиться уже неохото.

Обращайся. Смогу - помогу!

1374. funduk, 14 мая 2010, 16:30:38
С2,С4 нет у меня-не решал.
Так что, не будет заданий С5,С6?Сегодня уже решал вариант без них.
Или всё же на ЕГЭ они будут??

1375. Фантик, 14 мая 2010, 17:31:46
ДАЙТЕ ПОЖАЛУйСТА САЙТЫ ГДЕ ЧАСТЬ "С" С РЕШЕНИЯМИ

1376. Влад, 14 мая 2010, 17:57:12
В треугольнике ABC угол C равен 90^circ, AB = 8, sin A = 0,5. Найдите BC. Ответ:16 или 4

1377. Леонид , 14 мая 2010, 19:24:30
ВЛАДУ.ВС/8 =0,5, ВС=4


1374. funduk. БУДУТ, это точно.

1378. funduk, 14 мая 2010, 19:46:05
Просьба!
Кто достаточно прорешал задания С5,С6, написать, что надо уметь делать.=)
Вроде как С1 - уметь делать замену...

1379. funduk, 14 мая 2010, 19:51:53
А есть что-нибудь подобное этому сайту,только по др. предметам-физика и инф.??

1380. Леонид , 14 мая 2010, 23:18:41
1378. funduk. С5 и 6 непредсказуемы,там может быть что угодно.

1381. Ди, 15 мая 2010, 01:36:24
в этот четверг миновал пробный егэ (10 класс)
Заинтересовало задание С-3,как не мучилась,не смогла одолеть его, но хотелось бы узнать решение для себя.
Собственно,вот само задание:
Log3(x^2-x-3)+Log3(2x^2+x+3)=>Log3(x^2-2)^2+2+Log1/3(4)

вторую часть преобразовала в Log3(2,25(x^2-2)^2)
затем не знала что сделать,пыталась и скобки раскрыть,но выходила бессмыслица..(получаются третьи,четвертые степени)

Рассчитываю на совместные усилия.

1382. Леонид , 15 мая 2010, 06:14:20
ДИ (сходу не прописывая), левую часть запиши как логарифм произведения кв. трёхчленов. имеем основание 3 больше 1, функция взрстает, запишем н-во без лог. не меняя знака. далее многочлнн 4-й ст....

Какой регион? Почему так поздно пробный? По России он прошёл месяц назад. Всё ли верно записано в условии - посмотри внимательно!!! Может быть это задание просто учительница предложила? И что за ЕГЭ в 10 классе? Что - то у тебя не то!!!

1383. Ди, 15 мая 2010, 18:34:13
Леонид,я из Москвы.
Я немного не точно выразилась, у нас это была переводная контрольная , но ввиде ЕГЭ,варианты нам были присланы в школу.(как нам сказали все школы это пишут)
Тогда уточню ещё раз задание и здесь отпишусь.

1384. Мучительница, 15 мая 2010, 20:17:52
log3(x^2-x-3)(2x^2+x-3)>=log3(x^2-2)^2+2+log1/3(4)
x^2-x-3>0

4(x^2-x-3)(2x^2+2-3)>=(3x^2-6)^2
x^2-x-3>0
3x^2-6#0

3x^2-6=(x^2-x-3)+(2x^2+x-3)

Пусть a=x^2-x-3, b=2x^2+x-3. Первое неравенство системы принимает вид 4ab>=(a+b)^2, то a^2-2ab+b^2<=0, (a-b)^2<=0, а это возможно, только если
а=b.
Таким образом, x^2-x-3=2x^2+x-3, x^2+2x=0, x=0 или x=-2.
Условиям системы удовлетворяет x=-2

1385. fundukk, 15 мая 2010, 20:21:43
Задание В8(19967) у меня не совпадает с ответом.Т.е у вас не совпадает с моим=).
У меня получилось 12.5 .

1386. Мучительница, 15 мая 2010, 20:23:38
Ди предыдущее послание для тебяю Это по поводу неравенства с логарифмами из работы для 10 класса , которую писали 13 мая

1387. Любаша, 15 мая 2010, 20:47:55
Первая труба пропускает на 3 литра воды в минуту меньше,чем вторая. Сколько литров воды в минуту пропускает первая труба,если резервуар объемом 270 литров заполняет на 3 минуты дольше,чем вторая труба помогите решить

1388. катя, 15 мая 2010, 21:35:52
подскажите сайт где можно порешать задания с.

1389. Леонид , 15 мая 2010, 22:29:08
1385. fundukk, 15 мая 2010, 20:21:43.
Найдём СИН. А (ч/з триг. 1) = (2корня из 13)/13, то Тг.А = 2/3. По опред. Тг. в треуг. АСН (СН - высота) СН = 29/3 - ответ не идёт в формат ЕГЭ.

Вопрос к Lexxus.

1390. Леонид , 15 мая 2010, 22:38:11
1384. Мучительница, 15 мая 2010, 20:17:52. ЭТО ВАШЕ
"log3(x^2-x-3)(2x^2+x-3)>=log3(x^2-2)^2+2+log1/3(4)

4(x^2-x-3)(2x^2+2-3)>=(3x^2-6)^2". Куда делось +2 в правой части?


1391. Ди, 15 мая 2010, 22:45:55
Мучительница,спасибо,что отклинулись.
вот не поняла несколько моментов:
почему 3x^2-6#0,а не (3x^2-6)^2>0

откуда такое выражение?как вы его получили из выше написанного?
3x^2-6=(x^2-x-3)+(2x^2+x-3)

и ещё у вас опечатка со знаком: (2x^2+x+3),а не (2x^2+x-3)


1392. Ди, 15 мая 2010, 22:57:09
Леонид , я так поняла.
она сделала так...
log3(x^2-2)^2+2=2Log3(x^2-2)+2Log3(3)=2Log3((x^2-2)3)=Log3(3x^2-6)^2

1393. Ди, 15 мая 2010, 23:05:56
Любаша,простая задачка
1ая труба - скорость Х литр/мин
тогда 2ая труба - х+3 литр/мин

далее составляем уравнение
270/x-270/(x+3)=3
затем уже думаю дальнейший ход писать не надо
ответ у меня получился 15 литров в минуту

Я в шоке!У нас в школе сказали,что часть В принимают только как зачет,т.е. за него баллы не будут ставить.Баллы проставляются только за часть С(

1395. Lexxus, 15 мая 2010, 23:47:24
Я в шоке!У нас в школе сказали,что часть В принимают только как зачет,т.е. за него баллы не будут ставить.Баллы проставляются только за часть С(

Это неправда.

1396. ZajkaVseznaika, 16 мая 2010, 10:53:44
б7 какое кол-во вариантов заданий,пока сидела насчитала 26,при том что 2 из них на пробном мы решали в б3,это задание проще,чем некоторые очень сильно навороченные где корень на корне,присутствует танген и степени в корнях в одном выражении.
б3 сравните почти любое б3 с сайта и книжки с фипи,миоо,задания в книжках гораздо легче,чем те,которые были на пробном и есть на сайте.
б4 разнообразился недавно очень сильно,буквально как неделю,раньше нужно было знать треугольники,синусы,косинусы,тангенсы,катангенсы в худшем случае сторона содержала корень,теперь же такое кол-во фигур прорешать и как-то подготовиться по нововведениям невозможно просто.
б9 раньше это были как-правило параллелограммы и кубики,которые было считать реально,если захотеть,теперь же в описанные и вписанные фигуры,конусы,пирамиды,сферы,цилиндры.это сложнее гораздо.
б10 физические задачи.оставили бы одну среднюю скорость,прорешать все задачи,которые возможны в курсе физики нереально.+это егэ по математике,так каким же образом туда затесалась физика,а не литература например?может я задание по литературе увидеть хочу в б10,если уж протискивается другой предмет.
б8 как и в б7 очень много типов заданий,слишком много.
в прошлом году задания в части а мало того что с вариантами ответов были,так еще и проще раз в 5,чем у нас без вариантов ответов.мы козлы отпущения получается?сколько можно над нами издеваться?

1397. Леонид , 16 мая 2010, 14:16:18
Ни к чему этот никому ненужный, если так можно сказать, анализ содержания части В ЕГЭ 2010. Что явно не помогает подготовке. Все добавки, о которых пишите, здесь уже с декабря.

1398. муха, 16 мая 2010, 14:37:48

2синус124*косинус124/синус248

как такое решить?

1399. алёна, 16 мая 2010, 14:50:44
помогите пожалуйста решить задачу в12 ничего в них не понимаю((((
первый рабочий за час делает на 3 детали больше,чем второй,и заканчивает работу над заказом,состоящим из 340 деталей, на три часа раньше ,чем второй рабочий,выполняющий такой же заказ.сколько деталей в час делает второй рабочий?

1400. vika_sitnikova, 16 мая 2010, 17:13:30
Здравствуйте!
Помогите плиз решить В12.
Брюки дороже рубашки на 20% и дешевле пиджака на 46%. На сколько процентов рубашка дешевле пиджака?

муха,твое задание решается очень просто:
2синус124*косинус124/синус248=2синус124*косинус124/синус2*124=2синус124*косинус124/2синус124*косинус124=1

нижнюю часть уравнения расписываешь,как формулу двойного угла.

1402. Леонид , 16 мая 2010, 18:04:08
1401. безнадежный абитуриент, 16 мая 2010, 17:22:24

Что-то не понял, что написал. В числителе сверни по формуле
синус 248/синус 248 =1.

1400. vika_sitnikova, 16 мая 2010, 17:13:30. Смотри где-то выше. Уже надоела эта задача про штаны.

1399. алёна, 16 мая 2010, 14:50:44

СМОТРИ ЗДЕСЬ. ЭТО одно и тоже! 1393. Ди, 15 мая 2010, 23:05:56.
Просматривайте советы других.

Леонид,если свернуть,то ответ тоже самое получится(так даже проще,я не сразу обратила внимание)...
Просто я в знаменателе синус248 разложила как синус 124*2. И по формуле двойного угла синус2альфа=2синусальфа*косинус альфа разложила.

1404. Леонид , 16 мая 2010, 22:54:41
1401. безнадежный абитуриент, 16 мая 2010, 17:22:24
муха,твое задание решается очень просто:
2синус124*косинус124/синус248=2синус124*косинус124/синус2*124=2синус124*косинус124/2синус124*косинус124=1

нижнюю часть уравнения расписываешь,как формулу двойного угла.


Почитай как напмсано, я просто поправки в нёс. Тебе нравится (виднее) знаменатель расписать - делай. Но тобою не дан ответ и где там хоть одно уравнение. Формулы.... Да?

1405. Дарья, 16 мая 2010, 23:48:33
Помогите пожалуйста!

В правильной шестиугольной призме ABCDEF1B1C1D1E1F1,все ребра в которой равны 1,найдите расстояние от точки С до прямой FF1.

1404. Леонид , 16 мая 2010, 22:54:41
1401. безнадежный абитуриент, 16 мая 2010, 17:22:24
муха,твое задание решается очень просто:
2синус124*косинус124/синус248=2синус124*косинус124/синус2*124=2синус124*косинус124/2синус124*косинус124=1

нижнюю часть уравнения расписываешь,как формулу двойного угла.


Почитай как напмсано, я просто поправки в нёс. Тебе нравится (виднее) знаменатель расписать - делай. Но тобою не дан ответ и где там хоть одно уравнение. Формулы.... Да?

Ты не увидел ответ?!а написано 1!Ну формулу двойного угла любой ученик,наверно,должен знать наизусть......

1407. Мучительница, 17 мая 2010, 01:05:02
Дарья кажется совсем просто
расстояние равно двум радиусам описанной окружности, 6-уголник правильный, то две стороны. ответ 2
если не так, то мне пора спать!!!

1408. Леонид , 17 мая 2010, 07:48:40
РЕБЯТА ! Пишите свои С1-4 на пробном ЕГЭ в АПРЕЛЕ. И укажите РЕГИОН. Это ориентир на ЕГЭ. Из книжек не надо. Только свои и ваших одноклассников КИМ.

ЕГЭ ны яратам

1410. Леонид , 17 мая 2010, 08:54:00
1409. минегабделбарыймулла, 17 мая 2010, 08:44:57
ЕГЭ ны яратам

Всё сказал? Язык свой хорошо,наверное, знаешь. М О Л О Д Е Ц !

1411. vova, 17 мая 2010, 18:33:53
В треугольнике ABC угол C равен 90^circ, CH — высота, AB = 13, tg A = 5. Найдите BH.

1412. Ника, 17 мая 2010, 20:21:05
два велосипедиста одновременно отправились в 168-километровый пробег.первый едет со скоростью на 2 км в час большей второго,и прибывает к финишу на 2 часа раньше второго.найти скорость велосипедиста,пришедшего к финишу вторым.
Помогите решить задачу пожалуйста

1413. Леонид , 17 мая 2010, 21:35:53
1412. Ника, 17 мая 2010, 20:21:05. Не ленитесь просматривать, здесь "море" описано решений аналогичных задач. Ну и эта.
Скорость второго х (её надо найти). (х+2) - ск. 1-го.
Время первого 168/(х+2).
Время второго 168/х. Он тратит времени больше на 2, уравнение:
168/х - 168/(х+2) = 2. Приводи к общему знам., подобные,решай кв. ур-е. И все дела!

1414. неАйс, 19 мая 2010, 15:49:11
Ребят, а правда что в следствие разницы во времени, выпускники например владивостока решают ЕГЭшные задания, выкладывают в интернет ответы, и москвичи смогут получить ответы до окончания у них егэ? Ведь у всех по России будут одни и те же варианты?

1415. Андрей, 19 мая 2010, 15:49:58
ПОМОГИТЕ РЕШИТЬ ПОЖАЛУЙСТА!
Найдите площадь поверхности четырехугольной пирамиды(S),если в основании пирамиды лежит ромб с диагоналями 30 и 40,и все боковые грани пирамиды наклонены к плоскости основания под углом 30градусов

1416. Леонид , 19 мая 2010, 21:13:30
1414. неАйс, 19 мая 2010, 15:49:11. Нет, так не будет. Именно из-за поясов времени. На каждый регион неменее 40-50 вариантов не совпадающих.

1417. нимфа, 20 мая 2010, 10:04:30
ох.... егэ:( кто его только придумал.....к*злы

1418. Дэн, 20 мая 2010, 14:58:47
ДАВС тетраэдр Е-середина ДВ.М точка пересечения мелиан грани АВ С .Разложить векторы ЕМ по векторам ДА,ДВ,ДС.РЕШИТЕ ПОЖАЛУЙСТА!!!!!

1419. Инесса, 20 мая 2010, 15:34:32
Помогите пожалуйста...сложно справиться с задачами в12, которые на проценты и про части....приведу пример:
3 кг черешни стоят столько же, сколько 5 кг вишни, а 3 кг вишни-столько же, сколько 2кг клубники. На сколько процентов кг клубники дешевле кг черешни?
скажите алгоритм решения таких задач, что принимать за неизвестное? много ли таких задач в открытом банке? буду очень признательна...

1420. Олеся, 20 мая 2010, 16:03:19
площадь поверхности тетраэдра равен 1.2.Найдите площадь поверхности многогранника,вершинами которого являются середины сторон данного тетраэдра

1421. Андрей, 20 мая 2010, 21:42:57
ПОМОГИТЕ РЕШИТЬ ПОЖАЛУЙСТА!!
Найдите площадь поверхности четырехугольной пирамиды(S),если в основании пирамиды лежит ромб с диагоналями 30 и 40,и все боковые грани пирамиды наклонены к плоскости основания под углом 30градусов

1422. Леонид , 20 мая 2010, 23:35:02
половины диагоналей 15 и 20 , то сторона ромба по т. Пиф(диаг. ромба перпеикулярны) = 25. Высота ромба их площади его 1/2*30*40=25*н, н= 12. Половина 6. Линейный угол при основании 30 град. Апофема х, высота пир. х/2. По т. ПИф. ур-е х кв. - (х/2)кв.=36. Аповема х = 4 корня из 3.
Бокова пов. 1/2*4*25*4корня из3 = 200корней из 3. + пл. осн. 1/2*30*40 = 600. Поверхность пирамиды 200 корней из 3 +600. Разбирайся.

1423. МАЛАЯ, 21 мая 2010, 13:19:55
ЕГЭ задалбала

1424. Диана), 21 мая 2010, 15:03:20
если это не решу,то три в аттестат.помогите,плиз)))
найдите все решения системы
sin(2x+y)=0,
cos(x+y)=1,
удовлетворяющее условиям -п <или равно x < или равно п, -2п < или равно y <или равно -п

1425. vova, 22 мая 2010, 15:57:29
Боковые ребра треугольной пирамиды взаимно перпендикулярны, каждое из них равно 3. Найдите объем пирамиды.

1426. Александрор), 23 мая 2010, 15:58:08
Рёбра AD и BC пирамиды DABC равны 24см и 10см. Расстояние между серединами рёбер BD и AC равно 13см. Найдите угол между прямыми AD и BC.
Люди добрые помогите решить, битый час сижу, не выходит=(

1427. Anutka, 23 мая 2010, 19:46:17
А я ЕГЭ по математике пробник на 5 написала!!!!))))
Знаете, самое главное решать всё больше и больше всяких разных заданий,и всё получится)))

1428. сашка из воркуты, 23 мая 2010, 20:33:41
я вот не могу постоянно ЕГЭ онлайн писать,хочу скачать электронные тесты ЕГЭ поновее,подскажите где можно такие найти,а то что то не получается.А здесь вообще можно что то скачать?

1429. Леонид , 23 мая 2010, 20:42:54
1428. сашка из воркуты. здесь всё есть. Захои в Открытый банк заданий. Здесь в верху, в третьй строчке текст выделен. Кликни, жди, откроется, жми в зелёной Строке на Проверочные работы. Там можно проходить тренировочные тестирования - несколько работ с № 5 по № 14. И результат получишь.

САШКА воркутянин напиши свои с пробного, хотя бы С1 - С4. Надеюсь на тебя

1430. сашка из воркуты, 23 мая 2010, 20:56:06
вот у нас возник спор с преподом....за задание С6 даётся 4 балла я читала что баллы даются по убывающей: всё идеально решено-4 балла, если есть небольшая ошибка в вычислении, из за чего ответ в конце неверен,но решение правильное -3 балла, и так далее,1 балл даётся если мыслить начали правильно,но потом понаошибались..........а препод говорит что либо 0 либо 4 балла,если ответ неверен, то даже решение не смотрят........вот кому верить??

1431. Lexxus, 23 мая 2010, 21:04:38
вот у нас возник спор с преподом....за задание С6 даётся 4 балла я читала что баллы даются по убывающей: всё идеально решено-4 балла, если есть небольшая ошибка в вычислении, из за чего ответ в конце неверен,но решение правильное -3 балла, и так далее,1 балл даётся если мыслить начали правильно,но потом понаошибались..........а препод говорит что либо 0 либо 4 балла,если ответ неверен, то даже решение не смотрят........вот кому верить??

Ты гораздо более права. Критерии для разных заданий могут различаться, но в целом всё так, как ты написала.

1432. Галатея 777, 23 мая 2010, 22:12:27
Проверьте пожалуйста ответ:

в задании B10 (№ 28013) - 0,25
в задании B10 (№ 28014) - 0,67

1433. Алина, 23 мая 2010, 22:21:46
C2
В правильной шестиугольной призме ABCDEFA1B1C1D1E1F1,все ребра которой равны 1. Найдите расстояние от точки С до прямой A1B1

1434. Галатея 777, 23 мая 2010, 22:39:10
Каково ваше мнение, в онлайн тестировании ответ 0,67 выдает неверным
вот сайт: http://mathege.ru:8080/or/ege/StartTest?quizId=198
16 вариант В10
Программа не реагирует Ни на 0,66, ни на 0,67.

1435. сашка из воркуты, 23 мая 2010, 23:49:21
1431. Lexxus -спасибо, буду знать,при такой системе хоть на 1 балл можно надеяться,попытаться решить

1429. Леонид -имеешь ввиду сами задания?
загляни на сайт http://alexlarin.narod.ru там в разделе ЕГЭ есть " Диагностические работы МИОО 2010 года" => Диагностическая работа 13 мая 2010 года 10 класс там все эти задания и решения....

кстати, походу у меня был такой же вариант заданий ,что и у 1381. Ди .Судя по заданию С3))

P.S. Спасибо за объяснения))

1436. Кристина, 24 мая 2010, 06:27:03
ЕГЭ это ужасно зачем его вообще придумали например русский и биологию я сдам но с математикой у меня проблемы!!!!!!!!!! У меня нет слов я в тоске

1437. Танюча, 24 мая 2010, 14:45:51
Решите задачки пожалуйста по геометрии, а то я тупой человечик( заранее поськи =*** А вот и задачки:
1.В равнобедренном треугольнике АВС с основанием АС проведена медиана ВМ. На ней взята точка Д. Докажите равенство треугольников АВД и СВД.
2. В равнобедренном треугольнике АВС с основанием АС проведена биссектриса ДС. Найдите углі треугольника АВС, если угол АДС=60 градусов.
P.S. Надеюсь решите-поможите... НУ ВЫ ЖЕ УМНЫЕ!)

1438. Танюча, 24 мая 2010, 14:50:14
углы*)))

1439. Миха, 24 мая 2010, 16:30:09
Всем привет!Я думаю было хорошо дать грамоты тем кто занимает 1-3 места в коллективном разуме!Дать их можно 6 июня!Как вам мое предложение?

1440. Aleksandr, 24 мая 2010, 16:57:08
в треугольнике MNK биссектрисы пересекаются в точке О. Расстояние от точки О до стороны MN=6см.,NK=10см. Найти площадь треугольника NOK

1441. Л.М., 24 мая 2010, 21:30:04
Зачем? И потом, у участника строки 6 средний результат выше. А кто и как помогал другим? И ещё, в самом начале много баллов набрали за одни и теже задания. Позже это было отрегулировано. Так что 1-3 , мне думается, не лучшие результы. А что тренировались-молодцы, больше уверенности будет на ЕГЭ.

1442. сашка из воркуты, 24 мая 2010, 21:42:37
1437. Танюча 1) Док-во:
т.к. тр-ик АВД р/б,то сторона АВ будет равна стороне ВС, медиана-является и высотой и биссектрисой,значит она делит треугольник пополам => угол В тре-ка АВД будет равен углу В тре-ка СВД,а сторона ВД у них общая,так мы получаем,что эти тре-ки будут равны по двум сторонам и углу между ними.......ну вот как бы я сделала))

1443. L.D, 24 мая 2010, 21:50:13
Интересно, а что такое средний результат ? Есть же колонка доверие... И потом сначала на первых местах были совершенно другие люди как мне помнится...

1444. Ванда, 25 мая 2010, 14:03:19
В 10 № 28459 в банке заданий. никак не решу, помогите разобраться

1445. АНЯ, 25 мая 2010, 16:47:19
этот леонид у вас сдесь репетитор что ли

1446. funduk, 25 мая 2010, 17:05:54
Всё! Послезавтра первый ЕГЭ.Кто может дать полезные ссылки для подготовке к ЕГЭ по информатике??А то я стал нервничать. =)

1447. сашка из воркуты, 25 мая 2010, 17:37:34
1446. funduk : ооо инфу сдаёшь....потом расскажешь что там было?задания,сложность и т.д.))плиизззз, мне её тоже сдавать светит...уфф

1448. сашка из воркуты, 25 мая 2010, 19:13:55
помогите пожалуйста, не получается нормальный ответ...

Для определения эффективной температуры звёзд используют закон Стефана — Больцмана, согласно которому мощность излучения нагретого тела прямо пропорциональна площади его поверхности и четвёртой степени температуры:P = S&#963;T^4 , где &#963;=5,7*10^-8 — числовой коэффициент, S — площадь (в квадратных метрах), T — температура (в градусах Кельвина), а P — мощность (в ваттах). Известно, что некоторая звезда имеет площадь 181*10^15м квад,а излучаемая ею мощность не менее P=9,12*10^20 Вт. Определите наименьшую возможную температуру этой звезды (в градусах Кельвина).

1449. Леонид , 25 мая 2010, 21:18:20
1445. АНЯ, 25 мая 2010, 16:47:19. Это тебе мешает?

1450. кирилл, 25 мая 2010, 21:53:14
круто)))))))

1451. funduk, 26 мая 2010, 14:00:06
тишина...

1452. funduk, 26 мая 2010, 20:35:12

1453. svetlana, 26 мая 2010, 22:16:31
Помогите с задачей В10 (№ 28709):
"Cкороcть колеблющегоcя на пружине груза меняетcя по закону v(t)=10sin(пt/5) (cм/c), где t — время в cекундах. Какую долю времени из первых трех cекунд cкороcть движения превышала 5 cм/c? Ответ выразите деcятичной дробью, еcли нужно, округлите до cотых. "
Непонятно как записывать ответ. По времени получается 2,17 с, а если расчитывать ДОЛЮ от трёх секунд, то может быть надо ещё на три поделить (0,72 получится)?
Какой ответ писать 2,17 или 0,72 ?

1454. Оля, 26 мая 2010, 23:47:45
Я так рассуждаю: 10sin (pi. t/5) больше 5, sin (pi.t/5) , больше 1/2 - на ед. окружности: pi.t/5 больше pi/6, но меньше 5pi/6, т.е. t/3 больше 5/18 но меньше 25/18. это 25/18 - 5/18 = 20/18 = 10/9 примероно 1,11

1455. жека, 26 мая 2010, 23:56:34
правильной шестиугольной призме ABCDEF A1B1C1D1E1F1 всё рёбра которой равны 1 найдите расстояние от точки С до прямой F1E1

1456. Новогодний питон, 27 мая 2010, 10:11:50
у меня завтра ЕГЭ по алгебре. ВСЕМ УДАЧИ.

1457. julia, 27 мая 2010, 10:40:18
ПОМОГИТЕ РЕШИТЬ ЗАДАЧУ,ПОЖАЛУСТА!!!!Мне срочно надо до 13 ч!!
Основанием прямой призмы ABCA1B1C1 является прямоугольный треугольник ABC с прямым углом C.Через ребро CC1 проведено сечение CC1D1D, перпендикулярное к плоскости AA1B.Найдите площадь сечения,если объем призмы равен 10,2, AD=0,9, BD=2,5.

1458. Lexxus, 27 мая 2010, 12:51:29
Основанием прямой призмы ABCA1B1C1 является прямоугольный треугольник ABC с прямым углом C.Через ребро CC1 проведено сечение CC1D1D, перпендикулярное к плоскости AA1B.Найдите площадь сечения,если объем призмы равен 10,2, AD=0,9, BD=2,5.

Расписываем теорему Пифагора для прямоугольных треугольников ABC, ADC и BDC. Получаем систему из трех уравнений с неизвестными AC, BC и CD.

Аккуратно её решив, получим:
AC = 3*sqrt(17/2)/5
BC = sqrt(17/2)
CD = 3/2.

Объем призмы равен произведению площади треугольника ABC на высоту CC1, то есть
V = СС1*AC*BC/2, откуда высота призмы
СС1 = V/(AC*BC/2) = 10.2/2.55 = 4.

Площадь сечения CDD1C1 (а это прямоугольник) равна CD*CC1 = 3/2*4 = 6

1459. Ника, 27 мая 2010, 14:56:50
как сдавать?

1460. rasl, 27 мая 2010, 16:23:57
кто нибудь знает ответы на завтрршнее егэ по матике?если кто знает кидайте ссылку

1461. funduk, 27 мая 2010, 16:56:56
эх...
Не очень доволен инф-ой =(

1462. максим, 27 мая 2010, 21:37:47
народ,мне завтра сдавать ЕГЭ по математике,а я ваще ниче незнаю!!где можно ответы скачать?ответы к реальным ЕГЭ за 9 класс по математике!=)

1463. максим, 27 мая 2010, 21:40:29
ау не молчите))ппц как надо!!))

1464. сашка из воркуты, 27 мая 2010, 22:48:49
1461. funduk жаль,а что там было то?ну там задачи, примерные задания?

1465. Сергей, 28 мая 2010, 11:49:49
Народ решите плиз задачу!

B12 Двое рабочих, работая вместе, могут выполнить работу за 12 дней. За сколько дней, работая отдельно, выполнит эту работу первый рабочий, если он за два дня выполняет такую же часть работы, какую второй - за три дня?

1466. Леонид , 28 мая 2010, 13:18:37
Пусть 1-й за х дней (в ответ), второй за у. Произвоительность 1/х +1/у =1/12 - первое уравнение. 2/х=3/у - второе уравнение, из него у= 1,5х. Первое приведём к общему знам. 12х+12у=ху и вместо у поставь 1,5х.
х = 20

1467. jlby, 28 мая 2010, 13:58:09
1453 верно 0,72

1468. jlby, 28 мая 2010, 14:18:53
1432
b10 28014 0.67 верно
b10 28013 реши неравенство модуль cos pit>=(корень из 2 )/ 2

1469. jlby, 28 мая 2010, 15:05:46
Для 1444
b10 28459
Т.к. pv^1,4=const, то при максимальном давлении в 128 атм объём будет минимальным, реши уравнение 1*8^1,4=128*V^1,4, упрости его 2^4,2=2^7*V^1,4, отсюда V^1,4=2^(-2,8), V^1,4=(2^(-2))^1,4, отсюда
V=2^(-2), т.е. V= 0,25

1470. Леонид , 29 мая 2010, 00:04:07
Lexxus! Очень прошу о помощи в решении данной задачи. ПОЖАЛУЙСТА. Точка О – центр окружности радиуса 2. На продолжении радиуса ОМ взята точка А. Через точку А проведена прямая, касающаяся окружности в точке К. Известно, что угол ОАК равен 60&#8304;. Найдите радиус окружности, вписанной в угол ОАК и касающейся данной окружности внешним образом.

1471. Lexxus, 29 мая 2010, 14:29:06
Точка О – центр окружности радиуса 2. На продолжении радиуса ОМ взята точка А. Через точку А проведена прямая, касающаяся окружности в точке К. Известно, что угол ОАК равен 60&#8304;. Найдите радиус окружности, вписанной в угол ОАК и касающейся данной окружности внешним образом.

Хорошая задачка.



Для начала заметим, что у нас может быть два случая - один обозначен синим, другой - красным (частично).

Рассмотрим "синий" случай. Пусть C - центр искомой окружности, а r - её радиус, который мы ищем. CT - перпендикуляр к OA.
В прямоугольном треугольнике CTO:
OC = 2+r; CT = r. Сейчас найдем и третью сторону, OT.

В прямоугольном треугольнике CTA угол CAT = OAK/2 = 30 градусов, поэтому TA = CT/tg(30 гр.) = r*sqrt(3).
А из треугольника OAK, зная угол OAK и сторону OK, находим:
OA = 4/sqrt(3).

Отсюда OT = OA-TA = 4/sqrt(3)-r*sqrt(3).

Можем записать теорему Пифагора для треугольника CTO:
r^2+(4/sqrt(3)-r*sqrt(3))^2=(r+2)^2

Кстати, если точно таким же образом рассмотреть "красный" случай, то уравнение получится вот такое:
r^2+(r*sqrt(3)-4/sqrt(3))^2=(r+2)^2

Понятно, что оба уравнения приводятся к одному и тому же виду
9*r^2-36*r+4=0

Поэтому, решив это уравнение, мы получим ответ сразу для обоих возможных случаев:

r1 (синий) = 2-4*sqrt(2)/3 (примерно 0.11)
r2 (красный) = 2+4*sqrt(2)/3 (примерно 3.89)

1472. Леонид , 29 мая 2010, 15:50:36
Lexxus, огромное спасибо.

1473. Фанька, 31 мая 2010, 06:12:31
помогите,пожалуйста,с задачей..Натуральные числа a,b, c образуют возрастающую арифметическую прогрессию,причем все они больше 1000 и явлются квадратами натуральных чисел.Найдите наименьшее возможное.при указанных условиях,значение b.если можно,с объяснением...зараннее спасибо)

1474. Сергей, 31 мая 2010, 11:58:31
Ответы по всем предметам ня ЕГЭ 2010 года (все регионы) доступны на сайте http://ege-2010.info Заходите и скачивайте! В качестве бонуса есть уникальная методика для подготовки к сдаче ЕГЭ по всем предметам.

1475. Ванда, 1 июня 2010, 07:30:24
На сайте webmaht.exponenta.ru выложены 5 моделей досрочного экзамена по математике в апреле. Часть В - простая, обычная, а вот часть С мне показалась "заковыристой", особенно С2. Ко всем заданиям приложены ответы и решения. У меня просьба к Lexxus - не могли бы Вы посмотреть это С2, может как-то попроще можно сделать?

1476. Бамбр, 1 июня 2010, 08:51:36
Здравствуйте, уважаеые, помогите, пожалуйста с B8. Это с открытого банка задач, на этом сайте я почему-то подобного рода задания не смог найти.
http://i027.radikal.ru/1006/94/7d65895c5fb6.jpg
Ответь ведь 0.5?

1477. Lexxus, 1 июня 2010, 09:42:29
Это с открытого банка задач, на этом сайте я почему-то подобного рода задания не смог найти.

Плохо искал ;)
И то самое задание там тоже есть.

1478. гоша, 1 июня 2010, 12:09:54
У меня голова не работает, ПОМОГИТЕ!!!!!

1479. Игорь, 1 июня 2010, 13:23:01
Ребят, помогите пожалуйста кто-нибудь метод выполнения задания- в8
я не понимаю, когда ответ с минусом, а когда без минуса....от чего это зависит?
Заранее спасибо!

1480. L@mpa, 1 июня 2010, 13:28:47
Да уж, ЕГЭ это что то с чем то ....лично яч считаю, что это не поврка знаний, а тупое натаскивание на определенного типа заданий воти все! Ведь тренируют нас именно так!

1481. funduk, 1 июня 2010, 14:01:09
результаты где посатеть можня?

1482. ищущий правду, 1 июня 2010, 14:42:08
люди помогите с заданием пожалуйста!
Cкороcть колеблющегоcя на пружине груза меняетcя по закону v(t)=8sin(pi*t/5) (cм/c), где t тАФ время в cекундах. Какую долю времени из 2 cекунды cкороcть движения превышала 4 cм/c? Ответ выразите деcятичной дробью, еcли нужно, округлите до cотых.
я получил 5/6<=t<=25/6 скажите чтодальше делать? разделить на 2? и еще если у меня в ответе к заданию будет 1.66666 то что в ответ написать? 1.7?

1483. Альбик, 1 июня 2010, 16:07:55
Игорек, в общем в В8 все просто. как все гениальное
___________________________________
если угол от касательной/прямой острый - +
если тупой - -

1484. Александра, 1 июня 2010, 18:22:16
Б9 1)Диагональ куба равна корню из 12. найти его объём

1485. Альбик, 1 июня 2010, 18:31:58
во блин
первый раз такой бред вижу
покажите как решать кто знает?..

1486. Dron, 1 июня 2010, 19:02:11
Уважаемые юноши девушки напишите пожалуйста ответы ЕГЭ по алгебре который состоится 07.06.2010. Пожалуйста напишите буду очень благодарен. Я буду ждать.

1487. Леонид , 1 июня 2010, 20:29:05
диагональ куба д кв. = 3а кв. т. е. 12 = 3а кв., то а кв.=4, а=2. Объём а в кубе = 2 в кубе = 8.

1488. Олег, 1 июня 2010, 20:46:42
Lexxus,я что хотел спросить,ведь если тут решаются задачи и можно посмотреть правильный ответ или не правильный,то наверняка и решение есть,дак вот,где можно взять решения???

1489. svetlana, 1 июня 2010, 23:32:08
" ищущий правду " (1 июня, 14:42)

Верно то, что 5/6<=t>=25/6, т.е. скорость превышала 4см/с на протяжении времени 25/6 - 5/6 = 20/6 (с), но 2 секунды - это 12/6. Получается тогда 12/6 - 5/6 = 7/6 (с) или 1,17 с.
Но если расчитывать долю от 2 секунд, то вероятно надо бы разделить ещё на 2 и тогда ответ будет округлённо 0,59.

НЕПОНЯТНО КАКОЙ ОТВЕТ ХОТЯТ УВИДЕТЬ АВТОРЫ ЗАДАЧИ ? 1.17 или 0.59 ?

КТО-НИБУДЬ ЗНАЕТ ЧТО ПИСАТЬ В ОТВЕТЕ ?

1490. Игорь, 2 июня 2010, 12:39:19
C5. Дано F(x) = x^2 +4X + abs(x^2-(3/2)x -1) -a (abs( )- модуль)
Найти все а, при которых зачение функции не отрицательно .
Моя идея - найти через производную минимум всей функции без "-а", затем подставив значение х в точке минимума найти У и тогда при а < -y график будет подниматься над ОХ . Вопрос как брать производную от модуля?
Ну как идея?
по идее получится а<-2, а при моделировании на построителе графиков получается а<-1.75

1491. Александра, 2 июня 2010, 12:50:46
спасибо)
вот ещё несколько задачек Б( которые не могу решить(((

1)радиус основания первого конуса в 3 раза меньше чем радиус 2ого конуса. образующая 1ого в 2 раза больше чем образующая 2ого. чему равна площадь боковой поверхности 1ого конуса если площадь боковой поверхности 2ого равна 18 см в кв. ответ дайте в см кв.

2) шар объемом 6 м в кубе вписан в цилиндр. найти объем цилиндра.

3)кубик весит 10 г. сколько граммов будет весить кубик, ребро которого в 3 раза больше чем ребро первого кубика если оба кубика изготовлены из одинакового материала?

4)бильярдный шар весит 360 г. сколько граммов будет весить шар вдвое меньшего радиуса сделанный из того же материала?

5)если каждое ребро куба увеличить на 1, то площадь его поверхности увеличится на 30. найти ребро куба

6) во сколько раз увеличится объем правильного тетраэдра если все его ребра увеличить в два раза?

7) в куб с ребром 6 вписан шар Найти объем шара, деленный на пи.

помогите решить пожалуйста)))))

1492. Я, 2 июня 2010, 13:00:48
Админ, сделай, чтобы '+' при поиске можно было заменять на 'plus'. А то невозможно найти, если есть этот знак. Он просто не отображается.

1493. Lexxus, 2 июня 2010, 15:23:38
Админ, сделай, чтобы '+' при поиске можно было заменять на 'plus'. А то невозможно найти, если есть этот знак. Он просто не отображается.

Теперь ищет с плюсом.

1494. Игорь, 2 июня 2010, 16:30:39
Альбик
спасибо!)

1495. кристиннн, 2 июня 2010, 18:32:57
Первая труба пропускает на 2 литра воды в минуту меньше, чем вторая. Сколько литров воды в минуту пропускает первая труба, если резервуар объемом 255 литров она заполняет на 2 минуты дольше, чем вторая труба?

Как можно решить задачу без достаточных индексов???

1496. Александра, 2 июня 2010, 18:35:40
хочу здать егэ))))

1497. Наталья, 2 июня 2010, 19:07:02
Вот, кто хочет, может потренироваться. Это центральный регион.
С1-решить систему ур-ний:5х-2в степени игрек равно 3 плюс корень из 5х-х в квадрате.
2-е ур-ние:игрек-логарифм2(х в квадрате-1)равно 0

С2-через ребро основания правильной четырехугольной пирамиды проведена плоскость,которая делит площадь боковой поверхности пирамиды пополам.в каком отношении,считая от вершины,проведенная плоскость делит высоту пирамиды?

С4-хорда АВ и диаметр MN одной и той же окружности не пересекаются,а точка пересечения прямых AM и BN равноудалена от концов хорды АВ на расстояние 3.Найти радиус окружности,если уголANM равен 30градусов

С5-при каких значениях параметра а ур-ние х в квадрате-2а синус(косинус х)плюс а в квадрате синус2 равно 0 имеет единственное решение?

С6-если к десятичной записи натурального числа а приписать справа запятую,а потом некоторый бесконечный набор цифр,то получится десятичная запись такого иррационального числа с,что (2с-3)в квадрате равно 3а в квадрате-12с плюс 46.Найдите все во3можные значения числа с

С5-решить систему ур-ний 1-е ур-ние:хуz равно 10 2-е ур-ние:lg в квадрате х плюс lg в квадрате у плюс lg в квадрате z равно 8,5 3-е ур-ние:lg x умножить lg y умножить lg z равно -4,5

С6-найдите сумму всех трехзначных натуральных чисел n таких,что первая и последняя цифры числа n в квадрате равны 1

С5-При каких положительных значениях параметра a модуль разности корней уравнения ах в квадрате плюс 2х минус 2,25 равно 0 не больше расстояния между точками экстремума функции f(x)равно 2х в кубе минус 9х в квадрате минус 6ах плюс 13а в квадрате

С6 Необходимо добраться из пункта А в пункт В,которые находятся в лесу.Через пункты А и В проходят две прямые дороги,пересекающиеся под углом 60 градусов.По дорогам можно доехать на велосипеде за 32минуты(сначала от пункта А до перекрестка,потом от перекрестка до пункта В)Можно также дойти пешком по прямой тропинке до ближайшего участка дороги,проходящей через В,потом до пункта В доехать на велосипеде,затратив на весь путь более 1,6 часа,или же дойти пешком напрямую до пункта В за 4 умножить на корень из 619 минут.Найдите расстояние от пункта В до дороги,проходящей через А,если скорость передвижения на велосипеде равна 20км/ч,а пешком-5км/ч

1498. иван, 2 июня 2010, 19:17:22
Первая труба пропускает на 1 литр воды в минуту меньше, чем вторая. Сколько литров воды в минуту пропускает вторая труба, если резервуар объемом 110 литров она заполняет на 1 минуту быстрее, чем первая труба? какой ответ 10 или 11

1499. костик, 2 июня 2010, 19:26:31
ЗАДАНИЯ ЧАСТИ Б-8 много касячества перепроверяйте те где спрашивается найти сумму или кол-во точек каких либо там ибо народ зделает ошибки

1500. фундук, 2 июня 2010, 21:08:59
еее
69 баллов по информатике !
=)

1501. funduk, 2 июня 2010, 21:16:53
А русский долго ждать??

1502. Я, 2 июня 2010, 21:59:43
Первая труба пропускает на 2 литра воды в минуту меньше, чем вторая. Сколько литров воды в минуту пропускает первая труба, если резервуар объемом 783 литра она заполняет на 4 минуты дольше, чем вторая труба заполняет резервуар объемом 725 литров?

Решаю-решаю,ответ нецелый получается((

1503. ilo, 3 июня 2010, 07:44:28
пропускная способность первой трубы-х-2,второй-х,время заполнения 783-х литров первой трубой-783/х-2,второй-725/х, разница в 4 минуты приводит к уравнению: 783/х-2 - 785/х=4. После приведения к общему знаменателю 783х-725х+1450-4х2+8х=0; 2х2-33х-725=0; Д=6889=83 в квадрате, х=29, второй не удовлетворяет условию.Ответ:29-2=27

1504. ilo, 3 июня 2010, 08:02:15
какой ответ 10 или 11?
из 2-х корней -10 и11 по условию подходит только положительный 11,при условии что пропускная способность второй трубы-х,если пропускная способность второй трубы обозначена х+1 корни будут -11 и 10, подходит 10, подставим в х+1,ответ тот же-11.

1505. Gerik=), 3 июня 2010, 11:06:42
Всем привет!




Как это егэ осложняет(((

1506. Алексия, 3 июня 2010, 11:55:32
Я заметила ошибку в ответах, когда решала Егэ по математике демонстрационный вариант, в С1. У вас там ответ неправильный!!!

Как напечатать задания из открытого банка задач???

1507. Александра, 3 июня 2010, 14:23:29
решите пожалуйста задачи Б9 из поста 1491...
не может быть такого что никто не знает!
вот купила себе раб.тетр. с заданиями Б9 а там ответы есть а решений нет((((((

1508. ilo, 3 июня 2010, 15:14:03
1491 1) Sб.п прямопропорциональна R и l, если образующая увеличена в 2 раза, то и Sб.п. увеличится вдвое, если R уменьшен в 3 раза, то и Sб.п.уменьшается в 3 раза. Т.о. искомая Sб.п. составляет от данной 2/3S=18*2/3=12кв.см
2)Vш.=4/3пиRкуб. из этой формулы находим пиRкуб: 6*3/4=9/2
Vцилиндра=пиRкв.*h, h=2R, Vц.=2пиRкуб. 2*9/2=9куб.ед.

1509. ilo, 3 июня 2010, 15:37:04
1491 3)Vкуба=а в кубе.,если ребро увеличить в 3 раза V увеличится в куб раз, то есть в 27.масса прямопропорциональна плотности и V, т.к.плотность та же(материал один),значит-во сколько раз V, во столько и масса,т.е.в 27: 10г.*27=270г.
4)аналогично 3)
5)было ребро а, стало (а+1) 6(а+1)в квадрате-6а квадрат=30. а=2

1510. неАйс, 3 июня 2010, 16:17:04
Ребят почему правильный ответ 126 ?
http://live.mephist.ru/show/mathege2010/view/id/25887/

1511. неАйс, 3 июня 2010, 16:21:13
Алексия, там есть значок, наводя мышку вылазиет надпись печать, после того как выбрала можешь прям через браузер печатать, в файрфоксе файл-печать

1512. Lexxus, 3 июня 2010, 16:22:06
Ребят почему правильный ответ 126 ?
http://live.mephist.ru/show/mathege2010/view/id/25887/

Тоже думаешь, что 77?
А задание внимательно читать кто за тебя будет?

1513. неАйс, 3 июня 2010, 16:45:44
Lexxus, Ааа! Точно! вот так я и на пробном, заместо площади нашел объем. Просто привык в такого типа задачах в основном объем надо находить

1514. неАйс, 3 июня 2010, 17:29:56
а вот здесь http://live.mephist.ru/show/mathege2010/view/id/6141/ почему 500 получается? там же промежутки. в банке задач я не нашел кстати эту задачу, ее удалили что ли

1515. Lexxus, 3 июня 2010, 17:39:18
а вот здесь http://live.mephist.ru/show/mathege2010/view/id/6141/ почему 500 получается? там же промежутки. в банке задач я не нашел кстати эту задачу, ее удалили что ли

Потому и удалили, что формулировка дурацкая.
Но имелось в виду, конечно, "при какой минимальной температуре...", так что ответ - 500.

1516. jesful, 3 июня 2010, 17:42:32
Привет всем!
В задачи В12 №5679 походу ошибка в ответе.
Дело в том, что там к полученному значению икса(11) нужно прибавить ещё 4, т.к. нужно найти скорость первого, пришедшего на финиш.

Решение:
За х обозначаем скорость второго велосипедиста.
77/(х+4)=77/х+4
Упрощаем и получается:
х*х-4х-77=0
х=11 - скорость, пришедшего к финишу вторым
11+4=15
Ответ:15

Вот ссылочка:
http://live.mephist.ru/show/mathege2010/view/id/5679/

1517. бе, 3 июня 2010, 19:30:18
да ну это егэ!!!!!

1518. Леонид , 3 июня 2010, 20:35:54
Два велосипедиста одновременно отправились в 77-километровый пробег. Первый ехал со скоростью, на 4 км/ч большей, чем скорость второго, и прибыл к финишу на 4 часа раньше второго. Найти скорость велосипедиста, пришедшего к финишу первым. Ответ дайте в км/ч.

х - что просят в ответе. Это важно, много раз писали, х - ск. 1-го (он пришёл первым, его ск. и просят найти). (х-4) - ск. 2-го. Время 1-го 77/х, время 2-го 77/(х-4). 2-ой пришёл позже, ур-е: 77/(х-4) - 77/х = 4. Решаем х=11 - это и есть ответ. Читай и думай, друг, внимательно.

1519. Леонид , 3 июня 2010, 20:39:15
Lexxus! Прошу о помощи! Пожалуйста. С4

Радиус окружности, описанной около ∆ АВС, равен 13, Cos ВАС = - 5/13,
высота, проведённая к стороне ВС, равна 5. Найдите длину той хорды
АМ описанной окружности, которая делится пополам стороной ВС.

1520. Александра, 3 июня 2010, 22:23:42
Спасибо огромное)))
а вот ещё несколько задач кому не трудно попробуйте решите плиз)))

1) В цилиндрический сосуд в котором находится 6 дм воды, опущена деталь. при этом уровеень жидкости в сосуде поднялся в 1.5 раза. чему равен объем детали?

2)прямоугольный параллелепипед описан около цилиндра, радиус равен 2 и высота равна 2. найти объем параллелепипеда.

3) радиусы двух шаров 6 и 8 Найдите радиус шара площадь поверхности которого равна сумме площадей поверхностей данных шаров

1521. jesful, 4 июня 2010, 09:20:59
Спасибо, Леонид!

1522. Настасья, 4 июня 2010, 10:55:26
Люди обьясните как решать проще задания В11 блин иногда все так просто а иногда.......

1523. jlby, 4 июня 2010, 12:29:25
1482
тебе надо найти 7/6 от 2, т.е. 7/6:2=7/12=0,58(3). В ответе писать 0,58!

1524. Александра, 4 июня 2010, 13:17:59
кто-нибудь может решить B11задание, я вообще не понимаю как решать задания вот такого плана:

Найдите наименьшее значение функции
у=(х^2-7х+7)е^(х-5) на отрезке [4;6]

1525. Артем Ник, 4 июня 2010, 13:29:07
Ищи производную!
Находи точки минимума!
И проверяй значение функции в точках концов отрезка,который дан и в точках минимума!)

1526. Александра, 4 июня 2010, 14:03:03
производная равна (9х-х^2-14)е^(х-5)
теперь возникает вопрос,чтобы найти точки минимума нужно просто решить квадратное уравнение? а потом эти точки подставить в первоначальную функцию? или в "производную" функцию??

1527. ilo, 4 июня 2010, 14:16:06
производная: (2х-7)е^х-5+(х^2-7х+7)е^х-5=е^х-5(2х-7+х^2-7х+7)=е^х-5(х^2-5х), множитель e^х-5 в 0 не обращается ни при каком х(показательная функция), х^2-5х=0 при х=0 и х=5
у(4)=-5/е, у(6)=е ,у(0)=7/е^5, у(5)=-3, унаим.=-3 остальные значения можно не рассматривать,т.к. они не являются целыми,а в ответе указывается целое.
в 1520 2) основание цилиндра вписано в основание параллелепипеда(окружность вписана в квадрат) сторона квадрата 2R=4, Sосн.=16, V=16*2=32
3)S1=4пR=24п, S2=32п, в сумме 56п, S большого=4пR=56п, отсюда R=14

1528. ilo, 4 июня 2010, 14:33:15
прошу прощения у(0) не принадлежит отрезку[4,6], рассматривать не надо

1529. Александра, 4 июня 2010, 14:53:21
ilo, спасибо огромное!))))

1530. ilo, 4 июня 2010, 14:54:40
в 1520 1) 6дм кубических?

1531. Thecutelittledeadgirl, 4 июня 2010, 15:01:18
Кажется, в задании номер 16225 ошибка. Не знала, куда еще написать... Спасибо)

1532. Леонид , 4 июня 2010, 15:04:50
1519. Леонид , 3 июня 2010, 20:39:15
Lexxus! Прошу о помощи! Пожалуйста. С4

Радиус окружности, описанной около &#8710; АВС, равен 13, Cos ВАС = - 5/13,
высота, проведённая к стороне ВС, равна 5. Найдите длину той хорды
АМ описанной окружности, которая делится пополам стороной ВС.


1533. ilo, 4 июня 2010, 15:19:24
Игорь 1490 может быть квадратный трёхчлен под модулем раскрыть дважды: при х для которых он положителен и отрицателен, а не меньше -это когда больше или равно. может быть производная это круто для квадратичной функции,взять координаты вершины и смотреть,где у=а ниже или касается вершины, а при отрицательном выражении под модулем там вообще линейная функция получается.

1534. Екатерина, 4 июня 2010, 16:42:00
Помогите решить задачу!!!!!Плиз))))
В10 В шар радиуса корень из 34 вписан цилиндр с площадью основания 9пи.Найдите тангенс угла между образующей цилиндра и диагональю его осевого сечения.

1535. Александра, 4 июня 2010, 17:45:59
и кому не слжно объясните мне плиз как решить все тоже B11 задание но только с натуральными логарифмами(

Найти наибольшее значение функции у=In(7х)-7х+7 на отрезке [1/14 ; 5/14].
и ещё один вопрост как можно посчитать чему равен натуральный логарифм по основанию 1/2 то есть In 1/2 ?

1536. Алиюш, 4 июня 2010, 17:51:24
здравствуйте!!! А можно сюда писать во время ЕГЭ по математике???))))))

1537. оля, 4 июня 2010, 19:06:30
помогите решить Б10 плиз
После дождя уровень воды в колодце может повыситься.Мальчик измеряет время падения t небольших камешков в колодец и рассчитывает расстояние от повехности земли до уровня воды по формуле h=-5t^2. до дождя время падения камешков составляло 1с. На какую наименьшую высоту должен подняться уровень воды после дождя,чтобы измеряемое время изменилось больше, чем на 0.1 с? ответ выразите в метрах.

1538. пирожок с картошкой, 4 июня 2010, 19:13:17
Пожалуйста, объясните, когда при тригонометрических уравнениях ставить пн, а когда 2пн,н принадлежит Z(это в части С1)

1539. Леонид , 4 июня 2010, 19:17:13
1)ln1/2 - иррациональное число (основание е, с не 1/2; 1/2 под знаком логарифма или логарифмируемое число). Задание: бери производную
у штрих= 7/(7х) - 7 приравниваем 0, решаем х=1/7. Поставляй в функцию
у(1/7)= ln(7*1/7)-7*1/7 +7 = 0-1+7 =6

1540. Александра, 4 июня 2010, 20:21:54
Спасибо!)))всё с заданиями Б11 разобралась) может кто Б12 сможет решить?
Б12 Найдите двузначное число,которое в 2,5 раза больше суммы его цифр и в 3 раза больше произведения его цифр.
и ещё одна

Б12 Четыре рубашки дешевле куртки на 20 процентов.На сколько процентов шесть рубашек дороже куртки?

1541. ilo, 4 июня 2010, 20:58:07
До дождя расстояние от поверхности земли до воды:5*1^2=5,время падения после дождя уменьшается на 0,1с,т.к.вода поднимается, т.е. станет 0,9, снова рассчитываем расстояние 5*0,9^2=4,05.Значит уровень воды может подняться на наименьшую высоту5-4,05=95см=0,95м,дальше время изменится больше, чем на 0,1с. или так:h=5t^2. t<0,9, t^2<0,81, t^2=1/5h, 1/5h<0,81,h<4,05 h-расстояние от поверхности земли до воды

1542. Lexxus, 4 июня 2010, 22:45:04
Радиус окружности, описанной около ∆ АВС, равен 13, Cos ВАС = - 5/13,
высота, проведённая к стороне ВС, равна 5. Найдите длину той хорды
АМ описанной окружности, которая делится пополам стороной ВС.

Заставил, однако, глубоко задуматься :)



1. Во-первых, найдем BC. Как известно, угол, вписанный в окружность, опирается на вдвое большую дугу. Обозначим угол BAC за "альфа" (его косинус мы знаем). Тогда дуга BC (большая) равна 2альфа, а угол BOC, соответственно, 2пи-2альфа.

Хорда BC = 2R*sin(BOC/2) = 2*R*sin(пи-альфа) = 2*R*sin(альфа) = 2*R*sqrt(1-(5/13)^2) = 2*13*12/13 = 24.

2. Из прямоугольного треугольника BKO находим KO = sqrt(13^2-12^2) = 5

3. Теперь нам пора заметить, что у нас опять два случая. Как обычно, обозначим их красным и синим. Рассмотрим красный, а синий получится сам собой.

4. Из точки D отложим отрезок DP, параллельный и равный AT=5. KO=DP, оба они перпендикулярны BC...
В общем, я веду к тому, что точки M, O и P лежат на диаметре, параллельном BC, а треугольники ADT и DMP равны.

5. Рассмотрим прямоугольные треугольники ODM и ATD. Углы ADT и OMD равны, а значит, эти треугольники подобны (по двум углам).
Можем составить пропорцию:

OM/DM = AD/TD. Кстати, обозначим DM за x (То есть, в окончательном ответе нам нужно будет указать 2x).
Итак, 13/x = x/TD.

В свою очередь, TD найдем из треугольника ATD:
TD = sqrt(x^2-25)

Итак, вот и получилось уравнение:
13/x = x/sqrt(x^2-25)
Оно сводится к биквадратному
x^4-169*x^2+4225 = 0

У него есть два положительных корня:
x1 = sqrt(13/2*(13+sqrt(69))) (это для "красного" случая)
x2 = sqrt(13/2*(13-sqrt(69))) (это для "синего" случая)

Ну, и не забываем, что надо ещё умножить на два.


1543. Леонид , 4 июня 2010, 23:10:53
Lexxus! Какой Вы молодчина. Приогромнейшее спасибо. Как Вы успеваете всем ответить!
Как можно связаться с руковоителем Вшего проекта,выступающим с видеоконсультацией?

1544. Леонид , 4 июня 2010, 23:21:34
Lexxus...у
Ваше: "Хорда BC = 2R*sin(BOC/2) = 2*R*sin(пи-альфа) = 2*R*sin(альфа) = 2*R*sqrt(1-(5/13)^2) = 2*13*12/13 = 24".

(ВС проще звучит по теореме синусов, найдя син.А = 12/13 - триг.ед., ВС/син.А = 2R).

А вот п.4 - догадаться нелегко-выход на решение. Высший клас. ЗДОРОВО.

4. Из точки D отложим отрезок DP, параллельный и равный AT=5. KO=DP, оба они перпендикулярны BC...
В общем, я веду к тому, что точки M, O и P лежат на диаметре, параллельном BC, а треугольники ADT и DMP равны.

1545. Lexxus, 4 июня 2010, 23:25:17
Как можно связаться с руковоителем Вшего проекта,выступающим с видеоконсультацией?

Леонид, смею заметить, что у Вас не совсем (точнее, совсем не) верное представление о моём месте в структуре Мироздания вообще и в ЕГЭ в частности :)
В конкретно этом проекте (то есть, во всем, что есть на сайте live.mephist.ru) я работаю один, у меня тут нет ни руководителей, ни соучастников.

С И. В. Ященко (если Вы его имеете в виду) я даже не знаком.
Но если вы хотите с ним связаться, вам поможет веб-страница кафедры математики МИОО.

1546. надежда 2010, 5 июня 2010, 01:45:44
Александра, по задачам В11: можно просто брать точку из указанного промежутка и подставить в формулу функции, это и для функций с Е, и логарифмом, и с тригонометрией... Главное, чтобы при подстановке точки из промежутка ушло все "лишнее"

1547. gorigor, 5 июня 2010, 01:50:38
Что тута не работает? Мозги не работают. Для решения этих дебилоидных задач мозги нэ нужны

1548. надежда 2010, 5 июня 2010, 01:51:53
александра, подставь в формулу 214, логарифм уйдет, а остальное хорошо считается, и не надо заморачиваться с производной. Попробуй.

1549. надежда 2010, 5 июня 2010, 02:00:00
вообще-то не 214, а две четырнадцатых, это у меня проблема с компом, не вае печатает

1550. ilo, 5 июня 2010, 06:38:10
gorigor, ты у нас такой дурак по субботам или как? слышишь, сдвинь корону на бок, чтоб не висла на ушах.

1551. ilo, 5 июня 2010, 08:11:46
1540 2)у-цена куртки,4х-четырёх рубашек. В первом случае цена "дешёвых "рубашек сравнивается с ценой "дорогой" куртки,в таких случаях за 100% принимается то, с чем сравнивают у-100%, 1%-0,01у на сумму4хприходится 4х/0,01у% 100%-4х/0.01у=20%, 4х/0.01у=80% х=0,2у
во втором случае "дорогие" рубашки сравниваются с "дешёвой" курткой:снова, то,с чем сравнивают -у-100%, 6х/0,01у-приходится на сумму 6х(это больше 100%): 6х/0,01у-100%=4х/0,01у*1,5-100%=120%-100%=20%,из
4х/0,01у=80%чтобы получить 6х/0.01у умножаешь обе части на 1,5,
короче разницу в цене впервом случае у-4х/у=0,2, х=0,2уво втором случае
6х-у/у, вместо х подставь у, отношение вырази в %
в 1) двузначного не получается

1552. Асия, 5 июня 2010, 09:50:23
Lexxus, пожалуйста помогите с многогранниками, что-то совсем их не пойму( 5 лет прошло, эх геометрия, школа, нужно брату на ЕГЭ помочь), если есть парочка решенных, скиньте пожалуйста на почту.

1553. логорифм, 5 июня 2010, 11:06:27
Я хочу сказать следующее....Русский язык-егэ...плохой...давай товариш здават Армянский Йызык Хочу очэнь сдавать..и секаса оч с комиссией хочу!

1554. ilo, 5 июня 2010, 11:54:37
Екатерина 1534- R основания цилиндра из ф-лы: 9п=пR^2, R=3. Из прямоугольного треугольника в диагональном сечении гипотенуза 2sqrt34, один катет-образующая, второй-диаметр основания цилиндра-3*2=6,находим образующую по т.Пифагора 4*34-36=100=10^2 tg=6/10=0,6

1555. Владимир, 5 июня 2010, 13:22:49
на гипотенузе прямоугольного треугольника с катетами a и b как на стороне во внешнюю сторону построен квадрат. Найдите расстояние от вершины прямого угла треугольника до точки пересечения диагоналей квадрата.


Помогите, срочно нужно решение.

1556. Lexxus, 5 июня 2010, 14:13:38
на гипотенузе прямоугольного треугольника с катетами a и b как на стороне во внешнюю сторону построен квадрат. Найдите расстояние от вершины прямого угла треугольника до точки пересечения диагоналей квадрата.

Вообще-то, это не похоже на ЕГЭшную задачу.



На каждой из оставшихся сторон квадрата достроим по такому же треугольнику.
Получится квадрат со стороной (a+b). Точка, которую нам нужно найти, является точкой пересечения и его диагоналей, которая, как известно, отстоит от каждого из углов квадрата на
sqrt(2)/2*(длина стороны) = sqrt(2)*(a+b)/2

1557. Elli, 5 июня 2010, 15:33:31
Владимир! треуг САВ-прямоугольный, угол САВ-прямой, СВ-гипотенуза= корень из (акв+в кв), синус АСВ=в/(корень из(акв+в кв)), соs АСВ=а/(корень из(акв+в кв)), СЕ=половина диагонали квадрата,треугольник СЕВ-прямоугольный и равнобедренный,СЕ=(корень из(акв+в кв))/КОРЕНЬ ИЗ 2,угол ВСЕ=45 градусов,треугольник АСЕ-уголАСЕ=угол АСВ+45градусов. косинус АСЕ=косинус(АСВ+45)=((а/(корень из(акв+в кв)))*(корень из 2)/2)+(в/(корень из(акв+в кв)))*(корень из 2)/2)=(корень из 2*(а+в))/2корняиз(акв+в кв). по т косинусов АЕ квадр=а(а+в)-это после всех преобразований. соответственно АЕ=корень из(а кв+ав). это мои рассуждения.не могу сказать,что точно

1558. Elli, 5 июня 2010, 15:44:46
АААААААА! Я НЕ увидела решения Лексуса! Значит у меня неверно.
Lexxus! я не очень поняла запись
sqrt(2)/2*(длина стороны) = sqrt(2)*(a+b)/2 . это значит, что сторона равна а+в? ,т.е.(sqrt(2)*(a+b)/2 )/(sqrt(2)/2)

1559. Lexxus, 5 июня 2010, 15:50:06
АААААААА! Я НЕ увидела решения Лексуса! Значит у меня неверно.
Lexxus! я не очень поняла запись
sqrt(2)/2*(длина стороны) = sqrt(2)*(a+b)/2 . это значит, что сторона равна а+в? ,т.е.(sqrt(2)*(a+b)/2 )/(sqrt(2)/2)

Сторона большого квадрата - a+b, это, кажется, видно из рисунка.
Половина длины диагонали (как раз нужное нам расстояние) равна стороне квадрата, умноженной на корень из двух пополам.

1560. Elli, 5 июня 2010, 15:55:12
А)) у меня тоже было правильно, только я АЕ не до конца посчитала.АЕ квадрат= а кв+(а кв+в кв)/2- (а кв+ав)=(а+в) квадрат=> АЕ=а+в. вот!

1561. Elli, 5 июня 2010, 16:11:30
Ну да, я тупая((. потеряла минус в формуле косинус суммы и потеряла 2 в знаменателе.
Лексус! у тебя,как всегда красиво!

1562. Анна, 5 июня 2010, 17:38:21
Задание B1 (6191)

(показов: 794, ответов: 303)


В городе N живет 300000 жителей. Среди них 10 % детей и подростков. Среди взрослых 35% не работает (пенсионеры, домохозяйки, безработные). Сколько взрослых работает?

Ответ: 175500

Пожалуйста, кто-нибудь подскажите решение задачи!!!!
У меня ответ не получается!!!!

1563. ilo, 5 июня 2010, 18:37:41
Aнне: 10%-это 1/10 часть от 300тыс., т.е. 30тыс.детей и подростков, взрослых-300-30=270(тыс).из них 35% не работают,значит работают 65%- 270т.*0,65=175,5тыс.=175500чел.

1564. Анна, 5 июня 2010, 19:12:46
Задание B4 (4853)
(показов: 2349, ответов: 614)

В параллелограмме АВСД sinА= 0,8. Найдите cosВ .

Ответ: -0.6

помогите пож решить не знаю с чего начать

1565. ilo, 5 июня 2010, 19:44:10
cosB=cos(180-A)=-cosA
cosA из основного триг. т-ва=о,6(А-угол острый), а -cosA=-0,6

1566. Вероника-, 5 июня 2010, 20:10:14
7 -го уже ЕГЭ...пипец как ужасно страшно... А я умею решать только задания В и некоторые из С1 го могу решить...остальные С мы с учителями и решать не брались(((

1567. Анна, 5 июня 2010, 20:12:34
спасибо огр

1568. smoke, 5 июня 2010, 20:36:31
Груз маccой 0,06 кг колеблетcя на пружине cо cкороcтью, меняющейcя по закону v=2sinПt , где t — время в cекундах. Кинетичеcкая энергия груза, измеряемая в джоулях, вычиcляетcя по формуле E= m*v^2/2 , где m — маccа груза (в кг), v — cкороcть груза (в м/c). Определите, какую долю времени из первой cекунды поcле начала движения кинетичеcкая энергия груза будет не менее 3 * 10^ -2 Дж. Ответ выразите деcятичной дробью, еcли нужно, округлите до cотых.

напишите решение желательно подробно просто в других задачах такого же типа у меня ответ сходился а тут нет(
заранее спасибо!

1569. smoke , 5 июня 2010, 20:38:29
у меня 0,17 получается почемуто а правильный ответ 0,67 пишет...

1570. Анна, 5 июня 2010, 21:12:41

Задание B6 (27591)

(показов: 62, ответов: 17)


Найдите площадь треугольника, две стороны которого равны 8 и 12, а угол между ними равен 30.



Ответ: 24
дайте идею по решению плиз...

1571. ilo, 5 июня 2010, 21:20:48
Анне: S=1/2absina^b

1572. Ольга, 5 июня 2010, 21:23:08
Здесь есть решения заданий части С с досрочного экзамена?

1573. Анна, 5 июня 2010, 22:00:02
Задание B9 (25857)

(показов: 430, ответов: 99)


Объем параллелепипеда равен 6. Найдите объем треугольной пирамиды .



Ответ: 1

картинку не скопировать посм по номеру пож что-то ответ не получается у меня ответ 3 помогите ilo пож)))))))

1574. ilo, 5 июня 2010, 22:08:21
SMOK 0,06*v^2/2>3*10^-2; 0,06v^2>0,06; v^2>1; v<-1 или v>1; 2sinпt>1; sinпt>1/2; 5п/6+2пn<пt<п/6+2пn; 5/6+n<t<1/6+n, время задано в пределах 1с: 5/6<t<1/6, это 5/6-1/6=2/3=0,66...(знак везде больше либо равно)

1575. ilo, 5 июня 2010, 22:29:57
Vпир.=1/3Sосн.пир.*h; Vпар.=Sосн.пар.*h=6; высоты у них равны, Sосн.пир.=1/2Sосн.пар.; Vпир.=1/3*1/2Sосн.пар*h=1/6Sосн.пар.*h=1/6*6=1

1576. Анна, 5 июня 2010, 22:37:59
ilo !!! спасибо. а завтра вечером вы будете на связи???? будем дальше решать.......

1577. smoke , 5 июня 2010, 22:44:45
ilo большое спс )

1578. неАйс, 5 июня 2010, 22:51:04
объясните пожалуйста почему ответ 1 ?

http://live.mephist.ru/show/mathege2010/view/id/25857/

у меня 2 получается

1579. неАйс, 5 июня 2010, 23:02:47
ilo,спасибо! но почему вот такое решение неправильное, Vпир.=1/3*BB1*Sосн.пир=1/3*BB1*AB*BC*sin90=1/3Sосн.пар

1580. неАйс, 5 июня 2010, 23:09:44
разобрался, в формуле площади треугольника 1/2 забыл...

1581. надежда 2010, 6 июня 2010, 00:25:51
Александра, по поводу задачи: пусть Х- число десятков, Y- число единиц, тогда само искомое число: 10Х+Y, составим систему: 10Х + Y =2.5( Х + Y)
и
10Х + Y =3ХY, решаем, получим искомое число равно 15.

2) Пусть Х руб. стоит куртка, тогда 0,8Х стоят 4 рубашки, 0,2Х руб. стоит 1 рубашка, а 1.2Х руб стоят 6 рубашек, тогда 6 рубашек дороже куртки на 20%

1582. ilo, 6 июня 2010, 07:38:25
Надежде 2010: пожалуйста, сверим ответы к задаче из поста 1490?

1583. Asiya7, 6 июня 2010, 10:08:33
На рисунке изображен график функции F(x), определенной на интервале (-3,9). Найдите количество точек, в которых касательная к графику функции параллельна прямой y=-7.


Объясните как решать такие задачки, пожалуйста!

1584. Мотивация, 6 июня 2010, 10:14:37
В треугольнике ABC угол C равен 90 градусов, косинус А равен 10 деленное на два корня из восьмидесяти девяти. Найдите тангенс А . Есть решение?

1585. Ника, 6 июня 2010, 10:36:48
Мотивация...потеореме Пифагора СВ=16, тангенс = 16/10=1.6

1586. Ника, 6 июня 2010, 10:39:39
Asiya7 на графике начерти прямую у=-7 и смотри сколько раз оа пересекает график, это и будет ответ

1587. Настя, 6 июня 2010, 10:52:36
В сосуде было 12л. соляной кислоты. Часть кислоты отлили и сосуд долили водой.Затем снова отлили столько же и опять долили водой. Сколько жидкости отливали каждый раз, если в сосуде оказался 25%-ный раствор кислоты

1588. Asiya7, 6 июня 2010, 11:07:37
Ника, а если совсем не пересекает? и как с касательной быть?

1589. Юлия, 6 июня 2010, 11:20:40
в основании прямоугольной призмы лежит квадрат со стороной 5 .боковые ребра равны 7/п.найдите обьем цилиндра,описанного около этой призмы.
пожалуйста можно решение побыстрее,а то сижу только над этими задачами,завтра уже экз ,а с ними проблема,пожалуйста,заранее очень благодарна

1590. ника, 6 июня 2010, 11:21:03
Asiya7,сначала касательную начерти так чтоб она была пар-на у=-7...и потом смотри сколько раз касательная пересекает график...

1591. Людмила, 6 июня 2010, 11:26:39
http://live.mephist.ru/show/mathege2010/view/id/6419/ Как решается?

1592. Юлия, 6 июня 2010, 11:31:23
ПОЖАЛУЙСТА ПОМОГИТЕ)

в основании прямоугольной призмы лежит квадрат со стороной 5 .боковые ребра равны 7/п.найдите обьем цилиндра,описанного около этой призмы

1593. Людмила, 6 июня 2010, 11:36:44
На рисунке изображен график функции , определенной на интервале (-5, 6) . Определите количество целых точек, в которых производная функции отрицательна.
Как решать подобные задачи?

1594. надежда 2010, 6 июня 2010, 11:59:38
Asiya7 проблема легко снимается: посчитай на графике все горбы и впадины, это и будут ьеста(точки) касания.

1595. Л, 6 июня 2010, 12:11:56
Объем конуса равен 120. Через середину высоты параллельно основанию конуса проведено сечение, которое является основанием меньшего конуса с той же вершиной. Найдите объем меньшего конуса.



1596. Asiya7, 6 июня 2010, 12:17:13
Надежда, спасибо!

1597. smoke, 6 июня 2010, 12:37:56
Л тебе : т.к сечение через серидину то высота и радиус нового конуса будут в 2 раза меньше поэтому составляем отношение их обьемов . получаем что обьем нового конуса в 8 раз меньше первого т.е 120/8=15

1598. Elli, 6 июня 2010, 12:50:53
Помогите решить С4!!!
дан прямоугольный треугольник АВС С прямым углом при вершине В и углом а при вершине А. Точка D - середина гипотенузы. Точка С1 симметрична точке С относительно прямой ВD. Найдите угол АС1В.

1599. ыть, 6 июня 2010, 13:01:27
егэ - это вынос мозга. Я решаю гдето половину заданий, и сказать что мне страшно - ничего не сказать.

1600. Леонид , 6 июня 2010, 13:15:32
1598. Elli, 6 июня 2010, 12:50:53

Приве. Давненько не появлялась. Задачу только увидел, посмотрю чиркну. А из какого источника, что С4 ?

ПОСМОТРИ. С4-хорда АВ и диаметр MN одной и той же окружности не пересекаются,а точка пересечения прямых AM и BN равноудалена от концов хорды АВ на расстояние 3.Найти радиус окружности,если уголANM равен 30градусов.

Ясно, что через середину АВ диаметри перпеникуляр, коль речь идёт о равноудалённости на 3. Вопрос - обязательно ли диаметр МН паралелелен АВ- для убедительности. ЖДУ!

1601. Thecutelittledeadgirl, 6 июня 2010, 13:15:52
Задание B11 (3831)

Найдите точку максимума функции y = (x+17)*e в степени (17-x).

Ответ: -16
Помогите, пожалуйста, у меня получается точка минимума, в добавок другая!

1602. Сайка, 6 июня 2010, 13:53:05
всем ПРИВЕТ)))))) ВСЕ готовы К завтрашней Битве?)))))

1603. Светланка, 6 июня 2010, 14:02:16
длины двух сторон треугольника 4 и 6. найдите наибольшее возможное значение длины третей

1604. аля, 6 июня 2010, 14:23:38
света я думаю так: если а- третья сторона, то 4+6 больше а. отсюда а должен быть меньше 10, значит наибольшее возможное целое значение равно 9

правильно я рассуждаю????)))))

кто может помочь со сдачей ЕГЭ по матем, отзовитесь)))


1605. аля, 6 июня 2010, 14:32:13
можно ли ночью на 7 июня узнать максимально приближенные варианты ЕГЭ по матем???? если да, то откуда???

1606. терминатор, 6 июня 2010, 14:35:41
В треугольнике АВС АС=ВС, АВ=60,соsА=3/5 . Найдите высоту CН.
Подскажите как решается:)

1607. терминатор, 6 июня 2010, 14:42:02
Аля говорят на этом сайте что то типа этого...Правда ли это не знаю)http://abiturient.pro/

1608. Юля, 6 июня 2010, 14:42:15
Терминатор, ответ 4.
учитывая равные стороны треугольника, получается, что он равнобедренный. И, кстати, там АВ=6, а не 60. cosA=3/5. cosA=AH/AC => AC=5, AH=3. (AC)2(в квадрате, по т.Пифагора)=AH2+CH2
CH2=AC2-AH2
CH=(25-9)<- все это под корнем=4.
с Яндекса, да?)

1609. Thecutelittledeadgirl, 6 июня 2010, 14:44:06
Т. к. треугольник равнобедренный, то СН- высота, бессектриса, значит АН = 60:2=30. cosА=АН:АС, следовательно АС=50, по теореме Пифагора:СН = 40

1610. Thecutelittledeadgirl, 6 июня 2010, 14:46:48
Пожалуйста! Как это решить, чтоб этот ответ получился?!
Задание B11 (3831)

Найдите точку максимума функции y = (x+17)* e в степени (17-x).

Ответ: -16

1611. терминатор, 6 июня 2010, 14:51:37
нет *открытый банк задач егэ. там АВ=60 было...
все равно спасибо, помогли..)

1612. Thecutelittledeadgirl, 6 июня 2010, 15:14:37
Терминатор, а он у тебя работает?
У меня ни в какую не загружается.. (mathege.ru)

1613. терминатор, 6 июня 2010, 15:23:31
--------------------------------------------------------------------------------

Thecutelittledeadgirl, -16 никак не получается, может 18

1614. Thecutelittledeadgirl, 6 июня 2010, 15:27:28
И я так думаю! Спасибо, приятно иметь единомышленника)
Такой дан ответ на этом сайте. У меня, ко всему прочему, точка мин., а не мах получилась

1615. Светланка, 6 июня 2010, 15:27:50
подскажите пожалуйста, как решаются задания, когда дан график функции и надо найти производную в определенной точке. Аля, спасибо вам огромное

1616. терминатор, 6 июня 2010, 15:31:53
Thecutelittledeadgirl та же самая фигня..:(

1617. krisstal, 6 июня 2010, 15:49:42
Задание B11 (3831) - согласна что там отве не -16 а -18!!!!
или может я не правильно решаю??
там производная : с е вынести за скобку и останется (1+х+17) = (18+х)=0 х=-18

почему ответ то -16?(((((

1618. Thecutelittledeadgirl, 6 июня 2010, 15:57:40
krisstal, даже само задание сформулированно неправильно. Требуют мах. функции, а там только одна точка, и она -18, и она мин.!
Причем, кажется, таких ошибок здесь много...

1619. терминатор, 6 июня 2010, 16:14:09
а тут только 11классники?;)

1620. ilo, 6 июня 2010, 16:16:23
когда берётся производная от е^17-x как от сложной функции нужно ещё домножить на производную от аргумента, а это -1, 1-х-17=0, х=-16

1621. Serdzeedka, 6 июня 2010, 16:17:37
Помогите пожал.решить задачку:"Лодка в 8:00 вышла из пункта А в пункт В, расположенный в 15 км от А. Пробыв в пункте В 2 часа, лодка отправилась назад и вернулась в пункт А в 20:00. Определите (в км/ч) скорость течения реки, если известно, что собственная скорость лодки равна 4 км/ч."
Заранее спасибо!=)

1622. katya.16ru, 6 июня 2010, 16:22:32
на рисунке изображен график функции у=ф(х),определенной на интервале (-7;5).
найдите количество точек,в которых касательная к графику функции параллельна прямой у=-13. объясните алгоритм решения таких заданий

1623. Thecutelittledeadgirl, 6 июня 2010, 16:31:01
ilo! Спасибо огромное!
А я уже начала паниковать: завтра уже экзамен, а я В11 решать не умею)

1624. надежда 2010, 6 июня 2010, 16:39:25
Thecutelittledeadgirl В11 найдите точку максимума функции:

когда находишь производную, не забудь, что в степени (показатель) Х с минусом, ответ 16



ВСЕМ УДАЧИ!!!!!!!!!!!!!!

1625. Thecutelittledeadgirl, 6 июня 2010, 16:43:13
Serdzeedka
Примем v реки за х. Всего лодка в пути была 10 часов (20:00-8:00-2 часа стоянки).
Пусть туда лодка плыла ПО течению реки и ее v1=Vлодки+х, тогда обратно ПРОТИВ течения со V2=Vлодки -х. Т. к. расстояние и туда и обратно 15 км, то туда лодка плыла (t1=15/v1), а обратно (t2=15/v2).
Уравнение: t1+t2=10.
Все подставь, ответ х=2

1626. Thecutelittledeadgirl, 6 июня 2010, 16:44:48
Спасибо,
ВСЕМ 100 БАЛЛОВ!))

1627. Serdzeedka, 6 июня 2010, 16:48:45
Thecutelittledeadgirl,спасибо большое!!!!!=)

1628. надежда 2010, 6 июня 2010, 16:50:18
конечно же 16 с минусом!(см. выше).

katja.16ru прямая y=-13 параллельна оси ОХ, посчитай количество горбов и впадинок на графике функции, это и будет число искомых точек.

1629. терминатор я , 6 июня 2010, 16:51:11
опять облом...все больше и больше нового:)

1630. ilo, 6 июня 2010, 17:01:57
КАTУА.16RU чтобы касательная была // прямой, нужно, чтобы у них совпадали угловые коэффициенты, у=0х-13, к=0, Ккас=0тоже, а это в точках экстремумов.

1631. ilo, 6 июня 2010, 17:12:19
простите: экстремумов или перегибов.

1632. Thecutelittledeadgirl, 6 июня 2010, 18:31:17
Скажите, корень (х в квадрате - х)=-2корня из 3, такое может быть?
Да и вообще, корень может быть равен отрицательному числу? Именно не подкоренное выражение, а сам корень?

1633. ilo, 6 июня 2010, 18:40:38
если это ур-е, то корней нет, если корень квадратный,то он неотрицателен(больше или равен 0),если промежуточный результат, то может ошибка?

1634. ТоМуСьКа, 6 июня 2010, 18:54:15
Все Завтра удачи.
Ни пуха!
У меня последний экзамен 9 числа,вождение и все,я свободна буду.

1635. Thecutelittledeadgirl, 6 июня 2010, 19:01:17
Все, я поняла. Я как раз пыталась понять, почему у меня написано "нет решения".
Меня просто смущает, что корень из 4 может быть равен -2, а другие корни что, предатели родины?

1636. Rassti, 6 июня 2010, 19:01:36
Всем удачи! Через неделю много не пейте.

1637. ilo, 6 июня 2010, 19:13:27
Насте: раствора осталось 12л как и было, в этих 12л собственно кислоты 25%-1/4 часть раствора, т.е.3л.Остальные 9л-долитая вода, доливали два раза поровну,по 4,5л.

1638. Леночка, 6 июня 2010, 19:19:35
Привет! Ночью будут известны КИМы?

1639. Lika, 6 июня 2010, 19:32:08
Люди,родненькие,объясните В8=(
не пойму как находить наибольшее или наименьшее значение,или в каких точках функция положительна=(

1640. ilo, 6 июня 2010, 19:36:49
наверное, всё дело в D(f) самого корня,по определению для sqrt a a больше, либо равно 0, а с sqrt4 что взять?

1641. Мифка, 6 июня 2010, 19:37:37
Всем привет!Жаль что сегодня только наткнулся на этот сайт:(
Всем удачи завтро! и кстате если есть тут кто с Сахалина напишите свой вариант:)

1642. Thecutelittledeadgirl, 6 июня 2010, 19:51:51
Lika,
во-первых, всегда смотри, что изображено: функция или ее производная.
Еще запомни, что функция возрастает там, где производная положительна. Например, задание 6404. Изображена производная, на промежутке [0;6] она только и делает, что положительна, значит f. все время возрастает, а значит наибольшее значение она принимает в последней точке промежутка (6).

1643. Elli, 6 июня 2010, 19:54:07
Леонид! Завтра экзамен! схожу с ума((
С4-хорда АВ и диаметр MN одной и той же окружности не пересекаются,а точка пересечения прямых AM и BN равноудалена от концов хорды АВ на расстояние 3.Найти радиус окружности,если уголANM равен 30градусов.

Ясно, что через середину АВ диаметри перпеникуляр, коль речь идёт о равноудалённости на 3. Вопрос - обязательно ли диаметр МН паралелелен АВ- для убедительности. ЖДУ!
у мменя получился радиус=3.МН будет параллелен АВ-точно. по свойству секущих и окружности.т.е например т. пересечения прямых AM и BN будет Р. Тогда по свойству РА*РМ=РВ*РН, А т.к. РА=РВ,то РМ=РН И АМ=ВН, треугольники МАН И МВТ-РАВНЫЕ прямоугольные,высоты к диаметру-равные,поэтому АВ параллельно МН. Но разве это надо было доказывать? по-моему, это ясно из условия.ведь не сказано, что они не пересекаются внутри окружности,а просто, что не пересекаются(т.е.никогда)))

1644. Thecutelittledeadgirl, 6 июня 2010, 19:58:50
Задание 6427, изображена снова производная, точки экстремума - те точки, где производная = 0, значит считаем все точки, где кривая пересекает ОХ на [-4;4].

6422. На этот раз изображен график самой функции. Нам известно, что функция возрастает там, где производная положительна, значит и наоборот, производная положительна там, где функция возрастает. Мы видим, что таких промежутков несколько, но тех, что содержат целую точку (2) - тольок один.

1645. Elli, 6 июня 2010, 20:03:04
Леонид! я нашла решение своей задачи. на вот этом сайте.http://www.egetrener.ru/view_zadachi.php?zadachi=C4&page=3
жаль только, что я его поздно обнаружила((. Там видеоуроки, всё наглядно. но только не успею всё посмотреть уже,жаль.

1646. Thecutelittledeadgirl, 6 июня 2010, 20:04:01
Lika, пиши номер задания - объясню

1647. ilo, 6 июня 2010, 20:13:03
Lika: берёшь производную, приравниваешь её к 0 или смотришь, где она несуществует, проверяешь, лежат ли они на заданном отрезке,не лежат-отбрасывай, считаешь значения функции на концах промежутка и в тех точках,которые не отбросила.Берёшь самое большое число,которое получилось-унаиб.,самое малое-унаим.Лучше конкретно пример приведи.

1648. Ищущий Правду, 6 июня 2010, 20:13:25
Люди пожалуйста помогите с B11: экзамен на носу!http://s47.radikal.ru/i118/1006/c8/92e00c92960a.jpg

1649. Ololololo, 6 июня 2010, 20:13:39
http://live.mephist.ru/show/mathege2010/view/id/6139/

помогите решить,ваще не пойму(((((

1650. Ellen, 6 июня 2010, 20:15:38
Всем привет) Помогите, пожалуйста, решить задание с3.
(1/2)в степени log2(x^2-1)>1

1651. Lika, 6 июня 2010, 20:25:00
Thecutelittledeadgirl
а почему в 6404 не может быть 1?

1652. Darkstein, 6 июня 2010, 20:28:08
http://vkontakte.ru/id69665827 это моя страничка в контакте пишите туда, помогу с ц3 ))

1653. ilo, 6 июня 2010, 20:34:38
производная: -4sinx-27/п=0; sinx=-27|п<-1 в 0 не обращается-монотонная,
ищем значения f(x) на концах:
у(-2п/3)=-1/2+27/п*2п/3+3=20,5
у(0)=4+3=7
Унаиб.=у(-2п/3)=20,5

1654. trust, 6 июня 2010, 20:36:37
Дайте сайт где выкладывали егэ по русском и ин.языкам во Владивостоке.....мб там приблизительные задания по математике выложат?

1655. ilo, 6 июня 2010, 20:47:17
Ololo: Т1-300/Т1*100%>40%; сокращаем на 10,приводим к общему зн-лю:
10Т1-3000/Т1-4Т1>0; 10Т1-3000-4Т1>0; 6Т1>3000; Т1>500 (знак везде -больше либо равно) Тнаим.=500

1656. Алина, 6 июня 2010, 20:47:53
пожалуйста подскажите как решать...оч оч нужно))))))))
Зависимость объема спроса q (тыс. руб.) на продукцию предприятия-
монополиста от цены p (тыс.руб.) задается формулой q = 70 &#8722; 5p . Выручка
предприятия за месяц r (в тыс. руб.) вычисляется по формуле r ( p) = q &#8901; p .
Определите наибольшую цену p, при которой месячная выручка r ( p)
составит не менее 240 тыс. руб. Ответ приведите в тыс. руб.

1657. Денис, 6 июня 2010, 20:49:39
B8 (№ 6011) по математике кто может помочь с решением?

1658. Ищущий Правду, 6 июня 2010, 20:58:39
Спасибо ilo!

1659. ilo, 6 июня 2010, 21:05:09
Ellen: 1=(1/2)^0; log2(x^2-1)<0; 0=log21; x^2-1<1; x^2-2<0;
-sqrt2<x<sgrt2; наложим решение на D(f): x^2-1>0 -1<x<1
Ответ: -1<x<1

1660. Thecutelittledeadgirl, 6 июня 2010, 21:22:01
Lika, потому что ты путаешь производную и саму функцию! Тебе дан график производной, а спрашивают про ФУНКЦИЮ! То, что в точке 1 производная высоко не значит, что функция тоже должна быть самой большой.

1661. Thecutelittledeadgirl, 6 июня 2010, 21:29:29
Денис, так как прямая параллельна касательной, то k у них одинаковый. y=7x+11 и y=kx+b, похоже, не правда ли? Поэтому k=7.
Ищем производную от второй функции y'=2x+8=k,
y'=2x+8=7
x=-1/2

1662. Денис, 6 июня 2010, 21:35:55
Спасибо большое)

1663. Денис, 6 июня 2010, 21:40:31
простите за надоедливость но еще один вопрос.

B8 (№ 27486)

не подскажете как быть тут?
так же работать с производными или приравнять функции?

1664. денис, 6 июня 2010, 21:58:44
минимальный балл по русскому - 36 баллов.
http://www1.ege.edu.ru/index.php?option=com_content&task=view&id=817&Itemid=201

1665. funduk, 6 июня 2010, 22:02:21
У кого-нибудь есть электронный вариант различных конспектов,шпор,примеров с решениями??Выложите плис!
Или сюда,или funduk.funduk@ya.ru


Всем удачи завтра!!! =))

1666. funduk, 6 июня 2010, 22:10:02
ЕЕЕ...

Уже 3 форума накрылись =)))

1667. funduk, 6 июня 2010, 22:15:00
А я так хотел доразбирать задания С5...
И эти нервы...Не смог решить задание с1 варианта 1 Лысенко...И решение чтот не очень понял...

Всё очень плохо...

А тут тишина...

Разговор с самим собой...

Моожет пойти порешать?Да,иди. =))

1668. Thecutelittledeadgirl, 6 июня 2010, 22:18:59
Так же с производной, мне кажется. В этом случае ответ должен быть -1.

1669. фундук, 6 июня 2010, 22:21:57
А удачи мне поделать не кто не хочет?=))

1670. Денис, 6 июня 2010, 22:25:56
а почему не -3,66?
там же два корня получается. на каком основании его можно исключить?

1671. ilo, 6 июня 2010, 22:32:33
Денис: 3Хo^2+14Xo+7=-4; Xo=-1, Xo=-11/3, подставляем в уравнения касательной и ф-ции, -1 подходит, -11/3 нет,надо бы -11/3 еще раз проверить, а пока Хо=-1

1672. ilo, 6 июня 2010, 22:53:54
после проверки: угловые коэффициенты в точке -11/3 совпадают, но прямая у=-4х-11 касательной не будет.

1673. СТЕРВА, 6 июня 2010, 23:08:10
ПОМОГИТЕ МНЕ КТО НИБУДЬ ЗАВТРА НА ЕГЭ МОЙ НОМЕР АСИ 454221602

1674. Леонид , 6 июня 2010, 23:46:42
1643. Elli, 6 июня 2010, 19:54:07
Да параллельны - это просто поясняется углами: тр. АРВ и МРК правильные.. Иди от данного 30. ему пртивоположный 120. т.к. чет-к вписан в окр. и т.д. Пояснение д.б.

Твоя задача проста дуга 2а по впис. углу. другая пи-2а (тр. с 90 град -вписан). то искомый угол пи/2-а. Но врядли такие будут на ЕГЭ - это очень просто! Я видел там о чём ты пишешь, но мне кажется это просто приманка, судя по мониторингу и пробному в разных регионах их задания очень просты.

1675. Elli, 7 июня 2010, 00:23:31
Леонид! Спасибо за разъяснение! Удачи нам завтра на егэ! После экзамена отпишись, какие попались задачи в с части, я тож напишу. А то егэ пройдёт- и не будем на это сайт заходить - как-то даже жалко расставаться ))

1676. Allod-ONE, 7 июня 2010, 00:31:34
А косинус может равняться нулю? или быть меньшего него?

1677. killersss90, 7 июня 2010, 07:25:42
прямоугольный параллепипед описан около цилиндра радиус которого равен 2.объем параллепипеда равен 48. найдите высоту цилиндра

1678. Русь, 7 июня 2010, 07:38:48
помогите мне на ЕГЭ...умоляю вас))))
пишите в асю 363742430

1679. ilo, 7 июня 2010, 07:53:17
killersss90: в основании круг вписан в квадрат, сторона основания равна 4, площадь основания- 16 16*h=48 h=3

1680. Марина, 7 июня 2010, 08:19:18
1041 ответьте)) прошу, срочно!!!((

1681. Лиана, 7 июня 2010, 08:55:31
Найдете наибольшее значение функции у равно 6 кос х - 21/п+7 на отрезке [-2п/3;0]

В треугольмие абс угол с равен 90 градусов,кос б равен 4/5,аб равно 10.найдете ас

1682. Лиана, 7 июня 2010, 08:56:52
помогите пожалуйста срочно ребят

1683. ilo, 7 июня 2010, 09:14:30
Лиана, 1) 18
2) 6

1684. Persona-Prazdnik, 7 июня 2010, 09:48:58
Баржа в 10:00 вышла из пункта А в пункт В, расположенный в 15 км от А. Пробыв в пункте В 1 час 20 минут, баржа отправилась назад и вернулась в пункт А в 16:00. Определите (в км/час) скорость течения реки, если известно, что собственная скорость баржи равна 7 км/ч.


кто решит задачу .....тому закину на мобильный 500 рублей

помогите.....

1685. Лиана, 7 июня 2010, 10:13:36
ilo спасибо большое!!!!!

1686. Николай, 7 июня 2010, 10:48:11
Прямоугольный параллелипипед описан около цилиндра,радиус основания которого равен 3.Объем параллелипипеда равен 36.Найти высоту цилиндра. пожалуйста помогите.....

1687. Андрей, 7 июня 2010, 10:49:05
Катер в 10 00 вышел из а в пункт в,расположенный в 15от А.пробыв в пункте б 4 часа ,катер отправился назад и вернулся в пункт а в 18 00 того же дня!! Определите в км/ч собственную скорость катера,если известно ,что скорость теч реки равна 2 км/ч

1688. Никита, 7 июня 2010, 10:49:24
Треугольник АВС угол С равен 90 градусов, ав равен 30, ас равен 24 найдите син А


помогите)

1689. Серёга , 7 июня 2010, 10:50:35
Всем привет! Люди, помогите моей девушки.. У неё экзамен, она в аське сидит, не можем решить задачу.

В прямоугольном параллелепипеде АВ СД А1 В1 С1 Д1 известны ребра АВ=5 АД=12 СС1=3 Найдите угол между плоскостями ВД Д1 и АД В1

Заранее спасибо.. Если кто здесь есть, ответ если можно быстрей

1690. надежда 2010, 7 июня 2010, 10:56:24
Николай, высота цилиндра равна 1, радиус 3, то ребро основания 6, ов основании квадрат, его площадь 36, делим объем на площадь- получим 1

1691. Никита, 7 июня 2010, 11:00:40
Из пункта а в пункт в расстояние между которыми 50км,одновременно выехали автомобилист и велосипедист.известно что за час автомобилист проезжает на 40км больше велосипедиста.определите скорость велосипедиста если известно что он прибыл в пункт в на 4часа позже автомобилиста.ответ дать в км/ч



Ребята помогите плиз))

1692. надежда2010, 7 июня 2010, 11:01:16
никита, ответ 0.6

1693. Маша, 7 июня 2010, 11:04:04
Катер в 10:00 вышел из пункта А в пункт В ,расположенный в 15км от А . пробыв в пункте В 4часа ,катер отправился назад и вернулся в пункт А в 18:00 того же дня. Определите в (км /ч )собственную скорость катера ,если известно ,что скорость течения реки равна 2км /ч

1694. Серёга , 7 июня 2010, 11:04:35
Надежда 2010)))

А вы случайно не знаете ответ на мою задачу(((

Если знаете, помогите пожалуйста)))

1695. Маша, 7 июня 2010, 11:05:21
Никита, ответ 0.6

1696. Никита, 7 июня 2010, 11:07:25
Надежда благодарю))

1697. надежда2010, 7 июня 2010, 11:11:31
никита, по задаче: скорость велосипедиста 10км/ч

1698. Лина, 7 июня 2010, 11:12:46
помогите пожалуйста с этой задачей, срочно нужно решение!
В правильной треугольной пирамиде SАВС с основанием АВС известны ребра :АВ=30 под корнем 3,SС=34.найти угол образованный плоскостью основания и прямой АМ,где М-точка пересечения медиан грани SВС

1699. Марина, 7 июня 2010, 11:13:54
помогите плиз!!! Срочно надо задачку решить
Из пункта А в пункт В,расстояние между которыми 5О км,одновременно выехали мотоциклист и велосипедист.известно,что за час мотоциклист проезжает на 3О км больше чем велосипедист.определите скорость велика,если известно,что он прибыл в пункт В на 1,5 часа позже,чем мотоцикл.ответ в км

1700. Любовь, 7 июня 2010, 11:14:20
байдарка в 9 часов вышла из пункта А в пункт Б, расположенный в 15 км от А. Пробыв в пункте В 45 минут ,байдарка отправилась назад и вернулась в А в 16 часов того же дня. Определите (в км/ч) собственную скорость байдарки , если известно, что скорость течения реки равна 1 км/ч
срочно решите пожалуйста)))

1701. Vik, 7 июня 2010, 11:14:45
Помогите пожалста СРОЧНО!
y=4cosX+21/ПИ*x+9 на отрезке от [-2пи/3;0]

1702. Полина, 7 июня 2010, 11:15:04
Найдите наименьшее значение функции у=4х-4tgx+17 на отрезке [-4/П; 0]

кааак?

1703. Vik, 7 июня 2010, 11:16:32
И еще СРОЧНО!
4 в степени 2 плюс лог 11 по основаниям 4

1704. Никита, 7 июня 2010, 11:18:17
надежда2010
так ответ 0,6 или 10???

1705. марина, 7 июня 2010, 11:18:43
помогите пожалуйста! очень очень надо!
Катер в 11:00 вышел из пункта А в пункт В, расположенный в 15 км от А. Пробыв в пункте В 1 час 20 мин, катер отправился назад и вернулся обратно в пункт А в 15:00 того же дня. Определите собственную скорость катера , если известно что скорость течения реки равна 3км/ч

1706. полина, 7 июня 2010, 11:18:50
ВИК
4 в степени 2 плюс лог 11 по основаниям 4
16 умножить на 11
176

1707. Артур, 7 июня 2010, 11:21:28
Из пункта А в пункт В ,расстояние между которыми 40км ,одновременно выехали мотоциклист и велосипедист.известно,что за час мотоциклист проезжает на 50км больше ,чем велосипедист .определите скорость велосипедиста,если известно,что он прибыл в пункт В на 3часа 20минут позже мотоциклиста.ответ дайте в км/ч

1708. чЕЛОВЕК, 7 июня 2010, 11:21:35
Persona-Prazdnik, Баржа в 10:00 вышла из пункта А в пункт В, расположенный в 15 км от А. Пробыв в пункте В 1 час 20 минут, баржа отправилась назад и вернулась в пункт А в 16:00. Определите (в км/час) скорость течения реки, если известно, что собственная скорость баржи равна 7 км/ч.


кто решит задачу .....тому закину на мобильный 500 рублей

помогите.....


НА ВОПРОС ОТВЕТЬТЕ!!!!

1709. Екатерина , 7 июня 2010, 11:22:36
Людишки, помогите срочно, плииииз)))))

Моторная лодка в 11:00 вышла из пункта А в пункт В, расположенный в 15км от А. Пробыв в пункте В 1час 20 минут,лодка отправилась назад и вернулась в пункт А в 15:00 того же дня. Определите (в км/ч) скорость течения реки, если известно, что собственная скорость лодки равна 12 км/ч

1710. Андрей, 7 июня 2010, 11:23:21
Тетрадь стоит 40 рублей,какое наибольшее число таких тетрадей можно будет купить за 650 после понижения цены на 20 процентов

плз, помогитееее

1711. полина же, 7 июня 2010, 11:26:46
была 40 стала 32
20 штук

1712. Никита, 7 июня 2010, 11:27:05
Дан треугольник авс,ав-14,вс-10,са-12.точка д лежит на прямой вс так,что вд:дс=3:7.окружности вписанные в каждый из треугольников адс и адв касаются стороны ад в точках е и ф.найти длинну еф



ПОМОГИТЕ ПОЖАЛУЙСТА)))
СПАСИБО ЗА РАНЕЕ))

1713. KaterinaKa, 7 июня 2010, 11:29:29
y=8tg x - 8x- 2Пи +6 [-Пи/4; Пи/4
Pomogite Срочно=****

1714. Lexxus, 7 июня 2010, 11:30:51
Баржа в 10:00 вышла из пункта А в пункт В, расположенный в 15 км от А. Пробыв в пункте В 1 час 20 минут, баржа отправилась назад и вернулась в пункт А в 16:00. Определите (в км/час) скорость течения реки, если известно, что собственная скорость баржи равна 7 км/ч.

2 км/ч

Моторная лодка в 11:00 вышла из пункта А в пункт В, расположенный в 15км от А. Пробыв в пункте В 1час 20 минут,лодка отправилась назад и вернулась в пункт А в 15:00 того же дня. Определите (в км/ч) скорость течения реки, если известно, что собственная скорость лодки равна 12 км/ч

3 км/ч

Тетрадь стоит 40 рублей,какое наибольшее число таких тетрадей можно будет купить за 650 после понижения цены на 20 процентов

20

1715. Дима, 7 июня 2010, 11:32:54
Найдите наибольшее значение функции У = 2х-2tg-5 на отрез

1716. Любовь, 7 июня 2010, 11:33:00
ОЧЕНЬ СРОЧНОООООО!
ПОММОГИТЕ ПОЖАЛУУУЙСТА

байдарка в 9 часов вышла из пункта А в пункт Б, расположенный в 15 км от А. Пробыв в пункте В 45 минут ,байдарка отправилась назад и вернулась в А в 16 часов того же дня. Определите (в км/ч) собственную скорость байдарки , если известно, что скорость течения реки равна 1 км/ч

1717. Екатерина , 7 июня 2010, 11:33:37
Lexxus,



Спасибки тебе))))

1718. полина, 7 июня 2010, 11:33:57
лексус
Найдите наименьшее значение функции у=4х-4tgx+17 на отрезке [-4/П; 0]

реши мне это пожалуйста

1719. Николай, 7 июня 2010, 11:35:09
В прямоугольном параллелипипеде АВСDА1В1С1D1 известны ребра:АВ=5,АD=12,СС1=3.найти угол между плоскостями ВDD1 и АD1В1.
помогите пожалуйста!

1720. марина, 7 июня 2010, 11:36:07
Найдите наибольшее значение функции у=4tg-2tgx-П+9 На отрезке [0;П/3] - помогите ребят! пожалуйста!

1721. Андрей, 7 июня 2010, 11:36:27
спс большое, я вначале не так задачу понял, показалось, что после каждой покупки цена падает на 20%)))

1722. марина, 7 июня 2010, 11:37:23
Для много из предприятий-монополистов зависимость объема спроса на продукцию q (единиц в месяц) от ее цены p (тыс.руб.)задается формулой:q=16О-1Ор.определите максимальный уровень цены р (в тыс.руб.),при котором значение выручки предприятия за месяц r=qxр (x это умножить.)составит не менее 55О тыс.руб



1723. Серёга, 7 июня 2010, 11:37:31
В правильной треугольной пирамиде SABC с основанием ABC известны рёбра:AB=8v3, SC=10.найдите угол образованный плоскостью основания и прямой проходящий через середины рёбер AS и BC

1724. Николай, 7 июня 2010, 11:37:54
спасибо Надюшенька :* :*

1725. Маша, 7 июня 2010, 11:38:42
Помогите с задачей с катером)))
Катер в 10:00 вышел из пункта А в пункт В ,расположенный в 15км от А . пробыв в пункте В 4часа ,катер отправился назад и вернулся в пункт А в 18:00 того же дня. Определите в (км /ч )собственную скорость катера ,если известно ,что скорость течения реки равна 2км /ч

1726. Lexxus, 7 июня 2010, 11:40:00
Найдите наименьшее значение функции у=4х-4tgx+17 на отрезке [-4/П; 0]

17

1727. Никита, 7 июня 2010, 11:40:48
Из пункта а в пункт в расстояние между которыми 50км,одновременно выехали мотоциклист и велосипедист.известно что за час автомобилист проезжает на 30км больше велосипедиста.определите скорость велосипедиста если известно что он прибыл в пункт в на 1,5 часа позже автомобилиста.ответ дать в км/ч


помогите ребят))

1728. полина, 7 июня 2010, 11:42:24
лексус обожаю

то что марина написала задачу тоже ребят, кто может решите

1729. Lexxus, 7 июня 2010, 11:42:50
РЕБЯТ, СРАЗУ ПРЕДУПРЕЖДАЮ: Я НЕ БУДУ СЕЙЧАС РЕШАТЬ ЗАДАЧИ ЧАСТИ С - РЕШЕНИЯ СЛИШКОМ ДОЛГО ЗАПИСЫВАТЬ.

1730. Любовь, 7 июня 2010, 11:42:53
лексус, реши пожалуйста задачу)
ОЧЕНЬ НУЖНО..

байдарка в 9 часов вышла из пункта А в пункт Б, расположенный в 15 км от А. Пробыв в пункте В 45 минут ,байдарка отправилась назад и вернулась в А в 16 часов того же дня. Определите (в км/ч) собственную скорость байдарки , если известно, что скорость течения реки равна 1 км/ч

1731. Юрий, 7 июня 2010, 11:43:05
Срочно нужна помощь, помогите пожалуйста.....
3 в степени 2+logз 8
и
3 в степени 3+logз 11

1732. мари, 7 июня 2010, 11:43:32
Найдите наибольшее значение функции у=4tg-2tgx-П+9 На отрезке [0;П/3] - помогите пожалуйста!

1733. Никита, 7 июня 2010, 11:43:43
. Lexxus,


помоги пожалуйста)

1734. настя, 7 июня 2010, 11:45:01
7 в степени 2 +лог 7 в 4той степени

Прямоугольный параллелепипед описан около сферы радиуса 3.найти его объём.
ПОМОГИТЕ СРОЧНО РЕШИТЬ

1735. сергей, 7 июня 2010, 11:46:08
можете пожалуйста посчитать 7 в квадрате плюс log7в основание 6

1736. Ирена, 7 июня 2010, 11:48:15
В параллелепипеде ав равен 8.ад равен 6 и сс1 равен 3 найдите угол между плоскостями вдд1 и ад1в1 Помогите кто может..плиз!!!

1737. марина, 7 июня 2010, 11:48:19
сергей


можете пожалуйста посчитать 7 в квадрате плюс log7в основание 6

294

1738. полина, 7 июня 2010, 11:48:56
49 умножь на 7 мне лень

1739. Юрочка, 7 июня 2010, 11:49:27
очень нужна помощь...
Найдите наименьшее значение функции y=5x-5tgx+4 на отрезке(-п:4;о)

1740. Lexxus, 7 июня 2010, 11:49:38

байдарка в 9 часов вышла из пункта А в пункт Б, расположенный в 15 км от А. Пробыв в пункте В 45 минут ,байдарка отправилась назад и вернулась в А в 16 часов того же дня. Определите (в км/ч) собственную скорость байдарки , если известно, что скорость течения реки равна 1 км/ч

5 км/ч

Из пункта а в пункт в расстояние между которыми 50км,одновременно выехали мотоциклист и велосипедист.известно что за час автомобилист проезжает на 30км больше велосипедиста.определите скорость велосипедиста если известно что он прибыл в пункт в на 1,5 часа позже автомобилиста.ответ дать в км/ч

20 км/ч

1741. полина, 7 июня 2010, 11:49:50
марин нам нужна задача

добавь меня в аську 384600680

1742. Любовь, 7 июня 2010, 11:50:04
СРООООЧНО ПОМОГИТЕ. ОЧЕНЬ НУЖНООООООООО)

для одного из предприятий - монополистов зависимость объёма спроса на продукцию q (едениц в месяц) от её цены р (тыс.руб) задается формулой q=160- 10руб . определите максимальный уровеньцены в тыс руб, при котором значение выручки предприятия за месяц r=q*p составит не менее 550 тыс руб

Это часть С в ней надо писать не только ответ но и решение!

1744. Андрей, 7 июня 2010, 11:51:29
У=20tg(x)-20x +5pi-3

люди, выведите производную, пожалуйста

1745. сергей, 7 июня 2010, 11:51:33
1737. марина, 7 июня 2010, 11:48:19

спасибо ))

1746. Lexxus, 7 июня 2010, 11:51:44
Найдите наименьшее значение функции y=5x-5tgx+4 на отрезке(-п:4;о)

4

1747. Никита, 7 июня 2010, 11:51:45
Lexxus,

1748. Любовь, 7 июня 2010, 11:52:20
ЛЕКСУС, ОГРОМНЕЙШЕЕ СПАСИБО ТЕБЕ

1749. Альбик, 7 июня 2010, 11:52:40
а вот и я, родные=))
только с экзамена по математике.
с утра нашла ким с одним вариантом в интернете)) одно задание в егэ было такое же)
вооот. в части В все решила, а в С С1 и С6, короче хз как все.
задания не принесли удивления, все также, как и в демках.

1750. Никита, 7 июня 2010, 11:52:47
Lexxus,

спасибо)))

Дан треугольник авс,ав-14,вс-10,са-12.точка д лежит на прямой вс так,что вд:дс=3:7.окружности вписанные в каждый из треугольников адс и адв касаются стороны ад в точках е и ф.найти длинну еф



и эту пожалуйста если можно)))
спасибо за ранее))

1751. Любовь, 7 июня 2010, 11:54:46
ААА, НУ ЯСНО)
ВСЁ РАВНО СПАСИБО

лЕКСУС, помоги плииииз

7 в степени 2 +лог 7 в 4той степени

Прямоугольный параллелепипед описан около сферы радиуса 3.найти его объём.

1752. Человек, 7 июня 2010, 11:55:06
Спасибо лексус экзамен кончился=(( , ппц,

1753. Анюта, 7 июня 2010, 11:55:35
найдите наименьшее значение функции y=4tgx-4x-пи+7 на отрезке -пи/4;пи/4
огромная просьба помочь

1754. дима, 7 июня 2010, 11:55:51
Найдите наибольшее значение функции У = 2х-2tg-5 на отрез РЕШИТЕ ПОЖАЛУСТА.

1755. m@ri6k@, 7 июня 2010, 11:56:02
помогите плиз!!! Срочно надо задачку решить!!!!
Из пункта А в пункт В,расстояние между которыми 5О км,одновременно выехали мотоциклист и велосипедист.известно,что за час мотоциклист проезжает на 3О км больше чем велосипедист.определите скорость велика,если известно,что он прибыл в пункт В на 1,5 часа позже,чем мотоцикл.ответ в км

1756. Lexxus, 7 июня 2010, 11:56:26
У=20tg(x)-20x +5pi-3

люди, выведите производную, пожалуйста

20/(cos(x))^2-20

1757. полина, 7 июня 2010, 11:57:15
7 в степени 2 +лог 7 в 4той степени
в степени или по основанию?

1758. сашка666, 7 июня 2010, 11:57:20
Помогите задачу решить прямоугольный параллелепипед описан около сферы радиусом 4.5 найти объем параллелепипеда

1759. Никита , 7 июня 2010, 11:57:22
m@ri6k@

20 км/ч

1760. Loverman, 7 июня 2010, 11:57:24
1742. Любовь, 7 июня 2010, 11:50:04
СРООООЧНО ПОМОГИТЕ. ОЧЕНЬ НУЖНООООООООО)

для одного из предприятий - монополистов зависимость объёма спроса на продукцию q (едениц в месяц) от её цены р (тыс.руб) задается формулой q=160- 10руб . определите максимальный уровеньцены в тыс руб, при котором значение выручки предприятия за месяц r=q*p составит не менее 550 тыс руб

такая же задача - какой ответ все таки???

1761. Lexxus, 7 июня 2010, 11:58:21
найдите наименьшее значение функции y=4tgx-4x-пи+7 на отрезке -пи/4;пи/4

3

1762. марина, 7 июня 2010, 11:58:44
Найдите наибольшее значение функции у=4tg-2tgx-П+9 На отрезке [0;П/3] - помогите пожалуйста!

1763. полинэээ, 7 июня 2010, 11:59:09
1705 решите

1764. Lexxus, 7 июня 2010, 11:59:19
Помогите задачу решить прямоугольный параллелепипед описан около сферы радиусом 4.5 найти объем параллелепипеда

729

1765. любовь, 7 июня 2010, 11:59:42
в степенииии

1766. Маша, 7 июня 2010, 12:00:01
ну помогите с катером

1767. Машенька, 7 июня 2010, 12:00:44
Помогите плиз, потчти такая же
Б9 есть такое Прямоугольный параллелепипед описан около сферы радиуса 6,5. Найдите его объем.

1768. Lexxus, 7 июня 2010, 12:00:47
Найдите наибольшее значение функции у=4tg-2tgx-П+9 На отрезке 0;П/3 - помогите пожалуйста!

Напиши условие нормально.

1769. m@ri6k@, 7 июня 2010, 12:00:55
1759. Никита , 7 июня 2010, 11:57:22


спасибо огромное Никита

1770. сашка666, 7 июня 2010, 12:01:26
спасибо лексул)

1771. Никита, 7 июня 2010, 12:01:40
1742. Любовь, 7 июня 2010, 11:50:04
СРООООЧНО ПОМОГИТЕ. ОЧЕНЬ НУЖНООООООООО)

для одного из предприятий - монополистов зависимость объёма спроса на продукцию q (едениц в месяц) от её цены р (тыс.руб) задается формулой q=160- 10руб . определите максимальный уровеньцены в тыс руб, при котором значение выручки предприятия за месяц r=q*p составит не менее 550 тыс руб



каков ответ??

1772. настя, 7 июня 2010, 12:02:20
найти наименьшее значение У=16 танг.х-16х-4пи+3.на отрезке -пи на 4;пи на 4
плииииз, срочно надо

1773. Lexxus, 7 июня 2010, 12:02:53
Из пункта А в пункт В,расстояние между которыми 5О км,одновременно выехали мотоциклист и велосипедист.известно,что за час мотоциклист проезжает на 3О км больше чем велосипедист.определите скорость велика,если известно,что он прибыл в пункт В на 1,5 часа позже,чем мотоцикл.ответ в км

20 км/ч
Вы хоть проглядывайте предыдущие комментарии, я это уже писал.

1774. марина, 7 июня 2010, 12:03:40
ну функция у=4tg-2tgx-П+9 На отрезке от (0;пи разделить на 3)

1775. александр, 7 июня 2010, 12:03:58
Найти наибольшее значение функции Y=4tgx-4x+п-8 на отрезке [-п/4;п/4]

1776. m@ri6k@, 7 июня 2010, 12:04:55
ПОМОГИТЕ!!!!

для одного из предприятий - монополистов зависимость объёма спроса на продукцию q (едениц в месяц) от её цены р (тыс.руб) задается формулой q=160- 10руб . определите максимальный уровеньцены в тыс руб, при котором значение выручки предприятия за месяц r=q*p составит не менее 550 тыс руб

1777. Никита, 7 июня 2010, 12:05:28
Lexxus,

помоги плизз
для одного из предприятий - монополистов зависимость объёма спроса на продукцию q (едениц в месяц) от её цены р (тыс.руб) задается формулой q=160- 10руб . определите максимальный уровеньцены в тыс руб, при котором значение выручки предприятия за месяц r=q*p составит не менее 550 тыс руб

1778. Lexxus, 7 июня 2010, 12:05:58
для одного из предприятий - монополистов зависимость объёма спроса на продукцию q (едениц в месяц) от её цены р (тыс.руб) задается формулой q=160- 10руб . определите максимальный уровеньцены в тыс руб, при котором значение выручки предприятия за месяц r=q*p составит не менее 550 тыс руб

11

1779. Никита, 7 июня 2010, 12:07:40
спасибо Lexxus

1780. Lexxus, 7 июня 2010, 12:07:54
ну функция у=4tg-2tgx-П+9 На отрезке от (0;пи разделить на 3)

Четыре тангенса (???) минус два тангенса икс???
Успокойся, соберись и попробуй написать еще раз.

1781. Андрей, 7 июня 2010, 12:08:27
Lexxus, пасб большое

1782. Lexxus, 7 июня 2010, 12:10:30
Найти наибольшее значение функции Y=4tgx-4x+п-8 на отрезке [-п/4;п/4]

-4

1783. полинааа, 7 июня 2010, 12:10:49
лексус я на тебя буду молиться

Байдарка в 10.00 вышла из пункта А в пункт В, расположенный в 15 км от А. Пробыв в пункте В 1 час 20 мин. Байдарка отправилась назад и вернулась в пункт А в 18.00 того же дня. Определите (в км/ч) собственную скорость байдарки, если известно что скорость течения реки равна 3 км/ч

а?

1784. юляя, 7 июня 2010, 12:11:02
. настя, 7 июня 2010, 12:02:20
найти наименьшее значение У=16 танг.х-16х-4пи+3.на отрезке -пи на 4;пи на 4
плииииз, срочно надо

1785. Екатерина , 7 июня 2010, 12:11:16
Всем спасибо)) Экзамен Off.. Будем ждать результатов..=)))))


Лексус, тебе отдельное СПАСИБО=*

1786. Алексей, 7 июня 2010, 12:11:46
Пожалуйста....
Катер в 10:00 вышел из пункта А в пункт В,расположенный в 15 км от А. Пробыв в пункте В 4 часа катер отправился назад и вернулся в пункт А в 18:00 того же дня. Определите в км /ч собственную скорость катера если известно что скорость течения реки равна 2 км /ч.

1787. Никита, 7 июня 2010, 12:12:21
9 в степени 2+лог по основанию 9 в степени 2



помогите

1788. Диана, 7 июня 2010, 12:12:53
С пункта А в пункт Б, расcтояние между которыми 60 км, одновременно выехали мотоциклист и велосепедист, известно что за час мотоциклист проезжает на 50 км больше чем велосепедист. Определите скорость велосепедиста если известно что он прибыл в пункт Б на 5 часов позже мотоциклиста . Ответ дайте в киллометрах помогите

1789. сергей, 7 июня 2010, 12:13:01
решите уравнение пожалуйста

Y=12tgx-12x-3+6 на отрезке [-П/4;П/4]

1790. Lexxus, 7 июня 2010, 12:13:14

Байдарка в 10.00 вышла из пункта А в пункт В, расположенный в 15 км от А. Пробыв в пункте В 1 час 20 мин. Байдарка отправилась назад и вернулась в пункт А в 18.00 того же дня. Определите (в км/ч) собственную скорость байдарки, если известно что скорость течения реки равна 3 км/ч

6 км/ч

1791. юляя, 7 июня 2010, 12:13:18
настя, 7 июня 2010, 12:02:20
найти наименьшее значение У=16 танг.х-16х-4пи+3.на отрезке -пи на 4;пи на 4
плииииз, срочно надо

1792. KaterinaKa, 7 июня 2010, 12:13:23
катер в 11 00 вышле из пунката А в пункт В расположенный в 15 ка от А.пробыв в пункте В 1 час 20 минут. катер отправился назад и вернулся обратно в пункт А в 15 00 того же дня. определите (в км/ч)собственную скорость катера ,если известно,что скорость течения реки равна 3 км/ч

ОООООООООООООООООООООООООООЧень!помогите пожалуйста

1793. Еленочка, 7 июня 2010, 12:14:08
В11 у=4tgx-4x+п-9 (-п/4;п/4) найдите значение выражения на этом промежутке пожалууйстааа.....срочно...

1794. Екатерина , 7 июня 2010, 12:15:10
. KaterinaKa,



2 Км/ч

1795. Lexxus, 7 июня 2010, 12:15:14
С пункта А в пункт Б, расcтояние между которыми 60 км, одновременно выехали мотоциклист и велосепедист, известно что за час мотоциклист проезжает на 50 км больше чем велосепедист. Определите скорость велосепедиста если известно что он прибыл в пункт Б на 5 часов позже мотоциклиста . Ответ дайте в киллометрах помогите

10 км/ч

1796. Любовь, 7 июня 2010, 12:15:18
лексуууус, очень срочно, прошу..

прямоугольный параллелепипед описан около сферы радиуса 3 . найти его объем

1797. Дымка, 7 июня 2010, 12:15:33
Найти наименьшее значение функции у=14tg х - 28 х + 7Пи-6 На отрезке от 0до пи/3 включ

1798. ПОЛИНА, 7 июня 2010, 12:15:38
О ЛЕКСУС!!!!!!!!!!!!!!!!!!!!!! Я ВАС ЛЮБЛЮ

1799. Lexxus, 7 июня 2010, 12:16:07
прямоугольный параллелепипед описан около сферы радиуса 3 . найти его объем

216

1800. Машенька, 7 июня 2010, 12:16:33
Б9 Прямоугольный параллелепипед описан около сферы радиуса 6,5. Найдите его объем.
ЛЕКСУС Помогите плиз, вы похожую решили.

1801. Егор, 7 июня 2010, 12:16:52
Lexxus, пожалуйста помоги :
Найдите наименьшее значение функции
y=4tgx-4x-п+7 на отрезке [-п/4;п/4]
Спасибо!

1802. сергей, 7 июня 2010, 12:16:53
извините, я не правильно написал, мне надо найти значение выражения

Y=12tgx-12x-3П+6 на отрезке [-П/4;П/4]


помогите пожалуйста !!)))

1803. Александр, 7 июня 2010, 12:17:28
Из пункта А в пункт В,расстояниж между которыми 60 км, одновременно въехали автомобилист и велосипедист.известно что за час автомобилист проезжает на 110 км больше чем велосипедист определите скорость велосипедиста если известно что он прибыл в пункт в на 5.5 часа позже автомобилиста.ответ дайте в км,ч

1804. Машенька, 7 июня 2010, 12:17:41
Б9 Прямоугольный параллелепипед описан около сферы радиуса 5,5. Найдите его объем.
ЛЕКСУС Помогите плиз, вы похожую решили. 5,5!!! сори!!!

1805. Андрей, 7 июня 2010, 12:17:50
Основанием прямой призмы является прямоугольный треугольник с катетами9 и 9 .боковые ребра равны 4/на пи р. Найдите общем цилиндра,описанного около этой призмы

1806. Таня, 7 июня 2010, 12:17:51
для одного из предприятий - монополистов зависимость объёма спроса на продукцию q (едениц в месяц) от её цены р (тыс.руб) задается формулой q=160- 10руб . определите максимальный уровеньцены в тыс руб, при котором значение выручки предприятия за месяц r=q*p составит не менее 550 тыс руб


ПОМОГИТЕ ПЛИЗ!!!!!!!!!

1807. Lexxus, 7 июня 2010, 12:18:16
Найти наименьшее значение функции у=14tg х - 28 х + 7Пи-6 На отрезке от 0до пи/3 включ

8

1808. KaterinaKa, 7 июня 2010, 12:18:32
Екатерина ,


СПАСИБООООООООООООООООО=********

1809. Никита, 7 июня 2010, 12:18:45
прямоугольный параллелепипед описан около сферы радиуса 5,5 . найти его объем



Лексус помоги будь добр)

1810. санька, 7 июня 2010, 12:19:05
Байдарка в 10:00 вышла из пункта А в пункт В, расположенный в 15 км от А. Пробыв в пункте В 1час 20 мин, байдарка отправилась назад и вернулась в пункт А в 16:00 того же дня. Определить собственную скорость байдарки, если скорость реки 2 км/ч




1811. сашка666, 7 июня 2010, 12:20:16
Катер в 10.00 вышел из пункта А в пункт Б расположенный в 15 км от пункта А пробыв в пункте Б 4 часа катер отправился назад и вернулся в пункт А в 18.00 того же дня определить собственную скорость катера в км.ч если известно что скорость течения реки равна 2км.ч

1812. Lexxus, 7 июня 2010, 12:20:37
для одного из предприятий - монополистов зависимость объёма спроса на продукцию q (едениц в месяц) от её цены р (тыс.руб) задается формулой q=160- 10руб . определите максимальный уровеньцены в тыс руб, при котором значение выручки предприятия за месяц r=q*p составит не менее 550 тыс руб

11

Прямоугольный параллелепипед описан около сферы радиуса 5,5. Найдите его объем.

1331
(два * R) в кубе.

Ну, ребят, у вас что, совсем мозни отключились?

1813. Екатерина, 7 июня 2010, 12:20:40
Из пункта А в пункт В,расстояниж между которыми 60 км, одновременно въехали автомобилист и велосипедист.известно что за час автомобилист проезжает на 110 км больше чем велосипедист определите скорость велосипедиста если известно что он прибыл в пункт в на 5.5 часа позже автомобилиста.ответ дайте в км,ч
помогите пожалста!

1814. Катеринка, 7 июня 2010, 12:20:43
Прямоугольный параллелепипед описан около сферы радиуса 1,5. Найдите его обьём!!!

1815. m@ri6k@, 7 июня 2010, 12:21:33
В треугольнике авс АВ=15 СМ,ВС=8,СА=9.точка Д лежит на прямой ВС ТАК,ЧТО ВД:ДС=З:8.ОКРУЖНОСТИ, впиранные в каждый из треугольников АДС и АДВ,касаются стороны АД в точках Е и Ф.найдите длину отрезка ЕФ

1816. Настя., 7 июня 2010, 12:22:37
Никита,1331.

1817. Lexxus, 7 июня 2010, 12:22:44
Из пункта А в пункт В,расстояниж между которыми 60 км, одновременно въехали автомобилист и велосипедист.известно что за час автомобилист проезжает на 110 км больше чем велосипедист определите скорость велосипедиста если известно что он прибыл в пункт в на 5.5 часа позже автомобилиста.ответ дайте в км,ч

10 км/ч

1818. alex, 7 июня 2010, 12:23:24
Прямоугольный параллелепипед описан около сферы радиуса 2,5. Найдите его обьём! помогите пожалуйста!

1819. Таня, 7 июня 2010, 12:23:43
СПАСИБО ОГРОМНОЕ!!!!!!!!!!!!!!!!!!!!!!!

1820. Настя., 7 июня 2010, 12:24:07
Катеринка: 27

1821. сергей, 7 июня 2010, 12:24:28
Байдарка в10часов вышла из А в В,расположенную в 15км от А прибыв в пункте В1час20мин,байдарка отправилась назад и вернулась в пункт А в 18:00 того/же дня!определить в км/ч собственную скорость байдарки, если известно что скорость реки 3кмч?

помогите плиз )))

1822. ЛЕНА, 7 июня 2010, 12:24:47
решите уравнение плиз y= 8tg x- 8 x -2 пи+ 6 на отрезке -пи/4 до пи/4, наименьшее значение найти

1823. Никита, 7 июня 2010, 12:25:22
Катер в 10.00 вышел из пункта а в пункт В, расположенный в 15 км от А. пробыв в пункте В 4 часа , катер отправился назад и вернулся в пункт А в 18.00 того же дня. Определите собственную скорость катера , если известно , что скорость течения реки 2 км./ч


помогите с этой ПОЖАЛУЙСТА

1824. Lexxus, 7 июня 2010, 12:25:44
Катер в 10.00 вышел из пункта А в пункт Б расположенный в 15 км от пункта А пробыв в пункте Б 4 часа катер отправился назад и вернулся в пункт А в 18.00 того же дня определить собственную скорость катера в км.ч если известно что скорость течения реки равна 2км.ч

8 км/ч

1825. сергей, 7 июня 2010, 12:25:49
извините, я не правильно написал, мне надо найти наименьшее значение выражения

Y=12tgx-12x-3П+6 на отрезке [-П/4;П/4]


помогите пожалуйста !!)))

1826. Пупся, 7 июня 2010, 12:27:22
из пункта А в пункт В,расстояние между которыми 50км,одновременно выехали автомобилист и велосипедист.Известно,что за час автомобилист проезжает на 40км больше,чем велосипедист.Определите скорость велосипедиста,если известно.что он прибыл в пункт В на 4часа позже автомобилиста.

1827. ilo, 7 июня 2010, 12:28:06
Катеринка 27

1828. малышка, 7 июня 2010, 12:28:10
Решите пожалуйста.
Из пункта А в пункт В, расстояние между которыми 60 км., одновременно выехали мотоциклист и велосипедист. Известно что за час мотоциклист проезжает на 50 км больше чем велосипедист. Определить скорость велосипедиста если известно что по прибыл в пункт В на 5 часов позже мотоциклиста.

1829. Настя., 7 июня 2010, 12:28:21
Алекс.125

помогите у=5х-5тангенсх плюс 4 на отрезке от пи делить на 4 до 0

1830. Никита, 7 июня 2010, 12:28:22
Прямомугольный параллелепипед описан около сферы радиуса 2. Найдите его объем

1831. Никита, 7 июня 2010, 12:29:13
Прямоугольный параллелепипед описан около сферы радиуса 2,5. Найдите его обьём! помогите плиз!

1832. Андрей, 7 июня 2010, 12:29:28
Никита, я решал эту задачу, вышло 8 км/ч. с проверкой. похоже варианты одинаковы, у тебя есть ответ на:

Основанием прямой призмы является прямоугольный треугольник с катетами9 и 9 .боковые ребра равны 4/на пи р. Найдите общем цилиндра,описанного около этой призмы?

ПОМОГИТЕ!!!!

1833. Андрей, 7 июня 2010, 12:29:47
"Прямомугольный параллелепипед описан около сферы радиуса 2. Найдите его объем"


64

1834. m@ri6k@, 7 июня 2010, 12:29:50
Прямоугольный параллелепипед описан около сферы радиуса 3.5 . Найдите его ОБЪЕМ

1835. орлдр, 7 июня 2010, 12:30:40
1828. малышка, 7 июня 2010, 12:28:10
Решите пожалуйста.
Из пункта А в пункт В, расстояние между которыми 60 км., одновременно выехали мотоциклист и велосипедист. Известно что за час мотоциклист проезжает на 50 км больше чем велосипедист. Определить скорость велосипедиста если известно что по прибыл в пункт В на 5 часов позже мотоциклиста.
ответ 10км/ч

1836. Андрей, 7 июня 2010, 12:30:57
Прямоугольный параллелепипед описан около сферы радиуса 2,5. Найдите его обьём! помогите плиз!
--------------------------------

125

1837. Никита, 7 июня 2010, 12:31:34
Андрей спасибо__

1838. юляшка, 7 июня 2010, 12:31:37
помогите пожалуйста

В треугольнике авс АВ=15 СМ,ВС=8,СА=9.точка Д лежит на прямой ВС ТАК,ЧТО ВД:ДС=З:8.ОКРУЖНОСТИ, впиранные в каждый из треугольников АДС и АДВ,касаются стороны АД в точках Е и Ф.найдите длину отрезка ЕФ

1839. Андрей, 7 июня 2010, 12:31:49
Прямоугольный параллелепипед описан около сферы радиуса 3.5 . Найдите его ОБЪЕМ
-----------------------

343

1840. Андрей, 7 июня 2010, 12:31:50
Lexxus, помоги плз,

Основанием прямой призмы является прямоугольный треугольник с катетами9 и 9 .боковые ребра равны 4/на пи р. Найдите общем цилиндра,описанного около этой призмы

1841. Дымка, 7 июня 2010, 12:32:47
Спасибо большое Lexxus, выручили.

1842. сергей, 7 июня 2010, 12:32:55
полинааа

у нас варианты одинаковые ! ))))

1843. санька, 7 июня 2010, 12:33:21
Байдарка в 10:00 вышла из пункта А в пункт В, расположенный в 15 км от А. Пробыв в пункте В 1час 20 мин, байдарка отправилась назад и вернулась в пункт А в 16:00 того же дня. Определить собственную скорость байдарки, если скорость реки 2 км/ч
плз помогите

1844. Никита, 7 июня 2010, 12:34:03
В треугольнике авс АВ=15 СМ,ВС=8,СА=9.точка Д лежит на прямой ВС ТАК,ЧТО ВД:ДС=З:8.ОКРУЖНОСТИ, впиранные в каждый из треугольников АДС и АДВ,касаются стороны АД в точках Е и Ф.найдите длину отрезка ЕФ


ПОМОГИТЕ С ЭТОЙ,ВОПРОС ЖИЗНИ И СМЕРТИ__)))

1845. Lexxus, 7 июня 2010, 12:34:20
Основанием прямой призмы является прямоугольный треугольник с катетами9 и 9 .боковые ребра равны 4/на пи р. Найдите общем цилиндра,описанного около этой призмы?

162

1846. Никита, 7 июня 2010, 12:35:04
Дан треугольник авс,ав-14,вс-10,са-12.точка д лежит на прямой вс так,что вд:дс=3:7.окружности вписанные в каждый из треугольников адс и адв касаются стороны ад в точках е и ф.найти длинну еф
ПОМОГИТЕ С ЭТОЙ,ВОПРОС ЖИЗНИ И СМЕРТИ__)))

1847. ЛЕНА, 7 июня 2010, 12:35:50
найдите наименьшее значение y= 8tg x- 8 x -2 пи+ 6 на отрезке -пи/4 до пи/4, ПОЖАЛУЙСТА, оч. надо

1848. Андрей, 7 июня 2010, 12:36:06
Lexxus,

спс большое

1849. Любовь, 7 июня 2010, 12:37:04
найдите наименьшее значение функции у=4х-4tgх+17 [-пи/4;0]

ОЧЕНЬ СРОЧНОООО!
ЛЕКСУС, ДОРОГОЙ! ПОЖАЛУЙСТА, ПОМОГИ..

1850. Lexxus, 7 июня 2010, 12:37:35
Никита, повторяю: ЧАСТЬ С РЕШАТЬ НЕ БУДУ!

Байдарка в 10:00 вышла из пункта А в пункт В, расположенный в 15 км от А. Пробыв в пункте В 1час 20 мин, байдарка отправилась назад и вернулась в пункт А в 16:00 того же дня. Определить собственную скорость байдарки, если скорость реки 2 км/ч

7 км/ч. Уже было.

1851. Александр, 7 июня 2010, 12:37:41
1828. малышка, 7 июня 2010, 12:28:10
Решите пожалуйста.
Из пункта А в пункт В, расстояние между которыми 60 км., одновременно выехали мотоциклист и велосипедист. Известно что за час мотоциклист проезжает на 50 км больше чем велосипедист. Определить скорость велосипедиста если известно что по прибыл в пункт В на 5 часов позже мотоциклиста.
помогите пожалуйста!! было у кого эта задача в тестах?

1852. сергей, 7 июня 2010, 12:37:52
Треуг АBС угол С равен 90градусов ,АВ = 30 cм, АС=24см найти sinА


помогите мне пожалуйста, !!!)))

1853. юльчик, 7 июня 2010, 12:38:02
прямоугольный параллелепипед описан около сферы радиуса 3.5 . Найдите его ОБЪЕМ
пожалуйста помогите!!!!

1854. ilo, 7 июня 2010, 12:38:35
из пункта А в пункт В,расстояние между которыми 50км,одновременно выехали автомобилист и велосипедист.Известно,что за час автомобилист проезжает на 40км больше,чем велосипедист.Определите скорость велосипедиста,если известно.что он прибыл в пункт В на 4часа позже автомобилиста.
Ответ: 10

1855. drugoy10, 7 июня 2010, 12:39:03
ЛЕНА,
по-моему, -2!!!


1856. Lexxus, 7 июня 2010, 12:39:11
найдите наименьшее значение функции у=4х-4tgх+17 , -пи/4;0

17

1857. KaterinaKa, 7 июня 2010, 12:39:23
http://vkontakte.ru/photo8413743_167486394

1858. Александр, 7 июня 2010, 12:39:57
Для одного из предприятии-монополистов зависимость объема спроса на продукцию q (единиц в месяц)от ее цены p (тыс.руб.)задаетсю формулой: q=160-10pопределите максимальный уровень цены p (в тыс.руб.) при котором значение выручки предприятия за месяц r=qp составит не менее 550 тыс.руб.
была у кого эта задача? помогите решить

1859. юльчик, 7 июня 2010, 12:40:16
помогите пожалуйста

В треугольнике авс АВ=15 СМ,ВС=8,СА=9.точка Д лежит на прямой ВС ТАК,ЧТО ВД:ДС=З:8.ОКРУЖНОСТИ, впиранные в каждый из треугольников АДС и АДВ,касаются стороны АД в точках Е и Ф.найдите длину отрезка ЕФ

1860. Lexxus, 7 июня 2010, 12:40:47
Из пункта А в пункт В, расстояние между которыми 60 км., одновременно выехали мотоциклист и велосипедист. Известно что за час мотоциклист проезжает на 50 км больше чем велосипедист. Определить скорость велосипедиста если известно что по прибыл в пункт В на 5 часов позже мотоциклиста.
помогите пожалуйста!! было у кого эта задача в тестах?

Была, была, смотри выше.
10 км/ч

1861. ilo, 7 июня 2010, 12:41:11
юльчик 343

1862. drugoy10, 7 июня 2010, 12:41:19
сергей,
0,6 !!!

1863. Мария, 7 июня 2010, 12:41:36
Помогите,пожалуйста!!!

у=4х-2tgx-П+9 На отрезке от (0;пи разделить на 3)


1864. Lexxus, 7 июня 2010, 12:43:20
http://vkontakte.ru/photo8413743_167486394

Удали фотку. Там есть штрих-код, по нему можно определить, чей это КИМ.
Вполне могут аннулировать результат.

1865. сергей, 7 июня 2010, 12:43:36
Лекс, реши пожалуста эту

Треуг АBС угол С равен 90градусов ,АВ = 30 cм, АС=24см найти sinА


помоги мне пожалуйста, !!!)))

1866. Машенька, 7 июня 2010, 12:43:39
1857. KaterinaKa, 7 июня 2010, 12:39:23

Катеринка у тебя в Б8 какой ответ?

1867. drugoy10, 7 июня 2010, 12:44:44
Мария,
7 !!!

1868. Санек, 7 июня 2010, 12:45:00
Для одного из предприятии-монополистов зависимость объема спроса на продукцию q (единиц в месяц)от ее цены p (тыс.руб.)задаетсю формулой: q=160-10pопределите максимальный уровень цены p (в тыс.руб.) при котором значение выручки предприятия за месяц r=qp составит не менее 550 тыс.руб.
была у кого эта задача? помогите решить

1869. ilo, 7 июня 2010, 12:46:44
Треуг АBС угол С равен 90градусов ,АВ = 30 cм, АС=24см найти sinА

0,6

1870. drugoy10, 7 июня 2010, 12:46:54
Санек, 7 июня 2010, 12:45:00
Для одного из предприятии-монополистов зависимость объема спроса на продукцию q (единиц в месяц)от ее цены p (тыс.руб.)задаетсю формулой: q=160-10pопределите максимальный уровень цены p (в тыс.руб.) при котором значение выручки предприятия за месяц r=qp составит не менее 550 тыс.руб.
была у кого эта задача? помогите решить

11 !!!!!!!

1871. Пётр, 7 июня 2010, 12:47:11
найдите наибольшее значение функции y=10tgx-10x+4 на отрезке [-пи/4;0]
плизз помогите))

1872. Егор, 7 июня 2010, 12:47:18
Lexxus, не дождался помощи, пожалуйста!!
Найдите наименьшее значение функции
y=4tgx-4x-п+7 на отрезке [-п/4;п/4]
заранее спасибо!!!

1873. Мария, 7 июня 2010, 12:47:19
drugoy10 - спасибо большое))

1874. Машенька, 7 июня 2010, 12:48:35
1870. drugoy10, 7 июня 2010, 12:46:54
Санек, 7 июня 2010, 12:45:00
Для одного из предприятии-монополистов зависимость объема спроса на продукцию q (единиц в месяц)от ее цены p (тыс.руб.)задаетсю формулой: q=160-10pопределите максимальный уровень цены p (в тыс.руб.) при котором значение выручки предприятия за месяц r=qp составит не менее 550 тыс.руб.
была у кого эта задача? помогите решить

11 !!!!!!!

А училка говорит неправильно((((((

1875. надежда2010, 7 июня 2010, 12:49:17
мария : У=4Х-2tgХ-п+9 ответ -7

1876. Lexxus, 7 июня 2010, 12:49:28
Найдите наименьшее значение функции
y=4tgx-4x-п+7 на отрезке, -п/4;п/4

Вообще-то, дождался, я уже решал это.
Ответ: 3

1877. KaterinaKa, 7 июня 2010, 12:49:40
я не сдаю егэ)
подруге помогаю

1878. drugoy10, 7 июня 2010, 12:49:44
1872. Егор, 7 июня 2010, 12:47:18
Lexxus, не дождался помощи, пожалуйста!!
Найдите наименьшее значение функции
y=4tgx-4x-п+7 на отрезке [-п/4;п/4]
заранее спасибо!!!

3 !!!

1879. Санек, 7 июня 2010, 12:50:02
Машенька так 11 это правильный или не правильный ответ?)))

1880. Пётр, 7 июня 2010, 12:50:19
Машенька будет 110

1881. сергей, 7 июня 2010, 12:51:32
не дождался ответа !))))
найти наименьшее значение
Y=12tgx-12x-3П+6 на отрезке [-П/4;П/4]


помогите пожалуйста !!)))
убедительно прошу !))

1882. Lexxus, 7 июня 2010, 12:52:06
найдите наибольшее значение функции y=10tgx-10x+4 на отрезке -пи/4;0

4

1883. KaterinaKa, 7 июня 2010, 12:52:12
Y=12tgx-12x-3П+6 на отрезке [-П/4;П/4]

пожалуййстаааааааааааааааааааааааааааааа=***

1884. Санек, 7 июня 2010, 12:52:20
Для одного из предприятии-монополистов зависимость объема спроса на продукцию q (единиц в месяц)от ее цены p (тыс.руб.)задаетсю формулой: q=160-10 определите максимальный уровень цены p (в тыс.руб.) при котором значение выручки предприятия за месяц r=qp составит не менее 550 тыс.руб.
так какой тут ответ то будет????

1885. ilo, 7 июня 2010, 12:52:33
Для одного из предприятии-монополистов зависимость объема спроса на продукцию q (единиц в месяц)от ее цены p (тыс.руб.)задаетсю формулой: q=160-10pопределите максимальный уровень цены p (в тыс.руб.) при котором значение выручки предприятия за месяц r=qp составит не менее 550 тыс.руб.
была у кого эта задача? помогите решить
Ответ :11

1886. Егор, 7 июня 2010, 12:53:00
Лексус, Другой10
СПАСИБО!

1887. drugoy10, 7 июня 2010, 12:53:11
Машенька,
запутать пытается)
я уверен!)

1888. ЛЕНА, 7 июня 2010, 12:53:23
Спасибо я уже тоже решила, да -2!!!!!!!!!!!!!!!!!

1889. малышка, 7 июня 2010, 12:53:31
2+лог5 в 3 степени с основанием 5 пожалуйста люди решите

1890. Машенька, 7 июня 2010, 12:54:19
1877. KaterinaKa, 7 июня 2010, 12:49:40
я не сдаю егэ)
подруге помогаю

ну скажи что у подруги

1891. Lexxus, 7 июня 2010, 12:54:27
Для одного из предприятии-монополистов зависимость объема спроса на продукцию q (единиц в месяц)от ее цены p (тыс.руб.)задаетсю формулой: q=160-10pопределите максимальный уровень цены p (в тыс.руб.) при котором значение выручки предприятия за месяц r=qp составит не менее 550 тыс.руб.

Правильный ответ: 11!!!

найти наименьшее значение
Y=12tgx-12x-3П+6 на отрезке -П/4;П/4

-6

1892. надежда2010, 7 июня 2010, 12:54:43
егор, 4tgх-4х-п+7 наименьшее значение 3

1893. drugoy10, 7 июня 2010, 12:54:59
1883. KaterinaKa, 7 июня 2010, 12:52:12
Y=12tgx-12x-3П+6 на отрезке [-П/4;П/4]

пожалуййстаааааааааааааааааааааааааааааа=***

-6 !!!!

1894. Некитос!!, 7 июня 2010, 12:54:59
3 в степени 2+log8 по основанию 3 y=8tgx-8x-2П+6 найти наим значение на отрезке -п/4 и п/4

1895. Пётр, 7 июня 2010, 12:56:14
Из пункта А в пункт В,расстояние между которыми 60км, одновременно выехали автомобилист и велосипедист. Известно,что за час автомобилст проезжает на 110км больше,чем велосипедист . Определите скорость велосипедиста , если известно ,что он прибыл в пункт Вна 5,5 часа позже автомобилиста. Ответ дайте в кмч пожалуйста помогитееее)
помогите плиззз)

1896. Lexxus, 7 июня 2010, 12:56:36
2+лог5 в 3 степени с основанием 5 пожалуйста люди решите

Если ты имеешь в виду
2+(log_5(5))^3, то это равно 3.

1897. drugoy10, 7 июня 2010, 12:57:27
1889. малышка, 7 июня 2010, 12:53:31
2+лог5 в 3 степени с основанием 5 пожалуйста люди решите

5 !!!!

1898. Эля, 7 июня 2010, 12:58:35
Найдите наибольшее значение функции у=4х-2tgx-П+9 На отрезке 0;П/3
помогите

1899. Санек, 7 июня 2010, 12:58:53
Из пункта А в пункт В,расстояние между которыми 60км, одновременно выехали автомобилист и велосипедист. Известно,что за час автомобилст проезжает на 110км больше,чем велосипедист . Определите скорость велосипедиста , если известно ,что он прибыл в пункт Вна 5,5 часа позже автомобилиста. Ответ дайте в кмч пожалуйста помогитееее)
помогите плиззз)
ответ 10

1900. Lexxus, 7 июня 2010, 12:59:14
3 в степени 2+log8 по основанию 3

3^(2+log_3(8)) = 72

1901. сергей, 7 июня 2010, 12:59:24
решите эту пожалуста очень прошу !)))

Основанием прямой призмы является прямоугольный треугольник с катетами 9 и 7 .боковые ребра равны 6/П. Найти обьём цилиндра,описанного около этой призмы?

1902. drugoy10, 7 июня 2010, 13:00:08
1898. Эля, 7 июня 2010, 12:58:35
Найдите наибольшее значение функции у=4х-2tgx-П+9 На отрезке 0;П/3
помогите
7 !!!

1903. Цаца, 7 июня 2010, 13:00:43
В7-найти значение выражения-3 в степени 2+lоg4 по основанию 3

В11-найти наибольшее значение функции у=7х-7tg х-3 на отрезке [о;Пи/4].

Люди Помогите пожалуста

1904. Эля, 7 июня 2010, 13:01:06
Найдите наибольшее значение функции у=4х-2tgx-П+9 На отрезке 0;П/3
помогите очень срочно!!!
Лексус, прошу.

1905. тимофе, 7 июня 2010, 13:01:55
Байдарка в 10:00 вышла из пункта А в пункт В, расположенный в 15 км от А. Пробыв в пункте В 1час 15 мин, байдарка отправилась назад и вернулась в пункт А в 14:00 того же дня. Определить собственную скорость байдарки, если скорость реки 1 км/ч

1906. Эля, 7 июня 2010, 13:02:18
спасибо большоее!!!!

1907. Lexxus, 7 июня 2010, 13:02:20
В7-найти значение выражения-3 в степени 2+lоg4 по основанию 3

36

1908. Настя., 7 июня 2010, 13:02:27
РЕбят,всем спасибо))

1909. Алексей, 7 июня 2010, 13:02:53
Люди добрые помогите!!!

y+sinx=0
(2корень из sinx)-1)*(2y+5)

это система

1910. Артём, 7 июня 2010, 13:04:17
Прямоугольный параллелепипед описан около сферы радиуса 3. Найдите его объем
Lexxus плиззз

1911. сергей, 7 июня 2010, 13:04:21
ешите эту пожалуста очень прошу !)))

Основанием прямой призмы является прямоугольный треугольник с катетами 9 и 7 .боковые ребра равны 6/П. Найти обьём цилиндра,описанного около этой призмы?

1912. Андрей, 7 июня 2010, 13:05:12
1910. Артём, 216

1913. Маня), 7 июня 2010, 13:05:16
Найдите наибольшее значение функции
y=4tgx-4x-п+в на отрезке, -п/4;п/4
помогите решить,пожалуйстаааа

1914. Lexxus, 7 июня 2010, 13:05:53
В11-найти наибольшее значение функции у=7х-7tg х-3 на отрезке о;Пи/4.

-3

Найдите наибольшее значение функции у=4х-2tgx-П+9 На отрезке 0;П/3

7

Байдарка в 10:00 вышла из пункта А в пункт В, расположенный в 15 км от А. Пробыв в пункте В 1час 15 мин, байдарка отправилась назад и вернулась в пункт А в 14:00 того же дня. Определить собственную скорость байдарки, если скорость реки 1 км/ч

11 км/ч

1915. малышка, 7 июня 2010, 13:06:50
ой не правильно записала))) 2 в степени 3+лог5 с основанием 5

1916. Цаца, 7 июня 2010, 13:06:51
1907. Lexxus, 7 июня 2010, 13:02:20

СПАСИБО!!!!

а Б11 пожалуйста
найти наибольшее значение функции у=7х-7tg х-3 на отрезке [о;Пи/4].

1917. Андрей, 7 июня 2010, 13:06:56
http://filetransfer.jimm.org/sendfile.php?file=2a06966739b844761cd896b6

помогитеее, пллиииииииз, СРОЧНО

1918. Lexxus, 7 июня 2010, 13:07:01
Прямоугольный параллелепипед описан около сферы радиуса 3. Найдите его объем

Слабо самому 6 в куб возвести?
216

1919. Пётр, 7 июня 2010, 13:07:54
y+cosx=0
(5 корень из cos x-1)(5y-3)=0
система,решите плиззззз

1920. александр, 7 июня 2010, 13:08:08
Основанием прямоугольной призмы является прямоугольный треугольник с катерами 7 и 3. Боковые ребра равны 2/П. Найти общем цилиндра, описанного около призмы


помогите пожалуйста

1921. Цаца, 7 июня 2010, 13:08:09
Lexxus ты супер!!!

1922. drugoy10, 7 июня 2010, 13:08:23
1910. Артём, 7 июня 2010, 13:04:17
Прямоугольный параллелепипед описан около сферы радиуса 3. Найдите его объем
Lexxus плиззз
Сорри, Лексус прав)

1923. Артём, 7 июня 2010, 13:08:54
Спасибо большое

1924. Андрей, 7 июня 2010, 13:09:00
1917. Андрей, 7 июня 2010, 13:06:56
http://filetransfer.jimm.org/sendfile.php?file=2a06966739b844761cd896b6

помогитеее, пллиииииииз, СРОЧНО

подробный ответ нужен

1925. Маня), 7 июня 2010, 13:10:33
Найдите наибольшее значение функции
y=4tgx-4x-п+в на отрезке, -п/4;п/4
помогите решить,пожалуйстаааа
ПОМОГИИИТЕЕЕЕ

1926. Lexxus, 7 июня 2010, 13:10:44
Основанием прямоугольной призмы является прямоугольный треугольник с катерами 7 и 3. Боковые ребра равны 2/П. Найти общем цилиндра, описанного около призмы

29

1927. сергей, 7 июня 2010, 13:11:19
Основанием прямой призмы является прямоугольный треугольник с катетами 9 и 7 .боковые ребра равны 6/П. Найти обьём цилиндра,описанного около этой призмы?

1928. Буш, 7 июня 2010, 13:11:46
y=-0.25 x=-1^kП/4+Пк

1929. Lexxus, 7 июня 2010, 13:12:13

помогитеее, пллиииииииз, СРОЧНО

подробный ответ нужен

Часть С сегодня не решаю.

1930. юлия, 7 июня 2010, 13:12:20
байдарка вышла в 10.00 вышла из пункта А в пункт Б. Б находится от А в 15 км.
В Б она простояла 1ч2мин,потом вернулась в 16.00. Скорость реки 2 км/ч.
Какая скорость байдарки?
помогите,пожалуйста!

1931. катя, 7 июня 2010, 13:12:39
7в степени2+log7в4
помогитееееееее

1932. Анастасия, 7 июня 2010, 13:12:41
Найти все значения параметра а,при котором функция f(x) = x^2-2|x-a^2|-8*x, имеет более двух точек экстремума?
Помогите с решением.

1933. drugoy10, 7 июня 2010, 13:12:45
1925. Маня), 7 июня 2010, 13:10:33
Найдите наибольшее значение функции
y=4tgx-4x-п+в на отрезке, -п/4;п/4
помогите решить,пожалуйстаааа
ПОМОГИИИТЕЕЕЕ


y=4tgx-4x-п+в +8???

тогда 4 !!!

1934. Артем!!!!, 7 июня 2010, 13:14:26
лексус реши пожалуйста!!!!
Катер в 10.00 вышел из пункта А в пункт Б расположенный в 15 км от пункта А пробыв в пункте Б 4 часа катер отправился назад и вернулся в пункт А в 18.00 того же дня определить собственную скорость катера в км.ч если известно что скорость течения реки равна 2км.ч

оч надо!!

1935. Буш, 7 июня 2010, 13:15:06
y+sinx=0
(2корень из sinx)-1)*(2y+5)
ОТВЕТ:
y=-0.25 x=-1^kП/4+Пк

1936. тимофей, 7 июня 2010, 13:15:34
7в степени2+log7в4
ответ 53,ниче сложного ))))

1937. сергей, 7 июня 2010, 13:16:04
Основанием прямой призмы является прямоугольный треугольник с катетами 9 и 7 .боковые ребра равны 6/П. Найти обьём цилиндра,описанного около этой призмы?
решите это пожайлуста ! )))
прошу !!

1938. тимофей, 7 июня 2010, 13:16:52
7в степени2+log7в4
сорри ответ 196

1939. юлия, 7 июня 2010, 13:17:25
лексус,помоги,пожалуйста! очень очень надо!байдарка вышла в 10.00 вышла из пункта А в пункт Б. Б находится от А в 15 км.
В Б она простояла 1ч2мин,потом вернулась в 16.00. Скорость реки 2 км/ч.
Какая скорость байдарки?


1940. катя, 7 июня 2010, 13:17:36
тимофей
спасибо огромное)))))

из пункта А в пункт В,растояние между каторыми 50км,однавременно выехали автомобилист и велосипедист.Известно,что за час автомобилист проехал на 80км больше,чем велосипидист.Определите скорость велосипидиста если известно,что он прибыл в пункт В на 2часа позже автомобилиста.Ответ дайте в км/ч.
и вот эту помогите пожалуйста))

1941. Артем!!!!, 7 июня 2010, 13:18:23
Катер в 10.00 вышел из пункта А в пункт Б расположенный в 15 км от пункта А пробыв в пункте Б 4 часа катер отправился назад и вернулся в пункт А в 18.00 того же дня определить собственную скорость катера в км.ч если известно что скорость течения реки равна 2км.ч

лекс реши пожалуйста оч нада!!!!!!

1942. Маня), 7 июня 2010, 13:19:04
да,y=4tgx-4x-п +8....наибольшее
ответ -4 или 4????

1943. drugoy10, 7 июня 2010, 13:19:23
1934. Артем!!!!, 7 июня 2010, 13:14:26
лексус реши пожалуйста!!!!
Катер в 10.00 вышел из пункта А в пункт Б расположенный в 15 км от пункта А пробыв в пункте Б 4 часа катер отправился назад и вернулся в пункт А в 18.00 того же дня определить собственную скорость катера в км.ч если известно что скорость течения реки равна 2км.ч

оч надо!!


8 !!!

1944. Любовь, 7 июня 2010, 13:19:28
часть С кто - нибудь решит?

1945. Артем!!!!, 7 июня 2010, 13:20:53
ребят ну хоть кто нибудь помогите!!!!!!!!
Катер в 10.00 вышел из пункта А в пункт Б расположенный в 15 км от пункта А пробыв в пункте Б 4 часа катер отправился назад и вернулся в пункт А в 18.00 того же дня определить собственную скорость катера в км.ч если известно что скорость течения реки равна 2км.ч

1946. drugoy10, 7 июня 2010, 13:20:59
1942. Маня), 7 июня 2010, 13:19:04
да,y=4tgx-4x-п +8....наибольшее
ответ -4 или 4????

4!!!!!!!!!

1947. Анастасия, 7 июня 2010, 13:21:00
у кого нибудь были похожие задания про два экстремума функции?

1948. Андрей, 7 июня 2010, 13:21:24
Моторная лодка в 11 00 вышла из пункта а в пункт в,расположенный в15км от а. Пробыв в пункте в 1 час 15минут лодка.
Отправилась назад и вернулась в пункт а в 16 00 того же дня . Определите в км/ч собственную скорость лодки,если известно,что скорость течению реки равна 3км/ч

ПЛИИИИЗ, ПОМОГИТЕЕЕ

1949. Lexxus, 7 июня 2010, 13:21:29

из пункта А в пункт В,растояние между каторыми 50км,однавременно выехали автомобилист и велосипедист.Известно,что за час автомобилист проехал на 80км больше,чем велосипидист.Определите скорость велосипидиста если известно,что он прибыл в пункт В на 2часа позже автомобилиста.Ответ дайте в км/ч.

20 км/ч

1950. Марина =*, 7 июня 2010, 13:22:40
y+sinx=0
(2(корень sinx)-1)*(2y+5)
это система,помогите пожалуста! напишите решение очень прошу

1951. юлия, 7 июня 2010, 13:23:15
байдарка вышла в 10.00 вышла из пункта А в пункт Б. Б находится от А в 15 км.
В Б она простояла 1ч2мин,потом вернулась в 16.00. Скорость реки 2 км/ч.
Какая скорость байдарки?
помогите,пожалуйста!очень срочно надо! плиииииииииииииз!

1952. Маня), 7 июня 2010, 13:23:29
Байдарка в 10:00 вышла из пункта А в пункт В, расположенный в 15 км от А. Пробыв в пункте В 1час 15 мин, байдарка отправилась назад и вернулась в пункт А в 14:00 того же дня. Определить собственную скорость байдарки, если скорость реки 1 км/ч

ответ 9 км/ч(там нужно от 10 отнять 1)...

1953. катя, 7 июня 2010, 13:23:56
Lexxus
спасибо )**

1954. Денис, 7 июня 2010, 13:25:57
y=4ax+[x^2-6x+5]

при каких значениях а наименьшее значение фун-и меньше 1

1955. Маня), 7 июня 2010, 13:26:21
Спасибо,ребят,за помощь!!!!

1956. сергей, 7 июня 2010, 13:26:45
ребята, ну хоть кто нить, помогите пожалуйста !)))


Основанием прямой призмы является прямоугольный треугольник с катетами 9 и 7 .боковые ребра равны 6/П. Найти обьём цилиндра,описанного около этой призмы?

решите это пожайлуста ! )))
прошу !!

1957. drugoy10, 7 июня 2010, 13:26:51
1948. Андрей, 7 июня 2010, 13:21:24
Моторная лодка в 11 00 вышла из пункта а в пункт в,расположенный в15км от а. Пробыв в пункте в 1 час 15минут лодка.
Отправилась назад и вернулась в пункт а в 16 00 того же дня . Определите в км/ч собственную скорость лодки,если известно,что скорость течению реки равна 3км/ч

ПЛИИИИЗ, ПОМОГИТЕЕЕ


9!!!!

1958. Мария, 7 июня 2010, 13:28:02
y+cosx=0
(5 корень из cos x-1)(5y-3)=0

help!!))

1959. Буш, 7 июня 2010, 13:28:12
"1950. Марина =*, 7 июня 2010, 13:22:40
y+sinx=0
(2(корень sinx)-1)*(2y+5)
это система,помогите пожалуста! напишите решение очень прошу"
ОТВЕТ:
y=-0.25 x=-1^k arcsin1/4+Пк

1960. юлия, 7 июня 2010, 13:28:39
байдарка вышла в 10.00 вышла из пункта А в пункт Б. Б находится от А в 15 км.
В Б она простояла 1ч2мин,потом вернулась в 16.00. Скорость реки 2 км/ч.
Какая скорость байдарки?
помогите,пожалуйста!очень срочно надо! пожалуйста!!!!!!!!!!!!!

1961. drugoy10, 7 июня 2010, 13:31:43
1956. сергей, 7 июня 2010, 13:26:45
ребята, ну хоть кто нить, помогите пожалуйста !)))


Основанием прямой призмы является прямоугольный треугольник с катетами 9 и 7 .боковые ребра равны 6/П. Найти обьём цилиндра,описанного около этой призмы?

решите это пожайлуста ! )))
прошу !!

я бы написал 195, но я не уверен!

1962. алекс, 7 июня 2010, 13:32:49
найдите наибольшее значение функции у=2х-tg(x-5), на отрезке от о до пи/4, помогите пожалуйста кто знает

1963. Машенька, 7 июня 2010, 13:33:47
1960. юлия, 7 июня 2010, 13:28:39
байдарка вышла в 10.00 вышла из пункта А в пункт Б. Б находится от А в 15 км.
В Б она простояла 1ч2мин,потом вернулась в 16.00. Скорость реки 2 км/ч.
Какая скорость байдарки?
помогите,пожалуйста!очень срочно надо! пожалуйста!!!!!!!!!!!!!

7

1964. дмитрий, 7 июня 2010, 13:35:22
байдарка вышла в 10.00 вышла из пункта А в пункт Б. Б находится от А в 15 км.
В Б она простояла 1ч2мин,потом вернулась в 16.00. Скорость реки 2 км/ч.
Какая скорость байдарки? помогите,кто знает!

1965. drugoy10, 7 июня 2010, 13:36:35
1960. юлия, 7 июня 2010, 13:28:39
байдарка вышла в 10.00 вышла из пункта А в пункт Б. Б находится от А в 15 км.
В Б она простояла 1ч2мин,потом вернулась в 16.00. Скорость реки 2 км/ч.
Какая скорость байдарки?
помогите,пожалуйста!очень срочно надо! пожалуйста!!!!!!!!!!!!!

7 !!!!!!!!!!

1966. юлия, 7 июня 2010, 13:37:24
спасибо огромное))

1967. алекс, 7 июня 2010, 13:41:11
найдите наибольшее значение функции у=2х-tg(x-5), на отрезке от о до пи/4, помогите пожалуйста кто знает

1968. drugoy10, 7 июня 2010, 13:45:10
1967. алекс, 7 июня 2010, 13:41:11
найдите наибольшее значение функции у=2х-tg(x-5), на отрезке от о до пи/4, помогите пожалуйста кто знает
там точно скобки?
если без, то -5...

1969. Валерия, 7 июня 2010, 13:45:23
В .10.00 вышел из пункта А в пункт В катер , расположенный в 15 километрах от пункта А. Пробыв в пункте В 1час 15мин, катер отправился назад и вернулся в пункт А в 14.00 того же дня. Определите собственную скорость катера если известно что скорость течения 1км.ч.



Помогите решить!пожалуйста!осталось до конца экзамена 20 минут!!!!!!
выручайте!!!!!!!!!!!!!!!!!!!!!!!!!!!!!!!!!!!!!!!!!!!!!!!!!!!

1970. drugoy10, 7 июня 2010, 13:48:00
1969. Валерия, 7 июня 2010, 13:45:23
В .10.00 вышел из пункта А в пункт В катер , расположенный в 15 километрах от пункта А. Пробыв в пункте В 1час 15мин, катер отправился назад и вернулся в пункт А в 14.00 того же дня. Определите собственную скорость катера если известно что скорость течения 1км.ч.



Помогите решить!пожалуйста!осталось до конца экзамена 20 минут!!!!!!
выручайте!!!!!!!!!!!!!!!!!!!!!!!!!!!!!!!!!!!!!!!!!!!!!!!!!!!


11 !!!!!!!!!!

1971. пацанчик, 7 июня 2010, 13:48:10
Валерия,писали 2 ответа, 11 и 9,я думаю что 11 правильный. с тебя аська за то что выручил :))

1972. Иришка), 7 июня 2010, 13:48:52
помогите!!
3 в степени 3+log 11 по основанию 3

1973. Lexxus, 7 июня 2010, 13:49:23
В .10.00 вышел из пункта А в пункт В катер , расположенный в 15 километрах от пункта А. Пробыв в пункте В 1час 15мин, катер отправился назад и вернулся в пункт А в 14.00 того же дня. Определите собственную скорость катера если известно что скорость течения 1км.ч.

11 км/ч. Это уже было раза два.

найдите наибольшее значение функции у=2х-tg(x-5), на отрезке от о до пи/4, помогите пожалуйста кто знает

Неправильно записано условие.

1974. Lexxus, 7 июня 2010, 13:50:46
3 в степени 3+log 11 по основанию 3

3^(3+log_3(11)) = 297

1975. О/К, 7 июня 2010, 13:51:02
Мне и Алексу! пожалуйста, помогите!
найдите наибольшее значение функции у=2х-tg(x-5), на отрезке от о до пи/4, помогите пожалуйста кто знает

1976. drugoy10, 7 июня 2010, 13:51:14
1972. Иришка), 7 июня 2010, 13:48:52
помогите!!
3 в степени 3+log 11 по основанию 3

297 !!!

1977. О/К, 7 июня 2010, 13:51:29
Мне и Алексу! пожалуйста, помогите.
найдите наибольшее значение функции у=2х-tg(x-5), на отрезке от о до пи/4, помогите пожалуйста кто знает

1978. Сестра выпускницы, 7 июня 2010, 13:52:15
1967. алекс, 7 июня 2010, 13:41:11
найдите наибольшее значение функции у=2х-tg(x-5), на отрезке от о до пи/4, помогите пожалуйста кто знает
ответ 4, точно говорю, только что сестре решала

1979. Валерия, 7 июня 2010, 13:53:40
Пацанчик 573739564 ;D

1980. drugoy10, 7 июня 2010, 13:54:41
1978. Сестра выпускницы, 7 июня 2010, 13:52:15
1967. алекс, 7 июня 2010, 13:41:11
найдите наибольшее значение функции у=2х-tg(x-5), на отрезке от о до пи/4, помогите пожалуйста кто знает
ответ 4, точно говорю, только что сестре решала


а х чему равен? первый раз такое задание встречаю

1981. Цаца, 7 июня 2010, 13:54:42
У все кончилось. Всех заданий B на 4-ку не хватит наверное?

1982. Валерия, 7 июня 2010, 13:55:17
А решение задачи напишет кто-нить?????
там только ответ(

В .10.00 вышел из пункта А в пункт В катер , расположенный в 15 километрах от пункта А. Пробыв в пункте В 1час 15мин, катер отправился назад и вернулся в пункт А в 14.00 того же дня. Определите собственную скорость катера если известно что скорость течения 1км.ч.


1983. drugoy10, 7 июня 2010, 13:58:21
Валерия,

(15/х+1) + 15/х-1 = 2,3/4
путем несложных вычислений получаем 11)))

1984. Валерия, 7 июня 2010, 13:59:44
Спасибоуууу,но вроде уже поздняк метаться :D

1985. Дмитрий, 7 июня 2010, 14:00:09
Найдите все а при котором функция f(x)=x^x - |x-a^a| - 11x имеет одну точку максимума

1986. НЕЛИЧКА, 7 июня 2010, 14:01:27
!!!!!!!!!!!!!!!!!!!!!!!!!!!!!!!!!!!!!!!!!!!!!!!!!!!!!!!!!!!Из пункта А в пункт В , расстояние между которыми 50км , одновременно выехали автомобилист и велосипедист.известно что за час автомобилист проезжает на 80км больше,чем велосипедист.определите скорость велосипедиста,если известно что он прибыл в пункт В на 2часа позже автомобилиста.
ПОМОГИТЕ!!!!!!!!!!!!!!!!!!!!!! !!!!!!!!!!!!!!!!!!!!!!!!!

1987. drugoy10, 7 июня 2010, 14:03:39
1986. НЕЛИЧКА, 7 июня 2010, 14:01:27
!!!!!!!!!!!!!!!!!!!!!!!!!!!!!!!!!!!!!!!!!!!!!!!!!!!!!!!!!!!Из пункта А в пункт В , расстояние между которыми 50км , одновременно выехали автомобилист и велосипедист.известно что за час автомобилист проезжает на 80км больше,чем велосипедист.определите скорость велосипедиста,если известно что он прибыл в пункт В на 2часа позже автомобилиста.
ПОМОГИТЕ!!!!!!!!!!!!!!!!!!!!!! !!!!!!!!!!!!!!!!!!!!!!!!!

20!!!

1988. Димитрий, 7 июня 2010, 14:06:54
Найдите все а при котором функция f(x)=x^x - |x-a^a| - 11x имеет одну точку максимума
ЛЮДИ ДОБРЫЕ ПОМОГИТЕ!!!!!!!!!!!

1989. Dim, 7 июня 2010, 14:08:58
3 в степени 3+log 2 по основанию 3

1990. НЕЛИЧКА, 7 июня 2010, 14:09:22
СПАСИБО ОГРОМНОЕ))

1991. drugoy10, 7 июня 2010, 14:10:12
1989. Dim, 7 июня 2010, 14:08:58
3 в степени 3+log 2 по основанию 3

54!!!!

1992. Dim, 7 июня 2010, 14:10:27
3 в степени 3+log 2 по основанию 3

54???????????

1993. drugoy10, 7 июня 2010, 14:13:00
1992. Dim, 7 июня 2010, 14:10:27
3 в степени 3+log 2 по основанию 3

54???????????


да!!! 3 в третей умножить на 2!!!

1994. Dim, 7 июня 2010, 14:13:14
drugoy10
пасиб,просто проверял

1995. Катенька, 7 июня 2010, 14:13:35
!!!!!!!!!!!!!!!!!!!!!!!!!!!!!!!!!!!!!!!!!!!!!!!!!!!!!!!!!!!!!!!!!!!!!!!!!!!!!!!!!!!!!!!


Найдите все а при котором функция f(x)=x*x - |x-a*a| - 11x имеет одну точку максимума

1996. drugoy10, 7 июня 2010, 14:15:37
Всем Удачи!!!

1997. Dasha, 7 июня 2010, 17:08:54
Ребят, если АС=24, АВ=30, в прямоугольном треугольнике, чему будет равен sinА-? Если не трудно, ответьте пожалуйста!!!

1998. drugoy10, 7 июня 2010, 17:49:08
Dasha, 0,6

1999. OK, 7 июня 2010, 17:55:28
кто решал С5 из поста 1932 дайте ответ. (-5,-3)u(3,5)?

2000. yu5euy6e65e, 7 июня 2010, 17:56:24
0.6

2001. Dasha, 7 июня 2010, 18:03:34
У меня никак 0,6 не получается! Только 0,8!...

2002. ANDRESS, 7 июня 2010, 18:11:28
порог по русскому языку 67 баллов это точно? А 37 балов из 60 или переведеные из ста?

2003. drugoy10, 7 июня 2010, 18:13:12
Dasha,
сторона ВС равна 18, sinA - отношение ВС к АВ, т.е 18/30 или 3/5 или 0,6... ты видимо cosА нашла...

2004. Dasha, 7 июня 2010, 18:17:27
drugoy10, у меня про ВС ничего не было сказано...

2005. drugoy10, 7 июня 2010, 18:24:15
Dasha, ну как бы ВС=корню из 30 в квадрате - 24 в квадрате (теорема Пифагора), и, соответственно, равна 18...

2006. Dasha, 7 июня 2010, 18:32:51
Что помню:
В1-8(там про ручки было)
В2-2590
В3-6
В5-258000(точно не помню)
В6-6
В8-(-0,8)
У кого так было?

2007. Юлия, 7 июня 2010, 19:03:58
У меня вроде похожие ответы, только в В1 у меня 20 получилось

2008. ANDRESS, 7 июня 2010, 19:16:40
люди на какой сайт надо зайти чтоб оценки узнать?

2009. ANDRESS, 7 июня 2010, 19:26:19
Я сделал 10 правельно из B и одно из C1

2010. юлёк, 7 июня 2010, 21:09:06
у кого ни спроси, все практически всю первую часть и с1 сделали. интересно, какой порог будет? И ещё: ответы на с2 и с3 нормальные? в сысле, целые числа?

2011. МеТиСкА)), 7 июня 2010, 21:12:37
привет всем.вот читала ваши задания,почти задания у всех совпалают...а у меня нет(((обидно,даже не с кем свериться((

2012. ThDark)), 7 июня 2010, 22:46:56
Всем привет) У кого была задача в С2 про прав. 3-ую пирамиду.Стороны основания 20 корней из 3. боковый грани 29) Там что то про угол между прямой и плоскостью=)Плиз напишите ответ ппц как надо(((

2013. Леонид , 7 июня 2010, 22:59:55
Сначала напиши условие без "что то"! И что значит боковые грани 29?

2014. Мучительница, 8 июня 2010, 03:59:58
Задача из ЕГЭ 2010
С4 В тр-ке АВС АВ=10, ВС=5, СА=6. Точка Д лежит на прямой ВС так,что ВД:ДС=
=1:2. Окружности, вписанные в каждый из тр-ков АДС и АДВ, касаются стороны АД в точках Е и Ф. Найти длину отрезка ЕФ.
РЕШЕНИЕ
Т .к. точка Д лежит на прямой ВС, то возможны два случая.
Если точка Д внутри отрезка ВС, то ВД=53 и ДС=103 , если на луче СВ, то ВД=2,5, ДС=7,5.
из теоремы об отрезках касательных, выходящих из одной точки( они равны) получаем,что каждй такой отрезок равен р-а, р-в, р-с, где р-полупериметр, а,в,с-стороны треугольника( для нахождения отрезка касательной надо из полупериметра вычесть сторону треугольника, лежащую против вершины ттреуголника из которой проведены касательные)
Значит, ДФ=0,5(АВ+ВД+АД)-АВ= 0,5(ВД+ДС-АВ)
ДЕ=0,5(АД+ДС+АС)-АС=0,5(АД+ДС-АС)
ЕФ=ДЕ-ДФ
После вычислений получаем в первом случае ЕФ=176, во втором случе ЕФ=4,5.

2015. Мучителница, 8 июня 2010, 04:06:36
в предыдущем решении ВД=пять третьих, ДС=десять третьих
ЕФ=две целых пять шестых

2016. OK, 8 июня 2010, 06:06:07
Найти все значения параметра а,при котором функция f(x) = x^2-2|x-a^2|-8*x, имеет более двух точек экстремума?
решала так: нашла контрольную точку(где модуль =0):х=а^2

x<=a^2,f(x)=x^2-6x-2a^2;
x>a^2, f(x)=x^2-10x+2a^2;
у параболы на первом луче вершина Хо=3, на втором Хо=5
третий (острый) экстремум будет между ними, если 3<a^2<5,
заменила системой: a^2<5
a^2>3 ответ:-5<a<-3 и 3<a<5
пожалуйста, посмотрите, правильно?

2017. Валерия, 8 июня 2010, 09:47:58
люди на какой сайт надо зайти чтоб оценки узнать?

Андрес,на сайт администрации своей области-города!


а у кого нибудь была в11 такая:
У=20tg(x)-20x +5П-3 [-п/4; п/4]

какое там решение?

2018. Леонид , 8 июня 2010, 20:32:46
Вообще-то, большинство заданий С были прще, чем на диаеностических заданиях и пробном ЕГЭ по математике.

2019. Леонид , 8 июня 2010, 20:44:14
Lexxus, может быть такое "Извините, сервер перегружен" висит весь день на сайте о результатах ЕГЭ по Кемеровской области?

2020. Ксения, 8 июня 2010, 22:38:38
Валерия,у меня была похожее задание)

берешь производную от этого выражения,потом приравниваешь к нулю....и подставляешь -п/4,п4 и то что еще входит в этот промежуток,то что получилось когда вяла производную)

2021. Lexxus, 8 июня 2010, 22:48:23
Lexxus, может быть такое "Извините, сервер перегружен" висит весь день на сайте о результатах ЕГЭ по Кемеровской области?

Да, такое быть, безусловно, может.
У вас не слишком много народу - я думаю, всего-то около 15 тысяч сдающих ЕГЭ и, скажем, столько же просто интересующихся его результатами. Но и сервера, полагаю, куда менее производительные, чем, например, тот, что позволяет вам читать этот текст.

Когда администраторы вашего местного сервера увидели, что он не справляется с нагрузкой, они поступили радикально - просто повесили сообщение "Извините, сервер перегружен".

Во всяком случае, этот сценарий кажется мне наиболее правдоподобным.
И, предваряя следующий вопрос, скажу, что я, конечно же, понятия не имею, когда сайт станет вновь доступным.

2022. Валера, 8 июня 2010, 22:51:58
1-2x
2 =128

Какой ответ?

2023. мучительница, 9 июня 2010, 00:00:14
ВАЛЕРА, получится -3

2024. Валера, 9 июня 2010, 00:20:49
Коментарий скрыт (грубость — 7%)показать

2025. xxSp@yNxx, 9 июня 2010, 11:33:46
ЕГэ - необъективная хуйня)
А перевод из первичного балла в вузовскую систему ещё хуже.
Например,у нас в классе золотая медалистка (она её реально заслужила) и двоишница написали русский на 60 и 64 балла,причём двоишница получила 64,а отличница 60.Это даже не смешно,знаешь,что этот человек не мог так облажатся и всё видеть такой результат...

2026. cnosc, 9 июня 2010, 12:41:57
а у кого нибудь была в11 такая:
У=4tg(x)-4x -П+7 [-п/4; п/4]

2027. Леха М, 9 июня 2010, 13:39:14
у меня была такая! ответ -13 вроде....

2028. cnosc, 9 июня 2010, 14:02:10
разве не 3

2029. Валерия, 9 июня 2010, 14:59:52
Ксения,а у меня получилось что-то 291 вроде

2030. ксюша, 9 июня 2010, 16:04:02
на ЕГЭ очень трудно и я не хочу его сдавать

2031. Валерия Волгоград, 9 июня 2010, 16:54:00
2030. ксюша, 9 июня 2010, 16:04:02
на ЕГЭ очень трудно и я не хочу его сдавать

нет,егэ не трудно,это все решаемо. больше паники создают учителя!

2032. Валерия Волгоград, 9 июня 2010, 16:56:43
у меня 59 баллов, а у моей подруги-отличници 56!!!
конечно это ужасно.
у меня за С 12 баллов,а ей всего 3 поставили.

2033. cFg, 9 июня 2010, 21:38:25
Фу, слава богу , написал русский язык на 63 балла

2034. Dima, 10 июня 2010, 00:34:20
а физику тоже так будут решать?

2035. Dim , 10 июня 2010, 07:01:37
Кто нибудь из Башкирии знает свои результаты по русскому?

2036. Леонид , 10 июня 2010, 07:46:50
заходи htth://www.ege.edu.ru/ иди по ссылкам, ищи свой регион и т.д.

2037. Dim, 10 июня 2010, 08:34:16
Скажите уже кто-нибудь как узнать результаты в Башкирии?

2038. AleRon, 10 июня 2010, 08:47:58
УРААААААААААА! РУССКИЙ СДАЛ, 60 БАЛЛОВ ЧЕТВЕРКА! ДУМАЛ ПРОВАЛЮ! весь год балду гонял! Осталось математику узнать и завтра история:((((

2039. Андрей, 11 июня 2010, 11:00:51
Да не парьтесь :) ЕГЭ по физике - халява . Вот мне её в универ в воскресенье сдавать это да . Я бы лучше ЕГЭ сдал ещё раз )

2040. Катюша, Нефтеюганск, 11 июня 2010, 13:33:59
Катюшка, а зачем ее тогда выбрала?)

2041. Катюша, Нефтеюганск, 11 июня 2010, 13:35:27
2040. maxerka, 11 июня 2010, 13:24:15
свобода!!!!!физика была последней!Часть а(хз сколько правильно),часть в-наугад и с-1


неееее я только в части а наугад поставила)

2042. dura umnaya, 11 июня 2010, 17:11:55
сдали физику Мск.
было просто.
но вспомнить все формулы так и не удалось(.
ожидаю плохие результаты(.

2043. dura umnaya, 11 июня 2010, 17:13:35
сегодня писали физику.
было просто.
но вспомнить все формулы так и не удалось(.
ожидаю плохие баллы(.черт.

2044. dura umnaya, 11 июня 2010, 17:52:32
физика.проехали...
было легко.
но,черт,к почти 18 годам не запомнить формулу электромагнитных колебаний!!!
расстроена(.
в задаче на термодинамику условие неправильно прочитала.сосуд блин не горизонтально,а вертикально и поршень поехал!!

2045. dura umnaya, 11 июня 2010, 17:53:31
блин!!
третий раз набираю сообщение!!
почему не публикуется?!! ><

2046. Леонид , 11 июня 2010, 22:36:05
К чему на всех страичках пишите одну и ту же ерунду?

2047. Lexxus, 11 июня 2010, 23:48:02
К чему на всех страичках пишите одну и ту же ерунду?

На всех страничках все пишут разную ерунду. Это я напутал, переделывая вывод комментариев.

2048. Киса, 12 июня 2010, 16:12:39
Когда примерно будет известна математика???

2049. Кирилл, 15 июня 2010, 00:57:42
оБЩЕСТВО ответы?выложат когда и где?от помощи ребят не откажусь

2050. Dim, 15 июня 2010, 06:49:16
Ну чё там с математикой есть примерные сроки?

2051. Elli, 16 июня 2010, 10:45:40
у меня 81 балл.
Леонид! что у тя? наверое 100?

2052. валюша, 21 июня 2010, 10:10:27
В прямоугольном параллелипипеде АВСDА1В1С1D1 известны ребра:АВ=5,АD=12,СС1=3.найти угол между плоскостями ВDD1 и АD1В1.
помогите пожалуйста!

2053. Людмила, 22 июня 2010, 13:02:31
здравствуйте.помогите решить 2 нормеа. B11 -
y= 28 tg x - 28 x - 7 П + 7
найти наименьшее значение функции на отрезке от - пи / 4 до пи / 4 в квадратных скобках.

можно в действия пжлст)

и С2 -
В правильной шестиугольной призме ABCDEF A1B2C3D4E5F6 все ребра равны 1. Найдите расстояние от точки C до прямой A1B1


2054. турал, 30 июня 2010, 01:59:44
точка А находится на расстоянии 7 см от каждой из 2 пересекающихся плоскостей расстояние от нее до прямой пересечения плоскостей 14 см. найдите угол между плоскостями

2055. Zheka, 30 июня 2010, 19:44:20
Доброго времени суток! Задачку одну решить не могу, может кто подсобит?
"Найти три числа больше (кажется) 20, квадраты которых образуют арифметическую прогрессию." Вроде всё условие. Заранее спасибо!

2056. Вика, 5 июля 2010, 18:31:38
Как можно написать

2057. Lili, 5 июля 2010, 18:57:17
Помогите пожалуйста.Найдите наименьшее знач.функции у=6+2п-8х-8корень2cos x на отрезке[0;п/2]

2058. Михаил, 6 июля 2010, 12:09:40
Всем доброго времени!
Lexxus, идея конечно классная, но скажи, есть ли спам-фильтр в открытом банке ЕГЭ?

2059. Мари, 8 июля 2010, 17:39:25
Здравствуйте!Lexxus,вы будете здесь,т.е на сайте в день проведения экзамена 12 июля?Чем мы хуже других(

2060. Lexxus, 8 июля 2010, 17:53:52
Здравствуйте!Lexxus,вы будете здесь,т.е на сайте в день проведения экзамена 12 июля?Чем мы хуже других(

Ох, где я, а где 12 июля?...
Но я попытаюсь.

2061. Ксения, 9 июля 2010, 09:40:49
мне надо найти сумму целых неравенств Х в степени 2-13х+36<0

2062. Жиган, 11 июля 2010, 00:01:28
У меня тоже 12 числа в 10.00 экзамен было бы не плохо если кто нибудь помог. . Если есть вариант помоги

2063. Vol4onko, 12 июля 2010, 00:30:40
Мне кажеться,что нас уже никто не спасет:)))осталось 9 с половиной часов до экзамена

2064. Мария, 17 июля 2010, 15:54:50
Здравствуйте!Скажите пожалуйста кто сдавал 12 июля математику,какое задание было под в11-на нахождение наибольшего значения функции от(у) на отрезке или другие??оч срочно(

2065. ученица, 26 июля 2010, 21:40:12
В основании прямой призмы лежит прямоугольный треугольник с катетами 9 и 3. Боковые ребры равны 5/п. Найдите объем цилиндра, описанного около этой призмы. ПОЖАЛУСТА РЕШИТЕ!!! Я РЕШАЮ, У МЕНЯ ОТВЕТ НЕ СХОДИТСЯ!!! правильный ответ 112,5 - а у меня 125 получается(((

2066. Liana, 31 июля 2010, 14:43:30
а будут ли ответы на вопросы ЕГЭ 2011г по математике?

2067. many, 8 августа 2010, 15:37:52
Два велосипедиста одновременно отправляются в 168-километровый пробег. первый едет со скоростью на 2 км/ч большей, чем второй, и прибывает к финишу на 2 часа раньше второго. Найти скорость велосипедиста, пришедшего к финишу вторым

2068. many, 8 августа 2010, 15:40:10
помогит решить пожалуйста?? очень срочно надо...

2069. Ilia_, 8 августа 2010, 17:00:43
Первый ехал со скоростью 14 км/ч, второй - 12 км/ч, в чём проблема?

2070. Л.М., 12 августа 2010, 14:54:58
В чём, в общих чертах, предполагается отличие задач открытого банка заданий ЕГЭ по математике в 2011 от 2010?

2071. Л.М., 26 августа 2010, 12:14:42
демоверсия 2011 показывает, что открытый банк заданий 2010 сохранится и 201год.

2072. андрей, 26 августа 2010, 17:55:04
помогите решить пример, 3a+5b5a+7b если 4a+3b=0

2073. Л.М., 27 августа 2010, 09:05:05
вырази из второго, например, а=-3в:4. Вместо а подставь в первое
Задание, наверное, упростить?

2074. надежда 2010, 4 сентября 2010, 11:10:16
а задания 2010 остаются в банке, или новые добавятся? А количество зарегестрированных остается и добавляютя новые участники? А то что-то далеко от старта начинать надо, чтобы выйти в первую хотя бы сотню.

2075. Lexxus, 4 сентября 2010, 12:42:46
а задания 2010 остаются в банке, или новые добавятся? А количество зарегестрированных остается и добавляютя новые участники? А то что-то далеко от старта начинать надо, чтобы выйти в первую хотя бы сотню.

Из официального банка задачи 2010 года никуда не делись - просто добавились новые. Так что тут тоже старые задачи никуда не исчезают.

Что касается участников, то, возможно, я разделю рейтинги по годам. Это, в принципе, логично.

2076. Леонид , 6 сентября 2010, 19:26:31
Lexxus! Приветствую Вас. Просмотрел банк заданий на 2011. Смысл заданий сохранился. В В9 добавлены несколько новым содержанием - задачки близкие к школьным.
Посоветйте, где можно отыскать несколько вариантов 2010 - второй части.

Не могли бы вы дать данные справляемости с заданиями В1-В12 в % по России или подсказать, где можно найти их.

С наступившим учебным годом.

2077. Lexxus, 7 сентября 2010, 12:20:53
Lexxus! Приветствую Вас. Просмотрел банк заданий на 2011. Смысл заданий сохранился. В В9 добавлены несколько новым содержанием - задачки близкие к школьным.

Ещё много новых типов заданий B4.

Посоветйте, где можно отыскать несколько вариантов 2010 - второй части.

Во-первых, если прямо тут по комментариям пособирать, то соберётся куча вариантов. Разве только С6 тут мало выкладывали.
Кроме того, у меня где-то завалялась хорошая книжка с заданиями части С. Практика показала, что задания там вполне соответствовали реальному экзамену по тематике и сложности. Найду - выложу.

Не могли бы вы дать данные справляемости с заданиями В1-В12 в % по России

Официальной статистики вроде бы нет. Могу посчитать только по своим данным.

С наступившим учебным годом.

Вас тоже. Буду рад, если и в этом году вы будете также часто тут бывать.

2078. Артём, 8 сентября 2010, 21:22:48
Люди я хочу спросить а тут нету тестов по химии и биологии ?

2079. Chameleon, 10 сентября 2010, 21:47:37
Может новость для обсуждения пора другую использовать?

2080. Иван, 13 сентября 2010, 22:33:20
Люди,помогите решить уравнение,sin^22x-sin^24x=1/2sin(3(2x-п/3))?Очень нужно,пожалуйста,готовлюсь к егэ,помогите!!!!!!=)

2081. ilo, 14 сентября 2010, 22:24:09
Иван,,sin^22x-sin^24x=1/2sin(3(2x-п/3))
(sin2x-sin4x)(sin2x+sin4x)=-1/2sin(п-6x)
-2sinxcos3x2sin3xcosx=-1/2sin6x
-sin2xsin6x=-1/2sin6x
1/2sin6x-sin2xsin6x=0
sin6x(sin2x-1/2)=0
1)sin6x=0 или 2)sin2x=1/2
из1:х=п|12+пn/6 входят в 2) х=(-1)^nп/12+пn/6

2082. ilo, 14 сентября 2010, 22:34:42
ошибочка вышла...
во 2)x=(-1)^nп/12+пn|2, осталось решить вопрос о входимости

2083. ilo, 14 сентября 2010, 22:50:13
Ответ: п/12+пn; 5п/12+пn; п/12+пn/6.

2084. Бахриев , 23 сентября 2010, 19:01:31
1) 3м2дм5см
+4м5дм3см=

2085. Бахриев, 23 сентября 2010, 19:04:52
Пажалуйста помогите мне срочно надо 1) 3м2дм5см
+4м5дм3см=
Решите по быстрей я вас прашу

2086. Леонид , 23 сентября 2010, 19:48:05
Ты в каком классе, чудак?

2087. Maxim, 24 сентября 2010, 13:55:38
помогите плз(

5sinx+4=|5cosx+2|

???? TT

2088. Леонид , 25 сентября 2010, 05:46:41
не понятно 5 умн. на синус или 5 в ст. сину. Если в ст., модуль опускай, т.к. левая часть и подмодуль положительны при всех х. далее рассуждай по виду ур-я

2089. Maxim, 26 сентября 2010, 18:31:51
ненет)извиняюсь))
5*SinX + 4 = | 5*CosX + 2 |

2090. Леонид , 26 сентября 2010, 19:04:00
5sinx - 5cosx =-2 (в левой части подмодуль положителен), - опустили знак модуля и перенесли. Примени к sinx и cosx формулы двойного угла. В правой части -2*1 (1 - как тригонометрическую сумма квадратов синусо и косинуса х/2). Всё в одну часть, подобные - однородное триг. ур-е, дели обе части на cos^2 (x/2). кв-е ур-е относительно тангенса и все дела - Я так понял твоё условие.

2091. марина, 27 сентября 2010, 18:07:21
решите задачу:из пункта А в пункт В,расстояние между которыми 27 км ,одновременно навстречу друг другу вышли два пешехода и встретились через 3 часа.пешеход который вышел из пункта А,приходит в пункт В на 1час 21мин.раньше чем другой приходит в пункт А.найдите скорость каждого пешехода.

2092. ilo, 29 сентября 2010, 08:37:16
марина,Первое уравнение получаем из условия, что до встречи шли 3 часа: v+u=9, второе из условия, что второй шёл дольше на 81/60ч: 27/u-27/v=81/60. После преобразований приходишь к квадратному ур-ю v^2+31v-180. отв.5 и 4

2093. ilo, 29 сентября 2010, 09:35:10
Леонид , квадратное уравнение 7t^2+10t-3=0?

2094. Леонид , 29 сентября 2010, 21:57:00
2093. ilo, И что? Уравнение Д = 100+84 = 184 - корень не извлекается.
-10 - 2 корня из 46 и -10 + 2 корня из 46. Откуда это уравнение, может составлено неверно. Или дано и решить, ответ написал, если так?

2095. ilo, 29 сентября 2010, 22:12:46
Леонид , 26 сентября ...кв-е ур-е относительно тангенса и все дела ...
не катит что-то

2096. Lexxus, 29 сентября 2010, 22:24:50
Леонид , 26 сентября ...кв-е ур-е относительно тангенса и все дела ...

Всё правильно, это же задачка типа C1. Корни могут быть и дурацкими.
Они там действительно дурацкие - что в одном, что в другом случае.

2097. ilo, 29 сентября 2010, 23:10:06
поясните, пжл, почему в левой(?) части подмодуль положителен? Подмодуль положителен в правой части при -п+arccos2/5<x<п-аrccos2/5. как установить что эти корни туда попадают?Пробую решать заменой cos на sin по ф-ле приведения, получаю, что надо сравнить дуги 5п/4+arcsin корень из2/5 и -п+аrccos2/5 Как?

2098. Lexxus, 29 сентября 2010, 23:37:42
Подмодуль положителен в правой части при -п+arccos2/5<x<п-аrccos2/5. как установить что эти корни туда попадают?Пробую решать заменой cos на sin по ф-ле приведения, получаю, что надо сравнить дуги 5п/4+arcsin корень из2/5 и -п+аrccos2/5 Как?

А вот тут я, честно говоря, затрудняюсь ответить.
Если аккуратно построить график левой и правой частей, то более-менее понятно, какие корни отбрасывать, но именно что "более-менее".
С калькулятором, естественно...

А вот без всего...

2099. Леонид , 30 сентября 2010, 17:12:39
2089. Maxim, 26 сентября Предлагаю, просто решить 2 уравнения:
5*SinX + 4 = | 5*CosX + 2 |
1) 5sinx +4=5cosx+2,
2) 5sinx+4=-5cosx - 2, решение там выше описано.
Выбрать, удовлетворяющие: в 1-м правая часть больше либо равно 0, во втором меньше 0. Пробуйте.


Lexxus, убери пожалйста безобразие № 2099 - оно портит наше общение.

2100. Lexxus, 30 сентября 2010, 17:34:21
Lexxus, убери пожалйста безобразие № 2099 - оно портит наше общение.

О, вы ещё не видели то, на что это было ответом.

2101. ilo, 30 сентября 2010, 21:05:49
5sinx+4=5cosx+2;
Sinx-sin(π/2-x)=-2/5
X=π/4-arcsin √2/5+2πk ϵ-π+аrccos 2/5≤x≤π-arccos 2/5
X=5π/4+arcsin √2/5+2πk,
Пытаюсь сравнить длины дуг 5π/4+arcsin √2/5 и-π+arccos 2/5
Или π/4+arcsin √2/5 и arccos 2/5
Найдём сумму arcsin-ов:arcsin √2/2+arcsin √2/5=arcsin (√46+2)/10
arccos2/5=arcsin √21/5
arcsin(√46+2)/10 первая дуга короче, корень не попадает в промежуток,
выбрать как предлагает Леонид не получается

2102. ilo, 30 сентября 2010, 21:11:42
Да, ребята....

2103. Леонид , 1 октября 2010, 21:36:00
Уважаемый LEXXUS, с наступающим Днём Учителя!!! Здоровья, творчества, терпения. Ваш труд выпускникам необходим.

2104. Lexxus, 1 октября 2010, 21:43:57
Уважаемый LEXXUS, с наступающим Днём Учителя!!! Здоровья, творчества, терпения. Ваш труд выпускникам необходим.

Спасибо, конечно, но к вам этот праздник имеет гораздо большее отношение, чем ко мне. Так что, как говорится, и вас туда же :)

2105. ilo, 2 октября 2010, 18:46:27
Всех с праздником!
2089. Maxim, 26 сентября
Л.ч. д.б неотрицательна и ещё 2 способа знаковыражения модуля – имеем совокупность 2-х систем: (D(f))
1.{█(5sinx+4≥0@5cosx+2≥0)┤
2.{█(5sinx+4≥0@5cosx+2<0)┤
Предварительно придётся установить взаимное расположение точек
π+arcsin 4/5 и –π+arccos 2/5
Вторая дуга длиннее, всё изобразить на тригонометрическом круге –получишь решение 1 системы:
-arcsin 4/5≤x≤π-arcsin 2/5
Ей соответствует уравнение 5sinx+4=5cosx+2
Решение: x_1=π/4-arcsin √2/5+2πk,kϵZ∈D(f) x_2=5π/4+arcsin √2/5+2πk,kϵZ∉D(f)
Это легко установить по кругу, по тому, каким четвертям принадлежат эти точки.
2. система имеет решение: π-arccos 2/5≤x≤π+arcsin 4/5
Соответствующее этой системе уравнение: 5sinx+4=-5cosx-2
x_3=π-arccos (3√2)/5+π/4+2πk,kϵZ x_4=-π+arccos (3√2)/5+π/4+2πk,kϵZ
Положение этих точек по кругу установить нельзя, приходится сравнивать длины дуг
π/4-arccos (3√2)/5 и π/4+arccos (3√2)/5 c arcsin 4/5
(раньше было как)
x_3∈D(f), x_4∉D(f)
Отв.: x_1,x_3

2106. lord, 5 октября 2010, 13:42:53
http://shareflare.net/download/89132.817dda0f24fffed360155e34878f/algebra___.zip.html
вот хороший сборник (алгебра в таблицах) к егэ по МАТЕМАТИКЕ!

2107. ponka, 7 октября 2010, 16:27:54
камень брошен с высоты 24 метра .высота h на которой находится камень зависит от времени t: h(t)=24-5t-t^2 ...сколько секунд камень буден падать???

2108. Марина, 7 октября 2010, 17:32:46
Здравствуйте!!! научите пожалуйста решать B 10!!
например, вот эту: В розетку электросети подключены приборы, общее сопротивление которых составляет Ом. Параллельно с ними в розетку предполагается подключить электрообогреватель. Определите (в омах) наименьшее возможное сопротивление этого электрообогревателя, если известно, что при параллельном соединении двух проводников с сопротивлениями и их общее сопротивление даётся формулой , а для нормального функционирования электросети, общее сопротивление в ней должно быть не меньше 30 Ом.

2109. Lexxus, 7 октября 2010, 17:42:45
камень брошен с высоты 24 метра .высота h на которой находится камень зависит от времени t: h(t)=24-5t-t^2 ...сколько секунд камень буден падать???

Нужно найти время, при котором h(t)=0, т.е. решить уравнение
24-5t-t^2=0
Корни этого уравнения: -8 и 3. Поскольку камень не может падать минус 8 секунд, то ответ 3.

2110. ilo, 7 октября 2010, 19:19:13
Марина, 7 октября
Если сильно не заморачиваться, то в заданную формулу подставить все данные из задачи и составить неравенство по условию "не меньше",т.е. больше, либо равно 30.А т.к требуется наименьшее, то это при условии равенства 30, все остальные значения больше. Значит - решить уравнение ....=30 относительно неизвестного (сопротивления обогревателя).
Это ни в коем случае не рекомендации к решению ВСЕХ задач В10. Чтобы научиться, надо решать.

2111. Felta, 7 октября 2010, 20:50:43
Катер 11.00 вышел из пункта А в пункт Б, расположенный в 15 км от А. Пробыв в пункте Б 1 час 20 мин. отправился назад и вернулся обратно в пункт А в 15.00 того же дня. Определить собственную скорость катера, если скор течения реки 3 км/ч
Если можно, с решением, плиз, умоляю

2112. Артем, 7 октября 2010, 20:58:28
Ребят, системы уравнений никогда не любил х)
Хэлп, пожалста
http://i081.radikal.ru/1010/03/2d4078499345.jpg

2113. Леонид , 7 октября 2010, 21:21:36
Ну и что там? Приравнивай числитель к нулю, находи Х, выбирай те х, которые отвечают ОДЗ и при них у. А если нужно, пиши сюда систему. Иначе из-за тебя надо прыгать туда-сюда.

2111. Felta, 7 октября 2010, 20:50:43
Таких задач здесь (выше) море решено,смотри, учись, не ленись.

2114. Маринка=)), 8 октября 2010, 20:35:49
Первый рабочий за час делает на 2 детали больше, чем второй рабочий, и заканчивает работу над заказом, состоящим из 192 деталей, на 4 часа раньше, чем второй рабочий выполняет заказ, состоящий из 224 таких же деталей. Сколько деталей делает за час второй рабочий?
помогите пожалуйста=)))

2115. ilo, 8 октября 2010, 21:32:55
1 2
производительность х+2 х
(кол-во в час)
заказ 192 224

t 192/x+2 < 224/x на 4


224/x-192/x+2=4 кв.ур-е х^2-6x-112=0 Отв.14

2116. настя, 9 октября 2010, 15:35:01
может ли кто нибудь помочь мне решить вот это
ДАНО ТОЧКИ А (-4 7 0) В ( -5 3 3 ) С ( 0 1 3)
НАЙТИ cos угла АВС И ТРЕУГОЛЬНИК АВС



пожалуйста помогите решить!

2117. Леонид , 10 октября 2010, 08:29:22
Девушка, о каких координатах пишешь? Где (;) между числами - иначе не видно о двухмерном или трёхмерном пространстве речь идёт?

2118. настя, 10 октября 2010, 12:22:41
ну Леонид вы все прям видите.представьте что я их подставила)

2119. Леонид , 10 октября 2010, 13:01:47
2116. настя, 9 октября 2010 Куда и что подставила? Находи координаты (вектора) сторон треугольника - Из координат конца вычесть координаты начала, Затем длину вектора (стороны) - корень кв. из суммы координат. Далее теорема косинусов для стороны АС - кв. стороны равен сумме кв. 2-х других сторон минус удвоенное произведение этих сторон на косинус угла между ними - подставляй выражай косинус.

Но я так и не понял, что и во что ты подставила.

НАРОД - пшите условие поняиным

2120. Леонид , 10 октября 2010, 13:05:49
поправка: корень квадратный из суммы квадратов координат вектор (стороны)

Точка А(4;0;7) так надо понимать?

2121. настя, 10 октября 2010, 16:30:19
а вы Леонид простите кто, учитель?

2122. Леонид, 11 октября 2010, 18:53:06
настя,
я не учитель - я школьник причем второгодник

2123. настя, 11 октября 2010, 18:59:05
у.и в каком классе сейчас?

2124. Леонид, 11 октября 2010, 19:05:36
настя я сейчас в 11 классе

2125. настя, 11 октября 2010, 19:09:17
ну удачной сдачи егэ тебе и не воспринимайте все так серьезно мой вам совет!

2126. Леонид, 11 октября 2010, 19:13:05
Настя, спасибо большое, тебе тоже удачно сдать ЕГЭ, а я наверное пойду в армию !

2127. настя, 11 октября 2010, 19:17:31
а я уже сдала егэ)

2128. Леонид, 11 октября 2010, 19:19:20
Молодец Настя, а насколько баллов

2129. настя473, 11 октября 2010, 19:24:34
не не очень много.хотя мучилась целый год. алгебра 52 русский 61 общество 52

2130. Леонид, 11 октября 2010, 19:27:21
165 баллов не очень много, на бюджет не потянешь

2131. настя, 11 октября 2010, 19:29:05
ну вобще то я уже на 1 курсе учусь и бюджет мне не нужен был

2132. Леонид, 11 октября 2010, 19:33:44
Готовься к сессии, 1 сессия самая сложная ! И не забывай главное, что первые 2 курса ты работаешь на зачетную книжку, остальные она на тебя.

2133. настя, 11 октября 2010, 19:35:27
откуда ты все это знаешь?)

2134. Леонид, 11 октября 2010, 19:40:38
Настя друзья рассказали

2135. настя, 11 октября 2010, 19:42:55
ладно спасибо за совет ты кстати из какого города?

2136. Леонид, 11 октября 2010, 19:45:50
Настя, а тебе зачем ?

2137. настя, 11 октября 2010, 19:47:17
просто интересно не хочешь не говори

2138. Леонид, 11 октября 2010, 19:52:04
Настя не обижайся !
я сходил бы с тобой в парк или кино !

2139. настя, 11 октября 2010, 19:55:00
да я вобще не про это

2140. Леонид , 11 октября 2010, 19:56:08
Здраствуй Lexxus! Кто - то под моим именем шпарит. С № 2122 - это не мой диалог. Просьба удалите их.

2141. Леонид, 11 октября 2010, 19:59:48
Настя не переживай ! УДАЧИ ТЕБЕ

2142. Lexxus, 11 октября 2010, 20:22:15
Здраствуй Lexxus! Кто - то под моим именем шпарит. С № 2122 - это не мой диалог. Просьба удалите их.

Так сходу нее нашлось точных данных, но полагаю, что в России насчитывается не один десяток тысяч Леонидов. И, даже будучи стопроцентно уверенным, что цитирую сейчас "Того Самого" Леонида, не считаю себя вправе лишать других возможности использовать это имя.

Во всяком случае, до тех самых пор, пока вдруг не решу сделать здесь регистрацию.

2143. Леонид , 12 октября 2010, 07:11:38
Но он так в диалог вступил, можно сказать, удачно, хотя и пишет, что второгодник-двоечник. Насте лапши на уши вешает. А та верит.

2144. Алeкс, 12 октября 2010, 09:51:50
А ты что им зaвидуeшь ? Кстaти второгодник нe обязaтeльно двоeчник, он можeт из-зa болeзни нa второй год остaлся.

2145. Леонид , 12 октября 2010, 19:54:39
ЧЕМУ? Чудак.

2146. Тимур, 12 октября 2010, 20:23:09
Здравствуйте! Помогите пожалуйста решить задачу. Пытался подойти со всех сторон к решению, но в любом случае получается огромный дискриминант, из которого не вычитается корень целым числом. Что то я запутался в ней, видимо =="

Из пункта А в пункт В, расстояние между которыми 40км, одновременно выехали мотоциклист и велосипедист. Известно, что за час мотоциклист проезжает на 50км больше, чем велосипедист. Определите скорость велосипедиста, если известно, что он прибыл в пункт В на 3 часа 20 минут позже мотоциклиста. Ответ дайте в км/ч.

За хотя бы какую-нибудь помощь, заранее благодарен.

2147. Гришa, 12 октября 2010, 20:58:33
ОТВЕТ НА ЗАДАЧУ - СКОРОСТЬ 100500 КМ/Ч

2148. Lexxus, 12 октября 2010, 21:01:05
Но он так в диалог вступил, можно сказать, удачно, хотя и пишет, что второгодник-двоечник. Насте лапши на уши вешает. А та верит.

Ну и что? Не вижу проблемы :)

Из пункта А в пункт В...

Тимур, полистай этот топик. Раз десять уже тут такую задачу решали.

2149. Тимур, 12 октября 2010, 21:45:14
да, я листал Тт и решал по примеру .-." а не получается и всё ==" я уже правда как только не пробовал хХ вот запутался где то и всё хХ
я посмотрел все задачи, но с такими цифрами выше не было .-.
помогите всё таки пожалуйста тТ

2150. Леонид , 13 октября 2010, 00:56:21
Из пункта А в пункт В, расстояние между которыми 40км, одновременно выехали мотоциклист и велосипедист. Известно, что за час мотоциклист проезжает на 50км больше, чем велосипедист. Определите скорость велосипедиста, если известно, что он прибыл в пункт В на 3 часа 20 минут позже мотоциклиста. Ответ дайте в км/ч.

За Х, что просят в задаче - скорость велосипедиста. То скорость мотоциклиста Х+50. Время велосипедиста 40/х , время мотоциклиста 40/(х+50) и он затратил времени меньше - уравнение 40/х - 40/(х+50) = 10/3 (время 3ч20мин - 3 1/3 часа или 10/3). Решаем уравнение: к общему знаменатею

120х - 120(х+50) = 10х(х+50). 120х-120х-6000= 10х^2 + 500х.

х^2 +50х - 600=0. х= 60. Проверяй, решая, вычисления.

2151. нaстя, 13 октября 2010, 12:47:44
Мнe никто *лaпшу нa уши нe вeшaeт* просто ты в концe нaшeго рaзговорa нaчaл мeтaться. Мол я ни я и лошaдь нe моя. Нe по мужски сдeлaл.

2152. Kattu, 13 октября 2010, 12:51:34
Здравствуйте, уважаемый Lexxus! К задачам под номерами 6941, 5417, 7371, 13995 даны неправильные ответы. Можно ли в этом открытом банке на Вашем сайте их исправить?

2153. Леонид , 13 октября 2010, 14:33:48
2151. нaстя, 13 октября . Ты о чём? И перед кем метаться?

2154. нaстя, 13 октября 2010, 14:59:53
Пeрeд мною конeчно. Снaчaло приглaшaл в кино, a кaк только я откaзaлaсь,нaчaл втирaть, что зa нeго кто-то другой пишeт

2155. Lexxus, 13 октября 2010, 15:25:51
Пeрeд мною конeчно. Снaчaло приглaшaл в кино, a кaк только я откaзaлaсь,нaчaл втирaть, что зa нeго кто-то другой пишeт

За не го действительно писал кто-то другой. И вообще, кончайте тут фигнёй заниматься.

2156. Lexxus, 13 октября 2010, 15:48:10
Здравствуйте, уважаемый Lexxus! К задачам под номерами 6941, 5417, 7371, 13995 даны неправильные ответы. Можно ли в этом открытом банке на Вашем сайте их исправить?

Ответы к заданиям 6941, 5417 и 13995 исправлены.
Ответ к №7371 и так правильный. Внимательно читаем задание.

2157. Kattu, 13 октября 2010, 16:57:03
Ответ к №7371 и так правильный. Внимательно читаем задание.
Я задание читаю внимательно. Там дан график функции. Точки экстремума по графику функции - это точки максимума и минимума. Складываем -3+ (-1) + 0 + 1+ 4 + 5 + 6 + 7 + 8 = 27. Так что ответ не правильный.
Еще ответ не правильный в номере 7925

2158. Lexxus, 13 октября 2010, 17:12:30
Я задание читаю внимательно.

О, пардон, палец соскочил, смотрел задание 7381.
Исправил.

2159. Lexxus, 13 октября 2010, 17:16:30
Kattu, большое спасибо за помощь.

2160. Леонид , 13 октября 2010, 18:17:09
2155. Lexxus, 13 октября . СПАСИБО.

2161. Леонид , 13 октября 2010, 18:20:21
2149. Тимур, 12 октября . Почему молчим. смотри запись 2190

2162. ЮЛИЯ87, 14 октября 2010, 07:39:02
помогите решить, пожалуйста!
из пункта А в пункт В, расп-ый ниже по течению реки, отправился плот. Одновременно на встречу ему из пункта В вышел катер. встретив плот, катер сразу развернулся и пошел назад. Какую часть пути от А до В проплывет плот к моменту возвращения катера в пункт В, если скорость катера в стоящей воде втрое больше скорости течения реки?

2163. Викинг 1234, 14 октября 2010, 10:53:29
помогите решить, пожалуйста!
из пункта А в пункт В, расп-ый ниже по течению реки, отправился плот. Одновременно на встречу ему из пункта В вышел катер. встретив плот, катер сразу развернулся и пошел назад. Какую часть пути от А до В проплывет плот к моменту возвращения катера в пункт В, если скорость катера в стоящей воде втрое больше скорости течения реки?

2164. викинг 1234, 14 октября 2010, 11:23:39
НУ и что не кто не знает ответа на эту задачу????

2165. Леонид , 14 октября 2010, 19:26:59
Мил человек, Викинг, откуда взята эта задача? В банке заданий ЕГЭ такого типа задач нет! Читай, разбирайся.
х - ск. плота и реки, 3х - собств. ск. катера. Пусть весь путь 1. Плот до встречи проплыл у км. - за время у/х часов. Катер до встречи проплыл (1-у)км., за время (1-у)/2х (скорость против течения 3х-х=2х). Их время до встречи одинаковое,- уравнение у/х= (1-у)/2х. решаем, умножив обе части на 2х, получим 2у= 1-у, 3у=1. у=1/3.

После встречи: плот проплыл а км., за время а/х. Катер проплыл (1-у) - тот же путь, за время (1-у)/4х (ск. катера по течению 3х+х=4х).
Время одно, уравнение а/х=(1-у)/4х. Умн. на 4х, имеем 4а=1-у. 4а = 1 - 1/3 (подставили у). 4а=2/3. а = 1/6. Всего плот проплыл 1/3+1/6= 1/2 всего пути.
УРА-а-а-а-а-а.

2166. Свeтa, 14 октября 2010, 20:54:57
Лeонид помоги пожaлуйстa рeшить зaдaниe. Сфeрa кaсaeтся конусa в eдинствeнной точкe (А) a тaкжe плоскости, в которой лeжит основaниe конусa, в точкe В. Пусть D . Диaмeтрaльно противоположнa точкe А , прямaя DA проходит чeрeз цeнтр конусa. Нaйти высоту конусa, eсли eго рaдиус 2,a рaдиус сфeры 3.

2167. ilo, 14 октября 2010, 21:22:19
2159. Lexxus, 13 октября, поясните,пжл, какие ответы всё же правильные в №№ 6941,7925,13995, т.к.в поиске заданий ответы неверные. Задание 6941 по-моему вовсе некорректно, т.к. нигде нет понятия что "целые точки" это выбранные из числовых промежутков, у которых координаты - целые числа.

2168. Lexxus, 14 октября 2010, 21:34:31
2159. Lexxus, 13 октября, поясните,пжл, какие ответы всё же правильные в №№ 6941,7925,13995, т.к.в поиске заданий ответы неверные.

Верные ответы на все эти задания - именно те, что отображаются после слова "ответ".
Задание 6941 по-моему вовсе некорректно, т.к. нигде нет понятия что "целые точки" это выбранные из числовых промежутков, у которых координаты - целые числа.

В школьном курсе математики используется достаточно много терминов, которые специально не вводятся и точно не определяются, но по идее интуитивно понятны.

2171. Леонид , 14 октября 2010, 22:44:14
2166. Свeтa, 14 октября. Проверь всё ли верно в условии. Что значит ... через центр конуса? - я такого понятия не встречал.

Общая мдея по вашему условию - это в осевом сечении окружность вписана в угол при основании конуса, её центр лежит на биссектрисе этого угла... Здесь я не могу делать чертёх Посмотри условие внимательно.

Пересечение медиан ....?

2172. tolerop, 15 октября 2010, 00:10:32
не могу преобразовать в произведение сомножителей: Дайте хотя бы идею...

1/а + 1/в + 1/с = 1/(а+в+с)

2173. tolerop, 15 октября 2010, 02:04:32
Ура! Сам решил.
Спасибо всё равно...

2174. Kattu, 16 октября 2010, 16:49:02
Здравствуйте, уважаемый Lexxus! К задачам под номерами 77187, 77189, 77213 даны неправильные ответы. Можно ли в этом открытом банке на Вашем сайте их исправить?

2175. Lexxus, 16 октября 2010, 17:03:30
К задачам под номерами 77187, 77189, 77213 даны неправильные ответы

А также 77191, 77193, 77195, 77207, 77209, 77211, 77215, 77225, 77227, 77229 и 77231, правильно?
Вы просто потрясающе оперативны. Ответы к этим заданиям определены (как "неправильные") буквально несколько часов назад.

Я совсем не уверен, какие ответы к этим заданиям будут считаться правильными на экзамене. Формулировка условия очень неоднозначная.

2176. 89081911647, 16 октября 2010, 17:53:20
а как мне пройти егэ по математики 9 класс?????

2177. Леонид , 16 октября 2010, 20:21:22
Например, №77225 ответ 25 000 считаю верным , другие не смотрел.

2178. Радик, 16 октября 2010, 20:50:17
Задание B8 (7815). Я не пойму, точка 19 - точка экстремума. Однако, нам дан промежуток [0,19] И на точке 19 заканчивается график. Следовательно она не меняется с + на - . А останавливается.

2179. Kattu, 16 октября 2010, 23:07:58
Уважаемый lexxus! Я согласна с тем, что задания некорректны и могут трактоваться по разному, но все-таки, если прочитать условие, то фраза "цена золота на момент закрытия торгов" определяет цену в дни торгов, и не определяется в выходные. Поэтому добавленные в ответы к задачам дни нельзя учитывать. В те дни когда биржа не работает, не может определяться цена золота на момент окончания торгов, которых в эти дни нет. Спасибо.

2180. Леонид , 16 октября 2010, 23:15:58
Интересная постановка проблемы о выходных днях. Торги могут проводиться в любые дни. В условии что-нибудь сказано о выходных?

2181. Kattu, 16 октября 2010, 23:26:03
Меня не интересует когда могут проводиться торги, на графиках в этих заданиях нет точек в эти дни, как, интересно, Вы определяете цену, может был "черный вторник" и цена скаканула. Здесь цена определяется только по точкам, а не по отрезкам их соединяющим, (читайте условие) , и в определенные дни точек просто нет, нет и цены товара на момент окончания торгов.

2182. Kattu, 16 октября 2010, 23:32:17
показана цена золота на момент закрытия биржевых торгов во все РАБОЧИЕ дни с 11 по 27 июля 2000 года. Обратите внимание на выделенное слово!!! Это из задания 77213...

2183. Леонид , 16 октября 2010, 23:52:43
Ответ 11. Жирные точки - цена золота отмечены во все дни. Причём тут выходные?

2184. Kattu, 17 октября 2010, 00:02:59
Конечно, правильный ответ-11, а выставлен как правильный -13. Цена золота отмечена только в рабочие дни, поищите на горизонтальной оси 15,16,22, 23 числа... Боюсь не найдете, так как это выходные дни , а в условии РАБОЧИЕ!!!

2185. Lexxus, 17 октября 2010, 01:56:49
Леонид, Kattu, в ваших доводах, безусловно, есть резон.

Я адресовал этот вопрос заведующему кафедры математики МИОО Ивану Валериевичу Ященко. Он, насколько я знаю, сам является разработчиком заданий, так что как он ответит (в случае, если ответит, конечно), так и правильно.

2186. Kattu, 17 октября 2010, 02:59:46
Уважаемый Lexxus, не тот случай, что бы беспокоить столь важного человека, при внимательном прочтении заданий ситуация предельно ясна. Во всех этих заданиях написано "РАБОЧИЕ ДНИ", они же и отмечены точками на графике... Спасибо.

2187. Lexxus, 17 октября 2010, 03:10:22
Уважаемый Lexxus, не тот случай, что бы беспокоить столь важного человека, при внимательном прочтении заданий ситуация предельно ясна. Во всех этих заданиях написано "РАБОЧИЕ ДНИ", они же и отмечены точками на графике... Спасибо.

Что ж, считайте, что вы меня убедили. Впрочем, я не вижу причин, чтобы лишний раз не побеспокоить кого бы то ни было, коль скоро это беспокойство ограничивается одним письмом.

2188. Kattu, 17 октября 2010, 03:23:41
Уважаемый Lexxus! Спасибо, что уделили внимание моим объяснениям...
Честно говоря, давно волнует ответ к заданию с наибольшим количеством показов и ответов 27529. Большинство ответило 4, Ваш ответ 3, а по-моему правильный 2. Там в задании есть слово "МЕЖДУ", вряд ли 2 мм осадков находится между 2 мм и 8 мм... Посмотрите,пожалуйста, когда будет время...
Спасибо.

2189. Lexxus, 17 октября 2010, 04:00:02
Честно говоря, давно волнует ответ к заданию с наибольшим количеством показов и ответов 27529. Большинство ответило 4, Ваш ответ 3, а по-моему правильный 2. Там в задании есть слово "МЕЖДУ", вряд ли 2 мм осадков находится между 2 мм и 8 мм... Посмотрите,пожалуйста, когда будет время...

Ещё одно задание с явно некорректным условием. Но мне всё-таки кажется, что в таких случаях более правильно считать, что имеется в виду отрезок, а не интервал, т.е. правильный ответ - 3. Повторюсь, что это лишь моё мнение. Разработчик вполне мог думать иначе.

2190. Kattu, 17 октября 2010, 10:36:08
Уважаемый Lexxus! Конечно, Вы рассуждаете как математик, и возможно, Вы были бы правы, если бы не одно но... По словам разработчиков эти задачи даны, как задачи с практическим содержанием и рассуждать на тему отрезков и интервалов здесь неуместно. Как бы любой человек ответил на вопрос: Какие числа лежат между 2 и 8. Ответ однозначный: 3,4,5,6,7. Математики обычно говорят, числа между 2 и 8 включительно. А так задание как всегда некорректное. Спасибо.

2191. Катерина, 17 октября 2010, 17:50:46
Помогите решить и объяснить пожалуйста:
площадь поверхности куба равна 50.найдите его диагональ

2192. Леонид , 17 октября 2010, 19:13:32
куб- площадь поверхности - площадь шести квадратов, то площадь одного 50/6, ребро куба а = 5/на корень из 3. Диагональ куба Д = а корней из 3. Значит ребро 5.

2193. Катерина, 18 октября 2010, 14:25:41
спасибо тебе огромное)))значит я правильно все решила))))

2194. дура))), 18 октября 2010, 18:16:16
помогите плизззз
5^9*2^8/10^7

2195. Леонид , 18 октября 2010, 22:22:29
5^8*5*2^8:10^7= 10^8*5:10^7=10:5=2, т.к. 5^9 = 5^8*5, 5^8*2^8 = 10^8

2193. Катерина, 18 октября - оправданий не надо. Запрос не соответствует оправданию.

2196. ilo, 19 октября 2010, 00:02:22
дура))), 18 октября 2010, 18:16:16
помогите плизззз
10^8*5/10^7=10*5=50

2197. Леонид , 19 октября 2010, 07:42:17
2195. Леонид . Верно 50. *5 лшибочно занисал :5.

2198. Kattu, 19 октября 2010, 12:52:57
Lexxus! Открытый банк пополнился новыми задачами B1. Не могли ли Вы их добавить на этом сайте! Спасибо.

2199. Lexxus, 19 октября 2010, 19:08:05
Lexxus! Открытый банк пополнился новыми задачами B1. Не могли ли Вы их добавить на этом сайте! Спасибо.

Спасибо за информацию. В ближайшие дни постараюсь обновить.

2200. Александр, 20 октября 2010, 10:35:11
Из пункта А в пункт В,расстояние между которыми 60км,одновременно выехали мотоциклист и велосипедист.Известно,что за час мотоциклист проезжает на 40 км больше чем велосипедист.Определите скорость велосипедиста,если известно что он прибыл в пункт В на 2 часа позже мотоциклиста.Ответ дайте в км/ч.

2201. Леонид , 20 октября 2010, 13:08:25
скорость велосипедиста (её надо найти) х, тогда мотоциклиста (х+40). Время велосипедиста 60/х, время мотоциклиста 60/(х+40). Велосипедист затратил на 2 часа больше - уравнение: 60/х - 60/(х+40) = 2. К общему знаменателю и разделив на 2, получим х в кв. +40х - 1200=0. х=20.


таких задач выше очень много ршено. смотрите.

2202. Леонид , 21 октября 2010, 06:53:16
Телефонная компания предоставляет на выбор три тарифных плана.

Тарифный план Абонентская плата Плата за 1 минуту разговора
Повременный 135 руб. в месяц 0,3 руб.
Комбинированный 255 руб. за 450 мин. в месяц 0,28 руб. за 1 мин. сверх 450 мин. в месяц.
Безлимитный 380 руб. в месяц


Абонент выбрал наиболее дешевый тарифный план, исходя из предположения, что общая длительность телефонных разговоров составляет 650 минут в месяц. Какую сумму он должен заплатить за месяц, если общая длительность разговоров в этом месяце действительно будет равна 650 минут? Ответ дайте в рублях.
Как понимать 1-ю строчку? 650*0,3+135 = 330? вторая 311 и третья 380. смысл безлимитки? Или делаю что-то не так? Не 135 же в первой? Тогда смысл залачи? Это прототип №26677. Lexxus,растолкуйте? Зациклило меня.

Здравствуйте, нужно решение С5 и С4, сама написала, но не уверена, что правильно.
С5 Найдите все значения а, при каждом из которых наибольшее значение функции f(x)=x^2-7|x-a|-3x на отрезке [-6;6] принимает хотя бы на одном из концов этого отрезка.
С4 Две окружности, касающиеся прямой в точках А и В, пересекаются в точках С и Д, причем АВ=8, СД=15. Найдите медиану СЕ треугольника АВС.

2204. Игорь, 21 октября 2010, 16:35:12
В7. Найдите боковое ребро четырёхугольной призмы, если сторона ее основания равна 20, а площадь поверхности равна 1200?

Обьясните пожалуста как решать такие задания.

2205. Игорь, 21 октября 2010, 18:18:41
Ошибочка вышла.
То задание не В7 а В9

2206. Леонид , 21 октября 2010, 18:47:18
2204. Игорь, 21 октября . Призма случайно не является рямой?

2207. Игорь, 21 октября 2010, 18:56:36
Нет.
Правильная четырёхугольная призма...

2208. Леонид , 21 октября 2010, 20:13:42
1200 = (20*20)*2 + 4*20*H (2 площади основания + площадь боковой поверхности = площади всей поверхности).
1200 = 800 + 80Н, Н =(1200-800):80, Н= 5.

Что же так невнимательно пишем условие?

2209. Игорь, 21 октября 2010, 21:09:26
Погоди...т.е боковое ребро призмы равно 5 ? (спасибо за помощь)


Помоги пжл ещё решить В12
В12. Первые 120 км автомобиль ехал со скоростью 60 км/ч, следующие 100 км со скоростью 70 км/ч, а затем 90 км со скоростью 90 км/ч.
Найдите среднюю скорость автомобиля на протяжении всего пути. Ответ дать в км/ч.

2210. Леонид , 21 октября 2010, 21:22:47
Пробуем, если я правильно понял: весь путь 120+100=90= 300 км.
Время на первом участке пути 120:60 = 2 часа, на втором 100:70= 10/7 часа, на третьем 90:90 = 1 час. Время затраченное на весь путь 2+10/7 + 1 = 31/7 часа. Находим срелнюю скорост- путь делим на время 310: 31/7 = 310*7/31 = 70 км/ч

В правильной и прямой призмах высота и боковое ребро одно и то же.
Дерзай!!!

2211. Игорь, 21 октября 2010, 21:42:02
Спасибо за помощь.
А ты (вы) учитель математики? Как часто на этом сайте?
Можно задать еще несколько вопросов по части В и С?

2212. Игорь, 21 октября 2010, 22:09:37
Леонид, а почему у тебя 2+10/7+1=31/7? должно же быть 13/7 и тогда 310*7/13=166,9 км/ч
Что то несходится...

2213. ilo, 21 октября 2010, 22:42:59
Игорь, 21 октября 2010, 22:09:37
2=14/7; 1=7/7; 14/7+10/7+7/7=31/7

2214. Игорь, 21 октября 2010, 22:53:30
ilo, 21 октября 2010, 22:42:59

Понял)

2215. Игорь, 21 октября 2010, 23:01:27
В11. Найдите наибольшее значение функции y=tgx-x+п4-11
на отрезке [-п/4,п/4]

2216. Леонид , 22 октября 2010, 00:05:38
2211. Игорь, 21 октября . Бываю часто. Пиши, только условие внимательно! iLo - показано верно, я не стал так подробно писать. При сложении дробей приводим к общему знаменателю - общий подход.

В11. Бери производную у щтрих = 1/sin^2 x - 1. Приравнивай 0, Sin^2 x =1.
x=-pi/2 + 2pik и x= pi/2 + 2pik При к=0 (-pi/2 и pi/2 - хорошо видно на единичной окружности. Да?), другие тем более не попадают в указанный промежуток в задании. Подставляем в данную функцию концы указанного промежутка и выбираем, что просят.
Находи у, подставив в данную функцию -pi/4: -1 -pi/4 + pi/4 - 11 = -12.
Находи у, подставляя в данную функцию pi/4: 1 - pi/4 + pi/4 - 11 = -10 - это наиболшее - ответ.

Задание алгоритмичнон - о наибольшем (наименьшем) значении функции на промежуике с помощью производной. Проверь, вдруг ошибочка - эти вещи здесь сложно печатать.

2217. Игорь, 22 октября 2010, 00:44:25
Леонид , 22 октября

Я тоже теперь часто сдесь буду...так и подучусь)))

С6. Наибольшее целое число, не превосходящее число х, равно
х''(в квадрате)+6/7
Нужно найти все такие значения х.
Можешь обьяснить как решать...пошагово?

2218. Игорь, 22 октября 2010, 00:49:42
Пояснение:

х(в квадрате)+6
(дробь)7

2219. Леонид , 22 октября 2010, 07:30:20
Всё понятно под №2215, Игорь? Почему не пишешь?

С6, так (х^2 + 6):7 меньше или равно х ? Не может быть С6 - это же простое кавдратное неравенство. Причём, если сказано целое число, значит скобка нацело делится на семь...Посмотри ещёраз условие для уточнения, потом подумаем.

2220. Игорь, 22 октября 2010, 08:08:16
Да, 7 делит скобку нацело. А в задании нет больше меньше или равно нулю.
просто выражение "икс в квадрате прибавляем шесть и все это дробью делим на 7"

№2215, я примерно знаю как решать но не все задания такого типа...
y=tgx-x+п/4-11

на отрезке [-п/4,п/4]

Как ты нашел производную у'? при чем тут Sin? если в задании tg?

2221. Лаура, 22 октября 2010, 08:28:39
Помогите решить задачу, срочно ПОЖАЛУЙСТА

2222. Лаура, 22 октября 2010, 08:31:09
Велосипидист проехал в первый день 65 км, во второй день 39 км. За сколько часов проедет за два дня велосипидист , если едет на постоянной скорости

2223. Игорь, 22 октября 2010, 08:42:25
2222. Лаура, 22 октября
Условие не корректно, проверь.

2224. Леонид , 22 октября 2010, 08:50:53
2220. Игорь, 22 октября . С6 - написано "не превосходящее" - значит меньше или равно.

Игорь, выучи формулы и правила дифференцирования. На твой вопрос: производная тангенса есть единица делённая на квадрат синуса. Меня ошоломил вопрос- откуда синус взялся.

Народ, пишите точное условие, в математике без ваших загадок, мыслей требуется достаточно не мало.

2225. Игорь, 22 октября 2010, 09:07:45
2224. Леонид , 22 октября

C6. да, как решить такой пример?

Игорь, выучи формулы и правила дифференцирования. На твой вопрос: производная тангенса есть единица делённая на квадрат синуса. Меня ошоломил вопрос- откуда синус взялся.-------------обязательно выучу.

2226. Kattu, 22 октября 2010, 12:06:40
Игорь!!! Лучше не учите эту формулу, дело в том, что производная от тангенса - единица деленная на квадрат косинуса. Удачи!

2227. Kattu, 22 октября 2010, 12:09:59
Lexxus! Открытый банк пополнился новыми задачами B3. Не могли ли Вы их добавить на этом сайте! Спасибо.

Здравствуйте, а почему мне никто не отвечает????
хоть кто-нибудь может помочь????????????????????
С5 Найдите все значения а, при каждом из которых наибольшее значение функции f(x)=x^2-7|x-a|-3x на отрезке [-6;6] принимает хотя бы на одном из концов этого отрезка.
С4 Две окружности, касающиеся прямой в точках А и В, пересекаются в точках С и Д, причем АВ=8, СД=15. Найдите медиану СЕ треугольника АВС.

2229. Lexxus, 22 октября 2010, 13:46:21
С5 Найдите все значения а, при каждом из которых наибольшее значение функции f(x)=x^2-7|x-a|-3x на отрезке [-6;6] принимает хотя бы на одном из концов этого отрезка.


Для начала, прикинем, как ведет себя функция на этом отрезке при изменении параметра a:



При x>a: f(x) = f1(x) = x^2-10x+7a
При x<a: f(x) = f2(x) = x^2+4x-7a
При x=a: f(x) = x^2-3x = a^2-3a

Функция может достигать максимального значения либо на границах отрезка, либо в точках максимума (если они есть), либо в особой точке (где выражение под модулем меняет свой знак, т.е. при x=a).

f1'(x) = (x^2-10x+7a)' = 2x-10. Экстремум в точке x=5, и это точка минимума (производная меняет знак с отрицательного на положительный).
f2'(x) = (x^2+4x-7a)' = 2x+4. Экстремум в точке x=-2, и это тоже точка минимума.
Так что максимумов у функции нет. Следовательно, наибольшего значения функция f(x) может достичь только либо на одной из границ отрезка [-6;6], либо в точке x=a.

Если a < -6 или a > 6, то функция всегда принимает максимальное значение на одной из границ отрезка, поскольку особая точка лежит вне его.

Если a принадлежит [-6;6], то условие выполняется, когда справедливо хотя бы одно из неравенств:

(1): f2(-6)>=f(a) (значение функции в левой границе отрезка больше ее значения в особой точке)
(2): f1(6)>=f(a) (значение функции в правой границе отрезка больше ее значения в особой точке)

(1): 36-24-7a >= a^2-3a
a^2+4a-12 <= 0
a принадлежит [-6;2]

(2): 36-60+7a >= a^2-3a
a^2-10a+24 <=0
a принадлежит [4;6]

Таким образом, функция f(x) принимает своё наибольшее значение на отрезке [-6;6] при всех значениях a от минус бесконечности до 2 включительно и от 4 включительно до плюс бесконечности.
a ∈ (–∞;2]∪[4;+∞)

2230. Леонид , 22 октября 2010, 15:21:52
С4 полное и правильное условие???

Lexxus, спасибо БОЛЬШОЕ!!!

да, прям так было написано....

2233. Леонид , 22 октября 2010, 15:41:38
№2215, я примерно знаю как решать но не все задания такого типа...
y=tgx-x+п/4-11

на отрезке [-п/4,п/4]

Как ты нашел производную у'? при чем тут Sin? если в задании tg?

ИГОРЬ!!! Извиняюсь, производная тангенса есть 1 : Cos^2 x, рассуждения теже. Сможешь или по новой написать. Там cos x = 1, cos x = -1.
х = 2pi k, x = pi + 2pik, в промежуток попадает х=0, при котором у=-10 с чем-то. ответ -10.
Поспешил-насмешил из-за печатания в разных алфавитах.

2234. Игорь, 22 октября 2010, 16:48:41
2233. Леонид , 22 октября 2010

Леонид, я переделал сам.Спасиб.

Lexxus, а вы можете решить С6 из поста 2217?

2235. Игорь, 22 октября 2010, 16:57:51
2226. Kattu, 22 октября.

Kattu, я не буду учить именно то что сдесь мы обсуждали, просто повторю данную тему с преподавателями...

(tg u)' = 1/ cos2u&#215; u' так ведь?

2236. Леонид , 22 октября 2010, 17:14:02
Молодец игорь, в перечне таких заданий всего 4 вида, отличаются остальные только числами.

Чтобы увидедь добавленные, зайдите mathege.ru щёлкнуть каталог по заданиям, откроется таблица В1-В12 откуда набираются задания на ЕГЭ. В любом из В щёлкните прототипы и листайте(стрелки там есть вверху справа) - это шаблоны всех заданий. От них другие отличаются только числами.

2237. Леонид , 22 октября 2010, 17:32:46
ИГОРЬ6 С6. Наибольшее целое число, не превосходящее число х, равно
х''(в квадрате)+6/7
Нужно найти все такие значения х.
Неравенство (x^2 +6)/7 меньше или равно х, в одну часть и умножив на 7, будем иметь x^2 - 7x +6 меньше или равно 0, (х-1)(х-6) меньше или равно 0. Методом интервалов х : 1,2,3,4,5,6. Проверяем каждое
х=1, (1^2 +6)/7 меньше или равно1.верное
х=2? (2^2 +6)/7 меньше или равно 2, 10/7 не целое , не идёт и т.д.
Верное будет ещё при х=6. Ответ: 1;6. Я так,как понял задание, рассуждал.

2238. Lexxus, 22 октября 2010, 18:15:38
Lexxus, а вы можете решить С6 из поста 2217?

С6. Наибольшее целое число, не превосходящее число х, равно
х''(в квадрате)+6/7
Нужно найти все такие значения х.

х(в квадрате)+6
(дробь)7

Насколько я понял, ты силился написать
(x^2+6)/7
(то есть икс в квадрате плюс шесть, и всё это поделить на семь).

Если так, то:



Значение функции f(x) = (x^2+6)/7 (линия красного цвета) не превышает значения x (линия синего цвета) при x, принадлежащем отрезку [1;6] (это можно установить, решив неравенство (x^2+6)/7 <= x).

Внутри этого отрезка функция f(x) принимает целые значения 1, 2, 3, 4, 5 и 6 (это видно по графику, но при решении лучше взять производную, убедиться, что на этом отрезке нет экстремумов и вычислить значения f(x) на границах полученного отрезка, т.е. f(1) и f(6) ).

Значит, нужные нам значения x находятся из уравнений
f(x) = 1, f(x) = 2, и т.д., f(x) = 6

Но еще нам нужно убедиться, что значение f(x) - именно ближайшее целое число <= x, т.е. между f(x) и x не затесалось больше ни одного целого числа. Для этого достаточно решить неравенство

f(x)+1 <= x

(f(x)+1 - линия зелёного цвета на графике)

Убеждаемся, что оно решений не имеет, т.е. значение x всегда меньше f(x)+1.

Поэтому нам подходят все корни уравнений
f(x) = 1, f(x) = 2, и т.д., f(x) = 6, принадлежащие отрезку [1;6], т.е., соответственно,

1, 2*sqrt(2), sqrt(15), sqrt(22), sqrt(29) и 6

(sqrt - это квадратный корень, если что :)

2239. Писька норушка, 22 октября 2010, 19:17:30
А разве линии не должны пересекаться

2240. Игорь, 22 октября 2010, 21:34:33
Леонид , Lexxus благодарю за помощь...я понял как решать.

Вот задание С6 только другой вариант...обьясните пжл.

С6. Перед каждым из чисел 2, 3, …, 6 и 10, 11, …, 20 произвольным
образом ставят знак плюс или минус, после чего к каждому из
образовавшихся чисел первого набора прибавляют каждое из
образовавшихся чисел второго набора, а затем все 55 полученных
результатов складывают. Какую наименьшую по модулю и какую
наибольшую сумму можно получить в итоге?

2241. Игорь, 22 октября 2010, 22:50:12
B10. Зависимость объeма спроса q (тыс. руб.) на продукцию предприятия-монополиста от цены p (тыс. руб.) задаeтся формулой q=190-10p. Выручка предприятия за месяц r (в тыс. руб.) вычисляется по формуле r(p)=q*p. Определите наибольшую цену p, при которой месячная выручка r(p) составит не менее 880 тыс. руб. Ответ приведите в тыс. руб.

Обьясните как решать пожалуста.

2242. Игорь, 22 октября 2010, 23:13:59
В12. Из пункта А в пункт В, расстояние между которыми 30 км, одновременно выехали автомобилист и велосипедист. Известно, что в час автомобилист проезжает на 100 км больше, чем велосипедист. Определите скорость велосипедиста, если известно, что он прибыл в пункт В на 1 час 15 минут позже автомобилиста. Ответ дайте в км/ч.

Обьясните пожалуста как решать такие задачи?

2243. ilo, 22 октября 2010, 23:48:54
Игорь, 22 октября 2010, 22:50:12
можно посмотреть посты 2108 и 2110, относительно первой задачи, ко второй обозначь параметры движения А и В
S 30-A 30-B
v 100+x-A x-B
t 30/100+x < 30/x на 5/4ч
лучше сделать в форме таблицы,время перевести в часы 15мин=1/4ч

2244. Игорь, 22 октября 2010, 23:59:23
2243. ilo, 22 октября 2010, 23:48:54

А по подробней можно?как рассуждать, что за чем делать?

2245. Леонид , 23 октября 2010, 07:56:10
Игорь, за х ск. велосипедиста, заметь - её надо найти, тогда ск. автомобилиста (х+100). уравнение 30/х - 30/(х+100) =5/4 (1ч.15мин). в одну часть, к общ. знамен...

2246. Игорь, 23 октября 2010, 08:31:20
Угу.

B10. Зависимость объeма спроса q (тыс. руб.) на продукцию предприятия-монополиста от цены p (тыс. руб.) задаeтся формулой q=190-10p. Выручка предприятия за месяц r (в тыс. руб.) вычисляется по формуле r(p)=q*p. Определите наибольшую цену p, при которой месячная выручка r(p) составит не менее 880 тыс. руб. Ответ приведите в тыс. руб.

Как сдесь быть?

2247. Леонид , 23 октября 2010, 09:27:14
в формулу r(p): (190 - 10р)р больше или равно 880, -10р^2 +190p -880 больше или равно 0,
р^2 - 19p + 88 меньше или равно 0. корни 8 и 11, для неравенства [8;11]- цена, 11- наибольшая.

Игорь, если не секрет, где проживаешь? Откуда задания берёшь?

2248. Игорь, 23 октября 2010, 10:30:07
2247. Леонид , 23 октября 2010

Заданий у меня много, беру из зборников свежих, с сайтов таких как этот ну и преподы нагружают.В общем к ЕГЭ готовлюсь.
Я из России, Ставропольского края г.Буденновск

А ты где живешь?

2249. Леонид , 23 октября 2010, 10:40:16
Далеко. В Кузбассе. В Краснодрском на море бывал, в Ставрополье немного.
Игорь, здесь задания все те, из которых набираются КИМы ЕГЭ. Этого вполне достаточно для подготвки решения заданий В. С - надо ловить из переписки здесь же с ребятами других регионов России.

Молодец- готовишься автивно. Толк будет.

2250. Леонид , 23 октября 2010, 10:42:26
Да, забыл посоветовать. Нарешивай по темам. Набей руку на В1, потом В2 и т.д.

2251. Игорь, 23 октября 2010, 13:42:59
Леонид, спасибо за советы...буду учиться.вот мы в школе логарифмические уравнения еще не проходили а их полно в заданиях.что делать?

И кстати это отличный сайт:)

2252. Леонид , 23 октября 2010, 15:58:06
Логарифмы изучают взависимости от того, по какому учебнику учитесь. Но к январю должны всё пройти, затем повторять. Тема обширная, задания разнообразные есть в В3, 7, 11, !1 и С 3. Построена тема на степенях - знать надо хорошо. В первую очередь усвоить определение логарифма, действия логарифмирования. Начнёте изучать - уши торчком, внимательно, осмысленно. Как со степенями? Если хорошо, с логарифмами проблем особых не будет. Производную изучили? Нажми В8 - графичкские - внимание!!!

2253. Игорь, 23 октября 2010, 18:56:31
Буду надеяться что вскоре мы пройдем эту тему...
Со степенями вроде нормально...разбираюсь, но тоже надо дорабатывать.
Производную учили еще в 10 классе, но также есть пробелы.

Вот именно с В8 у меня трудности.Не знаешь что поискать в интернете чтобы понять хорошо тему?или ты обьясни:)

2254. Леонид , 23 октября 2010, 19:04:40
Попробую в ходе общения.

2255. Игорь, 24 октября 2010, 11:17:04
В9. Объем первого цилиндра равен 12 м&#179;. У второго цилиндра высота в три раза
больше, а радиус основания – в два раза меньше, чем у первого. Найдите
объем второго цилиндра. Ответ дайте в кубических метрах.

Леонид, как решить?

2256. Леонид , 24 октября 2010, 12:15:16
объём первого pi R^2 H. выходим на второй цилиндр pi (R/2)^2 (3H) =
= (pi R^2 H) *3/4 в скобках замени объём первого цилиндра и умнохай на 3/4.
Я не понял в условии 12m&#179/

Принцып решения большинства задач в В9 - записывай формулу данного, делай в ней изменения по условию (здесь радиуса и высоты), выделяй исходную формулу (у меня она в скобках), подставляй и считай. Думаю, Игорёк, всё понял здесь?

2257. Леонид , 24 октября 2010, 12:21:53
Игорь, зайди на mathege.ru Там увидешь "тренировочные работы" - заходи, будет польза.

2258. Игорь, 24 октября 2010, 14:47:05
2256. Леонид , 24 октября 2010, 12:15:16
объём первого pi R^2 H. выходим на второй цилиндр pi (R/2)^2 (3H) =
= (pi R^2 H) *3/4 в скобках замени объём первого цилиндра и умнохай на 3/4.
Я не понял в условии 12m&#179/

в общем то понял.В условии 12м(в кубе) не отобразилось.

Да я заходил уже...Там только задания, но не обьясняется как их решать(((

2259. Игорь, 24 октября 2010, 14:51:34
В9. Прямоугольный параллелепипед описан около цилиндра, радиус основания и высота которого равны 1. Найдите объем параллелепипеда.

Леонид ,вот задача с mathege.ru ...Какая сдесь требуется формула для нахождения обьёма?

2260. Леонид , 24 октября 2010, 15:11:51
описан около цилиндра - диаметр основания цилиндра и сторона онованая параллелепипеда 2. объём парал-да - площадь основания на высоту : 2*2*1=4.

От туда именно включают задания на ЕГЭ

2261. Игорь, 24 октября 2010, 15:42:27
В9. В основании прямой призмы лежит квадрат со стороной 3.
Боковые ребра равны 6/п . Найдите объем цилиндра, описанного около этой призмы.

Сейчас прохожу один из 2х вариантов, как решить?

2262. Игорь, 24 октября 2010, 16:15:35
Леонид, Сейчас прошел 2-ой вариант набрал 7 баллов, не решил В7 В8 В9 В11 В12.

В9. В основании прямой призмы лежит квадрат со стороной 3.
Боковые ребра равны 6/п . Найдите объем цилиндра, описанного около этой призмы.

В12. Два велосипедиста одновременно отправились в 56-километровый пробег. Первый ехал со скоростью, на 1 км/ч большей, чем скорость второго, и прибыл к финишу на 1 час раньше второго. Найти скорость велосипедиста, пришедшего к финишу первым. Ответ дайте в км/ч.

Обьясни пожалуста. :)

2263. Леонид , 24 октября 2010, 19:26:15
2261. Игорь. в основании квадрат вписан в круг - по т. Пифагора находи диаметр - диагональ квадрата: 3 корня из 2,то радиус 1,5 корня из 2. Объём цилиндра V= pi R^2 H = pi (1,5 Корней из 2)^2 * 6/pi = 2,25*2*6=27 (проверь внимательнее).

В12. первый ехал с большей скоростью- значит пришёл первым к финишу, пусть его ск. х, тогда скорость 2-го (х-1).

уравнение: 56/(х-1) - 56/х = 1. к общ. знам. 56х - 56х +56 = х в кв. - х и.т.д. Ещё раз, во избежании ошибки в записи ответа, за х бери то, что просят найти.

что было В7.8,11?

2264. Леонид , 24 октября 2010, 22:28:05
http://live.mephist.ru/show/mathege2010/view/B8. Сюда, ИГОРЬ, заходишь? Там, смотри по меткам, есть решённые, можно тренироваться,где ответы сразу выдают.

2265. Игорь, 24 октября 2010, 22:32:06
2263. Леонид , 24 октября 2010, 19:26:15

В7 было логарифмическое уравнение.
В8 был график зависимости расстояния от времени движения авто, и надо было найти среднюю скорость...я не знаю как.
В11 было логарифмическое уравнение. но труднее того что в В7

2266. Леонид , 24 октября 2010, 23:56:27
С логарифмами нтебе ещё рано - не изучали. В8 - такого содержания не встречал - там всё на производную. Или что свежее появилось? Полистаю В8.

2267. Игорь, 25 октября 2010, 07:50:39
2266. Леонид , 24 октября 2010, 23:56:27

Да там был вариант без логарифмов и без производной, вот я решал тот который без производной, там В8 новое.

2264. Леонид , 24 октября 2010, 22:28:05
http://live.mephist.ru/show/mathege2010/view/B8. Сюда, ИГОРЬ, заходишь? Там, смотри по меткам, есть решённые, можно тренироваться,где ответы сразу выдают.-------------Я вечером проверю :)

2268. Леонид , 25 октября 2010, 09:26:24
В8 - видимо, просто так вписано, в банке заданий таких нет. Несколько настораживает, а может быть и не надо заморачиваться. За 2 часо проехал 100 км, следовательно скорость 100:2= 50 км.ч.

2269. Игорь, 25 октября 2010, 14:54:01
2268. Леонид , 25 октября

Я В8 в ответе ввел 50, этот результат и оказался правильным ответом.

http://live.mephist.ru/show/mathege2010/view/B8. Сюда, ИГОРЬ, заходишь?---------------------сюда заходил, ответы конечно есть, но я не знаю как их получить, незнаю как решать.Леонид, научи пжл.

уравнение: 56/(х-1) - 56/х = 1. к общ. знам. 56х - 56х +56 = х в кв. - х и.т.д. Ещё раз, во избежании ошибки в записи ответа, за х бери то, что просят найти.-----------------тут получается что? х^2-x+1=0

2270. маделин, 25 октября 2010, 15:32:20
суппер сайт))))

2271. Игорь, 25 октября 2010, 15:54:34
А тооооо :)

2272. Леонид , 25 октября 2010, 16:02:43
Когда зайдёшь, читай внимательно в верху ссылки. Нажми "Решение заданий" откроется, Нажми "Тип заданий - это В1-В12. Нажми решённые - будут появлятся задания. Реши, ответ впиши в окошечко и жди - появляется быстро. и т.д. Пробуй. Если нажмёшь "нерешённы" - ответов не будет.Заготовь бумагу и ручку.

Ты куда думаешь поступать? Как математик твой? Как класс?

Игорь, Псебай, Момтовое далеко от Вас?

2273. Леонид , 25 октября 2010, 16:04:41
Мостовое

2274. Леонид , 25 октября 2010, 16:55:04
ИГОРЮ: получается что х^2-x+1=0 - разве?: x^2 - x - 56=0? корни 8 и -7, ответ 8.

2275. Игорь, 25 октября 2010, 16:55:47
2272. Леонид , 25 октября 2010, 16:02:43

Поступать думаю в СГУ (Ставропольский государственый университет) или в СГАУ (Ставропольский государственный аграрный университет)...Таких поселков(городов) не знаю-первый раз слышу. А что?

уравнение: 56/(х-1) - 56/х = 1. к общ. знам. 56х - 56х +56 = х в кв. - х и.т.д. Ещё раз, во избежании ошибки в записи ответа, за х бери то, что просят найти.-----------------тут получается что? х^2-x+1=0 ??????????????

2276. Игорь, 25 октября 2010, 16:58:11
x^2 - x - 56=0? корни 8 и -7, ответ 8.
Все понял:)

2277. Игорь, 25 октября 2010, 17:01:03
Леонид, блин не пойму где сдесь решение: http://live.mephist.ru/show/mathege2010/view/B8/solved/ ???????????????????

2278. Игорь, 25 октября 2010, 17:45:11
B10. Камень брошен вертикально вверх. Пока камень не упал, высота, на которой
он находится, описывается формулой h (t) = &#8722;5t^2 +18t (h – высота в метрах,
t – время в секундах, прошедшее с момента броска). Найдите, сколько
секунд камень находился на высоте не менее 9 метров.

Леонид, обьясни как решать пжл.

2279. СветланаЯ, 25 октября 2010, 17:59:20
ПОМОГИТЕ ПЛИЗ СРОЧНО! ЗАВТРА СДАТЬ НЕОБХОДИМО!
в правильной треугольной пирамиде АВСS с основанием АВС известны ребра: АВ= 5 корней из 3, SC= 13. найти угол, образованный плоскостью основания и прямой, проходящей через середину ребер АS и ВС.!!!!!!!!!!!
заранее огромное спасибо!

2280. Светлана, 25 октября 2010, 18:00:07
Это задание С4 2011 года!

2281. Lexxus, 25 октября 2010, 18:46:28
в правильной треугольной пирамиде АВСS с основанием АВС известны ребра: АВ= 5 корней из 3, SC= 13. найти угол, образованный плоскостью основания и прямой, проходящей через середину ребер АS и ВС.

Это задание С4 2011 года!

Лень мне чертеж рисовать-загружать, так что на пальцах.

1. Поскольку SABC - правильная пирамида, то ABC - равносторонний треугольник, а остальные грани - равные между собой равнобедренные треугольники.
То есть все стороны основания равны 5*sqrt(3), а все боковые ребра равны 13.

2. Пусть D - середина BC, E - середина AS, SH - высота, опущенная из точки S к основанию пирамиды, EP - высота, опущенная из точки E к основанию пирамиды.

3. Найдем AD из прямоугольного треугольника CAD по теореме Пифагора. Получится 15/2 = 7.5.

4. Поскольку пирамида правильная, точка H - это точка пересечения высот/медиан/биссектрис треугольника ABC, а значит, делит AD в отношении 2:1 (AH=2*AD).

5. Найдем SH из прямоугольного треугольника ASH. AH=AD*2/3 = 5, AS = 13, по теореме Пифагора SH = sqrt(13^2-5^2) = 12.

6. Треугольники AEP и ASH оба прямоугольные и имеют общий угол A, следовательно, подобные. По условию, AE = AS/2, значит, и AP = AH/2, и EP = SH/2.

7. Осталось рассмотреть прямоугольный треугольник EDP (нас как раз интересует угол EDP).
EP = SH/2 = 6;
DP = AD*2/3 = 5;

Тангенс угла EDP = EP/DP = 6/5,
Угол EDP = arctg(6/5)

2282. Леонид , 25 октября 2010, 21:00:19
2278. Игорь, 25 октября 2010, И ещё- пропечатывай словами, без заморочек. Хорошо?

Неравенство: 5t^2 + 18t больше или равно 9 (сказано не менее 9). переноси 9, решай квадратное н-во (их надо научиться хорошо решать, - они очень часто внутри других заданий встречаются). В официальном открытом банке этой задачи нет - здесь корни не извлекаются. В похожих:найди корни, оба должны быть положительными, от большего отними меньший - это ответ.

что то не понял, пиши.

2283. Леонид , 25 октября 2010, 21:03:25
В четвёртой строке, слева "Решение заданий"

2284. эндже, 25 октября 2010, 21:05:47
на дне сосуда,имеющего форму куба со стороной 30см лежит деталь,остальной объем сосуда заполнен водой.Когда деталь вынули,уровень воды понизился на 10см.Найдите объем детали в кубических сантиметрах.
ПОЖАЛУЙСТА ПОМОГИТЕ!!!!!

2285. Леонид , 25 октября 2010, 21:10:43
Это в 100 км от Армавира в Краснодарском - родственники живут, почти на границе с Ставропольским краем - Смотрел брошенный у вас симпатичны городок Медногорск.

2286. Леонид , 25 октября 2010, 21:13:52
Открытый банк задач ЕГЭ по математике 2011
Более 35000 реальных задач ЕГЭ 2011 года
Тесты ЕГЭ Онлайн • Вопросы, просьбы, предложения

--------------------------------------------------------------------------------

Решение заданий • Просмотр заданий • Поиск заданий • Коллективный разум • Прогресс

--------------------------------------------------------------------------------

Привет, solo4848
Тип заданий: Все • B1 • B2 • B3 • B4 • B5 • B6 • B7 • B8 • B9 • B10 • B11 • B12
Решённость: Все • совсем без ответов • нерешённые • решённые

--------------------------------------------------------------------------------

001 • 002 • 003 • 004 • 005 • 006 • » • 150

--------------------------------------------------------------------------------
Задание B8 (6404)

(показов: 1886, ответов: 953)

2287. ilo, 25 октября 2010, 23:18:40
Был объём воды 30*30*30=27000; стал 30*30*20=18000 Объём, который занимала деталь 27000-18000=9000 куб см

2288. ilo, 25 октября 2010, 23:22:00
эндже, 25 октября 2010, 21:05:47
проще 30*30*10=9000

2289. Игорь, 25 октября 2010, 23:30:06
2286. Леонид , 25 октября.

Насчет городов не знаю...не слышал.
Дело в том что неравенство: -5t^2 + 18t Так я пишу нормально,все отображается,а когда уже нажимаю "у меня все" оказывается муть...


значит....-5t^2 + 18t>=9
-5t^2 + 18t-9>=0
D=144. sqrt=12
x1=0.6 x2=3
t=3-0.6
t=2.4
Ответ: 2.4с

Леонид, ты это имел ввиду?таково решение и ответ.или я что-то не понял?

2290. Леонид , 26 октября 2010, 07:57:18
Да, всё верно, МОЛОДЦОМ,
но в первой записи у тебя не было "минуса" перед 5. Почему там, как ты говоришь, муть , здесь пропечатано всё чётко. Пробовал ещё выйти на "Решение заданий", "Тип заданий", "Решённые"?
Как ты печатаешь знаки меньше и больше? Научи.

2291. Игорь, 26 октября 2010, 14:58:07
2290. Леонид , 26 октября 2010, 07:57:18

Я тогда скопировал и сдесь вставил, отобразилось сразу нормально но после загрузки сервером получилась муть.

Сейчас попробую выйти.

Знаки легко, если у тебя комп то переходи на английский и печатай их нажав на шифт и знак одновременно ну или шифт держи и нажимай на знак, они находятся в правом нижнем углу клавы, где Alt Ctrl.у меня они на месте русских б и ю
<<<< = >>>> :)

2292. Игорь, 26 октября 2010, 15:15:44
Lexxus, сдесь на этом сайте есть подробное решение заданий В1-В12, или только их просмотр и ответы?если есть то обьясните где.

2293. Леонид , 26 октября 2010, 15:59:44
<>< Спасибо - это пробы получились. Решений В я, на пример, не встречал.

2294. Lexxus, 26 октября 2010, 16:46:04
Lexxus, сдесь на этом сайте есть подробное решение заданий В1-В12, или только их просмотр и ответы?если есть то обьясните где.

Кое-какие задачи достаточно подробно разбирались прямо в этой ленте комментариев.

Вот это ещё полезно будет почитать.

2295. Игорь, 26 октября 2010, 16:59:29
2293. Леонид , 26 октября 2010
В11. Найдите наибольшее значение функции.

у=2cosx+sqrt3x-sqrt3п(дробь)3. на отрезке (0;п/2)

2296. Игорь, 26 октября 2010, 17:33:33
Lexxus, спасибо за полезную инфу.Конечно не плохо бы было если бы там был разбор каждого задания В1-В12 т.е. идет описание(какие ошибки,что надо знать) далее самая распространеная задачка-и ее подробное решение.Эту ленту комментариев обязательно проверю:)

2297. Леонид , 26 октября 2010, 18:50:56
у =2cos x + (корень из 3)*x - (корень из 3)pi /3 на ( ; ) или на [ ; ] ???- это очень важно-какие скобки!!!

1. находи производну у штоих = -2sin x + корень из 3, прирвнивай её 0. Выбирай х входящий в указанный прмежуток - это pi/3. Находи значение ф-ии в этой точке и на концах промежутка. Выбирай наибольшее из них. Как сумел по твоему условию. Игорь к условию - пристальное внимание.

2298. Игорь, 26 октября 2010, 20:54:59
Леонид, скобки квадратные (включая) [0;п/2]
К условию...присмотрюсь.

попробую....
у=2cosx+sqrt3x-sqrt3п(дробь)3
у'=-2sinx+sqrt3=0
y=-2sin*п/3+sqrt3+2Пк
y=-2*sqrt3/2+sqrt3+2Пк

Фигня какая-то)))))))))))))
Дальше не знаю.обьясни плиз, понятней когда ты решаешь полностью, с обьяснением :)


2299. Леонид , 27 октября 2010, 07:27:28
-2sinx=-sgrt3, sinx=sgrt3/2, x=pi/3 + 2pik, k=0 x=pi/3 входит в [0;pi/2].
Это легко делается на единичной окружности (черти). V3/2 (V-корень, короче пчатать) отмечаем на оси ОУ по определению синуса, проводим прямую параллельно ОХ до пересечения с окружностью - в промежуток попадает только pi/3. Это, пожалуй главное в таких заданиях - приём.
1)Находим у(0)= (не в призводную подставляем, а в функцию) = 2cos 0 +V3*0 - V3pi/3 = 2 -V3pi/3 - не вычисляется для бланка ответа- не теряем время.
2) y(pi/3) = 2cos pi/3 +V3*pi/3 - V3pi/3 = 2*1/2=1.
3) y(pi/2)=2cos pi/2 +V3*pi/2- V3pi/3=0+... то же "плохой" ответ - не для банка. Ответ 1. Если просчичат с калькулятором,так и будет. Разбираясь, смотри внимателно, могут быть опечатки-прыгать приходится и не всё как надо печатается.

2300. Игорь, 27 октября 2010, 07:48:50
Буду разбираться.....

2301. Игорь, 27 октября 2010, 08:08:32
2299. Леонид , 27 октября 2010, 07:27:28

В9. Прямоугольный паралелепипед описан около цилиндра.Радиус основания цилиндра равен 3.Обьем пералелепипеда равен 72.найти высоту цилиндра


2302. Леонид , 27 октября 2010, 09:51:58
Прямоугольный паралелепипед описан около цилиндра-сторона основания равна диамету, т.е. 6. Из объёма пар-да 6*6*H=72, H=2-это и есть высота цилиндра. А ты, что не на уроках, судя по времени?

2303. Kattu, 27 октября 2010, 15:12:50
Уважаемый Lexxus! Я нашла ошибки ответах в номерах 6999, 24429! Исправьте из пожалуйста! Спасибо!

2304. Игорь, 27 октября 2010, 15:47:39
Леонид, у нас уроки начинаются в 8.30 я успеваю с утра сдесь побывать и на уроки двигаю.идти 10-13 мин. :)

В9. Обьем цилиндра равен 1 см^3.Радиус основания уменьшили в 2 раза а высоту увеличили в 3 раза.Найдите обьем получившегося цилиндра.Ответ дайте в см^3.

2305. Леонид , 27 октября 2010, 16:53:17
2258. Игорь, 24 октября 2010, 14:47:05
2256. Леонид , 24 октября 2010, 12:15:16
объём первого pi R^2 H=1. Выходим на второй цилиндр pi (R/2)^2 (3H) =
= (pi R^2 H) *3/4 в скобках замени объём первого цилиндра и умнохай на 3/4.


Показывал тебе, как работать с такими задачами. 1*3/4 = 0,75

2306. Игорь, 27 октября 2010, 17:00:31
2305. Леонид , 27 октября 2010, 16:53:17
Спасибо :)


Леонид, В9. Площадь осевого сечения цилиндра равна 29. Найдите площадь боковой поверхности цилиндра, деленную на п.

2307. Игорь, 27 октября 2010, 17:32:17
В4. В треугольнике АВС угол С равен 90 градусов, угол А равен 30 градусов, АВ=sqrt3. Найдите АС.

Леонид, я знаю как решать такие задание но именно с вот этим возникли трудности...тут кажется есть какая-то теорема или формула связанная с углом 30 градусов в прямоугольном треугольнике.Помого пжл.

2308. Игорь, 27 октября 2010, 17:32:27
В4. В треугольнике АВС угол С равен 90 градусов, угол А равен 30 градусов, АВ=sqrt3. Найдите АС.

Леонид, я знаю как решать такие задание но именно с вот этим возникли трудности...тут кажется есть какая-то теорема или формула связанная с углом 30 градусов в прямоугольном треугольнике.Помоги пжл.

2309. Леонид , 27 октября 2010, 17:48:09
2307. Игорь, 27 октября 2010, 17:32:17
В4. В треугольнике АВС угол С равен 90 градусов, угол А равен 30 градусов, АВ=sqrt3. Найдите АС.

Можно подойти по-разному. По определению кос А = АС:АВ,
(корень из 3)/2 = (корень из 3)/АВ, АВ=2(корень из 3)/(корень из 3) = 2

Цилиндр: S сеч. = 2RH = 29. Бок. пов.ц. =2piRH = 29pi , делим на pi = 29.

Игорь, ты по какому принципу подбираешь задачи. Учись по темам, иначе каша будет в голове.

2310. Игорь, 27 октября 2010, 17:58:07
Леонид, я решаю сборник типовых вариантов заданий 2010 года под редакцией Семенова и Ященко.Беру вариант и решаю весь кроме С.Ну или В1-В12 сразу в 10ти вариантах прохожу.что не знаю прошу тебя помочь :)А что?

2311. Игорь, 27 октября 2010, 18:03:37
2309. Леонид , 27 октября 2010, 17:48:09
В4. В треугольнике АВС угол С равен 90 градусов, угол А равен 30 градусов, АВ=sqrt3. Найдите АС.

Можно подойти по-разному. По определению кос А = АС:АВ,
(корень из 3)/2 = (корень из 3)/АВ, АВ=2(корень из 3)/(корень из 3) = 2

Леонид, в сборнике ответом на это задание В4 является 1.5
т.е ответ не 2 а 1.5. ????????????????????7

2312. Леонид , 27 октября 2010, 18:37:52
Что- то я перевернул, записывая сходу. По другому: катет ВС против угла 30 равен половине гипотинузы, тюе. (корень из 3)/2. Далее по т. Пифагора
(корень из 3)^2 - ((корень из 3)/2)^2 = 3 - 3/4 = 12/4 - 3/4 = 9/4. Извлекаем корень 3/2=1,5. Проверяй всегда, я печатую не записывая предвариельно на бумаге и приходится переключаться на алфавиты.

Да это автор ЕГЭ. Но советую, прорешай сначала все В1, потом В2...

Книга беднее - открытый банк лучше. Зайди, как я тебе говорил и шпарь

2313. Игорь, 27 октября 2010, 18:56:45
Леонид, катет ВС против угла 30 равен половине гипотинузы, вот что я забыл.Адальше зная ВС уже по теореме Пифагора :)

У меня есть еще одна книжка, она по объемнее под редакцией Лысенко и Кулабухова, это нормалек?называется "подготовка к ЕГЭ 2011" допущено ФИПИ.

Вот банк то 2010 года, а я в 2011 сдавать буду, или это не играет роли?задание будут такие же?

2314. Леонид , 27 октября 2010, 19:09:52
не могу сказать,книги того автора я невидел. Банк 2011 сохранился, добавлены задания (смысл их тотже, кроме векоров в В6), вряд ли добавленные успели попасть в книжки.

2315. Игорь, 27 октября 2010, 19:21:28
В треугольнике АВС угол С равен 90 градусов, угол А равен 30 градусов. Найдите синус угла ВАD.

Вот чтобы найти ВАD возьмем развернутый угол DAC=180
BAD=DAC-BAC;
BAD=180-30=150
sin BAD=Sin 150
Sin150 распишем как Sin90+sin30+sin30=1+1/2+1/2=2
У меня получилось 2 а в ответе 0.5

Леонид, какова моя ошибка?

2316. Игорь, 27 октября 2010, 19:32:12
Леонид, а банк 2011 находитсе сдесь http://www.mathege.ru/

Или сдесь http://live.mephist.ru/show/mathege2010/view

2317. Леонид , 27 октября 2010, 19:36:05
Sin150 распишем как Sin90+sin30+sin30 - прости, безобразие -=1+1/2+1/2=2
У меня получилось 2 а в ответе 0.5
Игорь между синусом и скобкой нет знака умножения,- раскрывать скобку сдесь ни в коем случае нельзя: туловище - шея - голова. Здесь к формуле приведения sin150 = sin(90 + 60) = cos 60 = 1/2 = 0,5/

2318. Игорь, 27 октября 2010, 19:53:20
Леонид, sin150=(90+60) это понятно, но почему потом =cos60???
это формула приведения?

2319. Игорь, 27 октября 2010, 19:55:53
http://www.legionr.ru/

Леонид, вот сайт разработчиков той книги, зайди посмотри.

2320. Леонид , 27 октября 2010, 20:34:02
Да Игорь - формула приведения. После равно ты упустил sin.

Правило: 1) если выделяем 90 или 270 функцию меняем на кофункцию,
если выделяем 180 или 360 функцию не меняем;
2) знак приведённой функции (в конце) определяем по знаку приводимой (в самом начале) в зависиости от того, в какой четверти оканчивается её угол.

В учебнике нет такого правила - рекомендую на все случаи формул приведения.

2321. Леонид , 27 октября 2010, 20:41:38
посмотрел на ту книжку - пойдёт для тренировки на маленьькую троечку.

2322. Игорь, 27 октября 2010, 20:55:24
А правил только 2?

Наш учитель сказала прикупить такую книжку,может ты не ту глянул, по мне так нормальная, есть краткий теоретический справочник, и варианты заданий ЕГЭ 2011, и еще куча задач чтобы "набивать руку" :)
Может что посоветуешь из похожей литературы?

2323. Игорь, 27 октября 2010, 21:03:03
C1. Решите систему уравнений
{2^x=siny
{2^-x=2siny+1

Леонид, помоги пжл. :)

2324. Леонид , 27 октября 2010, 22:01:15
Это С1. первое оставим, во втором 2siny = 2^-x - 1. siny= 2^-x/2 - 1/2,
Уравнение новое: 2^x = 1/2* (1/2^x) - 1/2, Пусть 2^x = a, a = (1/2)* 1/a -1/2 умножим обе части на 2а: 2a^2=1 - a, 2a^2 +a-1=0. a=1/2 и a= -1-не походит,т.к. 2^x>0.

^x=1/2; x=-1. siny=1/2. y=(-1)^k pi/6+2pik. Прописывая, проверяй -муторно печатать, понимаеш?

2325. Игорь, 27 октября 2010, 22:16:07
Леонид, понимаю, спасиб.

Леонид, а ты учитель?или где только учишься?

2326. Леонид , 27 октября 2010, 22:32:22
Разобрался с решением,Игорь?

2327. Игорь, 28 октября 2010, 07:49:38
Не совсем, почему 1 уравнение оставляем?

2328. Игорь, 28 октября 2010, 08:05:31
В11. найдите наибольшее значения функции.
у=9х-8sinx+7 на отрезке [-п/2;0]
у'=9-8cosx
9-8cosx=0
Леонид, а что дальше?

2329. Леонид , 28 октября 2010, 11:13:26
2327. Игорь, 28 октября 2010, 07:49:38
Не совсем, почему 1 уравнение оставляем?
Идея в том, что во 2-м уравнении выразили синус, на основании этого (в 1-м ур. видно, чему равен синус) - синусы и приравниваем, получаем показательное уравнение, в котором степень с отрицательным показателем записываем с показателем положительным...

у=9х-8sinx+7 на отрезке [-п/2;0]
Производная y* = 9-8cosx - всюду положительная, значит ф-я возрастающая, её наибольшее зн-е вправом конце промежутка. Находим y(0)=7. Что не понял пиши.

2330. Леонид , 28 октября 2010, 13:31:54
2328. Игорь. 9-8cosx=0
Леонид, а что дальше? Делал верно, можно продолжать по общему алгоритму:
Уравнение 8cosx=9, cosx = 9/8 - решений нет, т.к. -1<=cosx<=1 (знаешь?). Это значит на указанном промежутке производная не принимает 0!!! Наибольшее значение функции следует искать на концах промежутка!!!! Находим значения данной функции на концаз промежутка:
у(-pi/2)= 9(-pi/2) - 8sin(-pi/2) +7 = -9pi/2 +8 +7= -9pi/2 +15

y(0)= 9*0 - 8sin0 + 7 = 7 - это больше, чем -9pi/2 + 15/


2331. мак, 28 октября 2010, 15:56:28
tg x - 3/корень из -sin x = 0
пожалуйста

2332. Игорь, 28 октября 2010, 17:44:37
Леонид, для чего сдесь надо умножить на 2а?
умножим обе части на 2а: 2a^2=1 - a, 2a^2 +a-1=0. a=1/2 и a= -1-не походит,т.к. 2^x>0.


решений нет, т.к. -1<=cosx<=1 (знаешь?).----------это правило?

2333. Леонид , 28 октября 2010, 18:15:13
2332. Игорь, 28 октября 2010, 17:44:37
Леонид, для чего сдесь надо умножить на 2а?
умножим обе части на 2а: 2a^2=1 - a, 2a^2 +a-1=0. a=1/2 и a= -1-не походит,т.к. 2^x>0.

Здесь уже умноженное. Смотри раньше6есть 2 и а в замнателях,- умножаем, чтобы избавиться от дробей - роиём решения.

решений нет, т.к. -1<=cosx<=1 (знаешь?).----------это правило?

Синус и косинус не могут быть меньше -1 и больше 1 - на единичной окружности из определения и отношение катета к гипотенузе, а гипотенуза больше катета, поэтому меньше или равно 1. Голосом и рисунком пояснить быстрее. Здесь приходится много печ.

2334. Игорь, 28 октября 2010, 18:15:36
В12. Каждый из двух рабочих одинаковой квалификации может выполнить заказ за 15ч.Через 5ч после того, как один из них приступил к выполнению заказа, к нему присоелинился второй рабочий, и работу над заказом они долвели до конца уже вместе. За сколько часов был выполнен весь заказ?

Леонид, обьясни как решать пжл. :)

2335. Леонид , 28 октября 2010, 19:27:27
1/15 производительность каждого рабочего, производительность вместе 1/15+1/15=2/15.
5/15 = 1/3 всей работы выполнил первый рабочий, когда работал один- осталос 2/3 работы для совместной. Если они работали вместе х часов, то остальную работу выполнили 2/15 х = 2/3, то х = 2/3 : 2/15, х = 5 ч. Да один работал 5 ч., всего времени 5+5 =10 - ответ.

как дела на уроках математики? Учительница гоняет? В каком возрасте? Как класс по математике?

2336. Игорь, 28 октября 2010, 19:51:30
Да дела не очень, у меня трояк :( Учитель 6.5 из 10 баллов, это я его так оцениваю...ему 58 лет вроде.и не гоняет он ни кого, ему по барабану...домашней нет 2, и родителей зови. :) Класс у нас 22 чела из них 2 четверки 12 троек и остальные дятлы еще те...)))Во как......

2337. Игорь, 29 октября 2010, 09:08:46
В11.Найдите наименьшее значение функции.
у=7sinx-8x+9 на отрезке [-3п/2;0]
y'=7cosx-8
7cosx-8=0
7cosx=8
cosx=8/7 решений нет так как -1<=cosx<=1
Производная не принимает 0 на указанном промежутке.

у(-3п/2)=7sin(-3п/2)-8(-3п/2)+9=-7+8 3п/2+9=8 3п/2+2=4*3п+2

у(0)=7уsin0-8*0+9=9

4*3п+2>9

Ответ: 9

Леонид, проверь я правильно сделал?заранее спасибо!

2338. Леонид , 29 октября 2010, 13:23:22
Игорь. Извини, от эмоций: МОЛОТОК!!! в смысле молодец. Всё верно. Нетренируйся на В11, алгоритм решения один для всех, но есть и разница...

2339. Леонид , 29 октября 2010, 13:35:25
2331. мак, 28 октября 2010, 15:56:28
tg x - 3/корень из -sin x = 0
пожалуйста
РЕШИЛИ?

2340. Игорь, 29 октября 2010, 18:05:49
Леонид, а в чем разница?в каких случаях?

2341. Игорь, 29 октября 2010, 18:20:27
Леонид, я вот решал задачу В6 варианта 8 и мой ответ не сошелся с тем который должен быть.

В6. Найдите площадь S круга. В ответе укажите S/п. Размер каждой клетки 1см на 1см.Ответ дать в квадратных сантиметрах.

Я нашел радиус. R=3
S= пR^2
S=п3^2
S=п9
В ответе укажите S/п
п9/п=9 п сокращаем. значит площадь круга равна 9.
А в книжном ответе 8
Леонид, радиус круга который взят на рисунке приблизительно 3 но точное значение наверно 2.8 и в квадрате 7.84 но все равно 8 никак не получается.
Что скажешь по данному примеру?

2342. Леонид , 29 октября 2010, 18:40:23
2340. Игорь, 29 октября 2010, 18:05:49
Леонид, а в чем разница?в каких случаях? В выборе точек, принадлежащих промежутку. Можно коротко, а можно долго перебором К, решая триг. ур-я, приравняв производную ). Есть задания на нахождение точки минимума или максимума. Попадутся, разберёмся.

В записи № 2238 читай натренеруйся, а то я шарахнул НЕТРЕНЕРУЙСЯ.

Твоё В6 рисунок там есть? Там не указана заштрихованной часть круга? В общем мне надо видеть.

2343. Леонид , 29 октября 2010, 18:48:10
Забыл, ход решения верный, но верно ли радиус найден?

2344. Игорь, 29 октября 2010, 19:38:42
Леонид, ну поставь точку на бумаге в клеточку, отмерь радиус 2.8 и получится тоже самое что и дано в книжке.И ничего не заштриховано.

Вот Lexxus графики чертит,он бы мог изобразить данный круг с радиусом 2.8

Это задание.
В6. Найдите площадь S круга. В ответе укажите S/п. Размер каждой клетки 1см на 1см.Ответ дать в квадратных сантиметрах.

2345. Игорь, 29 октября 2010, 19:46:24
Леонид, может быть у тебя возможно есть такая книжка.
ЕГЭ 2010 математика под редакцией Семенова и Ященко. 8 вариант, задание В6

2346. Игорь, 29 октября 2010, 19:47:04
Леонид, может быть у тебя есть такая книжка?
ЕГЭ 2010 математика под редакцией Семенова и Ященко. 8 вариант, задание В6

2347. Леонид , 29 октября 2010, 20:04:58
Игорь, помочь не могу, книжек не имею, пользуюсь только открытым банком задач. На рисунке чистый кржок? Что-то не могу ни понять в чём дело, ни подогнать.
как ты утверждаешь - чистый кружок - тогда опечатка в ответах книги.

2348. Игорь, 29 октября 2010, 20:38:43
В10. Электрическая цепь напряжением 220В защищена предохранителем, рассчитанным на силу тока 8А.Найдите наименьшее сопротивление, которое может быть у электроприбора, включенного в эту цепь, чтобы предохранитель продолжал работать.Сила тока в цепи I вязана с напряжением U соотношением I=U/R, где R-сопротивление электроприбора.(ответ выразите в омах).

Леонид, обьясни пжл. как решать?

2349. Леонид , 29 октября 2010, 21:10:08
I*R = U, R = 220:8 , R= 27,5. это просто, в банке задачки заморочнее - пугают большими условиями и формулами.

2350. Игорь, 29 октября 2010, 23:24:11
Действитльно легкая, я и сам бы смог сделать :) спасиб, Леонид.

В12. Обьемы ежемесячной добычи газа на первом, втором и третьем месторождениях относятся как 7:6:14. Планируется уменьшить месячную добычу газа на первом месторождении на 14% и на втором-тоже на 14%. На сколько процентов нужно увеличить месячную добычу газа на третьем месторождении, чтобы суммарный обьем добываемого газа за месяц не изменился?

Леонид, как решить? :)

2351. Леонид , 30 октября 2010, 11:22:30
ИГОРЮ. Требуется, чтобы и до, и после изменения общий объем добычи, равный 7+6+14, сохранился. Составляем уравнение:
7+ 6 + 14 = 7 &#8729; (1 - 14/100) + 6 &#8729; (1 - 14/100) + 14 &#8729; (1 + x/100), где x - это как раз на сколько процентов нужно увеличить объем добычи на третьем месторождении.
Решаем, получаем в ответе 13.

Вообще-то эта задача не из банка заданий, она не из простых по сравнению с уровнем сложности В12.

2352. Игорь, 31 октября 2010, 00:30:26
Леонид, 7+ 6 + 14 = 7 &#8729; (1 - 14/100) + 6 &#8729; (1 - 14/100) + 14 &#8729; (1 + x/100), эта строка глюканула :(

Я эту задачу брал не из банка.А из сборника, ну ты знаешь, он 2010 года.Решил что надо понять алгоритм решения, а вдруг встретится на ЕГЭ!?

2353. Леонид , 31 октября 2010, 07:48:11
что-то долго не было. такой задачи не будет, она не отвечает уровню сложности. Позже, в течении дняповторю. Не теряй время, решай В12 из банка. А почему абра-кадабры появились не знаю.

2354. Игорь, 31 октября 2010, 08:54:09
Леонид, интернет не подключалcя

В12. Первую половину трассы автомобиль проехал со скоростью 38 км/ч, а вторую со скоростью 57 км/ч.Найдите среднюю скорость автомобиля на протяжении всего пути.


2355. Леонид , 31 октября 2010, 11:44:35
Пксть весь путь 2. То 1/38 - время, затраченное на у-юполовину пути. 1/57 -время, - на 2-ю половину пути. Всё время будет (1/57 + 1/38).
Тогда средняя скорость на всём пути есть 2:(1/38 + 1/57) =
= 2:(3/114 + 2/114) = 2:(5/114)= 228:5 = 45,6 км.ч.

Весь путь взят за 2, чтобы проще было считать. Можно было за А, то полпути 0,5А - больше писанины и вычислений.










2356. Леонид , 31 октября 2010, 12:37:40
к записям 2350 - 2353.
Требуется,чтобы и до, и после общий объём добычи, равный в частях 7+6+14, сохранился. Пусть на х процентов нужно увеличить добычу на третьем месторождении.
На первом: 7(1-14/100). На 2-м: 6(1-14/100). На 3-м 14(1-х/100).

Уравнение: 7+6+14 = 7(1-14/100) + 6(1-14/100) + 14(1-х/100).
27=7*0,86 + 6*0,86 + 1400(100-x)... x = 13

14% = 14/100. Уменьшили 1-14/100. 1 - на одну чась. Могли бы обозначить за А, но увеличило бы объём записей и преобразований.
Игорь, ещё один вопро - твои предки казаки?

2357. Алена, 31 октября 2010, 16:09:17
помогите решить пару заданий

2358. Алена, 31 октября 2010, 16:11:50
найдите площадь треугольника, вершины которого имеют координаты (1;6), (10;6), (2;8)

2359. Алена, 31 октября 2010, 16:16:31
в сосуд, имеющий форму правильной треугольной призмы, налили воду. Уровень воды достигает 4см. На какой высоте будет находиться уровень воды, если ее перелить в другой такой же сосуд, у которого сторона основания в 2раза больше, чем у первого? Ответ дайте в сантиметрах. Пожалуйста решите кто нибудь!

2360. Леонид , 31 октября 2010, 18:09:40
АЛЁНА, Игорь тоже пригодится: объём воды сохраняется!!! Объём призмы правильной треугольной по формуле: V = (a^2 корней из 3)/4 *Н =
= (a^2 корней из 3)/4 *4 (сторону основания увеличиваем в 2 раза, ноавя высота - уровень h).
(2a)^2 корней из 3*4. 4*a^2 корней из 3 *4 = a^2 корней из 3 *h ,
делим обе части на a^2 корней из 3- получаем 4*4=16.

Отметьте указанные точки на клетчатом листе в координатной плоскости ХОУ, получим треуглдьник с основанием 9 и высотой 2. Площадь 1/2*9*2=9

2361. Игорь, 31 октября 2010, 20:42:39
Леонид, предки РУССКИЕ! :)

В12. Из пункта А в пункт В вниз по течению реки отправились одновременно моторная лодка и байдарка. Скорость течения реки равна 3 км/ч. Последнюю 1/7 часть пути моторная лодка шла с выключенным мотором, и ее скорость относительно берега была равна скорости течения. На той части пути, где моторная лодка шла с выключенным мотором, ее скорость была на 2 км/ч больше скорости байдарки.Найдите скорость байдарки в неподвижной воде, если в пункт В байдарка и моторная лодка прибыли одновременно.

Леонид, замудреная задачка :) как с ней быть?

2362. Игорь, 31 октября 2010, 21:22:21
Произвольные задачи.
Проценты, сплавы, смеси.
1. Имеется кусок сплава меди с оловом общей массой 24 кг, содержащий 45% меди. Сколько чистого олова надо прибавить к этому куску сплава, чтобы полученный сплав содержал 40% меди?

2.Сплав меди с цинком, содержащий 5 кг цинка, сплавили с 15 кг цинка.В результате содержание меди в сплаве понизилось по сравнению с первоначальным на 30%. Какова была первоначальная масса сплава, если известно, что она была меньше 20 кг?

3. Стоимость 20 мячей до уценки составляла 900 руб. Какое максимальное количество мячей можно приобрести на ту же сумму после их уценки на 10%

Леонид, обьясни пжл. как решать?

2363. Леонид , 31 октября 2010, 22:10:56
стоимость 1 мяча до уценки 900:20 = 45 р.
Уценили на 10%, мяч стал стоить 45*0,9 = 40,5р. На 900 р приобретём 900:40,5=22 мяча.
Игорь, казаки - русский свободолюбивый народ.

Задачи на сплавы на ЕГЭ не предлагаются, их нет в банке данных. Их можно отнести к одним из сложных задач. Ты почему не прислушиваешься к советам. Не сообщаешь понятно ли. Решённые задачи (Алёна) к тебе обращаюсь там, читал?

В12. Из пункта А в пункт В вниз по течению реки отправились одновременно моторная лодка и байдарка. Скорость течения реки равна 3 км/ч. Последнюю 1/7 часть пути моторная лодка шла с выключенным мотором, и ее скорость относительно берега была равна скорости течения. На той части пути, где моторная лодка шла с выключенным мотором, ее скорость была на 2 км/ч больше скорости байдарки.Найдите скорость байдарки в неподвижной воде, если в пункт В байдарка и моторная лодка прибыли одновременно.

1 - весь путь. Скорость м.л. по течению (х+3), затратила время до выключения мотора (6/7)/(х+3), где х - ск. м.л., на остальной путь её время (1/7)/х. Далее мне не понятно : как ск. м.л., плывущей со ск. реки (мотор выключен) может быть больше ск. байдарки на 2км.ч.- получается байдарка плыла медленнее скороти реки....? Посмотри условие точнее.


2364. Леонид , 1 ноября 2010, 04:24:43


Алёнка (Игорь) поправка в записи 2360 (поторопился, извиняюсь) в строке
(2a)^2 корней из 3*4. 4*a^2 корней из 3 *4 = a^2 корней из 3 *h, - должно быть (2a)^2 корней из 3/4*h = (a^2 корней из 3)/4 *4 ,
(4a^2 корней из3)/4*h=(a^2 корней из 3)/4*4 Делим на (a^2 корней из 3)/4. Получим 4h =4, h=1-станет уровень.

2365. Игорь, 1 ноября 2010, 07:38:54
2363. Леонид , 31 октября 2010,

Условие я правильно написал, это точно.Замудренная задача :)
Поправку вижу,уже изменил у себя.

Нет не казаки.

2366. Леонид , 1 ноября 2010, 08:27:15
По новой с поправкой: 1 - весь путь. Скорость м.л. по течению (х+3), затратила время до выключения мотора (6/7)/(х+3), где х - ск. м.л., на остальной путь её время (1/7)/3 !!! . Далее мне не понятно : как ск. м.л., плывущей со ск. реки (мотор выключен) может быть больше ск. байдарки на 2км.ч.- получается байдарка плыла медленнее скороти реки....? Посмотри условие точнее.

коль на 1/7 пути скорость м. лодки 3, как и скорость реки (мотор выключен) - скорость Б. здесь на 2 меньше скорости м.л., т.е 1 чтоли это её собственная ск.)

Надо к LEXXUS Обратиться. ПОМОГИТЕ.

В12. Из пункта А в пункт В вниз по течению реки отправились одновременно моторная лодка и байдарка. Скорость течения реки равна 3 км/ч. Последнюю 1/7 часть пути моторная лодка шла с выключенным мотором, и ее скорость относительно берега была равна скорости течения. На той части пути, где моторная лодка шла с выключенным мотором, ее скорость была на 2 км/ч больше скорости байдарки.Найдите скорость байдарки в неподвижной воде, если в пункт В байдарка и моторная лодка прибыли одновременно.


2367. Леонид , 1 ноября 2010, 09:16:25
Вот так!!!
х-ск. м.л. 6/7(х+3)- её время на путь до отключения двигателя. (1/7)/3 = =1/21 - её время на путь при отключённом двигателе. Всё время м.л. на весь путь 6/7(х+3) + 1/21.

Далее: х-2 собственная скорость байдарки на 1/7 пути, то на всём пути
(х-2+3 = х+1)- с учтом течения реки 3 км.ч. Время байдарки на весь путь
1/(х+1). Время их одинаковое - уравнение 6/7(х+3) + 1/21 = 1/(х+1). В одну часть, к общему, уравнение х кв. + х - 42=0, х=6-ск. м.л., то байдарки на 2 меньше, 4.

Да, поднакручено условие с понятием "собственных ск."

2368. Любаня-новичок, 2 ноября 2010, 20:21:18
Привет всем!!!пожалуйста, помогите мне с решением такого задания!!!
Коэффициент полезного действия некоторого двигателя определяется формулой n=(T1-T2)*100%/T1. При каких значениях температуры нагревателя Т1 КПД этого двигателя будет равен 80%, если температура холодильника Т1=200 К?

2369. Игорь, 2 ноября 2010, 20:41:40
Леонид, задача конечно не простая, и как то странно уровень сложности распологается, то совсем легко то трудно.И кто только следит за этим?!
Спасибо что решил :) А для чего ты про предков спрашивал?

2370. Игорь, 2 ноября 2010, 20:57:12
Леонид, вот еще В12. обьясни как решать.

В12. При настройке станка выпущены 3 пробные партии деталей с высоким процентом брака. В первой партии 20 деталей, из которых 40% бракованных. Во второй партии 14 деталей, из которых лишь 3 бракованных. В третьей партии 30 деталей, из которых не более чем треть деталей без брака. Было решено, что партия с наибольшим числом небракованных деталей будет передана настройщику, а остальные пойдут на переплавку.Сколько бракованных деталей получит настройщик?

2371. Леонид , 2 ноября 2010, 22:22:02
2369. Игорь, 2 ноября 2010. Просто ради интереса. В Кубани много казаков проживают. Ты разобрался с той задачей ? В ней вдумчивое, да в любой, - внимательное чтение необходимо.

Ты где опять выкопал такую задачу? В банке заданий их нет!!! У нас уже час ночи, для интереса заглянул (что так долго не было, активно отдыхаешь на каникулах?), давай завтра по- вашему. Сначала надо на бумаге, а то сходу ляпсусы из-за спешки получаются и времени уж много.
Уровень сложности? Наверное регулируют. Например В8 полегче, В5 потруднее...

2372. Леонид , 2 ноября 2010, 22:47:12

1-я партия 2-я партия 3-я партия
деталей всего 20 14 30

бракованных 20*0,4=8 3

небракованных 12 11 <=30*1/3=10, <= 10

Передадут настройщику 1-ю партию 20 деталей с наибольшим числом небракованных 12 деталей. Значит 8 бракованных получит настройщик.

2373. wedima, 3 ноября 2010, 15:20:15
В боковой стенке цилиндрического бака вблизи дна закреплен кран. После его открытия вода начинает вытекать из бака, при этом высота столба воды в нём меняется по закону
H(t) = 7,2-1,92t+0,128t2,
где t – время в минутах.
В течение какого времени вода будет вытекать из бака?
Решение:
Так как вода будет вытекать из бака при условии, что уровень больше нуля, то получаем:
7,2-1,92t+0,128t2&#8805;0
Решим данное неравенство.
7,2-1,92t+0,128t2=0
D=b2-4ac=(-1,92)2-4*(0,128)*7,2=3,684-3,684=0
D=0, следовательно система имеет одно решение.
t=(-b±&#8730;(b^2-4ac))/2a = -1,92/0,256=7,5
t=7.5
Ответ: вода вытекает за 7.5 минут

2374. Татьяна, 5 ноября 2010, 10:44:22
У меня вопрос: как можно скачать весь банк открытых заданий, чтобы решать, не прибегая к связи в Интернете?

2375. Норжима, 6 ноября 2010, 08:37:14
найдите наибольшее значение функции у=cos x на промежутке[2;3]

2376. Настя, 6 ноября 2010, 09:15:46
[2;3] - вторая четверть. y = cosx во второй четверти убывающая, след-но её наибольшее значение cos2.
Или: Производная у* = - sin x. y* =0, sinx=0. x = pi n. нет попадающих в [2;3].

2377. Игорь, 6 ноября 2010, 10:27:24
Леонид, сейчас редко потому что у меня тариф для интернета уже закончился на этот месяц. С 15-го буду как штык :)

Ту задачу разобрал, ну и наврятли с таким уровнем сложности они будут на ЕГЭ.

С1. Решите систему уравнений.
siny=x-6
cosy=x-7

Леонид, обьясни как решать :)

2378. Леонид , 6 ноября 2010, 12:18:45
А что за тариф, я в месяц плачу 500 и сколько угодно работаю (безлимитный), у меня интернет проводной. Где взял эту систему. Ой, ПРИВЕТ, забыл поздоровоться.
x=siny+6, x=cosy+7, выразили х в 1-м и 2-м уравнениях, приравняв х, получаем уравнение: siny+6=cosy+7, siny-cosy - 1=0. Применим формулы двойного угла для синуса и косинуса и распишем 1, перенесём в одну часть:
2sin(y/2)cos(y/2) -cos^2(y/2)+sin^2(y/2) -sin^2(y/2) - cos^(y/2)=0- подобные.
2sin(y2)cos(y/2) -2cos^2(y/2)=0. Общий множитель за скобки 2cos(y/2) (sin y/2 - cos y/2) = 0. Произведение равно 0
1. cos y/2=0, y/2=pi/2+pk |*2, y=pi+2pik - подставляем вето у во 2 - е ур-е: cos(pi+2pik)=x-7, -1=x -7, x=6. (6;pi+2pik)

2. sin y/2 - cos y/2 = 0 |:cos y/2. tg y/2 = 1, y/2 = pi/4 + pik, y = pi/2 + 2pik- вместо у во 2-е ур-е: cos(pi/2+2pik) = x-7, 0=x-7, x=7. (7;pi/4+2pik).

Област допустимых значений:-1<=siny<=1, -1<=x-6<1 |+6, 5<=x<=7(*).
и -1<=x-7<=1 |+7 , 6<=x<=8(**). Общее ОДЗ из (*) и (**) 6<=x<=7. Ответ:(6;pi+2pik) и (7;pi/4+2pik).

Про детали и настройщика разобрался? Пошто молчишь? Печатал там в виде талицы красиво, расползлось.

2379. Lina, 7 ноября 2010, 14:29:46
Высота прямой призмы ABCA1B1C1 равна 18. Основание призмы-треугольник ABC, площадь которого равна 12, AB=5. Найдите тангенс угла между плоскостью ABC1 И плоскостью основания призмы.

Можете решить, пожалуйста!

2380. Леонид , 7 ноября 2010, 16:08:39
Угол между указанными в условии плоскостями - это всё равно, что угол образованный высотами треугольника АВС1 и АВС - с общей стороной АВ - их высоты прводятся в одну точку Н (не середина АВ). Высоту СН находим из данной площади Треугольника: 12 = 1/2 АВ*CH. Отсюда СH =24:5= 4,8. В треугольнике С1НС - прямоугольный (призма прямая) по определению tg C1HC = C1C:CH = 18:2,4 = 7,5/

2381. Жаклин, 7 ноября 2010, 16:34:45
Будьте любезны, решите, плииииз!

C1 log^2 числа x по основанию 2 + log числа x по основанию 2 - 1 = (корень из 5-x^2)^2 + x^2

C2 Найдите все значения x, при каждом из которых расстояние между соответствующими точками графиков функций f(x) = log числа (3x+13) по основанию 2 и g(x) = 5,5 меньше, чем 0,5




2382. Lina, 7 ноября 2010, 17:32:49
Леонид, спасибо огроомное!
Только одно не поняла: в конце вы написали, что CH=2,4, а до этого вычислили и получилось 4, 8. Наверно, опечатка или я не поняла.

2383. Леонид , 7 ноября 2010, 23:09:06
Да, невнимательность: СН =4,8 ТО 18:4,8=3,75

2384. оля, 7 ноября 2010, 23:34:26
2
ax +bx+c помогите пожалуйста я в 4 классе.

2385. Леонид , 7 ноября 2010, 23:41:59
2381. Жаклин, 7 ноября 2010, 16:34:45. Запись правой части 1-го не понял.
2-е:
Растояние - это модуль, запише: | log (3x+13) по осн.2 - 5,5| <0,5. К двойному неравенству по определению модуля: -0,5<log(3x+13) по осн.2 -5,5<0,5. Прибавим ко всем частям неравенства 5,5 - получим: 5<log(3x+13) осн.2<6. 5 и 6 заменим логарифмами с осн.2: log32 осн.2< log(3x+13) осн.2<log65 осн.2
Логарифмическая функция с основанием 2 , большим 1,- возрастающая. Освобождаемся от логарифмов 32<3x+13<64 |-13, 19<3x<51 |:3,
6 1/3<x<17.
ОДЗ: 3x+13>0, x>-4 1/3. Ответ: 6 1/3<x<17. Разбираясь, проверяйте. Да, это задание не может быть С2, С2-стереометрия, для С3 - простое

2386. Жаклин, 8 ноября 2010, 21:08:35
Спасибо большое!!!
Я как раз C1 сама сделала)
это просто старый вариант егэ) где-то 2008-2009.

Можете решить вот эту.

C5 Найдите все значения a, при каждом из которых оба числа 3sina+5 и 9cos2a-36sina-18 являются решением неравенства в числителе (25x-3x^2+18)*sqrt(x-1), в знаменателе log(модуль(x-7))-1 осн.4 >=0

2387. дима, 8 ноября 2010, 21:32:06
Задание B8 (8049) посмотрите че за брехня там ответ единица а ваш коллективный разум мне тычет что два, хотя большинство сказало 1

2390. дима, 9 ноября 2010, 11:15:06
2388 Blackduard мы сюда не отвлекаться заходим а решать.
2389 леонид я с тобой согласен ответ реально 1 а номер я правильно назвал получается колективный разум тупит

2391. Kattu, 9 ноября 2010, 13:21:19
Уважаемый Lexxus! Вот номера заданий с неправильными ответами: 24429, 6999, 6459, 6493, 7803, 8049, 7535, 24133, 6955, 28551, 27068. Можно ли исправить в них ответы? На сайте открытого банка добавили новые прототипы B7, следовательно и новые задания. Всего там 56830 задач. Не могли бы Вы добавить новые задачи на этот сайт? Спасибо

2392. Леонид , 9 ноября 2010, 13:34:58
2390. дима, 9 ноября. Дима, каждый имеет право на ошибу. Ты выражай своё несогласие без, типа, тупит. Lexxus просматривает все наши записи и если есть ошибка, он поправит либо нас либо ошибку и сообщит об этом. БУДЕМ УВАЖИТЕЛЬНЫМИ друг к другу. Хорошо?

2393. dimageniy, 9 ноября 2010, 19:06:42
уважаемый леонид? я внимательно прочел ваше обращение и решил с вами согласиться я очень расчитываю на Lexxus ))

а ещё я хочу спросить у Kattu вот вы идете первой , а вы 11 класс и много ли вы сидите в сутки в открытом банке? Спасибо
P.S. я заметил что тут пишут свои реальные логины поэтому я тоже напишу свой

2394. Жаклин, 9 ноября 2010, 21:35:45
Пожалуйста, решите! Очень надо!

C5 Найдите все значения a, при каждом из которых оба числа 3sina+5 и 9cos2a-36sina-18 являются решением неравенства в числителе (25x-3x^2+18)*sqrt(x-1), в знаменателе log(модуль(x-7))-1 осн.4 >=0

2395. мишечка, 9 ноября 2010, 22:02:17
где можно сдать в инете пробный еге без смс

2396. Kattu, 9 ноября 2010, 23:13:14
Уважаемый Lexxus! В открытом банке уже около 57000 задач, может быть стоит подумать о размещении в вашей базе только прототипов, постепенно добавляя их вместе с открытым банком. Сейчас там всего 1500 прототипов. Представляете 56000 задач, а у нас в коллективном разуме люди решили максимум чуть больше 4000. В противном случае ваша замечательная идея постепенно отомрет за не надобностью. Большинство задач так и не будут никогда решены. Спасибо.

2397. Lexxus, 10 ноября 2010, 00:32:29
Задание B8 (8049) посмотрите че за брехня

И правда, брехня. Исправлено.

Вот номера заданий с неправильными ответами: 24429, 6999, 6459, 6493, 7803, 8049, 7535, 24133, 6955, 28551, 27068. Можно ли исправить в них ответы?

24429, 6999, 7803, 7535, 24133, 6955, 28551, 27068 - исправлены;

6493 - там ответ вообще пока не определён (впрочем, всё равно спасибо за сигнальчик, привёл формулировки этой и подобных задач в божеский вид);

6459 - формулировка не очевидная, но я склонен скорее согласиться с коллективным разумом. Впрочем, если приглядеться к этой задаче ещё внимательнее, то ответ к ней вообще невозможно определить, поскольку неясно, какая размерность у h в приведённой формуле.

2398. Жаклин, 10 ноября 2010, 17:10:39
Никто не может решить или никто не пробовал?
Ну, плиииз!

C5 Найдите все значения a, при каждом из которых оба числа 3sina+5 и 9cos2a-36sina-18 являются решением неравенства в числителе (25x-3x^2+18)*sqrt(x-1), в знаменателе log(модуль(x-7))-1 осн.4 >=0

2399. Lexxus, 10 ноября 2010, 18:26:51
C5 Найдите все значения a, при каждом из которых оба числа 3sina+5 и 9cos2a-36sina-18 являются решением неравенства в числителе (25x-3x^2+18)*sqrt(x-1), в знаменателе log(модуль(x-7))-1 осн.4 >=0

Ну, насколько я понял, неравенство вот такое:
(25x-3x^2+18)*sqrt(x-1)/(log_4(|x-7|)-1) >= 0,
т.е. в знаменателе (логарифм по основанию 4 от |x-7|)-1.

1. Итак, для начала решим неравенство.
1.1. В числителе есть корень, значит, x>= 1
1.2. Квадратный двучлен в числителе раскладывается на -3(x+2/3)(x-9)
1.3. Разберемся со знаменателем.
1.3.1. Заметим, что x не может быть равен 7
1.3.2. Решим неравенство log_4(|x-7|)-1>0
|x-7| > 4 => x < 3 или x > 11
Поскольку мы решаем неравенство, и для нас важен только знак, то можно считать, что знаменатель ведет себя точно также, как (x-3)(x-11) (но только не надо забывать, что точку x=7 нужно "выколоть").

1.4. Итак, наше неравенство можно представить как систему:
x>=1
x не равен 7
-3(x+2/3)(x-9)/((x-3)(x-11)) >=0

Методом интервалов получим решение:
x принадлежит [1;3) и [9;11)

2. Теперь посмотрим на выражение 3sin(a)+5.
Поскольку значения синуса лежат внутри отрезка [-1;1], то это выражение может принимать значения в пределах отрезка [2;8].
То есть во второй полуинтервал из решения неравенства оно точно не попадает, а в первый попадает, если оно меньше 3, т.е.
3*sin(a)+5 < 3
sin(a) < -2/3
Итак, sin(a) может лежать в полуинтервале [-1;-2/3)

3. Осталось разобрать последнее условие - что 9cos2a-36sina-18 тоже является решением неравенства.
cos(2a) = 1-2sin^2(a) => выражение превращается в
9(1-2sin^2(a))-36sin(a)-18 = -18sin^2(a)-36sin(a)-9

заменим sin(a) на t и посмотрим, как ведёт себя функция y(t)=-18t^2-36t-9
на уже найденном полуинтервале [-1;-2/3).
y'(t) = -36t-36
единственный экстремум - в точке -1, и это максимум. Следовательно, функция на рассматриваемом полуинтервале всюду убывает.
y(-1) = 9
y(-2/3) = 7

Это значит, что наше второе выражение является решением неравенства только в том случае, если оно равно 9, т.е. когда sin(a) = -1

Так что ответ -
a = -пи/2 + 2пи*n

2400. Света, 10 ноября 2010, 19:17:41
Очень круто придумали, молодцы!)

2401. Жаклин, 10 ноября 2010, 21:22:38
Lexxus, спасибо огромное!!! очень помогли!
Только sina=-1 это наверно -п/2+2пn

2402. Lexxus, 11 ноября 2010, 00:02:12
Только sina=-1 это наверно -п/2+2пn

Совершенно верно :)

2403. Оля, 11 ноября 2010, 12:15:27
Всем привет, помогите пожалуйста решить задачку:в треугольнике АВС угол С=90 градусов,АС=16, cosA=4/5. Найти длину сторон ВС. Помогите очень нужно.

2404. dimageniy, 11 ноября 2010, 12:58:53
Оля, надеюсь в косинусах ты разбираешься. из определения косинусов находишь чему равна гипотенуза из равенства4/5=16/АВ , АВ=25
из основного тригонометрического тождества ( SINвквадрате A +COSвквадратеA =1 получаем SIN A=3/5 и из определения синуса получаем ВС= АВ* SIN A = 25*3/5=15

ОТВЕТ ВС=15

2405. Леонид , 11 ноября 2010, 13:54:21
Оля, короче: найдите АВ по определению косинуса. ВС - по теореме Пифагора.

2406. dimageniy, 11 ноября 2010, 14:35:13
ну так даже лучше

2407. Натали, 11 ноября 2010, 21:12:55
Ребят, не получается, помогите и объясните пожалуйста :
в треугольнике АВС угол с равен 90 АВ=5, tg В=4/3.найти ВС.
Вот, а дальше, не знаю как:
tg В=4/3 ->АС/ВС=4/3?

2408. dimageniy, 11 ноября 2010, 21:32:05
пусть одна часть равна Х то tg B =4x/3x
по теореме пифагора 16х в квадрате +9х в квадрате=25
х в квадрате равен 1
х=1 , значит СА=4 СВ=3
ответ ВС=3

2409. Арсений, 12 ноября 2010, 02:19:55
К задачи Задание B8 (7063) дан неправильный ответ - 6, тогда как в реальности правильный ответ - 3.

2410. dimageniy, 12 ноября 2010, 08:06:05
Арсений действительно ,вы правы. Интересно почему система считает что отве=6, ведь там вариантов 3 больше. Lexxus объясните пожалуйста

2411. Леонид , 12 ноября 2010, 08:28:34
Ребяты, ответ 6 правильный. Судите: дан график функции, производная отрицательна там, где функция убывает. Идите по склонам графика слева направо (вниз) и считайте точки на ОХ с целыми координатами на линии графика.

2412. Леонид , 12 ноября 2010, 09:30:35
Одна мысль. В В12 появились новые задач. Хороши ли некоторые из них? Например, два поезда (пассажирский и грузовой) с приличными скоростями движутся в одном направлении по параллельным путям ...
Ответ, скорее всего, страшная трагедия. А Вы как думаете?

2413. dimageniy, 12 ноября 2010, 11:58:55
да леонид, вы правы тут невнимательность подвела.

2414. Елена Ил, 12 ноября 2010, 19:26:24
Решала задания. В №5301 ответ не верный, там посчитана длина окружности, а площадь круга равна радиус в квадрате умножить на пи, в ответе должно быть 16

2415. Елена Ил, 12 ноября 2010, 19:30:38
Решала задания. В зданииВ6 (№5301) ответ не верный, там посчитана длина окружности, а площадь круга равна радиус в квадрате умножить на пи, в ответе должно быть 16

2416. Kattu, 12 ноября 2010, 19:54:58
Уважаемый Lexxus! В открытом банке уже около 57000 задач, может быть стоит подумать о размещении в вашей базе только прототипов, постепенно добавляя их вместе с открытым банком. Сейчас там всего 1500 прототипов. Представляете 56000 задач, а у нас в коллективном разуме люди решили максимум чуть больше 4000. В противном случае ваша замечательная идея постепенно отомрет за не надобностью. Большинство задач так и не будут никогда решены.

2417. Леонид , 12 ноября 2010, 22:48:13
№5301. Найти площадь защтрихованной части, т.е. полукруга, радиус которого 4. Площадь круга piR^2 = 16pi. Полукруга 8pi, для записи ответа надо разделить на pi. Ответ 8. Ни какой длины окружности там не посчитано.

2418. Арсений, 13 ноября 2010, 00:21:07
По поводу задачи B8 (7063) - в отрицательной части шкалы OX отрезки убывания функции не попадают на целые значения X - в этих значениях находятся точки экстремума, в которых производная=0. Крайние точки (-6 и 5) в интервал не входят. Остается отрезок убывания функции, расположенный в положительной части OX. В этом отрезке только 3 целых точки (x=2,x=3,x=4).

2419. Леонид , 13 ноября 2010, 00:48:37
Верно

2420. Kattu, 13 ноября 2010, 12:29:47
Lexxus! Спасибо большое

2421. Lexxus, 13 ноября 2010, 13:18:53
Lexxus! Спасибо большое

Да я, вроде бы, выше предложение пока (!) проигнорировал - за что спасибо-то?

А вообще, я планирую отделить прототипы и попытаться собрать базу решений (не ответов, а именно решений) каждого из них.
Но работа эта слабо поддается автоматизации, поэтому может затянуться.

Ещё есть мысль сделать отдельную базу по решённым здесь задачам части C. Это проще, но тоже пока на уровне общей идеи.

2422. Kattu, 13 ноября 2010, 14:49:14
Lexxus! Прототипы с решениями мысль хорошая, но процесс действительно может затянуться. Если есть возможность, лучше отделить прототипы и прорешать их хоть коллективным разумом, хоть как. Пока их меньше 1500. Удачи!

2423. natka, 13 ноября 2010, 17:23:25
Пожалуйста, объясните почему в следующей задаче(В10№6115) в ответе выбран наименьшей корень, хотя спрашивают про наибольший??????

Зависимость температуры (в градусах Кельвина) от времени (в минутах) для нагревательного элемента некоторого прибора была получена экспериментально и на исследуемом интервале температур задаётся выражением T(t)~=~T_0+at+bt^2, где T_0~=~1160 К, a~=~34 К/мин, b~=~-0,2 К/(мин)2. Известно, что при температурах нагревателя свыше 2000 К прибор может испортиться, поэтому его нужно отключать. Определите (в минутах) через какое наибольшее время после начала работы нужно отключать прибор.

2424. Leila, 13 ноября 2010, 18:02:48
Всех приветствую! И прошу помочь мне,наверное,с самым простым заданием В9:
Найти боковое ребро правильной четырёхугольной призмы, если сторона её основания равна 20, а площадь поверхности равна 1200.
(Извиняюсь,если вдруг повторяюсь с этим заданием)

2425. dimageniy, 13 ноября 2010, 18:20:26
2424. Leila

S=1200
S(осн)=20*20=400
S(бок)=1200-2S(осн)=1200-800=400
S(одной бок грани)=400/4=100
S(одной бок грани)=20*на высоту
высота=100/20=5


ОТВЕТ 5

2426. Леонид , 13 ноября 2010, 18:49:14
2425. dimageniy - МОЛОДЕЦ. Хороший, краткий, понятный расклад задачи. Всё верно!

2427. Леонид , 13 ноября 2010, 19:13:57
2423. natka/

Подставляем данные в формулу Т(t): 1160 + 34t -0,2t^2 <=2000,
-0,2t^2 + 34t - 840<=0 |*10 : (-2). t^2 - 170t + 4200>=0. Решаем квадратное неравенство: ветви параболы вверх D = 12100, t1=30; t2=140. t<=30 и t>=140- решения неравенства, t=30. Что значит в записях ~?
Если взять t>=30, прибор сгорит - говорит квадратичное неравенство.

2428. dimageniy, 13 ноября 2010, 19:40:51
2426. Леонид спасибо стараюсь

2429. Кира, 13 ноября 2010, 21:14:09
мастер и ученик могли бы выполнить заказ за 6 часов. Мастер, работая вместе с учеником 4 часа, помог выполнить часть задания. Оставшуюся работу ученик закончил самостоятельно за 6 часов. за сколько часов мог бы выполнить заказ ученик, работая без помощи мастера?

2430. ilo, 13 ноября 2010, 22:52:36
natka, 13 ноября 2010, 17:23:25
Пожалуйста, объясните почему в следующей задаче(В10№6115) в ответе выбран наименьшей корень, хотя спрашивают про наибольший??????
Прибор может работать при условии, что температура <=2000 град. Решением неравенства 1160+34t-0,2t^2<=2000 является временной интервал от 0-начала работы до 30 мин и от 140 мин до бесконечности. В промежуток времени от 30 мин до 140 мин температура превысит 2000, а, следовательно, после включения может пройти самое большее 30 мин., после чего прибор надо отключить

2431. Leila, 14 ноября 2010, 07:23:53
2425. dimageniy, Спасибо огромное) Всё оказалось намного проще,чем я думала)
У меня ещё к вам вопрос:
задание С3 (неравенство)
x2-6x+8/x-1 - x-4/x2-3x+2 меньше или равно 0
Привела к общему знаменателю - получила выражение, с которым не знаю, что делать....

2432. Leila, 14 ноября 2010, 07:25:06
2425. dimageniy, Спасибо огромное) Всё оказалось намного проще,чем я думала)
У меня ещё к вам вопрос:
задание С3 (неравенство)
x^2-6x+8/x-1 - x-4/x^2-3x+2 меньше или равно 0
Привела к общему знаменателю - получила выражение, с которым не знаю, что делать....

2433. Леонид , 14 ноября 2010, 08:20:51
2432. Leila, 14 ноября! Видимо получили кубический многочлен и не справились. Приводите к общему знаменателю, разложив всё на множители, там, в чеслителе появится общий множитель...
(х-2)(х-4)/(х-1) - (х-4)/(х-3)(х-4)<=0. К общему знаменателю: ( (х-2)^2*(x-4) - (x-4) )/(x-1)(x-2). (х-4) за скобки, имеем (x-4)*((x-2)^2 - 1) всё это делённое на (х-1)(х-2). (x-4)(x^2 - 4x + 3)/(x-1)(x-2)<=0.
(x-4)(x-1)(x-3)/(x-1)(x-2)<=0. Сокращаем на (х-1), не забыв потом 1 выбросить, если она попадёт в решение (сокращение - деление - на 0 делить нельзя). Имеем: (x-4)(x-3)/(x-2)<=0. Мнтодом интервалов x<2 U 3<=x<=4. Выбросим 1:(-~;1)U(1;2)U[3;4]. Разбираясь, проверяй, вдруг где опечатка.
~ - бесконечность у меня.

2434. natka, 14 ноября 2010, 11:57:20
Артем вот решение твоей задачки:

1. Найдем время, которое катер затратил на весь путь: 18-10-4=4ч
2. Пусть х-собств. скорость катера, тогда х-2 скорость по теч., а х+2 скорость против теч.
3. Составим уравнение:
15/х-2 - 15/х+2=4
4. Решаем уравнение. Приводим к одинаковым знаменателям и получаем след. квадратное уравнение:
2х в кадрате-15х-8=0
Получаем, что х=8 и -0,5(не удовлетв)
5. Ответ: 8
но за правильность я конечно не ручаюсь, но это тоже результат.

2435. dimageniy, 14 ноября 2010, 18:59:54
задание 7929 ответ 1 , а там написано что 2 . меня эти нестыковки начинают бесить

2436. Alla, 14 ноября 2010, 20:09:55
А беситься не надо, потому что правильный ответ 2. (ты видимо перепутал точки максимума и минимума

2437. Kattu, 14 ноября 2010, 20:21:57
В номере 7929 ответ 2

2438. Леонид , 14 ноября 2010, 20:28:16
2435. dimageniy, ответ 2. Производная с (-) на (+) - там график производной - читай внимательно, то функция с убывания на возрастание. Бдишь? В этих заданиях пристальность внимания, и понимание свойств функции по производной. Другие задания, наоборот, где по графику функции надо судить о производной.

2439. dimageniy, 14 ноября 2010, 21:37:21
да да, я понял просто на графике вместо 16 показалось 18 вот и ошибка спасибо за поправку

2440. Леонид , 14 ноября 2010, 22:40:56
Предлгаю задачу В 12 из добавленных в открытый банк ЕГЭ 2011.

Первый велосипедист выехал из посёлка по шоссе сокоростью 15 км/ч. Через час после него со скоростью 10 км/ч из того же посёлка в том же направлении выехал второй велосипедист, а ещё через после этого - третий. Найдите корость третьего велосипедиста, если сначала он догнал второго, а через 2 часа 20 минут после этого догнал первого. Ответ дайте в км/ч. (мой ответ 25).

Как бы вы решали эту задачу.

Прошу LEXXUS Ваше решение, если можно, с рисунком. Верен ли мой ответ? И решал "заморочисто" - м.б. Ваше решение проще.

Вообщет - то добавленные на 2011 задачи сложнее в плане осмысления. В 2010 г. на егэ В 12 дали верные ответы около половины выпускников.

2441. Леонид , 14 ноября 2010, 22:45:24
Поправка к задаче в записи под № 2440: "..., а ещё через ЧАС после этого - третий."

2442. Alla, 15 ноября 2010, 12:57:29
Леониду.
Не уверена, что мое решение проще.
х- скорость третьего
t - время, через которое третий догнал второго
t+1 - время второго, через которое третий догнал второго
Получаем, tx=10(t+1)

Через 2ч20 минут после этого
t+7/3 - время второго (от начала его движения), через которое он догнал первого
t+13/3 - время первого, через которое его догнал третий (первый был в пути на 2 часа дольше)
Получаем, 15(t+13/3)=x(t+7/3)

Решаем системы двух уравнений:
tx=10(t+1)
15(t+13/3)=x(t+7/3)
Получаем два ответа для х: 25 и 8,...
Скорость третьего не может быть меньше скорости второго
Ответ: 25

2443. Alla, 15 ноября 2010, 13:00:32
Подобная задача была в прошлом году на экзамене в 9 классе, дети ее решали проще, не могу вспомнить как. Можно эту задачу решить с помощью уравнения.

2444. Леонид , 15 ноября 2010, 14:50:20
Да, АЛЛА, так. Хотелось бы решение придумать попроще, пока не нахожу.

2445. Alla, 15 ноября 2010, 16:43:25
До выхода третьего первый шел уже 2 часа, т.е. прошел 30 км - это расстояние между третьим и первым на момент начала движения третьего. Скорость сближения первого и третьего (х-15) при условии, что х - скорость третьего. Значит время встречи 30/(х-15)

Ан-но, расстояние между третьим и вторым на момент выхода третьего - 10 км, скорость сближения (х-10). Время встречи 10/(х-10).

Разница во времени встреч 2ч20мин.

Получаем уравнение: (30/(х-15))-(10/(х-10))=7/3

2446. Alla, 15 ноября 2010, 16:44:49
Леонид!
Напишите, пожалуйста, понравился ли этот вариант решения

2447. Kattu, 16 ноября 2010, 13:11:31
Lexxus! Спасибо большое за прототипы, сделанные так быстро, но в прототипах к заданию В11 не все прототипы, их там на много больше. Возможно ли их добавить. Спасибо!

2448. Lexxus, 16 ноября 2010, 14:09:04
в прототипах к заданию В11 не все прототипы, их там на много больше

Хм, и правда. Странно, должны были скопироваться. Ладно, добавлю их чуть позже.

2449. svetlna, 16 ноября 2010, 15:04:47
В4 Помогите пожалуста, не знаю как решать((( в треугольнике угол c равен 90 ch высота ab=39 tgA=2/3 найдите ah

2450. Alla, 16 ноября 2010, 15:30:43
tgA=2/3. значит СВ=2х, АС=3х, по теореме Пифагора выражаешь АВ через х, приравниваешь к 39, находишь х (т.е. все стороны треугольника найдены, числа иррациональные, в этой программе писать не удобно)

Затем используешь две формулы площади треугольника: (АВ*СН)/2=(СВ*АС)/2

2451. svetlana, 16 ноября 2010, 15:34:40
Спасибо огромное!!

2452. Леонид , 16 ноября 2010, 16:07:19
2445. Alla, 15 ноября 2010, 16:43:25
Моё мнение - это решение красивее и доступнее для понимания. Спасибо!

2453. Игорь, 16 ноября 2010, 17:29:02
Привет Леонид, как дела?
Тариф у меня такой же почти как у тебя но есть ограничение, в месяц не более 2гб.
Интернет безпроводной.

2454. Леонид , 16 ноября 2010, 20:50:39
Здорово, Игорёк! Уменя не ограничений в гб. На 500р скольхочу выхожу. Нарешал много? Слушай, это у Вас теракты страшные были?

2455. Лена, 16 ноября 2010, 22:23:04
Мастер и ученик могли бы выполнить заказ за 6 часов. Мастер, работая вместе с учеником 4 часа, помог выполнить часть задания. Оставшуюся работу ученик закончил самостоятельно за 6 часов. За сколько часов мог бы выполнить заказ ученик, работая без помощи мастера?

2456. ilo, 17 ноября 2010, 08:21:09
Leila, 14 ноября 2010, 07:25:06 x^2-6x+8/x-1 - x-4/x^2-3x+2 меньше или равно 0
Привела к общему знаменателю - получила выражение, с которым не знаю, что делать....
Если получила выражение (х-4)(х-1)(х-3)/(х-3)(х-2)<=0, то решить можно методом интервалов, учитывая, что 1-корень чётной кратности и при переходе через него функция знак не меняет.Получаются два промежутка - полуинтервал (2;3] и луч [4;до беск), которые надо объединить

2457. ilo, 17 ноября 2010, 08:24:22
в знаменателе (х-1)(х-2)-опечатка моя

2458. ilo, 17 ноября 2010, 08:48:59
Лена, 16 ноября 2010, 22:23:04

1/6 часть всей работы мастер и ученик выполняют за час, за 4 часа они выполнят 1/6*4=2/3 всей работы,останется ученику выполнить 1/3 часть за 6 часов,значит всю работу он бы сделал за 18.

2459. Леонид , 17 ноября 2010, 11:05:01
2457. ilo, 17 ноября .Ответ дн неверный, судя по нервенству:
(х-4)(х-1)(х-3)/(х-1)(х-2)<=0, решая методом интервлов ( не могу нарисовть здесь). Ответ. Кратный корень отбрасывается (это сокращение, делить на 0 в научном мире запрещено) (-~;1)U(1;2)U[3;4]
LEXXUS, покажите народу метод интервалов рисунком. Пожалуйста.

2460. Lexxus, 17 ноября 2010, 11:41:42
LEXXUS, покажите народу метод интервалов рисунком. Пожалуйста.


Извиняюсь за качество, но рисовать в нормальной программе и выкладывать рисунок мне лень.


2461. anton, 17 ноября 2010, 11:55:17
помогите решить В4! в треугольнике АВС угол С равен 90 градусам, СН-высота,АВ=15, tg A=7.найти АН.

2462. Леонид , 17 ноября 2010, 14:56:40
2460. Lexxus, 17 ноября 2010. Огромное спасибо. Чётко, красиво, понятно!!!

2461. anton, 17 ноября . В треуг. СНА по определению тангенса 7 =СН : АН, то СН = 7 АН. Теперь свойство пропрциональных отрезков в прямоуг. треуг.
СН^2 = AH*BH, CH^2 = AH(AB - AH). Подставляем (7x)^2 =x*(15-x), где АН=х. 49x^2=15x-x^2. 50x^2 = 15, x^2=3/10. В ответ не идёт (встречаются такие задачи их убирают), может я где-то опечатку допустил. Решая проверь. Эта задача м.б. решена и по другому. Может быть что-то в твоём условии?

2463. ilo, 17 ноября 2010, 15:56:03
Да, действительно, у меня взяты промежутки, где функция в лев части положительна, а по условию должна быть неположительной, ошибочка вышла.

2464. Игорь, 17 ноября 2010, 16:20:12
Леонид, нарешал не много так как месяц с поломанной рукой ходил :)
Да теракты недалеко от нас, ужас.

2465. ilo, 17 ноября 2010, 16:36:11
anton, 17 ноября 2010, 11:55:17 Эта задача м.б. решена и по другому.
Отрезок AH обозначить х. Тогда tgA=AH/x=7 или AH=7x;
tgB=ctgA=1/7; AH/15-x=1/7; или 7AH= 15-x; тогда 49х=15-х; 50x==15; 10x=3;x=0,3

2466. Леонид , 17 ноября 2010, 17:51:53
Отрезок AH обозначить х. Тогда tgA=AH/x - как так?
AH/15-x=1/7 - как так, если АН=х? х/15-х? Непонятны увязки!


2467. Леонид , 17 ноября 2010, 18:18:40
Поправка: СН^2 = AH*BH, CH^2 = AH(AB - AH). Подставляем (7x)^2 =x*(15-x), где АН=х. 49x^2=15x-x^2. 50x^2 = 15*х - здесь х упустил раньше.
5х(10х-3)=0. х=0- не подходит, 10х-3=0, х=0,3


2468. ilo, 17 ноября 2010, 19:20:34
Леонид , 17 ноября 2010, 17:51:53
Отрезок AH обозначить х. Тогда tgA=AH/x - как так?
AH/15-x=1/7 - как так, если АН=х? х/15-х? Непонятны увязки!
Высота, проведённая из вершины прямого угла прямоугольного треугольника разбивает его на два подобных- ACH и BCH.Из АСH сторону AH обозначим за х, тогда tgA=CH/x=7;Откуда СH=7x Теперь из BCH находим tgB=ctgA=CH/BH=CH/15-x.=1/7. Откуда 7CH=15-x; т.к. CH=7x; 7*7x=15-x; 49x=15-x; x=0,3

2469. Леонид , 17 ноября 2010, 22:25:28
Да, сравните свои пред. записи и мои вопросы. Здесь видно. Там не фигурирует СН. И в этом мой вопрос.

2470. Лилия, 18 ноября 2010, 16:50:59
Леонид,здравствуйте!Просмотрела все задания:узнала многое,но не могу решить задачу.Помогите пожалуйста!!!!!!!!!
В прямоугольном параллелепипеде ABCDA1B1C1D1 известны три измерения AB=5,BC=12,CC1=7.Найдите угол между плоскостями CB1D1 и AB1D1.

2471. Лилия, 18 ноября 2010, 17:03:13
И еще одна непростая задача С5.Очень срочно!Я верю,что поможете!
Решите уравнение:x^2+(a-6)^2=модуль выражения x-a+6 плюс модуль выражения x+a-6.

2472. Лилия, 18 ноября 2010, 22:08:26
Я погибаю!Помогите же кто-нибудь решить задачи!

2473. Леонид , 18 ноября 2010, 22:25:05
2470. Лилия, 18 ноября.
Здесь чертёж, а значит и решение может описать только LEXXUS.
Сложно без чертежа. Треуг-ки - плоскости в условии - равные. Имеют общую сторону В1Д1 - равные: СВ1=АД1 и СД1=АВ1, В1Д1 - общая. Искомый угол-угол между их высотами СН = АК, тоже равными. Но - (Н и К, не совпадают - высоты пересекают основание ближе к меньшей стороне) - в этом сложность задачи...

2474. ilo, 18 ноября 2010, 22:56:15
Углом между плоскостями можно считать угол между нормалями к этим плоскостям.Нормалью к AB1D1 можно считать диагональ A1C, т.к.угол между диагональю куба и плоскостью, проведённой через концы трёх рёбер куба,выходящих из той же вершины прямой. А к В1D1C перпендикулярна диагональ AC1.Обе диагонали лежат в диагональной плоскости AA1C1C и равны корень из 218/2(из прямоугольного треугольника, скажем, AC1C) после по теореме косинусов:49=2*218/4+-2*218/4*cosx; cosx=60|169; x=arccos60|169;
Можно ввести систему координат с началом в точке В: A(5;0;0); C1(0;12;7) вектор AC1(-5;12;7); A1(5;0;7); C(0;12;0) вектор A1C(-5;12;-7). Находим скалярное произведение этих векторов: 25+144-49=120; модули этих векторов равны корню из218. делим скалярное произведение на произведение модулей-ответ тот же.

2475. Леонид , 18 ноября 2010, 23:44:47
У нас же не куб. И 120/218 неравно 60/169

2476. Настя, 18 ноября 2010, 23:47:02
И с нормалями, что - то не то.

2477. ilo, 19 ноября 2010, 10:27:33
Облом. Это не куб. Моё решение не верно. Признаю свою ошибку. Тогда остаётся построить плоскости аналитически: A(5;0;0) B1(0;0;7)D1(5;12;7) AB1D1: -84x +35y-60z+420; Вектор нормали: N1(-84;35;-60) C(0;12;0) B1(0;0;7)D1(5;12;7)
CB1D1: -84x+35y+60z-35*12; N2(-84;35;60) Модули векторов N1 и N2 равны 109, а скалярное произведение 4681, cosx=4681/11881; x=arccos 4681/11881; чщрошо бы проверить ещё раз. Если решать через площади, то с этими данными,мне кажется будет ещё заморочнее находить площади по формуле Герона, а по ней высоту.

2478. Леонид , 19 ноября 2010, 12:40:12
Задача хорошая, но угол между нормалями (перпендикулярами к плоскостям) ни как не равент углу между плоскостями.

2479. ilo, 19 ноября 2010, 13:17:44
Угол между плоскостями равен углу между их нормалями - скалярное произведение нормалей разделить на произведение длин нормалей. Корянов рекомендует в своём сборнике для подготовки к ЕГЭ , а он входит в аппеляционную комиссию.

2480. ilo, 19 ноября 2010, 13:32:11
Уточню: cosx=|A1*A2+B1*B2+C1*C2| / sgrt(A1^2+B1^2+C1^2)*sgrt(A2^2+B2^2+C2^2), где A1,B1,C1- координаты вектора нормали N1, а A2,B2,C2-вектора N2 из задачи.

2481. Тимур, 19 ноября 2010, 13:53:26
Помогите решить задачу - супертрейлер длина которого составляет 18 метров, проезжает мимо километрового столба за 9 сек. За какое время он на той же скорости проедет мост длиной 36 метров?

2482. Тимур, 19 ноября 2010, 14:43:39
спасибо всем сам решил

2483. Alla, 19 ноября 2010, 15:03:03
"2478. Леонид ,
Задача хорошая, но угол между нормалями (перпендикулярами к плоскостям) ни как не равент углу между плоскостями."

Абсолютно верно Леонид, это разные углы, но технология такая существует, только надо говорить не об угле , а о его косинусе с поправкой на знак. (хотя вопрос очень спорный, если угол между плоскостями острый и равен "а", то между нормалями равен (180 - "а"), ну а учитывая, что за угол между прямыми и плоскостями мы принимаем острый угол, то получаем, что углы имеют равные градусные меры)


2484. Леонид , 19 ноября 2010, 16:34:10
2481. Тимур, 19 ноября 2010, 13:53:26
Помогите решить задачу - супертрейлер длина которого составляет 18 метров, проезжает мимо километрового столба за 9 сек. За какое время он на той же скорости проедет мост длиной 36 метров?

Мимо столба он прходит как раз свю длину 18м за 9 сек, значит его скорость 2м/с. Расстояние в 36м он проходит с той же ск. 2м/с. Т.е. за время 36:2= 18сек. Или 0,3 часа - бывает ответ просят дать в др. единицах измерения нежели, чем получили. Заморочки маленькие для невнимательных.Имей в виду!

2485. ilo, 19 ноября 2010, 16:42:30
Чтобы полностью пройти мост, трейлер должен пройти его длину 36м+18м-длина самого трейлера,всего 54м.тогда54:2=27с или так :54 в 3раза больше 18 значит и время в 3 раза больше. Передняя точка проходит весь мост 36м,а чтобы полностью его пройти, надо дождаться, когда и задняя точка мост минует.

2486. ilo, 19 ноября 2010, 16:59:15
Alla, 19 ноября 2010, 15:03:03
"2478. Леонид Угол между пересекающимися плоскостями можно вычислить1) как угол между прямыми, лежащими в этих плоскостях и перпендикулярными к линии их пересечения.
2) как угол треугольника. если удаётся включить линейный угол в некоторый треугольник;
3)как угол между перпендикулярными им прямыми
4)используя формулу скалярного произведения в том числе в координатной форме

2487. Леонид , 19 ноября 2010, 17:22:38
Уважаемый LEXXUS, нужна помощь. Что то не клеится! Можно методом координат, вычислив высоту указанных треугольников (у них высоты эти равны - треугольники равные). Но больно уж заморочисто - по формуле Герона, стороны с радикалами. Да ещё вычислять координаты оснований высот, а потом координаты высот - векторов. И всюду радикалы. Чтобы пойти через скалярное произведение векторов. Это один ход решения.
Или оду из высот параллельно перенести, чтобы их основания были в одной точки и пойти по теореме косинусов. Но с длиной стороны против искомого угла возникает проблемы.

В общем, нужна помощь! Пожалуйста!

В прямоугольном параллелепипеде ABCDA1B1C1D1 известны три измерения AB=5,BC=12,CC1=7.Найдите угол между плоскостями CB1D1 и AB1D1.

2488. Alla, 19 ноября 2010, 22:13:26
Леонид
Очень не удобно описывать решение, но Вы человек умный, я думаю вам достаточно написать алгоритм
1) вводите систему координат
2) определяете координаты вектора АВ1{a.b.c} и АД1{n.m.t} (определяющих одну плоскость)
3) Составляете систему для определения координат нормали к плоскости {x.y.z}
ax+by+cz=0
nx+my+tz=0
В этой системе решений б/м (так же как и б/м нормалей к плоскости) задаете значение любой координаты нормали, например х, а остальные вычисляете с помощью системы
4) Аналогично определяете координаты нормали ко второй плоскости.
5) определяете косинус угла между нормалями (с помощью скалярного произведения) - это косинус угла между плоскостями.

Успехов!

2489. Lexxus, 19 ноября 2010, 22:54:29
В прямоугольном параллелепипеде ABCDA1B1C1D1 известны три измерения AB=5,BC=12,CC1=7.Найдите угол между плоскостями CB1D1 и AB1D1.

Я не думаю, что тут имеет смысл заморачиваться с координатным методом. И тем более - с формулой Герона.
Высоту CH1 треугольника CB1D1, все стороны которого мы находим из теоремы Пифагора (ну да, с радикалами, ничего страшного), найдем, записав теорему Пифагора для прямоугольных треугольников CH1B1 и CH1D1.
Там получится два уравнения с двумя неизвестными, нормально решается.

Получится CH1 = 109/13.

А угол между плоскостями будет равен удвоенному углу между CH1 и H1H (высотой, опущенной из H1 на плоскость ABCD, равной 7)

Так что в ответе будет 2*arccos(HH1/CH1) = 2*arccos(91/109)

2490. Арсений, 19 ноября 2010, 23:13:15
Не понял, почему в задании B8 (7393) ответ 3 - у меня -31 получается.
Там же вроде сумму значений x у точек эстремума данного графика функции.
А их значения: 1, 2, -2, -5, -8, -9, -10.


2491. Игорь, 19 ноября 2010, 23:23:08
Помогите решить пжл.
В11. найдите наименьшее значение функции у=(x-7)e^x-6 на отрезке [1;7]

2492. Леонид , 20 ноября 2010, 00:52:17
2489. Lexxus, 19 ноября 2010, 22:54:29. Спасибо. Я думал пойти с помошью т. Пифагора, показалось, что там будет неменьше заморочек с радикаламию Прорешаю.

Игорь, привет. Ты чего не здороваешься. Алгоритм как и раньше. Бери производную, приравнивай её к нулю. Находи значения функции на концах промежутка и в точках, ему принадлежащих. Выбирай наименьшее.

Это общий алгоритм. Для данного: y* = -7*e^(x-6) + (x-7)*e^(x-6) = e^(x-6)(-7+x-7) = e^(x-6)(x-14) =0. (x-14)=0. x=14- не принадл. [1;7]. Находим: y(1)=-6*e^(-5) и y(7)=0- наибольшее.

Здесь применяли производную произведения. Будь осторожен, когда показатель был бы не (х-7), а (7 - х), появится знак (-) посередине производной - неграмотно выразился - ты должен понять. производная сложной функции (х-7)штрих = 1, (7-х)штрих = -1

2493. Леонид , 20 ноября 2010, 01:05:24
2490. Арсений, 19 ноября 2010, 23:13:15
Не понял, почему в задании B8 (7393) ответ 3 - у меня -31 получается.
Там же вроде сумму значений x у точек эстремума данного графика функции.
А их значения: 1, 2, -2, -5, -8, -9, -10.

Не понятно "ответ3 - у меня -31 получается". Так что же у тебя! Это график ФУНКЦИИ - экстремумы в вершинках, там производная равна 0 (касательные в них параллельны ОХ). Это сложи(тобой виписанное). Ответ -31.

2494. Леонид , 20 ноября 2010, 07:43:56
2489. Lexxus, 19 ноября 2010. У меня снова вопрос по этой задаче. Да, СН1 найдём с помощью т. Пифагора. Но ведь АН1 не является высотой в треуг. АВ1Д1, чтобы как Вы пишите "А угол между плоскостями будет равен удвоенному углу между CH1 и H1H (высотой, опущенной из H1 на плоскость ABCD, равной 7)"- т.е. АН1С - ни есть линейный угол искомого двугранного угла. Основания высот указанных плоскостей (треугольников не приходятся в одну точку). Вразумите.

2495. ilo, 20 ноября 2010, 08:49:41
Спасибо, всё получилось!

2496. ilo, 20 ноября 2010, 09:38:34
из поста 2477 x=arccos 4681/11881 =2arccos91/109 - проверено, поэтому решение тоже правильно

2497. Леонид , 20 ноября 2010, 09:41:12
Да, угол то в любом случае получается. Но является ли он линейным. Основания высот треугольников здесь не приходятся в одну точку. Я имею в виду АН1С, СН1 перпенд В1Д1, а АН1 нет.

2498. Lexxus, 20 ноября 2010, 12:13:19
Но ведь АН1 не является высотой в треуг. АВ1Д1, чтобы как Вы пишите "А угол между плоскостями будет равен удвоенному углу между CH1 и H1H (высотой, опущенной из H1 на плоскость ABCD, равной 7)"- т.е. АН1С - ни есть линейный угол искомого двугранного угла. Основания высот указанных плоскостей (треугольников не приходятся в одну точку).

Основания высот, действительно, не совпадают. Но это не имеет значения.
Найдя угол между HH1 и CH1, мы нашли угол между плоскостями CB1D1 и BDD1B1. Таким же образом мы можем найти угол между AB1D1 и BDD1B1. Результат получится такой же, поскольку треугольники CB1D1 и AB1D1 равны.
Так что мы просто умножили найденный угол на два. Вот и всё.

2499. Леонид , 20 ноября 2010, 13:33:20
Спасибо.Замучил, ну ничего -для пользы дела.

2500. ilo, 20 ноября 2010, 13:36:27
Леонид , 20 ноября 2010, 09:41:12
Да, угол то в любом случае получается. Но является ли он линейным.
Угол между высотой СH1 и высотой AH2 можно заменить на угол между CH1 и отрезком, параллельным AH2, а это всё равно высота AB1D1, просто совместить H2 и H1.

2501. Леонид , 20 ноября 2010, 14:17:48
2500. ilo, 20 ноября
Да. треуг. СН1 ? (? - параллельно перенесенная А у АН2, совмещая Н1 и Н2) - равнобедренный (высоты равные, как в равных треуг.), поэтому можем рассматривать половину угла - его косинус.

2502. Леонид , 20 ноября 2010, 18:30:06
2485. ilo, 19 ноября 2010. Да верно, опечатался, не указав собственной длины. Таких задач и посложнее много добавлено в открытый банк заданий к ЕГЭ на 2011. Есть даже обгоны с приличными скоростями по параллельным путям пассажирского и товарного поездов. Кстата, от туда и задача, по которой мы общались - 3 велосипедиста через час....

2503. Лена, 21 ноября 2010, 17:42:32
помогите пожалуйста с решением
в10

Масса радиоактивного вещества уменьшается по закону M(t)=Mo*2^-t/T.
В лаборатории получили вещество,содержащее времени Мо=12мг изотопа ксенона-133,период полураспада которого равен Т=5,3суток. в течении скольких суток масса изотопа ксенона-133 в веществе будет превосходить 3мг?


в12

велосипедист отправился с некоторой скоростью из города А в город В,расстояние между которыми равно 88км. возвращаясь из В в А,он ехал поначалу с той же скоростью,но через один час пути вынужден был сделать остановку на 15мин. после этого он продолжил путь в А,увеличив скорость на 2км/ч,и в результате затратил на обратный путь столько же времени,сколько на путь из А в В. найдите скорость велосипедиста на пути из А в В.

2504. настя, 21 ноября 2010, 20:06:07
хелпми!
1.)вычислите:cos10П/3+sin1500
2.)постройте график функции y=sin(3x-П/2)+4
3.)постройте график функции y=-ctg(x-П/4)+2
4.)решите уравнение графически yx=1

2505. Леонид , 21 ноября 2010, 20:44:14
2503. Лена, х - ск. из А в В, то время 88/х.

Время обратно: 1ч + (88-х)/(х+2) + 1/4, т.к (1 ч пути, за час прошёл х, осталось (88-х) и скорость увеличена на 2,т.е. (х+2), а время (88-х)/(х+2), 1/4 ч - это 15 мин.). Уравнение: 88/х = 1 +(88-х)/(х+2) + 1/4. К общему знаменателю, раскрыв скобки, подобнык, получится уравнение:
88*4x + 88*4*2 - 88*4x +4x^2 = 5X^2 + 10X/ X^2 + 10X - 704=0, x=22. Проверяй, просматривая решение.

2506. dimageniy, 21 ноября 2010, 21:14:28
помогите пожалуйста
1)синусы углов треугольника образуют арифметическую прогрессию, причем наименьший из углов =10 градусов. чему равна величина одного из двух других углов округленного до целого числа градусов

2) произведение корней уравнения (x+2)*x^3=2*x+1

2507. Лена, 22 ноября 2010, 07:45:45
спасибо большое Леонид)))

2508. Леонид , 22 ноября 2010, 08:50:16
Для 2503. Лена, 21 ноября.
Просто подставляй данные в формулу: 12*2^-t/5,3>3, 2^-t/5,3>2^-2,
-t/5,3>-2 - т.к. показательная ф-я с основанием 2 большем 1 возрастающая.
t/5,3<2. t< 10,6. 10,6. ???

2509. Леонид , 22 ноября 2010, 16:56:50
LEXXUS, в записи 2056
Найти произведение корней уравнения (x+2)*x^3=2*x+1. У меня вопрос следующего плана. Решив уравнение - оно очень простое: x^4 + 2X^3 - 2x - 1=0. (x^4 - 1) + (2x^3 - 2x) = 0. (x^2 - 1)(x^2 + 1) + 2x(x^2 - 1) = 0/
(x^2 - 1)(x^2 + 1 + 2x) = 0. (x^2 - 1) (x + 1)^2 = 0. (x-1)(x+1)(x+1)^ = 0.
Корни: х = 1, х = -1 и х =-1. Те.е. два кратных корня! Вопрос: В произведение по заданию, брать все три корня (ответ: 1)? Или два корня, без учёта кратного (ответ:-1)? Как правильно поступать в таких ситуациях?
Я бы взял, два разных числа, являющихся корнями. Ответ: -1.
Или грамотность задания? Прошу разяснить, пожалуйста?
Например, мы говорим, что кв.ур-е с Д=0 имеет один или 2 равных корня, в ответ пишем одно значение. А здесь?

2510. Лена, 22 ноября 2010, 19:33:53
ПЛИИИЗ((
СИЖУ ТУПЛЮ ЧЕ ТО((

В12.Расстояние между пристанями А и В равно 48км. Отчалив от пристани А в 10ч утра,теплоход проплыл по течению реки с постоянной скоростью до пристани В.После трехчасовой стоянки у пристани В теплоход отправился в обратный рейс и прибыл в А в тот же день в 22.00ч.
Найдите скорость теплохода в неподвижной воде,ели скорость течения реки 4км/ч

в12
четыре рубашки дешевле куртки на 20%.на сколько процентов шесть рубашек дороже куртки?

2511. Лена, 22 ноября 2010, 19:43:07
четыре рубашки дешевле куртки на 20%
на сколько процентов шесть рубашек дороже куртки

2512. ilo, 22 ноября 2010, 21:59:43
настя, 21 ноября 2010, 20:06:07
хелпми!
1.)вычислите:cos10П/3+sin1500
2.)постройте график функции y=sin(3x-П/2)+4
3.)постройте график функции y=-ctg(x-П/4)+2
4.)решите уравнение графически yx=1
1) cos10pi/3=-1/2,sin1500-?,может 1500pi?, тогда 0
2) если можешь построить y=sinx, то его нужно сжать вдоль оси х в 3 раза, затем сдвинуть вправо на pi/2, и, наконец поднять вверх на 4 единицы.
3)строишь ctgx, сдвигаешь на pi/2 вправо, отображаешь симметрично оси х и поднимаешь на 2 единицы вверх
4)построить гиперболу у=1/х,решением будет любая точка,принадлежащая графику.

2513. Леонид , 22 ноября 2010, 22:24:28
Пусть цена куртки Х р. - 100%, цена 4 рубашек 80%, одной 80:4=20%, 6 рубашек 20*6=120%. По отношению к 100% куртки, цена 6 рубашек дороже 120%-100%=20% Ответ:20

2514. Лиза, 23 ноября 2010, 21:12:49
Пожалуйста решите, очень надо, срочно, плиииииз!

Ребро AD пирамиды DABC перпендикулярно плоскости основания ABC. Найдите расстояние от вершины A до плоскости, проходящей через середины ребер AB, AC и AD, если AD=2 корня из 5, AB=AC=10, BC=4 корня из 5.

2515. Леонид , 23 ноября 2010, 21:48:24
Искомое расстояние - это высота треугольника, вершинами которого являются середины ребер указанного в условии сечения. У этого треугольника стороны: корен из 5 (вершина - на середине АД),половина высота треугольника АВС, высота которого легко находится по т. Пифагора в его половинке, т.к. треугольник АВС - равнобедренный) и третья сторона - высота сечения - находится по т. Пифагора в половинке треугольника сечения). Искомое растояние находится из приравнивания формул площади треугольника: половина произведения осеования на высоту и полупроизведения катктов - треугольник, в котором имкомое расстояние - прямоугольный в вершине А. ЛИЗА, без чертежа сложно, но коль Вы взялись за эту задачу, внимательно прочитав предложенный ход решения, разберётесь и справитесь. АД - высота пирамиды, постройте сечение через середины и по описанию. УСПКХОВ. Рад, если смог помочь.

2516. Леонид , 23 ноября 2010, 21:54:23
Извиняюсь не совсем правильно выразился в первом предложении - искомое расстояние это высота треугольника, вершины которого:А, середина АД и середина высоты треугольника АВС из А. И далее по тексту выше.

2517. Лиза, 23 ноября 2010, 22:11:35
Что-то у меня совсем не сходится все с чертежом(

2518. Леонид , 23 ноября 2010, 22:38:23
Обознач, наприме: К - середина АД, М - середина АС, Р - середина АВ - соедини эти точки - сечение по условию. Проведи высоту в треуг. АВС из А , она проходит через саредину ВС - точка Е. Эта высота пересекает МР в её середине Т. Треугольник получили КАТ - его высота из А на КТ и есть искомое расстояние. АК= корню из 5, ТР= 1(половина МР), АР=5 - находи по т. Пифагора АТ. Далее, в треуг АКТ находи КТ. Всё известно в треуг. АКТ - прямоугольный. Площадь: 1/2АК*AT=1/2KT*на искомое расстояние (перпендикуляр - высота из А к КТ).

Чертежи, здесь мы чертить не можем

2519. ilo, 23 ноября 2010, 23:07:06
Леонид , 23 ноября,
TP=sgrt5?

2520. Лиза, 23 ноября 2010, 23:15:18
да-да, я тоже заметила)

2521. Леонид , 23 ноября 2010, 23:28:15
Лиза я вычисления не проводил. Похоже, прикидкой, что так. По критериям, если верные чертёж и ход решения, при незначительной вычислительной ошибке поставят 2 из 2 баллов. Посмотри ещё разок своё решение, проверь. У нас уже поздно - 2 ночи

2522. Лиза, 23 ноября 2010, 23:30:52
все равно спаисибо большое, очень помогли!
я решила, у меня ответ странный получился, но вроде так и должно быть)
а где вы живете, что у вас 2 часа ночи?

2523. Леонид , 24 ноября 2010, 08:33:18
Кузбасс

2524. ilo, 24 ноября 2010, 08:58:38
Лиза, 23 ноября 2010,у меня ответ странный получился
У Корянова приводится ответ к этой задаче 2. Высоту прямоугольного треугольника можно найти по формуле h=ab/c, где a=sgrt5, b=2sgrt5-катеты, с=5-гипотенуза.

2525. Леонид , 24 ноября 2010, 11:16:33
2524. ilo, 24 ноября. Не оспариваю, как Вы написали о нахождении высоты прямоугольного треугольника, но это надо помнить. Запомнить всего невозможно.
Я писал тоже самое, но по чертежу 1/2 ав=1/2сh. Согласны? Приём сравнения! Ещё приём:вводим "вспомогательную" переменную. Находим одно и тоже с "двух сторон", приравниваем, находим переменную - открыт путь ко всему решению!!!
Где срабатывае принцип решения геометрических "ищи треугольник". Если известно 3 его элемента - разрешима любая задача. Т.К. о треугольнике мы больше всего знаем. Очеь важно знать о приёмах.

2526. Kitovik, 24 ноября 2010, 18:17:53
Пожалуйста помогите ответь на олимпиаду по информатике КИТ 7класс,
Вот на картинке вопросы.... Это олимпида очень важна для меня: С 1-9 я сделал.
10- Анаграммы - это головоломки, которых переставляются бквы в словах Расшифруйте и Выберите лишнее слово...
А) нерсак Б) тренпир В) моторин Г) туравиакла Д) кокните
12. Кроссворд.
2-Указательное устро-во ввода информации об относительном перемещении для компьютера--??
4-устройство печати цифровой информации на твердый носитель.
13-Для шифровки каждой буквы слова используются двузначные числа. Известно что буква "к" кодирована числом 15. Среди слов "торт", "ежик", "станок", "радуга" есть слова, кодируемые последовательностью цифр : 35291815, 303113241115. Какая последовательность цифр является кодом слова "китенок"?
А)151831241115 Б) 15181211243515 В) 15183135241115 Г) 151831153524 Д) 15181135243015

2527. Игорь, 24 ноября 2010, 18:38:00
Привет Леонид.как дела?Подсоби пжл...

В11.Найдите наибольшее значение функции.
у=4tgx-4x+pi-7 на отрезке [-pi/4;pi/4].

2528. Леонид , 24 ноября 2010, 19:29:41
1)y* = 4/cos^2 x - 4. 2) 4/cos^2 x - 4 = 0, cos^2 x = 1, cos x =1 или cos x=-1
x = 2pi k или x = pi + 2pik соответственно. Какие из них попадают в
[-pi/4;pi/4] ? x = 0 - из х=2pik при k=0.

Находим зн-я ФУНКЦИИ на концах промежутка и в точках ему принадлежащих

1)y(0) = pi -7 - не для бланка ответов; 2) y(-pi/4) = -4 +pi + pi - 7 = -10 + 2pi - можно и не считать - не для бланка ответов; 3)y(pi/4)= = 4 - pi + pi -7 = -3. Ответ: -3 - наибольшее.

вообщето видно, что призводная неотрицательная, то ф-я возрастающая. Значит наибольшее значение в правом конце промежутка.

на эту тему запись № 2491 и 2492 и раньше были...ты здавал вопрос откуда синус взялся запись 2233 на мою опечатку тебе советовали не учить таких формул. Припомни всё, почитай , будешь смело решать такие задания.
А дела - нормально.

2529. Леонид , 24 ноября 2010, 19:45:25
Опечаточка не 2233, а 2216 и 2220

2530. Лилия, 24 ноября 2010, 21:10:17
Спасибо за решение задачи с плоскостями!
Lexxus, помогите пожалуйста решить задание:
Решите уравнение:x^2+(a-6)^2=модуль выражения x-a+6 плюс модуль выражения x+a-6.

2531. Deni, 25 ноября 2010, 01:12:50
Уважаемый Lexxus! Очень хорошо, что разместили прототипы! Но при решении прототипов В12 выскакивают другие задания, в основном из недавно размещенных. Проверить легко по номеру задания. И именно при решении...просмотр заданий выдает прототипы...
Удачи!

2532. Фаридка, 25 ноября 2010, 11:14:09
(время 3ч20мин - 3 1/3 часа или 10/3) а как вот это определять? а если 1час 24минуты как представить?

2533. Леонид , 25 ноября 2010, 12:04:27
Либо что - то с задачей, либо с решением у Вас, либо решать с 10/3 - в первом случае. Во втором: 2ч + 24мин = 2ч + 24/60ч (минута 1/60 часа) = 2 + 2/5 (сократили на 12) = 2 + 4/10 = 2,4

2534. ксюша, 26 ноября 2010, 18:03:50
синусы углов треугольника образуют арифметическую прогрессию,причем наименьший из углов равен 10 градусов. чему равна величина одного из двух других углов, округленная до целого числа градусов?

2535. ксюша, 26 ноября 2010, 18:11:11
всем привет,помогите пожалуйста решить(

2536. Инга, 27 ноября 2010, 22:09:37
Помогите, пожалуйста решить:
4tg&#8289;&#12310;х/2&#12311; +2 tg&#8289;&#12310;х/4&#12311; +8 ctg&#8289;х=tg&#8289;&#12310;х/12&#12311;-tg&#8289;&#12310;х/8&#12311;

2537. Леонид , 27 ноября 2010, 22:58:11
это что такое? # - № ? 7 = & ?

2538. Фии, 28 ноября 2010, 21:37:28
Из пункта А в пункт В, расстояние между которыми 30 км, одновременно выехали автомобилист и велосипедист. Известно, что в час автомобилист проезжает на 100 км больше, чем велосипедист. Определите скорость велосипедиста, если известно, что он прибыл в пункт В на 1 час 15 минут позже автомобилиста. Ответ дайте в км/ч.


Прошу решить

2539. ilo, 29 ноября 2010, 15:30:40
Фии, 28 ноября
x-скорость велосипедиста,х+100-автомобилиста,уравнение:30/х-30/х+100=5/4;
квадратное уравнение будет X^2+100x-2400=0; x=20;Ответ:20.

2540. «КриSтин@», 29 ноября 2010, 18:56:46
люди, как посмотреть, правильно ли я решила?? ((

2541. Леонид , 29 ноября 2010, 19:19:23
Маленькая поправочка: x-скорость велосипедиста,(х+100) -автомобилиста,уравнение:30/х-30/(х+100)=5/4;
квадратное уравнение будет х^2+100x-2400=0; x=20;Ответ:20.

2542. ilo, 30 ноября 2010, 16:29:23
Лилия, 18 ноября
Kонтрольными будут точки х=а-6 и х=6-а
Т.к. справа неотрицательное выражение, то рассмотреть те случаи, когда левая часть неотрицательна, т. е. произведение (x-(a-6))(x+(a-6)) неотрицательно, если множители одного знака или хотя бы один равен 0.А это есть подмодульные выражения, т.е. рассмотреть случаи, когда подмодули имеют одинаковые знаки или хотя бы один из них равен0, т.е. для случаев a<6; a>6 или a=6. При х<a-6 или x<6-a подмодули отрицательны, при x>6-a или x>a-6 оба подмодуля положительны. В 1 случае уравнение имеет вид x^2+2x-(a-6)^2=0; во втором x^2-2x-(a-6)^2=0;

2543. Игорь, 30 ноября 2010, 22:42:09
Привет Леонид, помоги решить.

Вычислить:
2^3+log числа 9 по основанию 2
(sqrt7):4+log числа 0.5 по основанию sqrt7
6^-3log числа 2 по основанию 6
5^-2log числа 3 по основанию 5

Решить уравнения:
lg x=1
lg x=-2
lg x=3
lg x=-4
log x по основанию 9=1/2
log x по основанию 8=1/3

Заранее спасибо!

2544. евгения, 1 декабря 2010, 09:10:08
Можете пожалуйста пояснить,,,открытый банк---значит какое либо задание из предложенных(в банке) будет на экзамене егэ????? я правильно понимаю???????

2545. Леонид , 1 декабря 2010, 14:55:37
2^3 *2^(log9по осн.2) =(показатели степени сложены, записади в виде произведения ст.)= 8 (возвели)*9( по основному логарифмическому тождества) = 72
2-е. не понял
3-е. (6^log 2 по осн.6)^-3 (коль показатели перемножены, записали степень в степени) = (основное лог. тождество) и = 2^-3 = 1/(2^3) = 1/8 = 0,125
4-e. x=10^1 (по определению логарифма) =10.
5-е. x = 10^3 =1000.
6-e. x = 10^-4 = 0,0001.
7-e. x=9^(1/2) = 3.
9-e. x=8^(1/3) = 2.

2546. Александра, 2 декабря 2010, 18:27:14
Здравствуйте!Подскажите, пожалуйста, как решить эту задачу: В равнобедренном треугольнике ABC угол при вершине В равен 120 градусов. Расстояние от точки М, лежащей внутри треугольника, до основания треугольника равно 1/корень из 3,а до боковых сторон равно 3. Найдите АС.

2547. ilo, 2 декабря 2010, 22:59:18
т.к. расстояние от М до боковых сторон равны - М лежит на биссектрисе,мед.,выс., проведённой к АС. BM=3/sin60=2sgrt3. BH=7/sgrt3.1/2AC=7/sgrt3*tg60;AC=14.

2548. Леонид , 3 декабря 2010, 00:10:46
2544. евгения, 1 декабря . Да, женя, от туда и будут.

2549. Мария, 5 декабря 2010, 22:48:25
Помогите решить, пожалуйста!


В треугольнике АВС АВ=8, угол А=углу В, cos А=45. Найдите биссектрису СН.

2550. Эльдар, 6 декабря 2010, 13:45:15
теплоход проходит от пристани A до пристани B ро течении реки за 3ч, а против теченияза 4 часа. За сколько часов проплывает это растояние плот?
помогите пожалуйста очень прошу.....

2551. Lexxus, 6 декабря 2010, 14:50:48
теплоход проходит от пристани A до пристани B ро течении реки за 3ч, а против теченияза 4 часа. За сколько часов проплывает это растояние плот?

Смотри.
Предположим, S - это расстояние от A до B, Vt - собственная скорость теплохода в стоячей воде, Vr - скорость течения реки.

Нам известно, что:
а) S/(Vt+Vr) = 3
б) S/(Vt-Vr) = 4

А найти нам надо
S/Vr (плот плывет со скоростью течения реки).

Решать удобно так: "перевернем" уравнения а) и б):
(Vt+Vr)/S = 1/3, (Vt-Vr)/S = 1/4

Вычтем второе из первого:
(Vt+Vr-Vt+Vr)/S = 1/3 - 1/4 = 1/12
2Vr/S = 1/12
Vr/S = 1/24

Значит, S/Vr = 24
Ответ - 24 часа.

2552. Яночка, 7 декабря 2010, 16:58:55
помоги пожайлуста срочно

синусы углов треугольника образуют арифметическую прогрессию,причем наименьший из углов равен 10 градусов. чему равна величина одного из двух других углов, округленная до целого числа градусов?

2553. dimageniy, 7 декабря 2010, 20:10:03
фига се че у вас досих пор математический чемпионат решают?


а ответ 151

2554. Игорь, 7 декабря 2010, 23:42:26
Привет Леонид, спасибо за помощь!
Вот примерчик, не знаю как решать,обьясни...

Найдите значение выражения 8sin 5π/12*cos 5π/12

2555. Lexxus, 8 декабря 2010, 10:35:30
Найдите значение выражения 8sin 5&#960;/12*cos 5&#960;/12

Не тупи. Готовая формула синуса двойного угла.

2556. Наталья, 8 декабря 2010, 12:55:33
помогите решить задачу: Волнистый попугай дешевле хохлатого на 37, 5%. На сколько процентов хохлатый попугай дороже волнистого?

2557. Lexxus, 8 декабря 2010, 13:12:17
помогите решить задачу: Волнистый попугай дешевле хохлатого на 37, 5%. На сколько процентов хохлатый попугай дороже волнистого?

Помогаю:

(волнистый) = (хохлатый)*(1-37.5/100)
(хохлатый) = (волнистый)*(1+x/100)

2558. Леонид , 8 декабря 2010, 16:19:43
Найдите значение выражения 8sin 5&#960;/12*cos 5&#960;/12 = 4*2sin 5pi/12*cos 5pi/12 = 8*sin 2* 5pi/12 = 8*sin 5pi/6 = 8sin(pi - pi/6) = 8sin pi/6 = 8* 1/2 = 4.
Игорь, как дела учебные?

2559. Ириска, 8 декабря 2010, 17:39:45
В прямоугольном параллелепипеде `ABCDA_1B_1C_1D_1` известны ребра: `AB=35`, `AD=12`, `CC_1=21`. Найдите угол между плоскостями ABC и `AD1B
Ребят, помогите пожалуйста!


2560. Ириска, 8 декабря 2010, 17:47:04
*ABC B A1DB опечаталась..

2561. Lexxus, 8 декабря 2010, 17:52:42
В прямоугольном параллелепипеде `ABCDA_1B_1C_1D_1` известны ребра: `AB=35`, `AD=12`, `CC_1=21`. Найдите угол между плоскостями ABC и `AD1B

Поскольку у нас прямоугольный параллелепипед, то и AD, и AD1 перпендикулярны AB. Поэтому искомый угол - это, например, угол D1AD.
Он равен
arctg(DD1/AD) = arctg(CC1/AD) = arctg (21/12).

2562. Lexxus, 8 декабря 2010, 17:59:27
*ABC B A1DB опечаталась..

Другое дело. А то какая-то уж больно простая задачка.

Тут уже лучше рисунок нарисовать. Сейчас убегаю, попозже, если не забуду.

Но в двух словах - опускаем перпендикуляры к DB из A и A1, доказываем, что они попадают в одну точку H, находим их длины из треугольников ADB и A1DB соответственно (все их стороны мы можем найти по теореме Пифагора), а затем находим угол A1HA из одноименного треугольника.
Это и будет искомый угол между плоскостями.

2563. Ириска, 8 декабря 2010, 18:10:17
Lexxus,огромное спасибо,вы мне очень помогли!!

2564. Ириска, 8 декабря 2010, 18:52:25
Но все равно я не могу их длины найти...у меня наверно скоро мозги сломаются от 6 часов геометрии(

2565. Laz, 8 декабря 2010, 19:28:48
Помогите пожалуйста решить Б11
найдите наибольшее значение функции
y= 2cos x+ корень из 3 x - корень из П/ 3
на отрезке [0; п/2]

2566. Laz, 8 декабря 2010, 19:31:24
извиняюсь я устранил ошибку
помогите пожалуйста решить Б11
найдите наибольшее значение функции
y= 2cos x+ корень из 3 *x - корень из3 П / 3
на отрезке [0; п/2]

2567. ilo, 8 декабря 2010, 21:07:39
Laz, 8 декабря
производная ф-ции = -2sinx+sgrt3; sinx=sgrt3/2; x=п/3-точка, принадлежащая [0;п/2] подозрительная на экстремум.
y(0)=2-sgrt3*п/3<0;
y(п/3)=2*1/2+sgrt3п/3-sgrt3п/3=1;
y(п/2)=0+sgrt3п/2-sqrt3п/3=sqrt3п/6<1;
Отв.: yнаиб.=y(п/3)=1.

2568. евгений, 8 декабря 2010, 23:08:20
добрый вечер леонид. вы поясняли задачу. вот:2541. Леонид , 29 ноября 2010, 19:19:23
Маленькая поправочка: x-скорость велосипедиста,(х+100) -автомобилиста,уравнение:30/х-30/(х+100)=5/4;
квадратное уравнение будет х^2+100x-2400=0; x=20;Ответ:20.

объясните пожалуйста откуда взялись оба уровнения?

2569. евгений, 8 декабря 2010, 23:10:16
ну или ilo поясни пожалуйста

2570. Людмила, 8 декабря 2010, 23:37:16
6. Игорь Найдите площадь S круга. В ответе укажите S/п. Размер каждой клетки 1см на 1см.Ответ дать в квадратных сантиметрах.

Я нашел радиус. R=3
S= пR^2
S=п3^2
S=п9
В ответе укажите S/п
п9/п=9 п сокращаем. значит площадь круга равна 9.
А в книжном ответе 8
Леонид, радиус круга который взят на рисунке приблизительно 3 но точное значение наверно 2.8 и в квадрате 7.84 но все равно 8 никак не получается.
Что скажешь по данному примеру?

Ответ: Рассмотри прямоугольный треугольник, в котором радиус является гипотенузой , а катеты - точное количество клеточеки по теореме Пифагора найди его

2571. Людмила, 8 декабря 2010, 23:43:28
Мария, 5 декабря 2010, 22:48:25
Помогите решить, пожалуйста!


В треугольнике АВС АВ=8, угол А=углу В, cos А=45. Найдите биссектрису СН.

Треугольник равнобедренный, поэтому СН является и высотой. Теперь рассмотри прямоугольный треугольник АСН

2572. xxx, 9 декабря 2010, 00:10:03
в треугольнике АВС АВ=ВС,АС=5,sin С=0.6,СH-высота.Найдите AH.

2573. ilo, 9 декабря 2010, 09:08:03
евгений, 8 декабря 2010,ну или ilo поясни пожалуйста
Оба прошли путь 30км.Время, за которое они это сделали определяем разделив пройденный путь на соответствующую скорость.Велосипед ехал медленнее(скорость у него меньше, а время больше), находим на сколько 1ч15мин выражаем в ч:1ч 15 мин=1ч+1/4ч=5/4ч

2574. ilo, 9 декабря 2010, 09:22:15
евгений, 8 декабря
квадратное ур-е получается так:дополнительный множитель к первой дроби 4х+400, ко второй 4х, к третьей х^2+100x; 120х+12000-120х-5х^2-500х=0;после приведения подобных имеем то, что имеем

2575. ilo, 9 декабря 2010, 09:38:26
xxx, 9 декабря углы А и С равны, косинусы тоже AH=ACcosA, cosA=0,8

2576. лёля, 9 декабря 2010, 10:50:52
помогите решить пожалуйста
Найдите косинус а ,если синус а =2корень 65 и а принадлежит (п2;п)

2577. Игорь, 9 декабря 2010, 11:45:34
Леонид, учеба нормально.ты как?
Помоги решить в11.
Нужно найти наибольшее значение функции у=12tgх-12х+3*п-6 на отрезке [-п/4,п/4]
срочно нужно:)

2578. Леонид , 9 декабря 2010, 16:31:55
ПУТЁМ! Игорю. Уже были такие задания у нас с тобой,- пока только план решения:
1) находим производную функции;
2) приравниваем производную к 0 и решаем уравнение;
3) среди найденных корней в этом уравнении выбираем те, которые входят в данный отрезок;
4) находим значения функции в концах отрезка и выбранных - входящих в этот отрезок (подстановкой в данную функцию);
5) выбирем (здесь) наибольшее в пункте 4 - это ответ.
делай, в чём ещё проблемы в таких заданиях. Игорь, все В11 такие и найти точку максимума (минимума). Только числа и функуии разные. Усваивай - пора с тами заданиями чувствоваить себя "АССОМ"

2579. Раула, 9 декабря 2010, 19:33:36
здравствуйте) Леонид, помогите пожалуйста. завтра сдавать, а я абсолютно не знаю как решать...
обычно делаю без особых проблем, а тут как ступор...
1) С1
решить неравенство ( х^3 * 5^ log числа х по основанию 2) / (х^log числа 2^[ по основанию 6) < (х^log числа 2^х по основанию 6)
2) С2
Дан шар радиусом 7 с центром в точке О и два его сечения с центрами в точках О1 и О2. Точка О лежит на отрезке О1О2, длина которого равна 6. площадь одого из сечений в 2 раза больше площади другого.Найдите радиус меньшего сечения.

2580. Леонид , 9 декабря 2010, 21:40:08
(х^log числа 2^[ по основанию 6) < (х^log числа 2^х по основанию 6) - в этой части непонятна запись 2^[ .

Проьую С2 - Сечения параллельны. В сечении шара, пусть точка М - точка большего сечения на прверхности шара, К - такая же - меньшего сечения.
Площадь S(O1) = 2S(O2), т.е. pi r1^2 = 2 pi r2^2, r1^2=2 r2^2.
Из треуг ОО1М r1^2 = 49 - x^2, где О1О = х. Из треуг. ОО2К r2^2 = 49 -(6-x)^2. Уравнение: 49-x^2 = 2*(49 - (6-x)^2) - найдём х и легко радиус любого сечения. Это 1-й случай.

Случай 2-й - на рассмотреть. когда данные сечения не параллельны. Без чертежа не могу понятным сделать решение. А чертить мы здесь не можем.
И ещё, могут ли М и К совпасть? Исследовать надо. Ну вот, что смог.


2581. Леонид , 9 декабря 2010, 21:48:17
Да! Коль все три центра на одном отрезке, случая непараллельности ссечений не может быть, не может быть тогда и совпадения М и К. Если я верно понял задачу.

2582. Игорь, 9 декабря 2010, 21:51:46
Леонид, а у меня наибольшее 6 получилось.

y'=12/cos^2x-12
y'=0 12/cos^2x-12=0
cos^2x=1
cos x=sqrt1
cos x=+-1
y(-pi/4)=12 tg(-pi/4)-12(-pi/4)+3pi-6=-12-6=-18
y(0)=12 tg0-12*0+3pi-6=3pi-6=3.42
y(pi/4)=12 tg pi/4-12pi/4+3pi-6=12-6=6
Ответ: 6 наибольшее.

Леонид где я ошибся?

2583. Раула, 9 декабря 2010, 22:44:02
Леонид,
логарифм числа 2 по основанию 6, извините, заучилась
у меня получился ответ Х> 4
но как то сомнительно

а во втором я сделала, но что то ответ больно неподходящий корень квадратный из 24

но я делала так, что r1^2 =R^2 - x^2 , тк r2^2=2* R1^2, то я просто подставила и получилось
2(R^2-x^2)=^-(6-x)^2

2584. Леонид , 9 декабря 2010, 23:21:18
ошибочка в Y(-pi/4) = -12+ 3pi + 3pi - 6 - не идёт в ответ.
вторая строка 3пи - 6 не идёт в ответ
послкднее верно 6 - это и ответ. Смотри при двух выч. ошибках отыет верный - это случайность.
ты кула дел пи вр 2-м случае? В 1-м случае минус потерял.
НО Алгоритм решения верный.

2585. Леонид , 9 декабря 2010, 23:37:52
2583. Раула - смотри моё решение, у тебя одно уравнение, а 2 неизвестных величины. Первое лучше напиши внимательно поновой

2586. Юлия, 10 декабря 2010, 13:44:07
ПОМОГИТЕ РЕШИТЬ ЗАДАЧУ : В равнобедренном треугольнике АВС с основанием АС ,боковая сторона АВ равна 8см,а высота ,проведённая к основанию равна 3 корня из 7. Найдите косинус угла А

2587. Леонид , 10 декабря 2010, 18:03:37
Можно так: найдите по т. Пифагора половину основания AB^2 - BH^2 = 64 - 63 = 1. cos A = AH:AB = 1:8 = 0,125, где ВН - высота к основанию АС и все дела.

2588. Юлия, 10 декабря 2010, 20:47:08
Ой спасибо вам большое Леонид))))))))))))))

2589. Юлия, 10 декабря 2010, 20:58:59
Как решить вот эту задачу:коэффициент полезного действия некоторого двигателя определяется формулой Т1-Т2:Т1*100%. При каком наименьшем значении температуры нагревания КПД этого двигателя будет не менее 37,5%,если температура холодильника Т2=270

2590. Юличка________, 11 декабря 2010, 14:43:01
сначало правильно оформи:) T1-T2/T1 *100%
(T1-270)*100/T1=37,5
решаем уравнение
100T1-2700=37.5T1
62.5T1=27000
T1=432
ответ : 432

2591. Леонид , 11 декабря 2010, 15:27:08
что же правильно - со скобками в решении и без скобок в условии.
решение предложено, если в условии (T1 - T2)* 100/T1, но ведь записано не так запрашиваемым. И с действиям умножения и деления ??? Какое же услови?

2592. Юлия, 11 декабря 2010, 16:34:50
Спасибо большое правильный ответ)))))) Сможите подсказать вот это: В прямоугольном треугольнике АВС угол С =90градусов,АВ =корень из 13,АС=3.Найти ctg

Ещё вот эту :Велосепедист выехал с постоянной скоростью из города А в город В,расстояние между которыми равно 108 км. На следующий день он отправился обратно со скоростью на 3 км/ч больше прежней . В пути он сделал остановку на 3часа и в результате затратил на обратный путь столько же времени,сколько на путь из А в В. Найдите скорость велосипедиста на пути из А в В

2593. Леонид , 11 декабря 2010, 18:15:15
х- от А до В, обратного пути ск. (х+3).
Время от А до В 108/х. Время от В до А. 108/(х+3) и плюс 3 ч. Уравнение:
108/х = 3 + 108/(х+3), 108(х+3) = 3х(х+3) + 108х, 3х кв. + 9х - 324 = 0,
х кв. + 3х -108 = 0, х=-12 и х=9. Ответ:9
Задача 1) ctg ???

2594. Лена, 11 декабря 2010, 19:18:53
Помогите решить из В4:
В треугольнике АВС угол С=90гр., СН - высота, АВ=12, tg=2/3. Найдите АН.

2595. Юлия, 11 декабря 2010, 22:19:25
Леонид да ctg.

2596. Леонид , 11 декабря 2010, 22:35:34
Понятно, что котангенс. Но какого угла?

2597. Юлия, 12 декабря 2010, 13:05:37
ой точно угла А

2598. Леонид , 12 декабря 2010, 13:47:03
Что значит tg=2/3 Какого угла? Ленка, ... ???


2597. Юлия, 12 декабря 2010, 13:05:37
ой точно угла А

По т. Пифагора катет ВС^2 = AB^2 - AC^2 = 13 - 9 = 4. BC = 2. ctgA= AC/BC= 3/2 = 1,5

2599. Игорь, 12 декабря 2010, 18:15:10
Лнонид, я когда решал на бумаге все было правильно, тебе печатал по памяти на спех,помню что ответ 6 при у= (п/4)

2600. Леонид , 12 декабря 2010, 18:28:58
Ответ верный, коль советуешься не спеши печатать

2601. настя, 12 декабря 2010, 19:48:01
помогите решить неравенство sin2x<1/2

2602. Юлия, 12 декабря 2010, 20:30:37
вот это задание совсем не понимаю: Зависимость температуры (в градусах Кельвина) от времени (в минутах) для некоторого прибора была получена экспериментально и на исследуемом интервале температура задаётся выражением: Т =Т нулевое +а*t+b*t^2 .где Т нулевое= 296К, а=5 К/мин,b= -1/8 К/мин^2. Известно ,что при нагреве прибора свыше 338 градусов,он может выйти из строя,поэтому его нужно отключить. Определите .через какое время в минутах после начала работы нужно отключить прибор.

2603. Леонид , 12 декабря 2010, 21:34:05
296 +5t -1,8t^2<=338, решайте это квадратное неравенство.
1,8t^2 - 5t +42>=0

2604. ilo, 13 декабря 2010, 00:17:07
настя, 12 декабря
-7pi/6<2x<pi/6; -7pi/12<x<pi/12;

2605. ilo, 13 декабря 2010, 00:39:41
Лена, 11 декабря
Посмотри пост 2468 и определись с углом для tg.

2606. ilo, 13 декабря 2010, 08:24:30
настя, 12 декабря
добавить+ 2pin в обеих частях неравенств

2607. Юлия , 13 декабря 2010, 13:56:47
так там корень из дискрименанта не получается

2608. настя, 13 декабря 2010, 14:34:10
ilo, спасибо большое, у меня почему то получалось в левой части 5рi/6(((и поэтому дальше все решение было не верным!если не трудно то решите пожалуйста еще такое уравнение( или хотя бы напишите ответ)
дробь, в числители cos5x(1-корень из двух*sin3x), в знаменателе sin 3x. вся дробь равна нулю.(для уточнения, в числителе под корнем только 2, а sin3x без корня) буду очень признательна и благодарна за решение))

2609. Ириска, 13 декабря 2010, 19:39:27
в правильной четырехугольной пирамиде sabcd, все ребра которой равны 1, найдите расстояние от середины ребра sb до плоскости scd

2610. ilo, 13 декабря 2010, 20:44:13
настя, 13 декабря у меня почему то получалось в левой части 5рi/6
точки5pi/6 и -7pi/6 на тригонометрическом круге совпадают,а при записи промежутка слева должно быть записано меньшее значение-примитивно можно объяснить так
Из условия равенства дроби 0 следуют уравнения:cos5x=0;1-sqrt2sin3x=0;
1-oe: 5x=pi/2+pin; x=pi/10+pin/5;2-oe: 3x=pi/4+2pin или 3x=3pi/4+2pin; x=pi/12+2pin/3 или x=pi/4+2pin/3;x не должен быть равен pin/3, ни один из этих корней с этими точками не совпадает, поэтому в ответе все три группы корней

2611. Lexxus, 13 декабря 2010, 21:23:15
в правильной четырехугольной пирамиде sabcd, все ребра которой равны 1, найдите расстояние от середины ребра sb до плоскости scd




Нам нужно найти расстояние от точки К до плоскости SCD.
Пусть точка L - середина ребра AS. Поскольку пирамида правильная, то расстояние от L до плоскости SCD также равно искомой величине.
А значит, прямая KL параллельна плоскости SCD, и от любой её точки (включая точку K) до этой плоскости расстояние одинаково.

Пусть SN - высота треугольника SDC, а SM - высота треугольника SAB.
Поскольку все ребра пирамиды равны, то эти треугольники правильные, и их высоты равны
SN = SM = sqrt(3)/2

Точка P является серединой отрезков SM и LK.

Искомое расстояние будет равно длине перпендикуляра PQ, опущенного из P к SN.

Из равнобедренного треугольника SNM по теореме Пифагора находим угол NSM, который будет равен 2arcsin(1/sqrt(3)).

Из прямоугольного треугольника SPQ, зная, что его гипотенуза SP равна половине SM, т.е. sqrt(3)/4, находим:
PQ = SP*sin(NSM) = (sqrt(3)/4)*sin(2*arcsin(1/sqrt(3))) =
= sqrt(3)/4*1/sqrt(3)*sqrt(1-(1/sqrt(3))^2) =
= sqrt(3)/4*1/sqrt(3)*sqrt(2)/sqrt(3) =
= sqrt(2)/2/sqrt(3) = 1/sqrt(6)

Ответ: 1/sqrt(6)
(один на корень из шести)

2612. Ириска, 13 декабря 2010, 21:56:38
спасибо огромное,Lexxus!!

2613. АнтонЯ, 13 декабря 2010, 22:23:07
У меня завтра экзамен сможет мне кто нибудь помочь?

2614. настя, 14 декабря 2010, 00:36:58
ilo,спасибо вам большое за помощь=))очень вам признательна! удачи вам=)

2615. Kattu, 14 декабря 2010, 10:13:40
Здравствуйте Lexxus! В открытом банке опять добавили задания, их там уже 63182. Точно добавили В8 и В11. Если можно, добавьте их сюда! В номерах 7075, 6993, 7945 ошибки в ответах. Исправьте их пожалуйста! Спасибо!

2616. Елена, 14 декабря 2010, 15:42:39
Помогите пожалуйста с заданиев B4 -
В треугольнике ABC AC=BC, AB=72, cosA=12/13. Найдите высоту CH

2617. Надежда, 14 декабря 2010, 21:04:16
Решите пожалуйста :)
№1) Стороны основания правильной шестиугольной пирамиды равны 6, боковые ребра равны 5. Найдите площадь боковой поверхности пирамиды.

№2) Найдите площадь боковой поверхности правильной шестиугольной призмы, сторона основания которой равна 2, а высота 6

2618. ilo, 15 декабря 2010, 09:26:11
Елена, 14 декабря
CH=AHtgA;AH=36; tg^2(A)=1/cos^2(A)-1; tg(A)=5/12;CH=15.

2619. Фии, 15 декабря 2010, 14:20:44
Фии, 28 ноября 2010, 21:37:28
Из пункта А в пункт В, расстояние между которыми 30 км, одновременно выехали автомобилист и велосипедист. Известно, что в час автомобилист проезжает на 100 км больше, чем велосипедист. Определите скорость велосипедиста, если известно, что он прибыл в пункт В на 1 час 15 минут позже автомобилиста. Ответ дайте в км/ч.


Прошу решить


2541. Леонид , 29 ноября 2010, 19:19:23
Маленькая поправочка: x-скорость велосипедиста,(х+100) -автомобилиста,уравнение:30/х-30/(х+100)=5/4;
квадратное уравнение будет х^2+100x-2400=0; x=20;Ответ:20.

у мня не получается такое квадратное уравнение, не моглм бы рассписать? очень прошу!

2620. Фии, 15 декабря 2010, 14:25:13
Решенные. B6. №22171. не получается решить, помогите кто нибудь

2621. любашка, 15 декабря 2010, 15:01:43
Помогите пожалуйста решить задачу(((.....Найдите боковое ребро правильной четырёхугольной призмы,если сторона ее основания равна 20,а площадь поверхности равна 1200???????помогите...

2622. любашка, 15 декабря 2010, 15:10:50
????ПОМОГУТ НЕТ???

2623. Леонид , 15 декабря 2010, 15:23:11
Sполн = Sбок. + 2Sосн. 1200 = Sбок +2*20*20, Sбок. = 400. Sбок.= =Росн.*H, 400 = 4*20*H, 400 = 80*H, H = 5. Можно и по дргому решить.

2624. Леонид , 15 декабря 2010, 15:27:34
№22171 Там треугольник с основанием 2 и высотой 5. Площадь 1/2*2*5 = 5

2625. ilo, 16 декабря 2010, 08:49:52
Фии, 15 декабря
посты 2573, 2574.

2626. Игорь, 16 декабря 2010, 18:13:51
Привет Леонид, как дела?Вот задание не знаю ка решить, помоги....

В9. В треугольнике АВС угол А равен 30 градусов, сторона ВС равна 10 см, Н-точка пересечения высот.Найти радиус окружности,описанной около треугольника ВСН.

2627. Леонид , 16 декабря 2010, 20:03:18
Привет, Игорь! Задачу написал правильно? В9 - задачи стереометрическе - где взял эту. Скдя по условию, ожет быть несколько случаев. Треугольник АВС - равнобедренный, тупоугольный, остроугольный и даже прямоугольный.

2628. Игорь, 16 декабря 2010, 22:48:10
Эту взял из своего сборника, там свободные задания идут вот и это попалось.Ладно забудь.

Найти наибольшее значение функции y=4cos x-21/pi*x+9 на отрезке [-2pi/3,0]

Леонид, у меня наибольшее 13. А у тебя?

2629. Феликс, 16 декабря 2010, 22:55:52
помогите плииизз...
в правильной четырехугольной пирамиде высота равна 6, боковое ребро равно 10. Найдите объем

2630. Игорь, 16 декабря 2010, 23:06:19
Найти область значений функции y=2x-1(дробь)x+1

2631. Леонид , 17 декабря 2010, 09:35:34
2628. Игорь, 16 декабря 2010, 22:48:10
Ошибочка где-то, у меня 21 Находили значение функции в -2пи/3, у =-2+14+9
В другом задании не понял запись функции


2632. Игорь, 17 декабря 2010, 11:34:43
у=2х-1/х+1

2633. Игорь, 17 декабря 2010, 11:40:54
Леонид, ошибку нашел.спасибо:) да 21 получается

2634. Леонид , 17 декабря 2010, 12:49:23
обл. опред. х не равен 0,т.е. (-беск.;0)U(0;+беск.)

2635. ilo, 17 декабря 2010, 19:08:13
Феликс, 16 декабря
(1/2d)^2=100-36=64; 1/2d=8; d=16-диагональ основания; d=sgrt2*a; a-сторона основания; a=16/sgrt2; V=1/3a^2*h; h-высота пирамиды V=1/3*256/2*6=256куб.ед

2636. Игорь, 17 декабря 2010, 22:00:14
Леонид, спасиб.

(9-x^2)*sqrt2-x-x^2=0

2637. Катя, 17 декабря 2010, 23:24:23
Я новенькая тут подскажите где можно найти решение к заданию...И есть ли решение вообще?

2638. сергей, 18 декабря 2010, 20:01:13
в треугольнике авс угол с равен 90 ас=25 тангенс угла а =4/5 найти ас

2639. Леонид , 18 декабря 2010, 22:13:57
Странно!!! ас=25 и найти надо ас????

2640. Кира, 18 декабря 2010, 22:17:42
Семья состоит из мужа, жены и их дочери студентки. Если бы зарплата мужа увеличилась вдвое, общий доход семьи вырос бы на 67%. Если бы стипендия дочери уменьшилась втрое, общий доход семьи сократился бы на 4%. Сколько процентов от общего дохода семьи составляет зарплата жены?

2641. Леонид , 18 декабря 2010, 23:40:18
Эта задача не из ЕГЭ. Она повышенного уровня сложности из Второй части ГИА в 9 классе - есть в сборнике.

2642. Екатерина, 19 декабря 2010, 09:32:21
Помогите решить задачу! Из двух пунктов, расстояние между которыми 28 км, выходят одновременно навстречу друг другу два пешехода. Если бы первый не задержался на 1 ч на расстоянии 9 км от места своего отправления, то встреча пешеходов произошла бы на полпути. После остановки первый пешеход увеличил скорость на 1 км/ч, и встреча произошла на расстоянии 4 км от того места, где задержался первый. Найти скорости пешеходов.

2643. Леонид , 19 декабря 2010, 10:59:57
Не егэшная задача - два ответа на ЕГЭ не просят

2644. 19Dec2010, 19 декабря 2010, 11:32:38
Здравствуйте, знаю, что не в тему, но я просто не нашла, где тут форум для обсуждения задач по физике...задача легкая, в ней физического только формула, а остальное.....вообщем задача на проценты, соотношение.....я думаю математики смогут помочь...... я решила и получила вариант ответа -3, но все мне доказывают, что правильный -1, хочу чтобы ВЫ помогли мне.....
Если в ОТКРЫТОМ сосуде увеличить абсолютную температуру газа на 25%, то концентрация молекул газа (газ считать идеальным) изменится?
1.-уменьшится на 25%
2.-увеличится на 25 %
3.-уменьшится на 20%
4.-не изменится
используй формулу Р=nKT , где Р-это давление, n-концентрация, K-const, Т- абс. температура
Мое решение: 1)из ранее написанной формулы выразила концентрацию- n1=P/KT1 2) Т1=100%
Х= 25% из этого условия=> х=1/4=> T2=T1+X=5/4Т1
3)n2=P/КТ2=Р/К*5/4Т1=0,8 Р/КТ
Получается, что концентрация уменьшилась на 20%
а мне говорят, что т.к. n=P/KT => n обратнопропорционально T, а т.к. Т увеличилась на 25%, то n уменьшится на 25%, если это так почему же я решаю у меня получается, что на 20% уменьшается......значит я где -то делаю ошибку.......пжл, найдите ее..........ОЧЕНЬ ЖДУ ОТВЕТА!!!! ПОЖАЛУЙСТА ПОМОГИТЕ!!!!!!

2645. Юлия, 19 декабря 2010, 15:14:46
помогите пожалуйста решить уравнение (1-cos t) (ctg+1) И вот это (sin t+1) (tg t -1)

2646. ilo, 19 декабря 2010, 15:46:32
19Dec2010, 19 декабря
решение верно. увеличение на 25% как правильно посчитано соответствует увеличению в 1,25 раза, т.к.величины обратно пропорциональны, то концентрация уменьшается в 1,25 раза и становится равной 0,8n это тоже верно посчитано, значит концентрация по сравнению с первоначальной изменилась на 0,2n,Что составляет (0,2n/n)*100%=20%нельзя утверждать, что батон хлеба на лотке на столько же процентов дешевле,чем в магазине,на сколько процентов в магазине дороже, чем на лотке.Вы правы.

2647. Leo, 19 декабря 2010, 16:27:13
не могу решить задачу и чертеж постоить не могу...

дана треугольная призма АВСА1В1С1 в основании которой лежит прямоугольный треугольник с углом С = 90, а угол В = 30.Гипотенуза АВ=корень из 21, высота призмы = 1/2.Точка N лежит на медиане основания СО, причем СN:NO=2:1.точка М лежит на диагонале СВ1 боковой грани СС1В1В.АМ и АN пересекаются.Определите угол между МВ1 и плоскостью АМN.

2648. Леонид , 19 декабря 2010, 18:29:36
та задача С2 решаема, но некоторые вещии в условии задачи вызывают вопросы. Пересмотрите условие! Написано "АМ и АN пересекаются" - козе понятно - у без этого видно, т.к. них точка А общая. И еще - не указан вид призмы. Значит она наклонная, надо думать? Кроме того, ничего не указано о М - угол искомый многозначен из-за М.

2649. ilo, 19 декабря 2010, 18:48:42
Кира, 18 декабря
изменение общего дохода в случае, когда зарплата мужа увеличилась вдвое, составило зарплату мужа или 0,67 общего дохода,т.о. m=0,67о.д.Подробно:2m+g+d-(m+g+d)/m+g+d=0,67; m/о.д.=0,67 Так же найти изменение общего дохода,если стипендия дочери уменьшилась втрое, т.е.стала1/3d-изменение общего дохода составило 2/3d (2/3d)/о.д.=0,04откуда
стипендия дочери d=0,06о.д.Если зарплата мужа 67%,стипендия дочери 6%,то зарплата жены 27% от общего дохода

2650. ilo, 19 декабря 2010, 19:09:40
Екатерина, 19 декабря
если бы пешеходы встретились на полпути, то имели бы равные скорости v=14/t,где t-время до встречи,именно с такой скоростью шёл до встречи второй,а первый с этой скоростью шёл 9км,потом увеличил скорость на 1км/ч ,скорость стала (14/t)+1=(14+t)/t и следующие 4км он прошёл за 4t/(14+t). Всего до встречи было потрачено времени 9t/14+1+4t/(14+t)=15t/14. После преобразований получается квадратное уравнение 3t^2+7t-98=0, t=14/3ч: V2=3км/ч; v1=4км/ч после увеличения.

2651. Юлия, 19 декабря 2010, 20:46:37
помогите пожалуйста решить уравнение (1-cos t) (ctg+1) И вот это (sin t+1) (tg t -1)

2652. 19Dec2010, 19 декабря 2010, 20:53:35
спасибо большое ilo!!!

2653. Игорь, 19 декабря 2010, 21:28:22
Леонид, здарова :) помоги решить уравнение плиз...

(9-x^2)*sqrt2-x-x^2=0, второй множитель весь под квадратным корнем

2654. Леонид , 19 декабря 2010, 22:01:42
9-x^2=0, x=3,x=-3, при условии если 2-x - x^>=0, x^2 + x - 2 <=0, здесь корни Х1=-2 и Х2=1 - трёхчлен неотрицательный -2<=X<=1!!! 3 и -3 в найденный промежуток не поппадают, следовательно решениями являются числа, при которых подкоренной кв. трёхчлен равен 0. Т.е. решения -2 и 1.
Как, Игорёк, учёба. Что - то воюете между собой на Кубани?

Там, выше Юлька по уравнениям обрпщается, а где они - уравнения? М.Б. упростить надо?

2655. pro100maria, 20 декабря 2010, 03:25:36
Доброго времени суток.
Огромная просьба, помогите решить след. задания, пожалуйста.

B11.
Найдите наибольшее значение функции y= 6cosx - 6x + 4 на отрезке [0; 3П/2]

C1.
Решите систему уравнений:
{ y+cos x = 0 ;
(3 cos x (cos x под корнем) - 1) (3y-2) = 0.

C2.
В прямоугольном параллелепипеде ABCDA1B1C1D1 известны ребра: AB = 6, AD = 8, CC1 = 16. Найдите угол между проскостями ABС и A1DB.

Заранее благодарю и надеюсь на помощь. Спасибо.

2656. Like_a_Dream, 20 декабря 2010, 07:15:27
Байдарка в 10 00 вышла из пункта а в пункт б расположенный в 15 км от а. Пробыв 1ч 20мин в пункте б байдарка отправилась назад и вернулась в пункт б в 16 00. Определите собственную скорость? Скор течении реки 2 км/ч

помогиииие срочно пжл

2657. ilo, 20 декабря 2010, 15:12:57
pro100maria, 20 декабря
C2.Могу предложить план решения:Найти все стороны треугольников ABD и A1BD.Найти высоту треугольника A1BD(если трудно-поясню)Дальше можно использовать формулу cosfi=S1/S,где S1-площадь проекции A1BD на плоскость нижнего основания, S-площадь самого треугольника A1BD, fi-искомый угол.Или найти высоту треугольника ABD на BD и искать как угол между высотами.
C1.из 2-го cosx>=0,из 1-го cosx=-y, y<=0
из условия равенства нулю произведения3sqrt(-y)-1=0 или 3y-2=0; из 1-го y=-1/9; из 2го y=2/3-не удовлетворяет условию y<=0;Т.о.cosx=1/9;x=(+-)arccos1/9+2pin; Ответ:(arccos1/9+2pin;-1/9) (-arccos1/9+2pin;-1/9)
B11.производная -6sinx-6=0;sinx=-1;x=-pi/2+2pin, принадлежит точка3pi/2.Считаем значения ф-ции в 0-y(0)=10, и в 3pi/2-y(3pi/2)<0; yнаиб.=y(0)=10

2658. Юлия, 20 декабря 2010, 15:30:09
Помогите (1-cos t) *(ctg+1) И вот это (sin t+1) *(tg t -1)


2659. pro100maria, 20 декабря 2010, 16:07:01
ilo, за задание B11 спасибо огромное. Вспомнила и поняла, как это делается.

А вот с С2 и С1 проблемы.

Ответ в С1 - "Ответ:(arccos1/9+2pin;-1/9) (-arccos1/9+2pin;-1/9)".. это конечный ответ без упрощений?

А С2.. не совсем поняла, т.к. в геометрии совершенно не сильна. Если возможно, не могли бы вы это объяснить, используя числа.. Пробный егэ уже завтра... очень надеюсь на вашу помощь.

2660. pro100maria, 20 декабря 2010, 16:16:10
И еще одна, просьба...

Как я написала выше - я в геометрии абсолютно не смыслю... Поэтому прошу помощи с заданием B4.

В треугольнике ABC угол С = 90 градусов,
AB = корень кв. из 13,
Sin A = 3 / корень кв. из 13.
Найдите AC.

2661. Leo, 20 декабря 2010, 16:16:25
Леонид,
задача пересмотрена...условие совпадает с написанным выше?..может быть расположение точки М как-то находится в процессе решения?...всё ещё прошу помочь мне...(не могу решить задачу и чертеж постоить не могу...

дана треугольная призма АВСА1В1С1 в основании которой лежит прямоугольный треугольник с углом С = 90, а угол В = 30.Гипотенуза АВ=корень из 21, высота призмы = 1/2.Точка N лежит на медиане основания СО, причем СN:NO=2:1.точка М лежит на диагонале СВ1 боковой грани СС1В1В.АМ и АN пересекаются.Определите угол между МВ1 и плоскостью АМN.)

2662. ilo, 20 декабря 2010, 16:36:52
в С-1 записан ответ.
По С-2: ABC-плоскость нижнего основания,A1BD-треугольник стороны которого-диагонали гранейA1B, A1D,BD.По теореме Пифагора BD=10; A1B=2sqrt73; A1D=8sqrt5;Сделай выносной чертёж-треугольник BA1D.Найдём высоту к BD. Пока ясно?

2663. ilo, 20 декабря 2010, 16:56:43
pro100maria, 20 декабря И еще одна, просьба...
AC/AB=cosA; cosA из основного тригонометрического тождества sqrt(1-9/13)= 2/sqrt13; AC=AB*cosA=sqrt13*(2/sqrt13)=2;

2664. Тамирлан, 20 декабря 2010, 17:12:52
помогите сделать мне B1 B2 B4

2665. ilo, 20 декабря 2010, 17:29:45
Тамирлан, 20 декабря
Да пожалуйста...

2666. pro100maria, 20 декабря 2010, 17:42:06
ilo, даже не знаю, как благодарить. Спасибо огромное за помощь.

2667. ilo, 20 декабря 2010, 18:22:43
Из треугольника A1BD найдём высоту A1H: Сначала по теореме косинусов найти cosB:320=292+100-2*10*2sqrt73cosB; cosB=9/(5*sqrt73), затем по основному тригонометрическому тождеству sinB=(4*sqrt109)/(5*sqrt73);sinB=h/2sqrt73; откуда h=(8sqrt109)/5;S A1BD=8sqrt109; cosfi=24/(8sqrt109)=3/sqrt109,где 24- площадь прямоугольного треугольника ABD,находится легко.fi-угол между плоскостями из формулы fi=arccos3/sqrt109;

2668. настенька, 20 декабря 2010, 18:24:37
ребята, помогите пожалуйста, решить задачку,
2cos2x+sin2x>tgx
пыталась решать через универсальную подстановку, но к ответу не могу прийти! помогите пожалуйста, очень нужно)

2669. ilo, 20 декабря 2010, 18:32:29
Eсли формулу не давали, то в треугольнике ABD найти тем же путём высоту она будет равна 24/5 и из прямоугольного треугольника, где 16 второй катет найти сosfi=(24/5)/(8sqrt109/5)=3/sqrt109/ Ответ тот же.

2670. Леонид , 20 декабря 2010, 22:43:18
2661. Leo, 20 декабря 2010, 16:16:25.
По условию точка М лежит на АВ1, то
МВ1 - это же на прямой СВ1, т.е. МВ1 лежит в плоскости АМN - значит угол между ними 0 градусов. Причём тут тогда все те данне, что указаны в условии.
К сведению: медиана СО из прямого угла прямоугольного треугольника, значит СО = половине гипотенузе 1/2 корня из 21. Легко находятся СN и NO по данному отношению. И причём тут высота призмы. Что то с условием задачи?

2671. настенька, 21 декабря 2010, 14:21:17
ребят, а что мне с моей задачкой никто не поможет?( ну хотя бы намекните какой ответ получится)

2672. ilo, 21 декабря 2010, 15:15:23
настенька, 21 декабря
Если выразить tg через синус и косинус двойного угла,получится дробь cos 2x(2+cos2x+sin2x)/1+cos2x>0 Всё, что могу.А это неравенство дано в условии или следствие другого?

2673. ilo, 21 декабря 2010, 16:30:00
настенька, 21 декабря
Лучше так:
2 * cos 2x + sin 2x > tg x
2 * (cos^2 x - sin^2 x) + 2 * sin x * cos x > sin x/cos x
2 * cos^2 x - 2 * sin^2 x + 2 * sin x * cos x - sin x/cos x > 0
(2 * cos^3 x - 2 * sin^2 x * cos x + 2 * sin x * cos^2 x - sin x)/cos x > 0
(2 * cos x * (cos^2 x - sin^2 x) + sin x * (2 * cos^2 x - 1))/cos x > 0
(2 * cos x * cos 2x + sin x * cos 2x)/cos x > 0
cos 2x * (2 * cos x + sin x)/cos x > 0
cos 2x * (2 + tg x) > 0
Получаем: cos 2x > 0, tg x > -2, cos x <> 0
или cos 2x < 0, tg x < -2, cos x <> 0

2674. ilo, 21 декабря 2010, 17:09:20
настенька, 21 декабря ну хотя бы намекните какой ответ
-pi/4+pin<x<pi/4+pin и (объединение) pi/2+pin<x<pi-arctg2+pin

2675. Ольга, 21 декабря 2010, 19:02:57
основание прямой треугольной призмы треугольника АВС, в котором угол С=60 и АВ=ВС=8.на ребре ВВ1 отмечена точка Р так,что ВР:РВ1=3:5.найти тангенс между плоскостями АВС и АСР,если расстояние между прямыми ВС и А1С1 равно 16

2676. Ольга, 21 декабря 2010, 19:03:53
помогите пожалуйста!

2677. Юлия, 21 декабря 2010, 20:40:58
помогите решить уравнение 2cos^квадрате t-1=0

2678. настенька, 21 декабря 2010, 21:09:14
ilo, это неравенство дано в условии!( я пробовала сделать замену sin2xи cos2x через универсальную подстановку, дошла до дроби: в числителе (tgв квадрате 2х - 1)(тангенс х+2), в знаменателе 1 + тангенс в квадрате х и вся эта дробь меньше нуля, а дальше не получается!!!(( помогите , очень нужно!

2679. Юлия, 21 декабря 2010, 21:12:02
Ещё вот это помогите пожалуйста просто щас скоро сессия и я не понимаю немного sin3x+ корень из 3cos 3x=0

2680. настенька, 21 декабря 2010, 21:41:30
2679. Юлия, 21 декабря 2010, 21:12:02
Ещё вот это помогите пожалуйста просто щас скоро сессия и я не понимаю немного sin3x+ корень из 3cos 3x=0,



я попробовала решить , ответ получился: плюс минус pi/6+2/3pin, может кто еще решит, если такой же ответ получится, то ммогу написать ход решения)

2681. ilo, 21 декабря 2010, 22:45:16
настенька, 21 декабря
посмотри внимательнее
2673-решение, 2674-ответ,что не ясно-поясню

2682. ilo, 21 декабря 2010, 23:34:59
Юлия, 21 декабря
sin3x+ корень из 3cos 3x=0; делишь на cos3x: tg3x+sqrt3=0; tg3x=-sqrt3;
3x=-pi/3+pin;x=-pi/9+pin/3;

2683. ilo, 21 декабря 2010, 23:46:49
Юлия, 21 декабря
2cos^квадрате t-1=0; cos^2t=1/2; cost=(+-)sqrt2/2;cost=sqrt2/2; t=(+-)pi/4+2pin; cost=-sqrt2/2; t=(+-)3pi/4+2pin; объединяем: t=pi/4+pin/2;

2684. ilo, 21 декабря 2010, 23:57:50
Ольга, 21 декабря
CC1-общий перпендикуляр скрещивающихся прямых-высота призмы BP:PB1=3:5; BP=16:8*3=6; PB1=2*5=10; ABC- равносторонний,BH=(sqrt3/2)*8=4sqrt3; PC=AP=10; -по теореме Пифагора; PH=sqrt(100-16)=sqrt84=2sqrt21;cosfi=BH/PH=4sqrt3/2sqrt21=2/sqrt7; fi-искомый угол.

2685. Анюта, 22 декабря 2010, 15:03:25
Помогите,пожалуйста решить задачу- В прямоугольном параллелепипеде ABCDA1B1C1D1 известны рёбра АВ=8,АD=6 CC1=7.Как можно скорее.жду....

2686. Юлия , 22 декабря 2010, 15:31:45
эо не так решается там надо по другому мы это не проходили

2687. Анюта, 22 декабря 2010, 16:29:10
Это опять я...просто от этой задачи зависит какая у меня выйдет оценка за полугодие((((

2688. dimageniy, 22 декабря 2010, 19:21:25
анют а что найти то нужно

2689. Леонид , 22 декабря 2010, 19:31:59
2685. Анюта, 22 декабря 2010, 15:03:25
Вы прочитали,что написали? Ищи то, не знаю что!

2690. Юлия, 22 декабря 2010, 22:28:56
Блин ну помогите пожалуйста решить вот эти уравнения 1-sin^квадрате t=0
6cos^квадрате х+7sin х-8=0

2691. Анюта, 23 декабря 2010, 13:36:04
Ой,простите...торопилась,не заметила... В прямоугольном параллелепипеде ABCDA1B1C1D1 известны рёбра АВ=8,АD=6 CC1=7.Найдите угол между плоскостями СD1В1 и AD1B1.
Ещё раз прошу...кто сможет-помогите...


2692. dimageniy, 23 декабря 2010, 15:52:06
2690. Юлия
первое решу тут и думать нечего сразу ответ по основному тригонометрическому тождеству cos^2 t=0
cos t=0
t=arccos(0)=пи/2+пи*N, где N множество натуральных чисел

второе решается как квадратное уравнение только сперва cos^2 x переводишь в
1-sin^x получаем
-6sin^x+7sinx-2=0 пусть sinx=t тогда
-6t^2+7t-2=0
D=49-48=1
t1=0.5
t2=2/3
подставляешь вместо t sinx и решаешь c помошью arcsin ( просто не хочу дальше писать мне лень очень)

2693. Леонид , 23 декабря 2010, 15:59:05
Ищите в ранних записях. Эту задачу уже решали здесь. Записи 2484-2489

2694. dimageniy, 23 декабря 2010, 16:12:48
АНЮТ вообщем задачка нетрудная но написать ее сюда очень тяжело ответ кажется 83,5 градуса
хотя попозже можно постараться написать сюда у меня сейчас времени нет только напишу я так как смогу а ты должна будешь постараться понять если че напиши мне на e-mail

2695. Анюта, 23 декабря 2010, 17:36:07
2694. dimageniy, 23 декабря 2010, 16:12:48,а если без градусов?
там вроде ответ должен получится 2arctg24/35.
я не смогу тебе написать на е-mаil,если не трудно будет,то сам напиши мне туда решение...

2696. dimageniy, 23 декабря 2010, 19:30:11
все анют задачу решил сейчас с твоим ответом совпадает ты мне свой id вконтакте скажи пожалуйста чтоб знать кому помог у меня http://vkontakte.ru/id62305060

2697. Настя, 23 декабря 2010, 19:37:00
Помогите,пожалуйста,я совсем не понимаю задачи типа В1=((
заранее спасибо!

2698. dimageniy, 23 декабря 2010, 19:53:17
анют ответ 2*arctg (5/7) решение скину по фотке вконтакте http://vkontakte.ru/id62305060

2699. dimageniy, 23 декабря 2010, 19:56:54
настя пиши вконтакте я тебе видио перекину с решением В1

2700. настенька, 23 декабря 2010, 19:58:27
2674. ilo, 21 декабря 2010, 17:09:20
настенька, 21 декабря ну хотя бы намекните какой ответ
-pi/4+pin<x<pi/4+pin и (объединение) pi/2+pin<x<pi-arctg2+pin,
я еще раз перерешала, по твоему способу , и по своему через универсальную подчтановку всё вроде понятно, кроме окончательного ответа( почему там pi/2+pin<x<pi-arctg2+pin, у меня получается вместо этого выражения такое:
-pi/2+pin, - arctg2+pin!) но все равно спасибо за помощь)

2701. Анюта, 23 декабря 2010, 20:09:41
dimageniy,набери в контакте Анна Арасланова.на Аве моё фото,а под фотографией написано "Аня".Жду)))

2702. ilo, 23 декабря 2010, 21:55:59
записаны симметричные дуги, у меня верхняя, у тебя нижняя, при n=1 ты получаешь мою, я при n=-1 твою.Хорошо бы посмотреть решение через универсальную подстановку,есть время-выложи.

2703. Юлия, 23 декабря 2010, 22:11:21
решите вот это пожалуйста 4sin^квадрате х+4cos х -1=0

2704. Игорь, 23 декабря 2010, 22:11:59
Привет Леонид, учеба нормально...ты как?у вас снег выпал?Прикинь,у нас солнце каждый день +18 днем, по ходу пролетаем мы со снегом на Новый год.Поясни что ты имел ввиду за разборки?

Неделю назад начали изучать логарифмы.ВЫот уравнение и неравенство немогу решить...помоги пжл.

lg^2x+4lg(10*x)=1


log (x-1) по основанию 1/5<=-2

Заранее спасибо.

2705. Леонид , 23 декабря 2010, 23:52:55
Лорогой, у нас неделю уже за 40 мороза, а сегодня ещё с ветерком - заметь, пл сибирски. Ха-ха-ха! Смешно? В станице, кажется Кущевской?
lg^2 x + 4lg10 + 4lg x (применили логарифм произведения в сумму логарифмов и на 4 = скобки раскрыли) - 1 = 0. Пусть lg x = a, то имеем уравнение: a^2 + 4a + 3 = 0 (логарифм числа по его основанию равен 1,т.е. lg10=1). a1 = - 1, a2= -3.
1) lg x = -1, x= 0,1. 2)lg x = -3, x=0,001. ОДЗ х>0-оба значения а - подходят.

log(x-1) осн.1/5 <= log 25 осн. 1/5. . т.к. (1/5)^ -2 = 25. Основание 1/5<1- логарифмическая функция убывающая (важнейий момент при решении логарифмических неравеств), то x - 1>= 25, x >= 26. ОДЗ x-1>0, x > 1. Ответ x>=26

2706. Игорь, 24 декабря 2010, 18:02:15
Леонид, 40 мороза это для нас жесть:)про разборки ничего не знаю, только в Москве вот вот были.

уравнение немного не понял как решать,а неравенство понятно!

2707. Леонид , 24 декабря 2010, 20:15:03
что, конкретно,не понял в решении уравнения? Оно очень простое-алгоритмичное. 4lg(10x) = 4(lg10 + lg x) = 4(1+lg x)=4+4lg x, а затем квадратное относительно логарифма х...

2708. Snoopy, 25 декабря 2010, 16:56:24
Пристани А и В расположены на озере,расстояние между ними 280км.Баржа отправилась с постоянной скоростью из А в В.На следуйщий день после прибытия она отправилась обратно со скоростью на 4км/ч больше прежней,сделав по пути остановку на 8 часов.В результатее она затратила на обратный путь столько же времени,сколько на путь из А в В.Найдите скорость баржи на пути из А в В.Ответ дайте в км/ч

2709. Snoopy, 25 декабря 2010, 18:42:16
Помогите пожалуйсто!!!

2710. timmy, 26 декабря 2010, 16:15:44
стоимость участия в семинаре-3000руб. с человека. Группам от организаций предоставляются скидки: от 3 до 10 человек- 5%; более 10 человек- 8%. Сколько должна заплатить организация, направившая на семинар группу из 12 человек?

2711. ilo, 26 декабря 2010, 21:06:17
Snoopy, 25 декабря
Время туда равно времени обратно, его и находим. 280/v=280/(v+4)+8 В чём трудность?

2712. ilo, 26 декабря 2010, 21:14:42
Анюта, 23 декабря
Ещё раз прошу...кто сможет-помогите...
План такой: Доказать равенство треугольников A1B1D1 и B1D1C, найти все их стороны и высоты и по теореме косинусов найти угол между высотами к стороне B1D1.

2713. ilo, 26 декабря 2010, 21:30:13
timmy, 26 декабря
3тыс.*12=36т со всей компании 36тыс*0,08=2,88тыс.;-скидка; 36-2,88=33,12тыс.=33120руб.

2714. Анюта, 27 декабря 2010, 14:30:02
в правильной треугольной пирамиде апофема боковой грани равна 2корень из6 и образует с плоскостью основания угол 45градусов. найдите объём пирамиды.

2715. ilo, 27 декабря 2010, 15:27:32
Анюта, 27 декабря Из равнобедренного треугольника SHO найди высоту,где S-вершина пирамиды противоположная основанию ABC, Н-основание апофемы,О-основание высоты пирамиды.Отрезок OH равен высоте и равен1/3 медианы(высоты,Биссектрисы, медианы)основания, найти медиану правильного треугольника а по ней сторону основания.Дальше остаётся только подставить в формулу объёма пирамиды.

2716. Рауль, 27 декабря 2010, 18:38:02
В треугольнике MNK угол N=45 градусам, угол M=30 градусам, NK=5. Найдите MK.

2717. Анюта, 27 декабря 2010, 18:53:35
И какой ответ получился?у меня 16 получается.

2718. ilo, 27 декабря 2010, 19:23:49
Маловато получилось. h=2sqrt3; m=6sqrt3; a=12; S=36sqrt3;

2719. Ирама, 27 декабря 2010, 19:32:21
помогите решить задачу в треугольнике авс вписан ромб так, что угол с у них общий, а остальные три вершины мнк лежат на сторонах треугольника. Найдите площадь ромба, если угол асв=120 градусов, ам:мс=2:3 и вс=15 см

2720. ilo, 27 декабря 2010, 20:35:17
Ирама, 27 декабря
Из подобия треугольников AMN и ABC найти сторону ромба MN, дальше в формулу S=a^2sinC

2721. Анюта, 27 декабря 2010, 21:15:56
2718. ilo, 27 декабря 2010, 19:23:49,если не трудно,сфоткай и отправь мне решение http://vkontakte.ru/id43603401...

2722. Валерия, 27 декабря 2010, 21:27:15
Помогите пожалуйста решить задачу!
теплоход проходит по течению реки до пункта назначения 513 км и после стоянки возвращается в пункт отправления. Найдите скорость течения, если скорость теплохода в неподвижной воде равна 23 км/ч, стоянка длится 8 ч, а в пункт отправления теплоход возвращается через 54 ч после отплытия из него. Ответ дайте в км/ч.

2723. dimageniy, 27 декабря 2010, 21:34:16
Lexxus вы тут как бы главный, можно поинтересоваться почему у меня показывает уровень доверия ноль, хотя совсем недавно было 74%

p.s. я в рейтинге 109 примерно если смотреть через общий рейтинг то уровень доверия нормальный

2724. Evgetor, 29 декабря 2010, 09:00:10
привет всем
у меня вопрос такой, вот по этой задачке
http://live.mephist.ru/show/mathege2010/view/id/6989/
правильный ответ 6, но я почему то сомневаюсь в его правильности
я думаю ответ 4
кто-нибудь может обьяснить почему 6??
буду очень благодарен

2725. dimageniy, 29 декабря 2010, 10:55:49
я тоже согласен там правильный ответ 4

2726. Леонид , 29 декабря 2010, 16:27:08
2724. Evgetor, 29 декабря 2010, 09:00:10. ОТВЕ: 4

2727. Серёжа, 30 декабря 2010, 17:05:00
Цена на электрический чайник была повышена на 22% и составила 3050 руб.СВколько рублей стоил чайник до повышения цены?

Напишите пожалуйста решение и ответ!!!
Заранее спасибо

2728. Evgetor, 1 января 2011, 16:35:29
2727. Серёжа, 30 декабря 2010, 17:05:00

стоймость чайника до повышения цены берём за Х
тобишь повышение на 22% выражаем как 0.22Х
дальше это складываем и приравниваем к стоймости после повышения
получается вот так
х+0.22х=3050
1.22х=3050
х=2500

ответ: до повышения цены чайник стоил 2500 руб

2729. Леонид , 1 января 2011, 22:25:18
Проще и сразу: цена увеличена на 22%, т.е. по отношению первоначальной 122% или в частях 1,22. Находим первоначальную чену: 3050:1,22 = 2500

2730. Фии, 2 января 2011, 19:32:38
Точки A, B, C, расположенные на окружности, делят ее на три дуги, градусные величины которых относятся как 1:3:5 . Найдите больший угол треугольника ABC. Ответ дайте в градусах.

ответ:100

http://live.mephist.ru/show/mathege2010/view/id/27868/



объясните пожайлуста решение

2731. Мade in Rай, 2 января 2011, 19:35:07
Сосулька упала с крыши с высоты 30м.Высота h(t), на которой находится падающая сосулька,зависит от времени падения t(c) следующим образом
h(t)=30-5t2.
Определите,сколько секунд сосулька будет находится на высоте не менее 10,8м?

Помогите решить пожалуйста)))
Извините,если такая задача уже была=)

2732. Фии, 2 января 2011, 20:19:53
2731. Мade in Rай, там наверное 30t и 10 метров?

если так, тогда
30t- -5t^2-10=0
5t^2-30t+10=0
решшаем ур-е, и получаем х=5;1
5-1=4
ответ:4

2733. Made in Rай, 2 января 2011, 22:08:00
Фии,нет=)
именно такое условие)))
Правда задачу я уже решила=)Вот решение,может кому пригодиться)
10,8<=30-5*t^2
t^2<=19,2/5
t^2<=3,84
t<=1,96 сек

2734. Lexxus, 2 января 2011, 23:04:44
Lexxus вы тут как бы главный, можно поинтересоваться почему у меня показывает уровень доверия ноль, хотя совсем недавно было 74%

Периодически программа сбрасывает результат из-за ошибки обращения к базе данных. Само восстановится.

2735. Valentiya, 4 января 2011, 07:12:01
Два автомобиля отправляются в 420-километровый пробег. Первый едет со скоростью на 10 км/ч большей, чем второй, и прибывает к финишу на 1 час раньше второго. Найти скорость автомобиля, пришедшего к финишу вторым.
Помогите с решением=( Я получила систему и когда начала подставлять значения очень сильно что-то намудрила и запуталась... Заранее спасибо

2736. Леонид , 4 января 2011, 08:53:17
зачем система? Пусть искомая скорость пришедшего вторым Х, то пришедшего первым (х + 10). Время второго 420/х время первого 420/(х+10). Уравнение:
420/х - 420/(х+10)=1. к общ. знам. ... Вообщето, выше такиз задач решено очень много. Просматривыйте. Есть все возможные, которые будут на егэ, другие отличаются только числами. Не ленитесь, ищите, систематизируйте - это хорошо помагает в подготовке.

2737. ВАся, 6 января 2011, 12:13:28
В треугольнике АВС АС=ВС=12 sinВ=корень из 15/4.Найдите АВ

2738. Вася, 6 января 2011, 16:04:25
Камень брошен вертикально вверх. Пока камень не упал, высота, на которой он находится, описывается формулой ( — высота в метрах, — время в секундах, прошедшее с момента броска). Найдите, сколько секунд камень находился на высоте не менее 9 метров. Как решать, помогите

2739. Вася, 6 января 2011, 16:09:35
Камень брошен вертикально вверх. Пока камень не упал, высота, на которой он находится, описывается формулой h(t)=-5t^2+18t (h — высота в метрах, t — время в секундах, прошедшее с момента броска). Найдите, сколько секунд камень находился на высоте не менее 9 метров.

2740. Денис, 7 января 2011, 19:11:07
В треугольнике АВС угол С равен (0 градусов, АВ=5, cosА=0,8. Найдите ВС.

2741. Денис, 7 января 2011, 19:28:10
В треугольнике АВС угол С равен 60 градусов, АВ=5, cosА=0,8. Найдите ВС.

2742. Асечка, 8 января 2011, 12:00:13
Помогите решить задачки плис.,очень надо.
1.АВСD-параллелограмм,ВDпринадлежит альфа,АС перпендикулярен альфа, АВ=5см.Чему равен периметр параллелограмма?

2)треугольник АВС-правильный,АВ=3см,высота АМ и СК пересекаются в точке О. DOперпендикулярен АВС,DО=1см
Чему равно расстояние от точкиD до вершин треугольника?

3) параллелограмм,AD=4см,CD=3см
МС перпендикулярен АВС,МDперпендикуляренAD.Чему равна площадь параллелограмма.
Спасибо.

2743. Асечка, 8 января 2011, 12:52:12
надо решить за 4 дня плисссссссссс,помогите

2744. Made in Rай, 8 января 2011, 15:17:53
Помогите пожалуйста решить В10:

Найдите наименьшее значение функции y=e^4x + e^-4x -5

2745. Maria, 8 января 2011, 16:10:17
Решите, пожалуйстаа!
Найдите точку минимума функции y=(x+5)^2*e^5-x

2746. Кристина, 8 января 2011, 21:48:16
В основании четырехугольной пирамиды SАВСД лежит квадрат АВСД со стороной (3*на корень из 10):5. Длины всех боковых ребер 3, точка М-середина ребра AS.Через прямую ВМ паралельно диагонали АС проведена плоскость.Определите величину угла (в градусах) между этой плоскостью и SAC.

2747. Kristi, 8 января 2011, 22:03:32
С3 Решите неравенство
log_((x+2)^2)(x(x+1)(x+3)(x+4))>1

Будьте добры, решите, плииииз

2748. Леонид , 9 января 2011, 11:23:06
что с основанием логарифма?

2749. Kristi, 9 января 2011, 11:57:19
основание: (x+2)
сам логарифм в квадрате
а это число: (x(x+1)(x+3)(x+4))

2750. Lexxus, 9 января 2011, 14:58:26
log_((x+2)^2)(x(x+1)(x+3)(x+4))>1
основание: (x+2)
сам логарифм в квадрате
а это число: (x(x+1)(x+3)(x+4))


То есть
(Логарифм (x(x+1)(x+3)(x+4)) по основанию (x+2)) в квадрате?

Условие точно правильно записано? Как-то это на бумажке ну совсем не решается.

2751. Kristi, 9 января 2011, 15:09:06
Да, все верно, еще раз перепроверила)
только не основание в квадрате, а логарифм)

2752. Lexxus, 9 января 2011, 15:52:48
В основании четырехугольной пирамиды SАВСД лежит квадрат АВСД со стороной (3*на корень из 10):5. Длины всех боковых ребер 3, точка М-середина ребра AS.Через прямую ВМ паралельно диагонали АС проведена плоскость.Определите величину угла (в градусах) между этой плоскостью и SAC.


Поскольку плоскость проведена через прямую ВМ паралельно диагонали АС, то ей принадлежит и прямая, параллельная AC и проходящая через точку M (назовём её MN).
Таким образом, нам нужно найти угол между плоскостями SAC и BMN, пересекающимися по прямой MN.



По условию, пирамида SABCD - правильная, а значит, высота SO делит диагонали основания пополам. Кроме того, точка N делит ребро SC пополам, следовательно, BM = BN, а точка P делит пополам отрезки MN и SO.

Отрезок BP (принадлешащий плоскости BMN) перпендикулярем MN, отрезок OP (принадлежащий плоскости SAC) перпендикулярер MN. Поэтому угол между плоскостями - это угол BPO, который мы найдём из одноименного прямоугольного треугольника.

BO = AO = (половина диагональ квадрата) =
= 3*sqrt(10)/5*sqrt(2)/2 = 3*sqrt(5)/5.

PO = (половина катета SO прямоугольного треугольника ASO) =
= sqrt(3^2-(3*sqrt(5)/5)^2)/2 = 3*sqrt(5)/5.

То есть, BO = PO, а значит, треугольник BPO не только прямоугольный, но и равнобедренный, и угол BPO = 45 градусов.

Ответ: 45 градусов.

2753. Леонид , 9 января 2011, 18:27:00
(Логарифм в квадрате числа(x(x+1)(x+3)(x+4)) по основанию (x+2)) >1.
Надо идти путями: 1) ЛОГ х(х+1)(х+3)(х+4) осн (х+2)> ЛОГ (х+2) осн (х+2),
2) ЛОГ х(х+1)(х+3)(х+4) осн (х+2)< ЛОГ (х+2)^(-1) осн (х+2),

1) и 2) рассматривать два случая: а) если 0<x+2<1, освобождаясь от ЛОГ знак неравенства сменить, т.к. функция убывающая;
б) х+2>1, освобождаясь от ЛОГ знака неравенства не менять, т.к. ф-я возрастающая. Перейдём, тем самым от логарифмичеих неравенств к рациональным. Это общий алгоритм решения - продолжайте сами- много надо печатать

2754. alfa20, 10 января 2011, 00:06:01
Асечка, в №1-что-то не то в условии.
№2. OA=OB=OC=R описанной около тр.АВС окр-ти. Тогда R=a/sqrt(3)=3/sqrt(3)=sqrt(3), DA=DB=DC=SQRT(1+3)=2.
№3.По обратной теореме о трёх перпендикулярах, CD перп-на AD. Тогда АВСД-прямоугольник, его площадь равна 4*3=12.

2755. Асечка, 10 января 2011, 13:33:13
Спасибо,большое=)

2756. Жанна, 10 января 2011, 17:19:25
я чета не поняла! почему до сих пор все ответы на задания не определены???? вы че, обалдели совсем7 а как детям тогда готовиться??? сами не определили еще ответы, так это дети должны определять их??*??

2757. Асечка, 11 января 2011, 05:00:49
alfa20,Я условие 1го проверила,то же самое,так что оно правильное.А что во 2м значит sqrt?

2758. Виктуар, 11 января 2011, 06:45:50
Привет YOU,я Виктуар помогите решить задачки плис.треугольник АВС,уголС=90градусов.ADперпендикулярен АВС.CD=12см,BD=13см.Чему равна длина отрезка Вс?

АВСВ-ромб.BFпринадлежит АВС.
Какой плоскости перпендикулярна прямая Ас?

Aпринадлежит АВС,ВD,биссектриса треугольника АВС.Чему равен угол (альфа,ВD)?
Диогональ Ас квадрата ABCDперпендикулярна некоторой плоскости альфа,проходящей через точку А.Чему тогда равен диогональ ВD,?
BD перпен.ABCD.ВК, ВИ и ВМ-медиана,биссектриса и высота треугольника АВС.К какой плоскости прямая АС перпен. ?

2759. Юлия, 11 января 2011, 10:54:08
я ничего не понимаю(((

2760. Юлия, 11 января 2011, 10:59:07
в треугольнике АВС угол С равен 90 градусов.АВ равно 5,cosА равен 0,8.Найдите ВС.

Помогите))

2761. Юлия, 11 января 2011, 11:03:50
это задание В4((

2762. Иван, 11 января 2011, 12:57:06
первый рабочий за час делает 2 детали больше,чем второй рабочий,и заканчивает работу над заказом,состоящим из 621 деталей,на 4 раза раньше,чем второй рабочий выполняет заказ,состоящий из 675 таких же деталей.Сколько деталей делает в час первый рабочий ? Помогите пожалуйста

2763. козел, 11 января 2011, 13:06:51
Коментарий скрыт (грубость — 19%)показать

2764. ильзия, 11 января 2011, 13:11:19
Коментарий скрыт (грубость — 40%)показать

2765. ДУМА ТИЛЕСЕ, 11 января 2011, 13:20:04
Коментарий скрыт (грубость — 89%)показать

2766. Иван, 11 января 2011, 13:38:59
Коментарий скрыт (грубость — 13%)показать

2767. Kristi, 11 января 2011, 18:56:33
там где с логарифмами, все-таки, наверно, основание в квадрате
что-то у меня не получается решить, помогите, пожалуйста

2768. Леонид , 11 января 2011, 21:27:45
Для Виктории!!! Пусть х дней понадобится на выполнение всей работы 1-у рабочему (просят по условию), 2-у у дней. Их производительность соответственно 1/х и 1/у. Общая призводительность 1/6. Уравнение (1): 1/х + 1/у = 1/6.

1-й за 2 дня со своей производительность выполнит работы 2/х, 2-й за 3 дня выполнит 3/у. И т.к. работа будет одтнаковой уравнение (2): 2/х = 3/у. Надо решить систему из (1) и (2) уравнений. В первом уравнении, приведя к общему знаменателю, получим 6х + 6у = ху. В уравнении (2) 2у =3х, где у = 3/2 х. Подставляем в новое (1) 6х + 6(3/2 х) = х (3/2 х). 6х + 9х = 3/2 х кв. Решайте квадратное уравнение, получаем х.

2769. alfa20, 11 января 2011, 21:47:48
Асечке
Возможно, в №1 так: Т.к. АС перп. альфа, то АС перп. любой прямой, лежащей в альфа, тогда АС перп. ВД. Тогда АВСД-ромб со стороной 5 см. Его
Р=4*5=20.
sqrt - это квадратный корень

2770. Асечка, 12 января 2011, 08:29:13
alfa20,Спасибки те большое,ваще помог=)(*_*)

2771. Елена18, 12 января 2011, 16:38:01
Народ помогите пожалуйста!!!! Как решить

кАЖДЫЙ ИЗ ДВУХ РАБОЧИХ ОДИНАКОВОЙ КВАЛИФИКАЦИИ МОЖЕТ ВЫПОЛНИТЬ ЗАКАЗ ЗА 15 Ч. ЧЕРЕЗ 5 Ч. ПОСЛЕ ТОГО, КАК ОДИН ИЗ НИХ ПРИСТУПИЛ К ВЫПОЛНЕНИЮ ЗАКАЗА, К НЕМУ ПРИСОЕДИНИЛСЯ ВТОРОЙ РАБОЧИЙ, И РАБОТУ НАД ЗАКАЗОМ ОНИ ДОВЕЛИ ДО КОНЦА УЖЕ ВМЕСТЕ. ЗА СКОЛЬКО ЧАСОВ БЫЛ ВЫПОЛНЕН ВЕСЬ ЗАКАЗ?

ЗАРАНЕЕ БОЛЬШОЕ СПАСИБО!!!

2772. Кирилл, 12 января 2011, 17:50:30
За 5 часов рабочий выполнил 1/3 заказа. Осталось 2/3. Далее, рабочий за час выполняет 1/15 заказа, вдвоем за час 2/15 заказа.Следовательно, после того, как присоединился второй рабочий, они выполнили 2/3 заказа за (2/3)/(2/15) = 5 часов. Итого, весь заказ был выполнен за 10 часов.

2773. гелька, 13 января 2011, 14:37:45
решите мне задачу: В 9 найдите боковое ребро правильной четырехугольной призмы, если сторона ее основания равна 20, а площадь поверхности равна 1200

2774. гелька, 13 января 2011, 14:39:57
ну помогите же кто нить

2775. Lexxus, 13 января 2011, 14:45:40
найдите боковое ребро правильной четырехугольной призмы, если сторона ее основания равна 20, а площадь поверхности равна 1200


Площадь поверхности правильной четырехугольной призмы равна
2*(сторона основания в квадрате) + 4*(сторона основания * боковое ребро)

Чему она равна, тебе известно. Отсюда можешь найти боковое ребро.

2776. Кирилл, 13 января 2011, 14:52:22
площадь поверхности равна сумме площадей двух оснований и сумме площадей четырех граней , т.е. 2*20^2+4*20*h , где h-ребро, и , по условию задачи, она равна 1200.
Следовательно, 2*20^2+4*20*h=1200;
h = 5

2777. гелька, 13 января 2011, 14:53:06
ответ будет 100?????????

2778. гелька, 13 января 2011, 14:55:29
у меня не такой ответ!!!

2779. Кирилл, 13 января 2011, 15:02:07
гелька, я не понял тебя. Ты утверждаешь, что ответ будет 100 или что ?

2780. гелька, 13 января 2011, 15:03:51
первую треть трассы автомобиль ехал со скоростью 80 км/ч, вторую треть со скоростью 30км/ч, а последнюю со скоростью 60 км/ч. найдите среднюю скорость автомобиля на протяжении всего пути. ответ дайте в км/ч. ПЛИЗЗЗ

2781. гелька, 13 января 2011, 15:05:31
извини пожалуйста ти все правильно решьл это я не так посчитала, реши пожалуйсто ещё****

2782. Кирилл, 13 января 2011, 15:06:27
Гелька, ты совсем плохая что ли ? (80+30+60)/3

2783. гелька, 13 января 2011, 16:21:59
я так пробовала решить там число не целое и не десятичное получается! как я в бланк то примерное запищу??????????

2784. гелька, 13 января 2011, 16:29:18
радиус основания первого круга в 3 раза больше, чем радиус сонования второго конуса , а образующаа первого конуса в 2 раза меньше, чем образующаа второго. чему равна площадь боковой поверхности первого конуса, если площадь боковой поверхности второго равна 18 (см квадратных)

2785. геля, 13 января 2011, 16:48:10
ау лиди мне кто нить окажет помощь?

2786. Леонид , 13 января 2011, 18:50:13
S = pi*RL боковая первого, s =pi*(R/3)*(2L) = 18- боковая второго через элементы первого. Из боковой второго 18=pi*(1/3)* 2RL Умножим обе части на 3/2. 18* 3/2 = pi*RL = 27 -боковая первого

2787. Катерина, 14 января 2011, 18:49:29
Решить систему уравнений

16^(cosx) -10 * 4^(cosx) + 16=0
корень из7+ 2sinx=0

Леонид очень прошу решите мне это задание.

2788. Lexxus, 14 января 2011, 19:49:47
Решить систему уравнений

16^(cosx) -10 * 4^(cosx) + 16=0
корень из7+ 2sinx=0


В первом уравнении делаешь замену
t = 4^cos(x)

t^2-10*t+16 = 0

t1 = 2, t2 = 8

1) 4^cos(x) = 2, cos(x) = 1/2
2) 4^cos(x) = 8, cos(x) = 1.5 > 1, так не бывает.

Так что из первого уравнения получается
x = (+/-) 1/2 + 2*пи*n

А вот второе уравнение ты наверняка неправильно сюда переписала.
Там, может, всё-таки, не 7, а 3?

2789. катя =), 15 января 2011, 00:09:52
круто всё придумано, только вот бы ещё разбор задач был...

2790. катя =), 15 января 2011, 00:15:19
пжл...люди, помогите решить задачу: В9. прямоугольный параллелепипед описан около сферы радиуса 8. Найдите его объём.
заранее Вам огромное спасибо)

2791. Катерина, 15 января 2011, 07:30:26
простите пожалуйста да очень спешила написать...



там корень из y + 2sinx=0


моя невнимательность черт ее дери

2792. Evgetor, 15 января 2011, 11:00:58
привет всем

насчёт вот этого задания чо скажете

http://live.mephist.ru/show/mathege2010/view/id/25757/

у меня получается ответ 937.5

2793. Evgetor, 15 января 2011, 11:06:31
2790. катя =), 15 января 2011, 00:15:19

если радиус сферы 8
значит стороны параллелепипеда по 16
Vпараллелепипеда = abc
тоесть 16*16*16=4096

2794. гелька, 15 января 2011, 13:49:23
найдите наибольшее значение функции у=3х-3tgх-8 на отрезке [0;п/4]
ПОЖАЛУЙСТА ПОМОГИТЕ...........

2795. ГЕЛЬКА, 15 января 2011, 14:04:54
НУ ПОМОГИТЕ ДОБРЫЕ ЛЮДИ........

2796. Lexxus, 15 января 2011, 14:23:29
найдите наибольшее значение функции у=3х-3tgх-8 на отрезке


Функция y=tg(x) - всюду возрастающая, и возрастает она всюду быстрее, чем функция y=x.
Поэтому функция 3х-3tg(х) всюду убывает, а значит, наибольшего значения достигает на левой границе отрезка.
3*0-3*tg(0)-8 = -8

2797. ГЕЛЬКА, 15 января 2011, 14:36:13
из пунктаА в пункт В по течению реки отправились одновременно моторная лодка и байдарка. скорость течения реки равна 3 км/ч. последнюю 1/7 часть пути моторная лодка шла с выключеным мотором, и ее скорость относительно берега была равна скорости течения. на той части пути, где моторная лодка шла с выключеным мотором, ее скорость была на 2км/ч больше скорости байдарки. найдите скорость байдарки в неподвижной воде, если в пункт В байдарка и моторная лодка прибыли одновременно.??????

2798. гелька, 15 января 2011, 14:37:04
вообще не умею решать такие задачи((((((

2799. ilo, 15 января 2011, 17:12:52
гелька, 13 января 2011, 16:29:18
радиус основания первого круга в 3 раза больше, чем радиус сонования второго конуса , а образующаа первого конуса в 2 раза меньше, чем образующаа второго. чему равна площадь боковой поверхности первого конуса, если площадь боковой поверхности второго равна 18 (см квадратных)
S б.п. конуса=pirl - прямопропорциональна r и l. Если r 1-го основания увеличить в 3 раза, то и Sб.п. увеличится втрое, если l уменьшить вдвое, то и Sб.п. уменьшится вдвое.Т.о.18 надо умножить на 3 и разделить на2, т. е умножить на 3/2.

2800. ilo, 15 января 2011, 17:32:14
Evgetor, 15 января 2011, 11:00:58

насчёт вот этого задания чо скажете
Vц.=1125pi, выделенная часть составляет 1/6 Vц.=187,5

2801. ilo, 15 января 2011, 17:37:14
ГЕЛЬКА, 15 января 2011
посмотри пост 2367 может это -то

2802. ilo, 15 января 2011, 18:21:46
гелька, 13 января 2011, 15:03:51
первую треть трассы автомобиль ехал со скоростью 80 км/ч, вторую треть со скоростью 30км/ч, а последнюю со скоростью 60 км/ч. найдите среднюю скорость автомобиля на протяжении всего пути. ответ дайте в км/ч. ПЛИЗЗЗ
Время, затраченное на первую треть пути-(1/3S)/80;вторую-(1/3S)/30;третью треть-(1/3S)/60.Всё время-S/240+S/180+S/180=S/48; v=48 (Скорость средняя находится делением всего пути на всё время: S:S/48=48)

2803. ГЕЛЬКА, 15 января 2011, 19:32:55
из пунктаА в пункт В по течению реки отправились одновременно моторная лодка и байдарка. скорость течения реки равна 3 км/ч. последнюю 1/7 часть пути моторная лодка шла с выключеным мотором, и ее скорость относительно берега была равна скорости течения. на той части пути, где моторная лодка шла с выключеным мотором, ее скорость была на 2км/ч больше скорости байдарки. найдите скорость байдарки в неподвижной воде, если в пункт В байдарка и моторная лодка прибыли одновременно.?????? ПОМОГИТЕ

2804. Звёздочка, 16 января 2011, 11:34:35
Дан куб ABCDA1B1C1D1 Найти косинус между прямыми CD и BD1. Помогите пожалуйста:)

2805. гелька, 16 января 2011, 13:46:43
ааааа

2806. Несчастный школьник Хех), 16 января 2011, 18:04:29
Найдите наименьшее значение функции y=5x-5tgx+4 на отрезке(-п:4;о)

2807. Леонид , 16 января 2011, 18:38:57
2806. Несчастный школьник Хех), 16 января 2011. Смотри запись 2796 - похожее.

2808. ilo, 16 января 2011, 19:33:39
Звёздочка, 16 января 2011, Помогите пожалуйста:)
Угол между ВD1 и CD равен углу между ВD1 и AB параллельной CD.Из треугольника ABD1 найти угол АВD1 по теореме косинусов: 2a^2=3a^2+a^2-2sqrt3a^2cosABD1

2809. геля, 16 января 2011, 20:24:51
а мене даже и не помогает никто........ ну и ладно чо блин!!

2810. Aja, 17 января 2011, 16:33:11
есть вопрос:
в разделе "совсем без ответов" задания, которые никто не решил или к которым по неопределенным проблемам никто не может написать ответ, который получили?

например в задании 5037. это B9. у меня получилось 9. и очень даже просто.

разъясните ситуацию. )

2811. Lexxus, 17 января 2011, 17:20:37
в разделе "совсем без ответов" задания, которые никто не решил или к которым по неопределенным проблемам никто не может написать ответ, который получили?

Чаще всего это задания, которые просто никто не пытался решать. В базе всё еще есть больше 2500 заданий, которым вообще пока не повезло хоть кому-нибудь попасться (то есть они гарантированно входят в группу "совсем без ответов").

2812. Aja, 17 января 2011, 23:17:04
спасибо, все ясно )

2813. Юлия, 18 января 2011, 14:45:54
помогите решить :в треугольнике АВС угол С равен 90 градусов,синус В равен 6 корней из 6. Найдите АС


2814. ilo, 18 января 2011, 16:14:59
Юлия, 18 января 2011, Синус таким не бывает.

2815. надежда2010, 18 января 2011, 19:59:22
люди, помогите: Решить чсистему : Хквадрат -2Z= -Уквадрат -2
Z + У меньше или равно Х

2816. надежда2010, 19 января 2011, 11:07:39
Несчастный школьник, большинство заданий В-11 на наиболшее или наименьшее значение можно решить так: поскльку в ответе не должно быть никаких корней, пи, е, и т.д., то просто бери и подставляй в формулу функции какойлибо конец промежутка данного или точку внутри него, она легко просматривается ( чтобы убрался логарифм или е), а для синуса(косинуса) это чаще всего ноль, для тангенса может быть и пи/4. Проверь! Работает!

2817. Юлия, 19 января 2011, 16:15:12
Блин всю задачу попутала ,простите пожалуйста,плиз
В треугольнике АВС угол С равен 90 градусов синус В =0,2,ВС = 6 корней из 6 Найти АС

2818. Ириска, 19 января 2011, 17:23:04
Ребята, помогите пожалуйста!Буду очень вам благодарна!
Из вершины G треугольника GHP проведен перпендикуляр GQ. Из точки Q опущен перпендикуляр на сторону HP. Найдите условие, при котором этот перпендикуляр пройдет через одну из вершин H или P треугольника.

Из вершины угла к его плоскости проведена наклонная, которая составляет со сторонами угла равные углы. Докажите, что ортогональной проекцией этой наклонной является биссектриса данного угла.

Докажите, что ортогональная проекция угла на плоскость, проходящую через одну из его сторон, меньше, равна или больше данного угла, смотря по тому, является ли данный угол соответственно острым, прямым или тупым

2819. ilo, 19 января 2011, 19:01:16
Юлия, 19 января
из основного тригонометрического тождества найти cos^2B=1-(1/5)^2=24/25;
cos B=(2sqrt6)/5; Затем tgB=1/(2sqrt6) или sqrt6/12; СА/СB=tgB; CA=CBtgB=3; sqrt6-корень из 6.

2820. Игорь, 19 января 2011, 20:22:02
Привет Леонид, как дела? С Новым Годом кстати!!!

2821. Игорь, 19 января 2011, 21:47:55
Помогите пжл...

В11:Найти наибольшее значение функции.

y=ln(7x)-7x+7 на отрезке [1/14;5/14]

B12:Расстояние между пристанями A и В равно 48 км.Отчалив от пристани А в 10 часов утра, теплоход проплыл по течению реки с постоянной скоростью до пристани В. После трехчасовой стоянки у пристани В теплоход отправился в обратный рейс и прибыл в А в тот же день в 22.00.
Найдите скорость теплохода в неподвижной воде, если скорость течения реки равна 4 км/ч.

Заранее спасибо!

2822. Мария, 19 января 2011, 22:30:26
спасибою

2823. Леонид , 20 января 2011, 08:39:14
Игорю! y=ln(7x)-7x+7 на отрезке [1/14;5/14] - наибольшее зн-е ф-и?
1) y" = 1/(7x)*7 - 7 = 1/x - 7. 2) 1/x - 7 = 0, x=1/7 = 2/14 - знаменателям на отрезке - видно, что принадлежит отрезку. 3) значения данной функции в 1/14, 2/14, 5/14 - подставляем ф функцию: 1) у(1/7) = ln(7 * 1/7) - 7 * 1/7 +7 = 0 -1+7 = 6. 2) y(2/7)=ln(7 * 2/7) - 7 * 2/7 + 7 = ln2+5 -не красивый ответ. 3) y(5/7) = ln(7 * 5/7) - 7 * 5/7 + 7 = ln5 +2 - не красивый ответ. Ответ:1) 6

2824. Игорь, 20 января 2011, 09:06:20
Списибо большоe Лeонид,выручил.

2825. надежда2010, 20 января 2011, 09:56:27
Леонид, я всетаки еще раз пишу свой способ (см 2816) В-11 в нахождении наибольшего(наименьшего) значения, экономит время и силы, работает в 99 случаях из 100: у=ln(7x)-7x+7 на отрезке[1/14; 5/14]- ясно, что красивый ответ на концах промежутка не получится, а внутри только одна точка есть, чтобы логарифм убрался, это2/14, подставляеми получаем 6. Это способ для "самых умных", многие и производную не найдут, не то что исследовать функцию. Просьба: посмотри систему в 2815, не соображу, как к ней подойти...Пожалуйста...

2826. Игорь, 20 января 2011, 10:27:24
Леонид,поможешь разобраться с В12 пост номер 2821?

2827. alfa20, 20 января 2011, 10:40:15
Надежда2010. Попробуй так:
Из уравнения: ,{(2z=x^2+y^2@2z&#8804;2x-2y,)&#9508;тогда
x^2+y^2+2&#8804;2x+2y ,
(x-1)^2+(y+1)^2&#8804;0,тогда (т.к.сумма квадратов неотрицательна) x=1,y=-1,z&#8804;2

2828. Юлия, 20 января 2011, 14:31:46
Помогите пожалуйста :Кузнец куёт уникальные кинжалы,пробивающие любую кольчугу. Зависимость объёма спроса на продукцию q(единиц в месяц) от её цены p(руб) задаётся формулой q=442-17p. Определите максимальный уровень цены p(руб),при котором значение выручки за месяц r=q*p составит не менее 1173

2829. ilo, 20 января 2011, 15:37:23
Юлия, 20 января
q из формулы подставить в значение выручки и решить неравенство (442-17p)*p>=1173; p^2-26p+69<=0; D=400; p1=3;p2=23;решением неравенства будет отрезок[3;23]Максимальное значение p на этом отрезке 23

2830. Леонид , 20 января 2011, 21:10:29
2825. надежда2010, 20 января 2011, 09:56:27
Там есть даже почти устные, когда виден знак проиводнй. И на Ваши 99% случаев надеяться нельзя! как правило большинство попадают в 1%. Не обманывайте народ.

2831. Юлия , 21 января 2011, 19:53:57
помогите решить:два велосепедиста одновременно отправились в пробег протяженностью 84 км. Первый ехал со скоростью на 5км/ч большей скорости второго,и прибыл к финишу на 5 часов раньше второго.Найти скорость велосипедиста ,пришедшего к финишу вторым

2832. Марина, 21 января 2011, 20:12:56
Всем добрый вечер!! Нуждаюсь в помощи)). С6. Произведение нескольких различных простых чисел делится на каждое из этих чисел, уменьшенное на 1. Чему может быть равно его произведение? Спасибо.

2833. Lexxus, 21 января 2011, 21:13:49
С6. Произведение нескольких различных простых чисел делится на каждое из этих чисел, уменьшенное на 1. Чему может быть равно его произведение?

Очень просто.
Произведение нескольких различных простых чисел может делиться только на эти же самые простые числа и на единицу.
Это значит, что каждое из этих простых чисел, уменьшенное на единицу, является либо другим простым числом из набора, либо произведением нескольких из них, либо единицей.
Единственное простое число, при уменьшении которого на единицу получается также простое число - это 3.
А единственное простое число, при уменьшении которого на единицу получается единица, - это 2.
Так что, первый ответ - 2*3 = 6.

Следующий ответ может получиться, если предыдущий ответ, увеличенный на единицу, является простым числом.
6+1 = 7 - это простое число, поэтому второй ответ - 2*3*7 = 42.

Следующим членом произведения может стать либо 2*7+1, либо 3*7+1, либо 2*3*7+1, если это простые числа. 2*7+1=15, 3*7+1=22 - не простые.
2*3*7+1 = 43 - а вот это простое число (тут уж придется проверять, перебирая делители).
Значит, третий ответ - 2*3*7*43 = 1806.

Чтобы доказать, что больше таких чисел нет, надо убедиться, что
2*43+1, 3*43+1, 7*43+1, 2*3*43+1, 2*7*43+1, 3*7*43+1 и 2*3*7*43+1 - не простые числа.

2834. (: ДИАНА :), 21 января 2011, 22:12:53
пож-та помогите. Найти все значения а, такие, что для любого х выполняется неравенство |х+1|+2|х+а|> 3-2х.

2835. ilo, 21 января 2011, 22:22:32
Юлия , 21 января
x-скорость того,кто пришёл вторым; x+5-первым;время первого 84/(х+5);второго 84/х. Уравнение 84/х-84/(х+5)=5; сводится к квадратному x^2+5x-84=0 откуда х=12

2836. ilo, 21 января 2011, 23:59:54
(: ДИАНА :), 21 января
перепиши в виде:2x+|x+1|+2|x+a|>3, левую часть обозначаешь F(x), контрольные точки,которые в 0 модуль обращают-x=-1 и x=-a.Рассмотри 2 случая:1)-а лежит левее -1, т.е.-а<=-1, т.е. а>=1 2)-a лежит правее -1, т.е. -а>-1,т.е.а<1. При а>=1 функция до -а убывает,после возрастает, т.е.в -а имеет минимум.При а<1 минимум в точке -1.Чтобы F(x) >3 для любого х, минимум должен лежать выше 3, т.е. F(-a)>3 это при а>=1и F(-1)>3 при а<1.Решаешь неравенства, получаешь при а<-3/2 F(x)>3 для любых х.

2837. Куки, 22 января 2011, 08:00:59
Два ребра прямоугольного параллелепипеда выходящие из одной вершини равны 11 и 15. Площадь поверхности этого параллелепипеда равна 538.найдите третье ребро выходящее из той же вершины

2838. Куки, 22 января 2011, 08:27:17
Найдите наименьшее значение функции у=2cos x - 13x + 4 на отрезке [-3п деленая на 2;0]

Помогите решить!!!

2839. (: ДИАНА :), 22 января 2011, 17:20:42
Спасибо огромное!!!!!!!!!!!!!!!

2840. Василиса, 22 января 2011, 17:40:20
здравствуйте. очень очень прошу помочь с решением. заранее благодарю :)Решите в натуральных числах уравнение n!+5n+13=k^2, где n! = 1*2*...*n -произведение всех натуральных чисел от 1 до n.

2841. Леонид , 23 января 2011, 07:55:27
y* = -2sin x - 13 - очевидно, производная отрицательная. Значит функция убывающая - ищите наименьшее значение функции, подставляя в её формулу леевую границу промежутка, х = 0. у(0) = 2 + 4 = 6 - ответ. Это случай, когда задание выполняется, практически, устно. Для Куки

2842. Анастасия, 23 января 2011, 12:23:10
Здравствуйте. Помогите пожалуйста с задачей.
В треугольнике АВС угол С прямой. tgA=(9 на корень из 19)/19. найти sinA

2843. zvedya, 23 января 2011, 16:01:24
кто ни будь помоги решить задачу...
из части в9.
Объем правильной шестиугольной пирамиды 6.Сторона основания равна 1.найти боковое ребро.

решала решала и не дорешала...

2844. zvedya, 23 января 2011, 16:08:55
это задание С3

log основания х+2 числа(36+16х-х в квадрате)-1/16 log в квадрате основания х+2 числа(х-18)в квадрате больше или равно 2

2845. ilo, 23 января 2011, 16:27:42
Василиса, 22 января
Если число в левой части рассматривать при n>=5, то n! делится на 2 и 5,т.е оканчивается 0, а само число оканчивается 3 или 8.В правой части квадрат натурального числа, который не может оканчиваться 3 или 8.Остаётся перебрать натур. числа от 1 до 4.Ответ n=2,k=5.

2846. Лилька, 23 января 2011, 16:32:03
помогите пожалуйста решить!

Пусть sinx-cosx=1,1 Найдите sin2x.

2.Найдите наибольшее значение функции y=4*sinx-3cosx

3.При каком наименьшем a один из корней уравнения x^2-2*a*(x-2)-4=0 в 5 раз больше другого?

4.Найдите остаток от деления многочлена x^21+x^10+x^5+3 на двучлен x-1



2847. ilo, 23 января 2011, 16:41:47
Анастасия, 23 января
sin^2A=tg^2(A)/(1+tg^2(A)); sinA=0,9.

2848. Лилька, 23 января 2011, 17:21:50
помогите пожалуйста)))
срочно надо!!

2849. ilo, 23 января 2011, 17:57:41
zvedya, 23 января
задание С3
log_(18-x)(x+2)-1/4log^2_|x-18|-2>=0;
Раскрываем модуль, x>18 не рассматриваем,т.к. выр-е под знаком log в этом случае отрицательно; при x<18: log_(18-x)+1-1/4lod^2_(18-x)-2>=0; Замена: log_(18-x)=t; (t-2)^2<=0;t=2; (x+2)^2=18-x; x1=-7-не удовл усл.x+2>0; x2=2; Отв.x=2.

2850. zvedya, 23 января 2011, 20:43:20
В4
в треугольнике АВС угол равень 90 градусов,АВ равен 10,ВС равен 8 найти cosA

2851. Анастас, 24 января 2011, 10:59:58
Всем доброго времени суток!!! Рубят, помогите кто нить решить задачу, очень срочно надо!!! буду о4 признательна, если поможете!!!!! вот задача: за время тренировки гоночные автомобили ехали с постоянной скоростью. при этом первый автомобиль, пройдя 76 кругов обогнал второй на 3 круга. Третий автомобиль стартовал на 953 секунд позже, чем первые два и проехал до окончания тренировки на 10776 метров меньше, чем первый. А если бы он выехал одновременно с первым, то обогнал бы его на 27371метров. Скорость второго автомобиля была равной 130 км/ч. Чему была равна скорость первого и третьего автомобилей? Сколько минут длилась тренировка?

2852. Света, 24 января 2011, 17:44:45
В арифметической прогрессии an найдите:
n и а1, если аn = -7,5, d= -2,5, Sn=75.

2853. Айжан, 25 января 2011, 18:31:57
катер плыл по течению от пристани А до пристани В 3 часа , обратно от пристани В до пристани А плыл 4 часа с той же собственной скоростью.Сколько часов будет плыт0ь это расстояние плот??
а)16 часов. в)24 часа. с)20 часов. д)15 часов.

2854. Вася, 26 января 2011, 10:44:43
Камень брошен вертикально вверх. Пока камень не упал, высота, на которой он находится, описывается формулой h(t)=-5t^2+18t (h — высота в метрах, t — время в секундах, прошедшее с момента броска). Найдите, сколько секунд камень находился на высоте не менее 9 метров. Ну ответьте кто-нибудь!?

2855. cer34, 26 января 2011, 12:24:55
В треугольнике абс твои с равен 90 градусов.косинус а равен 0.1 , бс равен 3корней из 11 .найти аб

2856. ilo, 26 января 2011, 12:57:31
Вася,
-5t^2+18t>=9; 5t^2-18t+9<=0; 0.6<=t<=3; 3-0.6=2.4

2857. ilo, 26 января 2011, 13:06:29
cer34, 26 января
sinA=sqrt(1-0.01)=sqrt(99/100)=(3sqrt11)/10; sqrt- корень из...
sinA=BC/AB; AB=BC/sinA=10;

2858. ilo, 26 января 2011, 13:25:06
Света, 24 января
В формулу n-го члена an=a1+(n-1)d подстравляешь an и d, выражаешь а1=2,5n-10; подставляешь в формулу суммы , получишь квадратное уравнение n^2-7n-60=0; n=17;a1=32,5

2859. ((ВерОника)), 26 января 2011, 16:35:45
Помоги те решить! «в треугольнике АВС стороны ВС образуют с основанием АС угол равный 30 градусов.а высота проведенная из вершины В делит основание на отрезки AD=12см и DC=5корней из 3 . найдите боковые стороны треугольника»

2860. зуля, 26 января 2011, 18:49:36
помогите решить-) плииз))радиус основания первого конуса в три раза меньше, чем радиус основания второго конуса, а образующая первого конуса в два раза больше, чем … Чему равна площадь боковой поверхности конуса, если площадь боковой поверхности второго равна восемнадцати см^2.

2861. ilo, 27 января 2011, 00:01:35
зуля, 26 января S б.п. конуса=pirl - прямопропорциональна r и l. Если r 1-го основания увеличить в 3 раза, то и Sб.п. увеличится втрое, если l уменьшить вдвое, то и Sб.п. уменьшится вдвое.Т.о.18 надо умножить на 3 и разделить на2, т. е умножить на 3/2.

2862. =)) , 28 января 2011, 07:42:53
здравствуйтее =)) очень пожалуйста,помогитее..(в основании прямой призмы АВСА1В1С1,лежит прямоугольный треугольник АВС ,угол С=90 градусов,АС=4см,ВС=3см,.Через сторону АС и вершину В1 проведена плоскость,уголВ1АС=60.Найти площадь боковой поверхности призмы. =))

2863. ilo, 28 января 2011, 21:35:39
AC перпендикулярна CB1, т.к. перпендикулярна её проекции CB. следовательно треугольник ACB1 прямоугольный B1C=ACtgB1AC; BB1 найти из BCB1 по теореме Пифагора.Площадь боковой поверхности=произведению периметра основания на BB1.

Log2(6+x)=4
Решите пожалуйста,а то я забыл как это решается

2865. Леонид , 29 января 2011, 01:16:34
Здорово, "Серый". Что же так? 2^4 =6+x, 16 = 6+x, x = 6. Если это ЕГЭ-шный пример - ответ 6 и ничего не надо делать, т.к. корень в единственном числе. Вообще-то надо проверить, или найти ОДЗ: 6+x>0, x>-6. Всё нормальненько, т.к. корень 6 больше (-6). Уловил всё, Серый?

2866. Сергей26, 29 января 2011, 10:49:31
Перед каждым из чисел 11,12, ..., 19 и 2,3, ..., 6 произвольным образом ставят знак плюс или минус,после чего от каждого из образовавшихся чисел второго набора,а затем все 45 полученных результатов складывают.Какую наименьшую по модулю и какую наибольшую сумму можно получить в итоге?


Решите пожалуйста оч над

2867. ksana, 30 января 2011, 16:09:56
в треугольнике АБС; СН-высота; АД-биссектриса.
угол БАД =25 градусов. Найти угол АОС.

помогите решить.

2868. lera, 30 января 2011, 20:05:28
x^1+lg x = 10x
помогите решить пожалуйста..

2869. Леонид , 30 января 2011, 21:04:36
Числа равны, равны и их логарифмы по одному основанию. Прологарифмируем, здесь, по основаию 10, т.к. здесь уже имеется такой логарифм.Да, (1+lgx) - это всё, думаю, в показателе? Будем иметь - cм. самое 1-е предлжение: lg(x^(1+lgx)) =lg(10x). Логарифм степени слева и логарифм произведения справ: (1+lgx)*lgx = lg10 +lgx, lgx+lg^2 x=1+lgx, lg^2 x=1, lgx=1 или lgx = -1, x = 10 или х=1/10 - записали по определению логарифма. Оба подходят по ОДЗ или сделайте проверку.

2870. Юлия, 31 января 2011, 16:58:18
помогите пожалуйста решить задачу:Моторная лодка прошла против течения реки 144км и вернулась в пункт отправления,затратив на обратный путь на 3 часа меньше. Найдите скорость лодки в неподвижной воде,если скорость течения равна 2км/ч

2871. ;), 1 февраля 2011, 10:12:48
В ответе к задаче 5055 ошибка!!! Площадь ромба равна половине произведения диагоналей.

2872. Татьяна, 1 февраля 2011, 10:32:46
Помогите,пожалуйста.
в треугольнике АВС угол С равен 90 градусов, ВС=6,АС=10. Найдите синус В.
В ответе 0,8.
а с моим решением не сходиться.(

2873. ARA, 1 февраля 2011, 11:22:13
2870. Юлия, 31 января 2011, 16:58:18
помогите пожалуйста решить задачу:Моторная лодка прошла против течения реки 144км и вернулась в пункт отправления,затратив на обратный путь на 3 часа меньше. Найдите скорость лодки в неподвижной воде,если скорость течения равна 2км/ч

А ОТВЕТ 14???

2874. ARA, 1 февраля 2011, 11:28:04
ТАТЬЯНА , ЧТО ТО НЕ ТО , ДЛЯ ЭТОГО НАМ НАДО БУДЕТ НАЙТИ AB, AB - ЭТО ГИПОТЕНУЗА, ЧТОБЫ ЕГО НАЙТИ НАДО ВОЗВЕСТИ КАТЕТЫ В КВАДРАТ И СЛОЖИТЬ .... ЕСЛИ ЭТО СДЕЛАЕМ ТО ПОЛУЧИМ КОРЕНЬ ИЗ 136, НО ЭТО НЕ ИЗВЛЕКАЕТСЯ...., ЧТО Т В УСЛОВИИ НЕ ПРАВИЛЬНО.

2875. ARA, 1 февраля 2011, 11:41:04
в треугольнике АВС угол С равен 90 градусов, ВС=6,АС=10. Найдите синус В.

ТАТЬЯНА Я ДУМАЮ , ВМЕСТО AC ДОЛЖНО БЫТЬ AB, ТОГДА ЭТО ПИФАГОРОВА ТРОЙКА ПОЛУЧАЕТСЯ. И AC=8. И ВСЁ. Синус - противолежащий КАТЕТ на гипотинузу. SINB=AC/AB, SIND=6/10=0.6

2876. ARA, 1 февраля 2011, 11:44:32
ТОЧНЕЕ SINB=8/10=0.8

ПРОСТО БЫСТРО ПИСАЛ , НЕ ЗАМЕТИИЛ..

2877. Татьяна, 1 февраля 2011, 12:23:43
ARA,спасибо большое,я просто так и подумала,что опечатка)

2878. Ксю, 1 февраля 2011, 13:57:28
Сплав меди с цинком, содержащий 80 г цинка, сплавлен со 100 г цинка. В результате содержание цинка в сплаве повысилось по сравнению с первоначальным на 20%. Сколько граммов меди в сплаве?

2879. Светлана, 1 февраля 2011, 18:07:37
Помогите, пожалуйста решить)
Найдите наибольшее значение функции y=11cosx+12x-7 на отрезке [-3п/2; 0]

2880. ARA, 1 февраля 2011, 19:50:53
Ксю А МОЖНО ОТВЕТ УЗНАТЬ???

2881. ARA, 1 февраля 2011, 20:11:09
Светлана!!
Найди производную !
после этого приравняй к нулю, получается что корней нет, значит и нет точек экстремума , значит наиб. значение = y(0)=11*cos(0)+12*0-7=11-7=5!!

2882. Иван, 1 февраля 2011, 21:58:22
Ребята, помогите пожалуйста решить: в правильной треугольной пирамиде SABC с основанием ABC известны ребра: AB=корень из трёх, SC=2корня из трех. Найдите угол, образованный плоскостью основания и прямой CN, где N - середина ребра AS.

2883. СЕРГЕЙ, 1 февраля 2011, 23:06:33
помогите пожайлуста решить...В ПРЯМОУГОЛЬНОМ ТРЕУГОЛЬНИКЕ АВС С ПРЯМЫМ УГЛОМ С ГИПОТЕНУЗА АВ РАВНА 18/НА КОРЕНЬ ИЗ 5 А КОСИНУС А =2/3.НАЙДИТЕ ВЫСОТУ СД ЭТОГО ТРЕУГОЛЬНИКА..... ПОМОГИТЕ ПОЖАЛУЙСТА

2884. Анастасия, 3 февраля 2011, 09:54:28
2883. СЕРГЕЙ, 1 февраля 2011, 23:06:33
помогите пожайлуста решить...В ПРЯМОУГОЛЬНОМ ТРЕУГОЛЬНИКЕ АВС С ПРЯМЫМ УГЛОМ С ГИПОТЕНУЗА АВ РАВНА 18/НА КОРЕНЬ ИЗ 5 А КОСИНУС А =2/3.НАЙДИТЕ ВЫСОТУ СД ЭТОГО ТРЕУГОЛЬНИКА..... ПОМОГИТЕ ПОЖАЛУЙСТА

CD = 20 если нужно, могу расписать решение

2885. ARA, 3 февраля 2011, 11:06:03
АНАСТАСИЯ , А ТЫ УВЕРЕНА ЧТО 20???

У МЕНЯ ПОЛУЧИЛОСЬ 4......

2886. Helen, 3 февраля 2011, 11:22:04
ответ № 4671 должен быть 28

2887. ARA, 3 февраля 2011, 11:29:52
2886. Helen, 3 февраля 2011, 11:22:04
ответ № 4671 должен быть 28

чё за ответ????
эт какая задача?

2888. Анастасия, 3 февраля 2011, 14:42:55
2885. ARA, 3 февраля 2011, 11:06:03
АНАСТАСИЯ , А ТЫ УВЕРЕНА ЧТО 20???

У МЕНЯ ПОЛУЧИЛОСЬ 4......

Нам дана гипотенуза и косинус угла А. Косинус А=АС/АВ. Следовательно можно составить пропорцию АС/АВ=2/3. подставим значение АВ, получается АС/(18*на корень из 5)=2/3. АС=12* на корень из 5. теперь рассмотрим треугольник АСД. он прямоугольный т.к. СД-высота, следовательно Синус А=СД/АС. теперь найдем синус А. он равен корень квадратный (1 - косинус в квадрате А).
Синус А = корень (1 - 4/9) = (корень из 5)/3.
СД/(12*на корень из 5) = (корень из 5)/3.
СД = (12*корень из 5 * на корень из 5)/3=(12*5)/3 = 20

2889. Юлия, 3 февраля 2011, 16:19:34
Да ответ 14

2890. ARA, 3 февраля 2011, 17:12:56
АНАСТАСИЯ, у нас гипотенуза это 18 разделить на корень из пяти, а у тебя умножить...

2891. ARA, 3 февраля 2011, 17:19:48
ща я моё решение напишу....

2892. ARA, 3 февраля 2011, 17:33:22
гипотенуза = 18 разделить на корень из 5. Косинус угла А=2 разделить на 3!!. Косинус А=АС/АВ. АС/АВ=2/3. получим АС/(18 РАЗДЕЛИТЬ на корень из 5)=2/3. АС=12 РАЗДЕЛИТЬ на корень из 5.
2)находим синус А , sinA=1-cos^2A, sinA=1-4/9, sinA=корень из 5 / на 3
3)треугольник ACD ПРЯМОУГ. sinA=CD/AC, корень из 5 / на 3 = CD/ на 12/на корень из пяти. ( корень из пяти пойдёт вверх) и получаем.
CD * 3 * на корень из 5 = корень из 5 * 12, сокращаем всё на 3 * корень из пяти. получаем CD=4

ОТВЕТ: 4

2893. Анастасия, 3 февраля 2011, 18:06:55
Ой да, тогда 4. Извиняюсь за ложную информацию!

2894. Анастасия, 3 февраля 2011, 18:09:34
ход решения то был правильным, единственное подставила не то значение и ответ получился другой

2895. ARA, 3 февраля 2011, 18:57:27
я с тобой полностью согласен!

2896. АлИсКа, 4 февраля 2011, 17:04:44
Помогите решить!
В понедельник акции компании подорожали на некоторое число процентов, а во вторник подешевели на то же число процентов.Врезультате они стали стоить на 4 % дешевле,чем при открытии торгов в понедельник.на сколько процентов подорожали акции в понедельник?

2897. АлИсКа, 4 февраля 2011, 17:12:08
Помогите решить,ПоЖаЛуЙсТа!
Скорость автомобиля,разгоняющегося с места старта по прямолинейному отрезку пути длино L км с постоянным ускорением А км/ч^2,вычисляется по формуле скорость=под корнем:2LА.определите наименьшее ускорение,с которым должен двигаться автомобиль,чтобы,проехав 0,8 км,приобрести скорость не менее 100 км/ч.ответ выразите в км/ч^2/

2898. ARA, 4 февраля 2011, 19:01:55
АлИсКа, ПЕРВУЮ ЗАДАЧУ МОЖНО ПОПРОБОВАТЬ... , А ВТОРУЮ ВООБЩЕ НЕ МОГУ УСЛОВИЕ ПОНЯТЬ...

№1) ПУСТЬ ЭТО ЧИСЛО ПРОЦЕНТОВ НА КОТОРОЕ ПОДОРОЖАЛИ И ПОДЕШЕВЕЛИ = X. ТОГДА ПОЛУЧАЕМ УРАВНЕНИЕ (1+X/100)*(1-X/100)=(1-4/100)
ПО ФСУ ПОЛУЧАЕМ :
1 - X в квадрате / 10000 = 1 - 4/100
сокращаем единицы : X в квадрате /10000 = 4/100
40000 = 100*X В КВАДРАТЕ
X1=20
X2=-20 ПК.
ОТВЕТ : 20%

2899. Юлия, 4 февраля 2011, 20:11:26
помогите пожалуйста решить задачу:Моторная лодка прошла против течения реки 144км и вернулась в пункт отправления,затратив на обратный путь на 3 часа меньше. Найдите скорость лодки в неподвижной воде,если скорость течения равна 2км/ч


2900. Светлана, 4 февраля 2011, 20:55:01
помогите пожалуйста решить: в проугольном треугольнике с прямым углом С .. АВ=25, cosA=0,28. найдите ВС.

2901. Юлия, 4 февраля 2011, 21:53:13
Свет эт легко решать

2902. ARA, 4 февраля 2011, 22:14:35
ЮЛЯ ТВОЯ ЗАДАЧА ТОЖ, НЕ ОЧЕНЬ СЛОЖНАЯ...

2903. ONV, 5 февраля 2011, 17:16:36
Из города А в город B выехал грузовик, а через час следом за ним выехал легковой автомобиль. Через 2 часа после выезда легковой автомобиль догнал грузовик и приехал в пункт B на 3 часа раньше, чем грузовик. Сколько часов потратил на дорогу от А до B грузовик?
Помогите пжл...

2904. LORD, 5 февраля 2011, 21:31:39
http://live.mephist.ru/show/mathege2010/view/id/26656/


ЛЮДИ , А ПОЧЕМУ ТУТ ОТВЕТ НЕ ПРАВИЛЬНЫЙ?
НАДО 2, А ТУТ 3

2905. ilo, 5 февраля 2011, 23:26:21
LORD, ПОЧЕМУ ТУТ ОТВЕТ НЕ ПРАВИЛЬНЫЙ?
15-2x=9; 2x=6; x=3

2906. Наталья, 6 февраля 2011, 14:50:43

Помогите пожалуйста решить!!!
длина линии пересечения сферы и плоскости,проходящая через конец диаметра под углом 60 градусов равна 5п си. наити диаметр сферы.

2907. Натуля, 6 февраля 2011, 18:54:47
Помогите!!!!!
В боковой стенке цилиндрического бака вблизи дна закреплен кран.После его открытия вода начинает вытекать из бака,при этом высота столба в нем меняется по закону H(t)=4-0,4t+1100t2,где t-время в минутах ,H-высота в метрах.В течении какого времени вода будет вытекать из бака?

2908. Натуля, 6 февраля 2011, 19:52:41
там 1/100t (в кводрате)

2909. Игорь, 7 февраля 2011, 10:59:06
Помогите пожалуйста решить у-е
(lоg 4 по основанию х) - 1=4lоg sqrt х по основанию 2

2910. Lexxus, 7 февраля 2011, 12:07:19
(lоg 4 по основанию х) - 1=4lоg sqrt х по основанию 2

log_x(4) - 1 = 4 * log_2(sqrt(x)) - точно правильно? А то числа плохие получаются.

2 * log_x(2) - 1 = 2 * log_2(x)
2 / log_2(x) - 1 = 2 * log_2(x)

Пусть t = log_2(x)

2/t - 1 = 2t
2 - t = 2t^2

2t^2 + t - 2 = 0
D = 17

t1 = (-1 + sqrt(17))/4
t2 = (-1 - sqrt(17))/4

x1 = 2^((-1 + sqrt(17))/4)
x2 = 2^((-1 - sqrt(17))/4)

2911. Анастасия, 7 февраля 2011, 16:47:37
2897. АлИсКа, 4 февраля 2011, 17:12:08
Помогите решить,ПоЖаЛуЙсТа!
Скорость автомобиля,разгоняющегося с места старта по прямолинейному отрезку пути длино L км с постоянным ускорением А км/ч^2,вычисляется по формуле скорость=под корнем:2LА.определите наименьшее ускорение,с которым должен двигаться автомобиль,чтобы,проехав 0,8 км,приобрести скорость не менее 100 км/ч.ответ выразите в км/ч^2/

У меня получается ответ: 6250. но не уверена что такое возможно. был бы хоть самолет, а то автомобиль с таким ускорением. У вас нигде ответ нельзя посмотреть на эту задачу? если что решение могу написать

2912. Анастасия, 7 февраля 2011, 16:58:54
2899. Юлия, 4 февраля 2011, 20:11:26
помогите пожалуйста решить задачу:Моторная лодка прошла против течения реки 144км и вернулась в пункт отправления,затратив на обратный путь на 3 часа меньше. Найдите скорость лодки в неподвижной воде,если скорость течения равна 2км/ч

Составляем уравнение: 144/(x-2) - 144/(x+2) = 3.
Приводим к общему знаменателю и переносим 3 в левую часть:
(144(x+2) - 144(x-2) - 3(x-2)(x+2)) / (x-2)(x+2) = 0
Раскрываем скобки, приводим подобные и получается: 144x+288-144x+288-3x^2+12=0
-3x^2+588=0
x^2=196 x=14 км/ч

2913. Анастасия, 7 февраля 2011, 17:03:48
2900. Светлана, 4 февраля 2011, 20:55:01
помогите пожалуйста решить: в проугольном треугольнике с прямым углом С .. АВ=25, cosA=0,28. найдите ВС.

cosA = AC/AB. AC/AB=0,28
AC/25=28/100 AC = 28*25/100 = 7
по теореме Пифагора найдем BC. BC = корень квадратный (AB^2 - AC^2)
BC = корень квадратный (625 - 49) = корень квадратный из 576 = 24.
Ответ: 24

2914. Анастасия, 7 февраля 2011, 17:14:56
2907. Натуля, 6 февраля 2011, 18:54:47
Помогите!!!!!
В боковой стенке цилиндрического бака вблизи дна закреплен кран.После его открытия вода начинает вытекать из бака,при этом высота столба в нем меняется по закону H(t)=4-0,4t+1100t2,где t-время в минутах ,H-высота в метрах.В течении какого времени вода будет вытекать из бака?
2908. Натуля, 6 февраля 2011, 19:52:41
там 1/100t (в кводрате)

Наталья, вода будет вытекать по половине параболы, следовательно нужно это уравнение приравнять к 0.
4 - 0,4t + 0,01t^2 =0. Для удобства подсчетов разделим обе части на 0,01
Получаем: t^2 - 40t + 400 =0. Решаем квадратное уравнение, т.к. дискриминант равен 0, у нас получается один корень. t = 20
Ответ: вода будет вытекать в течение 20 минут

2915. Настя, 7 февраля 2011, 18:45:25
помогите решить) в треугольнике АВС АС=ВС, высота СН равна 1,5, sinА=3/5. Найдите АВ.

2916. Анастасия, 7 февраля 2011, 19:11:01
2915. Настя, 7 февраля 2011, 18:45:25
помогите решить) в треугольнике АВС АС=ВС, высота СН равна 1,5, sinА=3/5. Найдите АВ.

Рассмотрим треугольник ACH. sinA = CH/AC. sinA = 1,5/AC = 3/5
AC = 1,5*5/3 = 2,5.
Теперь по тереме Пифагора найдем AH. AH = корень квадратный (AC^2 - CH^2)
AH = корень квадратный (2,5^2 - 1,5^2) = корень квадратный (6,25 - 2,25) = 2.
Т.к. AC=BC, следовательно треугольник ABC - равнобедренный, значит CH и высота и медиана, следовательно AB = AH*2. AB=2*2=4
Ответ: AB=4

2917. Настя, 7 февраля 2011, 19:16:34
спасибо, поняла)
а аот эту: В треугольнике АВС АВ=ВС, высота СН равна 2,5, sinА=корень из 5/5. Найдите АВ.
Решаю так же, у меня не получается(((

2918. Настя, 7 февраля 2011, 19:17:59
ой, не АВ=ВС, а АС=ВС!)

2919. самое классное имя, 7 февраля 2011, 19:23:16
помогите плизики.... От старта отъезжают 2 машинки и через некоторое время движутся по законам S1(t) = 0,25 t и S2(t)=1,25t - 0,25t^2(квадрат) На каком расстоянии от старта они поравняются?
и ещё... В основании треугольной пирамиды с высотой h лежит правильный треугольник со стороной а. ПРи каком а пирамида еммет наибольший объём если 2а+h=5

2920. Анастасия, 7 февраля 2011, 19:27:34
2917. Настя, 7 февраля 2011, 19:16:34
спасибо, поняла)
а аот эту: В треугольнике АВС АВ=ВС, высота СН равна 2,5, sinА=корень из 5/5. Найдите АВ.
Решаю так же, у меня не получается(((
2918. Настя, 7 февраля 2011, 19:17:59
ой, не АВ=ВС, а АС=ВС!)

решение абсолютно тоже, давай только разбиремся с вычислениями.
корень из 5/5 = 2,5/AC. AC = 12,5/корень из 5. избавляемся от иррациональности, то есть домножаем на корень из 5,
получается (12,5*корень из 5)/5=2,5*корень из 5.
AH = корень квадратный (AC^2-CH^2) AH=корень квадратный (31,25-6,25) = 5
AB=10

2921. Анастасия, 7 февраля 2011, 19:30:20
2919. самое классное имя, 7 февраля 2011, 19:23:16
помогите плизики.... От старта отъезжают 2 машинки и через некоторое время движутся по законам S1(t) = 0,25 t и S2(t)=1,25t - 0,25t^2(квадрат) На каком расстоянии от старта они поравняются?

Скажите, это какое задание из В части? В9, В10?

2922. самое классное имя, 7 февраля 2011, 19:41:04
B9 это кажется

2923. Анастасия, 7 февраля 2011, 19:43:11
интересные у вас задания, что одно, что второе. ниразу такие не встречала. про пирамиду, на мой взгляд некорректно задание составлено. у меня не получается конкретного ответа

2924. Настя, 7 февраля 2011, 19:48:18
Анастасия, спасибо большое)запуталась в вычислениях, поэтому у меня и не получилось)))спасибо)

2925. самое классное имя, 7 февраля 2011, 19:50:54
про пирамиду мне тоже не нравиться....просто прикидкой число вроде бы между 1,5 и 2 получается
a=1, V= 0,66; а=1,5 V=1,5; a=1,6 V=1,536; a=2 v= 1,33
а как решить не понимаю хоть убейте.....

2926. Анастасия, 7 февраля 2011, 19:51:39
Не за что)) обращайтесь)

2927. Анастасия, 7 февраля 2011, 19:59:23
самое классное имя, у меня получается что примерно при 1,7 наибольшее.

2928. самое классное имя, 7 февраля 2011, 20:00:09
ну нужен наверно точный ответ...часть B как никак...и решение хочу :D

2929. Анастасия, 7 февраля 2011, 20:09:04
h=5-2a
V=1/3*Sоснования*H
S правильного треугольника = корень из 3*a^3 / 4
V=!/3*корень из 3*a^3 / 4 * (5-2a) = корень из 3*a^2(5-2a) / 12
Вот в конечное выражение подставляла разные значения a, при 1,7 V будет наибольшим. уже даже при 1,71 меньше

2930. Настя, 7 февраля 2011, 20:22:35
В треугольнике АВС угол С равен 90 градусов, sinА=о,31. Найдите соsВ.
sinA будет равен cosB? почему?)

2931. Анастасия, 7 февраля 2011, 20:31:11
2930. Настя, 7 февраля 2011, 20:22:35
В треугольнике АВС угол С равен 90 градусов, sinА=о,31. Найдите соsВ.
sinA будет равен cosB? почему?)

получается sinA=BC/AB отношение противолежащего катета к гипотенузе
cosB=BC/AB отношение прилежащего катета к гипотенузе
Вот почему равны

2932. Настя , 7 февраля 2011, 20:36:53
ну я совсем...)
спасиб)

2933. Саня, 7 февраля 2011, 21:11:16
Парни помогите решить задачу по геометрии 10 класс : Дан прямоугольный параллепипед ABCD A1B1C1D1. Найти двугранный угол B1ADB , если AC=6под корнем 2 , AB1=4 под корнем 3 , ABCD - квадрат !

2934. Саня, 7 февраля 2011, 21:12:24
И девушки !!!

2935. Игорь, 7 февраля 2011, 22:01:02
2910. Lexxus, 7 февраля 2011, 12:07:19

Да Лексус, извиняюсь написал неправильно но решил сам,можешь посмотреть правильно ли?

(log_x(2)-1=4log_2(sqrt х)
Решение
ОДЗ: x>0
(1/log_2(x))-1=4log_2(sqrt х)
(1/log_2(x))-1=4log_2(x^1/2)
(1/log_2(x))-1-2log_2(x)=0
1-log_2(x) - 2log^2_2(x)=0
Пусть log_2(x)=а тогда,
1-а-2a^2=0
2a^2+a-1=0
D=9
a1=1/2
a2=-1
Если а=1/2 то log_2(x)=1/2
x=sqrt2
Если a=-1 то log_2(x)=-1
x=1/2
Оба решение удовл. ОДЗ x>0

Ответ: sqrt2 и 1/2

2936. ilo, 8 февраля 2011, 14:38:16
самое классное имя
ну нужен наверно точный ответ...часть B как никак...и решение хочу :D
Задача на оптимизацию,а не В
Записываешь формулу объёма, в которую подставляешь h, выраженное через а.
получаешь фунцию V(a) ,от которой надо взять производную и найти наибольшее значение.Ответ:5/3

2937. ilo, 8 февраля 2011, 16:19:51
самое классное имя, помогите плизики.... На каком расстоянии от старта они поравняются?
Поравняются-значит пройдут равные пути.Приравняешь-получишь квадратное уравнение,найдёшь t=4,в любую формулу.S=1

2938. Светлана, 8 февраля 2011, 16:59:34
Помогите, пожалуйста)
в треугольнике abc угол c равен 90 sinA 3/5. найдите cos B

2939. Анастасия, 8 февраля 2011, 17:06:11
2938. Светлана, 8 февраля 2011, 16:59:34
Помогите, пожалуйста)
в треугольнике abc угол c равен 90 sinA 3/5. найдите cos B

sinA=cosB получается sinA=BC/AB отношение противолежащего катета к гипотенузе
cosB=BC/AB отношение прилежащего катета к гипотенузе
cosB=0,6

2940. Светлана, 8 февраля 2011, 17:17:20
Анастасия, большое спасибо)

2941. Валерия, 8 февраля 2011, 18:40:00
965. Lexxus, 8 апреля 2010, 20:17:40
Из пункта А в пункт В выехал мотоциклист и одновременно из В в А выехал автомобилист.Мотоциклист прибыл в В через 2 часа после встречи,а автомобилист в А через 30 минут после встречи.Сколько часов был в пути автомобилист?

Допустим, x - это часть пути, пройденная мотоциклистом (М) до встречи, а заодно и часть его полного времени в пути до момента встречи.
Тогда (1-x) - часть пути (и времени), пройденная автомобилистом (А) до встречи.
За tm и ta обозначим время в пути мотоциклиста и автомобилиста соответственно.

Мы знаем:
tm*x = ta*(1-x) (они выехали одновременно)
tm*(1-x) = 2 (М после встречи ехал еще два часа)
ta*x = 0.5 (А после встречи ехал полчаса)
Система из трех уравнений с тремя неизвестными.

Аккуратно выражаем из второго уравнения tm, из третьего - x, подставляем всё в первое и решаем его относительно ta.

НАПИШИТЕ ЭТО УРВНЕНИЕ ОТНОСИТЕЛЬНО tа, ПОЖАЛУЙСТА:)

2942. Made in Rай, 8 февраля 2011, 19:42:09
найдите наименьшее значение функции y=5ctg x +10x -2.5п +5 на промежутке (0;п/2]

2943. ilo, 8 февраля 2011, 23:49:18
Валерия,можно так:
0,5Va-путь до встречи мотоциклиста или путь после встречи автомобилиста;
2Vm-путь до встречи автомобилиста или путь после встречи мотоциклиста;
время до встречи автомобилиста-2Vm/Va равно
времени до встречи мотоциклиста-0,5Va/Vm;
0,5Va/Vm=2Vm/Va; 0,5Va^2=2Vm^2; Va^2=4Vm^2; Va=2Vm; откуда время до встречи автомобилиста 2Vm/Va=2Vm/2Vm=1. 1+2=3-был в пути автомобилист.

2944. Валерия, 9 февраля 2011, 12:43:40
ilo, спасибо))))))

2945. ilo, 9 февраля 2011, 15:55:50
Made in Rай y'=-5/sin^2(x) равна 0 в pi/4+pin/2-отсюда указанному промежутку принадлежит pi/4,y'не существует в 0, но 0 не из указанного промежутка.Ищем значение функции в pi/4 и в pi/2. В pi/4-10,в pi/2-5pi/2+5, наименьшее-10

2946. Мade in Raй, 9 февраля 2011, 16:16:23
Спасибо огромное,ilo))))

2947. настя, 9 февраля 2011, 16:57:23
Помогите пожалуйста решить)я решаю, у меня получается 1350, а ответ другой)

Интернет-провайдер предлагает три тарифных плана:

тарифный план Абонентская плата Плата за трафик
план "0" нет 3,5 рублей за 1 Мб
план "700" 750 руб. за 700 Мб 3 руб. за 1 Мб сверх 700 Мб
трафика в месяц
план "1000" 1050 руб. за 1000 Мб 2,5 руб. за 1 Мб сверх 1000 Мб
трафика в месяц

Пользователь планирует, что его трафик составит 900 Мб и, исходя из этого выбирает наиболее дешевый тарифный план. Сколько должен заплатить пользователь за месяц, если его трафик действительно будет равен 900 Мб?

2948. Кирилл, 9 февраля 2011, 18:36:07
Ответ - 1050 р. Надо выбрать тариф 1000

2949. Настенька), 9 февраля 2011, 23:08:16
Первый велосипедист выехал из поселка по шоссе со скоростью 18 км/ч. Через час после него со скоростью 15 км/ч из того же поселка в том же направлении выехал второй велосипедист, а еще через час после этого — третий. Найдите скорость третьего велосипедиста, если сначала он догнал второго, а через 4 часа 20 минут после этого догнал первого. Ответ дайте в км/ч.

2950. Ксения, 9 февраля 2011, 23:24:12
помогите решить!!!
диаметр окружности основания цилиндра равен 26, образующая цилиндра равна 21. Плоскость пересекает его основания по хордам длины 24 и 10. Найдите тангенс угла между этой плоскостью и плоскостью основания цилиндра!!!

плииииз,помогиииите

2951. АЛИК , 10 февраля 2011, 09:12:35
Лодка в 5:00 вышел из пунктаА в пунктВ,расположенный в 30 км от А. Пробыв в пункте В 2 часа, лодка отправилась назад и вернулась в пункт А в 23:00тогоже дня определите(в км/ч)собственую скорость лодки,если известно что скорость течения реки равна1км/ч


2952. ilo, 10 февраля 2011, 13:47:18
Ксения
Задача имеет два решения-когда хорды по одну сторону от оси цилиндра и по разные.В первом случае расстояние от центра окружности основания хорды длиной 24 равно5,а хорды длиной 10-12, между ними 5+12=17.Во втором случае 12-5=7.В первом случае tg равен 21/17 ,во втором 3.Всё находится по теореме Пифагора.

2953. Ксения, 10 февраля 2011, 14:45:25
ооо,ilo,спасибо вам огромное!!!

2954. Натуля, 10 февраля 2011, 15:53:39
Прямоугольный параллелепипед описан около цилиндра ,радиус основания и высота которого равны 11.Найдите объём параллепипида.

2955. артур, 10 февраля 2011, 18:49:47
вычеслить sin 300

2956. Ed, 10 февраля 2011, 20:29:44
Lexxus, исправь, пожалуйста, в задаче 27595 ответ с 30 на 50.
Площади относятся как _квадрат_ коэффициента подобия.

2957. Erofey, 10 февраля 2011, 21:42:39
Есть какаяни будь информация о егэ на 2011 год? Какая нисшая точка сдачи?

2958. Ed, 11 февраля 2011, 01:04:43
http://live.mephist.ru/show/mathege2010/view/id/27093 и http://mathege.ru:8080/or/ege/ShowProblem.html?probId=27093 - разные задачи!

2959. Олечка, 11 февраля 2011, 12:31:47
Пожалуйста! Покажите алгоритм решения В11...особенно где эти синусы, косинусы, тангенсы!!!

2960. ilo, 11 февраля 2011, 15:23:46
Олечка, например?

2961. Lexxus, 11 февраля 2011, 15:34:43
Lexxus, исправь, пожалуйста, в задаче 27595 ответ с 30 на 50.
Площади относятся как _квадрат_ коэффициента подобия.

Слушаю и повинуюсь.

http://live.mephist.ru/show/mathege2010/view/id/27093 и http://mathege.ru:8080/or/ege/ShowProblem.html?probId=27093 - разные задачи!

И правда, разные. Но не думаю, что это смертельно.

2962. Юлия, 11 февраля 2011, 19:02:16
помогите решить: в Треугольнике АВС АС=ВС,АВ=10,Sin A =12/13 Найдите высоту СН

И вот эту пожалуйста :Моторная лодка прошла по течению реки 140 км и вернулась в пункт отправления,затратив на обратный путь на 3 часа больше. Найдите скорость течения,если скорость лодки в неподвижной воде равна 17 км/ч

2963. ilo, 11 февраля 2011, 20:13:10
Юлия, 11 февраля
по основному тригонометрическому тождеству cosA=5/13; tgA=12/5;CH=AH*tgA;
CH=5*12/5=12
Задача:
140.(17-x)-140/(17+x)=3; сводится к квадратному уравнению 3*x^2+280x-867=0;
один корень отрицательный, второй=3;Ответ:3.

2964. ilo, 11 февраля 2011, 21:39:01
Юлия, 11 февраля
В задаче 140/(17-x)- опечатка

2965. Игорь, 11 февраля 2011, 22:08:12
Помогите решить!

В12: Из пункта А в пункт Водновременно выехфли 2 авто.первый проехал с постоянной скоростью весь путь.Второй проехал первую половину пути со скоростью меньшей скорости первого на 10 км/ч, а вторую половину пути со скоростью 63 км/ч, в результате чего прибыл в пункт В одновременно с первым авто.Найдите скорогсть первого авто если известно что она больше 30 км/ч.Ответ в км/ч

2966. Игорь, 11 февраля 2011, 22:15:29
В7: Найдите sin(7п/2-a) если sin a=0.8 и а принадлежит отрезку (п/2;п)

В9: В прямоугольном паралелепипеде два ребра выходящие из однойи той же вершины равны 2 и 8.Площадь поверхности паралелепипеда равна 132.Найдите третье ребро выходящее из той же вершины.

Эти задания были в пробном ЕГЭ 2011, сегодня писали репитиционную,помогите решить пжл!

2967. Lexxus, 12 февраля 2011, 13:09:54
В7: Найдите sin(7п/2-a) если sin a=0.8 и а принадлежит отрезку (п/2;п)

7п/2 = 2п+3п/2 = 3п/2
Мы знаем, что sin(3п/2-a) = -cos(a).
Поскольку а у нас лежит во второй четверти, то
cos(a) = -sqrt(1-(sin(a))^2) = -sqrt(1-0.64) = -0.6
sin(7п/2-a) = -cos(a) = 0.6


2968. Lexxus, 12 февраля 2011, 13:13:27
В9: В прямоугольном паралелепипеде два ребра выходящие из однойи той же вершины равны 2 и 8.Площадь поверхности паралелепипеда равна 132.Найдите третье ребро выходящее из той же вершины.

Если a, b, c - стороны параллелепипеда, то площадь его поверхности равна
2ab + 2ac + 2bc

2*2*8 + 2*2*c + 2*8*c = 132
32 + 20*c = 132
c = 5

2969. Азателло, 12 февраля 2011, 17:36:31
Люди добрые, помогите!!!
Не могу решить следующие задачи из части В9:
№ 27170
№ 27178
№ 27182
№ 27184
№ 27204
№ 27207
№ 27208
Буду очень благодарен, если Вы мне поможете!

2970. Игорь, 12 февраля 2011, 18:57:43
В12: Из пункта А в пункт Водновременно выехфли 2 авто.первый проехал с постоянной скоростью весь путь.Второй проехал первую половину пути со скоростью меньшей скорости первого на 10 км/ч, а вторую половину пути со скоростью 63 км/ч, в результате чего прибыл в пункт В одновременно с первым авто.Найдите скорогсть первого авто если известно что она больше 30 км/ч.Ответ в км/ч

2971. Люция, 12 февраля 2011, 19:33:41
Как и где скачать новый (на 2011) банк задач?

2972. Леонид , 12 февраля 2011, 21:45:00
х - ск. 1-го - найти её просят. (х-10) ск. 2-го на первой половине пути. Пусть весь путь 2 (без дробей чтобы обойтись). 1/(х-10) + 1/63 - време 2-го на половинках пути и на весь путь. Время 1-го 2/х. Уравнение: 1/(х-10) + 1/63 = 2/х. 63х +х(х-10) -2(х-10)63. x^2 -73x -1260=0....

2973. Игорь, 13 февраля 2011, 12:23:19
2972. Леонид , 12 февраля 2011, 21:45:00

Леонид, уравнение: x^2-73x+1260
D=289
x1=45
x2=30 - не удовл. условию задачи
Ответ: 45

2974. ilo, 13 февраля 2011, 16:18:36
Азателло,Люди добрые, помогите!!!
27170-радиус описанной окружности составляет 2/3 высоты треугольника,найти высоту 2sqrt3:2/3=3sqrt3;она равна sqrt3/2a, откуда a=3sqrt3:(sqrt3/2)=6.Дальше ясно: S=P*h=3a*h=18*2;
27178-V=1/3*a^2*12=4*a^2; откуда a^2=200:4=50;a=5sqrt2; 1/2d=1/2*5sqrt2*sqrt2=5;дальше классический треугольник:5,12,13.или по т.Пифагора.
27182 и 27184и 27207 объёмы разбиты на четыре равные по объёму пирамиды и призмы, поэтому надо просто поделить заданный объём на 4.
27204Объём конуса прямопропорционален S осн. и высоте, высота не меняется, площадь основания уменьшилась в 6 раз, значит и объём фигуры тоже уменьшился в 6 раз, т.е.Vф=1/6Vкон.=1/18S*h=1/18*pi144*27=216pi.Осталось поделить на pi.

2975. Леонид , 13 февраля 2011, 22:32:35
--------------------------------------------------------------------------------

2973. Игорь, 13 февраля 2011, 12:23:19 ДА

2976. Арсений, 13 февраля 2011, 23:41:38
Не пойму, почему в задаче B9 (25867) ответ 0.1. У меня получается 0.15.

2977. Илья, 14 февраля 2011, 02:18:23
помогите пожалуйста! найдите наибольшее значение функции y= In(x+3)7 -7x на отрезке [-2,5; 0]

2978. Леонид , 14 февраля 2011, 07:07:18
Уважаемый LEXXUS, огромная просьба к Вам!!! Могут ли среди ГРАФИЧЕСКИХ заданий В8 (понимаю, может быть что угодно), но конкретно - производная не существует, а точка экстремума есть ? Я таких не встречал в открытом банке заданий, Если это возможно сделайте такой пример - здесь или, если не затруднит, на мой эл. адрес. Очень прошу! Говорят, что такого рода задания встречались в наборе заданий последне диагностической работе от 10 феврая (какого не знаю) - не встречал. Ведь вопрос сам по себе не из простых.
И второе - аналитический способ нахождения углового коэффициента по двум выбранным точкам - в В8 производная в точке по графику функам функции и касательной (точки на касательной выделены)? Очень прошу. Заранее премного благодарен! С уважением Леонид.

2979. ilo, 14 февраля 2011, 10:41:01
Азателло,в №27208 R шара=1,9:2=0,95, т.е шар касается куба S=4piR^2=3,8pi,осталось разделить на pi.

2980. Игорь, 14 февраля 2011, 16:19:16
Помогите решить задачу.

В12:Четыре рубашки дешевле куртки на 20%. На сколько процентов шесть рубашек дороже куртки?

2981. Налька, 14 февраля 2011, 16:44:52
подскажите, пожалуйста как решать)
задание типа В10:
Зависимость температуры (в градусах Кельвина) от времени (в минутах) для нагревательного элемента некоторого прибора была получена экспериментально и на исследуемом интервале температур задается выражением T(t)=T0+at+bt2, где Т0=520 К, а=22 К/мин, b=-0,2 К/мин2. Известно, что при температурах нагревателя свыше 1000 К прибор может испортиться, поэтому его нужно отключать. Определите (в минутах) через какое наибольшее время после начала работы нужно отключать прибор.
заранее спасибо)))

2982. Lexxus, 14 февраля 2011, 17:01:24
Могут ли среди ГРАФИЧЕСКИХ заданий В8 (понимаю, может быть что угодно), но конкретно - производная не существует, а точка экстремума есть ? Я таких не встречал в открытом банке заданий, Если это возможно сделайте такой пример - здесь или, если не затруднит, на мой эл. адрес. Очень прошу! Говорят, что такого рода задания встречались в наборе заданий последне диагностической работе от 10 феврая (какого не знаю) - не встречал. Ведь вопрос сам по себе не из простых.

Я тоже таких не встречал. Но почему бы и нет, собственно? Вот, скажем, функция
y = (|x|-4)^2



В случае графических заданий B8 нет принципиальной разницы, дифференцируема ли функция на заданном интервале, или нет.

Там везде формальные правила:
1. Если написано, что изображен график ФУНКЦИИ, то любой "горб" ("острый" он или "тупой") - это экстремум.
2. Если написано, что изображен график ПРОИЗВОДНОЙ ФУНКЦИИ, то экстремумы функции будут там, где график пересекает ось абсцисс.

2983. Lexxus, 14 февраля 2011, 17:11:23
аналитический способ нахождения углового коэффициента по двум выбранным точкам - в В8 производная в точке по графику функам функции и касательной (точки на касательной выделены)?


Что-то в этом роде?



Выделено две точки с координатами (-5,-3) и (-2,-9).

Совсем аналитический способ.

Уравнение прямой:
y = k*x + b.

Подставляем в это уравнение координаты обеих точек:
-3 = k*(-5) + b
-9 = k*(-2) + b

-3-(-9) = k*(-5-(-2))
k = 6/-3 = -2

2984. Леонид , 14 февраля 2011, 19:11:00
LEXXUS! Огромное спасибо! За приведённые обоснования! Вот, если на абстрактном графике производной, хотя бы в одной из точек пересечения с ОХ - пустая точка. Что в ней? Разве можем в такой ситуации сделать вывод о максимуме или минимуме? Эта точка может оказаться, через неё проходит ассимтота ГРАФИКА ФУНКЦИИ, а может быть и нет. Ведь так?

2985. Lexxus, 14 февраля 2011, 19:23:03
Вот, если на абстрактном графике производной, хотя бы в одной из точек пересечения с ОХ - пустая точка. Что в ней? Разве можем в такой ситуации сделать вывод о максимуме или минимуме?

Нет, конечно же, не можем.
Вот, например, функция f(x) = x^2 + |x|/x (красная линия на рисунке):



Её производная - функция 2*x (синияя линия) с выколотой точкой x=0 (как раз в том единственном месте, где график пересекает ось абсцисс).

У f(x), как видно по рисунку, вообще нет экстремумов.

2986. Игорь, 14 февраля 2011, 19:43:11
Найдите наибольшее значение функции sqrt-21+10x-x^2 (все под корнем) на отрезке [4;6]

Я решал, нашел производную...
Нашел значения ф-и на начале и конце промежутка...но там получаются ответы с корнем...

Помогите плиз....

2987. Lexxus, 14 февраля 2011, 20:01:53
Найдите наибольшее значение функции sqrt-21+10x-x^2 (все под корнем) на отрезке [4;6]

Я решал, нашел производную...
Нашел значения ф-и на начале и конце промежутка...но там получаются ответы с корнем...


Плохо ты искал производную :)

(sqrt(-21+10*x-x^2))' = 1/2*1/sqrt(-21+10*x-x^2)*(10-2*x) =
= (10-2*x)/2/sqrt(-21+10*x-x^2)

Найдем нули производной. Один видно сразу: 10-2*x = 0, x=5.

В знаменателе - квадратный трехчлен (-21+10*x-x^2) под квадратным корнем.
Корни этого квадратного трехчлена - 3 и 7, между которыми он положителен и знак не меняет.

Значит, внутри отрезка [4;6] единственный ноль производной - это x=5, и в этой точке она меняет знак с положительного на отрицательный. То есть это точка максимума функции.

sqrt(-21+10*5-5^2) = sqrt(4) = 2.

Ответ: 2

2988. ilo, 14 февраля 2011, 21:55:16
Налька, 14 февраля 2011, 16:44:52
подскажите, пожалуйста как решать)
задание типа В10:
Зависимость температуры (в градусах Кельвина) от времени (в минутах) для нагревательного элемента некоторого прибора была получена экспериментально и на исследуемом интервале температур задается выражением T(t)=T0+at+bt2, где Т0=520 К, а=22 К/мин, b=-0,2 К/мин2. Известно, что при температурах нагревателя свыше 1000 К прибор может испортиться, поэтому его нужно отключать. Определите (в минутах) через какое наибольшее время после начала работы нужно отключать прибор.
Подставив заданные коэффициенты получаешь T(t)=520+22t-0,2t^2<=1000 при этом условии прибор на испортится,решаешь неравенство получаешь два луча t<=30,t>=80.После 30" после начала работы t=0 температура достигнет 1000 и прибор надо отключать.

2989. Леонид , 14 февраля 2011, 23:33:40
Для Ильи. Как мог попасть на мой адрес - это первое? Второе - о какой пирамиде? речь???. Наверное - правильная шестиугольная призма? Пиши,Илья, сюда. А что так, с С2, если пишешь не очень в математике. Давай с простых. Потом и по сложнее получатся.

2990. Леонид , 14 февраля 2011, 23:42:03
2985. Lexxus, 14 февраля 2011, 19:23:03

Ещё раз огромное спасибо!!! Полностью удовлетворён!!! Всё хорошо. Я прав, что нет такого рода заданий в В8, чтобы надо было установить качество эестремума по абстрактному графику производной в пустой точке - не имеющей смысла. Тем более, что это не возможно сделать. СПАСИБО! И вопрос такого рода серьёзный - базовым быть не может.

2991. ilo, 15 февраля 2011, 09:11:12
Арсений, 13 февраля 2011, 23:41:38
Не пойму, почему в задаче B9 (25867) ответ 0.1. У меня получается 0.15.
Lexxus, гляньте,пожалуйста.

2992. Lexxus, 15 февраля 2011, 09:30:33
Не пойму, почему в задаче B9 (25867) ответ 0.1. У меня получается 0.15.
Lexxus, гляньте,пожалуйста.

Объем пирамиды равен трети объема призмы с тем же основанием и высотой. Объем призмы ABCA1B1C1 равен половине объема параллелепипеда.
Значит, объем пирамиды ABCB1 равен одной шестой объема параллелепипеда.

2993. Илья, 15 февраля 2011, 19:11:28
Всем привет!Леонид,адрес твой не сложго вычислить,ладно буду тут писать,просто не хочу казаться тупым)да речь идет о призме,я значит опечатался)мне уже подсказали как эту задачу решать,будут вопросы,обязательно задам!

2994. Илья, 15 февраля 2011, 19:27:51
Решали пробный ЕГЭ по математике неделю назад,так там почти ни одного примера с демоверсии нет,и написали плохо(

2995. Леонид , 15 февраля 2011, 20:10:05
Привет Пиши здесь, Илья. Смогу - помогу. Что было на пробном С1-С3, не сохранил?

2996. dimageniy, 16 февраля 2011, 13:05:37
9^x -3^(x+7)-243=0 решите пожалуйста вопрос очень очень важный

2997. Ленуська, 16 февраля 2011, 14:49:51
Спомощью графиков решите систему уравнений..
РЕШИТЕ ПОЖАЛУЙСТА!!!!!!!!!!!!!!!


система 1)ху=6
2)2х-3у=6

2998. Лилька, 16 февраля 2011, 16:33:12
Помогите пожалуйста решить задачу.
Треугольник АВС— прямоугольный (С = 90°), А = 30°, АС = а, DC перпендикулярна ABC, DC = а корней из 3, делённое на 2. Чему равен угол между плоскостями ADB и АСВ.

2999. Илья, 16 февраля 2011, 18:08:10
Леонид,нет,с1 неравенство надо было решить,даже системы не было

3000. налька, 16 февраля 2011, 20:50:09
в сосуд, имеющий форму правильной треугольной призмы, налили воду. Уровень воды достигает 36 см. На какой высоте будет находиться уровень воды, если ее перелить в другой сосуд той же формы, у которого сторона основания в 3 раза больше, чем у первого? ответ в см.
Помогите, пожалуйста)


3001. настя, 16 февраля 2011, 20:55:35
в основании прямой призмы лежит квадрат со стороной 5. Боковые ребра равны 7/п. Найдите объем цилиндра,, лписанного около этой призмы.

3002. Yuru, 17 февраля 2011, 11:18:34
http://live.mephist.ru/show/mathege2010/view/id/28137/
Ответ просят выразить в минутах,а не в часах !
Правильный ответ 45 !

3003. Lexxus, 17 февраля 2011, 11:48:07
3002. Yuru, 17 февраля 2011, 11:18:34
http://live.mephist.ru/show/mathege2010/view/id/28137/
Ответ просят выразить в минутах,а не в часах !
Правильный ответ 45 !

Справедливо.

3004. ilo, 17 февраля 2011, 14:05:50
налька,V=Sосн.*h; Sосн.пропорциональна квадрату стороны, если сторона основания увеличилась в 3 раза, то Sосн.- в 9раз.По условию V не изменяется, значит высота,на которой будет находиться уровень воды уменьшится в9 раз- 36:9=4

3005. Игорь, 17 февраля 2011, 16:19:07
2987. Lexxus, 14 февраля 2011, 20:01:53

Понятно :)

В12:В 2008 году в городском квартале проживало 40000 человек. В 2009 году, в результате строительства новых домов, число жителей выросло на 10%, а в 2010 году на 9% по сравнеию с 2009 годом. Сколько человек стало проживать в квартале в 2010 году?

Помогите пожалуйста...

3006. алекс2010, 17 февраля 2011, 16:48:18
2008 год-40000человек, 2009год +10% (10%от 40000 =40000х0,1=4000),итак в 2009году было 44000человек, 2010год + еще 9% (9% от 44
000=44000х0,09=3960) итого44000+3960= 47960 чел. Вроде так.

3007. пашка2010, 17 февраля 2011, 16:55:24
не могу дотумкать, помогите кто-нибудь....

Х в 4 степени + З(2Х+1) квадрат меньше либо равно 4Х квадрат (2Х+1)

3008. Игорь, 17 февраля 2011, 19:10:15
3006. алекс2010, 17 февраля 2011, 16:48:18
2008 год-40000человек, 2009год +10% (10%от 40000 =40000х0,1=4000),итак в 2009году было 44000человек, 2010год + еще 9% (9% от 44

000=44000х0,09=3960) итого44000+3960= 47960 чел. Вроде так.


В том то и дело что так не правильно...Я так решал, с ответом не сходится, тут не может быть так просто потому что задача В12

3009. пашка2010, 17 февраля 2011, 20:11:16
Игорь, а какой ответ? Задача знакомая, вроде решал уже...

3010. Игорь, 17 февраля 2011, 21:02:47
3009. пашка2010, 17 февраля 2011, 20:11:16
Игорь, а какой ответ? Задача знакомая, вроде решал уже...


Я не знаю Паша,вот сдесь написал ее чтобы помогли понять как решать...а то вдруг на ЕГЭ попадется.

3011. Игорь, 17 февраля 2011, 21:07:33
В12: В 2008 году в городском квартале проживало 40000 человек. В 2009 году, в результате строительства новых домов, число жителей выросло на 1%, а в 2010 году — на 9% по сравнению с 2009 годом. Сколько человек стало проживать в квартале в 2010 году?

Вот правильный вариант задачи, тот с ошибкой :(

3012. Игорь, 18 февраля 2011, 08:50:55
С6: Перед каждым из чисел 5,6,...,9 и 12,13,...17 произвольным образом ставят знак плюс или минус,после чего к каждому из образовавшихся чисел первого набора прибавляют каждое из образовавшихся чисел второго набора,а затем все 30 полученных результатов складывают.Какую наименьшую по подулю и какую наибольшую сумму можно получить в итоге?

3013. анастасия, 18 февраля 2011, 09:45:19
В треугольнике АВС угол С равен 90градус, sinA=7/8, AC=подкорнем15.найдитеАВ

3014. Игорь, 18 февраля 2011, 10:43:35
Sin А=ВС/АВ=7/8
АВ=8
Ответ:8

3015. ilo, 18 февраля 2011, 19:03:47
3013. анастасия, 18 февраля 2011, 09:45:19
В треугольнике АВС угол С равен 90градус, sinA=7/8, AC=подкорнем15.найдитеАВ

Прежде найти cosA из основного тригонометрического тождества-sqrt15/8;
AB=AC/cosA=8;

3016. Игорь, 18 февраля 2011, 19:55:56
3012. Игорь, 18 февраля 2011, 08:50:55
С6: Перед каждым из чисел 5,6,...,9 и 12,13,...17 произвольным образом ставят знак плюс или минус,после чего к каждому из образовавшихся чисел первого набора прибавляют каждое из образовавшихся чисел второго набора,а затем все 30 полученных результатов складывают.Какую наименьшую по подулю и какую наибольшую сумму можно получить в итоге?



В ответе получается 1 и 645.

645 можно получить выполнив эти действия: к каждому из образовавшихся чисел первого набора прибавляют каждое из образовавшихся чисел второго набора,а затем все 30 полученных результатов складывают.

А вот наименьшее положительное по модулю это 1.

3017. Александр Шмуратко, 19 февраля 2011, 18:38:37
2534. ксюша, 26 ноября 2010, 18:03:50
Углы треугольника образуют арифметическую прогрессию, причем наименьший из углов равен 10 градусов. Чему равна величина одного из двух других углов, округленная до целого числа градусов?


Есть решение. Продаю за 300 руб. Моя почта: as@glagolspb.ru

3018. Игорь, 19 февраля 2011, 21:06:28
В9: Прямоугольный параллелепипед имеет измерения равные а=4 в=4 h=3; из него вырезан куб сторона которого равна 1.Найдите площадь поверхности пераллелепипеда.

3019. Леонид , 19 февраля 2011, 22:32:29
Игорёк, а о вырезанном кубе ничего не сказано? Может он вырезан от верштны или какой-то грани? Явно, не насквозь, судя по размерам. Это же важно. Об объёме в твоём условии можно ответить, а о поверзности ? Поверхность куба 6, поверхность параллелепипеда 80. И...?

Александр Шмуратко, 19 февраля 2011, 18:38:37 - ДА! Шустёр! В смысле - шустрый! Решать то умеешь? Такой здесь первый.

3020. ксюша, 20 февраля 2011, 00:18:30
Александр Шмуратко
спасибо,конечно,но решение найденно..
причем бесплатно

3021. Игорь, 20 февраля 2011, 09:51:47
Леонид, куб вырезан из верхней правой части. 3-х сторон у параллелепипеда нехватает.И нужно найти площадь поверхности пераллелепипеда без вырезанного куба.

У меня 69 получалось.

S=2Sосн+Sбок
Sбок=Ph
S=2*3*4+16*3=72
Sпов.куба=3
S=72-3=69

Но это неправильный ответ.

3022. VladislavOfficiaL, 20 февраля 2011, 14:14:14
У меня почему-то не возможно в ответе записать в виде дроби. / вот этот знак не вводится в ответ.Что через точку писать ответ?

3023. Леонид , 20 февраля 2011, 15:40:02
Игорь - куб вырезан, как угол параллелепипеда? Если это тот случай - То поверхность не изменится - это один из интересных случаев. Что отрезается - появляктся на срезах. Поверхность прямоугольного параллелепипеда: 2ав + 2вс + 2ас - в любом случае. Твоё: 2(4*4 + 4*3 +4*3) = 80. Формулы у тебя записаны верно. Ph = 16*3=48. Площадь основания 4 х 4 = 16, а 2-х 32 , итого 80. Твоя ошибка, ты 3 втащил, как сторону основания , э это - высрта. Ответ: 80.

3024. я, 20 февраля 2011, 15:58:07
я много чего сказать могу,но я промолчу

3025. милана, 20 февраля 2011, 16:00:21
кто знает высшую алгебру

3026. Лёлька, 20 февраля 2011, 16:13:23
Уважаемые, помогите, пожалуйста, решить систему

|(5xy+7x)/(y-1) - 5x=6
<
|log (x^3-4x+49)/(2y+5) по основанию 7=2-2log (7+4x) по основанию 49

пожалуйста, помогите, если можете! :)

3027. ilo, 20 февраля 2011, 17:03:04
Лёлька, Из 1-го: 5xy+7x=(6+5x)(y-1);после раскрытия скобок y=2x+1;
2-е:log_(x^3-4x+49)/(2y+5)=log_49/(7+4x);подставляем y=2x+1:
(x^3-4x+49)(7+4x)=49(4x+7);после раскрытия скобок:4x^4+7x^3-16x^2-28x=0;
x(x^2(4x+7)-4(4x+7));x(x^2-4)(4x+7)=0;(0;1)-не явл.решением-при y=1 обращается в0 знаменатель дроби;(2;5)-решение; (-2;-3)-не решение,т.к. не удовлетворяет условию 7+4x>0;(-7/4;-5/2)-не решение, т.к. не удовлетворяет условию 7+4x>0; Ответ: (2;5)

3028. Лёлька, 20 февраля 2011, 17:06:02
ilo, огромное спасибо!!!

3029. Лёлька, 20 февраля 2011, 18:04:42
ilo, можно еще вопросик?!

log_ - вместо _ что писать?
спасибо)

3030. ilo, 20 февраля 2011, 18:43:32
всё приводим к основанию 7

3031. Игорь, 20 февраля 2011, 21:02:12
Леонид, нужно найти площадь поверхности параллелепипеда без площади поверхности куба. Sпов.куб=6
Sпов.парал=80-6=74

Чего то я не понимаю :(

3032. настя, 20 февраля 2011, 21:11:24
в основании прямой призмы лежит квадрат со стороной 5. Боковые ребра равны 7/п. Найдите объем цилиндра, описанного около этой призмы.
помогите, пож-та)

3033. ilo, 20 февраля 2011, 21:39:38
настя,a4=Rsqrt2;R=5/sqrt2;Sосн.=25pi/2;V=25pi/2*7/pi=175/2=87,5куб.ед.sqrt-корень из...

3034. ilo, 20 февраля 2011, 21:47:25
Александр Шмуратко, 19 февраля 2011, 18:38:37
[b]

Есть решение. Продаю за 300 руб. Моя почта: as@glagolspb.ru

Полно, голубь. не греши,
Убери свои гроши,-
Мы ведь энто не для денег.
Мы ведь энто для души.

3035. настя, 20 февраля 2011, 22:03:07
ilo, спасибо)

3036. Игорь, 20 февраля 2011, 22:03:17
В12: В 2008 году в городском квартале проживало 40000 человек. В 2009 году, в результате строительства новых домов, число жителей выросло на 1%, а в 2010 году — на 9% по сравнению с 2009 годом. Сколько человек стало проживать в квартале в 2010 году?

Помогите пожалуйста :)

3037. Izolda, 21 февраля 2011, 00:26:35
Не понимаю: задача 19365. Правильный ответ по сайту 0.6, хотя по распределению ответов выше процент у 0.8, это во-первых. а во-вторых, ответ действительно 0.8. так почему отображается 0.6?

Игорь, 40000*1,01*1,09=44036


3038. Леонид , 21 февраля 2011, 01:11:01
Игорь: 40 000*1,01 (101%) = 40 400. 40 400* 1,09 (109%) = 44 034 . Рассуждения 40 000 было 100%, стало 101% или 1,01 часть числа - находится умножением. Было 40 400 - 100%, стало 109 или 1,09 часть числа - находим умножением.
Что с параллелепипедом и вырезанным кубом?

3039. Izolda, 21 февраля 2011, 09:55:37
Леонид, все же 44036

3040. КСЕНИЯ, 21 февраля 2011, 14:32:23
можно вопрос,кто это решает?

3041. КСЕНИЯ...., 21 февраля 2011, 14:33:19
можно вопрос,кто это решает?

3042. Леонид , 21 февраля 2011, 14:49:22
3039. Izolda, 21 февраля 2011, 09:55:37 цитата

Леонид, все же 44036
Просто опечатка. На концаз множителей 9 и 4 --- 6.

3043. Леонид , 21 февраля 2011, 15:03:57
3037. Izolda, 21 февраля 2011, 00:26:35
19365 ответ 0,8

3044. Игорь, 21 февраля 2011, 21:58:17
Леонид, ты уверен что эта задача В12 так легко решается...как В1 почти :(

С той В9 вообще мутные замуты...показал учителю своему он сказал, что я правильно решил и в ответе 69.

3045. Илья, 21 февраля 2011, 22:17:13
Всем привет!Помогите плиз)начну с самого простого)для вас,но не простого для меня) и так:
Прямоугольный параллепипед описан около цилиндра,радиус основания и высота которого равны 8.Найдите объем параллепипеда.
P/s решите как можнее понятнее,чтоб я вник..

3046. Илья, 21 февраля 2011, 22:33:13
я вот решил,но не знаю правильно это или нет:
V=a*b*c,
Одна из сторон прямоугольного параллелепипеда равна высоте цилиндра, т.е. 8.5. А две другие стороны равны диаметру основания цилиндра, т.е. 17.
V=8.5*17*17=2456.5

3047. надежда, 22 февраля 2011, 09:40:54
помогите пожалуйста решить задачу В9: Кубик весит 800г. Сколько граммов будет весить кубик, ребро которого в 2 раза меньше, чем ребро первого кубика, если оба кубика изготовлены из одинакового материала?

3048. Izolda, 22 февраля 2011, 18:05:45
Надежда. Ребро в 2 раза меньше, значит, объем меньше в 2*2*2 раз, то есть в 8 раз. И масса тоже. Итого 800/8=100

3049. Зуля, 22 февраля 2011, 19:57:47
Помогите пожалуйста
Решить уравнения
log4(x +1)+log4(x+1)^2=3
5+lg^2икс=-4lx

решить систему уравнений
{log2(x+y)+2log4(x-y)=5}
{3^1+2log3(x-y)=48}

3050. Даша, 22 февраля 2011, 20:57:47
Помогите!!!
в треугольнике АВС угол С равен 90, СН - высота, АВ=29, tgА=3/7

3051. Леонид , 22 февраля 2011, 21:17:18
3045. Илья, 21 февраля 2011, 22:17:13
Так как параллелепипед оптиса около цилиндра, радиуса 8, то сторона основания (квадрат - пар-ед прямоугольный) 16. Объём параллелепипеда равен 16*16 *8 =2048.

3052. Леонид , 22 февраля 2011, 21:22:26
3046. Илья, 21 февраля 2011, 22:33:13
Где взял 8,5 Выше ты пишешь 8. Если с 8,5 - решение верное.

3053. Леонид , 22 февраля 2011, 21:31:09
3031. Игорь, 20 февраля 2011, 21:02:12 цитата

Леонид, нужно найти площадь поверхности параллелепипеда без площади поверхности куба. Sпов.куб=6
Sпов.парал=80-6=74

Чего то я не понимаю :(
Игорёк! Сделай на пластелине - вырежь кубик от него. Увидешь, что с кубиком не всё отойдёт - срезы же тже имеют площадь - квадратики. А поэтому так отнимать нельзя. Понятно написал? У нас девчёнки, чистя картошку на жарево, делали эти модельки, а потом жарили. И с задачей разбиались. Попробуй - 3 пользы. Советую - серьёзно, а то за шутку примешь.

3054. Зуля, 22 февраля 2011, 21:48:32
Помогите пожалуйста
Решить уравнения
log4(x +1)+log4(x+1)^2=3
5+lg^2икс=-4lx

решить систему уравнений
{log2(x+y)+2log4(x-y)=5}
{3^1+2log3(x-y)=48}


Люди добрые пожалуйста помогите решить, я никак не могу))ниче не получается-)

3055. Илья, 23 февраля 2011, 10:59:40
8,5 там Леонид....опечатался..

3056. стас, 23 февраля 2011, 12:52:37
помогите с решением
за 1 квартал цена телевизора понизилась на 10%, а за 2 квартал на 15%.На сколько процентов снизилась цена на телевизор за два квартала?

3057. Lexxus, 23 февраля 2011, 13:12:19
3056. стас, 23 февраля 2011, 12:52:37
за 1 квартал цена телевизора понизилась на 10%, а за 2 квартал на 15%.На сколько процентов снизилась цена на телевизор за два квартала?

Когда говорят, что что-то понизилось на P процентов, это значит
(новое значение) = (старое значение)*(1-P/100)

Значит,

(цена после 1 квартала) = (начальная цена)*(1-10/100)
(цена после 2 квартала) = (цена после 1 квартала)*(1-15/100)
т.е.
(цена после 2 квартала) = (начальная цена)*(1-0.1)*(1-0.15)

(1-0.1)*(1-0.15) = 0.9*0.85 = 0.765 = 1-0.235 = 1-23.5/100

(цена после 2 квартала) = (начальная цена)*(1-23.5/100)
Ответ: 23.5%

3058. stas, 23 февраля 2011, 13:29:30
около треугольника со сторонами 6, 8 и 10 описана окружность S.Найдите радиус окружности,касающейся меньшей стороны треугольника и окружности S.

3059. незнакомка, 23 февраля 2011, 14:12:50
помогите решить задачу
Время,затрачиваемое автобусом на прохождение расстояния 325км, при составлении нового расписания движения автобусов сокращенно на 40мин.Найдите среднюю скорость движения автобуса по новому расписанию, если она на 10км/ч больше средней скорости, предусмотренной старым расписанием.

3060. Илья, 23 февраля 2011, 16:57:26
Всем привет!По халяве достал реальный вариант ЕГЭ по алгебре...из отдела образования..я буду выкладывать примеры...сам буду решать и вы решайте,и скажите,можно ли сюда фото выкладывать?мне потмоу что отсканированные передали...я б сюда залил и всё..

3061. Стас, 23 февраля 2011, 17:02:33
Илья!неприменно заливайте.Вы сможете помочь многим людям.Я думаю администрация сайта вам поможет,не правда ли?

3062. Илья, 23 февраля 2011, 17:22:00
как сюда изображение вставлять?я б прям кинул у меня два варианта ЕГЭ..

3063. Lexxus, 23 февраля 2011, 17:33:54
3062. Илья, 23 февраля 2011, 17:22:00
как сюда изображение вставлять?я б прям кинул у меня два варианта ЕГЭ..

Залей на любой файлообменник и кинь сюда ссылку.
Ну или пришли мне на lexxus@mephist.ru, я тут выложу.

3064. незнакомка, 23 февраля 2011, 21:26:05
помогите решить задачу
Время,затрачиваемое автобусом на прохождение расстояния 325км, при составлении нового расписания движения автобусов сокращенно на 40мин.Найдите среднюю скорость движения автобуса по новому расписанию, если она на 10км/ч больше средней скорости, предусмотренной старым расписанием.
кто -нибудь может мне помочь?

3065. ilo, 23 февраля 2011, 22:17:12
3054. Зуля, 22 февраля 2011, 21:48:32
Помогите пожалуйста
Решить уравнения
log4(x +1)+log4(x+1)^2=3;

log_(x+1)+2log_|x+1|=3;D(f):x+1>0;x>-1;
при этом условии модуль раскрывается |x+1|=x+1;
3log_(x+1)=3; x+1=4;x=3;

3066. Ксенька, 23 февраля 2011, 22:18:25
помогите решить)оч нужно_
дан правильный тетраэдр МАВС с ребром 1. найдите расстояние между прямыми AL и МО, где L-середина ребра МС, О-центр грани АВС

3067. Илья, 23 февраля 2011, 22:22:16
3063. Lexxus, 23 февраля 2011,17:33:54


как сюда изображение вставлять?я б прям кинул у меня два варианта ЕГЭ..

Залей на любой файлообменник и кинь сюда ссылку.

Ну или пришли мне на lexxus@mephist.ru, я тут выложу.

Я отослал тебе на емайл,2 варианта...Будем решать))

3068. ilo, 23 февраля 2011, 22:57:38
3054. Зуля, 22 февраля 2011, 21:48:32


решить систему уравнений
{log2(x+y)+2log4(x-y)=5}
{3^1+2log3(x-y)=48}


Люди добрые пожалуйста помогите решить, я никак не могу))ниче не получается-)

Из 2-го:3*3^log_(x-y)^2=48;(x-y)^2=16;x-y=4 или x-y=-4-не отвечает D(f):x-y>0;
В 1-ое: log_(x+y)+2log_4=5; log_(x+y)=3;x+y=8;Осталось решить систему :1)x-y=4;2)x+y=8;x=6;y=2; Ответ:(6;2)

3069. Lexxus, 24 февраля 2011, 00:42:27
3066. Ксенька, 23 февраля 2011, 22:18:25
дан правильный тетраэдр МАВС с ребром 1. найдите расстояние между прямыми AL и МО, где L-середина ребра МС, О-центр грани АВС

1. Расстояние между двумя скрещивающимися прямыми - это длина перпендикуляра, опущенного из одной прямой, к плоскости, параллельной этой прямой и содержащей вторую прямую.
2. Строим проекцию AK отрезка AL на плоскость ABC. Плоскость AKL перпендикулярна плоскости ABC, параллельна прямой MO и содержит прямую AL. Значит, искомая длина - это длина перпендикуляра ON, опущенного из точки O к AK.



3. Дальше всё можно найти по теореме Пифагора либо методом координат.
В ответе получится 1/(2*sqrt(7))

3070. Арсений, 24 февраля 2011, 03:06:09
Проверьте правильность ответа к заданию B9 (4969). Кажется, правильный ответ - 22.5

3071. ilo, 24 февраля 2011, 08:27:45
3064. незнакомка, 23 февраля 2011, 21:26:05
помогите решить задачу
Время,затрачиваемое автобусом на прохождение расстояния 325км, при составлении нового расписания движения автобусов сокращенно на 40мин.Найдите среднюю скорость движения автобуса по новому расписанию, если она на 10км/ч больше средней скорости, предусмотренной старым расписанием.
кто -нибудь может мне помочь?

х- скорость по старому расписанию,х+10- по новому;325/х-35/(х+10)=2/3; 325*3*(х+10)-325*3*х=2*(х^2+10x); сводится к квадратному x^2+10x-1625*3=0; D=140^2; x=75;второе значение условию не отвечает.

3072. ilo, 24 февраля 2011, 08:57:57
3058. stas, 23 февраля 2011,&nbsp;13:29:30
около треугольника со сторонами 6, 8 и 10 описана окружность S.Найдите радиус окружности,касающейся меньшей стороны треугольника и окружности S.

Вписанный треугольник прямоугольный:100=64+36;Её R=5; Провести линию центров,она проходит через точку касания. R=2r+средняя линия, параллельная стороне, равной 8.5=2r+4;r=0,5;

3073. Леся, 24 февраля 2011, 18:57:56
помогите плиз....в равнобедренном треугольнике АВС угол В-прямой,АС=2корня из 2,ВД-медиана.вычислить скалярное произведение векторов ВД И АС,ВД И ВС,ВД И ВД...очень надо ..нам дали каждому задачи домой как на зачет...

3074. незнакомка, 24 февраля 2011, 20:42:20
Ило огромное спасибо!!!!!!

3075. Игорь, 24 февраля 2011, 20:59:20
В12: Расстояние между пристанями А и В равно 48 км. Отчалив от пристани А в 10 часов утра, теплоход проплыл по течению реки с постоянной скоростью до пристани В. После трехчасовой стоянки у пристани В теплоход отправился в обратнный рейс и прибыл в А в тот же день в 22.00. Найдите скорость теплохода в неподвижной воде, если скорость течения реки равна 4 км/ч.

Помогите решить...

3076. альбина)), 24 февраля 2011, 21:04:43
ПРИВЕТИК))))ПОМОГИТЕ ПОЖАЛУЙСТА РЕШИТЬ УРАВНЕНИЕ ПО В3.
log2(4-X)=3
ЗАРАНЕЕ СПАСИБО...

3077. Игорь, 24 февраля 2011, 21:21:28
3076. альбина)), 24 февраля 2011, 21:04:43
ПРИВЕТИК))))ПОМОГИТЕ ПОЖАЛУЙСТА РЕШИТЬ УРАВНЕНИЕ ПО В3.

log2(4-X)=3

ЗАРАНЕЕ СПАСИБО...


Ответ: -4

3078. ilo, 24 февраля 2011, 21:48:23
3075. Игорь, 24 февраля 2011, 20:59:20
В12:

Помогите решить...

Х-собственная,х+4-по течению,х-4-против.Уравнение:48/(х+4)+48/(х-4)=9;48х+48*4+48х-48*4-9х^2+9*16=0;квадратное 3x^2-32x-48=0;D=1600;x=12, другой не отвечает условию

3079. Игорь, 24 февраля 2011, 22:25:25
Большое спасибо, ilo

3080. Игорь, 24 февраля 2011, 22:44:46
С1: (2sin^2(х)-cosx-2)log_sinx(x^2)=0

ilo, помоги пжл...

3081. Леся, 24 февраля 2011, 22:46:11
помогите плиз....в равнобедренном треугольнике АВС угол В-прямой,АС=2корня из 2,ВД-медиана.вычислить скалярное произведение векторов ВД И АС,ВД И ВС,ВД И ВД...очень надо ..

3082. ilo, 25 февраля 2011, 07:28:09
3080. Игорь, 24 февраля 2011, 22:44:46
С1: (2sin^2(х)-cosx-2)log_sinx(x^2)=0

ilo, помоги пжл...


(sin^2(x)-cosx-2)log_sinx(x^2)=0;
D(f):sinx>0;sinx не равен1;x не равен 0;
Из условия равенства 0 произведения:
1)2-2cos^2(x)-cosx-2=0;cosx(2cosx+1)=0;cosx=0 или cosx=-1/2;
X=pi/2+pin,не отвечает условию sinx не равен 1 и sinx>0;(pi/2-первому, -pi/2-второму);x=(+-)2pi/3+2pin; -2pi/3+2pin не отвечает условию sinx>0;2)log_sinx(x^2)=0;x=(+-)1; -1 не отвечает условию sinx>0; Окончательно x=1;x=3pi/2+2pin (n везде из Z).

3083. Леонид , 25 февраля 2011, 07:42:28
Уважаемый LEXXUS! Просьба дать комментарий! В заданиях В3 есть тригонометрические уравнения, в которых просят найти наибольший отрицательный корень!!! Ответ - это числовое наибольшее отрицательное? Например, из -150 градусов и - 30 градусов, наибольшее -30 или по геометрической величине - 150. Просьба, приведите пример решения задания из открытого банка заданий - уравнения с косинусом или синусом. Пожалуйста! Заранее ВАМ благодарен.

3084. Илья, 25 февраля 2011, 11:12:47
Всем привет!помогите решить задачу...с реального ЕГЭ B9...

Сторону основания правильной четырехугольной призмы объем которой равен 15,увеличили в 1,2 раза а высоту в 1,5 раза.Найдите объем полученной призмы.


объясните как решать..(Заранее спасибо)

3085. Леонид , 25 февраля 2011, 14:28:10
V = abc = aac =15 - объём данной примы. Увеличим стороны основания и высоту, как скаано в условии: V = 1,2a*1,2a*1,5c = 1,2*1,2*1,5aac = =1,2*1,2*1,5*15=32,4. Всё понятно?

3086. ann, 25 февраля 2011, 15:20:25
помогите пожалуйста!!! укажите наименьшее число из области определения функции y=5tg(в квадрате)x+2 (пожалуйста напишите решение поподробней, каму не сложна помогите!)

3087. Илья, 25 февраля 2011, 15:46:27
Леонид,спасибо,всё понятно...ответ у меня такой же...просто надо было с кем то проверится...чтоб узнать правильно или нет!

3088. Игорь, 25 февраля 2011, 15:59:17
С2: В правильной шестиугольной призме A...F1, все ребра которой равны 1, найдите косинус угла между прямыми АВ1 и BD1.

3089. ann, 25 февраля 2011, 16:08:43
HELP ME...помогите пожалуйста!!! укажите наименьшее число из области определения функции y=5tg(в квадрате)x+2 (пожалуйста напишите решение поподробней, каму не сложна помогите!)

3090. ilo, 25 февраля 2011, 16:51:24
3088. Игорь, 25 февраля 2011, 15:59:17
С2: В правильной шестиугольной призме A...F1, все ребра которой равны 1, найдите косинус угла между прямыми АВ1 и BD1.

решение и чертёж


Изменил Lexxus (с депозита неудобно качать)

3091. Игорь, 25 февраля 2011, 22:16:04
Спасибо ilo.

3092. Дмитрий32, 26 февраля 2011, 11:34:51
Найдите наименьшее значение функции у=20тангенс х-20х-5п-7 на отрезке [-п/4;п/4]???????
ПОМОГИТЕ ПЛИИИИЗЗЗЗ!!!!!!!!!!!!!!!!!1 СРОЧНО!!!!!!!!!!!!

3093. Дмитрий32, 26 февраля 2011, 11:35:58
Найдите наименьшее значение функции у=20тангенс х-20х-5п-7 на отрезке [-п/4;п/4]???????
ПОМОГИТЕ ПЛИИИИЗЗЗЗ!!!!!!!!!!!!!!!!!1 СРОЧНО!!!!!!!!!!!!!!

3094. дима, 26 февраля 2011, 11:37:58
Найдите наименьшее значение функции у=20 тангенс х-20х-5п-7 на отрезке [-п/4;п/4]
?????????????????????????????????

3095. Little Girl, 26 февраля 2011, 12:42:38
Дима!
1) найди область определения ф-ии
2) найди производную
3) найди область определения производной
4) приравняй производную к нулю
5) когда найдешь х, один не войдет в отрезок, другой должен подойти
6) подставь в ф-ию вместо х то, что нашел и концы отрезков
7) то, что будет наименьшим, будет ответом

=))

3096. Чебурашка)), 26 февраля 2011, 12:49:25
Доброго дня всем!
Кто может, помогите, пожалуйста, решить уравнение для всех a
25^x+a^2(a-1)5^x-a^5=0

3097. катя, 27 февраля 2011, 11:50:51
После встречи двух теплоходов один из них пошел на юг,а другой на запад.через 2 ч.После встречи расстояние между ними было 60км.Найдите скорость каждого теплохода,если известно,что скорость одного из них на 6км.ч больше скорости другого.
ПОМОГИТЕ РЕШИТЬ...

3098. Lexxus, 27 февраля 2011, 12:16:27
3096. Чебурашка)), 26 февраля 2011, 12:49:25
Кто может, помогите, пожалуйста, решить уравнение для всех a
25^x+a^2(a-1)5^x-a^5=0

1. Делаем замену t = 5^x, получаем квадратное уравнение относительно t:
t^2 + a^2*(a-1)*t - a^5 = 0

2. Дискриминант:
D = a^4*(a-1)^2+4*a^5 = a^4*(a^2-2*a+1+4a) = a^4*(a^2+2a+1) =
= a^4*(a+1)^2 = (a^2*(a+1))^2, всегда больше или равен нулю.

3. Решения относительно t:
t1 = (-a^2*(a-1)-a^2*(a+1))/2 = -a^2*(a-1+a+1)/2 = -a^3
t2 = (-a^2*(a-1)+a^2*(a+1))/2 = -a^2*(a-1-a-1)/2 = a^2

4. Вернемся к первоначальной замене:
5^x = t
Значение показательной функции может быть только строго положительным.

Решение 5^x = -a^3 имеет место при
-a^3 > 0
a^3 < 0
a < 0.
И в этом случае x = log5(-a^3)

Решение 5^x = a^2 имеет место при
a^2 > 0
a не равно 0.
И в этом случае x = log5(a^2)

Ответ:
При a < 0: x = log5(-a^3), x = log5(a^2);
При a = 0: решений нет;
При a > 0: x = log5(a^2)

3099. незнакомка, 27 февраля 2011, 12:23:20
вы не знаете миним.балл по матиматике?

3100. незнакомка, 27 февраля 2011, 12:24:43
математике т .е

3101. Игорь, 27 февраля 2011, 13:37:32
3099. незнакомка, 27 февраля 2011, 12:23:20
вы не знаете миним.балл по матиматике?


Он меняется каждый год, балл не знаю, но знаю, что нужно решить 7 заданий части Б, чтобы двойки не было.

3102. Чебурашка))), 27 февраля 2011, 14:13:53
3099. незнакомка, 27 февраля 2011, 12:23:20
вы не знаете миним.балл по матиматике?


в 9 классе - 7 заданий, в 11 - 5 заданий.

Lexxus, большое спасибо за решение!!!

3103. Lexxus, 27 февраля 2011, 15:26:24
Пара присланных мне вариантов ЕГЭ по математике:

Вариант №3: B1-B5
Вариант №3: B6-B10
Вариант №3: B11-C6:


Вариант №6: B1-B5
Вариант №6: B6-B10
Вариант №6: B11-C6:

3104. Игорь, 27 февраля 2011, 19:27:16
Довольно сложные задания.

3105. Вася, 27 февраля 2011, 19:36:04
2622. любашка, 15 декабря 2010, 15:10:50
????ПОМОГУТ НЕТ???

1200=2х20х20 +4х20ха ,где а -боковое ребро . Из этого уравнения найдем
а .
Решение: 1200=800+80а 1200-800=80а 400=80а а=5
Ответ: 5

3106. Катя, 27 февраля 2011, 21:10:04
После встречи двух теплоходов один из них пошел на юг,а другой на запад.через 2 ч.После встречи расстояние между ними было 60км.Найдите скорость каждого теплохода,если известно,что скорость одного из них на 6км.ч больше скорости другого.
ПОМОГИТЕ РЕШИТЬ...
Лексус помоги решить...

3107. незнакомка, 27 февраля 2011, 21:24:43
А можно узнать, есть ли у вас ответы на задания, которые вы тут разместили?

3108. alfa20, 27 февраля 2011, 22:40:10
Катя, 27 февраля 2011, 21:10:04
Траектория - прямой угол. Расстояние между ними - гипотенуза треугольника. Обозначив через х скорость первого, (х+6)-скорость второго, получим, что через 2 часа (по теореме Пифагора):
(2x)^2+(2(x+6))^2=60^2.
Ответ: 18 и 24 км/ч.

3109. VladislavOfficiaL, 28 февраля 2011, 10:18:37
WaazzzuuuP!!!!!!!!! Особенно сибирякам!

Помогите пожалуйста решить уравнение.Чета я совсем туплю.

X^1+Lgx=10x


3110. Lexxus, 28 февраля 2011, 12:56:19
3109. VladislavOfficiaL, 28 февраля 2011, 10:18:37
X^1+Lgx=10x

Имеется в виду
x^(1+lg(x)) = 10*x ?

Тут два корня.

x*x^lg(x) = 10*x
x^lg(x) = 10 = x^log_x(10)
log_10(x) = log_x(10)
log_10(x) = 1/log_10(x)
(log_10(x))^2 = 1

log_10(x1) = 1, log_10(x2) = -1
x1 = 10, x2 = 1/10

3111. VladislavOfficiaL, 28 февраля 2011, 15:08:29
3110. Lexxus, 28 февраля 2011, 12:56:19
3109. VladislavOfficiaL, 28 февраля 2011, 10:18:37
X^1+Lgx=10x

Имеется в виду
x^(1+lg(x)) = 10*x ?

Тут два корня.

x*x^lg(x) = 10*x
x^lg(x) = 10 = x^log_x(10)
log_10(x) = log_x(10)
log_10(x) = 1/log_10(x)
(log_10(x))^2 = 1

log_10(x1) = 1, log_10(x2) = -1
x1 = 10, x2 = 1/10



Получается Lg^2*x=1 x1=1 x2=-1 или Lg*x^2=1 x1=1 x2=-1 Что правильно?

3112. Lexxus, 28 февраля 2011, 15:25:52
3111. VladislavOfficiaL, 28 февраля 2011, 15:08:29
Что правильно?

Я же специально скобочки ставлю, чтобы никогда не было разночтений.
(log_10(x))^2 - значит "десятичный логарифм икс, и всё это в квадрате". Это то же самое, что "логарифм в квадрате икс".

3113. VladislavOfficiaL, 28 февраля 2011, 15:32:59
Меня смутил тот факт,что у Ларичева задание под №1170

вот ссылка
http://reshebnic.narod.ru/Alimov-1092-1205.pdf

там x=0 и решений нет.Не верное решение что ли?

3114. VladislavOfficiaL, 28 февраля 2011, 15:42:01
Извиняюсь у Алимова

3115. VladislavOfficiaL, 28 февраля 2011, 15:48:02
ОДЗ x>0 значит ответ x=1

3116. Илья, 28 февраля 2011, 16:17:38
Дорогие пользователи.скоро поступят еще пару вариантов реального егэ,как будут сразу отошлю LexxusУ на почтовый ящик

3117. VladislavOfficiaL, 28 февраля 2011, 16:42:20
3115. VladislavOfficiaL, 28 февраля 2011, 15:48:02
ОДЗ x&gt;0 значит ответ x=1

Thanks ^_^

3118. Lexxus, 28 февраля 2011, 16:54:05
3113. VladislavOfficiaL, 28 февраля 2011, 15:32:59
http://reshebnic.narod.ru/Alimov-1092-1205.pdf

там x=0 и решений нет.Не верное решение что ли?

Не верное там решение.
Там написано:
lg^2(x)=1, а потом, с какого-то перепугу, x=0. Чушь.

Хреновый там решебник.

Да ты просто подставь в исходное уравнение значения x=10 и x=1/10.
Всё сойдется.

3119. VladislavOfficiaL, 28 февраля 2011, 16:58:29
Да я уже сам догнал,что это полный bullshit :(((

3120. Илья, 28 февраля 2011, 17:10:33
Люди,помогите решить вариант №6,B12 задачу выше изложенную,заранее спасибо

3121. baart, 28 февраля 2011, 18:31:54
Помогите решить плс
В треугольнике АВС АС=СВ=10СМ угол А =30градусов
ВК - перпендикуляр к плоскости треугольника и равен 5корней из 6 см
Найдите расстояние от точки К до АС

3122. ilo, 28 февраля 2011, 18:35:49
3120. Илья, 28 февраля 2011, 17:10:33
Люди,помогите решить вариант №6,B12 задачу выше изложенную,заранее спасибо
1-ая труба одна за x часов,вторая(с меньшей производительностью) за х+10;
1/х-такую часть бассейна 1-ая заполняет за час,1/(х+10)-2-ая;За 12 часов они заполнят весь бассейн, т.е.12/х+12/(х+10)=1; к общему знаменателю:(24х+120)/х(х+10)=х^2+10x/x(x+10); x^2-14x-120=0;x=6,второй корень не отвечает условию. Ответ:за16 та. что с меньшей производительностью.

3123. Lexxus, 28 февраля 2011, 19:22:18
3121. baart, 28 февраля 2011, 18:31:54
В треугольнике АВС АС=СВ=10СМ угол А =30градусов
ВК - перпендикуляр к плоскости треугольника и равен 5корней из 6 см
Найдите расстояние от точки К до АС

Проведем высоту BH к прямой AC.
Расстояние от K до AC будет равно длине отрезка KH.

KH можно найти по теореме Пифагора из прямоугольного треугольника KBH, зная BH.
BH можно найти из прямоугольного треугольника ABH, зная AB и угол A.
AB = AC*2*cos(угла A) = 10*2*sqrt(3)/2 = 10*sqrt(3)
BH = AB*sin(угла A) = 10*sqrt(3)*1/2 = 5*sqrt(3)
KH = sqrt(BH^2+BK^2) = sqrt(25*3+25*6) = sqrt(225) = 15

Ответ: 15 см

3124. Илья, 28 февраля 2011, 19:23:58
ilo спасибо!
но у меня при решении получилось х=20;x=-6 )))я что-то никак не пойму)

3125. Илья, 28 февраля 2011, 19:25:34
отрицательная работа не может быть)))и не может быть больше 12 часов,объясни может я где ошибку сделал при вычислении?

3126. ilo, 28 февраля 2011, 20:12:18
3125. Илья, 28 февраля 2011, 19:25:34
отрицательная работа не может быть)))и не может быть больше 12 часов,объясни может я где ошибку сделал при вычислении?

Это не работа ,а время,ты прав.отрицательным быть не может.Больше 12 может, потому что одна труба медленнее заполняет бассейн,чем вместе,12 ч -это вместе.
3124. Илья, 28 февраля 2011, 19:23:58
ilo спасибо!
но у меня при решении получилось х=20;x=-6 )))я что-то никак не пойму)

x^2-14x-120=0;D=196+480=676=26^2;x_1,2=(-14+-26)/2;x=6;

3127. Хелп ми..., 28 февраля 2011, 20:12:39
Периметр тупоугольного треугольника равен 45 см,а одна из его сторон больше другой на 12 см.Найдите стороны треугольника

3128. ilo, 28 февраля 2011, 20:17:45
3124. Илья, 28 февраля 2011, 19:23:58
ilo спасибо!
но у меня при решении получилось х=20;x=-6 )))я что-то никак не пойму)

Я допускаю oшибку при вычислении корней,исправляю:
x_1,2=14+-26/2;x=20-верно,второй -6 не отвечает условию

3129. Илья, 28 февраля 2011, 20:43:54
я туплю малясь)))ответ будет 20?так?

3130. Алина, 28 февраля 2011, 20:46:10
решите плиз
В треугольнике ABC AC=BC,угол С равен 62градуса.найдите внешний угол CBD

3131. ilo, 28 февраля 2011, 21:16:35
3129. Илья, 28 февраля 2011, 20:43:54
я туплю малясь)))ответ будет 20?так?

Так.А дальше решал? С1,например?

3132. Леонид , 1 марта 2011, 16:11:26
Для ИЛЬИ!!! В1 50:2=25 бвхлитровок, 50:3=16,.. 3-литровок (17), 25-17=8
В2 +, В3 по опред лог. (там не видно осн.) 1,6х+36,8=(1/6)^-2. 1,6x+36,8=36. 1,6x=36-36,8. 16x=-8. x=-0,5. В6 - там параллелограм с основанием 6 и высотой 3, площадь 6х3=18. В7 там степень, показатель которой сумма, запишем в виде произведения степеней (-0,3)^-1 * (0,3)^ (log 9 с основанием 0,3)= (-10/3) умножить на 9 (по основному лог. тождеству) =-30. В8 смотри точки пересечения графика с линиями параллельными оси ОХ: -3-2+0+2+3+4 = 4. В9 - аналогичное решение тебе показывал - смотри запись № 3085. В10 - можно решать двумя спообами- без проихводной: это парабола, ветви вниз, пересекает ОХ 40t - 5t^2=0. -5t(8-t)=0. t будут 0 и 8. Абссциса вршины параболы (0+8):2=4. Подставляем в h(t): h(4)=160 -5*16 =80. Пиши, всё ли понял. За варианты спасибо. Открыл только что, день не было дома. Успехов! Проблемы - степени и логарифмы. Надо наверстать, м.б. с репетитором. Заданий на них предлагают много и часто.

3133. фяц, 1 марта 2011, 17:49:04
Коментарий скрыт (грубость — 24%)показать

3134. ощдол, 1 марта 2011, 17:51:44
Коментарий скрыт (грубость — 64%)показать

3135. панкпрап, 1 марта 2011, 17:53:53
Коментарий скрыт (грубость — 48%)показать

3136. Танюшка))), 1 марта 2011, 19:55:34
Помогите, пожалуйста решить задачу
Двое рабочих, работая вместе, могут выполнить работу за 12 дней. За сколько
дней, работая отдельно, выполнит эту работу первый рабочий, если он за два
дня выполняет такую же часть работы, какую второй – за три дня?

заранее спасибо!!!!!!!!!!!

3137. Pandacho, 1 марта 2011, 19:58:56
Объем куба равен 27. Найдите площадь его поверхности.

MA.OB10.B9.80/innerimg0.jpg

3138. VladislavOfficiaL, 2 марта 2011, 04:33:44
3136. Танюшка))), 1 марта 2011, 19:55:34
Двое рабочих, работая вместе, могут выполнить работу за 12 дней. За сколько
дней, работая отдельно, выполнит эту работу первый рабочий, если он за два
дня выполняет такую же часть работы, какую второй – за три дня?



Производительность первого x, второго - y, вся работа - 1
Далее читаем задачу:
Двое рабочих, работая вместе, могут выполнить работу за 12 дней.
(x+y)12=1
за два дня выполняет такую же часть работы, какую второй – за три дня
2x=3y отсюда x=32y
Подставим в первое уравнение
(32y+y)12=1
52y=112
y=130
x=120
Производительность первого 120, значит он выполнит всю работу за 20 дней

3139. VladislavOfficiaL, 2 марта 2011, 05:32:29
Извини деление не проставил.

2x=3y отсюда x=3/2y
Подставим в первое уравнение
(3/2y+y)12=1
5/2y=1/12
y=1/30
x=1/20
Вся работа =1 Значит 1 делим на 1/20 получаем 20 дней.

Вот так думаю понятней.

3140. VladislavOfficiaL, 2 марта 2011, 05:39:04
3137. Pandacho, 1 марта 2011, 19:58:56
Объем куба равен 27. Найдите площадь его поверхности.




Объем куба равен а^3, где а- это длина ребра куба. Площадь одной его стороны равна а^2, а сторон у него 6.
а^3=27 => a=3 => a^2=9 => площадь поверхности куба равна 9*6=54
Площадь поверхности куба равна 54

3141. VladislavOfficiaL, 2 марта 2011, 05:50:44
Кто решил Вариант №3: B11-C6: В12?Если не трудно напишите ответ и свое решение.

3142. ilo, 2 марта 2011, 08:10:40
3141. VladislavOfficiaL, 2 марта 2011, 05:50:44
Кто решил Вариант №3: B11-C6: В12?Если не трудно напишите ответ и свое решение.

В12: х-по плану 1-й,у-второй;х+48-факт 1-й;у+0,1у-факт 2-й;Система:1)х+у=700;2)х+48+1,1у=800;х=400;у=320;48/400=0,12; Ответ:12%
C1-сверим ответы?

3143. ilo, 2 марта 2011, 09:57:00
C-1. вариант3
Решить систему уравнений:
■({■(cosy√(4sinx+1)=0;@6cos2x+cos2y+2sinx=3;)┤@D(f):4sinx+1≥0;sinx≥-1/4;@■(Из 1-го:1)cosy=0;или2)4sinx+1=0;sinx=-1/4;@1)если cosy=0;〖sin〗^2 y=1;6cos²x-6sin²x+cos²y-sin²y+2sinx=3;@■(6(1-sin^2 x)+0-1+2sinx=3;@6sin²x-sinx-1=0;@■(sinx=1/2;sinx=-1/3;-не отвечает условию sinx≥-1/4; @[■(x=π/6+2πn;@x=5π/6+2πn;)n∈Z┤@■((π/6+2πn;π/2+πn);(5π/6+2πn;π/2+πn)@2)sinx=-1/4;cos²x=15/16;@■(подставим в 6cos²x-6sin²x+cos²y-sin²y+2sinx=3;@90/16-6/16+cos²y-sin²y+2∙(-1/4)=3;@■(cos2y=-7/4;корней нет,не отвечает условию-1≤cos2y≤1@Ответ:@(π/6+2πn;π/2+πn);(5π/6+2πn;π/2+πn) )))))))

3144. ilo, 2 марта 2011, 10:06:25
или<a href="http://depositfiles.com/files/fpkq9ifmy">здесь/a>

3145. ilo, 2 марта 2011, 10:09:38
<a href="http://depositfiles.com/files/fpkq9ifmy">здесь</a>

3146. VladislavOfficiaL, 2 марта 2011, 11:26:10
3145. ilo, 2 марта 2011, 10:09:38
&lt;a href="http://depositfiles.com/files/fpkq9ifmy"&gt;здесь&lt;/a&gt;


С 1 это ппц.

3147. ilo, 2 марта 2011, 14:30:17
3146. VladislavOfficiaL, 2 марта 2011, 11:26:10


С 1 это ппц.

C-3 тоже методом рационализации

3148. VladislavOfficiaL, 2 марта 2011, 15:10:56
Не могу нарисовать чертеж Вариант №3: B11-C6: к С2 Помоги please ^_^

3149. VladislavOfficiaL, 2 марта 2011, 17:44:39
3148. VladislavOfficiaL, 2 марта 2011, 15:10:56
Не могу нарисовать чертеж Вариант №3: B11-C6: к С2 Помоги please ^_^


Кто решил эту задачу напишите ответ.У меня сторона основания пирамиды равна корню квадратному из 6 .Дальше находим площадь двух боковых перпендикулярных граней +площадь двух наклонных.Ответ не очень красивый.

3150. ilo, 2 марта 2011, 18:13:01
решение и чертёж


3151. VladislavOfficiaL, 2 марта 2011, 19:08:04
Спасибо,но я не могу открыть твои файлы.А можно на почту сбросить?

3152. Lexxus, 2 марта 2011, 20:29:35
ilo, я позволил себе сделать жаждущим путь к решению более коротким и слегка изменил пост.

Да, кстати. Советую сохранять документы в формате DOC (который "Документ Word 97-2003"): так ими сможет воспользоваться значительно большее количество народа.

3153. ilo, 2 марта 2011, 20:34:28
3152. Lexxus, 2 марта 2011, 20:29:35
ilo, я позволил себе сделать жаждущим путь к решению более коротким и слегка изменил пост.

Да, кстати. Советую сохранять документы в формате DOC (который "Документ Word 97-2003"): так ими сможет воспользоваться значительно большее количество народа.

Так лучше,учту.

3154. ilo, 2 марта 2011, 21:18:58
3151. VladislavOfficiaL, 2 марта 2011, 19:08:04
Спасибо,но я не могу открыть твои файлы.А можно на почту сбросить?

C-2 вариант 6: http://depositfiles.com/files/87uvgi6kx
Так читается?

3155. Ника, 2 марта 2011, 21:57:01
люююдиии,завтра пишу экзамен по математике,кто сможет помочь во время экзамена???
прошу вас,очень нужна помощь....((((

3156. Ника, 2 марта 2011, 22:12:14
поможет кто, нет?????

3157. VladislavOfficiaL, 3 марта 2011, 04:20:29
3154. ilo, 2 марта 2011, 21:18:58
3151. VladislavOfficiaL, 2 марта 2011, 19:08:04
Спасибо,но я не могу открыть твои файлы.А можно на почту сбросить?

C-2 вариант 6: http://depositfiles.com/files/87uvgi6kx
Так читается?


Cool :))Thanks!!!

http://depositfiles.com/files/fpkq9ifmy ---этот файл тоже в doc можно?^_^

3158. ilo, 3 марта 2011, 08:19:58
3157. VladislavOfficiaL, 3 марта 2011, 04:20:29

Cool :))Thanks!!!

http://depositfiles.com/files/fpkq9ifmy ---этот файл тоже в doc можно?^_^

http://depositfiles.com/files/3s0uyquqg

3159. VladislavOfficiaL, 3 марта 2011, 09:37:04
Thank!!! Для меня С1 это что-то запредельное ппц.

3160. VladislavOfficiaL, 3 марта 2011, 10:21:57
Вариант №3: B11-C6: Задача С1 Рассматриваем второе уравнение.После того,как оговорил что Cosy=0 и Sin^y=1 Потом все расписал через Cos двойного угла.Когда выносил за скобки 6Cos^2x-6Sin^2x=6(1-Sin^2x) Как так? Куда Cos^2x Исчез?

3161. VladislavOfficiaL, 3 марта 2011, 10:24:19
3158. ilo, 3 марта 2011, 08:19:58
3157. VladislavOfficiaL, 3 марта 2011, 04:20:29

Cool :))Thanks!!!

http://depositfiles.com/files/fpkq9ifmy ---этот файл тоже в doc можно?^_^

http://depositfiles.com/files/3s0uyquqg


Вариант №3: B11-C6: Задача С1 Рассматриваем второе уравнение.После того,как оговорил что Cosy=0 и Sin^y=1 Потом все расписал через Cos двойного угла.Когда выносил за скобки 6Cos^2x-6Sin^2x=6(1-Sin^2x) Как так? Куда Cos^2x Исчез?

3162. ilo, 3 марта 2011, 11:39:01
при перепечатывании пропущено слагаемое,подробно так:
6cos^2(x)-6sin^2(x)+0-1+2sin(x)=3;выразим всё через cosx: 6(1-sin^2(x))-6sin^2(x)-4+2sin(x)=0;6-sin^2(x)-6sin^2(x)-4+2sin(x)=0дальше верно
спасибо за замечание

3163. ilo, 3 марта 2011, 11:55:50
3161. VladislavOfficiaL, 3 марта 2011, 10:24:19

Вариант №3: B11-C6: Задача С1 Рассматриваем второе уравнение.После того,как оговорил что Cosy=0 и Sin^y=1 Потом все расписал через Cos двойного угла.Когда выносил за скобки 6Cos^2x-6Sin^2x=6(1-Sin^2x) Как так? Куда Cos^2x Исчез?

Здесь без опечатки:http://depositfiles.com/files/krzv6pq3q

3164. ilo, 3 марта 2011, 12:01:20
3162. ilo, 3 марта 2011, 11:39:01
при перепечатывании пропущено слагаемое,подробно так:
6cos^2(x)-6sin^2(x)+0-1+2sin(x)=3;выразим всё через cosx: 6(1-sin^2(x))-6sin^2(x)-4+2sin(x)=0;6-sin^2(x)-6sin^2(x)-4+2sin(x)=0дальше верно
спасибо за замечание

Здесь в последнем уравнении:6-6sin^2(x)-6sin^2(x)-4+2sin(x)=0

3165. VladislavOfficiaL, 3 марта 2011, 12:45:46
3162. ilo, 3 марта 2011, 11:39:01
при перепечатывании пропущено слагаемое,подробно так:
6cos^2(x)-6sin^2(x)+0-1+2sin(x)=3;выразим всё через cosx: 6(1-sin^2(x))-6sin^2(x)-4+2sin(x)=0;6-sin^2(x)-6sin^2(x)-4+2sin(x)=0дальше верно
спасибо за замечание


Cos2y=-7/4 число у тебя отрицательное.
У меня Cos2y=7/4 получилось положительное,но также не отвечает по условию.Ответы сошлись.Разобрался,спасибо!

3166. VladislavOfficiaL, 3 марта 2011, 13:20:52
3165. VladislavOfficiaL, 3 марта 2011, 12:45:46
3162. ilo, 3 марта 2011, 11:39:01
при перепечатывании пропущено слагаемое,подробно так:
6cos^2(x)-6sin^2(x)+0-1+2sin(x)=3;выразим всё через cosx: 6(1-sin^2(x))-6sin^2(x)-4+2sin(x)=0;6-sin^2(x)-6sin^2(x)-4+2sin(x)=0дальше верно
спасибо за замечание


Cos2y=-7/4 число у тебя отрицательное.
У меня Cos2y=7/4 получилось положительное,но также не отвечает по условию.Ответы сошлись.Разобрался,спасибо!


Ответы у тебя верные.Я сам ступил,в правую часть переносил знак не поменял Cos2y=-7/4

3167. Nastya, 3 марта 2011, 17:11:10
ПОМОГИТЕ, СРОЧНО!!!

часть С. №С3
дроб:
log с основанием2 * (3*2^(x-1) - 1) / x >= 1

3168. Lexxus, 3 марта 2011, 18:04:02
3167. Nastya, 3 марта 2011, 17:11:10
log с основанием2 * (3*2^(x-1) - 1) / x >= 1

Не знаю, насколько это получилось срочно, но решение такое.

ОДЗ.
1. x не равен нулю.
2. 3*2^(x-1)-1 > 0
2^(x-1) > 1/3
x > log_2(1/3)+1 = log_2(2/3)
Примерно вычисляем, что log_2(2/3) - это где-то между -1 и 0.

Теперь собственно неравенство.
(log_2(3*2^(x-1)-1) - x)/x >= 0

Ноль знаменателя (x=0) понятен, разбираемся с числителем

log_2(3*2^(x-1)-1) = x
log_2(3*2^(x-1)-1) = log_2(2^x)
3*2^(x-1)-1 = 2^x
(3*2^(x-1)-1)/2^x = 1
3*2^(-1) - 1/2^x = 1
3/2 - 2^(-x) = 1
2^(-x) = 1/2
x = 1 (можно было и угадать, на самом деле).

Итак, у нас две точки, где выражение меняет знак: 0 и 1
Прикидываем, какой у него знак будет, например, при x=2:
(log_2(5)-2)/2 - это больше нуля.
Значит,
при x>1 - "+"
при 0<x<1 - "-"
при x<0 - "+"

А, учитывая ОДЗ, получаем
Ответ: (log_2(2/3), 0) и [1,бесконечность)

3169. Наташа, 3 марта 2011, 20:12:53
Помогите пожалуйста! Очень срочно нужно решение этой задачи!

Дан прямоугольный параллелепипед ABCDA1B1C1D1, основание которого - квадрат ABCD. AC = 6 sqrt (2) см. B1B = 4 sqrt (3) см. Вычислите градусную меру двугранного угла B1ADB.

3170. Даша, 3 марта 2011, 20:19:41
Всем доброго вечера! Помогите пожалуйста!!!

Точка M равноудалена от всех вершин равнобедренного прямоугольного треугольника ABC ( угол C = 90 градусов), AC = BC = 4 см. Расстояние от точки M до плоскости треугольника равно 2 sqrt (3) см.
а) Докажите, что плоскость AMB перпендикулярна плоскости ABC.
б) Найдите угол между прямой MC и плоскостью ABC.


заранее спасибо!

3171. VladislavOfficiaL, 4 марта 2011, 06:33:30
3157. VladislavOfficiaL, 3 марта 2011, 04:20:29
3154. ilo, 2 марта 2011, 21:18:58
3151. VladislavOfficiaL, 2 марта 2011, 19:08:04
Спасибо,но я не могу открыть твои файлы.А можно на почту сбросить?

C-2 вариант 6: http://depositfiles.com/files/87uvgi6kx
Так читается?

Cool :))Thanks!!!

http://depositfiles.com/files/fpkq9ifmy ---этот файл тоже в doc можно?^_^
Вариант №6: B11-C6: C2 сейчас разбираю задачку и никак не пойму? Почему у тебя треугольники,площади которых ты нашел имеют одну и туже высоту?

3172. ilo, 4 марта 2011, 09:37:39
Думаю, что так получается при заданных условиях.Проверь вычисления-может ошибка?

3173. ilo, 4 марта 2011, 09:52:11
хотя фигня получается-наклонная равна перпендикуляру-посмотрю
^Вариант №6: B11-C6: C2 сейчас разбираю задачку и никак не пойму? Почему у тебя треугольники,площади которых ты нашел имеют одну и туже высоту?[/quote]

3174. VladislavOfficiaL, 4 марта 2011, 10:02:04
3172. ilo, 4 марта 2011, 09:37:39
Думаю, что так получается при заданных условиях.Проверь вычисления-может ошибка?


Треугольник АВ1С равносторонний.S= АС/2 *h сторона В1С-это гипотенуза она нам известна Значит по теореме Пифагора находим h.Ты указываешь h треугольника СВ1В как так? Я не понимаю? Объясни пожалуйста^_^

3175. VladislavOfficiaL, 4 марта 2011, 10:25:12
3174. VladislavOfficiaL, 4 марта 2011, 10:02:04
3172. ilo, 4 марта 2011, 09:37:39
Думаю, что так получается при заданных условиях.Проверь вычисления-может ошибка?


Треугольник АВ1С равносторонний.S= АС/2 *h сторона В1С-это гипотенуза она нам известна Значит по теореме Пифагора находим h.Ты указываешь h треугольника СВ1В как так? Я не понимаю? Объясни пожалуйста^_^


Я когда нашел h =а Потом через SinВ1НВ= В1В/В1H нахожу угол=корню из2/2
=пи/4 или 45 градусов. Ответы у нас сходятся.Мне не ясно зачем ты находил В1В? Когда по условию и так ясно,что она в два раза меньше диагонали? Я решил эту задачу немного по-другому,но ответ тот же.В твоем решении я так и не разобрался.

3176. ilo, 4 марта 2011, 10:28:23
3174. VladislavOfficiaL, 4 марта 2011, 10:02:04
3172. ilo, 4 марта 2011, 09:37:39
Думаю, что так получается при заданных условиях.Проверь вычисления-может ошибка?


Треугольник АВ1С равносторонний.S= АС/2 *h сторона В1С-это гипотенуза она нам известна Значит по теореме Пифагора находим h.Ты указываешь h треугольника СВ1В как так? Я не понимаю? Объясни пожалуйста^_^

Исправь:высота к стороне АС равна sqrt(3/2a^2-a^2/2)=a-здесь ошибка вычислительнаяДальше S AB1C=1/2*asqrt2*a=(sqrt2/2)a^2; cos=1/2,угол-pi/6
Здесь исправленоhttp://depositfiles.com/files/dp0al4gts

3177. ilo, 4 марта 2011, 10:42:43
3175. VladislavOfficiaL, 4 марта 2011, 10:25:12


Я когда нашел h =а Потом через SinВ1НВ= В1В/В1H нахожу угол=корню из2/2
=пи/4 или 45 градусов. Ответы у нас сходятся.Мне не ясно зачем ты находил В1В? Когда по условию и так ясно,что она в два раза меньше диагонали? Я решил эту задачу немного по-другому,но ответ тот же.В твоем решении я так и не разобрался.

Там не нужен sin B1HB-это угол между BH и HB это следует из того , что треугольник B1HB-равнобедренный, а надоугол между плоскостями

3178. VladislavOfficiaL, 4 марта 2011, 11:00:22
3176. ilo, 4 марта 2011, 10:28:23
3174. VladislavOfficiaL, 4 марта 2011, 10:02:04
3172. ilo, 4 марта 2011, 09:37:39
Думаю, что так получается при заданных условиях.Проверь вычисления-может ошибка?


Треугольник АВ1С равносторонний.S= АС/2 *h сторона В1С-это гипотенуза она нам известна Значит по теореме Пифагора находим h.Ты указываешь h треугольника СВ1В как так? Я не понимаю? Объясни пожалуйста^_^
Исправь:высота к стороне АС равна sqrt(3/2a^2-a^2/2)=a-здесь ошибка вычислительнаяДальше S AB1C=1/2*asqrt2*a=(sqrt2/2)a^2; cos=1/2,угол-pi/6
Здесь исправленоhttp://depositfiles.com/files/dp0al4gts


Просто жесть!!!!!! Как ты берешь отношение двух площадей,если треугольники не подобны?

3179. ilo, 4 марта 2011, 11:24:03
У Корянова при подготовке к ЕГЭ в сборнике даются рекомендации и эта формула,по ней решаются ряд задач,когда угол между плоскостями сложно определить другим способом.Послать рекомендации?

3180. VladislavOfficiaL, 4 марта 2011, 11:29:02
3179. ilo, 4 марта 2011, 11:24:03
У Корянова при подготовке к ЕГЭ в сборнике даются рекомендации и эта формула,по ней решаются ряд задач,когда угол между плоскостями сложно определить другим способом.Послать рекомендации?


А то?! Я же хочу знать где и в чем мои просчеты:))

3181. ilo, 4 марта 2011, 11:38:37
можно координатно-векторным методом попробовать,а вообще на угол между плоскостями это последнее,чтобы въехать, надо порешать на угол между прямыми,прямой и плоскостью

3182. VladislavOfficiaL, 4 марта 2011, 11:41:14
Во скачал вот с этого сайта:
http://www.ctege.org/content/view/1260/42/
То что ты хотел мне отправить?

3183. ilo, 4 марта 2011, 12:05:48
3182. VladislavOfficiaL, 4 марта 2011, 11:41:14
Во скачал вот с этого сайта:
http://www.ctege.org/content/view/1260/42/
То что ты хотел мне отправить?

http://depositfiles.com/files/as2td3e95

3184. ilo, 4 марта 2011, 12:12:30
То, я в тему только. В сборнике,если по порядку неплохая система.

3185. VladislavOfficiaL, 4 марта 2011, 12:31:16
Вообще я слышал Денещеву и ее соаавторов нужно скачивать или покупать для подготовке к ЕГЭ Она в прошлом году рецензировала ЕГЭ по матишу

3186. obts, 4 марта 2011, 13:18:43
При решении заданий открытой базы части В нажимаю в разделе "Решённость" "все", но программа отправляет меня в "нерешённые". И еще было бы неплохо нажимать "ответ" энтером, а не мышкой.

3187. ilo, 4 марта 2011, 13:21:01
и раньше её рекомендовали,не знаю,какой сборник надо,чтобы там была система ,а просто наобум решать набор задач сложно.В выложенных вариантах много чего проверяется.

3188. VladislavOfficiaL, 4 марта 2011, 13:30:18
У тебя и так все зашибись,мне бы твои знания ,не переживалбы по поводу сдачи ЕГЭ Я чем больше просматриваю учебного материала,тем тупее становлюсь.Ващее жесть.

3189. obts, 4 марта 2011, 13:36:13
3185. VladislavOfficiaL, 4 марта 2011, 12:31:16
Вообще я слышал Денещеву и ее соаавторов нужно скачивать или покупать для подготовке к ЕГЭ Она в прошлом году рецензировала ЕГЭ по матишу

Л.О.Денищева

3190. VladislavOfficiaL, 4 марта 2011, 14:12:38
3189. obts, 4 марта 2011, 13:36:13
3185. VladislavOfficiaL, 4 марта 2011, 12:31:16
Вообще я слышал Денещеву и ее соаавторов нужно скачивать или покупать для подготовке к ЕГЭ Она в прошлом году рецензировала ЕГЭ по матишу

Л.О.Денищева


Денищева Лариса Олеговна:))

3191. ilo, 4 марта 2011, 14:13:21
3188. VladislavOfficiaL, 4 марта 2011, 13:30:18
Я чем больше просматриваю учебного материала,тем тупее становлюсь.Ващее жесть.

Аналогично

3192. obts, 4 марта 2011, 14:43:51
задача 56629 не решается? в ответе корень?

3193. Lexxus, 4 марта 2011, 15:04:11
3192. obts, 4 марта 2011, 14:43:51
задача 56629 не решается? в ответе корень?

Нет, в ответе хорошее, добротное, целое число :)

3194. obts, 4 марта 2011, 15:07:45
3193. Lexxus, 4 марта 2011, 15:04:11
3192. obts, 4 марта 2011, 14:43:51

задача 56629 не решается? в ответе корень?

Нет, в ответе хорошее, добротное, целое число :)

площадь треугольника= 0.5*х*х*син30
1225=0.25*х*х
х*х=5000
х=50скрт(2)

3195. Lexxus, 4 марта 2011, 15:27:09
3194. obts, 4 марта 2011, 15:07:45
1225=0.25*х*х
х*х=5000


1225*4 не равно 5000 ;)

3196. Kattu, 4 марта 2011, 15:36:59
Lexxus! Исправьте, пожалуйста, ошибки в заданиях 4969, 19365, 5417. Спасибо!

3197. VladislavOfficiaL, 4 марта 2011, 15:39:37
3195. Lexxus, 4 марта 2011, 15:27:09
3194. obts, 4 марта 2011, 15:07:45
1225=0.25*х*х
х*х=5000


1225*4 не равно 5000 ;)


Ответ 70см.

3198. obts, 4 марта 2011, 15:43:16
3197. VladislavOfficiaL, 4 марта 2011, 15:39:37
3195. Lexxus, 4 марта 2011, 15:27:09

3194. obts, 4 марта 2011, 15:07:45

1225=0.25*х*х

х*х=5000



1225*4 не равно 5000 ;)



Ответ 70см.

ок. заклинило

3199. VladislavOfficiaL, 4 марта 2011, 15:54:04
3198. obts, 4 марта 2011, 15:43:16
3197. VladislavOfficiaL, 4 марта 2011, 15:39:37
3195. Lexxus, 4 марта 2011, 15:27:09

3194. obts, 4 марта 2011, 15:07:45

1225=0.25*х*х

х*х=5000



1225*4 не равно 5000 ;)



Ответ 70см.

ок. заклинило

S=a*b*Sin30 так как а=в=х Sin30=1/2 Ответ 70см.

3200. VladislavOfficiaL, 4 марта 2011, 15:59:03
3199. VladislavOfficiaL, 4 марта 2011, 15:54:04
3198. obts, 4 марта 2011, 15:43:16
3197. VladislavOfficiaL, 4 марта 2011, 15:39:37
3195. Lexxus, 4 марта 2011, 15:27:09

3194. obts, 4 марта 2011, 15:07:45

1225=0.25*х*х

х*х=5000



1225*4 не равно 5000 ;)



Ответ 70см.

ок. заклинило
S=a*b*Sin30 так как а=в=х Sin30=1/2 Ответ 70см.


S=a*b/2*Sin30 так как а=в=х Sin30=1/2 Ответ 70см.

3201. VladislavOfficiaL, 4 марта 2011, 17:28:31
У меня почему-то кнопка ответить не работает.Почему?

3202. Ириска, 4 марта 2011, 20:47:11
Ребята,помогите пожалуйста,буду очень благодарна!
Точка K, не принадлежащая плоскости равностороннего треугольника, удалена от каждой его вершины на расстояние см, а от каждой его стороны – на 2 см. Найдите расстояние от точки K до плоскости треугольника.

Угол между плоскостями двух равнобедренных треугольников ABC и BCD, имеющих общую боковую сторону BC, равен 900. Найдите расстояние между точками A и D, если основание каждого треугольника равно a, а каждая боковая сторона равна b.

Внутри двугранного угла из точки M, принадлежащей его ребру, проведен к нему перпендикуляр, на котором отложен отрезок MN, в два раза больший своей ортогональной проекции на одну из граней двугранного угла. Найдите угол, который образует MN с другой гранью, если двугранный угол равен 100.

Через данную точку проведите прямую, параллельную данной плоскости и перпендикулярную данной прямой.

спасибо)

3203. Lexxus, 4 марта 2011, 21:16:03
3196. Kattu, 4 марта 2011, 15:36:59
Lexxus! Исправьте, пожалуйста, ошибки в заданиях 4969, 19365, 5417. Спасибо!

Исправлено, спасибо.

3186. obts, 4 марта 2011, 13:18:43
При решении заданий открытой базы части В нажимаю в разделе "Решённость" "все", но программа отправляет меня в "нерешённые".

Чёрт, уже год никто не замечал. Спасибо, поправил.

3186. obts, 4 марта 2011, 13:18:43
И еще было бы неплохо нажимать "ответ" энтером, а не мышкой.

Во время подведения указателя мышки к кнопке можно лишний раз задуматься о правильности данного ответа :)

3204. obts, 5 марта 2011, 12:24:15
[quote]3202. Ириска, 4 марта 2011, 20:47:11
Ребята,помогите пожалуйста,буду очень благодарна!

Точка K, не принадлежащая плоскости равностороннего треугольника, удалена от каждой его вершины на расстояние см, а от каждой его стороны – на 2 см. Найдите расстояние от точки K до плоскости треугольника.

На сколько см удалена точка К от вершин?
Допустим в треугольной пирамиде КАВС КА=КВ=КС=3 см.
Рассмотрим равнобедренный треугольник КАВ. КА=КВ=3, КН=2 высота. По т. Пифагора АН=корень(9-4)=корень5, АВ=2корня5.
Рассмотрим равносторонний треугольник АВС. АВ=ВС=АС=2скрт5. По т. Пифагора СН=корень(20-5)=корень15.
КО высота пирамиды. О лежит на СН, и делит его СО=2ОН.
Рассмотрим треугольник СКО. СК=3, СО=2/3*корень15, О прямой угол. По т. Пифагора КО=корень(9-4/9*15)=корень(21/9)=(корень21)/3

3205. obts, 5 марта 2011, 12:37:15
в задаче 47501 угол ВАД=76, АД биссектриса, след-но ВАС=152. треугольник тупоугольный и бис-са АД не пересекает высоту СН.

3206. obts, 5 марта 2011, 12:40:16
и еще по поводу оформления сайта. чтобы узнать номер задания я вынужден вводить ответ, даже если уверен, что он неправильный. исправьте, пож-та.

3207. obts, 5 марта 2011, 12:46:41
нельзя ли поменять хронологию комментов, чтобы видеть последние вверху?

3208. VladislavOfficiaL, 6 марта 2011, 08:13:34
Задание B4 (4859) Объясните,как решали это задание.

3209. Lexxus, 6 марта 2011, 10:20:30
3206. obts, 5 марта 2011, 12:40:16
и еще по поводу оформления сайта. чтобы узнать номер задания я вынужден вводить ответ, даже если уверен, что он неправильный. исправьте, пож-та.

Так и задумано. Статистика решаемости заданий при этом получается более объективной.

3207. obts, 5 марта 2011, 12:46:41
нельзя ли поменять хронологию комментов, чтобы видеть последние вверху?

Только если в качестве опции. Я подумаю над этим.

3210. VladislavOfficiaL, 6 марта 2011, 11:46:02
3201. VladislavOfficiaL, 4 марта 2011, 17:28:31
У меня почему-то кнопка ответить не работает.Почему?


Меня кто-нибудь услышит?Lexxus ты же здесь рулишь?

3211. VladislavOfficiaL, 6 марта 2011, 11:50:09
3209. Lexxus, 6 марта 2011, 10:20:30
3206. obts, 5 марта 2011, 12:40:16
и еще по поводу оформления сайта. чтобы узнать номер задания я вынужден вводить ответ, даже если уверен, что он неправильный. исправьте, пож-та.

Так и задумано. Статистика решаемости заданий при этом получается более объективной.

3207. obts, 5 марта 2011, 12:46:41
нельзя ли поменять хронологию комментов, чтобы видеть последние вверху?

Только если в качестве опции. Я подумаю над этим.


Вообще можно поставить рульную прокрутку.Lexxus я думаю тебе не составит труда поставить скрипт.

3212. obts, 6 марта 2011, 13:26:22
3208. VladislavOfficiaL, 6 марта 2011, 08:13:34
Задание B4 (4859) Объясните,как решали это задание.


Меньшее основание равнобедренной трапеции равно 6. Высота трапеции равна 10. Тангенс острого угла равен 2. Найдите большее основание.

тангенс=противолежащий катет/прилежащий. тогда проекция боковой стороны на основание = 5. Большее основание = 5+6+5=11

3213. VladislavOfficiaL, 6 марта 2011, 15:55:06
3212. obts, 6 марта 2011, 13:26:22
3208. VladislavOfficiaL, 6 марта 2011, 08:13:34
Задание B4 (4859) Объясните,как решали это задание.


Меньшее основание равнобедренной трапеции равно 6. Высота трапеции равна 10. Тангенс острого угла равен 2. Найдите большее основание.

тангенс=противолежащий катет/прилежащий. тогда проекция боковой стороны на основание = 5. Большее основание = 5+6+5=11

В этом я разобрался.Извиняюсь не то задание написал.
Вот то что я хотел узнать
4853

3214. obts, 6 марта 2011, 16:38:39
3213. VladislavOfficiaL, 6 марта 2011, 15:55:06
3212. obts, 6 марта 2011, 13:26:22

3208. VladislavOfficiaL, 6 марта 2011, 08:13:34

Задание B4 (4859) Объясните,как решали это задание.



Меньшее основание равнобедренной трапеции равно 6. Высота трапеции равна 10. Тангенс острого угла равен 2. Найдите большее основание.



тангенс=противолежащий катет/прилежащий. тогда проекция боковой стороны на основание = 5. Большее основание = 5+6+5=11

В этом я разобрался.Извиняюсь не то задание написал.

Вот то что я хотел узнать

4853


по основному тригонометрическому тождеству косА=0,6. косВ=кос(180-А)=-косА=-0,6

3215. Лёлька, 7 марта 2011, 11:52:21
Помогите, пожалуйста, решить В10!!!


По закону Ома для полной цепи сила тока, измеряемая в амперах, равна I=e/R+r, где E - ЭДС источника (в вольтах), r=2 Ом - его внутреннее сопротивление, R - сопротивление цепи (в Омах). При каком наименьшем сопротивлении цепи сила тока будет составлять не более 50% от силы тока короткого замыкания Iкз=E/r? (Ответ выразите в Омах).

3216. ilo, 7 марта 2011, 12:31:19
[quote]3215. Лёлька, 7 марта 2011, 11:52:21
Помогите, пожалуйста, решить В10!!!

e/(R+2)<=1/2*e/2; разделим обе части ур-я на e: 1/(R+2)-1/4<=0; (4-R-2)/(R+2)*4<=0; 2-R<=0;R>=2(Oм)Наименьшее 2Ом.

3217. Лёлька, 7 марта 2011, 12:40:03
Ilo, благодарю!!!:)

3218. obts, 8 марта 2011, 21:01:20
с праздником, милые девушки!!!

3219. Женечка)), 9 марта 2011, 20:08:54
помогите, пожалуйста!!!


1)Через вершину к треугольника мкп проведена прямая кн, перпендикулярная плоскости треугольника.кн=15, мк=кп=10, мп=12.Найти расстояние от точки м до прямой.

2)дан прямоугольный параллелепипед авсда1в1с1д1.
найди двугранный угол в1адв, если ас=6 корней из 2х,ав1=4корня из 3х.АВСД-квадрат.


3)основание тетраэдра давс-треугольник авс со сторонами 13, 14, 15.расстояние от точки д до сторон треугольника равно 5.
Найти.расстояние от точки д до плоскости авс.д не принадлежит плоскости треугольника.

пожалуйста, распишите решение, сегодня!!!!!!!!
заранее благодарна.прсто мы не может решить с подругами((((

3220. Lexxus, 9 марта 2011, 21:43:02
3219. Женечка)), 9 марта 2011, 20:08:54
1)Через вершину к треугольника мкп проведена прямая кн, перпендикулярная плоскости треугольника.кн=15, мк=кп=10, мп=12.Найти расстояние от точки м до прямой.

До какой прямой?

3221. Lexxus, 9 марта 2011, 21:46:19
3219. Женечка)), 9 марта 2011, 20:08:54
2)дан прямоугольный параллелепипед авсда1в1с1д1.
найди двугранный угол в1адв, если ас=6 корней из 2х,ав1=4корня из 3х.АВСД-квадрат.

Градусная мера двугранного угла B1ADB будет равна углу B1AB. AB находишь как сторону квадрата с известной диагональю. Косинус искомого угла = AB/AB1

3222. Lexxus, 9 марта 2011, 22:03:04
3219. Женечка)), 9 марта 2011, 20:08:54
3)основание тетраэдра давс-треугольник авс со сторонами 13, 14, 15.расстояние от точки д до сторон треугольника равно 5.
Найти.расстояние от точки д до плоскости авс.д не принадлежит плоскости треугольника.

Поскольку расстояние от точки D до всех ребер треугольника ABC одинаково, то высота DO, опущенная из точки D к плоскости ABC, попадает в центр окружности O, вписанной в треугольник ABC.
Находишь радиус вписанной окружности, а потом и искомую высоту как катет прямоугольного треугольника с гипотенузой 5 и катетом, равным найденному радиусу.

3223. наталья, 10 марта 2011, 09:36:42
предлогаю обсудить решение задачи в10 28219 про гудок поезда

3224. ilo, 10 марта 2011, 17:31:02
245/(1-v/300)-245>=5;245*300-250(300-v)>=0;250v>=1500;v>=6м/с(21,6км/ч)

3225. VladislavOfficiaL, 10 марта 2011, 18:37:40
Задание B10 (28127) Как решается эта задача? Помогите please.

3226. ilo, 10 марта 2011, 20:46:26
3225. VladislavOfficiaL, 10 марта 2011, 18:37:40
Задание B10 (28127) Как решается эта задача? Помогите please.

20t+4t^2/2<=1200;умножаем обе части на 2:40t+4t^2<=2400;t^2+10t-2400<=0; D=2500;t_1=-30;t_2=20.Решение неравенства: -30<=t<=20;Время после начала работы t_0=0.Значит, от 0 до t=20 угол меньше 1200,предельное время 20,после 20 угол будет больше 1200.Ответ:20

3227. VladislavOfficiaL, 11 марта 2011, 11:38:08
3226. ilo, 10 марта 2011, 20:46:26
3225. VladislavOfficiaL, 10 марта 2011, 18:37:40
Задание B10 (28127) Как решается эта задача? Помогите please.

20t+4t^2/2<=1200;умножаем обе части на 2:40t+4t^2<=2400;t^2+10t-2400<=0; D=2500;t_1=-30;t_2=20.Решение неравенства: -30<=t<=20;Время после начала работы t_0=0.Значит, от 0 до t=20 угол меньше 1200,предельное время 20,после 20 угол будет больше 1200.Ответ:20


Спасибо.Как оказывается все просто.Когда начинаешь читать задание,вроде все так ужасно.Стоит только немного подумать и ответ готов:))

3228. Катя, 11 марта 2011, 17:22:39
В треугольнике АВС угол С равен 90 градусов, синусА= 7/8, АС= квадратный корень из 15. Найти АВ
Помогите пожалуйста решить.

3229. Rusalka, 11 марта 2011, 18:38:04
Привет! помогите пожалуйста решить задачки очень надо)
1) В правильной четырехугольной пирамиде ,высота =3м,боковое ребро=5. Найти объем пирамиды?
2) Сторона основания правильной четырехугольной пирамиды 8 см ,боковое ребро наклонено к плоскости основания под углом 45градусов . Найти объем пирамиды?

3230. ilo, 11 марта 2011, 18:38:29
3228. Катя, 11 марта 2011, 17:22:39
В треугольнике АВС угол С равен 90 градусов, синусА= 7/8, АС= квадратный корень из 15. Найти АВ
Помогите пожалуйста решить.

По основному триг. тождеству-cosA=sqrt(1-46/64)=sqrt15/8;AC/AB=cosA;AB=AC/cosA;AB=8.

3231. Катя, 11 марта 2011, 19:16:27
Спасибо)))

3232. Катя, 11 марта 2011, 19:31:08
Rusalka, 11 марта 2011, 18:38:04
1) В правильной четырехугольной пирамиде ,высота =3м,боковое ребро=5. Найти объем пирамиды?
V= 1/3 *h*а&#178;
V=1/3 * 3 * 5&#178;= 25
ответ: 25
если что не очень уверенна(((

3233. Катя, 11 марта 2011, 19:34:11
[quote]3232. Катя, 11 марта 2011, 19:31:08
Rusalka, 11 марта 2011, 18:38:04
1) В правильной четырехугольной пирамиде ,высота =3м,боковое ребро=5. Найти объем пирамиды?
V= 1/3 *h*а в квадрате
V=1/3 * 3 * 5 в квадрате= 25
ответ: 25
если что не очень уверенна(((

3234. rusalka, 11 марта 2011, 19:49:33
Катюш,спасибо)

3235. Rusalka, 11 марта 2011, 19:52:20
Помогите! В основании прямой трехугольной призмы лежит прямоугольный треугольник с катетами 8 и 6. Найти объем призмы,если её боковая поверхность 120.

3236. Катя, 11 марта 2011, 20:00:44
А первый ответ сходиться?

3237. Катя, 11 марта 2011, 20:04:36
В основании прямой треугольной призмы лежит прямоугольный треугольник с катетами 8 и 6 см. Найти боковое ребро призмы, если ее боковая поверхность равна 120 квадратных сантиметров.


Решение.
Сначала найдем гипотенузу основания призмы.

AB2 = AC2 + BC2
AB2 = 82 + 62
AB2 = 64 + 36
AB = &#8730;100
AB = 10

Обозначим боковое ребро призмы как h . Боковое ребро одновременно является и высотой призмы, поскольку по условию задачи призма является прямой. Тогда площадь боковой поверхности призмы является суммой площадей трех прямоугольников - ACC1A1, CBB1C1 и ABB1A1 или, если подставить известные значения катетов основания призмы, то

10h + 6h + 8h = 120
24h = 120
h =5

Ответ: ребро прямоугольной призмы с прямоугольным треугольником в основании равно 5 см.

3238. Катя, 11 марта 2011, 20:07:15
АВ= квадратный корень из 100

3239. катрин), 11 марта 2011, 21:16:44
Пожалуйста помогите найти точки минимума функции у=(7-х)е в степени 7-х.


3240. Анастасия, 12 марта 2011, 15:52:54
Помогите, пожалуйста, решить задачу.
В кубе ABCDA1B1C1D1, найдите косинус угла между плоскостями BA1C1 и BA1D1

3241. Lexxus, 12 марта 2011, 18:15:48
3240. Анастасия, 12 марта 2011, 15:52:54
В кубе ABCDA1B1C1D1, найдите косинус угла между плоскостями BA1C1 и BA1D1

Пусть ребро куба равно 1.
Тогда BA1C1 - это равносторонний треугольник со стороной sqrt(2).
А BA1D1С - это прямоугольник со сторонами A1D1 = BC = 1, A1B = D1C = sqrt(2).

Угол между плоскостями будет равен углу С1NM, где N и M - это середины отрезков A1B и D1C соответственно.

Этот угол найдем из треугольника С1NM по теореме коминусов:
С1M^2 = C1N^2+MN^2-2*C1N*MN*cos(C1NM).

C1M = половине диагонали грани куба = sqrt(2)/2.
NM = ребру куба = 1.
C1N = высоте равностороннего треугольника BA1C1 = sqrt(2)*sqrt(3)/2 =
= sqrt(6)/2.

cos(C1NM) = (3/2 + 1 - 1/2)/2/(sqrt(6)/2) = 2/sqrt(6)

В ответе будет arccos(2/sqrt(6))

3242. Илья, 13 марта 2011, 08:55:24
Доброе утро всем!Я болел меня так долго не было и не смог представить вам еще варианты реального ЕГЭ,к вечеру пришлю еще один вариант ЕГЭ Lexxus'у,потом он вам выложит сюда

3243. Анастасия, 13 марта 2011, 10:31:28
Lexxus, спасибо большое!!

3244. Lexxus, 13 марта 2011, 11:03:09
3242. Илья, 13 марта 2011, 08:55:24
Доброе утро всем!Я болел меня так долго не было и не смог представить вам еще варианты реального ЕГЭ,к вечеру пришлю еще один вариант ЕГЭ Lexxus'у,потом он вам выложит сюда

Спасибо, жду.

3245. глючик, 13 марта 2011, 17:20:58
привети к кто знает помогите мне пжл. сумма цифр двузначного числа равна 5. если к каждой цифре добавить по 2, то получится число, которое на 1 меньше удвоенного первоначального. найдите это число. желательно с решением=***

3246. LADY))), 13 марта 2011, 17:39:28
ну, допустим первоначальным числом было число 23, т.к. 2+3=5. Затем мы к каждой цифре добавляем 2 (т.е. 2+2=4 и 3+2=5) получилось число 45. Это число как раз-таки и меньше удвоенного первоначального, которое =46. Ответ: 23.

3247. глючик, 13 марта 2011, 17:43:18
а можно мне объяснить с помощью букв А и В, и решение через систему, извини но мне так будет легче понять, просто таких чисел много и мне не удастся сидеть и годать. заранее спасибо

3248. Lexxus, 13 марта 2011, 17:44:19
3245. глючик, 13 марта 2011, 17:20:58
сумма цифр двузначного числа равна 5. если к каждой цифре добавить по 2, то получится число, которое на 1 меньше удвоенного первоначального.

Фраза "если к каждой цифре добавить по 2, то получится число, которое на 1 меньше удвоенного первоначального" математическим языком записывается так:

10*(x+2)+(y+2) = 2*(10x+y)-1,
где x и y - это первая и вторая цифра искомого числа, а 10x+y - само это число.

Приводим подобные и получаем
10x+y = 23

Ответ: 23

3249. глючик, 13 марта 2011, 17:54:15
спасибо большое , вот еше одно напишите мне только план работы а решить попробую сама : сумма числителя и знаменателя дроби равна 13. если числитель дроби увеличить на 7, а знаменатель уменьшить на 7. то получится дробь, обратная даннной. найдите эту дробь=**

3250. LADY))), 13 марта 2011, 17:58:33
нет...не могу объяснить с помощью А и В...я не знаю как...т.к. это задание я сделала методом подбора и при чем всё получилось с первого раза)))

3251. глючик, 13 марта 2011, 18:00:47
ммм я ваще не умею подбором у меня время много занимает, кстате это задание В12

3252. LADY))), 13 марта 2011, 18:06:14
ну насчет подбора это как повезет...может сразу получиться а может нет...я так делаю тока тогда когда по формулам или как то по другому не получается...и в конце концов ответы правильные)))

3253. глючик, 13 марта 2011, 18:09:41
вот еше одно напишите мне только план работы а решить попробую сама : сумма числителя и знаменателя дроби равна 13. если числитель дроби увеличить на 7, а знаменатель уменьшить на 7. то получится дробь, обратная даннной. найдите эту дробь=**

3254. LADY))), 13 марта 2011, 18:16:05
опять же подбором...и получается 3/10

3255. глючик, 13 марта 2011, 18:36:12
а можешь дать решие ну то как ты решала:))

3256. LADY))), 13 марта 2011, 18:52:41
ну я написала просто все возможные варианты когда сумма получается 13, ну к примеру 1 и 12, потом 2 и 11, 3 и 10 ну и смотрела че подходит и подошло 3 и 10)))

3257. глючик, 13 марта 2011, 19:21:26
ммм спасибо большое =** а ты не с можешь помочь с еще ондной такого же типа??? пжл))

3258. глючик, 13 марта 2011, 19:38:20
найдите двузначное число, которое в 3 три раза больше произведения его цифр.если перставить цифры этого числа в обрасном порядке. то отношение полученного числа к данному будет равно 7/4

3259. глючик, 13 марта 2011, 19:55:55
ну хоть кто то здесь есть???

3260. LADY))), 13 марта 2011, 20:40:37
да...уже есть))) я! щас подумаю над твоим заданием...)))

3261. LADY))), 13 марта 2011, 20:58:19
это число 24!

3262. катрин), 15 марта 2011, 20:06:31
помогите пожалуйста решить(((
Решите систему уравнений 16 в степени lgх - 2*4 в степени lgх +1=0
6 в степени у+10 cos -1=o

3263. Lexxus, 15 марта 2011, 20:20:34
3262. катрин), 15 марта 2011, 20:06:31
Решите систему уравнений 16 в степени lgх - 2*4 в степени lgх +1=0
6 в степени у+10 cos -1=o

Задолбался ребусы решать.
Первое уравнение, видимо,
16^lg(x) - 2*4^lg(x) + 1 = 0
А вот второе
y+10 косинус чего минус что равно чего?

Скобками пользоваться религия запрещает?

3264. катрин), 15 марта 2011, 22:10:43
второе уравнение
6^у+10 cosх-1=0
извини((

3265. natasha, 16 марта 2011, 07:40:25
В треугольнике АВС угол С равен 90' АВ=18, cosA ,5 найдите АС

3266. Lexxus, 16 марта 2011, 10:44:04
3264. катрин), 15 марта 2011, 22:10:43
второе уравнение
6^у+10 cosх-1=0
извини((

Вот опять. Что под знаком косинуса? x или (x-1)? Допустим, ты имеешь ввиду:
6^y + 10*cos(x) - 1 = 0

Решаем систему:
16^lg(x) - 2*4^lg(x) + 1 = 0
6^y + 10*cos(x) - 1 = 0

В первом уравнении делаем замену t = 4^lg(x)
t^2 - 2*t + 1 = 0
(t - 1)^2 = 0
t = 1
4^lg(x) = 1 => lg(x) = 0 => x = 1

Решаем втрое уравнение.
6^y = 1 - 10*cos(1)

Вообще-то, можно считать, что 1 примерно равен pi/3, т. е. cos(1) примерно равен 1/2. Поэтому правая часть уравнения меньше нуля, и оно неразрешимо относительно y.
Впрочем, даже если изначально имелось ввиду
6^y + 10*cos(x - 1) = 0,
то оно и в этом случае решений не имеет.

3267. Василиса, 16 марта 2011, 17:16:31
3123. Lexxus, 28 февраля 2011, 19:22:18
[quote]3121. baart, 28 февраля 2011, 18:31:54
В треугольнике АВС АС=СВ=10СМ угол А =30градусов
ВК - перпендикуляр к плоскости треугольника и равен 5корней из 6 см
Найдите расстояние от точки К до АС

Проведем высоту BH к прямой AC.
Расстояние от K до AC будет равно длине отрезка KH.

KH можно найти по теореме Пифагора из прямоугольного треугольника KBH, зная BH.
BH можно найти из прямоугольного треугольника ABH, зная AB и угол A.
AB = AC*2*cos(угла A) = 10*2*sqrt(3)/2 = 10*sqrt(3)
BH = AB*sin(угла A) = 10*sqrt(3)*1/2 = 5*sqrt(3)
KH = sqrt(BH^2+BK^2) = sqrt(25*3+25*6) = sqrt(225) = 15

Слушай а как такая формула АВ получилась??


3268. таня, 16 марта 2011, 22:35:38
реши задачу плизз
внешний угол треугольника равен 39 градусов.докажите что треугольник тупой

3269. Lexxus, 16 марта 2011, 22:39:23
3268. таня, 16 марта 2011, 22:35:38
реши задачу плизз
внешний угол треугольника равен 39 градусов.докажите что треугольник тупой

Тупой бывает вопрос или, скажем, человек. А треугольник бывает тупоугольным ;)

3270. Василиса, 16 марта 2011, 22:48:14
Lex, вопрос жизни и смерти :DD
Откуда такая дивная формула:
BH можно найти из прямоугольного треугольника ABH, зная AB и угол A.
AB = AC*2*cos(угла A) = 10*2*sqrt(3)/2 = 10*sqrt(3)
BH = AB*sin(угла A) = 10*sqrt(3)*1/2 = 5*sqrt(3)???

3271. катрин), 17 марта 2011, 08:01:00
Lexxus, спасибо большое))))

3272. Илья, 17 марта 2011, 18:14:00
Всем привет!
В правильной шестиугольной призме A...F1,все ребра которой равны 1,найдите косинус угла между прямыми AB1 и BE1.

срочно пожалуйста решите,если можно с черчежом и по яснее,спасибо заранее!

3273. ilo, 17 марта 2011, 19:25:26
3272. Илья, 17 марта 2011, 18:14:00
Всем привет!
В правильной шестиугольной призме A...F1,все ребра которой равны 1,найдите косинус угла между прямыми AB1 и BE1.

срочно пожалуйста решите,если можно с черчежом и по яснее,спасибо заранее!

Посмотри здесь http://depositfiles.com/files/rn4m9029p

3274. Илья, 17 марта 2011, 19:35:33
ilo спасибо,но я решал по другому и там застрял,я через вектора находил!

3275. Марина, 18 марта 2011, 18:47:04
Помогите решить задачу,пожалуйста. Дано треугольник АВС,АА1 иСС1-медианы,АА1пересекает СС1в точке.М принадлежит АВ,NпринадлежитВС,MNIIAC,АС=12.Найти MN

3276. ТАНЯ, 19 марта 2011, 07:32:52
помогите, пожалуйста решить, срочно!
найти наибольшее значение функции y=9х-8sin x+7 на отрезке [-пи/2;0]

3277. Елена, 19 марта 2011, 15:41:48
Пожалуйста, помогите.
Площадь основания и боковых граней прямой треугольной призмы равны соответственно 480; 312, 200 и 128 см квадратных. Найдите высоту призмы.

3278. Вася, 20 марта 2011, 11:21:35
помогите

3279. Айгуль, 20 марта 2011, 17:32:50
Апофема правильной четырехугольной пирамиды равна 2а, высота пирамиды а корень из 2. Найдите: а)сторону основания пирамиды; б) угол между боковой гранью и основанием; в) площадь поверхности пирамиды; г) расстояние от центра основания пирамиды до плоскости боковой грани.
Помогите пожалуйста. Люди добрые. Спасибо заранее.

3280. коокса1, 20 марта 2011, 19:22:33
помогите решить задание, кто-нибудь, заранее благодарна!!!
Прямая У=3Х+1 является касательной к графику функции ах в квадрате +2х+3 . Найдите a.

3281. Nastya, 20 марта 2011, 20:09:31
Пожалуйста, помогите решить систему (из ЕГЭ С1):
(2* sinx^2 + 3 sin x + 1) / корень из (-y) = 0;
y = -cos x

3282. ilo, 21 марта 2011, 14:13:17
3281. Nastya, 20 марта 2011, 20:09:31
Пожалуйста, помогите решить систему (из ЕГЭ С1):
(2* sinx^2 + 3 sin x + 1) / корень из (-y) = 0;
y = -cos x

D(f): y<0; -pi/2+2pin<x<pi/2+2pin; 2*sin^2(x)+3sinx+1=0 , если sinx=-1 не из D(f); sinx=-1/2;x=-pi/6+2pin; y=-sqrt3/2;Ответ: (-pi/6+2pin:-sqrt3/2)

3283. Ecology, 21 марта 2011, 16:14:05
Клиент хочет арендовать автомобиль на сутки для поездки протяженностью 700 км. В таблице приведены характеристики трех автомобилей и стоимость их аренды. Помимо аренды клиент обязан оплатить топливо для автомобиля на всю поездку. Какую сумму в рублях заплатит клиент за аренду и топливо, если выберет самый дешевый вариант?

Автомобиль Топливо Расход топлива на 100 км Арендная плата за 1 сутки
1. Дизельное 7 3700
2. Бензин 10 3200
3. Газ 14 3200

Цена дизельного топлива 19 р. за литр, бензина 22 р. за литр, газа 14 р. за литр.
Ответ 3298 воспринимается как неправильный. почему???

3284. Lexxus, 21 марта 2011, 16:26:14
3283. Ecology, 21 марта 2011, 16:14:05
Ответ 3298 воспринимается как неправильный. почему???

Потому что как правильный воспринимается только правильный ответ, а это - неправильный.

3285. ^-^, 21 марта 2011, 17:03:18
в основании прямой призмы АВСА1В1С1,лежит прямоугольный треугольник АВС ,угол С=90 градусов,АС=4см,ВС=3см,.Через сторону АС и вершину В1 проведена плоскость,уголВ1АС=60.Найти площадь боковой поверхности призмы.

пожалуйста, помогите решить

3286. Елена, 21 марта 2011, 17:15:32
Пожалуйста помогите решить задачи:в треугогльнике АВС угол С равен 90градусов,АВ 130,АС 50.Найдите тангенс угла А.В треугольнике АВС угол С равен 90 градусов,АВ 15,ВС 3 кв.корень из 21.Найдите косинус угла А.ПОЖАЛУЙСТА ПОМОГИТЕ РЕШИТЬ!!!

3287. Lexxus, 21 марта 2011, 17:38:57
3285. ^-^, 21 марта 2011, 17:03:18
в основании прямой призмы АВСА1В1С1,лежит прямоугольный треугольник АВС ,угол С=90 градусов,АС=4см,ВС=3см,.Через сторону АС и вершину В1 проведена плоскость,уголВ1АС=60.Найти площадь боковой поверхности призмы.

1. По теореме Пифагора, AB = 5. Чтобы найти площадь боковой поверхности, которая равна произведению периметра основания на высоту призмы, нам не хватает только высоты.
2. Из прямоугольного треугольника ACB1: AB1 = AC/cos(B1AC) = 2*AC = 8.
3. Зная AB и AB1, по теореме Пифагора находим BB1: sqrt(8^2-5^2) = sqrt(39).
4. Площадь боковой поверхности призмы
S = (AB+BC+AC)*BB1 = 12*sqrt(39)

Ответ: 12*sqrt(39)

3288. ilo, 21 марта 2011, 18:28:41
3286. Елена, 21 марта 2011, 17:15:32
Пожалуйста помогите решить задачи:в треугогльнике АВС угол С равен 90градусов,АВ 130,АС 50.Найдите тангенс угла А.В треугольнике АВС угол С равен 90 градусов,АВ 15,ВС 3 кв.корень из 21.Найдите косинус угла А.ПОЖАЛУЙСТА ПОМОГИТЕ РЕШИТЬ!!!

1) cosA=5/13(по определению);sinA=12/13(по основному тригонометрическому тождеству)tgA=2,4(по определению);
2)sinA найти по определению. cosA=0,4 по основному тригонометрическому тождеству

3289. глючик, 21 марта 2011, 19:23:28
решите пожалуйста: сумма цифр двузначного числа равна 8. Если к каждой цифре прибавить по 2, то получитса число, которое на 5 больше удвоенного первоначального. найдите это число.

3290. Колян, 21 марта 2011, 20:38:30
В треугольнике АВС АС=ВС, АВ=18, cosА=3/5. Найдите высоту СН.
Помогите пожалуйста решить

3291. Кристина, 21 марта 2011, 21:35:40
Помогите посчитать,нет калькуратора с логарифмами 10*log(3,6)25

3292. Леонид , 21 марта 2011, 22:22:43
непонятно, что написано

3293. ilo, 22 марта 2011, 08:48:32
3290. Колян, 21 марта 2011, 20:38:30
В треугольнике АВС АС=ВС, АВ=18, cosА=3/5. Найдите высоту СН.
Помогите пожалуйста решить

sinB=cosA=CH/CB;CB по теореме Пифагора; CH=CB*sinB

3294. VladislavOfficiaL, 22 марта 2011, 13:25:36
3290. Колян, 21 марта 2011, 20:38:30
В треугольнике АВС АС=ВС, АВ=18, cosА=3/5. Найдите высоту СН.
Помогите пожалуйста решить

В равнобедренном треугольнике высота делит сторону АВ/2 ,тогда АН=9.
cosA=СН/АН=>9/СН=3/5 => СН=5*9/3
Ответ:15

3295. Кристина, 22 марта 2011, 15:04:19
Помогите посчитать.т.е. Отсюда: &#916;t = 10*log(&#947;}(&#965;2/&#965;1) = 10*log(3,6)25 (т.е. 10 умножить на логарифм двадцати пяти по основанию 3,6)

3296. Kристина, 22 марта 2011, 15:08:23
Посчить помогите.Отсюда: &#916;t = 10*log(&#947;}(&#965;2/&#965;1) = 10*log(3,6)25 (т.е. 10 умножить на логарифм двадцати пяти по основанию 3,6)

3297. Кристина, 22 марта 2011, 15:12:17
=10*log(3,6)25 (т.е. 10 умножить на логарифм двадцати пяти по основанию 3,6) нужен посчитанный ответ этого равенства

3298. VladislavOfficiaL, 22 марта 2011, 15:58:26
3297. Кристина, 22 марта 2011, 15:12:17
=10*log(3,6)25 (т.е. 10 умножить на логарифм двадцати пяти по основанию 3,6) нужен посчитанный ответ этого равенства


Ты условие запиши правильно.На моем мониторе абракадабра:))

3299. Викуля, 22 марта 2011, 18:27:58
log2(6 + x) = 4 - объясните подробное решение, пожалуйста!!

3300. кристулик, 22 марта 2011, 19:53:36
решите уравнение 1-cosx/2=tgx/4

3301. VladislavOfficiaL, 22 марта 2011, 19:58:25
3299. Викуля, 22 марта 2011, 18:27:58
log2(6 + x) = 4 - объясните подробное решение, пожалуйста!!


(6+x)=2^4 => (6+x)=16
x=16-6
x=10
Проверка Log_2(6+10)=4=>Log_22^4=4
4=4

3302. кристулик, 22 марта 2011, 20:11:30
вычислите 2sin5xcos3x-sim8x , если sinx + cosx = корень из 0,6

3303. кристулик, 22 марта 2011, 20:14:14
вычислите 2sin5xcos3x-sin8x , если sinx + cosx = корень из 0,6

3304. Ден, 22 марта 2011, 21:10:10
Помогите с задачей плиз))
В основании прямой призмы ABCA1B1C1 лежит равнобедренный треугольник ABC, у которого основание BC равно 3. Боковая поверхность призмы равна 32. Найдите площадь сечения призмы плоскости проходящей через CB1 параллельно высоте основания AD. Расстояние от A до плоскости сечения равно 6/5.
Пожалуйста помогите)) заранее благодарен))

3305. Лена, 22 марта 2011, 21:30:10
Помогите пожалуйста!!!! Умоляю !!!!! Правильная 3 угольная пирамида основание =2 корень 3, высота=2. Наити угол наклона бокового ребра к плоскости основания. Спасибо.

3306. Lexxus, 22 марта 2011, 23:17:01
3304. Ден, 22 марта 2011, 21:10:10
В основании прямой призмы ABCA1B1C1 лежит равнобедренный треугольник ABC, у которого основание BC равно 3. Боковая поверхность призмы равна 32. Найдите площадь сечения призмы плоскости проходящей через CB1 параллельно высоте основания AD. Расстояние от A до плоскости сечения равно 6/5.

Эту задачу составлял изувер-извращенец.



1. Разберёмся с сечением. Поскольку оно параллельно AD, то его плоскости принадлежит прямая LK, параллельная AD и проходящая через середину CB1. Отрезок LK равен AD, а поскольку K - середина CB1, то L - середина AA1.
2. Раз L - середина AA1, то LC = LB1, а значит, треугольник CLB1 - равнобедренный, и его площадь, которую нам надо найти, равна CB1*LK/2.

3. Пусть x = AD = LK, у = AA1 = BB1 = CC1.
Тогда из условий, что площадь боковой поверхности призмы равна 32, а BC=3, получаем
(AB+BC+AC)*AA1 = 32
y*(2*sqrt((3/2)^2+x^2)+3) = 32, или

y*(3+sqrt(9+4*x^2)) = 32    (1)

4. Расстояние AH от точки A до плоскости CLB1 равно расстоянию от A до прямой LM, параллельной CB1 и проходящей через точку L.
LAM - прямоугольный треугольник, где AM = DC = 3/2, AL = y/2.
Его площадь равна
1/2*AM*AL = 1/2*LM*AH.
Отсюда получаем
1/2*3/2*y/2 = 1/2*6/5*sqrt((3/2)^2+(y/2)^2)

y = 4/5*sqrt(9+y^2)    (2)

5. Из уравнения (2) находим, что высота призмы y = 4.
6. Из уравнения (1), зная y, находим, что высота основания призмы x = 2.

7. Площадь треугольника CLB1

S = x*sqrt(3^2+y^2)/2 = 2*sqrt(9+16)/2 = 5

Ответ: 5

3307. ilo, 23 марта 2011, 09:18:55
3305. Лена, 22 марта 2011, 21:30:10
Помогите пожалуйста!!!! Умоляю !!!!! Правильная 3 угольная пирамида основание =2 корень 3, высота=2. Наити угол наклона бокового ребра к плоскости основания. Спасибо.

SABC-правильная треугольная пирамида; Искомый угол,скажем.между SB и BH, BH-высота основания,находится sqrt3*a/2=3;Рассмотрим треугольник SOB,О- основание высоты; BO=2/3BH=2;Прямоугольный треугольник SOB-равнобедренный,искомый угол 45 градусов.

3308. Кристина, 23 марта 2011, 09:19:42
это условие моей задачи,просто нужно посчитать ответ который получится:

дельта t=10*log(y)(v2/v1)=10*log(3.6)25=

(т.е. 10 умножить на логарифм двадцати пяти по основанию 3,6)

3309. ilo, 23 марта 2011, 12:26:25
3308. Кристина, 23 марта 2011, 09:19:42
это условие моей задачи,просто нужно посчитать ответ который получится:

дельта t=10*log(y)(v2/v1)=10*log(3.6)25=

(т.е. 10 умножить на логарифм двадцати пяти по основанию 3,6)

В Mathcad получается 22,8.

3310. ilo, 23 марта 2011, 12:52:50
3303. кристулик, 22 марта 2011, 20:14:14
вычислите 2sin5xcos3x-sin8x , если sinx + cosx = корень из 0,6

2sin5xcos3x-sin8x=2*1/2(sin(5x+3x)+sin(5x-3x))-sin8x=sin2x;Во втором рав-ве возводишь в квадрат обе части: sin^2(x)+2sinx*cosx+cos^2(x)=0,6; 1+sin2x=0,6;sin2x=-0,4;

3311. Я Геля)), 23 марта 2011, 18:02:12
дан треугольник ABC, угол B=90 градусов,угол А=альфа,точка Д середина гипотенузы, С1 симметрична точке С относительно прямой ВД,найти угол АС1В.
Помогите решить,пожалуйста,срочно!!

3312. Collins, 23 марта 2011, 18:42:09
В правильной треугольной призме АВСА1В1С1 известны ребра АВ=6 квадратный корень из 3, ВВ1= 5. Точка М-середина ребра В1С1, а точка Т- середина А1М. Найдите угол между плоскостью ВСТ и прямой АТ


Помогите решить пожалуйста((((((((

3313. Ден, 23 марта 2011, 18:56:46
Lexxus)) спасибо огромное))

3314. Кристина, 23 марта 2011, 21:04:52
Спасибо большое В Mathcad !!!

3315. Kimik, 24 марта 2011, 08:44:56
Привет всем! Нужна быстрая помощь! Вот:
Теплоход проходит по течению реки до пункта назначения 465 км и после стоянки возвращается в пункт отправления . Найти скорость течению если скорjcnm теплохода в неподвижной воде равна 22км ч . Стоянка длиться 8часов. А в пункт теплоход возвращается через 52 часа после отплытия от него/

3316. maksimrussia, 24 марта 2011, 10:01:28
2950. Ксения, 9 февраля 2011, 23:24:12
диаметр ок
2950. Ксения, 9 февраля 2011, 23:24:12
диаметр ок

3317. ilo, 24 марта 2011, 22:39:55
3312. Collins, 23 марта 2011, 18:42:09
В правильной треугольной призме АВСА1В1С1 известны ребра АВ=6 квадратный корень из 3, ВВ1= 5. Точка М-середина ребра В1С1, а точка Т- середина А1М. Найдите угол между плоскостью ВСТ и прямой АТ


Помогите решить пожалуйста((((((((

Ответ:arccos19/181
Чертёж и решение http://depositfiles.com/files/conq2ytzs

3318. денис, 25 марта 2011, 07:34:58
через вершину К треугольника МКР проведена прямая МН перпендикулярная к плоскости треугольника известно что МР=15 см МК=КР=10 см МР=12см найдите расстояние от точкиН до прямой МР

3319. Леонид , 25 марта 2011, 09:11:04
3312. Collins, 23 марта 2011, 18:42:09, 3318. денис, 25 марта 2011, 07:34:58
Ребята, вы где взяли эти задачи? Ответьте, если нетрудно, назвав регион.

3320. денис, 25 марта 2011, 09:15:31
74 регион

3321. Леонид , 25 марта 2011, 09:35:58
где брали, кто давал эти задачи? Денис, прочитай свою задачу - помоему ты там с обозначениями напутал. МР и 12 и 15 ? и с перпендекуляром МН не то, коль он проводится из точки К? Смотри, пиши.

3322. денис, 25 марта 2011, 09:55:39
через вершину К треугольника МКР проведена прямая МН перпендикулярная к плоскости треугольника известно что KN=15 см МК=КР=10 см МР=12см найдите расстояние от точкиН до прямой МР

3323. Lexxus, 25 марта 2011, 11:10:42
3322. денис, 25 марта 2011, 09:55:39
через вершину К треугольника МКР проведена прямая МН перпендикулярная к плоскости треугольника известно что KH=15 см МК=КР=10 см МР=12см найдите расстояние от точки Н до прямой МР

Из точки K опустим высоту KL к MP.

Из прямоугольного треугольника KLM (KM=10, ML=MP/2=6) находим
KL = sqrt(10^2-6^2) = 8.

Искомое расстояние от точки H до MP будет равно длине отрезка HL, который мы находим из прямоугольного треугольника HKL (KH=15, KL=8)
HL = sqrt(15^2+8^2) = 17

Ответ: 17 см

3324. ilo, 25 марта 2011, 12:06:45
Lexxus,помогите,пожалуйста!
Надо решит совокупность: x^lgx>5;x^lgx<-2

3325. radik, 25 марта 2011, 12:45:22
Помогите решить:
Найдите наибольшее значение функции у=ln(x+8)^3-3х на отрезке [-7,5;0]

3326. Lexxus, 25 марта 2011, 13:09:31
3324. ilo, 25 марта 2011, 12:06:45
Надо решит совокупность: x^lgx>5;x^lgx<-2

По основному логарифмическому тождеству,
x^lg(x) = (10^lg(x))^lg(x) = 10^((lg(x))^2)
Показательная функция строго положительна, поэтому у второго неравенства решений нет.

10^((lg(x))^2) > 5
(lg(x))^2 > lg(5)

[lg(x) < -sqrt(lg(5)); lg(x) > sqrt(lg(5))]
[x < 10^(-sqrt(lg(5))); x > 10^sqrt(lg(5))]

Учитывая ОДЗ (x > 0), в ответе получаем два интервала:

(0; 10^(-sqrt(lg(5)))) и (10^sqrt(lg(5)); +бесконечность)

Для справки,
10^(-sqrt(lg(5))) ~ 0.15
10^sqrt(lg(5)) ~ 6.86
Но оценивать это необязательно, поскольку нам достаточно знать, что первое меньше второго.

3327. ilo, 25 марта 2011, 14:01:17
3326. Lexxus, 25 марта 2011, 13:09:31
[quote]3324. ilo, 25 марта 2011, 12:06:45
Надо решит совокупность: x^lgx&gt;5;x^lgx&lt;-2


Спасибо большое!

3328. ilo, 25 марта 2011, 14:10:46
3325. radik, 25 марта 2011, 12:45:22
Помогите решить:
Найдите наибольшее значение функции у=ln(x+8)^3-3х на отрезке [-7,5;0]

y'=(-3x-21)/(x+8); не существует при x=-8-не лежит на указанном отрезке; y'=0 при x=-7-принадлежит указанному отрезку.
y(-7)=21;y(-7,5)=-ln8+22.5;y(0)=3ln8;yнаиб.=y(-7)=21;

3329. andr_o, 25 марта 2011, 22:17:40
как попасть в список коллективного разума?

3330. Андрей, 26 марта 2011, 13:58:16
Люди , а то что прислал Илья, это реальный вариант ЕГЭ 2010 года?
скажите плизз....

3331. Lexxus, 26 марта 2011, 14:06:36
3330. Андрей, 26 марта 2011, 13:58:16
Люди , а то что прислал Илья, это реальный вариант ЕГЭ 2010 года?
скажите плизз....

По уровню сложности заданий вполне похож на реальный.

3332. Сусик, 26 марта 2011, 21:12:26
Да, ну и задачки=)))))

3333. Таня, 27 марта 2011, 11:57:30
Где можно найти все решения части С? Срочно нужны образцы решений.

3334. Кристина, 27 марта 2011, 17:19:39
Прошу вычислить:
М=8,314*293* (10 в -13 степени)/4,0*(10в -11)*(1-1,0*(10в-3)*1,33)=


Прошу вычислить:
R=1,38*(10в -23)*293/6*3,14*1,0*(10 в -3)*4,0*(10 в 11)=

3335. Lexxus, 27 марта 2011, 17:39:47
3333. Таня, 27 марта 2011, 11:57:30
Где можно найти все решения части С? Срочно нужны образцы решений.

Подробные решения задач ЕГЭ по математике

3336. Наташа, 28 марта 2011, 12:48:44
Помогите решить!!! Диаметр окружности основания цилиндра равен 26, образующая цилиндра равна 21. Плоскость пересекает его основания по хордам длины 24 и 10. Найдите тнгенс угла между этой плоскостью и плоскостью основания цилиндра.

3337. Lexxus, 28 марта 2011, 14:30:40
3336. Наташа, 28 марта 2011, 12:48:44
Диаметр окружности основания цилиндра равен 26, образующая цилиндра равна 21. Плоскость пересекает его основания по хордам длины 24 и 10. Найдите тнгенс угла между этой плоскостью и плоскостью основания цилиндра.


Пусть AB=10 и C1D1 = 24 - хорды, по которым сечение пересекает основания цилиндра. Плоскости оснований параллельны, значит, AB и C1D1 тоже параллельны.

Опустив перпендикуляры из точек C1 и D1 к плоскости OAB, получим отрезок CD, равный C1D1. Пусть K, L и L1 - середины хорд AB, CD и C1D1 соответственно.

Угол между плоскостью сечения и плоскостью основания цилиндра будет равен углу L1KL. Его тангенс мы найдём из прямоугольного треугольника L1LK: tg(L1KL) = LL1/LK.

LL1 = образующей цилиндра = 21
LK = LO+OK.

Из прямоугольного треугольника CLO:
LO = sqrt(CO^2-CL^2) = sqrt(13^2-12^2) = 5

Из прямоугольного треугольника AKO:
OK = sqrt(AO^2-AK^2) = sqrt(13^2-5^2) = 12

LK = 5+12 = 17

Ответ: tg(L1KL) = LL1/LK = 21/17

3338. VladislavOfficiaL, 28 марта 2011, 15:00:50
Помогите решить уравнения.Чета я запутался.
1.(sqrt 4-x^2-sqrt x^2+2)*sqrt 4-x=0
2.(sqrt 16x^2+3-sqrt 15x^2+28)*sqrt 4-x=0

3339. Lexxus, 28 марта 2011, 15:05:20
3338. VladislavOfficiaL, 28 марта 2011, 15:00:50
1.(sqrt 4-x^2-sqrt x^2+2)*sqrt 4-x=0
2.(sqrt 16x^2+3-sqrt 15x^2+28)*sqrt 4-x=0

Будь другом, расставь скобочки, где начинаются и заканчиваются подкоренные выражения. Надоело ребусы решать.

3340. VladislavOfficiaL, 28 марта 2011, 15:09:52
скобкаsqrt 4-x^2-sqrt x^2+2скобка*sqrt 4-x=0
скобкаsqrt 16x^2+3-sqrt 15x^2+28скобка*sqrt 4-x=0
где есть скобка я там написал прописью:)

3341. VladislavOfficiaL, 28 марта 2011, 15:13:37
(sqrt (4-x^2)-sqrt( x^2+2))*sqrt( 4-x)=0
(sqrt (16x^2)+3-sqrt( 15x^2+28))*sqrt (4-x)=0
Вот расставил

3342. VladislavOfficiaL, 28 марта 2011, 15:15:52
Во втором расставил неправильно Вот так надо
(sqrt (16x^2+3)-sqrt( 15x^2+28))*sqrt (4-x)=0

3343. VladislavOfficiaL, 28 марта 2011, 15:22:14
3339. Lexxus, 28 марта 2011, 15:05:20
3338. VladislavOfficiaL, 28 марта 2011, 15:00:50
1.((sqrt (4-x^2)-sqrt( x^2+2))*sqrt (4-x)=0
2.((sqrt (16x^2+3)-sqrt( 15x^2+28))*sqrt( 4-x)=0

Будь другом, расставь скобочки, где начинаются и заканчиваются подкоренные выражения. Надоело ребусы решать.


Исправил

3344. Lexxus, 28 марта 2011, 15:38:07
3341. VladislavOfficiaL, 28 марта 2011, 15:13:37
(sqrt (4-x^2)-sqrt( x^2+2))*sqrt( 4-x)=0

ОДЗ:
(1) 4-x >= 0, x<=4
(2) 4-x^2 >=0, -2 <= x <= 2
(1)+(2) --> -2 <= x <= 2

Решаем:
1) sqrt(4-x) = 0
x = 4, не входит в ОДЗ.

2) sqrt(4 - x^2) = sqrt(x^2 + 2)
4 - x^2 = x^2 + 2
2*x^2 = 2
x^2 = 1
x = -1, x = 1

Ответ: x = -1, x = 1

Второе решается тем же манером.

3345. VladislavOfficiaL, 28 марта 2011, 15:56:33
Спасибо,теперь понятно.А то я решил нашел такие же значения (x)а вот с ОДЗ не совсем разобрался.

3346. VladislavOfficiaL, 28 марта 2011, 16:02:12
3344. Lexxus, 28 марта 2011, 15:38:07
3341. VladislavOfficiaL, 28 марта 2011, 15:13:37
(sqrt (4-x^2)-sqrt( x^2+2))*sqrt( 4-x)=0

ОДЗ:
(1) 4-x &gt;= 0, x&lt;=4
(2) 4-x^2 &gt;=0, -2 &lt;= x &lt;= 2
(1)+(2) --&gt; -2 &lt;= x &lt;= 2

Решаем:
1) sqrt(4-x) = 0
x = 4, не входит в ОДЗ.

2) sqrt(4 - x^2) = sqrt(x^2 + 2)
4 - x^2 = x^2 + 2
2*x^2 = 2
x^2 = 1
x = -1, x = 1

Ответ: x = -1, x = 1

Второе решается тем же манером.


а ОДЗ (X^2+2) >=0 не нужно находить?

3347. Lexxus, 28 марта 2011, 16:09:24
3346. VladislavOfficiaL, 28 марта 2011, 16:02:12
а ОДЗ (X^2+2) >=0 не нужно находить?

А зачем? x^2+2 всегда больше нуля.

3348. VladislavOfficiaL, 29 марта 2011, 15:21:46
Помогите решить
Lg(x^2+21x)+tgpi*x/2=2+tgpi*x/2

3349. ilo, 29 марта 2011, 17:30:53
3348. VladislavOfficiaL, 29 марта 2011, 15:21:46
Помогите решить
Lg(x^2+21x)+tgpi*x/2=2+tgpi*x/2

Прямо "последствия" Фо-мы-1...Типа x не равен (-1)^n*(2n-1) и вычёркивать?

3350. Кристина, 29 марта 2011, 19:42:24
Помогите решить!
В правильной шестиугольной призме A..F1, все ребра которой равны 1. найдите угол между прямыми ABD1
рис.http://otvet.mail.ru/question/55593520/

и вот эту
В правильной шестиугольной призме A..F1, все ребра которой равны 1. найдите угол ACD1
рис.http://otvet.mail.ru/question/55593417/

3351. кристинка, 29 марта 2011, 19:44:31
ПОМОГИТЕ ПОЖАЛУЙСТА((( первая труба наполняет бак объёмом 570 л.,а вторая труба-бак объёмом 530 л.Известно,что первый пропускает в минуту на 4 л.воды больше,чем второй.сколько литров воды в минуту пропускает вторая труба,если баки наполнены за одно и то же время?

3352. VladislavOfficiaL, 30 марта 2011, 03:04:03
3351. кристинка, 29 марта 2011, 19:44:31
ПОМОГИТЕ ПОЖАЛУЙСТА((( первая труба наполняет бак объёмом 570 л.,а вторая труба-бак объёмом 530 л.Известно,что первый пропускает в минуту на 4 л.воды больше,чем второй.сколько литров воды в минуту пропускает вторая труба,если баки наполнены за одно и то же время?

За x берем вторую трубу
(x+4) первая
Составляем уравнение:
570/(x+4)=530/x
570x=530x+530*4
40*x=53*40
x=53 это вторая труба
x+4=57 это первая труба.

3353. ilo, 30 марта 2011, 10:59:51
3350. Кристина, 29 марта 2011, 19:42:24
Помогите решить!
В правильной шестиугольной призме A..F1, все ребра которой равны 1. найдите угол между прямыми ABD1
рис.http://otvet.mail.ru/question/55593520/
здесь перечитай условие

и вот эту
В правильной шестиугольной призме A..F1, все ребра которой равны 1. найдите угол ACD1
рис.http://otvet.mail.ru/question/55593417/

а вот эту:
CD_1=sqrt2-можно найти по т.Пифагора или как диагональ квадрата со стороной 1.AC=sqrt3 можно найти из треугольника АВС как AC^2=BC^2+AB^2-2BC*ABcos120=1+1-2(-1/2)=3;AD_1=2 по т.Пиф. из ADD_1;
По теореме косинусов:AD_1^2=AC^2+CD_1^2-2AC*CD_1*cosACD_1;
4=3+2-2sqrt6cosACD_1;cos ACD_1=1/2sqrt6=sqrt6/12;ACD_1=arccos(sqrt6/12)

3354. Кристина, 30 марта 2011, 12:47:23
вычислите,плиз
8,314*293* (10 в -13 степени)/4,0*(10в -11)*(1-1,0*(10в-3)*1,33)=

3355. Леонид , 30 марта 2011, 17:53:26
что значит "найдите угол между прямыми АВД1" - в записи №3350 ?
А запись № 3354?

3356. Виктор, 30 марта 2011, 18:45:01
Здравствуйте! Помогите пожалуйста решить задачу.
В треугольнике ABC угол C равен 90°, cos A=0.55. Найдите sin B.

3357. maria1995-2012, 30 марта 2011, 20:46:06
С3
туплю не могу, понимаю, что легко, но один корень не получается(((
sqrt((2x-1)^4-(2x-1)^2)+(2x-1)^2>=0
заранее спасибо

3358. Виктория, 30 марта 2011, 20:48:56
Точка M равноудалена от всех вершин равнобедренного прямоугольного треугольника ABC ( угол C = 90 градусов), AC = BC = 4 см. Расстояние от точки M до плоскости треугольника равно 2 sqrt (3) см.
а) Докажите, что плоскость AMB перпендикулярна плоскости ABC.
б) Найдите угол между прямой MC и плоскостью ABC.
в) Найдите угол между МС и плоскостью АВС.
г) Найдите расстояние от точки Е (середины стороны АВ) до плоскости ВМС.

Заранее огромное мпасибо)) нужно оч срочно, завтра уже сдавать!!!!!

3359. ilo, 31 марта 2011, 10:00:38
3357. maria1995-2012, 30 марта 2011, 20:46:06
С3
туплю не могу, понимаю, что легко, но один корень не получается(((
sqrt((2x-1)^4-(2x-1)^2)+(2x-1)^2&gt;=0
заранее спасибо

sqrt((2x-1)^4-(2x-1)^2)>=-(2x-1)^2;
левая часть положительна или равна 0 везде в D(f):x=1/2;совокупность нер-в x<=0;x>=1;
правая часть равна 0 при x=1/2 и отрицательна при любых;x=1/2-решение неравенства,
ну и кроме того положительная левая больше отрицательной правой везде в D(f)
Ответ:x=1/2;x<=0;x>=1

3360. Илья, 31 марта 2011, 14:26:06
всем привет,помогите решить задачу!
Диагональ осего сечерия равна sqrt61 см,радиус основания равен 3,найдите высоту цилиндра.
Ответы:
1)sqrt51
2)12 см
3)5см
4)другой ответ
Пожалуйста распишите как решали!и если можно рисунок!

3361. Илья, 31 марта 2011, 14:36:58
у меня получилось ответ:5

3362. ilo, 31 марта 2011, 14:53:27
3360. Илья, 31 марта 2011, 14:26:06
всем привет,помогите решить задачу!
Диагональ осего сечерия равна sqrt61 см,радиус основания равен 3,найдите высоту цилиндра.
Ответы:
1)sqrt51
2)12 см
3)5см
4)другой ответ
Пожалуйста распишите как решали!и если можно рисунок!

Осевое сечение-прямоугольник со сторонами 3+3=6 и h .Рассмотри прямоугольный треугольник со сторонами sqrt61,6 и h; h=sqrt(61-6^2)=5

3363. Илья, 31 марта 2011, 15:06:48
так у меня так же,спасибо ilo!!
Образующая конуса наклонена к плоскости основания под углом 30 градусов и равна 8 см.Найдите площадь осего сечения конуса.
Ответы:8sqrt3
16sqrt3
4sqrt3
Др.ответ

3364. Илья, 31 марта 2011, 15:24:43
Я решил,получилось 16sqrt3,ilo реши,и ответ просто скажи,расписывать не надо!

3365. Илья, 31 марта 2011, 16:12:22
решите пожалуйста задачу
Радиус основания конуса равен 10 см,а высота 15 см.Найдите площадь сечения конуса плоскостью паралл.основанию находящейся на расстоянии 2 см от его вершины

3366. io, 31 марта 2011, 18:44:41
3364. Илья, 31 марта 2011, 15:24:43
Я решил,получилось 16sqrt3,ilo реши,и ответ просто скажи,расписывать не надо!
16sqrt3-верно
3365. Илья, 31 марта 2011, 16:12:22
решите пожалуйста задачу
Радиус основания конуса равен 10 см,а высота 15 см.Найдите площадь сечения конуса плоскостью паралл.основанию находящейся на расстоянии 2 см от его вершины

треугольники подобны -к=15/2 =7,5-во столько раз высота большего треугольника больше меньшего,r соответственно тоже10:15/2=4/3.Площадь круга 16/9pi

3367. andr_o, 31 марта 2011, 20:32:26
задание В4 (19401) правильный ответ 0,6 он и являтся самым популярным, а стоит 0,8 - вторая строчка. ПОЧЕМУ?

3368. Olenyka, 31 марта 2011, 23:14:36
Помогите - В 12:В первый год процент безработных снизился, во второй год в 2.5 раза больше, после 2-х лет % безработицы уменьшился на 60%. Какой % снижения безработицы за 1-й год?

3369. XoxoL, 1 апреля 2011, 11:49:43
Напишите подробно как решать задачи типа В4 И В5 пожалуйста!

3370. мушвиг, 1 апреля 2011, 15:02:06
2872. Татьяна, 1 февраля 2011, 10:32:46
Помогите,пожалуйста.
в треугольнике АВС угол С равен 90 градусов, ВС=6,АС=10. Найдите синус В.
В ответе 0,8.
а с моим решением не сходиться.(




дура попробуй решить по другому!!!!

3371. ЛяйсанКа, 1 апреля 2011, 20:23:40
Здраствуйте, помогите пж решить:

1)Окружность S проходит через вершину C прямого угла и пересекает его стороны в точках удаленных от вершины C на расстояние 14 и 48.Найти радиус окружности вписанной в данный угол и косающей окружности S.

2)Дано:SABCDEF-правильная пирамида
AB=1
SA=2
М-середина SC
Найти:SF,^BM

Просьба: расписать решение

3372. ilo, 1 апреля 2011, 21:34:45
3367. andr_o, 31 марта 2011, 20:32:26
задание В4 (19401) правильный ответ 0,6 он и являтся самым популярным, а стоит 0,8 - вторая строчка. ПОЧЕМУ?

Популярные не значит правильные.Правильный-0,8

3373. VladislavOfficiaL, 2 апреля 2011, 10:05:03
3370. мушвиг, 1 апреля 2011, 15:02:06
2872. Татьяна, 1 февраля 2011, 10:32:46
Помогите,пожалуйста.
в треугольнике АВС угол С равен 90 градусов, ВС=6,АС=10. Найдите синус В.
В ответе 0,8.
а с моим решением не сходиться.(




дура попробуй решить по другому!!!!


Сам попробуй с этими параметрами решить:))

3374. Илья, 2 апреля 2011, 19:00:43
Всем привет! помогите решить задачу,я не понимаю как их решать,поясните плиз!

Объем данного правильного тетраэдра равен 4 см3. Найдите объем правильного тетраэдра, ребро которого в 3 раза больше ребра данного тетраэдра. Ответ дайте в см3.

3375. Илья, 2 апреля 2011, 19:55:52
помогите плиз! не могу составить уравнение(
Моторная лодка прошла против течения 24 км и вернулась обратно, затратив на обратный путь на 20 мин меньше, чем при движении против течения. Найдите скорость (в км/ч) лодки в неподвижной воде, если скорость течения равна 3 км/ч.

3376. ilo, 2 апреля 2011, 20:17:51
3374. Илья, 2 апреля 2011, 19:00:43
Всем привет! помогите решить задачу,я не понимаю как их решать,поясните плиз!

Объем данного правильного тетраэдра равен 4 см3. Найдите объем правильного тетраэдра, ребро которого в 3 раза больше ребра данного тетраэдра. Ответ дайте в см3.

Выпиши формулы,выражающие площадь правильного треугольника, высоту тетраэдра и формулу объёма тетраэтра. Если теперь увеличить ребро в 3раза.понятно будет. что площадь основания увеличится в 9,а высота в 3 раза.ОбЪём как произведение увеличится соответственно в 27 раз,т.е станет равным 108

3377. Илья, 2 апреля 2011, 20:34:33
V=а^2/12*sqrt2
h=sqrt2/3*a
S=sqrt3 /4*a^2
эти формулы?

3378. ilo, 2 апреля 2011, 20:37:50
3375. Илья, 2 апреля 2011, 19:55:52
помогите плиз! не могу составить уравнение(
Моторная лодка прошла против течения 24 км и вернулась обратно, затратив на обратный путь на 20 мин меньше, чем при движении против течения. Найдите скорость (в км/ч) лодки в неподвижной воде, если скорость течения равна 3 км/ч.

24/(x-3)-24/(x+3)=1/3;

3379. ilo, 2 апреля 2011, 20:44:38
3377. Илья, 2 апреля 2011, 20:34:33
V=а^2/12*sqrt2
h=sqrt2/3*a
S=sqrt3 /4*a^2
эти формулы?

V=1/3Sосн.*h , увеличь а в 3 раза и убедись.как изменится Sосн.,h и V.

3380. ilo, 2 апреля 2011, 20:52:14
3377. Илья, 2 апреля 2011, 20:34:33
V=а^2/12*sqrt2
h=sqrt2/3*a
S=sqrt3 /4*a^2
эти формулы?

Первая формула не может быть формулой V, а в квадрате?..

3381. Илья, 2 апреля 2011, 21:03:30
в кубе,описался

3382. ilo, 2 апреля 2011, 21:06:16
S правильного треугольника=a^2sqrt3/4; h=sqrt3*a/2;

3383. ilo, 2 апреля 2011, 21:14:47
3381. Илья, 2 апреля 2011, 21:03:30
в кубе,описался

Тогда a^3*sqrt2/12; a увеличиваешь в 3 раза, V увеличивается в 3^3 =27 раз

3384. Илья, 2 апреля 2011, 22:30:58
спасибо ilo

3385. Lexxus, 3 апреля 2011, 00:14:08
3371. ЛяйсанКа, 1 апреля 2011, 20:23:40
Окружность S проходит через вершину C прямого угла и пересекает его стороны в точках удаленных от вершины C на расстояние 14 и 48.Найти радиус окружности вписанной в данный угол и косающейся окружности S.

Во-первых, заметим, что, как и обычно бывает в C4, тут может быть два случая - вторая окружность может касаться первой как изнутри (синие линии на рисунке), так и снаружи (красная линия).



Итак, AC = 14, BC = 48, угол C - прямой. Значит, AB является диаметром первой окружности, и он равен sqrt(14^2+48^2) = 50.
Точка O, являясь центром окружности, делит AB пополам. Значит, перпендикуляры, опущенные из неё к отрезкам AC и BC, тоже делят их пополам.

Пусть O1 - центр второй окружности, а R - её радиус. Рассмотрим прямоугольный треугольник OKO1 с гипотенузой OO1 и катетами, параллельными лучам угла.

В "синем" случае:
OK = 24-R
O1K = R-7
OO1 = 25-R

Пишем теорему Пифагора:
(24-R)^2 + (R-7)^2 = (25-R)^2
Решаем, получаем два корня - 0 и 12. Нулевой случай нас не сильно интересует.

В "красном" случае всё то же самое, только OK=R-24 и, что самое важное, OO1=25+R.
И там, решая такое же уравнение, получим второй корень 112.

Ответ: 12 и 112

3386. Smoke, 4 апреля 2011, 09:27:24
Задание B8 (7651)
(показов: 351, ответов: 195)

8 ответ, там тупо посчитать.имхо, откуда 9?

3387. Lexxus, 4 апреля 2011, 10:01:57
3386. Smoke, 4 апреля 2011, 09:27:24
8 ответ, там тупо посчитать.имхо, откуда 9?

Тупо считать - тупо ;)

3388. Катрин, 4 апреля 2011, 11:11:21
Народ, помогите пожалуйста.
как решается: найдите наименьшее значение функции y=6x-6tg+11 на отрезке [-П/4;0]..

3389. ЛяйсанКа, 4 апреля 2011, 16:03:44
Lexxus спасибо огромное за решение первой задачи.

3390. Дашка, 4 апреля 2011, 18:25:31
помогите пожалуйста с задачей:
в треугольнике АВС угол С равен 90. sin A=3/4 ,АС=6корень из 7. найдите АВ

3391. Katerina, 4 апреля 2011, 20:06:18
Помогите решить, пожалуйста.
В треугольнике АВС угол С равен 90 градусов СН - высота В С=8 СН= 4 корень из 3 найдите sinA

3392. виктория, 4 апреля 2011, 21:28:57
приветик...помоги пожалуйста... даны шесть троек нужно поставить между ними знаки и скобки чтобы получилось=тысяча

3393. Lexxus, 4 апреля 2011, 21:35:33
3392. виктория, 4 апреля 2011, 21:28:57
приветик...помоги пожалуйста... даны шесть троек нужно поставить между ними знаки и скобки чтобы получилось=тысяча

333*3+3/3 = 1000

3394. dominika, 5 апреля 2011, 13:22:09
3306. Lexxus, 22 марта 2011, 23:17:01
треугольник CLB1 - равнобедренный, и его площадь, которую нам надо найти, равна CB1*LK

Площадь треугольника равна половине произведения его стороны на высоту, так что если мы еще разделим CB1*LK пополам. И того вместо десяти получаем 5.

P.S. А насчет изувера я согласна, в точку!))

3395. Lexxus, 5 апреля 2011, 14:11:12
3394. dominika, 5 апреля 2011, 13:22:09
Площадь треугольника равна половине произведения его стороны на высоту, так что если мы еще разделим CB1*LK пополам. И того вместо десяти получаем 5.

P.S. А насчет изувера я согласна, в точку!))

Да, пардон, ошибся мальца. Спасибо.

3396. Валерия, 5 апреля 2011, 19:49:25
Помогите срочно, пожалуйста, с решением заданий! Заранее спасибо!
1) y=32 tgx-20x-5п+5, на промежутке [-п/4; п/4] найти y наим.

2) y=4x-ln(x+3)^4, на промежутке [-2,5; 0] найти y наим.

3) y=(18-x)*e^18-x, найти точку максимума.

4) y=(3x^2-33+33)*e^6-x, найти точку максимума.

3397. Dobrik), 5 апреля 2011, 20:45:20
Здравствуйте!
Помогите,пожалуйста, решить систему уравненьиц:
cos5x+sin4xsinx=0
3cosy+5cosx=3

Заранее огромное спасибо!

3398. Янка, 5 апреля 2011, 21:23:05
Апофема правильной четырехугольной пирамиды равна 2а, высота пирамиды а корень из 2. Найдите: а)сторону основания пирамиды; б) угол между боковой гранью и основанием; в) площадь поверхности пирамиды; г) расстояние от центра основания пирамиды до плоскости боковой грани.
Пожалуйста помогите(

3399. Котя, 5 апреля 2011, 21:25:18
Помогите пожалуйста решить задачу:
y=2x*3-15x*2+36 x+1

3400. Котя, 5 апреля 2011, 21:39:35
Помогите пожалуйста
DX
S ___
2x+7

3401. Kattu, 5 апреля 2011, 22:20:21
Lexxus! Исправьте ответ в задании 7945. В открытом банке добавили заданий, их уже 72231. В том числе добавили прототипы. Добавьте их на свой сайт, пожалуйста!

3402. Mila6kaM, 5 апреля 2011, 22:23:33
Добрый вечер, уважаемые пользователи.
Помогите разобраться с тем, КАК решать след. задания (т.е. опишите ход решения), пожалуйста.


С1.
Найдите наименьшее значение функции
f(x) = (2x+4) в 5й степени -4(2x+4) в 4й степени при |x+2| < либо = 1.


С2.
Найдите все значения x, при каждом из которых выражения
3x в квадрате log (2+3x) по основанию_3 -6xlog корень 3й степени из 2+3x по основанию 1/3 и 3x в квадрате +2x
принимают равные значения.


С3.
Найдите все значения а, при каждом из которых неравенство
a-(2 в степени x + 2 корень кв. из 6 * 2 в степени -x - 5) / (3Sin корень из x-1 - 4 (-4 не под корнем) ) - a < либо = 0
не имеет решений.


Заранее благодарю.

3403. Валерия, 5 апреля 2011, 22:27:29
Помогите срочно, пожалуйста, с решением заданий! Заранее спасибо!

1) y=32 tgx-20x-5п+5, на промежутке [-п/4; п/4] найти y наим.

2) y=4x-ln(x+3)^4, на промежутке [-2,5; 0] найти y наим.

3) y=(18-x)*e^18-x, найти точку максимума.

4) y=(3x^2-33+33)*e^6-x, найти точку максимума.

3404. Lexxus, 5 апреля 2011, 23:59:47
3402. Mila6kaM, 5 апреля 2011, 22:23:33
С1.
Найдите наименьшее значение функции
f(x) = (2x+4) в 5й степени -4(2x+4) в 4й степени при |x+2| < либо = 1.


f(x) = (2x+4)^5 - 4*(2x+4)^4, так?

|x+2| <= 1, то есть x принадлежит отрезку [-3;-1].

Действуем также, как и всегда, находя минимальное/максимальное значение функции на отрезке.

1. Сначала найдём значения функции на границах отрезка:
f(-3) = (-2)^5 - 4*(-2)^4 = -32 - 4*16 = -32 - 64 = -96
f(-1) = 2^5 - 4*2^4 = 32 - 64 = -32

2. Теперь выясним, нет ли у функции минимумов на интересующем нас отрезке. Найдём производную:

f'(x) = 2*5*(2x+4)^4 - 2*16*(2x+4)^3 = 2*(10x+20-16)*(2x+4)^3 =
= 2*(10x+4)*(2x+4)^3

Приравняем производную к нулю и найдём точки экстремума:

2*(10x+4)*(2x+4)^3 = 0

Получается, у функции две точки экстремума: x = -2 и x = -2/5

-2/5 лежит за границами интересующего нас отрезка. А чтобы определить, какой именно экстремум (максимум или минимум) в точке -2, найдём знак производной в одной из точек по левую и по правую стороны от точки -2. Например, в точках -3 и -1:

f'(-3) = 2 * (-16) * (-2)^3 = 2*(-26)*(-8) = 416 > 0
f'(-1) = 2 * (-6) * 2^3 = -96 < 0

Значит, левее от точки -2 функция f(x) возрастает, а левее - убывает. То есть, в точке -2 - максимум. А это значит, что минимального значения f(x) достигает на границе отрезка.

Ответ: -96

3405. Lexxus, 6 апреля 2011, 00:15:00
3401. Kattu, 5 апреля 2011, 22:20:21
Lexxus! Исправьте ответ в задании 7945.

Исправил.

3401. Kattu, 5 апреля 2011, 22:20:21
В открытом банке добавили заданий, их уже 72231. В том числе добавили прототипы. Добавьте их на свой сайт, пожалуйста!

ОК, постараюсь добавить в ближайшие дни.
Спасибо за информацию.

3406. Lexxus, 6 апреля 2011, 00:36:38
3402. Mila6kaM, 5 апреля 2011, 22:23:33
С2.
Найдите все значения x, при каждом из которых выражения
3x в квадрате log (2+3x) по основанию_3 -6xlog корень 3й степени из 2+3x по основанию 1/3 и 3x в квадрате +2x
принимают равные значения.

Это не C2, а С3.
Выражения, насколько я понял, такие:
3*(x^2)*log_3(2+3x) - 6*x*log_(1/3)((2+3x)^(1/3)) и
3*x^2 + 2x

Итак, для начала разберёмся с ОДЗ:
2+3x > 0, x > -2/3

Теперь упростим первое выражение:
3*(x^2)*log_3(2+3x) - 6*x*(-1)*(1/3)*log_3(2+3x) =
= (3*x^2 + 2x)*log_3(2+3x)

Приравняем первое и второе выражения:
(3*x^2 + 2x) = (3*x^2 + 2x)*log_3(2+3x)
x*(2+3x)*(1-log_3(2+3x)) = 0

x = 0, либо
2+3x = 0, x = -2/3 (это не входит в ОДЗ), либо
log_3(2+3x) = 1
2+3x = 3
x = 1/3

Ответ: x=0, x=1/3

3407. VladislavOfficiaL, 6 апреля 2011, 15:03:25
Помогите решить неравенство.
(1-x)*Lg(x+2)<0
Составляем системы Это к первой(1-x)>0
Это к первойLg(x+2)<0
Это ко второй(1-x)<0
Это ко второй Lg(x+2)>0
Получилось две системы
Я правильно записал системы?
Что дальше?

3408. VladislavOfficiaL, 6 апреля 2011, 16:28:49
3407. VladislavOfficiaL, 6 апреля 2011, 15:03:25
Помогите решить неравенство.
(1-x)*Lg(x+2)&lt;0
Составляем системы Это к первой(1-x)&gt;0
Это к первойLg(x+2)&lt;0
Это ко второй(1-x)&lt;0
Это ко второй Lg(x+2)&gt;0
Получилось две системы
Я правильно записал системы?
Что дальше?

Решать не нужно,сам разобрался))))))))))

3409. Алексей, 6 апреля 2011, 18:00:58
Помогите решить задачку (B8 №6007)
Прямая y=7x-5 параллельна касательной к графику функции
y=x в квадрате +6x-8.
Найдите абсциссу точки касания.

3410. Марина, 6 апреля 2011, 18:03:00
в прямоугольном треугольнике АВС биссектриса прямого угла С делит гипотенузу на отрезки 3 и 4.найдите площадь треугольника АВС помогите решить!!!

3411. ilo, 6 апреля 2011, 19:12:13
3409. Алексей, 6 апреля 2011, 18:00:58
Помогите решить задачку (B8 №6007)
Прямая y=7x-5 параллельна касательной к графику функции
y=x в квадрате +6x-8.
Найдите абсциссу точки касания.

Коэффициент касательной равен значению производной в точке касания.Производная ф-ции y'=2x+6; т.к касательная параллельна прямой 7x-5,то коэффициенты касательной и прямой равны 2x+6=7 при x=0,5

3412. незнакомка, 6 апреля 2011, 19:30:07
На рисунке ABE = 104°, DCF = 76°, ЛС = 12 см. Найдите сторону АВ треугольника ABC.

3413. миф, 6 апреля 2011, 20:03:04
найдите наименьшее значение функции y=4в степени log 4 (x+1)+ 5в степени log5 (7-х) + 2х2- 3

3414. MORGan, 7 апреля 2011, 08:09:06
Объём пирамиды 300корней из 3.надо найти площадь вписаного в основания круга ,если высота =12см

3415. ilo, 7 апреля 2011, 08:41:11
3410. Марина, 6 апреля 2011, 18:03:00
в прямоугольном треугольнике АВС биссектриса прямого угла С делит гипотенузу на отрезки 3 и 4.найдите площадь треугольника АВС помогите решить!!!

Биссектриса треугольника делит противоположную сторону,т. е. гипотенузу на отрезки, пропорциональные прилежащим сторонам,т.е. катетам.Пусть a и b - катеты: a/b=3/4;a=3/4b;b^2+9/16b^2=49; 25/16b^2=49;b^2=49*16/25;b=7*4/5=28/5;a=3/4*28/5=21/5;S=1/2*21/5*28/5=11,76

3416. Илья, 7 апреля 2011, 15:11:07
Всем Привет,тут кто нибудь в физике разбирается?

3417. мистер киса, 7 апреля 2011, 15:28:23
Новые прототипы В 5 - помогите разобраться -
1.) В магазине одежды объявлена акция: если покупатель приобретает товар на сумму свыше 10000 руб., он получает сертификат на 1000 рублей, который можно обменять в том же магазине на любой товар ценой не выше 1000 рублей. Если покупатель участвует в акции, он теряет право возвратить товар в магазин. Покупатель И. хочет приобрести пиджак ценой 9500 р., рубашку ценой 800 р. и галстук ценой 500 р. В каком случае И. заплатит за покупку меньше всего:
1) И. купит все три товара сразу.
2) И. купит сначала пиджак и рубашку, галстук получит за сертификат.
3) И. купит сначала пиджак и галстук, получит рубашку за сертификат.
В ответ запишите, сколько рублей заплатит И. за покупку в этом случае.

2.)Вася загружает на свой компьютер из Интернета файл размером 30 Мб за 28 секунд. Петя загружает файл размером 28 Мб за 24 секунды, а Миша загружает файл размером 38 Мб за 32 секунды. Сколько секунд будет загружаться файл размером 665 Мб на компьютер с наибольшей скоростью загрузки?

3418. марина , 7 апреля 2011, 17:01:16
в основании пирамиды АВСDT лежит ромб ABCD с острым углом А=60 и стороной корень из трех высота пирамиды АТ равна 2 определить расстояние от точки пересечения диагоналей оснований до плоскости СВТ. определить координаты точек когда решаешь системой координат не могу их определить!

3419. марина , 7 апреля 2011, 17:04:00
3415. ilo, 7 апреля 2011, 08:41:11
3410. Марина, 6 апреля 2011, 18:03:00

в прямоугольном треугольнике АВС биссектриса прямого угла С делит гипотенузу на отрезки 3 и 4.найдите площадь треугольника АВС помогите решить!!!

Биссектриса треугольника делит противоположную сторону,т. е. гипотенузу на отрезки, пропорциональные прилежащим сторонам,т.е. катетам.Пусть a и b - катеты: a/b=3/4;a=3/4b;b^2+9/16b^2=49; 25/16b^2=49;b^2=49*16/25;b=7*4/5=28/5;a=3/4*28/5=21/5;S=1/2*21/5*28/5=11,76

что означает ^ знак

3420. ilo, 7 апреля 2011, 19:40:42
3419. марина , 7 апреля 2011, 17:04:00
[quote]3415. ilo, 7 апреля 2011, 08:41:11
3410. Марина, 6 апреля 2011, 18:03:00
что означает ^ знак

^-степень

3421. ilo, 7 апреля 2011, 20:12:42
3417. мистер киса, 7 апреля 2011, 15:28:23
Новые прототипы В 5 - помогите разобраться -
1.) В магазине одежды объявлена акция: если покупатель приобретает товар на сумму свыше 10000 руб., он получает сертификат на 1000 рублей, который можно обменять в том же магазине на любой товар ценой не выше 1000 рублей. Если покупатель участвует в акции, он теряет право возвратить товар в магазин. Покупатель И. хочет приобрести пиджак ценой 9500 р., рубашку ценой 800 р. и галстук ценой 500 р. В каком случае И. заплатит за покупку меньше всего:
3) И. купит сначала пиджак и галстук, получит рубашку за сертификат.

Не ясно,как можно получить рубашку за сертификат,если по условиям акции пиджак и галстук стоит 10 000,а сертификат выдаётся,если покупатель приобретает товар на сумму свыше 10000 руб


2.)Вася загружает на свой компьютер из Интернета файл размером 30 Мб за 28 секунд. Петя загружает файл размером 28 Мб за 24 секунды, а Миша загружает файл размером 38 Мб за 32 секунды. Сколько секунд будет загружаться файл размером 665 Мб на компьютер с наибольшей скоростью загрузки?

Вася со скоростью 30/28=360/336; Петя - 28/24=392/336; Миша - 38/32=399/336;Наибольшая скорость у Миши 665:19/16=560"

3422. мистер киса, 8 апреля 2011, 00:05:52
для 3421 ilo Спасибо. У меня возник такой же вопрос по 1 задаче, но такое условие задачи в открытом банке - новое в прототипах задач В 5 , во второй у меня такой же ответ.Хотел проверить, правильно ли понял.

3423. andr_o, 8 апреля 2011, 09:38:12
Объясните,пожалуйста, решение задачи В8(7473) Точки экстремума 1,2,3,4,5,7,8,9 Их сумма равна 39. В ответе стоит 5?

3424. Lexxus, 8 апреля 2011, 10:51:25
3423. andr_o, 8 апреля 2011, 09:38:12
Объясните,пожалуйста, решение задачи В8(7473) Точки экстремума 1,2,3,4,5,7,8,9 Их сумма равна 39. В ответе стоит 5?

У заданий B8 самая высокая степень ошибок по невнимательности. Тут как раз такой случай (почему-то большинство находит между словами "график" и "функции" слово "производной"). Правильный ответ, конечно, 39.

3425. VladislavOfficiaL, 8 апреля 2011, 13:38:41
sqrt(2x+1)<x-1
решал так:Обе части возвел в квадрат
Получилась система:

2x+1<(x-1)^2
2x+1>=0
x-1>0
дальше
-x^2+4x<0
x>=-1/2
x>1

решая это:-x^2+4x<0 я могу записать так?
x^2-4x>0 дальше x(x-4)>0 x>0 и x>4
Я правильно делаю?

3426. VladislavOfficiaL, 8 апреля 2011, 14:50:21
3425. VladislavOfficiaL, 8 апреля 2011, 13:38:41
Помогите решить неравенство
sqrt(2x+1)<x-1
решал так:Обе части возвел в квадрат
Получилась система:

2x+1<(x-1)^2
2x+1>=0
x-1>0
дальше
-x^2+4x<0
x>=-1/2
x>1

решая это:-x^2+4x<0 я могу записать так?
x^2-4x>0 дальше x(x-4)>0 x>0 и x>4
Я правильно делаю?

А может за скобки нужно выносить-x(x-4)<0
Тогда -x<0 или x>0
(x-4)<0 или x<4 ???????????

3427. Kattu, 8 апреля 2011, 15:26:54
3425. VladislavOfficiaL, 8 апреля 2011, 13:38:41
неравенство x^2 - 4x > 0 решается методом интервалов. Приравнивается к нулю, на числовую прямую и проставляются знаки. Ответ от минут бесконечности до 0 и от 4 до плюс бесконечности.

3428. VladislavOfficiaL, 8 апреля 2011, 15:34:15
3427. Kattu, 8 апреля 2011, 15:26:54
3425. VladislavOfficiaL, 8 апреля 2011, 13:38:41
неравенство x^2 - 4x &gt; 0 решается методом интервалов. Приравнивается к нулю, на числовую прямую и проставляются знаки. Ответ от минут бесконечности до 0 и от 4 до плюс бесконечности.

Реши подробно вот это:
-x^2+4<0
Если можно подробно

3429. VladislavOfficiaL, 8 апреля 2011, 15:51:32
3428. VladislavOfficiaL, 8 апреля 2011, 15:34:15
3427. Kattu, 8 апреля 2011, 15:26:54
3425. VladislavOfficiaL, 8 апреля 2011, 13:38:41
неравенство x^2 - 4x &amp;gt; 0 решается методом интервалов. Приравнивается к нулю, на числовую прямую и проставляются знаки. Ответ от минут бесконечности до 0 и от 4 до плюс бесконечности.

Реши подробно вот это:
-x^2x+4<0
Если можно подробно

Вымерли что ли все? Кто поможет????

3430. марина, 8 апреля 2011, 16:01:33
3418. марина , 7 апреля 2011, 17:01:16
в основании пирамиды АВСDT лежит ромб ABCD с острым углом А=60 и стороной корень из трех высота пирамиды АТ равна 2 определить расстояние от точки пересечения диагоналей оснований до плоскости СВТ. определить координаты точек когда решаешь системой координат не могу их определить!

3431. Lexxus, 8 апреля 2011, 16:13:53
3430. марина, 8 апреля 2011, 16:01:33
в основании пирамиды АВСDT лежит ромб ABCD с острым углом А=60 и стороной корень из трех высота пирамиды АТ равна 2 определить расстояние от точки пересечения диагоналей оснований до плоскости СВТ

Если не указано, какая именно имеется в виду пирамида (например, прямая), у этой задачи бесконечно много решений.

3432. Lexxus, 8 апреля 2011, 16:47:12
3425. VladislavOfficiaL, 8 апреля 2011, 13:38:41

решая это:-x^2+4x<0 я могу записать так?
x^2-4x>0 дальше x(x-4)>0 x>0 и x>4


x(x-4) > 0 - правильно. Дальше чушь.

Выражение x(x-4) обращается в ноль при x=0 и x=4.
Если x>4, оба сомножителя положительны.
Если 0<x<4, один из сомножителей меньше нуля, а значит, и всё выражение меньше нуля.
Если x<0, оба сомножителя отрицательны, минус на минус даёт плюс. Выражение больше нуля.

Значит, в ответе будет
(-бесконечность, 0) и (4, +бесконечность)

3433. VladislavOfficiaL, 8 апреля 2011, 17:48:11
3432. Lexxus, 8 апреля 2011, 16:47:12
3425. VladislavOfficiaL, 8 апреля 2011, 13:38:41

решая это:-x^2+4x<0 я могу записать так?
x^2-4x>0 дальше x(x-4)>0 x>0 и x>4


x(x-4) > 0 - правильно. Дальше чушь.

Выражение x(x-4) обращается в ноль при x=0 и x=4.
Если x>4, оба сомножителя положительны.
Если 0<x<4, один из сомножителей меньше нуля, а значит, и всё выражение меньше нуля.
Если x<0, оба сомножителя отрицательны, минус на минус даёт плюс. Выражение больше нуля.

Значит, в ответе будет
(-бесконечность, 0) и (4, +бесконечность)


После того,как нахожу корни,черчу прямую и на ней ставлю две точки 0 и 4
Потом смотрю на само неравенство x(x-4) > 0 отмечаю все точки больше нуля,это -бесконечность и + бесконечность и потом подставляю методом крыш остальные значения x>=-1/2 и x>1 Только ответ не сходится,как в учебнике.
Там написалиX принадлежит квадратная скобка-1/2;4 ) Почему так ?

3434. Гулька, 8 апреля 2011, 18:34:47
Помогите пожалуйста решить задачи!!!! и зарание спасибо тем, кто поможет))))))
1 задача) в треугольнике АВС угол С равен 90 градусов, В= 60 градусов, АВ= 6 см. найдите ВС.
2 задача) в правельную 4х-угольную призму с ребром основания 6 и и высотой 4 вписан конус. найдите объем конуса.
3 задача) в правильной 4х-угольной пирамиде апофема образует с плоскостью основания угол 30 градусов. сторона основания пирамиды равна 12 см. найдите площадь поверхности пирамиды.

3435. коко, 8 апреля 2011, 18:56:38
Из пункта А ушел первый поезд . Через 2 часа из пункта А в томже направлении вышел второй поезд и через 10 часов догнал первый поезд. Найдите среднюю скорость второго поезда если сумма средних скоростей поездов равна 110 км/ч.

3436. ilo, 8 апреля 2011, 22:17:30
3433. VladislavOfficiaL, 8 апреля 2011, 17:48:11

Только ответ не сходится,как в учебнике.
Там написалиX принадлежит квадратная скобка-1/2;4 ) Почему так ?

Возьми хотя бы 0 из этого промежутка.подставь в исходное неравенство-левая часть отрицательна- мурня получается

3437. ilo, 8 апреля 2011, 22:33:12
3434. Гулька, 8 апреля 2011, 18:34:47
Помогите пожалуйста решить задачи!!!! и зарание спасибо тем, кто поможет))))))
1 задача) в треугольнике АВС угол С равен 90 градусов, В= 60 градусов, АВ= 6 см. найдите ВС.
2 задача) в правельную 4х-угольную призму с ребром основания 6 и и высотой 4 вписан конус. найдите объем конуса.
3 задача) в правильной 4х-угольной пирамиде апофема образует с плоскостью основания угол 30 градусов. сторона основания пирамиды равна 12 см. найдите площадь поверхности пирамиды.

1) угол А=30,АВ-гипотенуза,ВС лежит напротив угла в 30 градусов,значит равен3
2)в основании правильной призмы-квадрат,в него вписан круг-основание конуса,его диаметр 6,площадь-9pi;высота конуса совпадает с высотой призмы; Vконуса=1/3*9pi*4=12pi
3)Из прямоугольного треугольника,образованного апофемой,ееё проекцией на основание и высотой пирамиды по т.Пифагора найти апофему4sqrt3;Sбок.пов-сти =апофема*полупериметр=96sqrt3; осталось прибавить площадьоснования 144

3438. ilo, 8 апреля 2011, 22:43:33
3435. коко, 8 апреля 2011, 18:56:38
Из пункта А ушел первый поезд . Через 2 часа из пункта А в томже направлении вышел второй поезд и через 10 часов догнал первый поезд. Найдите среднюю скорость второго поезда если сумма средних скоростей поездов равна 110 км/ч.

Средняя скорость одного x, другого 110-х.Один из них шёл12 часов у него скорость меньше,другой 10 часов.(110-x)*12=10x-пути они прошли равные до встречи,откуда x=60, у другого значит 50.Второй шёл с большей скоростью,значит его скорость 60.

3439. VladislavOfficiaL, 9 апреля 2011, 05:02:58
3436. ilo, 8 апреля 2011, 22:17:30
3433. VladislavOfficiaL, 8 апреля 2011, 17:48:11

Только ответ не сходится,как в учебнике.
Там написалиX принадлежит квадратная скобка-1/2;4 ) Почему так ?

Возьми хотя бы 0 из этого промежутка.подставь в исходное неравенство-левая часть отрицательна- мурня получается

Ну да гон какой-то получается.Ответ не верный в учебнике значит. У меня получился :(4;+бесконечность)

3440. Леонид , 9 апреля 2011, 06:11:16
Уважаемый Lexxus, прошу о помощи в решении вот такой задачи. Пожылуйста! Нужно срочно. "В треугольнике АВС сторона ВС вдвое длиннее стороны АВ. На ВС выбрана такая точка Д, что ВС:СД=2:3. Отрезок АД пересекает биссектрису ВЕ в точке К. Найдите отношение ВК:КЕ."

3441. Lexxus, 9 апреля 2011, 11:11:40
3440. Леонид , 9 апреля 2011, 06:11:16
В треугольнике АВС сторона ВС вдвое длиннее стороны АВ. На ВС выбрана такая точка Д, что ВС:СД=2:3. Отрезок АД пересекает биссектрису ВЕ в точке К. Найдите отношение ВК:КЕ

По теореме о биссектрисе, она делит противоположную сторону в отношении, равном отношению двух прилежащих сторон.
То есть, AE/CE = AB/BC.
Значит, AE/CE = 1/2 = BD/CD.

Поэтому, по теореме, обратной теореме Фалеса, отрезки AB и DE параллельны.

Это значит, что угол EDA равен углу BAD и угол DEB равен углу ABE (как накрест лежащие). Значит, треугольники DEK и AKB подобны, и BK/KE = AK/KD.

А всё по той же теореме о биссектрисе, AK/KD = AB/BD = 3/2.

Ответ: 3/2

3442. Леонид , 9 апреля 2011, 11:24:30
БОЛЬШУЩЕЕ СПАСИБО!

3443. Катерина, 9 апреля 2011, 16:15:34
Добрый день. Пожалуйста, помогите решить и объясните, как выполнять след. задание

С1

Найдите все значения x, для которых точки графика функции
y= (в числителе) 64 в степени x +7 * 8 в степени x / (в знаменателе ) 5-4x лежат выше соответствующих точек графика функции
y= (в числителе) 8 / (в знаменателе ) 5-4x


3444. Леонид , 9 апреля 2011, 16:20:51
Катерина, это задание ЕГЭ до 2010 года. Запиши неравенство: та функция, крафик которой выше, больше той, график которой ниже. Там всё просто решается.

3445. Василий, 9 апреля 2011, 21:42:52
1. В равнобедренном прямоугольном треугольнике один из катетов лежит в плоскости a, а другой образует с ней угол 45. Найдите угол между гипотенузой данного треугольника и данной плоскостью.

2. Точка K, не принадлежащая плоскости равностороннего треугольника, удалена от каждой его вершины на расстояние (корень из 13) см, а от каждой его стороны – на 2 см. Найдите расстояние от точки K до плоскости треугольника.

3. Угол между плоскостями двух равнобедренных треугольников ABC и BCD, имеющих общую боковую сторону BC, равен 90. Найдите расстояние между точками A и D, если основание каждого треугольника равно a, а каждая боковая сторона равна b.

4. Внутри двугранного угла из точки M, принадлежащей его ребру, проведен к нему перпендикуляр, на котором отложен отрезок MN, в два раза больший своей ортогональной проекции на одну из граней двугранного угла. Найдите угол, который образует MN с другой гранью, если двугранный угол равен 100.

Помогите

3446. Марина, 10 апреля 2011, 13:29:40
В остроугольном треугольнике АВС проведены высоты АА1 и ВВ1. точкиА2 и В2 являются точками пересечения высот с описанной около треугольника АВС окружностью. высоты пересекаются в точке Р. отрезок АН=9,АН1=2,ВН=6.найдите отношение площадей треугольников АНВ2 и ВНА2

3447. ВиКуЛеЧкА Ч, 10 апреля 2011, 14:56:28
F(x)=-x3+3x2+9x-29 на отрезке [-1;4]

Объясните пожалуйста как это решить!!!)

3448. Леонид , 10 апреля 2011, 15:52:16
производная у штрих = 3х^2 + 6x + 9 = 0. или х кв. + 2х + 3 = 0. Корней нет. Значения функции на концах отрезка У(-1) = - 34. У(4) = 64 + 48 + 36 -29 = 119. Что надо НАИБОЛЬШЕЕ или НАИМЕНЬШЕЕ, бери из 119 и -34.

3449. Алёна, 10 апреля 2011, 15:56:22
кто-нибудь, помогите, пожалуйста, решить задачу:

ребро куба равно корень из 6. найдите расстояние между диагональю куба и диагональю любой его грани. ответ должен получиться 1. Буду Вам очень признательна! :)

3450. Ириска, 10 апреля 2011, 19:10:01
Ребята,помогите пожалуйста решить задачи!!Заранее спасибо.
1.Плоские углы трехгранного угла равны 45, 45 и 60. Найдите двугранный угол, образованный плоскостями равных плоских углов.
2.Докажите, что двугранный угол между смежными боковыми гранями любой правильной 4-угольной пирамиды является тупым.
3.Докажите, что любое сечение трехгранного угла с плоскими углами по 900, пересекающее все его ребра, является остроугольным треугольником.

3451. Lexxus, 10 апреля 2011, 19:17:06
3445. Василий, 9 апреля 2011, 21:42:52
В равнобедренном прямоугольном треугольнике один из катетов лежит в плоскости a, а другой образует с ней угол 45. Найдите угол между гипотенузой данного треугольника и данной плоскостью.

Треугольник ABC, угол C - прямой, BC принадлежит плоскости.
AC = BC = x, AB = x*sqrt(2)
Опустим перпендикуляр AA1 к плоскости a.

Искомый угол - угол A1BA.

Угол A1CA равен 45 градусов, угол AA1C - прямой. AA1 = AC*sin(45 градусов) = x/sqrt(2).

sin(A1BA) = AA1/AB = (x/sqrt(2))/(x*sqrt(2)) = 1/2

Угол A1BA = arcsin(1/2) = 30 градусов.

3452. ilo, 10 апреля 2011, 21:04:33
3445. Василий, 9 апреля 2011, 21:42:52

2. Точка K, не принадлежащая плоскости равностороннего треугольника, удалена от каждой его вершины на расстояние (корень из 13) см, а от каждой его стороны – на 2 см. Найдите расстояние от точки K до плоскости треугольника.
Помогите

КО- высота правильной пирамиды,боковые рёбра которой равны sqrt13, О-центр правильного треугольника,лежащего в основании, КН- апофема. OH=r, r=1/3высоты треугольника,лежащего в основании(h); h=sqrt3/2*a=sqrt3*sqrt6/2=3sqrt2/2;r=sqrt2/2; Из треугольника KOH: KO^2=4-r^2=7/2; KO=sqrt14/2;

3453. Евгения , 10 апреля 2011, 21:08:54
помогите пожалуйсто .
Найдите корень уравнения
В3 под корнем -72-17 = безкорня -х
Если уравнение имеет более одного корня, в ответе укажите меньший из них.

В4 в треуольнике АВС АС=ВС АВ=27.5 соs А 1 корень 5 и все это деленное на 5

3454. мистер киса, 10 апреля 2011, 21:16:56
Помогите решиь прототип В 8 - новое в банке заданий 2011 г.

Материальная точка движется прямолинейно по закону x(t) = 6t^2 - 48t + 17, где x — расстояние от точки отсчета в метрах,t — время в секундах, измеренное с начала движения. Найдите ее скорость (в метрах в секунду) в момент времени t = 9 с.

3455. Алёна, 10 апреля 2011, 21:38:30
3449. Алёна, 10 апреля 2011, 15:56:22
кто-нибудь, помогите, пожалуйста, решить задачу:

ребро куба равно корень из 6. найдите расстояние между диагональю куба и диагональю любой его грани. ответ должен получиться 1. Буду Вам очень признательна! :)



помогите, пожалуйста!!!((

3456. Lexxus, 10 апреля 2011, 21:53:23
3452. ilo, 10 апреля 2011, 21:04:33
КО- высота правильной пирамиды,боковые рёбра которой равны sqrt13, О-центр правильного треугольника,лежащего в основании, КН- апофема. OH=r, r=1/3высоты треугольника,лежащего в основании(h); h=sqrt3/2*a=sqrt3*sqrt6/2=3sqrt2/2;r=sqrt2/2; Из треугольника KOH: KO^2=4-r^2=7/2; KO=sqrt14/2;

Это неправильно.
Сторона лежащего в основании пирамиды треугольника будет равна 2*sqrt(13-4)=6.
r = 6*(sqrt(3)/2)/3 = sqrt(3)
KO = sqrt(4-3) = 1.

3457. мистер киса, 10 апреля 2011, 22:30:01
Помогите решить новые прототипы В 8 из открытого банка заданий 2011 год

Материальная точка движется прямолинейно по закону x(t)=6t^2-48+17 , где x— расстояние от точки отсчета в метрах, t — время в секундах, измеренное с начала движения. Найдите ее скорость (в метрах в секунду) в момент времени t=9 с.

3458. Lexxus, 10 апреля 2011, 22:34:59
3449. Алёна, 10 апреля 2011, 15:56:22
ребро куба равно корень из 6. найдите расстояние между диагональю куба и диагональю любой его грани.


3459. ilo, 11 апреля 2011, 12:12:59
3456. Lexxus, 10 апреля 2011, 21:53:23
3452. ilo, 10 апреля 2011, 21:04:33
КО- высота правильной пирамиды,боковые рёбра которой равны sqrt13, О-центр правильного треугольника,лежащего в основании, КН- апофема. OH=r, r=1/3высоты треугольника,лежащего в основании(h); h=sqrt3/2*a=sqrt3*sqrt6/2=3sqrt2/2;r=sqrt2/2; Из треугольника KOH: KO^2=4-r^2=7/2; KO=sqrt14/2;

Это неправильно.
Сторона лежащего в основании пирамиды треугольника будет равна 2*sqrt(13-4)=6.
r = 6*(sqrt(3)/2)/3 = sqrt(3)
KO = sqrt(4-3) = 1.

Грешу вычислительными ошибками.Спасибо.

3460. ilo, 11 апреля 2011, 12:19:07
3440. Леонид , 9 апреля 2011, 06:11:16
Уважаемый Lexxus, прошу о помощи в решении вот такой задачи. Пожылуйста! Нужно срочно. "В треугольнике АВС сторона ВС вдвое длиннее стороны АВ. На ВС выбрана такая точка Д, что ВС:СД=2:3. Отрезок АД пересекает биссектрису ВЕ в точке К. Найдите отношение ВК:КЕ."

Lexxus, где находится точка D в этой задаче,не получается чертёж и решение или опечатка?

3461. Lexxus, 11 апреля 2011, 13:37:17
3460. ilo, 11 апреля 2011, 12:19:07
Lexxus, где находится точка D в этой задаче,не получается чертёж и решение или опечатка?

Да, небольшая опечатка там. ВС:СД=3:2, а не 2:3.

3462. Наташа, 11 апреля 2011, 14:38:40
Наклонная AB образует с плоскостью a угол 45, прямая AC, лежащая в этой плоскости, составляет угол 45 с ортогональной проекцией наклонной AB на плоскость a. Найдите угол BAC.

в ответе написано 60 градусов.подскажите как решить

3463. ilo, 11 апреля 2011, 16:52:26
3462. Наташа, 11 апреля 2011, 14:38:40
Наклонная AB образует с плоскостью a угол 45, прямая AC, лежащая в этой плоскости, составляет угол 45 с ортогональной проекцией наклонной AB на плоскость a. Найдите угол BAC.

в ответе написано 60 градусов.подскажите как решить

Пусть а=45 величина угла между наклонной АВ и её проекцией на плоскость,в=45-величина угла между проекцией наклонной АВ и прямой АС, проведённой через основание той же наклонной в плоскости проекции, и с-величина угла между наклонной АС и прямой, проведённой через её основание в плоскости проекции. Тогда справедливо следующее соотношение: cosc=cosa*cosb(теорема о трёх косинусах) cosc=sqrt2/2*sqrt2/2=1/2; угол c=60

3464. Ирина, 11 апреля 2011, 17:14:57
прямая 3x+2y=c касается гиперболы y=b делённое на x с положительными координатами. Найдите координаты точки касания.

3465. Аня, 11 апреля 2011, 17:41:25
Дан ромб со стороной a и углом 45. Точка L удалена от всех прямых, на которых лежат стороны ромба, на расстояние b. Найдите расстояние от точки L до плоскости ромба.

помогите пожалуйста решить

3466. Ира, 11 апреля 2011, 18:04:28
Угол между плоскостями двух равнобедренных треугольников ABC и BCD, имеющих общую боковую сторону BC, равен 120. Расстояние между точками A и D равно m. Основание каждого треугольника равно a. Найдите боковые стороны треугольников.

поясните пожалуйста как найти стороны

3467. alfa20, 11 апреля 2011, 20:11:31
Lexxus, а в №19369 точно ответ 0,8? Что-то у меня 0,6 получается...

3468. Lexxus, 11 апреля 2011, 20:36:21
3467. alfa20, 11 апреля 2011, 20:11:31
Lexxus, а в №19369 точно ответ 0,8? Что-то у меня 0,6 получается...

Да, у меня тоже. Ошибка там.

3469. Кристина, 12 апреля 2011, 14:16:07
помогите задачу решить пожалуйста в основании прямой призмы ABCDA1B1C1D1 лежит прямоугольный треугольник(уголС=90) через сторону BC и вершину A1 проведена плоскость, угол ВА1С=30 , А1В=10 см найдите площадь боковой поверхности

3470. ilo, 12 апреля 2011, 15:28:32
3466. Ира, 11 апреля 2011, 18:04:28
Угол между плоскостями двух равнобедренных треугольников ABC и BCD, имеющих общую боковую сторону BC, равен 120. Расстояние между точками A и D равно m. Основание каждого треугольника равно a. Найдите боковые стороны треугольников.

поясните пожалуйста как найти стороны

Проведи высоты к ВС=x, дважды вырази площадь равнобедренного треугольника,из равенства найди проведёную высоту.Затем для равнобедренного треугольника сторонами которого являются проведённые высоты и AD записать теорему косинусов.Из равенства найти x.

3472. Ксенька, 13 апреля 2011, 16:14:50
Помогите пожалуйста решить задачу. с2

В правильной шестиугольной пирамиде SABCDEF, стороны основания равны 1, а боковые ребра равны 2, найдите угол между прямыми SF и BM, где М - середина ребра SC

спасибо заранее!

3473. Collins, 13 апреля 2011, 20:28:28
помогите пожалуйста решить задачу.
Каждое ребро правильного тетраэдра равно а. Найдите объем тетраэдра и вписанного в него конуса. (можно решить задачу для а=6)
очень надо((((

3474. стас, 13 апреля 2011, 21:24:12
Точка M равноудаена от всех вершин равнобедренного треугольника ACB (угол C = 90o), AC = BC = 4 см. Расстояние от точки M до плосости треугольника равно 2&#8730;3 см.
а) Докажите, что плоскость AMB перпендикулярна плоскости ABC.
б) Найдите угол межд прямой MC и плоскостью ABC

3475. стас, 13 апреля 2011, 21:24:55
пожалуйста решить, срочно надо!!!

3476. стас, 13 апреля 2011, 21:24:55
пожалуйста решить, срочно надо!!!

3477. Аргентина, 13 апреля 2011, 21:57:10
Помогите пжалуйста решить мне! буду очень бдагодарна =*
ABCT-пирамида. Ребро AT и CT делятся соответственно точками M и N в отношении 3:4 и 2:5. Найти отношение объемов многогранников MNTB и ACBT

3478. ilo, 13 апреля 2011, 22:13:23
3472. Ксенька, 13 апреля 2011, 16:14:50
Помогите пожалуйста решить задачу. с2

В правильной шестиугольной пирамиде SABCDEF, стороны основания равны 1, а боковые ребра равны 2, найдите угол между прямыми SF и BM, где М - середина ребра SC

спасибо заранее!

решение и чертёж http://depositfiles.com/files/2c2c7nvpm

3479. ilo, 13 апреля 2011, 22:50:46
3473. Collins, 13 апреля 2011, 20:28:28
помогите пожалуйста решить задачу.
Каждое ребро правильного тетраэдра равно а. Найдите объем тетраэдра и вписанного в него конуса. (можно решить задачу для а=6)
очень надо((((

Vт.=1/3Sh; S=a^2*sqrt3/4;высота тетраэдра=a^2-R^2=sqrt2*a/sqrt3; R=a/sqrt3;
Vт.=a^3sqrt2/12; Sосн. конуса=pi*r^2=a^2*pi/12;r=R/2=a/2*sqrt3=a*sqrt3/6; Vкон.=1/3Sосн.*h=pi*a^3*sqrt6/108;

3480. ilo, 13 апреля 2011, 23:33:00
3474. стас, 13 апреля 2011, 21:24:12
Точка M равноудаена от всех вершин равнобедренного треугольника ACB (угол C = 90o), AC = BC = 4 см. Расстояние от точки M до плосости треугольника равно 2&amp;#8730;3 см.
а) Докажите, что плоскость AMB перпендикулярна плоскости ABC.
б) Найдите угол межд прямой MC и плоскостью ABC

Точка M лежит на перпендикуляре,проведённом через центр описанной окружности-точку H,лежащую на середине гипотенузы.Треугольники AMH,HMB,CMH равны как прямоугольные по катетам.AB по теореме Пифагора равна 4sqrt2;AH=BH=CH=2sqrt2;Угол MCH найти из прямоугольного треугольника:MH/CH=tgMCH;

3481. ilo, 13 апреля 2011, 23:37:20
3475. стас, 13 апреля 2011, 21:24:55
пожалуйста решить, срочно надо!!!

MH перпендикуярна двум пересекающимся прямым СH и AB,следовательно перпендикулярна плоскости ABC/

3482. Инна, 14 апреля 2011, 01:24:46
Помогите пожалуйста

Окружность содержит вершины B , C и D трапеции ABCD и касается ей боковой стороны AB. BC параллельна AD. и BC = 4 , AD = 12. AВ пересекает окружность в точке E. AE = 3. Найти площадь треугольника ABD

3483. стас, 14 апреля 2011, 08:02:13
боьшое спасибо ilo

3484. Аська, 14 апреля 2011, 09:27:02
Скажите,как написать задачу сюда,чтобы помогли решить,у меня сейчас экзамен и очень надо.

3485. Дмитрий, 14 апреля 2011, 15:00:00
В прямоугольном треугольнике BCD с прямым углом угол С, угол B равен 30 , BC равен корень из 3
найти BD
Помоги решить пожалуйста!

3486. Дмитрий, 14 апреля 2011, 15:01:50
Из пункта А в пункт В выехал мотоциклист и одновременно из В в А выехал автомобилист.Мотоциклист прибыл в В через 2 часа после встречи, а автомобилист в А через 30 минут после встречи.Сколько часов был в пути автомобилист?
И это тоже пожалуйста!!!

3487. Светлана, 14 апреля 2011, 15:13:33
Помогите решить задачу 5 класс Из пункта А в пункт В выехала грузов.маш. одновременно с ней из пункта А выехала легков. маш. Грузов. маш. через 2 ч. после начала движения встретила лекковую и ещё через 3 часа прибыла в пункт В. Сколько времени потратила легковая маш. на пути из пункта В в пункт А?

3488. ilo, 14 апреля 2011, 17:53:27
3485. Дмитрий, 14 апреля 2011, 15:00:00
В прямоугольном треугольнике BCD с прямым углом угол С, угол B равен 30 , BC равен корень из 3
найти BD
Помоги решить пожалуйста!

ctg30=BC/BD;BD=BC/ctg30 или обозначить СD=x;AB=2x и применить т.Пифагора

3489. ilo, 14 апреля 2011, 18:29:11
3486. Дмитрий, 14 апреля 2011, 15:01:50
Из пункта А в пункт В выехал мотоциклист и одновременно из В в А выехал автомобилист.Мотоциклист прибыл в В через 2 часа после встречи, а автомобилист в А через 30 минут после встречи.Сколько часов был в пути автомобилист?
И это тоже пожалуйста!!!

C-место встречи;Va-скорость автомобилиста,Vm-скорость мотоциклиста; AC=Va*0,5-путь до встречи m или путь после встречи a;СВ=Vm*2-путь до встречи а или путь после встречи m;Vm*2/Va-время до встречи а;До встречи они ехали одно и то же время: Vm*2/Va=0,5Vm/Va;0,5*Va^2=2*Vm^2;Va^2=4*Vm^2;Va=2*Vm;Подставляем в выражение времени до встречи Vm*2/2*Vm=1; Осталось прибавить время после встречи.

3490. Валя, 17 апреля 2011, 14:13:09
Друзья!Помогите решить 3 задачи:
1).Основание прямой призмы-треугольник со сторонами 3см и 8см и углом между ними 60.Найти площадь боковой поверхности призмы,если высота призмы 15см.

2).Найти площадь полной поверхности правильной треугольной пирамиды,если её апофема равна 4см,а угол между апофемой и высотой пирамиды равен 30.

3).Найти площадь полной поверхности правильного тетраэдра,высота которого равна 6см.

3491. Диана, 17 апреля 2011, 16:20:50
Помогите, пожалуйста, решить:
sinx*cosx-6sinx+6cosx+6=0
)))))))))))))))

3492. Катя, 18 апреля 2011, 11:18:20
Пожалуйста помогите срочно .В треугольнике АВС угол С равен 90 градусов синА=корень 21 деленого на 5. найдите син В

3493. Валя, 18 апреля 2011, 18:38:44
3491. Диана, 17 апреля 2011, 16:20:50
Помогите, пожалуйста, решить:
sinx*cosx-6sinx+6cosx+6=0
)))))))))))))))


Всё очень просто:
sinx*cosx-6sinx+6cosx+6=0 -формула
Sin(x+6x)=0
Sin 7x=0
7x=ПК, К принадлежит Z }: 7
x=ПК/7, К принадлежит Z
Ответ :x=ПК/7, К принадлежит Z

3494. Виктор, 18 апреля 2011, 19:01:49
http://live.mephist.ru/show/mathege2010/view/id/5301/ - правильный ответ 16, исправьте

3495. Лёха, 18 апреля 2011, 19:04:39
Помогите пожалуйста решить

Сравнить числа А=2,6&#215;10-5 &#247;1,3&#215;10-4 и В=0,02

3496. Аня, 18 апреля 2011, 19:07:49
Основанием пирамиды МАВСД является квадрат АВСД,ребро МД перпендикулярно к плоскости основания АД=ДМ=а.Надо найти площадь поверхности.ПОМОГИТЕ РЕШИТЬ,,НАДО СРОЧНО,,ДО ЗАВТРА,,,С РИСУНКОМ ПОЖАЛУЙСТА)))ОЧЕНЬ ПРОШУ,,,ЗАРАНЕЕ СПАСИБО)

3497. Lexa, 18 апреля 2011, 19:10:06
Помогите пожалуйста решить

2,6&#215;10^-5 &#247;1,310^-4 и В=0,02

3498. Lexxus, 18 апреля 2011, 19:29:41
3496. Аня, 18 апреля 2011, 19:07:49
Основанием пирамиды МАВСД является квадрат АВСД,ребро МД перпендикулярно к плоскости основания АД=ДМ=а.Надо найти площадь поверхности.ПОМОГИТЕ РЕШИТЬ,,НАДО СРОЧНО,,ДО ЗАВТРА,,,С РИСУНКОМ ПОЖАЛУЙСТА)))ОЧЕНЬ ПРОШУ,,,ЗАРАНЕЕ СПАСИБО)

Да там всё просто и без рисунка.
Все боковые грани пирамиды - прямоугольные треугольники.

Площади треугольников ADM и CDM равны (a^2)/2

AM = CM = sqrt(a^2+a^2) = a*sqrt(2), откуда
площади треугольников BAM и BCM равны
(a^2)*sqrt(2)/2

Площадь боковой поверхности равна сумме площадей всех четырех треугольников:

Sб = 2*(a^2)/2+2*(a^2)*sqrt(2)/2 = a^2*(1+sqrt(2))

А площадь полной поверхности равна площади боковой плюс площадь основания:

Sп = a^2*(1+sqrt(2)) + a^2 = a^2*(2+sqrt(2))

3499. Аня, 18 апреля 2011, 19:42:24
Спасибо большое!)

3500. Евгения, 18 апреля 2011, 19:52:54
Помогите решить... В треугольнике АВС АС=ВС=5, sinА=4/5. Найдите АВ?
Теплоход скорость которого в неподвижной воде равна 20 кмч, проходит по течению реки до пункта назначения и после стоянки возвращается в исходный пункт. Найдите расстояние, пройденное теплоходом за весь рейс, если скорость течения равна 4 кмч, стоянка длится 3 часа, а в исходный пункт теплоход возвращается через 13 часов после отплытия из него. Ответ дайте в киллометрах?

3501. наталья, 19 апреля 2011, 07:27:06
Помогите, пожайлуста, определить значение выражения:Z=Xв кубе-9X, при X=&#8731;(&#8730;35+2&#8730;2) +&#8731;(&#8730;35-&#8730;8)

3502. Аделя, 19 апреля 2011, 10:19:30
Найдите все положительные значения параметра а ,при каждом из которых уравнение а^x = x имеет единственное решение

3503. Lexxus, 19 апреля 2011, 13:43:08
3502. Аделя, 19 апреля 2011, 10:19:30
Найдите все положительные значения параметра а ,при каждом из которых уравнение а^x = x имеет единственное решение

Пусть f(x) = a^x, g(x) = x.

Функция g(x) - непрерывная, строго возрастающая на всей области определения и может принимать любое значение от минус бесконечности до плюс бесконечности.

При 0 < a < 1 функция f(x) - непрерывная, строго убывающая на всей области определения и может принимать значения в интервале (0;+бесконечность). Поэтому при любых таких a уравнение f(x) = g(x) имеет ровно одно решение.



При a = 1 функция f(x) тождественно равна единице, и уравнение f(x) = g(x) также имеет единственное решение x=1.

При a > 1:
Производная функции h(x) = (a^x-x) равна
(a^x-x)' = a^x*ln(a)-1
Приравняем её к нулю:
a^x*ln(a) = 1
a^x = 1/ln(a)
x = -log_a(ln(a)).

У производной единственный ноль. Слева от этого значения функция h(x) убывает, справа - возрастает.

Поэтому она либо вообще не имеет нулей, либо имеет два нуля. И один корень она имеет только в том случае, когда он совпадает с найденным экстремумом.

То есть, нам требуется найти такое значение a, при котором функция
h(x) = a^x-x достигает экстремума и обращается в ноль в одной и той же точке. Иными словами, когда прямая y=x является касательной к графику функции a^x.



То есть

a^x = x
a^x*ln(a) = 1

Подставляем a^x = x во второе уравнение:
x*ln(a) = 1, откуда ln(a)=1/x, a = e^(1/x).

Снова подставляем во второе уравнение:
(e^(1/x))^x*(1/x) = 1
e^1 = x
x = e.

А это подставляем в первое уравнение:
a^e = e
a = e^(1/e)

Ответ: (0;1] и e^(1/e)

3504. Татьяна, 19 апреля 2011, 17:24:35
Помогите, пожалуйста, решить задачи по геометрии
1. Сторона основания правильной четырехугольной пирамиды равна 2 корень из трех, двугранные углы при основании равны 45. Определить объем пирамиды?

3505. Lexxus, 19 апреля 2011, 17:33:30
3504. Татьяна, 19 апреля 2011, 17:24:35
Сторона основания правильной четырехугольной пирамиды равна 2 корень из трех, двугранные углы при основании равны 45. Определить объем пирамиды?

Если в правильной четырехугольной пирамиде двугранные углы при основании равны 45 градусов, то высота этой пирамиды равна половине стороны основания.

Отсюда объем равен
1/3*sqrt(3)*(2*sqrt(3))^2 = 4*sqrt(3)

3506. Татьяна, 19 апреля 2011, 17:33:42
И еще 2 задачи:
2. В основании прямой треугольной призмы лежит прямоугольный треугольник с катетами 4 и 2 см. Определить боковое ребро призмы и ее объем, если ее боковая поверхность 120 кв.см.
3. Стороны основания прямого параллелепипеда равны 3 и 5 см., угол между ними равен 60. Большая диагональ параллелепипеда равна 10 см. Найдите боковое ребро параллелепипеда и его объем.

3507. Яна, 19 апреля 2011, 20:00:18
Найдите значение выражения 4cos90градусов-8sin60градусов

3508. настя), 19 апреля 2011, 21:37:48
Апофема правильной четырехугольной пирамиды равна 2а, высота пирамиды а корень из 2. Найдите: а)сторону основания пирамиды; б) угол между боковой гранью и основанием; в) площадь поверхности пирамиды; г) расстояние от центра основания пирамиды до плоскости боковой грани.
помогите)

3509. настя), 19 апреля 2011, 22:22:39
помогите, пожалуйста)
очень нужно, решается судьба четвертной оценки!

3510. Анастасия, 20 апреля 2011, 00:32:33
Помогите пожалуйста, а то что-то ничего не понятно(
Найти множество вершин С квадратов ABCD, у которых вершина A находится на данной прямой, а вершина B - в данной точке.
Где может находиться четвёртая вершина квадрата, если две его вершины лежат на одной стороне данного острого угла, а третья - на другой?
Дан остроугольный треугольник АВС. Вписать в него квадрат, у которого две вершины лежат на стороне АВ.
прошу помощи)

3511. Анастасия, 20 апреля 2011, 08:07:19
Если что , это три разных задачи:)

3512. Lexxus, 20 апреля 2011, 08:20:51
3509. настя), 19 апреля 2011, 22:22:39
Апофема правильной четырехугольной пирамиды равна 2а, высота пирамиды а корень из 2. Найдите: а)сторону основания пирамиды; б) угол между боковой гранью и основанием; в) площадь поверхности пирамиды; г) расстояние от центра основания пирамиды до плоскости боковой грани.

а) Половина стороны основания - это катет прямоугольного треугольника, у которого гипотенуза - апофема пирамиды, а второй катет - её высота.
Значит, сторона пирамиды равна
2*sqrt(4*a^2-2*a^2) = 2*a*sqrt(2).

б) угол между боковой гранью и основанием равен углу между найденным катетом и гипотенузой треугольника из пункта а). А поскольку его катеты оба оказались равны a*sqrt(2), то этот угол равен 45 градусов.

в) Площадь квадрата со стороной 2*a*sqrt(2) плюс 4 площади равнобедренных треугольников с основанием 2*a*sqrt(2) и высотой 2a:
(2*a*sqrt(2))^2 + 4*(1/2*(2*a*sqrt(2))*(2*a))

г) Из того же самого треугольника из пунктов а) и б), искомое расстояние - это длина высоты, опущенной из его прямого угла на гипотенузу. Она будет равна половине гипотенузы, то есть a.

3513. Анастасия, 20 апреля 2011, 12:48:44
3510. Анастасия, 20 апреля 2011, 00:32:33

Найти множество вершин С квадратов ABCD, у которых вершина A находится на данной прямой, а вершина B - в данной точке.
Где может находиться четвёртая вершина квадрата, если две его вершины лежат на одной стороне данного острого угла, а третья - на другой?
Дан остроугольный треугольник АВС. Вписать в него квадрат, у которого две вершины лежат на стороне АВ.

помогите пожалуйста(
срочно надо...

3514. ЛяйсанКа, 20 апреля 2011, 14:31:32
Основанием прямой призмы АВСА1В1С1 является равнобедренный треугольник, в котором АВ=АС=2sqrt(2), ВС=2.Высота призмы равна 1.Найдите градусную меру угла между ребром АС и диагональю А1В боковой грани.

заранее огромное спасибо.

3515. Настюшка, 20 апреля 2011, 19:44:28
помогите решить уравнение:
3сos^2 (x) - sinx-1 =0

3516. Марина, 20 апреля 2011, 19:47:39
в параллелограмме диагональ длина 28 образует со сторонами углы синусы которых равны 3/5 и 5/13 найдите площадь параллелограмма

в параллелограмме ABCD известны величины AB=6 AD=7 косинус угла А =3/7 окружность касается прямых AC и BD а также стороны CD в точке Е найдите длину отрезка DE
помогите пожалуйста решить срочно с рисунком если можно!

3517. юрик, 20 апреля 2011, 22:42:51
все не угадаешь, надо щас учиться решать

3518. Kattu, 21 апреля 2011, 00:38:58
Lexxus! Исправьте, пожалуйста, ответ к задаче 8587. Спасибо!

3519. танюся, 21 апреля 2011, 10:38:22
решите плиз-срочно.. это не мне.. знакомому

В турбопаскале7 решить:

y=П+(корень)(a+b)2(всмысле в квадрате)-2(корень заканчивается) * sin П/3

3520. Просто Max, 21 апреля 2011, 13:02:13
в правильной треугольной пирамиде сторона основания равна 4 корень из 3 см, а плоский угол при вершине пирамиды равен 90 градусам. Найдите высоту пирамиды

3521. Анастасия, 21 апреля 2011, 13:36:11
3510. Анастасия, 20 апреля 2011, 00:32:33
Найти множество вершин С квадратов ABCD, у которых вершина A находится на данной прямой, а вершина B - в данной точке.
Где может находиться четвёртая вершина квадрата, если две его вершины лежат на одной стороне данного острого угла, а третья - на другой?
Дан остроугольный треугольник АВС. Вписать в него квадрат, у которого две вершины лежат на стороне АВ.

ну помогите пожалуйста
не решу-2 в журнал((

3522. танюся, 21 апреля 2011, 14:08:08
3521. Анастасия

напишите письмо личное ..кому нить сразу на мыло..

удачи.

У меня не горит, я подожду еще,может кто ответит.

3523. Анастасия, 21 апреля 2011, 14:24:26
3522. танюся, 21 апреля 2011, 14:08:08
..

кому например?

3524. танюся, 21 апреля 2011, 14:32:25
3523. Анастасия
bravo93.09@mail.ru VladislavOfficiaL

lexxus@mephist.ru Lexxus

3525. катя, 21 апреля 2011, 16:14:10
помогите пожалуйста решить:
1. Основание пирамиды MABCD является квадрат ABCD. Ребро MD перпендикулярно к плоскости основания. AD =DM =а. Найдите S поверхности пирамиды.

3526. Ольга, 21 апреля 2011, 16:18:44
Основанием прямого параллелепипеда является параллелограмм ABCD, стороны которого равны и 2a, острый угол равен 45°. Высота параллелепипеда равна меньшей высоте параллелограмма. Найдите:
*
меньшую высоту параллелограмма;
*
угол между плоскостью ABC и плоскостью основания;
*
площадь боковой поверхности параллелепипеда;
*
площадь поверхности параллелепипеда.


3527. Анна, 22 апреля 2011, 12:31:42
Здравствуйте помогите пожалуйста решить задачку, буду очень признательна)))))))

Точка М равноудалена от всех вершин равнобедренного прямоугольного треугольника АВС(угол С=90гр).АС=ВС=4см.Расстояние от точки М до плоскости треугольника равно 2*корень из 3 см. Найдите расстояние от точки Е - середины стороны АВ - до плоскости ВМС.

3528. vfrcbv, 22 апреля 2011, 19:23:14
дан прямоугольник ABC c гипотенуза АС=16 ВС=12 отрезок SC=20 перпендикуляр к плоскостиABC. найдите длину вектора СS+CB+BA

3529. максим, 22 апреля 2011, 19:25:08
Здравствуйте помогите решить.
дан прямоугольник ABC c гипотенуза АС=16 ВС=12 отрезок SC=20 перпендикуляр к плоскостиABC. найдите длину вектора СS+CB+BA

3530. Lexxus, 22 апреля 2011, 23:22:40
3527. Анна, 22 апреля 2011, 12:31:42
Точка М равноудалена от всех вершин равнобедренного прямоугольного треугольника АВС(угол С=90гр).АС=ВС=4см.Расстояние от точки М до плоскости треугольника равно 2*корень из 3 см. Найдите расстояние от точки Е - середины стороны АВ - до плоскости ВМС.

Поскольку треугольник ABC прямоугольный и равнобедренный, то AE = CE = BE, а это значит, что E - это проекция точки M на плоскость ABC и ME = 2*sqrt(3).

Пусть D - середина BC.
Искомое расстояние будет равно длине перпендикуляра EH, опущенного из точки E к MD.
ED = AC/2 = 2.
Отсюда MD = sqrt(ME^2+ED^2) = sqrt(12+4) = 4.

Прямоугольные треугольники EHD и MED подобны (угол D общий), значит,
ED/MD = EH/ME.
Отсюда
EH = ME/2 = sqrt(3).

3531. Lexxus, 22 апреля 2011, 23:45:31
3529. максим, 22 апреля 2011, 19:25:08
дан прямоугольник ABC c гипотенуза АС=16 ВС=12 отрезок SC=20 перпендикуляр к плоскостиABC. найдите длину вектора СS+CB+BA

Какой нафиг прямоугольник? Откуда у него гипотенуза? Нахрена там BC дано?

3532. нася, 23 апреля 2011, 11:06:56
Здравствуйте)
Основанием прямого параллелепипеда является параллелограмм ABCD, стороны которого равны и 2a, острый угол равен 45°. Высота параллелепипеда равна меньшей высоте параллелограмма. Найдите:
*
меньшую высоту параллелограмма;
*
угол между плоскостью ABC и плоскостью основания;
*
площадь боковой поверхности параллелепипеда;
*
площадь поверхности параллелепипеда.
не могли бы вы решить пожалуйста?

3533. Galya, 23 апреля 2011, 19:30:15
Помогите решить,пожалуйста!
1.Основание пирамиды прямоугольник со сторонами 6 и 8 см.Высота пирамиды 12 и проходит через точку пересечения диагоналей основания. Найти боковые ребра пиирамиды.
2.DABC-правильная треугольная пирамида. Строна основания три корня из трёх. Боковое ребро 5, MC медиана треугольника ABC. Найти S треугольника MDC. Искала по сайту эту(2) задачу, так и не нашла.
Заранее огромное спасибо.

3534. Дарья, 23 апреля 2011, 23:28:07
Помоги,пожалуйста. очень Вас прошу.

ABCDA1B1C1D1-куб.Точки F и P- середины рёбер A1B1 и B1C1 соответственно.Постройте сечение куба плоскостью, проходящее через точки F и P ,и перпендикулярно плоскости DAB1. Вычислите периметр этого сечения,если длина рёбер куба равна 6 см.

Заранее огромнейшее Вам спасибо.

3535. Ксюша, 24 апреля 2011, 15:04:31
Помогите решить задачу , очень нужно..заранее спасибо!
Апофема правильной 4-х угольной пирамиды равна 2а. Высота пирамиды равна ( а корней из 3). Найдите сторону основания пирамиды; угол между боковой гранью и основанием; площадь полной поверхности

3536. Lexxus, 24 апреля 2011, 19:09:27
3532. нася, 23 апреля 2011, 11:06:56
Основанием прямого параллелепипеда является параллелограмм ABCD, стороны которого равны и 2a, острый угол равен 45°. Высота параллелепипеда равна меньшей высоте параллелограмма. Найдите:
*
меньшую высоту параллелограмма;
*
угол между плоскостью ABC и плоскостью основания;
*
площадь боковой поверхности параллелепипеда;
*
площадь поверхности параллелепипеда.

Насколько я понял, следует читать "стороны которого равны а и 2a". Если так, то:

* Меньшая высота параллелограмма равна a*sin(45 градусов) = a*sqrt(2)/2.

* Плоскость ABC и есть плоскость основания. Можно не ошибаться хотя бы при записи условия?

* Площадь боковой поверхности равна сумме площадей четырех прямоугольников. У двух из них стороны a и a*sqrt(2)/2, у двух других - 2a и a*sqrt(2)/2.
Sб = (a+a+2a+2a)*a*sqrt(2)/2 = 3*(a^2)*sqrt(2)

* Площадь полной поверхности равна Sб + 2*(площадь основания)
Sп = 3*(a^2)*sqrt(2)+2*(2*a)*a*sqrt(2)/2 = 5*(a^2)*sqrt(2).

3537. Lexxus, 24 апреля 2011, 19:16:16
3533. Galya, 23 апреля 2011, 19:30:15
1.Основание пирамиды прямоугольник со сторонами 6 и 8 см.Высота пирамиды 12 и проходит через точку пересечения диагоналей основания. Найти боковые ребра пирамиды.

Если высота пирамиды проходит через точку пересечения диагоналей основания, то пирамида прямая, и все её боковые рёбра равны между собой.

Пусть основание пирамиды - ABCD, E - вершина пирамиды, а P - точка пересечения диагоналей основания.

AC = sqrt(8^2+6^2) = 10, AP = AC/2 = 5.

Из прямоугольного треугольника APE: AE = sqrt(5^2+12^2) = 13.

3538. Lexxus, 24 апреля 2011, 20:23:01
3533. Galya, 23 апреля 2011, 19:30:15
DABC-правильная треугольная пирамида. Строна основания три корня из трёх. Боковое ребро 5, MC медиана треугольника ABC. Найти S треугольника MDC.

В правильной треугольной пирамиде основание - равносторонний треугольник. Значит, MC - высота и биссектриса.
Значит
MC = BC*sin(60 градусов) = 3*sqrt(3)*sqrt(3)/2 = 9/2.
DM = sqrt(DB^2-BM^2) sqrt(5^2-(3*sqrt(3)/2)^2) = sqrt(73)/2.
DC = 5;

А дальше - например, по формуле Герона:
Полупериметр p = (MC+DM+DC)/2

S = sqrt(p*(p-MC)*(p-DM)*(p-DC))

Но без калькулятора такую штуку считать - с ума сойдешь.
Можно иначе.

Пусть DP - высота пирамиды. Точка P - точка пересечения медиан/биссектрис/высот треугольника ABC, и мы знаем, что она делит их в отношении 2:1.
То есть, PC = MC*2/3 = 9/2*2/3 = 3.

Значит, высота пирамиды
DP = sqrt(DC^2-PC^2) = 4

Площадь треугольника MDC равна MC*DP/2 = 9/2*4/2 = 9.

3539. vera, 25 апреля 2011, 17:26:53
Прямая y=2x+6 является касательной к графику функции y=x^3+x^2+2x+6. Найдите абсциссу точки касания. помогите разобраться.спасибо.

3540. Дина, 25 апреля 2011, 18:30:55
Помогите пожалуйста решить задачку, сама её решила, но неуверена что правильно.Найдите объём цилиндра, площадь основания которого равна 1, а образующая равна 6 и наклонена к плоскости под углом 30 градусов.

3541. Юлия, 25 апреля 2011, 19:43:18
вычислить 2sin5xcos3x-sin8x , если sinx+cosx=корень из 0.6

помогите пожалуйста (

3542. Максим, 25 апреля 2011, 20:52:29
1)В прямоугольном параллелепипеде измерения равны 5, 7 и корени из 47. Наити диагональ параллелепипеда и син угла между диагональю и плоскостью его основаия
2) Точка n находится на расстоянии h от плоскости A проведены две наклонные np и mp где p и q основания наклонных , соответственно под углами 45 и 60 градусов. Наити: pq если угол poq равен 150 градусов , где О-основание перпендикуляра , то перпендикулярна А
3) дано АВСД квадрат, АБ=а, треугольник БСН тупоугольный равнобедр треугольник, угол Б равен 120 градусов, плоскость АБСД перпендикулярна плоскости БСН. наити площадь треугольника АДН
Можете помочь пожалуйста с решением этих задач

3543. Максим, 25 апреля 2011, 20:53:16
заранее спасибо)

3544. Galya, 25 апреля 2011, 22:22:57
Спасибо огромное!! Мало того, что 2 не будет, так ещё и поняла как решать эти 2 задачи)

3545. настя, 26 апреля 2011, 16:30:05
Пожалуйста помогите мне решить задачу по геометрии мне нужно прямо сейчас:ответ на задачу!:Сторона основания правильной треугольной пирамиды 3 см,а угол между боковой гранью и основанием пирамиды равен 45 градусам.Найдите площадь полной поверхности пирамиды???

3546. Кристинка, 26 апреля 2011, 19:40:21
Решите пожалуйста до завтра! Умоляю!Очень нужно!
Правильная четырехугольная призма пересечена плоскостью, содержащей две ее диагонали. Площадь полученно¬го сечения равна 60 см2, а сторона основания равна 6 см. Вычислите площадь боковой поверхности призмы
Спасибо большое заранее)

3547. Ариадна, 28 апреля 2011, 17:45:32
Пожалуйста помогите решить задание С4.

Дан прямоугольный треугольник АВС с прямым углом при вершине В и углом (альфа) при вершине А. Точка D - середина гипотенузы. Точка С1 симметрична точке С относительно прямой ВD. Найдите угол АС1В.

3548. Дарья, 28 апреля 2011, 17:51:06
В розетку электросети подключены приборы, общее сопротивление которых составляет R=60 Ом. Параллельно с ними в розетку предполагается подключить электрообогреватель. Определите (в омах) наименьшее возможное сопротивление Ry этого электрообогревателя, если известно, что при параллельном соединении двух проводников с сопротивлениями Rx и Ry их общее сопротивление даётся формулой R= , а для нормального функционирования электросети, общее сопротивление в ней должно быть не меньше 35 Ом.
НАПИШИТЕ ПОЖАЛУЙСТА РЕШЕНИЕ. НЕ МОГУ ПОНЯТЬ КАК РЕШАТЬ(

3549. Алёнка, 28 апреля 2011, 18:54:23
Решите, пожалуйста, кто-нибудь!!!

(cosx+sqrt(2)/2)(tg(x-П/4)-1)=0

сколько корней на отрезке [0;2П]
ответ должен быть "3".

3550. Lexxus, 28 апреля 2011, 20:16:08
3549. Алёнка, 28 апреля 2011, 18:54:23

(cosx+sqrt(2)/2)(tg(x-П/4)-1)=0

сколько корней на отрезке [0;2П]


Это уравнение равносильно совокупности:

1. система
cos(x)+sqrt(2)/2 = 0
x-pi/4 не равно pi/2+pi*n

x = (+/-)3*pi/4 + 2*pi*n
x не равно 3*pi/4 + pi*n

откуда
x = -3*pi/4 + 2*pi*n

2. уравнение
tg(x - pi/4) = 1
x - pi/4 = pi/4 + pi*n

x = pi/2 + pi*n

Значит, все корни уравнения:
x = -3*pi/4 + 2*pi*n, x = pi/2 + pi*n

На отрезке [0,2*pi] будет три корня: pi/2, 5*pi/4 и 3*pi/2.

3551. Lexxus, 29 апреля 2011, 00:36:17
3547. Ариадна, 28 апреля 2011, 17:45:32
Дан прямоугольный треугольник АВС с прямым углом при вершине В и углом (альфа) при вершине А. Точка D - середина гипотенузы. Точка С1 симметрична точке С относительно прямой ВD. Найдите угол АС1В.

Здесь у нас может быть два случая: alpha меньше 45 градусов и alpha больше 45 градусов:



Рассуждения, справедливые для обоих случаев:

BD = AC/2 = AD = CD.
Поскольку точка С1 симметрична точке С относительно прямой ВD, то СE = C1E и СС1 перпендикулярна BD. Значит, DC=DC1, BC = BC1, треугольники DCB и DC1B равны.
Углы DCB = CBD = DC1B = DBC1 = pi/2 - alpha.
Углы CDB = C1DB = pi - 2(pi/2 - alpha) = 2*alpha.
Из прямоугольного треугольника BEC1, угол BC1E = pi/2 - (pi/2 -alpha) = alpha.

DC1 = DC = AD, следовательно, треугольник ADC1 - равнобедренный. Угол при его вершине
ADC1 = pi - CDB - C1DB = pi - 4*alpha.
А углы при основании
DAC1 = DC1A = (pi - ADC1)/2 = 2*alpha.

Соответственные углы DAC1 = CDB, следовательно, прямые DB и AC1 параллельны. Следовательно, углы CED = AC1E = pi/2.

Осталось только заметить, что в первом случае
Угол AC1B = AC1E + BC1E = pi/2 + alpha

Во втором случае
Угол AC1B = AC1E - BC1E = pi/2 - alpha

А при alpha = 45° задача не имеет смысла, поскольку точки A и С1 совпадают.

3552. Я, 29 апреля 2011, 15:49:45
Помогите кто-нибудь, пожалуйста, решить с2

дана треугольная правильная призма lmnl1m1n1. ребро основания и высота равны 2sqrt(5). найдите расстояние от точки l1 до плоскости lm1t, где t - середина l1n1

3553. Lexxus, 29 апреля 2011, 23:44:50
3552. Я, 29 апреля 2011, 15:49:45

дана треугольная правильная призма lmnl1m1n1. ребро основания и высота равны 2sqrt(5). найдите расстояние от точки l1 до плоскости lm1t, где t - середина l1n1

Расстояние от точки L1 до плоскости LM1T будет равно длине перпендикуляра L1H, опущенного из точки L1 на LT.

L1H найдём из прямоугольного треугольника LL1T, зная, что LL1 = 2sqrt(5), L1T = sqrt(5).
LT = sqrt(5+4*5) = 5.

Прямоугольные треугольники LL1T и L1HT подобны по двум углам, следовательно,
L1H/L1T = LL1/LT
L1H = sqrt(5)*2*sqrt(5)/5 = 2

Ответ: 2

3554. Алёнка, 30 апреля 2011, 16:21:55
помогите, пожалуйста, решить задания из варианта экзаменационной работы!

В4: в треугольнике АВС АС=ВС=10, cosА=0,6. Найдите S тр.АВС.
В7: 9^(ln6/ln3)
B9: В окружность основания цилиндра вписан правильный треугольник. Найдите объём пирамиды той же высоты, что и цилиндр, в основании которого лежит этот треугольник, если объём цилиндра равен Пsqrt(3)
B10: Для одного из предприятий зависимость объёма спроса на продукцию q(единиц в месяц) от её цены р(тыс. руб) задаётся формулой q=120-10р. Определите максим. уровень цены р(тыс. руб), при котором значение выручки предприятия за месяц r=q*p составляет не менее 320 тыс. руб.
В11: Найдите наименьшее значение функции y=(x-7)e^(x-6) на отрезке [1;7]
С1: решите систему уравнений
2y+3cosx=0
(ln(cosx)+1)(y-1)=0
C2: В основании прямого параллелепиеда ABCDA1B1C1D1 лежит ромб ABCD, <ACA1=arcctg3, <DBD1=arcctg4, CC1=2. Найдите площадь боковой поверхности параллелепипеда.

помогите, пожалуйста!!! :)

3555. Леонид , 30 апреля 2011, 18:52:42
Попробовать решить просьба. Задача с досрочного ЕГЭ 2011. C4. Радиус окружности, вписанной в треугольник АВС,
площадь которого равна 210, в три раза меньше высоты,
проведенной из вершины А . Известно, что  ВС = 28.
Найдите сторону АС.  

3556. Lexxus, 30 апреля 2011, 23:11:47
3555. Леонид , 30 апреля 2011, 18:52:42
Радиус окружности, вписанной в треугольник АВС,
площадь которого равна 210, в три раза меньше высоты,
проведенной из вершины А . Известно, что ВС = 28.
Найдите сторону АС.

Мы знаем, что радиус вписанной окружности равен площади треугольника, поделенной на его полупериметр: r = S/p.
В свою очередь, площадь треугольника равна BC*h/2 (h - высота).

S = 28*h/2 = 14*h = 42*r = 42*S/p.
1 = 42/p.

Отсюда полупериметр p = 42.

Подставляем известные значения BC и p в формулу Герона:

sqrt(42*14*(42-x)*(42-y)) = 210 (x и y - это две неизвестные стороны треугольника).

Отсюда (42-x)*(42-y) = 75.
И ещё нам известен полупериметр: (x+y+28)/2 = 42.

Решаем эту систему из двух уравнений, получаем
{x = 17, y = 39} или {x = 39, y = 17}.

Ответ: 17 или 39

3557. Lexxus, 30 апреля 2011, 23:19:11
3554. Алёнка, 30 апреля 2011, 16:21:55
решите систему уравнений
2y+3cosx=0
(ln(cosx)+1)(y-1)=0

Эта система равносильна совокупности из двух систем:

1. {2y+3cos(x)=0, y-1=0, cos(x) > 0}
y = 1;
2+3cos(x) = 0;
cos(x) = -2/3 < 0, следовательно, у этой системы нет решений.

2. {2y+3cos(x)=0, ln(cos(x))+1 = 0}
ln(cos(x)) = -1
cos(x) = 1/e
x = ±arccos(1/e) + 2*pi*n.

2y+3/e = 0
y = -3e/2

Ответ:
{y = -3e/2; x = ±arccos(1/e) + 2*pi*n}

3558. Леонид , 1 мая 2011, 05:04:41
Уважаемый LEXXUS! Огромное спасибо за решение С4.

3559. Кристина, 1 мая 2011, 10:46:38
Помогите пожалуйста!срочно надо:( по плану первой бригаде нужно изготовить на 720 изделий больше,чем второй. Так как в первую бригаду добавили 3 человека,а во вторую-2 человека, то обе бригады выполнили план на 2 дня раньше срока, причем каждый рабочий изготавливал в день по 18 изделий. Найдите, сколько рабочих по плану должны были работать в обеих бригадах вместе?

3560. Кристина, 1 мая 2011, 11:02:22
И еще пожалуйста две задачки... 1.в тетраэдре АВСТ ребра АТ и СТ делятся соответственно точками М и N в отношениях 3:4 и 2:5. Найдите отношение объемов многогранников МNВТ и АСВТ. И ВТОРАЯ... 2. В прямоугольном треугольнике АВС Из вершины прямого угла С проведена высота СН равная 6, отрезок АН равен 3. В треугольнике АНС проведена биссектриса НЕ,а В треугольнике СНВ проведена биссектриса угла Н-НD.определите длину ED

3561. Алёнка, 1 мая 2011, 20:03:19
Lexxus, большое спасибо за решение!
А не могли бы вы помочь с другими заданиями, пожалуйста! :)

3562. Lexxus, 1 мая 2011, 20:42:45
3561. Алёнка, 1 мая 2011, 20:03:19
А не могли бы вы помочь с другими заданиями, пожалуйста! :)

Мне скучно разжёвывать решения простых заданий. Но тут бывают и те, кому не скучно.

3563. Энди, 1 мая 2011, 22:00:17
Алёнка, 30 апреля 2011, 16:21:55
B10: Для одного из предприятий зависимость объёма спроса на продукцию q(единиц в месяц) от её цены р(тыс. руб) задаётся формулой q=120-10р. Определите максим. уровень цены р(тыс. руб), при котором значение выручки предприятия за месяц r=q*p составляет не менее 320 тыс. руб.

q=120-10p; r=q*p
Подставляем q, получаем : r=(120-10p)p. Если выручка должна составлять не менее, значит она должна быть больше или равна(>=) 320 тыс. Тогда
(120-10p)p>=320
10p^2 -120p-320>=0
p^2-12p+32<=0
D=16
p=8 или р=4
Ну а максим.значение в тысячах я думаю не сложно выбрать: 8000

3564. лена, 2 мая 2011, 14:51:42
Стороны основания прямого параллелепипеда равны 3 и 5 см., угол между ними равен 60. Большая диагональ параллелепипеда равна 10 см. Найдите боковое ребро. помогите решить)

3565. лена, 2 мая 2011, 14:57:11
сторона основания правильной четырехугольной пирамиды равна а. найти полную поверхность пирамиды...
сегодня очень надо(((

3566. Алёнка, 2 мая 2011, 16:32:09
B9: В окружность основания цилиндра вписан правильный треугольник. Найдите объём пирамиды той же высоты, что и цилиндр, в основании которого лежит этот треугольник, если объём цилиндра равен Пsqrt(3)
В11: Найдите наименьшее значение функции y=(x-7)e^(x-6) на отрезке [1;7]
C2: В основании прямого параллелепиеда ABCDA1B1C1D1 лежит ромб ABCD, <ACA1=arcctg3, <DBD1=arcctg4, CC1=2. Найдите площадь боковой поверхности параллелепипеда.

Решите, пожалуйста кто сможет!

3567. Лилия, 2 мая 2011, 19:26:17
Помогите, пожалуйста :)
Здесь уже была представлена данная задача, правда, в ней было пропущено число.

Основанием прямого параллелепипеда ABCDA1B1C1D1 является параллелограмм ABCD, стороны которого равны а*корней из 2-х* и 2a, острый угол равен 45°. Высота параллелепипеда равна меньшей высоте параллелограмма. Найдите:
*
меньшую высоту параллелограмма;
*
угол между плоскостью ABC1 и плоскостью основания;
*
площадь боковой поверхности параллелепипеда;
*
площадь поверхности параллелепипеда.

3568. Александр, 2 мая 2011, 19:59:49
Задание B9 (25649)
(показов: 1057, ответов: 233)

Найдите площадь поверхности многогранника, изображенного на рисунке (все двугранные углы прямые).

http://live.mephist.ru/tests/mathege2010-3/GetPicture__picId-27287.png
b9.119

У меня вопрос по этой задаче.
Как 20 -4=54?

3569. Дмитрий, 3 мая 2011, 20:33:17
Баржа в 10:00 вышла из пункта А в пункт В, расположенный в 15 км от А. Пробыв в пункте В 45 минут, баржа отправилась назад и вернулась в пункт А в 16:00. Определите (в км/час) скорость течения реки, если известно, что собственная скорость баржи равна 7 км/ч.
Помогите решить пожалуйста!!

3570. Леонид , 3 мая 2011, 21:46:44
в движении баржа была 16.00 - 10.00= 6ч без 45 мин - 3/4 часа, т.е. 5 1/4 часа или 21/4 часа. время по течению реки 15/(х+7), против течения 15/(х-7). Уравнение: 15/(х+7) + 15/(х-7)= 21/4. К общему знаменателю 60(х-7) + 60(х+7)=21(х-7)(х+7) - раскрыть скобки, решить кв. ур-е. х- собственная скорость баржи.

3571. крячков, 4 мая 2011, 10:37:04
cложное задание с5

3572. Елена, 4 мая 2011, 12:57:23
Здравствуйте.Помогите пожалуйста.Совсем запуталась в задаче,не могу решить:( Основание пирамиды- ромб со стороной 10см и высотой 6см. Найдите объем пирамиды, если все двугранные углы при ее основании равны 45 градусов.

3573. Лили, 4 мая 2011, 15:14:26
камень брошен вниз с высоты 12 м
высота h на которой находится камень во время падения зависит от времени t
h(t) = 12 - 4t - 5t^2
сколько секунд камень будет падать?

3574. ГуЗя, 4 мая 2011, 16:29:07
ПриветикИ)))помогите решить ,пожалуйста!!!))))
Компания "Альфа" начала инвестировать средства в перспективную отрасль в 2001 году ,имея капитал в размере 5000 долларов.Каждый год, начиная с 2002 года,она получала прибыль, которая составляла 200% от капитала предыдущего года. А компания "Бета" начала инвестировать средства в другую отрасль в 2003 году, имея капитал в размере 10000 долларов., и начиная с 2004 года, ежегодно получала прибыль, составляющую 400% от предыдущего года.На сколько долларов капитал одной из компаний был больше капитала другой к концу 2006 года, если прибыль из оборота не изымалась????
заранее большое спасибо))))

3575. лена, 4 мая 2011, 17:28:57
в равностороннем треугольнике ABC проведена биссектриса AD. расстояние от точки до прямой AC равно 6см. найдите расстояние от вершины A до прямой BC

3576. Элина, 4 мая 2011, 18:10:33
4 Луч NK делит развернутый угол ANB на два угла ANK и KNB. Найдите градусную меру этих углов, если угол ANK больше угла KNB в 1,4 раза.

3577. Альберт, 4 мая 2011, 18:32:30
Помогите решить пожалуйста!!!))))
tg3xcosx + 3sinx = 0

3578. анастасия, 4 мая 2011, 23:37:12
окружность в точке О касается сторон АВ и ВС треугольника АВС в точке К и М соответственно так,что АК=5 см,СМ=7 см.при этом центр окружности лежит на стороне АС и делит ее в отношении 4:5.найдите стороны АВ и ВС

3579. Авеста, 5 мая 2011, 13:24:04
Не знаю решение уравнения 2(z+1)=2x2+y2
Определите тип поверхности, сделайте чертеж, нарисуйте сечение повехности с координатными плоскостями, найдите фокусы директрисы и асимптоты полученных кривых . Выясните по одну или по разные стороны от поверхности находятся точки А и В, и сколько точек с поверхностью имеет прямая АВ.
Помогите пожалуйста!



3580. Александра, 5 мая 2011, 13:24:35
Здравствуйте!
помогите пожалуйста найти промежутки убывания функции
g(x) = -1 -9x - 13x5 + 4cosx

3581. ВладимирS, 5 мая 2011, 15:46:42
Б9
Во сколько раз увеличится площадь пирамиды, если все ее стороны увеличить в 6 раз

3582. Эльвинка=), 5 мая 2011, 17:16:41
в правильной треугольной пирамиде сторона основания равна 4 корень из 3 см, а плоский угол при вершине пирамиды равен 90 градусам. Найдите высоту пирамиls// что и из чего найти...у мя ответ получается два корень из 5 ...чет такого ответа нет...

3583. Виктория, 5 мая 2011, 19:27:16
Основание пирамиды – прямоугольник со сторонами 6см и 8 см. Высота пирамиды равна 12 см и проходит через точку пересечения диагоналей основания. Найдите боковые ребра пирамиды.

3584. дмитрий, 5 мая 2011, 20:33:48
кубик весит 10 г. сколько граммов будет весить кубик, ребро которого в 3 раза больше чем ребро первого кубика если оба кубика изготовлены из одинакового материала?

3585. дмитрий, 5 мая 2011, 20:36:49
кубик весит 800 г. сколько граммов будет весить кубик ребро которого в 2 раза меньше чем ребро первого кубика если оба кубика изготовлены из одинакового материала?

3586. Леонид , 5 мая 2011, 20:50:44
ребро 1-го кубика а, его обём а^3 =800. (a/2)^3=800/8=100

3587. Артём, 5 мая 2011, 23:05:20
Основание прямой призмы прямоугольный треугольник с катетами 6 и 8 см. Найдите площадь боковой поверхности призмы если её наибольшая грань - квадрат.

3588. Гуля, 5 мая 2011, 23:22:44
В правильной шестиугольной призме ABCDEF A1B1C1D1E1F1, все ребра которой
равны 1, найдите расстояние от точки C до прямой A1B1.


3589. коля, 5 мая 2011, 23:49:18
В ТРЕУГОЛЬНИКЕ АВС УГОЛ С=90,СОS А=0,55,НАЙТИ SIN В???
ПОМОГИТЕ ПЛИЗ!!!
ОЧЕНЬ НУЖНО(((

3590. Леонид , 6 мая 2011, 05:53:41
Треугольник прямоугольный, углы А и В дополняют друг друга до 90. синус угла В 0,55

3591. танюся, 6 мая 2011, 11:35:00
а мне не ответили.. :(((((((((((((((((((((((((((((((((((((((((((((((((((((((((((((((((((((((((((((((((((((((((((((((((((((((((((((((((((((

3592. Иришка, 6 мая 2011, 16:20:23
Сторона правильной треугольной пирамиды равна 3 см а угол между боковой гранью и основанием пирамиды равен 45 градусов. найдите площадь полной поверхности пирамиды.

буду очень признательна.желательно как можно быстрее

3593. Дина, 6 мая 2011, 21:32:29
Помогите, пожалуйста, решить систему уравнений!!!!!!!! Очень надо!!!!!!!!!!!
{x^(x-2y)=36
{4(x-2y)+log_6(X)=9

3594. Леонид , 7 мая 2011, 17:12:10
4 не встепени ? Условие не понятно.

3595. диана, 7 мая 2011, 20:53:40
кто здесь химию шарит?????отзовитесь...

3596. Илья, 8 мая 2011, 00:37:01
x^(x-2y)=36 => x-2y=log_36(x)=1/2 * log_6(x). Подставляем во вторую строчку.

2*log_6(x)+log_6(x)=9 => log_6(x)=3 => x=216, x-2y = 1/3 => y=x/2 - 1/6= 107 5/6

Ответ: 216, 107 5/6

3597. Татьяна, 8 мая 2011, 15:58:03
помогите пожалуйста решить:
1. Основание пирамиды MABCD является квадрат ABCD. Ребро MD перпендикулярно к плоскости основания. AD =DM =а. Найдите S поверхности пирамиды.


Основанием прямого параллелепипеда является параллелограмм ABCD, стороны которого равны и 2a, острый угол равен 45°. Высота параллелепипеда равна меньшей высоте параллелограмма. Найдите:
*
меньшую высоту параллелограмма;
*
угол между плоскостью ABC1 и плоскостью основания;
*
площадь боковой поверхности параллелепипеда;
*
площадь поверхности параллелепипеда.

3598. Lexxus, 8 мая 2011, 17:45:05
3572. Елена, 4 мая 2011, 12:57:23
Основание пирамиды- ромб со стороной 10см и высотой 6см. Найдите объем пирамиды, если все двугранные углы при ее основании равны 45 градусов.

Если все двугранные углы при основании равны, то пирамида прямая.
Назовём ей ABCDE (E - вершина).
Пусть O - точка пересечения диагоналей ромба, а OH - высота, опущенная из точки O к одной из его сторон. OH = высота ромба пополам = 3.

В треугольнике ЕOH угол EOH равен 90 градусов, угол EHO по условию равен 45 градусов. Значит, этот треугольник прямоугольный и равнобедренный, и высота пирамиды EO = OH = 3.

Объем пирамиды равен 1/3*EO*(сторона ромба * высота ромба) =
= 1/3 * 3 * 10 * 6 = 60

Ответ: 60 куб. см.

3599. Lexxus, 8 мая 2011, 18:00:03
3564. лена, 2 мая 2011, 14:51:42
Стороны основания прямого параллелепипеда равны 3 и 5 см., угол между ними равен 60. Большая диагональ параллелепипеда равна 10 см. Найдите боковое ребро.

Пусть параллелепипед зовётся ABCDA1B1C1D1.
Боковое ребро CC1 можно найти из прямоугольного треугольника ACC1, в котором мы пока знаем только гипотенузу AC1 = 10.
Катет AC можно найти, например, как длину суммы векторов AB и AD, выходящих из одной точки:
AC = sqrt(AB^2+AD^2+2*AB*AD*cos(60 градусов)) =
= sqrt(9+25+15) = sqrt(49) = 7.

Боковое ребро параллелепипеда
CC1 = sqrt(AC1^2-AC^2) = sqrt(100-49) = sqrt(51)

Ответ: sqrt(51) см

3600. Lexxus, 8 мая 2011, 18:11:28
3573. Лили, 4 мая 2011, 15:14:26
камень брошен вниз с высоты 12 м
высота h на которой находится камень во время падения зависит от времени t
h(t) = 12 - 4t - 5t^2
сколько секунд камень будет падать?

Решаем квадратное уравнение 12 - 4t - 5t^2 = 0, выбираем положительный корень.

3601. Lexxus, 8 мая 2011, 18:21:07
3575. лена, 4 мая 2011, 17:28:57
в равностороннем треугольнике ABC проведена биссектриса AD. расстояние от точки до прямой AC равно 6см. найдите расстояние от вершины A до прямой BC

Если я правильно понял, шести сантиметрам равно расстояние от точки D до прямой AC. В таком случае:

В равностороннем треугольнике биссектриса - это ещё и медиана, и высота. Значит, расстояние от вершины A до прямой BC - и есть длина биссектрисы AD.

Пусть DH - высота, опущенная из точки D на прямую AC.
Прямоугольные треугольники ADC и AHD подобны по двум углам (общий угол DAH). Значит, угол ADH равен углу ACD и равен 60 градусов (как и любой угол в равностороннем треугольнике).
AD = DH/cos(60 градусов) = 6*2 = 12

Ответ: 12 см

3602. Lexxus, 8 мая 2011, 18:24:16
3576. Элина, 4 мая 2011, 18:10:33
Луч NK делит развернутый угол ANB на два угла ANK и KNB. Найдите градусную меру этих углов, если угол ANK больше угла KNB в 1,4 раза.

x - градусная мера угла ANK.
y - градусная мера угла KNB.

Решаем систему уравнений:
x+y = 180
x = 1.4y

3603. Lexxus, 8 мая 2011, 19:09:40
3577. Альберт, 4 мая 2011, 18:32:30
tg3xcosx + 3sinx = 0

Если cos(x) = 0, то sin(x) равен либо 1, либо -1, что при подстановке в уравнение не приводит ни к чему хорошему. Значит, на cos(x) можно всё поделить:

tg(3x) = -3*tg(x).

Воспользуемся формулой тангенса тройного угла. Если она нам незнакома, то выведем её из формул тангенса суммы и тангенса двойного угла:

tg(3x) = (3*tg(x)-(tg(x))^3)/(1-3*(tg(x))^2)

заменим tg(x) на t:

(3t-t^3)/(1-3*t^2) = -3*t

3t - t^3 = -3t + 9t^3
10t^3-6t = 0
t*(5t^2-3) = 0
t = 0; t^2 = 3/5
Три корня:
t1 = 0; t2=sqrt(3/5); t3=-sqrt(3/5)

Значит,
x = pi*n
x = arctg(sqrt(3/5))+pi*n;
x = arctg(-sqrt(3/5))+pi*n = -arctg(sqrt(3/5))+pi*n

3604. Lexxus, 8 мая 2011, 20:44:16
3578. анастасия, 4 мая 2011, 23:37:12
Окружность с центром в точке О касается сторон АВ и ВС треугольника АВС в точке К и М соответственно так,что АК=5 см,СМ=7 см.при этом центр окружности лежит на стороне АС и делит ее в отношении 4:5. Найдите стороны АВ и ВС


Рассмотрим треугольники ABO и CBO.
Их площади:

S(ABO) = 1/2*AO*BH = 1/2*AB*R
S(CBO) = 1/2*CO*BH = 1/2*BC*R

Отсюда AB/BC = S(ABO)/S(CBO) = AO/CO = 4/5

Обозначив KB=MB=x, получим
(x+5)/(x+7) = 4/5
5x+25 = 4x+28
x = 3

Значит, AB = x+5 = 8; BC = x+7 = 10

Ответ: AB = 8 см, BC = 10 см.

3605. Lexxus, 8 мая 2011, 21:06:58
3579. Авеста, 5 мая 2011, 13:24:04
Не знаю решение уравнения 2(z+1)=2x2+y2
Определите тип поверхности, сделайте чертеж, нарисуйте сечение повехности с координатными плоскостями, найдите фокусы директрисы и асимптоты полученных кривых . Выясните по одну или по разные стороны от поверхности находятся точки А и В, и сколько точек с поверхностью имеет прямая АВ.
Помогите пожалуйста!

А рожа не треснет?
Вот когда эллиптические параболоиды и прочие поверхности второго порядка войдут в школьную программу, тогда пожалуйста.

3606. Lexxus, 8 мая 2011, 21:20:46
3582. Эльвинка=), 5 мая 2011, 17:16:41
в правильной треугольной пирамиде сторона основания равна 4 корень из 3 см, а плоский угол при вершине пирамиды равен 90 градусам. Найдите высоту пирамиды

Пирамида - ABCS.
O - проекция точки S на плоскость ABC.
BH - высота (биссектриса, медиана) основания.

Из прямоугольного треугольника ASB: SB = 4*sqrt(3)/sqrt(2)
Из прямоугольного треугольника BHA: BH = 4*sqrt(3)*sqrt(3)/2 = 6
Точка O делит BH в отношении 2:1 (BO=2*OH), то есть BO = BH*2/3 = 4.
Из прямоугольного треугольника SOB: SO = sqrt(SB^2-BO^2) = sqrt(8) = 2*sqrt(2).

Ответ: 2*sqrt(2) см

3607. Fely, 9 мая 2011, 09:52:37
Помогите пожалуйста!

Плоские углы трехгранного угла равны 60 60 90. На его ребрах от вершины отложены равные отрезки ОА ОВ ОС. Найдите двугранный угол между плоскостью угла в 90 и плоскость АВС.

3608. Омар, 9 мая 2011, 12:25:15
Люди,а с 6 реально решить?кто умеет-напишите мне на почту)может и меня научите)заранее спасибо)

3609. DecemberFlower, 9 мая 2011, 18:20:30
Здравствуйте!
Уравнение
(81^cosx-4*(9^cosx)+3) / tgx+sqrt(3) = 0
Мои несчастные мозги вроде бы поняли, как его решать. Вопрос про отбор корней: в ответе написано pi/3 + 2pi*n. Буду благодарна, если кто-нибудь объяснит, куда дели 5pi/3.
Да, возможно, это очень тупой вопрос, но мне правда непонятно=(
Спасибо.

3610. оля, 9 мая 2011, 18:25:31
дано :треугольник АВС ,АС=ВС .СD-медиана,угол В=а,СD=h.Наидите АВ

3611. Fely, 9 мая 2011, 19:06:47
И вот еще, если не затруднит)
Каждый плоский угол трехгранного угла равен 60. На одном из его ребер отложен от вершины отрезок, равный 3 см, и из его конца опущен перпендикуляр на противоположную грань. Найдите длину этого перпендикуляра.

3612. DecemberFlower, 9 мая 2011, 21:18:12
3609. DecemberFlower, 9 мая 2011, 18:20:30
Здравствуйте!
Уравнение
(81^cosx-4*(9^cosx)+3) / tgx+sqrt(3) = 0
Мои несчастные мозги вроде бы поняли, как его решать. Вопрос про отбор корней: в ответе написано pi/3 + 2pi*n. Буду благодарна, если кто-нибудь объяснит, куда дели 5pi/3.
Да, возможно, это очень тупой вопрос, но мне правда непонятно=(
Спасибо.


После отправки своего вопроса на меня пришло озарение и я поняла, что у точки 5pi/3 тангенс равен -sqr(3), который обращает знаменатель в 0! Поэтому единственным решением этого уравнения будет pi/3 + 2pi*n.
Спасибо=)




3613. Lexxus, 10 мая 2011, 01:31:48
3612. DecemberFlower, 9 мая 2011, 21:18:12
После отправки своего вопроса на меня пришло озарение и я поняла, что у точки 5pi/3 тангенс равен -sqr(3), который обращает знаменатель в 0! Поэтому единственным решением этого уравнения будет pi/3 + 2pi*n.
Спасибо=)

Всегда пожалуйста :)

Ребят, помогите пожалуйста, я очень сильно туплю в алгебре, поэтому чему равен log8? а? и вообще как его найти то можно...(о ф-лах log я знаю).
спасибо)

3615. Альберт, 10 мая 2011, 14:27:13
Lexxus СПАСИБО ОГРОМНОЕ!!!!!

3616. Александр, 11 мая 2011, 16:38:12
Помогите решить 2 задачи по геометрии плиз!!!!!

Вот они
1:Стороны треугольника равны 29см, 25см, и 6 см.Из вершины меньшего угла треугольника восставлен перпендикуляр. равный 8 корней из 6 . Найти расстояние от конца перпендикуляра до меньшей стороны треугольника.


2:Концы данного отрезка длиной 15см находятся от плоскости на расстоянии 7 см и 16 см. Найти проекцию этого отрезка на данную плоскость.

Зарание спасибо !!!!

3617. Виталий, 11 мая 2011, 17:04:34
http://live.mephist.ru/show/mathege2010/search/%ED%E0%E8%E1%EE%EB%FC%F8%E5%E5+%E7%ED%E0%F7%E5%ED%E8%E5/100/
Задача 4625. Помогите решить пожалуйста и напишите, кто может ход решения. Заранее благодарю.

3618. rusalka93, 11 мая 2011, 20:37:04
помоги пожалуйста!
1)плоскость,перпендикулярная диаметру шара ,делит его на отрезки части 3 см и 9 см .Найти объем шара?
2)Образующая конуса равна 6 см,а угол между вершинами осевого сечения 60 градусов. Найти объем конуса?
3) В кубе АВСДА1В1С1Д1 ,точка Е середина А1В1. Найти косинус угла между АЕ и ВД1?

3619. Анютка^^, 11 мая 2011, 20:41:04
Помогите!!!!! Четыре рубашки дешевле кукртки на 20%. На сколько шесть рубашек дороже куртки? Это задача В12!!!

3620. Леонид , 11 мая 2011, 22:08:05
3617. Виталий, 11 мая 2011, 17:04:34
Найди второй катет по теореме Пифагора и тангенс угла по определению - отншение противолежащего катета к прилежащему.

3621. Орлия, 11 мая 2011, 22:10:51
Если не ошибаюсь то 6 рубашек будут дороже куртки на 20%.

3622. Дэна, 11 мая 2011, 22:29:53
Помогите пожалуйста!!!!!!!!!
1) ABCDA1B1C1D1-куб. Найти угол между прямой AB1 и плоскостью ABC1.
2) Ребра AD и BC пирамиды DABC равны 24 см и 10 см. Расстояние между серединами BD и AC равно 13 см. Найти угол между прямыми AD и BC.
3) Основание прямой призмы ABCA1B1C1- треугольник ABC, в котором AB=BC=20, AC=32. Боковое ребро призмы 24. Точка P лежит на ребре BB1 так, что BP:PB1=1:3. Найти тангенс угла между плоскостями A1B1C1 и ACP.


3623. лариса, 12 мая 2011, 10:41:36
срочно!!! помогите найти корень уравнения!!!! (1/6) в степени 6-х = 36

3624. Гу, 12 мая 2011, 12:03:16
В треугольнике АВС угол А равен 90 градусам, синус С равен корень из 11 делёное на 6, АС=5. Найти ВС




помогите пожалуйста) оч надо)

3625. ilnaz, 12 мая 2011, 16:12:28
помогите решить
8sin(в квад)2x+cos2x+1 / корень -sinx

3626. Blood_Mary, 12 мая 2011, 17:41:21
Помогите мне решить задачу:
В правильной 6-угольной пирамиде SABCDEF проведено сечение SBE, являющееся правильным треугольником. Найти угол между плоскостью SAC и плоскостью основания пирамиды.

Заранее огромное спасибо)

3627. Kitty, 12 мая 2011, 18:51:28
сроооооооооочно нужно,помогите пожалуйста решить задачу
в правильной треугольной пирамиде высота 4,а гол между боковым ребром и плоскостью основания равен 45градусов. найти объем пирамиды

заранее спааасибо

3628. Ural, 12 мая 2011, 21:10:46
C2. Основанием пирамиды является ромб АВСД, причем АВ=3. Боковые ребра АМ=4,ВМ=5,СМ=2корень из7, ДМ=5. Найдите периметр трапеции АВЕF, если известно что Е принадлежит СМ, F принадлежит ДМ, AF=FD=2,5.
Помиогите,пожалуйста.

3629. Aлександр, 12 мая 2011, 21:27:26
разви нельзя сделать так чтобы в школе сдавали обычный экзамены (как раньше) ??? чтоб получить аттестат! (что я закончил 11 кл.) а егэ сдавалибы в вузах при поступлений.(в школе проста готовили к егэ)

3630. Илья, 12 мая 2011, 22:29:03
Такая вот задача из С6. Печалит она меня... Как решать-то ? )

Число P состоит из 11 разных натуральных чисел. Какое наименьшее число различных натуральных делителей имеет это число, включая 1 и само число P.

3631. ряхимя умярова, 12 мая 2011, 22:40:59
алекс ларин,егэ,с2.основание правильной четырехугольной призмыАВСДАодинВодинСодинДодин-прямоугольникА ВСД,в котором А В=5бАД корень из33,найти тангенс угла между плоскостью грани ААодинДодинД призмы и плоскостью проходящей через серединуредра СД перпендикулярно прямой ВодинДбААодин равно корень из 3.спасибо

3632. алена, 12 мая 2011, 23:52:52
основание пирамиды-прямоугольник со сторонами 6 см и 8 см.высота пирамиды равна 12 см и проходит чеперз точку пересечения диагоналей основания.найдите боковые ребра пирамиды

3633. Леонид , 13 мая 2011, 05:38:42
3631. ряхимя умярова !!!
Что это А В=5бАД корень из33? И это прямой ВодинДбААодин равно корень из 3.?
Напечатуй почётче

3634. Ксюша, 13 мая 2011, 13:35:57
Высота правильной четырехугольной пирамиды равна корень из 6, а боковое ребро наклонено к плоскости основания под углом 60
Найти: а) Бок. ребро пирамиды

3635. Аня, 13 мая 2011, 16:56:45
В правильной треугольной усеченной пирамиде стороны оснований 4 и 1, боковое ребро 2. Найти высоту.

3636. Игорь, 13 мая 2011, 18:23:46
Здраствуйте!
В основании прямой призмы лежит квадрат со стороной 3.Боковые ребра равны пи/4.Найдите объем цилиндра,описанного около этой призмы.

3637. Илья, 13 мая 2011, 18:29:33
Всем добрый вечер!!!
поомогите пожалуйста...
В правильной шестиугольной пирамиде SABCDEF,стороны основания которой равны 1,а боковые ребра равны 2,найдите синус угла между прямыми AL и BM,где М-середина ребра SC,L-середина ребра SB.

Заранее спасибо!

3638. Lexxus, 13 мая 2011, 22:37:41
3630. Илья, 12 мая 2011, 22:29:03
Такая вот задача из С6. Печалит она меня... Как решать-то ? )

Число P состоит из 11 разных натуральных чисел. Какое наименьшее число различных натуральных делителей имеет это число, включая 1 и само число P.

Вообще-то, задача звучит так:

[i]Число P равно произведению 11 различных натуральных чисел, больших 1. Какое наименьшее число натуральных делителей (включая единицу и само число) может иметь число P?[/i]

Вообще-то, это достаточно сложно для понимания, но я попробую объяснить.

Любое натуральное число N представимо в виде произведения
N = (p1^k1)*(p2^k2)*... и т.д.,
где p1, p2 и т.д. - простые числа, а k1, k2 и т.д. - целые неотрицательные числа.

Например,
15 = (3^1)*(5^1)
72 = 8*9 = (2^3)*(3^2)

Так вот, общее количество натуральных делителей числа N равно
(k1+1)*(k2+1)*...

Итак, по условию, P = N1*N2*...*N11, где
N1 = (p1^k[1,1])*(p2^k[1,2])*...
N2 = (p1^k[2,1])*(p2^k[2,2])*...
...,
а это значит, что
P = (p1^(k[1,1]+k[2,1]+...+k[11,1]))*(p2^(k[1,2]+k[2,2]+...+k[11,2]))*...,

и общее количество натуральных делителей числа P равно

(k[1,1]+k[2,1]+...+k[11,1]+1)*(k[1,2]+k[2,2]+...+k[11,2]+1)*...

Это выражение принимает минимальное значение, если все числа N1...N11 являются последовательными натуральными степенями одного и того же простого числа, начиная с 1.

То есть, например,
N1 = 2^1 = 2,
N2 = 2^2 = 4,
N3 = 2^3 = 8,
...
N11 = 2^11 = 2048.

Тогда количество натуральных делителей числа P равно
1+(1+2+3+...+11) = 67.

3639. ilnaz, 14 мая 2011, 13:01:50
С1
помогите решить
8sin(в квад)2x+cos2x+1 / корень из -sinx

3640. Lexxus, 14 мая 2011, 13:32:15
3639. ilnaz, 14 мая 2011, 13:01:50
С1
помогите решить
8sin(в квад)2x+cos2x+1 / корень из -sinx

... равно нулю, я полагаю :)

ОДЗ:
-sin(x) > 0
sin(x) < 0
pi+2*pi*n < x < 2*pi+2*pi*n
(часть единичной окружности, лежащая в нижней полуплоскости)

8*(sin(2x))^2 + cos(2x) + 1 = 0
8 - (cos(2x))^2 + cos(2x) + 1 = 0
Делаем замену: cos(2x) = t

8*t^2 - t + 9 = 0
t1 = 18/16 > 1, не подходит
t2 = -16/16 = -1

cos(2x) = -1
2x = pi + 2*pi*n
x = pi/2 + pi*n

Если n четное, то x лежит в верхней полуплоскости, это не удовлетворяет ОДЗ. Нам годятся только решения для нечетных n, что можно иначе записать

x = -pi/2 + 2*pi*n

3641. Энди, 14 мая 2011, 17:02:37
Если не трудно, помогите вот с такой задачкой:
С6. Ученик должен был умножить двузначное число на трехзначное и разделить их произведение на пятизначное. Однако он не заметил знака умножения и принял записанные рядом двузначное и трехзначное числа за одно пятизначное. Поэтому полученное частное (натуральное) оказалось в три раза больше истинного. Найти все три числа.

3642. Lexxus, 14 мая 2011, 19:32:15
3641. Энди, 14 мая 2011, 17:02:37
Ученик должен был умножить двузначное число на трехзначное и разделить их произведение на пятизначное. Однако он не заметил знака умножения и принял записанные рядом двузначное и трехзначное числа за одно пятизначное. Поэтому полученное частное (натуральное) оказалось в три раза больше истинного. Найти все три числа.

Пусть двузначное число - x, трехзначное - y, пятизначное - z.
По условию,
(1000x+y)/z = 3xy/z, то есть
1000x + y = 3*x*y

Раз правая часть этого равенства делится на x, то и левая должна делиться на x, то есть

y = k*x, где k - натуральное число.

1000x + kx = 3*k*x^2
1000 + k = 3*k*x

x = (1000+k)/3k

По условию, 10<=x<=99

(1000+k)/3k >= 10
29k <= 1000
k < 35

(1000+k)/3k <= 99
296k >= 1000
k > 3

И еще нам известно, что 1000+k = 3*k*x, то есть (1000+k) делится на 3. Таких чисел между 3 и 35 десять штук:
5,8,11,14,17,20,23,26,29,32

Нам нужно найти среди них такие, что (1000+k) делится на k.

Без калькулятора - убиться веником. Короче, таких вариантов три:

1. k = 5, x = 67, y = 335
xy = 22445, и это единственное пятизначное число, на которое нацело делится и 22445, и 67335.

2. k = 8, x = 42, y = 336
xy = 14113, и это также единственное пятизначное число, на которое нацело делится и 14113, и 42336.

k = 20, x = 17, y = 340
xy = 5780, что противоречит условию.

Таким образом, у нас имеется два варианта:
67, 335 и 22445; 42, 336 и 14113

3643. Энди, 14 мая 2011, 20:30:25
Lexxus, спасибо огромное за помощь!!!:)

3644. Dinka5, 15 мая 2011, 07:23:51
Моторная лодка прошла против течения реки 255 км и вернулась в пункт отправления, затратив на обратный путь на 2 часа меньше. Найдите скорость лодки в неподвижной воде, если скорость течения равна 1 км/ч. Ответ дайте в км/ч.
ПОМОГИТЕ! ПОЖАЛУЙСТА! Составляю уравнение для задачи, а корень дискриминанта полностью не извлекается. так ведь не должно быть...(
255/(х-1)-255/(х+1)=2

3645. Lexxus, 15 мая 2011, 11:00:08
3644. Dinka5, 15 мая 2011, 07:23:51
ПОМОГИТЕ! ПОЖАЛУЙСТА! Составляю уравнение для задачи, а корень дискриминанта полностью не извлекается. так ведь не должно быть...(
255/(х-1)-255/(х+1)=2

Ну какой там нафиг дискриминант?
Домножаем всё на (x-1)(x+1), получаем
255(x+1)-255(x-1) = 2(x+1)(x-1)
255x+255-255x+255 = 2x^2-2
2x^2 = 512
x^2 = 256
x = 16 (корень -16 нас не устраивает).

3646. Antis, 15 мая 2011, 13:55:25
1. Через вершину угла С в равнобедренном треугольнике СDE проведена прямая СА, перпендикулярная к плоскости треугольника . Известно, что СА=35 дм, СD=12sqrt2(12 корней из 2) дм Найдите расстояние от точки А до прямой DE.

2. Дан прямоугольный параллелепипед ABCDA1B1C1D1. Найдите двугранный угол ADCA1, если AC=13см DC=5см AA1=12sqrt3(12 корней из 3)см.

3. основанием прямой призмы ABCDA1B1C1D1 является параллелограмм ABCD со сторонами 6 дм и 12 дм и углом, равным 60 градусам. Диагональ B1D призмы образует с плоскостью основания угол 30 градусов . Найдите площадь боковой поверхности призмы.

4. Сторона правильной треугольной пирамиды равна 3 см , а угол между боковой гранью и основанием пирамиды равен 45 градусов. Найдите площадь полной поверхности пирамиды.

5. основанием прямой призмы ABCDA1B1C1D1 является параллелограмм со сторонами 4 см и 4sqrt3( 4 корней из 3) и углом, равным 30 градусам. Диагональ AC1 призмы образует с плоскостью основания угол 60 градусов . Найдите площадь боковой поверхности призмы.

6.Высота основания правильной треугольной пирамиды равна 3 см, а угол между боковой гранью и основанием пирамиды равен 45 градусов. Найдите площадь полной поверхности пирамиды.

3647. Алинка, 15 мая 2011, 14:12:02
помогите пожалуйста.
С1. tgх + 4 /корень из - cos

3648. Алинка, 15 мая 2011, 14:15:57
С1.cos^2 + cosх / корень из 2sinx + корень из 3.

3649. Леонид , 15 мая 2011, 14:22:53
ох, и наврала, Алина, нарочно не придумкшь!!!!!

3650. Алинка, 15 мая 2011, 14:28:15
чё такое

3651. Алинка, 15 мая 2011, 15:04:06
С1.cos^2 + cosх / корень из (2sinx + корень из 3)

3652. Kristina, 15 мая 2011, 21:48:24
помогите пожалуйста!
в правильной четырёх угольной пирамиде диагональ основания равна 4 корня из 3 см, а двугранный угол при основании равно 60 градусов.найти площадь полной поверхности..
заранее спасибо!

3653. Леонид , 16 мая 2011, 05:15:52
Алинка, опять врёшь и спршиваешь, чё такое!!! Это что! Уравнение, неравенство...? Знак какой? А за "С" - берёшься. А 3647 посмотри - аргумент у косинуса?

3654. Илья, 16 мая 2011, 15:09:43
3637. Илья, 13 мая 2011, 18:29:33 цитата

Всем добрый вечер!!!
поомогите пожалуйста...
В правильной шестиугольной пирамиде SABCDEF,стороны основания которой равны 1,а боковые ребра равны 2,найдите синус угла между прямыми AL и BM,где М-середина ребра SC,L-середина ребра SB.

Заранее спасибо!

3655. Анютка, 16 мая 2011, 16:09:15
Основанием пирамиды служит треугольник со сторонами 6, 5 и 5 см. Боковые грани пирамиды образуют с ее основанием равные двугранные углы, содержащие по 45. Определите объем пирамиды.

3656. Леонид , 16 мая 2011, 18:25:03
Двугранные углы равны - знчит центр впианной окружности надо иметь ввду, причём её радиус будет равен высоте приамиды, так как углы по 45 град. Найди площадь тругольника основания, определив его высоту -4 (треуг-к равнобедренный), площадь его 12 - как полупроизведение основания на высоту. Затем радиус из формулы полупроизведения периметра (16) на радиус окружности. 12= 8 умн. на радиус- радиус 2 и высота пирамиды 2. И обём, наконец: 1/3 *12*2= 8

3657. Ляйсан, 16 мая 2011, 20:27:10
1.определите количество корней уравнения sin2x=sinx , принадлежащих интервалу (-3;3)
2.решите уравнение (tgx+1)(2sinx/2-корень из 2)=0.В ответ запишите отношение наименьшего положительного корня уравнения к числу пи.
3.найдите количество точек на отрезке [0;2pi], в которых функция y=1/tgx-1 не определена.
4.найдите отношение наименьшего по модулю корня уравнения sin^2 x=3sinxcosx-2cos^2 x к числу pi.

3658. Энди, 16 мая 2011, 21:16:27
Есть маааленькая просьбочка помочь вот с такой задачкой,а то она меня совсем угнетает...:
С5. Найдите все значения параметра а, при каждом из которых уравнение f(x)=|a+2| &#8731;x (на всякий случай напишу словами еще раз - модуль(а+2)*кубический корень из х) имеет 4 решения, где f - четная периодическая функция с периодом Т=163, определенная на всей числовой прямой, причем f(x)=ax^2, если 0<=x<=83.
Заранее огромнейшее спасибо:)

3659. Айдар, 16 мая 2011, 22:05:54
Please help.Я не могу решить задачу B12 Костя и Руслан выполняют одинаковый тест. Костя отвечает за час на 24 вопроса текста, а Руслан — на 30. Они одновременно начали отвечать на вопросы теста, и Костя закончил свой тест позже Руслана на 48 минут. Сколько вопросов содержит тест?

3660. мария, 16 мая 2011, 22:14:40
ПРИВЕТ!ОБЪЯСНИТЕ КАК РЕШАТЬ ЗАДАЧУ С ГОРОДОМ N.В городе N живет 1500000 жителей. Среди них 15% детей и подростков. Среди взрослых 40% не работает (пенсионеры, студенты, домохозяйки и т.п.). Сколько взрослых работает?

3661. Энди, 16 мая 2011, 22:58:48
Айдар, 16 мая 2011, 22:05:54
Please help.Я не могу решить задачу B12 Костя и Руслан выполняют одинаковый тест. Костя отвечает за час на 24 вопроса текста, а Руслан — на 30. Они одновременно начали отвечать на вопросы теста, и Костя закончил свой тест позже Руслана на 48 минут. Сколько вопросов содержит тест?

Вообщем, на сколько я поняла - это можно решить так:

Для начала выясним что 48мин=0,8часа
Далее, нам известно что за час Костя отвечает на 24 вопроса, а а Руслан — на 30,обозначив количество всех вопросов за х получаем: х24=х30+0,8
И решаем это уравнение:
ответ получился 96, если я все так посчитала:)

3662. Энди, 16 мая 2011, 23:03:31
3660. мария, 16 мая 2011, 22:14:40
ПРИВЕТ!ОБЪЯСНИТЕ КАК РЕШАТЬ ЗАДАЧУ С ГОРОДОМ N.В городе N живет 1500000 жителей. Среди них 15% детей и подростков. Среди взрослых 40% не работает (пенсионеры, студенты, домохозяйки и т.п.). Сколько взрослых работает?


Вычислим сколько вообще взрослых в этом самом городе N:
100-15=85% - всего взрослых
Если 40% не работают, то значит остальные 45% все-таки что-то делают:) Таким образом:
1500000*0,45=675 000

3663. Энди, 16 мая 2011, 23:06:07
3661. Энди, 16 мая 2011, 22:58:48
Вообщем, на сколько я поняла - это можно решить так:

Для начала выясним что 48мин=0,8часа
Далее, нам известно что за час Костя отвечает на 24 вопроса, а а Руслан — на 30,обозначив количество всех вопросов за х получаем: х24=х30+0,8
И решаем это уравнение:
ответ получился 96, если я все так посчитала:)


Уравнение должно быть таким: х:24=х:30+0,8 (вообщем там тоже самое, только разделить, а не умножить - не допечатала просто:)))

3664. люда, 17 мая 2011, 09:10:40
помогите решить задачу!
дан прямоугольный параллелепипед ABCDA1B1C1D1, AC=13, DC=5, AA1=12 КОРНЕЙ ИЗ 3. ВЫЧИСЛИТЕ ГРАДУСНУЮ МЕРУ ДВУГРАННОГО УГЛА ADCA1.
Заранее спасибо!

3665. Светлана, 17 мая 2011, 14:33:33
Решите пожалууйййстааааа!ОЧЕНЬ ПРОШУ!

Сторона основания правильной треугольной пирамиды 3см, двугранный угол при стороне очснования 45 градусов.Найдите площадь полной поверхности пирамиды

3666. KceH, 17 мая 2011, 15:23:43
Вашему сайту цены бы не было,если бы как-нибудь можно было приделать под каждым заданием его решение.Не понимаешь,как решается,открыл,посмотрел,понял.Это был бы лучший сайт на просторах Рунета.
С уважением.

3667. маша, 17 мая 2011, 20:02:37
высота треугольной пирамиды равна 6 см,двугранный угол при стороне основания равен 30 градусов.наяти площадь поверхности пирамиды

3668. Дмитрий, 17 мая 2011, 21:28:17
Пожалуйста помогите решить задачу № 28021!
Заранее благодарю)

3669. FanatOfSpartak, 18 мая 2011, 00:49:36
Люди добрые! Завтра (фактически, уже сегодня) у 10-х классов в Москве пробное ЕГЭ по математике, начинается в 9:00. Кто может, оставайтесь пожалуйста онлайн и решайте задачки, данные в комментариях

3670. саша, 18 мая 2011, 10:05:01
3664. люда, 17 мая 2011, 09:10:40
помогите решить задачу!
дан прямоугольный параллелепипед ABCDA1B1C1D1, AC=13, DC=5, AA1=12 КОРНЕЙ ИЗ 3. ВЫЧИСЛИТЕ ГРАДУСНУЮ МЕРУ ДВУГРАННОГО УГЛА ADCA1.
Заранее спасибо!
3664. люда, 17 мая 2011, 09:10:40
помогите решить задачу!
дан прямоугольный параллелепипед ABCDA1B1C1D1, AC=13, DC=5, AA1=12 КОРНЕЙ ИЗ 3. ВЫЧИСЛИТЕ ГРАДУСНУЮ МЕРУ ДВУГРАННОГО УГЛА ADCA1.
Заранее спасибо!
3664. люда, 17 мая 2011, 09:10:40
помогите решить задачу!
дан прямоугольный параллелепипед ABCDA1B1C1D1, AC=13, DC=5, AA1=12 КОРНЕЙ ИЗ 3. ВЫЧИСЛИТЕ ГРАДУСНУЮ МЕРУ ДВУГРАННОГО УГЛА ADCA1.
Заранее спасибо!

3671. саша, 18 мая 2011, 10:09:18
3670. саша, 18 мая 2011, 10:05:01
3664. люда, 17 мая 2011, 09:10:40
помогите решить задачу!
дан прямоугольный параллелепипед ABCDA1B1C1D1, AC=13, DC=5, AA1=12 КОРНЕЙ ИЗ 3. ВЫЧИСЛИТЕ ГРАДУСНУЮ МЕРУ ДВУГРАННОГО УГЛА ADCA1.
Заранее спасибо!
3664. люда, 17 мая 2011, 09:10:40
помогите решить задачу!
дан прямоугольный параллелепипед ABCDA1B1C1D1, AC=13, DC=5, AA1=12 КОРНЕЙ ИЗ 3. ВЫЧИСЛИТЕ ГРАДУСНУЮ МЕРУ ДВУГРАННОГО УГЛА ADCA1.
Заранее спасибо!
3664. люда, 17 мая 2011, 09:10:40
помогите решить задачу!
дан прямоугольный параллелепипед ABCDA1B1C1D1, AC=13, DC=5, AA1=12 КОРНЕЙ ИЗ 3. ВЫЧИСЛИТЕ ГРАДУСНУЮ МЕРУ ДВУГРАННОГО УГЛА ADCA1.
Заранее спасибо!
сейчас напишите мне в течении 5 минут и я помогу вам

3672. роман, 18 мая 2011, 11:12:41
Найдите наименьшее значение функции y = ( x-5 )в квадрате (x+2)-7 на отрезке

3673. роман, 18 мая 2011, 11:25:23
помогите пожалуйста решить эти 2 задачи.

В равнобедренной трапеции основания 53 и 29. Длина боковой стороны равна 13. Найдите высоту трапеции

3674. Francyz, 18 мая 2011, 23:13:50
Люди помогите пожалуйста...

найдите значение выражения 21 синус в квадрате альфа,если тангенс равен корень,а под корнем 3^11

3675. саша, 19 мая 2011, 08:17:36
3673. роман, 18 мая 2011, 11:25:23
помогите пожалуйста решить эти 2 задачи.

В равнобедренной трапеции основания 53 и 29. Длина боковой стороны равна 13. Найдите высоту трапеции
ответ будет равен 5

3676. Александра, 19 мая 2011, 16:43:26
помогите пожалуйста(((((((очень вас прошу(((( срочно очень((
Основанием прямой призмы A1B1C1D1ABCD является параллелограмм ABCD со сторонами 6см и 6&#8730; 3см и углом равным 60&#730;. Найдите площадь боковой поверхности призмы .

сторона основания правильной треугольной пирамиды равна 3см, а двугранный угол при стороне основания 45градусов. найти площадь полной поверхности пирамиды

3677. DecemberFlower, 19 мая 2011, 17:41:24
3673. роман, 18 мая 2011, 11:25:23
помогите пожалуйста решить эти 2 задачи.

В равнобедренной трапеции основания 53 и 29. Длина боковой стороны равна 13. Найдите высоту трапеции


Роман, не знаю, насколько верно мое решение, но получается 5:
(53-26)/2=12 - один из катетов треугольника; второй равен боковой стороне. По Пифагору высота = sqrt(13^2-12^2)=5.

3678. Татьяна, 19 мая 2011, 20:08:02
а почему 26? =) и какой именно из этих катетов

3679. Татьяна, 19 мая 2011, 20:11:24
а все поняла

3680. DecemberFlower, 19 мая 2011, 20:36:21
Да, опечатка, конечно, 53-29

3681. Леонид, 19 мая 2011, 22:39:06
помогоите решить пожалуйста)

В правильную четырехуголбную пирамиду вписан конус. Найдите отношение объемов конуса и пирамиды.
с решением если можно)

и еще одну...

В правильной треугольной пирамиде высота основания равна h, боковые ребра наклонены к основанию под углом a(альфа).Найдите объем пирамиды.

тоже решение хотелось бы если можно...

никак не могу эти задачи решить, а очень надо...

3682. Леночка, 19 мая 2011, 23:26:18
Пожалуйста помогите,завтра контрольная,а я не могу решить одну задачу:
Высота правильной четырехугольной пирамиды равна корень из 6 см,а боковое ребро наклонено к плоскости основания под углом 60 градусов
а)найдите боковое ребро пирамиды
б)найдите площадь боковой поверхности пирамиды
я на вас надеюсь..)))

3683. Лёви, 20 мая 2011, 06:32:08
Люди...пожалуйста,помогите решить!!очень прошу!
1)в правильной треугольной призме АВСА1В1С1 сторона основания равна 1,точка Е-середина А1С1.найдите |СЕ-СБ|
ЗЫ. |СЕ и СБ|-векторы по модулю.
2) основанием пирамиды MABC служит прямоугольный треугольник ACB(90 градусов),АС=6,ВС=8.Боковые ребра пирамиды наклонены к основанию под углом 60 градусов.Найдите |АС+ВМ+СВ|.
|АС+ВМ+СВ|векторы мо модулю.
3) АВСДА1В1С1Д1-параллелепипед.А1С пересекает В1Д в точке М,векторы А1С=хСМ.Найдите х.

можно с чертежом,пожалуйста??Т_Т

3684. саша, 20 мая 2011, 09:21:06
3677. DecemberFlower, 19 мая 2011, 17:41:24
3673. роман, 18 мая 2011, 11:25:23
помогите пожалуйста решить эти 2 задачи.

В равнобедренной трапеции основания 53 и 29. Длина боковой стороны равна 13. Найдите высоту трапеции


Роман, не знаю, насколько верно мое решение, но получается 5:
(53-26)/2=12 - один из катетов треугольника; второй равен боковой стороне. По Пифагору высота = sqrt(13^2-12^2)=5.
я же дал ответ роману еще раньше, а он не увидел

3685. Виктория, 20 мая 2011, 18:08:13
Задача 28159. У меня получается 10, а ответ неправильный. Проверила подстановкой, всё равно 10 получается. Что делать? =)

3686. Илья, 20 мая 2011, 23:26:11
Lexxus , спасибо за предыдущую задача с6 !!
Но я снова в смятении... По следующей причине:

Найдите сумму всех трехзначных натуральных чисел n,таких, что первая последняя цифры числа n^2 равны 1

3687. Илья, 21 мая 2011, 14:53:20
Последняя цифра квадрата - 1, значит последняя цифр самого числа - 9.

100<=n<=999
10000<=n^2<999999

Если n^2 пятизначное, то, учитывая, что первая цифра квадрата - 1,
10000<=n^2<=19999
100<=n<=141 => 101, 109, 111, 119, 121, 129, 131, 139, 141

Если n^2 шестизначное, то, учитывая, что первая цифра квадрата - 1,
100000<=n^2<=199999
316<n<448
319,441 и пары 32x, 33x, 34x, 35x, 36x, 37x, 38x, 39x, 40x, 41x, 42x, 43x, где x - 1,9. Сумма каждой пары даст 650, 670, ... , 870

Суммируем парами: 210+230+250+270+141=(по арифм. прогрессии)=141+960=1101
319+441+650+...+870=319+441+(650+870)/2*12=9120+319+441=9120+760=9880

Итого: 9880+1101=10981

3688. Илья, 21 мая 2011, 14:54:05
*Последняя цифра квадрата - 1, значит последняя цифра самого числа - 9 или 1.

3689. Ольга, 22 мая 2011, 08:01:03
Помогите пожалуйста решить задачу..
В кубе ABCDA1B1C1D1 найдите косинус угла между плоскостями AB1C1 и BA1D1.
Буду очень благодарна..заранее спасибо=)

3690. Анастасия, 22 мая 2011, 13:17:28
помогите решить задачку,если можно и решение тоже напишите..
Диаметр одного шара равен радиусу другого шара.Чему равно отношение их объёмов?

3691. Анжелла, 22 мая 2011, 14:32:53
пожалуйста, помогите решить задачу! Основанием прямой призмы ABCDA1B1C1D1 является параллелограмм ABCD со сторонами 4 см и 4корней из 3 и углом равным 30. Диагональ АС призмы образует с плоскостью основания угол в 60 град. Найдтите площадь боковой грани. Прошу решить пожаааалуйста СЕГОДНЯ!!!))

3692. Маша, 23 мая 2011, 01:20:04
Помогите пожалуйста решить!!!Заранее огромное спасибо!!!


1. Основание прямой призмы треугольник у которого стороны длиной 5 и 6 см образует угол в 30 градусев. Боковое ребро призмы равно 4см.Найдите объем призмы.

2. Найдите площадь поверхности прямоугольного параллелепипеда у которого диоганаль равна 13дм высота 12дм а одно из ребер основания 4дм

3. Осевое сечение цилиндра прямоугольник со сторонами 12 и 26 см.Найдите объем цилиндра если его высота равна меньше стороны осевого сечения.

4. Образующая конуса равна 8см а угол при вершине осевого сечения равен 60 градусев. Найдите площадь боковой поверхности конуса.

3693. андрей, 23 мая 2011, 15:10:14
народ помогите решить задачу, желательно с решением, в правильной четырехугольной пирамиде диагональ основания равна 8 корней из 2 и двугранным углом при основании 60 градусов, найти площадь полной поверхности??

3694. Надежда, 23 мая 2011, 17:43:11
помогите пожалуйста решить эти 3 задачи.

1. Сторона основания правильной четырехугольной пирамиды равна m, а двугранный угол при стороне основания пи. Определить радиус вписанного в нее шара.

2. Образующая конуса составляет с его осью угол пи. Определить отношение объема этого конуса, к объему описанного около него шара.

3. В правильной треугольной пирамиде плоский угол при вершине равен пи, а сторона основания m. Определить объем пирамиды.

3695. иринка, 24 мая 2011, 12:51:45
Один из углов равнобедренного треугольника на 90 градус больше другого.Найдите углы треугольника

3696. Илья, 24 мая 2011, 18:51:53
Углы x, x, 90+x. Или x, 90+x, 90+x - два варианта. Но во втором варианте сумма их >180 => не подходит.

x+x+x+90=180
x=30

3697. Ириска, 24 мая 2011, 19:05:36
Помогите с С2
В кубе ABCDA1B1C1D1 точки M,N,P - середины ребер A1B1,B1C1,DC. Найдите тангенс угла между прямыми MN и A1P.
В ответах 3,у меня получилось 1.
Подскажите, как решить правильно.

3698. рос, 24 мая 2011, 21:46:37
помогите пожалуйста!можно еще рисунок?заранее спасибо!
дан прямоугольный треуогльник с гипотенузой АС=16 см и катетом ВС=12 см.отрезок SC=20 см-перпендикуляр к плоскости АВС.
найдите:
а)вектор CS+вектор CB+вектор BA по модулю= вектор SA по модулю
б)найдите угол между прямой SA и плоскостью АВС


3699. Дмитрий, 26 мая 2011, 16:56:29
Помогите решить пожалуйста

В сосуд, имеющий форму правильной треугольной призмы, налили воду. Уровень воды достигает 16 см. На какой высоте будет находиться уровень воды, если ее перелить в другой такой же сосуд, у которого сторона основания в 4 раза больше, чем у первог

3700. Lexxus, 26 мая 2011, 19:33:42
3697. Ириска, 24 мая 2011, 19:05:36
В кубе ABCDA1B1C1D1 точки M,N,P - середины ребер A1B1,B1C1,DC. Найдите тангенс угла между прямыми MN и A1P.

Прямая MN параллельна прямой A1C1. Значит, искомый угол равен углу PA1C1.
Его можно найти из треугольника A1C1P.
A1C1 = sqrt(A1D1^2+D1C1^2) = sqrt(2)
C1P = sqrt(PC^2+CC1^2) = sqrt(1+1/4) = sqrt(5)/2
A1P = sqrt(A1D^2+DP^2) = sqrt(2+1/4) = 3/2

По теореме косинусов:
C1P^2 = A1C1^2 + A1P^2 - 2*A1C1*A1P*cos(PA1C1)
5/4 = 2 + 9/4 - 2*sqrt(2)*(3/2)*cos(PA1C1)

откуда cos(PA1C1) = sqrt(3)/2

Значит, угол PA1C1 равен pi/6, а его тангенс равен sqrt(3)/3

3701. Каменцев, 26 мая 2011, 20:04:09

В правильной четырёхугольной пирамидеABCDH c основанием ABCD является квадрат со стороной" a".Двугранный угол между плоскостью ADH и точкой C равен 120 градусов. Найти S боковой поверхности.

3702. Леонид , 27 мая 2011, 09:26:04
Задача С4. Ваш вариант решения?
В угол, равный arccos(-1/9), вписана окружность радиуса 3.
параллельно хорде, соединяющей точки касания, проведены
две касательные к окружности, в результате чего получилась
трапеция. Найдите площадь этой трапеции.

3703. Леонид , 27 мая 2011, 09:30:26
Что значит двугранный угол с пл.АДН и точкой С?

3704. Lexxus, 27 мая 2011, 20:15:16
3702. Леонид , 27 мая 2011, 09:26:04
В угол, равный arccos(-1/9), вписана окружность радиуса 3.
параллельно хорде, соединяющей точки касания, проведены
две касательные к окружности, в результате чего получилась
трапеция. Найдите площадь этой трапеции.


Нас интересует площадь трапеции ABCD, Её высота равна 2R=6. Осталось только найти полусумму оснований (AB+CD)/2.


Из прямоугольного треугольника OLH найдем OH:
OH = OL/sin(OHL).
Для удобства обозначим угол OHL буквой "a".

По формуле косинуса двойного угла:
cos(2a) = 1 - 2sin^2(a), откуда
sin(a) = sqrt((1-cos(2a))/2)
sin(a) = sqrt((1+1/9)/2) = sqrt(5)/3

Чтобы потом к этому не возвращаться,
cos(a) = sqrt(1-5/9) = 2/3
tg(a) = sqrt(5)/3/2*3 = sqrt(5)/2.

Итак, OH = OL/sin(a) = 9/sqrt(5)
Отсюда:
PH = 9/sqrt(3) - 3
QH = 9/sqrt(3) +3

AB = 2*PH*tg(a)
CD = 2*QH*tg(a)

(AB+CD)/2 = 2*9/sqrt(5)*sqrt(5)/2 = 9

S(ABCD) = 9*6 = 54

3705. Леонид , 27 мая 2011, 21:07:01
--------------------------------------------------------------------------------

3704. Lexxus, 27 мая 2011, 20:15:16
СПАСИБО! Я решил также. Думал, что есть решение короче! Необычность длинных записей с аркфункциями.

3706. Настёнка), 28 мая 2011, 01:01:06
log4(8 - 5x)=2log43 как решить?

3707. Радостный отличник)), 28 мая 2011, 01:45:41
покупаем ответы на ЕГЭ kupiege.com

3708. Леонид , 28 мая 2011, 07:38:31
3604. Lexxus, 8 мая 2011, 20:44:16. Есть предложение о другом решении, почти устное. По свойству биссектрисы: (7+х):5к = (5+х):4к, на к делим, имеем по правилу пропорции 28+4х = 25+5х, х=3, ответ 10 и 8.

3709. Игорь, 28 мая 2011, 19:05:20
помогите пожалуйста решить задачу

Образующая конуса наклонена к плоскости основания под углом 60°, а расстояние от центра основания до образующей равно &#8730;3 м. Найдите площадь полной поверхности конуса.

3710. Игорь, 28 мая 2011, 19:09:00





Образующая конуса наклонена к плоскости основания под углом 60°, а расстояние от центра основания до образующей равно 3 м. Найдите площадь полной поверхности конуса.

3711. x, 28 мая 2011, 23:28:49
в прямой призме абса1б1с1 угол с=90 гр, аб=8sqrt(2), высота равна 6. треугольник в основании равнобедренный. найдите угол между ас1 и сб1.

помогите, пожалуйста, с ответом!!!)

3712. Рауль, 29 мая 2011, 08:49:21
пожалуйста,объясните как решать задания тапа B8(6412).В других подобных где есть X я знаю,что надо находить ероизводную,а в этих невыходит.Ещё задача B8(6404) объясните как решать

3713. Леонид , 29 мая 2011, 14:43:52
6412. касательные параллельные у= в (просто число). Уравнение любой прямой у=кх + в, видно, что к=0, то есть производная равна 0 - это в вершинах графика. Сколько вершин у графика, столько и касательных параллельных у=в. Для всех заданий такого вида.

3714. Леонид , 29 мая 2011, 14:49:19
6404. Имеем график производной, который на [0;6] выше оси ОХ, то есть производная здесь положительная, то функция возрастает. Значит её наибольшее значение при х = 6.

3715. Рауль, 29 мая 2011, 17:42:40
Леонид спасибо

3716. Иван, 29 мая 2011, 18:13:10
Ответы на ЕГЭ есть тут! Хватит учить! Проще заплатить!

3717. Леонид , 29 мая 2011, 18:52:58
3715. Рауль, 29 мая 2011, 17:42:40. Успехов! Помогли мои разъяснения? необходимо внимательнейшее чтение условия. График чего на рисунке?
Вопрос- задания? это от свойства функции, на производную, если имеем график функции! От производной, к свойству функции, если имеем график производной! Внимательность. Знание взаимосвязи свойства и производной (например: функция ворастает по графику, то производная неотрицательная).

---------------------------------------------------------------------------------------------
НАРОД! Рекомендую не ползоваться заманивающими сайтами по ЕГЭ. Здесь всё есть, общайтесь, проверяйте себя на тестах. Всё для ученика и бесплатно. На других, продя тест, просят дть номер вашего телефона, что якобы подтверждает, что вы не спам. Дадут код, воспользовавшись им, получите результат. А это денюшки ваши! Ради чего? Занимайтесь здесь ! Здесь реальные возможные задания.

3718. K., 30 мая 2011, 11:54:08
На изготовление 48 деталей первый рабочий тра­тит на 8 часов меньше, чем второй рабочий на изготовле­ние 96 таких же деталей. Известно, что первый рабочий за час делает на 4 детали больше, чем второй. Сколько дета­лей в час делает второй рабочий?

3719. Арсений, 31 мая 2011, 21:56:32
К заданию B9 (5073) дан неправильный ответ 14. Правильный ответ - 3.

3720. Lexxus, 31 мая 2011, 22:54:45
3719. Арсений, 31 мая 2011, 21:56:32
К заданию B9 (5073) дан неправильный ответ 14. Правильный ответ - 3.

Если ты относишься к тем, кто шестого сдаёт математику, тебе надо срочно прекращать тупить.

3721. Каприза)), 1 июня 2011, 09:44:52
Помогите решить уравнение sin(-x)=cos пи

3722. катя, 1 июня 2011, 14:55:20
Найдите значение выражения frac{g(x +1)}{g(x )}, если g(x)=6^{x}.

3723. Виталий, 1 июня 2011, 16:53:13
В сосуд, имеющий форму правильной треугольной призмы, налили воду. Уровень воды достигает 9 см. На какой высоте будет находиться уровень воды, если ее перелить в другой такой же сосуд, у которого сторона основания в 3 раза больше, чем у первого? Обьясните пожалуйста принцип решения такой задачи. Заранее благодарю

3724. Лина, 2 июня 2011, 07:41:17
Имеются два слитка сплава золота и меди. Первый слиток содержит 230 г золота и 20 г меди, второй – 240 г золота и 60 г меди. От каждого слитка взяли по куску, сплавили их и получили 300 г сплава, в котором 84% золота. Определите массу (г) куска, взятого от первого слитка?Слабо решить?

3725. Марина, 2 июня 2011, 10:21:08
Помогите решить задачу!!! Призма.Правильная треугольная. Нужно найти объем призмы и площадь боковой поверхности призмы, если известно сторона основания а-2корень 3 и высота призмы Н-3

3726. Елена, 2 июня 2011, 12:37:27
сторона правильной треугольной пирамиды 3см.,а угол между боковой гранью и основанием пирамиды равен 45градусов.Найдите площадь полной поверхности пирамиды.

3727. Леонид , 2 июня 2011, 14:05:27
марина и лена, вы откуда берёте задачи, они же не егэшные. времени, что ли у вас много.

3728. mashyta93, 2 июня 2011, 18:41:42
Помогите решить пожалуйста

Первая труба пропускает на 1 литров воды в минуту меньше, чем вторая труба. Сколько литров воды в минуту пропускает вторая труба, если резервуар объемом 930 литров она заполняет на 1 минут раньше, чем первая труба?

3729. Ксения, 2 июня 2011, 22:50:36
задача(найти боковую поверхность правильной шестиугольной призмы если её меньшая диагональ равна d и составляет с плоскостью основания угол альфа...помогите пожалуйста ее решить...буду очень признательна)

3730. Lada, 3 июня 2011, 00:10:03
xy+x-y=7, x^2-xy^2=6 - система уравнений. Понимаю, что за 9 класс, но чистый гуманитарий. И школа была давно. помогите, пожалуйста.
И еще упростить: sqrt(4-3*sqrt(2))^4 - 24*sqrt(2) sqrt(14-3*sqrt(2))^4 + 6*sqrt(28)

3731. анна, 3 июня 2011, 09:49:19
1537. оля, 4 июня 2010, 19:06:30
помогите решить Б10 плиз
После дождя уровень воды в колодце может повыситься.Мальчик измеряет время падения t небольших камешков в колодец и рассчитывает расстояние от повехности земли до уровня воды по формуле h=-5t^2. до дождя время падения камешков составляло 1с. На какую наименьшую высоту должен подняться уровень воды после дождя,чтобы измеряемое время изменилось больше, чем на 0.1 с? ответ выразите в метрах.

3732. Леся, 3 июня 2011, 10:42:38
Прямоугольный параллелепипед описан около цилиндра,высота которого равна 16, V параллелепипеда равен 64.Найдите радиус цилиндра. помогите пожалуйста кто-нибудь эту задачку решить.

3733. snejka, 3 июня 2011, 18:51:56
3559. Кристина, 1 мая 2011, 10:46:38
Помогите пожалуйста!срочно надо:( по плану первой бригаде нужно изготовить на 720 изделий больше,чем второй. Так как в первую бригаду добавили 3 человека,а во вторую-2 человека, то обе бригады выполнили план на 2 дня раньше срока, причем каждый рабочий изготавливал в день по 18 изделий. Найдите, сколько рабочих по плану должны были работать в обеих бригадах вместе?


помогите пожалуйста решить эта задачу до завтра :)
я вроде в математике не нуб, но никак не получается :(

3734. snejka, 3 июня 2011, 19:03:17
3732. Леся, 3 июня 2011, 10:42:38
Прямоугольный параллелепипед описан около цилиндра,высота которого равна 16, V параллелепипеда равен 64.Найдите радиус цилиндра. помогите пожалуйста кто-нибудь эту задачку решить.


Vп=а(квадрат)*b
b=16
Vп=64
r=a/2

a^2=64/16
a^2=4
a=2
=>
r=1


3735. snejka, 3 июня 2011, 19:06:12
3731. анна, 3 июня 2011, 09:49:19
1537. оля, 4 июня 2010, 19:06:30
помогите решить Б10 плиз
После дождя уровень воды в колодце может повыситься.Мальчик измеряет время падения t небольших камешков в колодец и рассчитывает расстояние от повехности земли до уровня воды по формуле h=-5t^2. до дождя время падения камешков составляло 1с. На какую наименьшую высоту должен подняться уровень воды после дождя,чтобы измеряемое время изменилось больше, чем на 0.1 с? ответ выразите в метрах.



если я правильно поняла, уровень должен подняться на 0.95
но могла ошибиться :(

3736. Леонид , 3 июня 2011, 21:36:29
--------------------------------------------------------------------------------

3704. Lexxus, 27 мая 2011, 20:15:16 цитата

3702. Леонид , 27 мая 2011, 09:26:04
В угол, равный arccos(-1/9), вписана окружность радиуса 3.
параллельно хорде, соединяющей точки касания, проведены
две касательные к окружности, в результате чего получилась
трапеция. Найдите площадь этой трапеции.



задача, всётаки решается и проще, и короче. Провести высоту трапеции ВН и из треугольникаВСН по определению синуса находим ВС - зтого достаточно, т.к. ВС и есть полусумма оснований трапиеции(св-во описанного четырёхугольника около окружности), высота 6.

3737. Ластик, 4 июня 2011, 10:34:33
Честно я не сдаю в этом году ЕГЭ,
просто летом начинаю заниматься т.к. утго у меня с математикой)))
у меня вопрос как решить В4
В треугольнике АВС угол С равен 90градусов, АВ=5, АС=4. Найдите sinА

3738. snejka, 4 июня 2011, 13:24:33
3737. Ластик, 4 июня 2011, 10:34:33
Честно я не сдаю в этом году ЕГЭ,
просто летом начинаю заниматься т.к. утго у меня с математикой)))
у меня вопрос как решить В4
В треугольнике АВС угол С равен 90градусов, АВ=5, АС=4. Найдите sinА



по теореме Пифагора находишь третью сторону
даже не считая могу сказать она будет равна 3, т.к. данный треугольник египетский со сторонами 3, 4, 5.

далее
синус это отношение противолежащего угла к гипотенузе
т.е. sinA=3/5
т.к. в егэ необходимо писать десятичную дробь, то в ответ пойдет 0,6

3739. Каплан, 4 июня 2011, 15:12:04
произведение нескольких различных простых чисел делится на каждое из этих чисел,уменьшенное на 1.Чему может быть равно это произведение?

3740. Lexxus, 4 июня 2011, 15:21:17
3739. Каплан, 4 июня 2011, 15:12:04
произведение нескольких различных простых чисел делится на каждое из этих чисел,уменьшенное на 1.Чему может быть равно это произведение?


Решение задачи C6: Произведение нескольких различных простых чисел

3741. snejka, 4 июня 2011, 17:19:25
3740. Lexxus, 4 июня 2011, 15:21:17
3739. Каплан, 4 июня 2011, 15:12:04
произведение нескольких различных простых чисел делится на каждое из этих чисел,уменьшенное на 1.Чему может быть равно это произведение?


Решение задачи C6: Произведение нескольких различных простых чисел




мм..мне тоже вариантик с такой с6 попался :)

3742. каплан, 4 июня 2011, 21:27:47
ну вы мне решиииииите,или неет?:)

3743. snejka, 4 июня 2011, 21:46:32
3742. каплан, 4 июня 2011, 21:27:47
ну вы мне решиииииите,или неет?:)



тебе кинули выше ссылку на решение данного задания :)

3744. alya, 5 июня 2011, 05:28:33
Из А в В одновременно выехали два автомобилиста. Первый проехал с постоянной скоростью весь путь. Второй проехал первую половину пути со скоростью 27 км/ч, а вторую половину пути проехал со скоростью, на 18 км/ч большей скорости первого, в результате чего прибыл в В одновременно с первым автомобилистом. Найдите скорость первого автомобилиста. Ответ дайте в км/ч.

3745. snejka, 5 июня 2011, 09:05:44
3744. alya, 5 июня 2011, 05:28:33
Из А в В одновременно выехали два автомобилиста. Первый проехал с постоянной скоростью весь путь. Второй проехал первую половину пути со скоростью 27 км/ч, а вторую половину пути проехал со скоростью, на 18 км/ч большей скорости первого, в результате чего прибыл в В одновременно с первым автомобилистом. Найдите скорость первого автомобилиста. Ответ дайте в км/ч.


составляешь ур-ие:

S/V1=S/2*27 + S/2*(V1+18)

S - сокращается и остается 1-но ур-ие с 1-ой переменной
уравнение составляется по принципе:
находим время которое проехал первый S-все расстояние делится на скорость V1
затем ищем время второго, т.к. он проехал сначала пол пути, то мы половину пути S/2 делим на скорость на этом участке 27
и прибавляем время со второго участка где он ехал со скоростью на 18 больше первого
:)

3746. Энди, 5 июня 2011, 13:02:16
Помогите пожалуйста:
С5. Найдите все значения параметра а, при каждом из которых уравнение f(x)=|a+2|*кубический корень(x) имеет 4 решения, где f - четная периодическая функция с периодом Т=16/3, определенная на всей числовой прямой, причем f(x)=ax^2, если 0<=x<=8/3.
Заранее огромнейшее спасибо!

3747. Ecology, 5 июня 2011, 14:01:27
Высота трапеции равна 10, площадь равна 150. Найдите сред-нюю линию трапеции.

MA.OB10.B6.48/innerimg0.jpg
Ответ: 15
Мне кажется, ответ неверный, так как средняя линия трапеции равна полусумме её оснований, поэтому ответ: 7,5.

3748. snejka, 5 июня 2011, 14:11:16
3747. Ecology, 5 июня 2011, 14:01:27
Высота трапеции равна 10, площадь равна 150. Найдите сред-нюю линию трапеции.

MA.OB10.B6.48/innerimg0.jpg
Ответ: 15
Мне кажется, ответ неверный, так как средняя линия трапеции равна полусумме её оснований, поэтому ответ: 7,5.


все верно
средняя линия будет равна полусумме оснований
т.к. по формуле у нас получается площадь трапеции равна высоте умноженой на полусумму оснований
то для получения ответа надо 150/10 и получится 15

3749. Ecology, 5 июня 2011, 15:53:46
Ой, извините, пожалуйста. Я всё поняла. Спасибо.

3750. Виви, 5 июня 2011, 16:16:17
Помогите решить

3751. наталья, 5 июня 2011, 17:09:19
Имеются два сосуда. Первый содержит 100 кг, а второй — 20 кг раствора кислоты различной концентрации. Если эти растворы смешать, то получится раствор, содержащий 72% кислоты. Если же смешать равные массы этих растворов, то получится раствор, содержащий 78% кислоты.

3752. Виктор, 5 июня 2011, 17:27:57
http://live.mephist.ru/show/mathege2010/view/id/12731/ - правильный ответ - "-7" (без кавычек). Такая ошибка встречается мне уже не в первый раз.

3753. snejka, 5 июня 2011, 18:18:02
3751. наталья, 5 июня 2011, 17:09:19
Имеются два сосуда. Первый содержит 100 кг, а второй — 20 кг раствора кислоты различной концентрации. Если эти растворы смешать, то получится раствор, содержащий 72% кислоты. Если же смешать равные массы этих растворов, то получится раствор, содержащий 78% кислоты.


решается системой:
за Х берешь процент кислоты в первом расстворе за У соответственно во втором
составляешь уравнения
100*х+20*у=120*0,72
20*х+20*у=40*0,78

3754. Татьяна, 5 июня 2011, 19:31:06
Помогите решить задачу. В правильной треугольной пирамиде SABC сторона основания АВ=6, боковое ревроSA=5. Найдите расстояние между прямыми АВ и SA.

3755. Татьяна, 5 июня 2011, 19:33:43
Помогите решить задачу. В правильной треугольной пирамиде SABC сторона основания АВ=6, боковое ревроSA=5. Найдите расстояние между прямыми АВ и SС.

3756. Эльвира, 5 июня 2011, 22:36:36
в задании 5179 ответ не 17. А 18. Или я не права? Помогите,если не правильно.

3757. Олег, 6 июня 2011, 07:53:42
Бескорыстные создатели раздела по ЕГЭ - молодцы. Любят своё дело, должно быть. И сарказм с юмором воодушевляют. Ну, сарказм, потому что мы ж по одну сторону баррикад.

3758. Полина, 6 июня 2011, 07:57:50
прямоугольный параллелепипед описан около цилиндра радиус которого 3 объем параллелепипеда 36 найти высоту
помогите!!!!!!!!!!

3759. Лиля, 6 июня 2011, 09:52:57
Помогите плиз решить задачу срочно с ЕГЭ!!!!! в правильной 6-тиугольной призме АВСДЕFА1В1С1Д1Е1F1 стороны по 5, боковые по 11. найти расстояние от точки А до Е1Д1

3760. Жанна, 6 июня 2011, 10:18:29
В правильной шестиугольной призме абсдеф а1б1с1д1е1ф1 стороны основания которой равны 4 а боковые ребра равны 1 найдите расстояние От точки б до прямой ф1е1

3761. ilo, 6 июня 2011, 10:21:22
AE1 перпендикулярна Е1D1 по теореме о 3-х перпендикулярах AE1- искомое расстояние Из AE1E: AE1=sqrt(121+(5sqrt3)^2)=14
3759. Лиля, 6 июня 2011, 09:52:57
Помогите плиз решить задачу срочно с ЕГЭ!!!!! в правильной 6-тиугольной призме АВСДЕFА1В1С1Д1Е1F1 стороны по 5, боковые по 11. найти расстояние от точки А до Е1Д1
, где 5sqrt3= АЕ=sqrt3a, где а-сторона основания.

3762. приветя, 6 июня 2011, 10:42:06
В равнобедренную трапецию с периметром 20 вписана окружность. Точка касания делит боковую сторону в отношении 1:4. Прямая проходящая через центр окружности и вершину трапеции, отсекает от трапеции треугольник. Найдите его площадь.

3763. юля, 6 июня 2011, 10:44:46
ПОМОГИТЕ ПОЖАЛУЙСТА РЕШИТЬ...УМОЛЯЮ

В треугольнике АВС АD- биссиктриса. Угол С равен 102 градуса,угол САD Равен 5 градусов. Найдите угол В, ответ в градусах

3764. ЮЛЯ, 6 июня 2011, 10:49:19
Из пункта А в пункт В расстояние между которыми 50 км одновременно выехали мотоциклист и велосепедист. Известно что за час мотоцикл проезжает на 30км больше чем вело. определите скорость вело если известно что он пробыл в пункт В на 1,5 часа позже мото

ПОЖ-ТА ПОЛМОГИТЕ РЕШИТЬ ЕГЭ

3765. Пиветя, 6 июня 2011, 10:49:40
ПЛИИИИИЗ РЕШИТЕЕЕЕЕ!
В равнобедренную трапецию с периметром 20 вписана окружность. Точка касания делит боковую сторону в отношении 1:4. Прямая проходящая через центр окружности и вершину трапеции, отсекает от трапеции треугольник. Найдите его площадь.

3766. ilo, 6 июня 2011, 10:51:46
3760. Жанна, 6 июня 2011, 10:18:29
В правильной шестиугольной призме абсдеф а1б1с1д1е1ф1 стороны основания которой равны 4 а боковые ребра равны 1 найдите расстояние От точки б до прямой ф1е1

Искомое расстояние-BF1 находим из треугольника BF1F. BF=sqrt(a)=4sqrt3, где а- сторона основания.BF1^2=48+1=49 BF1=7; BF1- перпендикулярна E1F1 по теореме о трёх перпендикулярах, поэтому BF1E1- прямоугольный и BF1- искомое расстояние.

3767. ilo, 6 июня 2011, 11:00:39
BF=a*sqrt(a) - надо так, выше-опечатка

3768. Белка, 6 июня 2011, 11:05:32
В правильной шестиугольной приеме АВСDEFA1B1C1D1E1F1, стороны основания которой равны 3,а боковые ребра равны 4,найдите расстояние от С до прямой D1E1

3769. ilo, 6 июня 2011, 11:07:33
3767. ilo, 6 июня 2011, 11:00:39
BF=a*sqrt(a) - надо так, выше-опечатка

BF=a*sqrt3- так вот правильно

3770. ПОЦЫК, 6 июня 2011, 11:14:51
Коментарий скрыт (грубость — 17%)показать

3771. белоус саша, 6 июня 2011, 11:20:15
помогите срочно!!!найти синус альфа если кос альфа равен минус два корня из 6 и поделить на 5 . альфа принадлежит пи на два и пи

3772. саша, 6 июня 2011, 11:24:39
3771. белоус саша, 6 июня 2011, 11:20:15
помогите срочно!!!найти синус альфа если кос альфа равен минус два корня из 6 и поделить на 5 . альфа принадлежит пи на два и пи

3773. саша, 6 июня 2011, 11:32:44
помогите срочно!!!найти синус альфа если кос альфа равен минус два корня из 6 и поделить на 5 . альфа принадлежит пи на три и пи

3774. Мадина, 6 июня 2011, 11:37:43
Прямоугольный параллелепипед описан около цилиндра, радиус основания и высота которого равна 2. найти объем параллелепипеда. ПОМОГИТЕ ПОЖАЛУЙСТА!

3775. юля, 6 июня 2011, 11:41:20
Найдите cоs а, если sin а = дробь корень 21/5 (21 под корнем) , и а принадлежит (П/2;П)
ПОМОГИТЕЕЕЕЕЕЕЕЕ ПОЖАЛУЙСТА ПРОШУ!!!!!!!!!

3776. катя, 6 июня 2011, 11:48:16
помогите.срочно.
Найти син, если кос корень 19 делить на 10.

3777. ilo, 6 июня 2011, 11:48:46
3771. белоус саша, 6 июня 2011, 11:20:15
помогите срочно!!!найти синус альфа если кос альфа равен минус два корня из 6 и поделить на 5 . альфа принадлежит пи на два и пи

0,2

3778. Мурад, 6 июня 2011, 11:49:24
найдите наибольшее

значение функции у =х(в кубе )-9х(в кв)+24х-7 на отрезке [-1;3] помогите

3779. Алия, 6 июня 2011, 11:49:42
В правильной шестиугольной призме все ребра которой равны 5 найти расстояние от точки А до С1Д1

3780. Евгений , 6 июня 2011, 11:50:44
3776. катя, 6 июня 2011, 11:48:16
помогите.срочно.
Найти син, если кос корень 19 делить на 10.



Катя ты промежуток укажи !!!

3781. ilo, 6 июня 2011, 11:51:23
3774. Мадина, 6 июня 2011, 11:37:43
Прямоугольный параллелепипед описан около цилиндра, радиус основания и высота которого равна 2. найти объем параллелепипеда. ПОМОГИТЕ ПОЖАЛУЙСТА!

32

3782. Дарья, 6 июня 2011, 11:51:53
из пункта А в пункт Б .расстояние между которыми 50км.одновременно выехали мотоцикл и велосипед. За час мотоцикл проезжает на 30км больше.чем велосипед. Определить скорость велосипеда.если известно.что он прибыл в пункт Б на 1.5часа позже мотоцикла.
Прошу помочь.

3783. Евгений , 6 июня 2011, 11:53:44
3782. Дарья, 6 июня 2011, 11:51:53
из пункта А в пункт Б .расстояние между которыми 50км.одновременно выехали мотоцикл и велосипед. За час мотоцикл проезжает на 30км больше.чем велосипед. Определить скорость велосипеда.если известно.что он прибыл в пункт Б на 1.5часа позже мотоцикла.
Прошу помочь.



тока что решал =))) Скорость велосипеда 20 км/ч

3784. ЮЛЯ, 6 июня 2011, 11:55:22
ДАРЬЯ СКОРОСТЬ 20 КМ/Ч

3785. Мурад, 6 июня 2011, 11:56:57
найдите наибольшее

значение функции у =х(в кубе )-9х(в кв)+24х-7 на отрезке [-1;3]

3786. Дарья, 6 июня 2011, 11:57:06
[/quote]


тока что решал =))) Скорость велосипеда 20 км/ч[/quote]
Можете решение написать?)Или очень большое?

3787. ilo, 6 июня 2011, 11:57:25
3778. Мурад, 6 июня 2011, 11:49:24
найдите наибольшее

значение функции у =х(в кубе )-9х(в кв)+24х-7 на отрезке [-1;3] помогите

13

3788. Таня, 6 июня 2011, 11:57:45
Наидите наибольшое значение функции y=xв кубе+15x-15. Прямая, перпендикулярная гипотинузе прямоугольного треугольника с катетами 5и 12,отсекает от него четырехугольник в который можно вписать окружность.наидите площадь этого четырех угольника

3789. Мурад , 6 июня 2011, 12:02:05
найдите наибольшее

значение функции у =х(в кубе )-9х(в кв)+24х-7 на отрезке [-1;3] решение может кто нибудь написать

3790. юля, 6 июня 2011, 12:02:38
Найдите cоs альфа, если sin альфа = дробь корень 21/5 (21 под корнем) , и альфа принадлежит (П/2;П)

3791. Евгений, 6 июня 2011, 12:03:49
Помогите "В прямоугольную призму вписан правильный цилиндр , радиус которого 3 см , объем прямоугольной призмы 36 см кубических , какова высота цилиндра ??? " Я лично написал на ЕГЭ , что 1 , ведь площадь призмы это произведение трех измерений (высота , ширь и длина ) 2 радуса это ширина 2 радиуса длина 6 x 6 ... А высота тогда должна быть 1 , я прав , прошу умного человека помочь ! =)))

3792. Евгений, 6 июня 2011, 12:07:21
Дарья , а какое решение эта задача решается аналитически ...
НУ уравнение ( 50/x минус 50/x+30 = 1,5 ) Решай
1 к общему знаменателю
2 у тебя получится квадратное уравнение
3 корень 20
4 отрицательный корень посторонний

3793. Дарья, 6 июня 2011, 12:10:17
Спасибо!

3794. Мурад, 6 июня 2011, 12:16:44
найдите наибольшее

значение функции у =х(в кубе )-9х(в кв)+24х-7 на отрезке [-1;3] решение может кто нибудь написать срочно прошу

3795. Инна, 6 июня 2011, 12:17:49
Помогите, пожалуйста! Найдите sin(альфа), если cos(альфа)=-квадратный корень из 7/4, альфа принадлежит промежутку (пи;3пи/2)
Заранее спасибо

3796. юля, 6 июня 2011, 12:22:51
В ходе распада радиоактивного изотопа его масса уменьшается по закону m(t)=m нулевое*2 в степени 1/T, где m нулевое(мг)-начальная масса изотопа, t(мин)-время, прошедшее от начального момента, T(мин)-период полураспада.В начальный момент времени масса изотопа m нулевое=250мг.Период его полураспада T = 2 мин.Через сколько минут масса изотопа будет равна 62,5 мг?

3797. Евгений , 6 июня 2011, 12:23:54
3794. Мурад, 6 июня 2011, 12:16:44
найдите наибольшее

значение функции у =х(в кубе )-9х(в кв)+24х-7 на отрезке [-1;3] решение может кто нибудь написать срочно прошу



11 ответ

3798. Саша 198, 6 июня 2011, 12:25:40
В правильной 6ти угольной призме абцдеф стороны основания равны 3,а боковые ребра 4 найдите расстояние от точки ц до прямоми д1е1..... пожалуйста если кто знает , подскажите ) времени мало....сижу на Егэ..)

3799. юля, 6 июня 2011, 12:25:59
В ходе распада радиоактивного изотопа его масса уменьшается по закону m(t)=m нулевое*2 в степени 1/T, где m нулевое(мг)-начальная масса изотопа, t(мин)-время, прошедшее от начального момента, T(мин)-период полураспада.В начальный момент времени масса изотопа m нулевое=250мг.Период его полураспада T = 2 мин.Через сколько минут масса изотопа будет равна 62,5 мг?

ПОМОГИТЕ...ОЧЕНЬ ПРОШУ

3800. Саша198, 6 июня 2011, 12:27:37
рямая перпендикулярна гипотенузе прямугольного треугольника с катетами 6 и 8 и отсекают от него 4 угольник в который можно вписать окружность найти площадь этого четырехугольник....еще одна ...заранее благодарен...

3801. Муурад, 6 июня 2011, 12:28:29
Евгений , 6 июня 2011, 12:23:54 цитата

3794. Мурад, 6 июня 2011, 12:16:44
найдите наибольшее

значение функции у =х(в кубе )-9х(в кв)+24х-7 на отрезке [-1;3] решение может кто нибудь написать срочно прошу



11 ответ
мне решение нужно срочно !!!!!

3802. Лиза, 6 июня 2011, 12:30:47
в треугольнике АВС AD-биссектриса угол С=97 градусов угол CAD=5 градусам найти угол В?

3803. Лиза, 6 июня 2011, 12:32:06
на доске написано более 40, но менее 48 целых чисел. среднее арифм. этих чисел равно -3, средн, арифм. всех положительных из них равно 4, а сред. арифм. всех отрицат. из них равно -8.
а) сколько чисел написано на доске?
б) каких чисел написано больше, положительных или отрицательных?
в) какое наибольшее кол-во положит. чисел может быть среди них?

3804. ilo, 6 июня 2011, 12:43:17
3794. Мурад, 6 июня 2011, 12:16:44
найдите наибольшее

значение функции у =х(в кубе )-9х(в кв)+24х-7 на отрезке [-1;3] решение может кто нибудь написать срочно прошу

производная 3x^2-18x+24=0 при x=2 и х=4, 4-не из отрезка [-1;3];f(2)=13;f(-1)=отрицательно; f(3)=11 наибольшее 13

3805. Кракозябра, 6 июня 2011, 12:44:09
ПОМОГИТЕ ПОЖАЛУЙСТА!!!. ЕГЭ ИДЕТ! Срочно нужна помощь!!!В правильной 6-угольной призме ABCDA1B1C1D1. Все рёбра =1. Найти расстояние от точки C до прямой A1 C1 это с2?????????????

3806. Кракозябра, 6 июня 2011, 12:47:07
Y = x2-2x2-4x-2 на отрезке [0;7] наименьшее значение в11 а это как решить? Помогите кто разбирается!!!

3807. ilo, 6 июня 2011, 12:54:38
3799. юля, 6 июня 2011, 12:25:59
В ходе распада радиоактивного изотопа его масса уменьшается по закону m(t)=m нулевое*2 в степени 1/T, где m нулевое(мг)-начальная масса изотопа, t(мин)-время, прошедшее от начального момента, T(мин)-период полураспада.В начальный момент времени масса изотопа m нулевое=250мг.Период его полураспада T = 2 мин.Через сколько минут масса изотопа будет равна 62,5 мг?

ПОМОГИТЕ...ОЧЕНЬ ПРОШУ

ответ4

3808. Саша198 , 6 июня 2011, 12:59:08
1)В правильной 6ти угольной призме абцдеф стороны основания равны 3,а боковые ребра 4 найдите расстояние от точки ц до прямоми д1е1..... пожалуйста если кто знает , подскажите ) времени мало....

2)прямая перпендикулярна гипотенузе прямугольного треугольника с катетами 6 и 8 и отсекают от него 4 угольник в который можно вписать окружность найти площадь этого четырехугольник....заранее благодарен...
PLEASE HELP!!!!!!!!!!!!

3809. Мухтар, 6 июня 2011, 13:10:56
Из пункта А в В расстояние между которыми 50 км, одновременно выехали мотоциклист и велосипедист. Известно что за час мотоциклист проезжает на 30 км больше, чем велосипедист. Определите скорость велосипедиста если, если известно, что он прибыл в пункт В на 1,5 часа позже мотоциклиста. Ответ дайте в км/ч

Помогите пожалуйста решить задачку очень срочно!

3810. артём, 6 июня 2011, 13:24:11
В ходе распада радиоактивного изотопа его масса уменьшается по закону m(t)=m нулевое*2 в степени -1/T, где m нулевое(мг)-начальная масса изотопа, t(мин)-время, прошедшее от начального момента, T(мин)-период полураспада.В начальный момент времени масса изотопа m нулевое=150мг.Период его полураспада T = 3 мин.Через сколько минут масса изотопа будет равна 37,5 мг?

3811. Руслан, 6 июня 2011, 13:26:33
В правильной шестиугольной призме ABCDEFA1B1C1E1D1,стороны основания которой равны 3,а боковые ребра=4,найдите расстояние от точки C до прямой A1B1

3812. Динара, 6 июня 2011, 13:28:17
Помогите пожалуйста с примером буду примного благодарна


(4sin(в кв) х+ 12 cos x + 15) - корень квадратный -17cos x=0

3813. Руш, 6 июня 2011, 13:30:54
В правильной шестиугольной призме abcdefa1b1c1d1e1f1 все ребра которой =1. найти расстояное от точки С до прямой а1b1. заранее спасибо!!!!!!

3814. Динара, 6 июня 2011, 13:31:29
cosx= -П(пи) чему равен х? спасибо

3815. Леонид , 7 июня 2011, 17:22:37
Уважаемый LEXXUS! Подскажите, что это за сайт, предлагающий варианты этого года, платный или бесплатный? Заманчиво - задания С. Просят № т/ф, тогда дадут код. ПОСОВЕТУЙТЕ. тhttp://test-online1.com/ege/number.php

3816. Леонид , 7 июня 2011, 17:35:20
Уважаемый LEXXUS! Затрудняемся в решении задачи С4 ЕГЭ этого года. Помогите. Интнресно, многиели справились с этой задачей? Вот эта задача: В треугольнике PQR угол R равен 90 гр. RA - высота, QB - медиана. АВ пересекает QR в точке С, принадлежащей лучу QR. Окружность проходит через точки А и В и касается QR в точке D. Найти DR, если ВС=25, АВ = 11.

Пожалуйста, помогите решением с чертежом. Заранее спасибо. Как то не равномерно, в других вариантах С4 значительно проще, чем эта.

3817. Goro, 7 июня 2011, 19:16:19
Коментарий скрыт (грубость — 16%)показать

3818. Lexxus, 7 июня 2011, 23:28:30
3816. Леонид , 7 июня 2011, 17:35:20
В треугольнике PQR угол R равен 90 гр. RA - высота, QB - медиана. АВ пересекает QR в точке С, принадлежащей лучу QR. Окружность проходит через точки А и В и касается QR в точке D. Найти DR, если ВС=25, АВ = 11.

Постараюсь без чертежа объяснить, а то рисовать особо негде.

Тут может быть два случая: PR > QR и PR < QR

Для обоих случаев:

AB - медиана прямоугольного треугольника PAR, значит, PB=BR=AB=11.
Из прямоугольного треугольника CRB находим
CR = sqrt(BC^2-BR^2) = sqrt(625-121) = 6*sqrt(14)

Для случая PR > QR:
По теореме о касательной и секущей, CD^2 = BC*AC
CD = sqrt(25*(25-11)) = 5*sqrt(14)
DR = CR-CD = sqrt(14).

Для случая PR < QR:
По теореме о касательной и секущей, CD^2 = BC*AC
CD = sqrt(25*(25+11)) = 5*6 = 30
DR = CD-CR = 30-6*sqrt(14).

Ответ:
sqrt(14), 30-6*sqrt(14)

3819. Леонид , 8 июня 2011, 12:33:04
Огромное СПАСИБО за 3818. Lexxus, 7 июня 2011, 23:28:30. Ёлки - палки, а задача то проще и емкость меньше, чем о трапеции и треугольнике. Бывает, не скумекаешь сразу маленькую догадку - здесь о медиане.

3820. КАТЯ, 10 июня 2011, 12:26:46
ПОЖАЛУЙСТА ПОМОГИТЕ НУЖНО ИМЕННО НА СЕЙЧАС ПОМОЧЬ МОЕЙ СЕСТРЕ НА КОНТРОЛЬНОЙ!
РЕШИТЕ ЗАДАЧУ ХОТЬ Б ОДНУ!
1.найти объем и площадь боковой поверхности конуса образующая которого равна 14 см и наклонена к плоскости основания под углом 60 градусов
2.высота цилиндра равна 6 см отрезок соединяющий центр верхнего основания цилиндра с точкой окружности нижнего основания образует с плоскостью основания угол 30 градусов.Найти радиус основания и длину проведенного отрезка
3.диаметр основания цилиндра равен 6 см а расстояние между центрами оснований 4 см.Найти диагональ осевого сечения и площадь полной поверхности цилиндра

3821. Евгения, 10 июня 2011, 16:39:29
дано:
cos альфа равен 0.5
270 градусов<альфа<360 градусов
Вычислить sin альфа?

3822. Леонид , 14 июня 2011, 14:13:38
Передышка!

3823. vakliotova, 15 июня 2011, 07:46:40
помогите срочно решить


Найти площадь боковой поверхности правильной треугольной пирамиды, у которой головая грань лежит под углом 60 градусов к плоскости основания, радиус окружности описаной около основания 30м

3824. Юлия, 15 июня 2011, 22:06:59
нужна срочно помочь)))кто соображает в логарифмах, нахождения точки максимума, уравнения всяких косинусов и т.п
пишите 55-98-98-272
заранее спасибо!

3825. юлия, 16 июня 2011, 17:22:04
основание пирамиды и квадрат со стороной равной 3 каждая боковая грань наклонена к плоскости основания под одним и тем же углом. тангенс которого равен 4/3 найдите площадь боковой поверхности пирамиды?

3826. юлия, 16 июня 2011, 17:26:04
3825. юлия, 16 июня 2011, 17:22:04
основание пирамиды и квадрат со стороной равной 3 каждая боковая грань наклонена к плоскости основания под одним и тем же углом. тангенс которого равен 4/3 найдите площадь боковой поверхности пирамиды?
помогите пжалуйста решить срочно

3827. Марина, 16 июня 2011, 17:50:13
В правильной треугольной усеченной пирамиде стороны оснований 4 и 1, боковое ребро 2. Найти высоту.

3828. Ксения, 16 июня 2011, 17:54:43
ПОМОГИТЕ ПЛИИИИИИИИЗ=( f(x)=1-x(в четвертой степени)-x(в шестой степени) в промежутке (-3;3).....тут типо надо сначала f'

3829. Кирилл, 16 июня 2011, 22:14:25
всем привет, нужна помощь.вот такая задача:
В правильной треугольной пирамиде сторона основания
равна 2 , а угол между боковыми рёбрами равен 900.Найти площадь боковой поверхности пирамиды.

3830. шаша+, 17 июня 2011, 01:02:04
Ксения: F(x)=1-X4степени-X6 степени на промежутке(-3;3) : Найдем производную: F*(х)=-4Хкуб-6Хв 5 степени, приравниваем к нулю, получим, что Х=0, одна точка, находится в заданном промежутке (-3;3), значит наибольшее значение функции в этой точке, равно F(0)=1.

3831. Элеонора, 17 июня 2011, 14:49:23
Всем привет,большая просьба,нужна ваша помощь
хочу подать апелляцию на результат ЕГЭ по математике,для этого нужно разобрать задания С1,С2
у меня есть фотографии заданий,пожалуйста,если можете помочь,пишите на почту,буду оооочень признательна
newrock666@yandex.ru

3832. Леонид , 17 июня 2011, 20:21:34
а чем помогут твои фото для апелляции с1 и с2, надо же иметь твои решения и с чем не согласен

3833. Леонид , 17 июня 2011, 20:26:03
и адрес твой не выдают

3834. Ксения, 19 июня 2011, 18:53:24
Необходимо найти сумму корней x=arctg4 + pi*k (k принадлежит Z) на отрезке [0; 120*pi] и выяснить что больше: эта сумма или число 23040.

Благодарю.

3835. Вано, 20 июня 2011, 15:33:18
Помогите мне решить плиз Найти значение производной функции
y=x+e(x-сверху)cosx в точке xo=pi/2

3836. таня , 27 июня 2011, 23:05:24
Сторона квадрата равна 4 см. Точка, равноудаленная от всех вершин квадрата, находится на расстоянии 6 см от точки пересечения его диагоналей. Найдите расстояния от этой точки до вершин квадрата. ПОМОГИТЕ РЕШИТЬ ПОЖААААЛУЙСТА!!

3837. Леонид , 28 июня 2011, 02:00:57
задача С4 на ЕГЭ. Ваш вариант решения, LEXXUS, пожалуйста-случай, когда прямая проведена через вершину большего основания трапеции и центрр окружности.
Окружность радиуса 6 вписана в равнобедренную трапецию, большее основание которой 18. Прямая, проходящая через центр окружности и вершину трапеции, отсекает от трапеции треугольник. Найдите отношение площади этого треугольника к площади трапеции.

3838. aliksitar, 28 июня 2011, 21:56:40
Ln(0.225)=2.303 Lgx как преобразовать логарифм?

3839. Леонид , 29 июня 2011, 12:30:02
Lexxus, обратите, пожалйста, внимание на запись №3837! Очень нужно!

3840. Роджер, 29 июня 2011, 16:30:56
пожалуйста помогите решить задачу . Дан куб АВСДА1В1С1Д1.точки К.Л.М- середины ребер АД,Д1С1,В1С1 соответственно.точка Р- пересечение диагоналеи АА1В1В.определите взаимное расположение прямых а)В1К и ДС б)В1Д1 и А1С1

3841. роджер, 29 июня 2011, 16:35:41
желательно если это возможно прямо щас,буду очень благодарен

3842. роджер, 29 июня 2011, 17:01:23
очень жду

3843. роджер, 29 июня 2011, 21:48:45
блин срочно нужно решение задачи

3844. Хряпа, 29 июня 2011, 22:13:30
Роджер, 29 июня 2011, 16:30:56

А чего тут решать? B1K и DC - скрещивающиеся, B1D1 и A1C1 перпендикулярные. И нафига столько лишних точек в условии? O_o

3845. роджер, 30 июня 2011, 07:26:21
хряпа я тоже так думал что так просто. Там решения какие то должны быть

3846. Леонид , 30 июня 2011, 07:48:20
L E X X U S ! Пожалуйста, посмтрите запись обращение № 3837, 3839

3847. Леонид , 1 июля 2011, 23:53:22
Вы все в отпске? Или просто не хотите отвечать?

3848. Дима хочу в МИФИ, 4 июля 2011, 18:00:10
Lexxus что можете сказать о вашем институте смогу ли я с 238 баллов туда поступить?

3849. Lexxus, 5 июля 2011, 10:24:36
3837. Леонид , 28 июня 2011, 02:00:57
Ваш вариант решения, LEXXUS, пожалуйста-случай, когда прямая проведена через вершину большего основания трапеции и центрр окружности.
Окружность радиуса 6 вписана в равнобедренную трапецию, большее основание которой 18. Прямая, проходящая через центр окружности и вершину трапеции, отсекает от трапеции треугольник. Найдите отношение площади этого треугольника к площади трапеции.

3846. Леонид , 30 июня 2011, 07:48:20
L E X X U S ! Пожалуйста, посмтрите запись обращение № 3837, 3839
3847. Леонид , 1 июля 2011, 23:53:22
Вы все в отпске? Или просто не хотите отвечать?

Я не в отпуске. Просто сильно занят решением задач совсем другого рода.

Попробую на пальцах объяснить.

Пусть трапеция ABCD, AD = 18, прямая проходит через вершину A и пересекает сторону CD в точке E.
1. Найдем BC. Это можно сделать либо через тангенс угла A (зная, что тангенс угла A/2 равен отношению радиуса вписанной окружности к половине большего основания), либо составив систему уравнений из условий равенства сумм оснований и боковых сторон трапеции и теоремы Пифагора.
В любом случае, BC = 8.
AB = CD = (AD+BC)/2 = 13

2. Продолжим AE и BC до пересечения в точке F. Углы BFA и BAF равны. Значит, треугольник ABF - равнобедренный.

3. Треугольники AED и FEC подобны, а сумма их высот, опущенных из точки E, равна высоте трапеции (2r = 12). Зная коэффициент подобия, найдем высоту треугольника AED.

CF = BF - BC = AB - BC = 5.
Коэффициент подобия равен CF/AD = 5/18

Составляем систему уравнений
{x+y = 12, x/y = 5/18}

Решаем, получаем высоту треугольника AED = 216/23

4. Отношение площадей
(18*216/23/2)/(12*(18+8)/2) = 162/299

Как-то так.

3850. Леонид , 6 июля 2011, 07:57:59
Спасибо! Моё решение - ВС без тангенса, проведя ВО и через подобие, достроив треугольник подобный АОД. Ваш подход мне тоже понравился. Ещё раз Большое Спсибо. Хочу попросить ещё одну задачу

С 4. Окружность радиуса 6 вписана в равнобедренную трапецию, большее основание которой 18. Прямая, проходящая через центр окружности и вершину трапеции, отсекает от трапеции треугольник. Найдите отношение площади этого треугольника к площади трапеции.

3851. Lexxus, 6 июля 2011, 10:07:57
3850. Леонид , 6 июля 2011, 07:57:59

С 4. Окружность радиуса 6 вписана в равнобедренную трапецию, большее основание которой 18. Прямая, проходящая через центр окружности и вершину трапеции, отсекает от трапеции треугольник. Найдите отношение площади этого треугольника к площади трапеции.

Это у вас юмор такой? Не смешно.

3852. Леонид , 6 июля 2011, 16:20:20
Нет, УВАЖАЕМЫЙ, не юмор. Маху да. Извиняюсь! За заскок. Вот она:

С 4. Прямая, перпендикулярная боковой стороне равнобедренного треугольника с основанием, равным 12, отсекает от него четырёхугольник, в который можно вписать окружность.
Найдите площадь этого четырехугольника, если радиус окружности равен 3.

3853. Lexxus, 6 июля 2011, 19:12:42
3852. Леонид , 6 июля 2011, 16:20:20
С 4. Прямая, перпендикулярная боковой стороне равнобедренного треугольника с основанием, равным 12, отсекает от него четырёхугольник, в который можно вписать окружность.
Найдите площадь этого четырехугольника, если радиус окружности равен 3.

На каком этапе решения этой задачи у вас возникли трудности?

3854. Леонид , 6 июля 2011, 19:19:31
Выделив треугольники в четырёхугольнике (к 1/2rР) - равенство касательных, отрезок Х - затруднения в хорошей паре подобных треуг-в. Как иначе попроще и покороче пока не придумал. Спасибо! Не знаю, понятно ли высказался.

3855. Олег, 8 июля 2011, 01:30:25
Почему в задании № 266347 не написаны угловые параметры параллелепипеда? Ошибка составителей или параллелепипеды бывают простые и косоугольные вместо обычные и прямоугольных?
Надо составителя попросит решить задачу "Найти площадь треугольника ABC где AB=8, а AC=5."

3856. Леонид , 8 июля 2011, 15:22:06
LEXXUS! позалуйста, запись №3853 и 3854

3859. Lexxus, 8 июля 2011, 19:37:03
3856. Леонид , 8 июля 2011, 15:22:06
LEXXUS! позалуйста, запись №3853 и 3854

Разбиваем четырехугольник на четыре пары равных прямоугольных треугольников. У двух пар мы знаем катеты, ещё у двух пар мы знаем один катет и можем найти углы, зная, что окружность с известным радиусом вписана в равнобедренный треугольник с известным основанием.
Я не вижу в этой задаче вообще никаких сложностей.

3860. Леонид , 8 июля 2011, 21:48:48
Да, это всё понятно! "...ещё у двух пар мы знаем один катет и можем найти углы,- КАК? "... зная, что окружность с известным радиусом вписана в равнобедренный треугольник с известным основанием".- НУ И ЧТО? У меня проблема с парой равных треугольников - катеты - отрезки касательных.Спасибо за ответ, но ответа пути решения я не увидел.

3861. Дамир, 11 июля 2011, 10:46:35
помогите кто может??? Скорость автомобиль ,разгоняящегося с места старта по прямолинейному отрезку пути длинной l км с постоянным ускорением а км/ч2..вычисляется по формуле v= корень2la. Определите наименьщее ускорение с которым должен двигаться автомобиль.чтобы проехав 0,4 км.преобрести скорость не менее 160км.ч

3862. dnf, 16 июля 2011, 01:32:00
В задании 7733 http://live.mephist.ru/show/mathege2010/view/id/7733/ коллективный разум ошибается.
«На рисунке изображен график производной функции f(x), определенной на интервале (-9,2). В какой точке отрезка [-8, -2] f(x) принимает наибольшее значение.»
График на рисунке на отрезке лежит ниже нуля и только в точке -2 приходит в ноль. То есть функция на этом отрезке убывает, и наибольшее значение у неё в начале отрезка (-8), а большинство считает, что в конце (-2).


3863. Александр, 17 июля 2011, 11:20:58
Клиент взял в банке кредит 60000 рублей на год под 15% годовых. Он должен погашать кредит, внося в банк ежемесячно одинаковую сумму денег, с тем чтобы через год выплатить всю сумму, взятую в кредит, вместе с процентами. Сколько он должен вносить в банк ежемесячно?

Эта задача не совсем корректная, ответ 5750 руб. который указан как правильный на самомо деле неправильный. Банки так не считают. Решение более сложное чем простое умнгожение на 1,15 и деление на 12



3864. Всем привет!, 14 августа 2011, 04:12:22
Помогите найти дифференциалы функций
а)f(x)=x^2+х+1
и найти первообразные функции
а)f(x)=х/&#8730;(1^ +х^2 )

3865. Яна, 14 августа 2011, 04:25:49
Помогите найти дифференциалы функций
а)f(x)=x^2+х+1
и найти первообразные функции
а)f(x)=х/&amp;#8730;(1^ +х^2 )[/quote]

3866. Андрей, 16 августа 2011, 21:25:05
чему равна f(2) Если f(x) = корень 3-x? помогите пожалуста

3867. Rosinka*, 18 августа 2011, 23:37:35
помогите пожалуйста Определить значение выражения z= x в кубе минус 12x , если x равен 9 плюс корень из 17, все под корнем во второй степени плюс 9 минус корень из 17, все под корнем

3868. Леонид , 29 августа 2011, 07:21:06
Уважаемый, Lexxus! Появился проект или уже демоверсия на 2012. Вопросы: 1)срзранится ли эта страница - ибо произошли изменения?
2)что будет с открытым банком заданий?
Изменения не существены, нельзя ли сохранить номера, добавив просто В13 и В14. ?

3869. Lexxus, 2 сентября 2011, 19:20:33
3868. Леонид , 29 августа 2011, 07:21:06
Уважаемый, Lexxus! Появился проект или уже демоверсия на 2012. Вопросы: 1)срзранится ли эта страница - ибо произошли изменения?
2)что будет с открытым банком заданий?
Изменения не существены, нельзя ли сохранить номера, добавив просто В13 и В14. ?

Всё сохранится, новые задания добавятся к существующим. Работаем в обычном режиме.

3870. диана, 7 сентября 2011, 16:36:29
помогите пожалуйста найти множество значений функции
f(x)= 4 sin^2x-3sinx+2cos^2x

3871. Диана, 7 сентября 2011, 17:51:18
дан треугольник со сторонами 5,12 ,13.точка о лежит на большей стороне треугольника и является центром окружности касающихся двух других сторон.найдите радиус окружности

3872. динар, 7 сентября 2011, 19:34:30
найдите значение параметра а при которых касательная к графику функции y=cos7x+7 в точке с абсциссой а паралельна касательной к этому графику в точке с абсциссой ПИ/6

3873. Динар, 8 сентября 2011, 13:05:19
все не надо :)

3874. Cоня, 11 сентября 2011, 14:12:05
докажите что сумма 34 умножить на 85+34умноженое на36 делится на 11

3875. b spline, 11 сентября 2011, 15:33:36
3874. Cоня, 11 сентября 2011, 14:12:05
докажите что сумма 34 умножить на 85+34умноженое на36 делится на 11


34(85+36)=34*121 121 делится на 11, значит и произведение тоже

3876. Алёна, 12 сентября 2011, 14:44:37
Я не могу решить "Контрольную работу по алгебре и началам анализа за год. 10 класс(в форме ЕГЭ)(учебник Мардкович А.Г.) 1 вариант часть 1. " помогите пожалуйста, мне только часть А и часть В. за ранее спасибо.

3877. Ольга, 16 сентября 2011, 13:17:23
V=одна третья умножить100см умножить 16см
помогите решить

3878. Galina, 17 сентября 2011, 22:58:30
Помогите, пожалуйста, разобраться с "ЕГЭ по математике. 2009 год. Диагностическая работа" с заданием B4:Найдите сумму корней уравнения
(x+2)^1/3+(2x+1)^1/3=(3x+3)^1/3
Дан ответ -2,5. У меня получилось 3 корня -2; -1/2; -1. Получается сумма равна -3,5. Где ошибка?
Заранее благодарна за ответ

3879. Алла, 18 сентября 2011, 16:52:00
Galina, 17 сентября 2011, 22:58:30
У меня такие же ответы получились: -2; -1/2; -1.
Может быть ошибка в предложенных ответах?

3880. Владимир, 19 сентября 2011, 18:50:26
http://live.mephist.ru/show/mathege2010/view/id/17155/
http://live.mephist.ru/show/mathege2010/view/id/19269/
Прошу написать ход решений этих заданий т.к у меня получилось 16 и 1 соответственно и я почему то уверен что я прав)

3881. Ляйсан, 20 сентября 2011, 14:35:55
1.Точки А(14;-8;-1),В(7;3;-1),С(-6;4;-1),D(1;-7;-1) являются вершинами ромба ABCD.Найти острый угол ромба.
2.В кубе ABCDA1B1C1D1 точка М центр грани BB1CC1.Вычислите угол между векторами MD и BB1.

3882. Елена, 20 сентября 2011, 15:21:49
В-10 № 43967 Ответ 1.58

3883. Александр, 26 сентября 2011, 11:41:45
Найдите корень уравнения: под корнем 72-17х без корня = 2

3884. Александр, 26 сентября 2011, 11:45:25
В9. Два ребра прямоугольного параллелепипеда, выходящие из одной вершины, равны 3 и 4. Площадь поверхности этого параллелепипеда равна 94. Найдите третье ребро, выходящее из той же вершины.

В12. Имеются два сосуда. Первый содержит 30 кг, а второй — 20 кг раствора кислоты различной концентрации. Если эти растворы смешать, то получится раствор, содержащий 68% кислоты. Если же смешать равные массы этих растворов, то получится раствор, содержащий 70% кислоты. Какова концентрация кислоты (в процентах) в первом сосуде?


HELP!!!

3885. Леонид , 26 сентября 2011, 20:45:50
3883. Александр, 26 сентября 2011, 11:41:45
Неизвестное ребро Х, пл. пов-ти - уравнение 94 = 2*3*4 + 2*3*x + 2*4*x,
94= 24 +6x + 8x, 70=14x, x = 5.

Уравнение - возведи обе части в квадрат 72 -17х = 4, 17х = 68, х=4

Задачу долго писать.

3886. Kattu22, 29 сентября 2011, 22:35:44
Lexxus! Спасибо что подправили задания в соответствии с открытым банком заданий. Исправьте, пожалуйста, ошибку в номере 7395. Спасибо

3887. Lexxus, 30 сентября 2011, 00:29:49
3886. Kattu22, 29 сентября 2011, 22:35:44
Lexxus! Спасибо что подправили задания в соответствии с открытым банком заданий.

Не стоит благодарности. Давно пора было, но руки всё никак не доходили.

3886. Kattu22, 29 сентября 2011, 22:35:44
Исправьте, пожалуйста, ошибку в номере 7395. Спасибо

Сделано, спасибо.

3888. OS, 1 октября 2011, 05:43:12
3884. Александр, 26 сентября 2011, 11:45:25

В12. Имеются два сосуда. Первый содержит 30 кг, а второй — 20 кг раствора кислоты различной концентрации. Если эти растворы смешать, то получится раствор, содержащий 68% кислоты. Если же смешать равные массы этих растворов, то получится раствор, содержащий 70% кислоты. Какова концентрация кислоты (в процентах) в первом сосуде?


HELP!!!


с1-процентная концентрация 1-го раствора, с2- 2-го; система: 1-ое ур-е:
0,01*с1*30+0,01*20=0,68*(30+20); 2-е:0,01*с1*m+0,01*с2*m=0,7*(m+m); после упрощения 1-ое:3с1+2с2=340; 2-е: с1+с2=140; С2=80%; С1=60%.

3889. Kattu22, 1 октября 2011, 18:34:40
Lexxus! Есть такая проблема. Не все задания В5 старого банка перешли в В4 нового. Следовательно, сейчас в В5 путаница. Там встречаются и уравнения и задачи с практическим применением. Исправьте, пожалуйста! Спасибо!

3890. Kattu22, 2 октября 2011, 18:23:51
Lexxus! Есть такая еще одна проблема. Не все задания В4 старого банка перешли в В6 нового. Следовательно, сейчас в тоже В4 путаница. Там встречаются как задачи с практическим содержанием, так и геометрические задачи. Исправьте, пожалуйста! Спасибо!

3891. ЕЫК, 3 октября 2011, 17:44:53
С4
Высоты АА1 ВВ1, СС1 остроугольного
треугольника ABC пересекаются в точке Н так, что С1Н : НС =1:2,
АН= 6, НА1 = 3, АС = 12. Найдите радиус окружности,
описанной около треугольника С1ВА1.

Треугольник АА1С - прямоугольный АА1 = 9(катет); АС = 12(гипот)
СА1 = 3sqrt(7);
Треугольник СА1Н - прямоугольный СА1 = 3sqrt(7)(катет); НА1 = 3(катет);
СН = 6sqrt(2);
СН+НС1 = 6sqrt(2) + 3sqrt(2) = 9sqrt(2) ~ 12,728; => CC1>AC т.е катет больше гипотенузы.
Где у меня ошибка подскажите плиз

3892. Lexxus, 3 октября 2011, 23:22:09
3891. ЕЫК, 3 октября 2011, 17:44:53
С4
Высоты АА1 ВВ1, СС1 остроугольного
треугольника ABC пересекаются в точке Н так, что С1Н : НС =1:2,
АН= 6, НА1 = 3, АС = 12. Найдите радиус окружности,
описанной около треугольника С1ВА1.

Ошибка в условии.

3893. Lexxus, 3 октября 2011, 23:23:06
3889. Kattu22, 1 октября 2011, 18:34:40
Lexxus! Есть такая проблема.
3890. Kattu22, 2 октября 2011, 18:23:51
Есть такая еще одна проблема.

Кажется, всё поправил.

3894. Kattu22, 3 октября 2011, 23:28:28
Lexxus, 3 октября 2011, 23:23:06
Спасибо!

3895. iras, 5 октября 2011, 16:28:26
добрый день! помогите с вычислением производной функции при х=1: f(x)=x3-2cosx. заранее спасибо.

3896. astk8, 7 октября 2011, 21:52:10
как решаются вот такие уравнения
y=2 ^( 1-4х-х^(2))-21

3897. OS, 7 октября 2011, 23:38:50
3896. astk8, 7 октября 2011, 21:52:10
как решаются вот такие уравнения
y=2 ^( 1-4х-х^(2))-21

Это задание на нахождение наибольшего значения ф-ции.
Обозначим -x^2-4x+1=t, -беск.<t<5; 0<2^t<32; -21<2^t<11. Наибольшее 11.

3898. Татьяна, 10 октября 2011, 11:29:38
решение функции y=sinx на промежутке [п/2; 5п/6]

3899. Женя, 10 октября 2011, 19:10:14
Помогите пожалуйста!!!! =(
в прямом параллелепипеде стороны основания равны 5 и 6 см образуют угол 30 градусов. большая диагональ параллелепипеда образует с основанием угол 45 градусов.найти S боковое.

3900. Kattu22, 11 октября 2011, 17:45:22
Lexxus! Исправьте, пожалуйста, ответы к заданиям 6987, 7363. Спасибо

3901. ирина иванова, 12 октября 2011, 20:26:25
помогите пожалуйста! разобраться в решениях заданий В10 (теория вероятностей) не могу справиться с задачами. спасибо.

3902. Lexxus, 13 октября 2011, 17:04:34
3901. ирина иванова, 12 октября 2011, 20:26:25
помогите пожалуйста! разобраться в решениях заданий В10 (теория вероятностей) не могу справиться с задачами. спасибо.

Если в среднем из 500 собранных Васей грибов 12 оказываются белыми, 24 - подосиновиками, 17 - подберёзовиками, а остальные - бледными поганками, то вероятность откинуться при употреблении одного случайно выбранного гриба из числа собранных Васей равна

(500-12-24-17)/500 = 0.894 (что равно 89.4 процента)

Подавляющее большинство задач B10 решаются точно также, остальные - самую капельку посложнее, но логика такая же.

Главное - во всех этих задачах нужно найти именно вероятность, то есть число, не меньшее нуля и не большее единицы.

3903. Kattu22, 13 октября 2011, 21:27:33
Lexxus! Исправьте, пожалуйста, ошибку в номере 5406. Спасибо!

3904. Lexxus, 13 октября 2011, 22:58:47
3903. Kattu22, 13 октября 2011, 21:27:33
Lexxus! Исправьте, пожалуйста, ошибку в номере 5406. Спасибо!

Исправил. Предыдущие тоже.
Огромное спасибо за внимание к сервису.

3905. Kattu22, 14 октября 2011, 16:52:31
Lexxus! Исправьте, пожалуйста, ошибку в номере 21863. Спасибо

3906. екатерина, 14 октября 2011, 19:46:54
решите ур-е xy-x-y=2011 и найдите такое решение, при котором x(в квадрате)+y минимально

3907. Lexxus, 14 октября 2011, 21:00:43
3906. екатерина, 14 октября 2011, 19:46:54
решите ур-е xy-x-y=2011 и найдите такое решение, при котором x(в квадрате)+y минимально

Это задание бессмысленно. Выражение x^2+y, где x и y удовлетворяют приведенному уравнению, может принимать любое значение от минус бесконечности до плюс бесконечности.

3908. куки, 15 октября 2011, 13:04:52
задачка !вася задумал число, прибавил к нему 1,сумму умножел на 2,произведение разделил на 3 и отнял от результата 4.Получил 6 .какое число задумал вася?помогите пожайлуста!!!!плиизз

3909. Алексей, 15 октября 2011, 14:54:21
3908. куки, 15 октября 2011, 13:04:52
задачка !вася задумал число, прибавил к нему 1,сумму умножел на 2,произведение разделил на 3 и отнял от результата 4.Получил 6 .какое число задумал вася?помогите пожайлуста!!!!плиизз


Пусть х-это искомое число. вася прибавил к Х 1 сумму умножил на 2, произведение разделил на три вычел 4 и получил 6, значит (Х+1)*2/3-4=6 , (Х+1)*2/3=10 , (Х+1)*2=30; (Х+1)=15; Х=14. ответ: это число 14

3910. куки , 15 октября 2011, 16:25:56
№1задача.4карандаша и 3тетради стоят 96 руб.в 2 карандаша и 2 тетради 54 руб.сколько стоят 8 карандашей и 7тетрадей?
№2задача. три сосуда вместимостью 20 л. наполнены водой,причём в первом 11 л.во втором 7 л. а в третьем 6л.как разлить воду поровну по трём сосудам если разрешается переливать в сосуд только токое количество воды сколько в нём уже имеется ?
№3задача . назделите головку сыра тремя разрезами на 8 равных частей.
к задачи №3 покажите пожайлуста рисунок!!!!!!!!!!!плиизз!!!!!!!!!!:)

3911. куки , 15 октября 2011, 17:33:26
помогите пожайлуйста плиизз а то мне 2 поставят

3912. Михаил, 15 октября 2011, 19:13:30
корень угла A2 чему равна

3913. OS, 15 октября 2011, 19:49:28
В № 4053 Найти наименьшее значение функции y=(3x^2-18x+18)*e^(4-x);
y'=(6x-18)*e^(4-x)+(3x^2-18x+18)*e^(4-x)*(-1)=e^(4-x)*(6x-18-3x^2+18x-18)=
=e^(4-x)*(-3x^2+24x-36); y'=0 x=2,x=6 В х=2 знак производной меняется с + на - это точка max, а в ответе min. Где ошибка?

3914. Алексей, 15 октября 2011, 22:40:15
фигово то что правильный ответ определяется на основе какой ответ был получен от пользователей больше всего! Например я уверен в задании что сделал его правильно B13 ведь скорость не может быть отрицательной но как показал правильный ответ и такой может быть только в ответ записали без знака минус) как понимать...

3915. OS, 16 октября 2011, 08:01:14
3914. Алексей, 15 октября 2011, 22:40:15
фигово то что правильный ответ определяется на основе какой ответ был получен от пользователей больше всего! Например я уверен в задании что сделал его правильно B13 ведь скорость не может быть отрицательной но как показал правильный ответ и такой может быть только в ответ записали без знака минус) как понимать...

№ задания укажи

3916. куки , 16 октября 2011, 15:03:14
помогите плииззз умаляю !!!!!!!!!!!!!!!!!!помогите плиизззззззззззззз кто может !!!!!!!!!!!!!!!!!!! плиизззззззззз умаляю!!!!!!!!!!!

3917. катенька, 16 октября 2011, 17:23:47
в пространстве произвольным образом расположены два правильных n-угольника, расстояние между центрами которых равно d. найдите длину суммы всех векторов, начала которых лежат в вершинах первого, а концы- в вершинах второго n-угольника

3918. алекс, 16 октября 2011, 17:35:45
Три сосуда вместимостью 20 литров наполнены водой , причём в первом - 11литров, во втором - 7 литров,в третьем-6литров.Как разлить воду поровну по трём сосудам,если разрешается переливать в сосуд только токое количество воды,Сколько в нём уже имеется?


4 карандаша и з тетради стоят -96рублей., в два карандаша и две тетради-54рубля.Сколько стоят восемь карандашей и семь тетрадей ?

ПОМОГИТЕ ПОЖАЙЛУСТА
3914 АЛЕКСЕЙ!!!!!!!!!!!!!!!!!!!!!!!! это не куки!!!!!!!!!!

3919. алекс, 16 октября 2011, 18:02:29
или другой кто небуть помогите пожайлуста

3920. Lexxus, 17 октября 2011, 02:37:43
3917. катенька, 16 октября 2011, 17:23:47
в пространстве произвольным образом расположены два правильных n-угольника, расстояние между центрами которых равно d. найдите длину суммы всех векторов, начала которых лежат в вершинах первого, а концы- в вершинах второго n-угольника

Каждый такой вектор можно представить в виде суммы:
(вектор с началом в какой-либо вершине первого многоугольника и концом в его центре) + (вектор с началом в центре первого и концом в центре второго многоугольника) + (вектор с началом в центре и концом в какой-либо вершине второго многоугольника).

Всего таких векторов n^2.

Сумма всех первых и третьих слагаемых даёт 0, остаётся n^2 одинаковых векторов длины d. Ответ: d*n^2

3921. Lexxus, 17 октября 2011, 02:40:27
3914. Алексей, 15 октября 2011, 22:40:15
фигово то что правильный ответ определяется на основе какой ответ был получен от пользователей больше всего!

Я не буду вдаваться в подробности, но это не так. Там ещё учитывается "весомость мнения" каждого ответившего, вычисляемая на основании предыдущих их ответов.

3922. рома_вес, 18 октября 2011, 00:05:27
Задание B1 (20515)
неверный ответ определен...

3923. OS, 18 октября 2011, 15:44:17
3922. рома_вес, 18 октября 2011, 00:05:27
Задание B1 (20515)
неверный ответ определен...

Больному прописано0,25*3*21=15,75г;В упаковке 0,25*40=10г Наименьшее число упаковок - 2. Одной мало будет, чтобы вылечиться.Ответ верный.

3924. Ульяна, 18 октября 2011, 18:54:07
На изготовление 475 деталей первый рабочий затрачивает на 6 часов меньше, чем второй рабочий на изготовление 550 таких же деталей.Известно,что первый рабочий за час делает на 3 детали больше ,чем второй.Сколько деталей в час делает первый рабочий?
Как решить эту задачу системой?

3925. Kattu22, 18 октября 2011, 19:48:45
Lexxus! Есть проблема с текстом задания 28621. В открытом банке скорость равна 3, а на этом сайте, скорость равна 4. С числом 4 эту задачу в рамках ЕГЭ решить нельзя, ответ будет не целым числом. Исправьте, пожалуйста! Спасибо.

3926. Lexxus, 18 октября 2011, 22:38:32
3925. Kattu22, 18 октября 2011, 19:48:45
Есть проблема с текстом задания 28621. В открытом банке скорость равна 3, а на этом сайте, скорость равна 4. С числом 4 эту задачу в рамках ЕГЭ решить нельзя, ответ будет не целым числом. Исправьте, пожалуйста! Спасибо.

Вот этого я и боялся - вместо того, чтобы просто удалить старое задание и добавить новое, как они делали прежде, они начали исправлять тексты.
А сверять текст каждого задания у меня никаких вычислительных ресурсов не хватит.
Ладно, будем надеяться, что таких правок там не так уж много.

3927. Kattu22, 19 октября 2011, 00:44:36
Я готова, если мне или моим друзьям будут попадаться такие задания, о них сообщать. Но исправить текст такой задачи на этом сайте реально? Как я заметила, номер 28621 не исправлен.

3928. Lexxus, 19 октября 2011, 14:15:45
3927. Kattu22, 19 октября 2011, 00:44:36
Как я заметила, номер 28621 не исправлен.

Теперь исправлен.

3929. катенька, 19 октября 2011, 16:44:16
Прямая, параллельная основанию треугольника, делит его площадь пополам. В каком отношении она делит боковые стороны треугольника

3930. макс18, 19 октября 2011, 16:48:18
стрелок 10 раз выстрелил по стандартной мишени и выбил 90 очков. сколько было попаданий в семерку, восьмерку и девятку , если десяток было четыре, а других попаданий и промахов не было?

3931. @НгЕлОчЕк МеК@, 19 октября 2011, 20:08:35
Помогите пожалуйста кто нить... Мне срочно надо решить модульную работу...Заранее спасибо!!! а вот это надо и решить >>> Степенная функция y=x в степени -2n (n-натуральное число). Свойство. График. Исследование.

3932. @НгЕлОчЕк МеК@, 19 октября 2011, 20:16:52
и вот еще это надо... Определите знак sin600

3933. ОЛЬГА, 19 октября 2011, 21:17:35
Р/Б ТРАПЕЦИЯ ОПИСАНА ОКОЛО ОКРУЖНОСТИ РАДИУСОМ 2.НАЙДИТЕ S ТРАПЕЦИИ,ЕСЛИ КОСИНУС УГЛА ПРИ БОЛЬШОМ ОСНОВАНИИ ТРАПЕЦИИ РАВЕН 0,6

3934. оля, 20 октября 2011, 14:46:14
На стороне ВС треугольника АВС отмечена точка К. Известно, что сумма углов В и С равна углу АКВ, АК = 5 м, ВК = 16 м и КС = 2 м. Найдите сторону АВ.

3935. Анюта, 20 октября 2011, 20:48:11
помогите пожалуйста: площадь боковой поверности прав. треугольника призмы равна 48 см кв. найти длину бокового ребра призмы, если длина стороны основания равна 4 см.

3936. Kattu22, 20 октября 2011, 23:57:18
Lexxus! Исправьте, пожалуйста, ответ к задаче 283655. Спасибо

3937. лера, 21 октября 2011, 00:24:04
помогите пожалуйста решить....
в прямоугольном параллепипеде стороны основания равны 5 см и 12см, а диагональ параллепипеда наклонена к плоскости основания под углом 45 градусов. найдите высоту параллепипеда

3938. Анастасия, 21 октября 2011, 12:50:12
Здравствуйте! Помогите пожалуйста освоить Логарифмы. Мне примеры как решать, а дальше я сома разберусь!
Log3(1+x)=2 или Log5(-1+x)=2, Log7(x+9)= Log7(2х-9) (B5)
64 log4 13 (В общем B7)

И ещё задачи B10. Пример с этого сайта:
1.В чемпионате по гимнастике участвуют 64 спортсменки: 23 из Норвегии, 25 из Дании, остальные — из Швеции. Порядок, в котором выступают гимнастки, определяется жребием. Найдите вероятность того, что спортсменка, выступающая первой, окажется из Швеции.

2.Фабрика выпускает сумки. В среднем на 180 качественных сумок приходится две сумки со скрытыми дефектами. Найдите вероятность того, что купленная сумка окажется качественной. Результат округлите до сотых.

Заранее спасибо!

3939. Lexxus, 21 октября 2011, 14:51:00
3936. Kattu22, 20 октября 2011, 23:57:18
Lexxus! Исправьте, пожалуйста, ответ к задаче 283655. Спасибо

Ох, беда. Почти все поголовно в этих задачах ошибаются :(

Исправил.

3940. Lexxus, 21 октября 2011, 15:00:15
3938. Анастасия, 21 октября 2011, 12:50:12
Фабрика выпускает сумки. В среднем на 180 качественных сумок приходится две сумки со скрытыми дефектами. Найдите вероятность того, что купленная сумка окажется качественной. Результат округлите до сотых.

Вот, как раз.

Чтобы решить задачу, надо найти отношение количества качественных сумок к общему количеству сумок.
В задаче сказано, что в среднем выпускается 180 качественных и 2 бракованные. То есть в сумме - 180+2 = 182.
{качественные}/({качественные}+{бракованные}) = 180/182 ~ 0.98901..
Округляем до сотых, получаем 0.99.

Как я уже выше написал, почему-то большинство людей в этих задачах про сумки принимают первое число за общее количество, и делят
({качественные}-{бракованные})/{качественные}.
Это, само собой, неправильно.

3941. Анастасия, 21 октября 2011, 16:11:42
Lexxus, спасибо!

3942. татьяна, 21 октября 2011, 18:25:31
как исследовать ф-ию на экстемум y=3*x^2+3*x-1

3943. Тимур, 22 октября 2011, 19:58:45
На клетчатой бумаге изображён прямоугольник m x n клеток (m и n взаимно просты m>n) диагональ прямоугольника НЕ ПЕРЕСЕКАЕТ ровно 116 клеток прямоугольника. Найдите все возможные m и n

3944. Тимур, 23 октября 2011, 15:23:39
А всё не надо сам решил =)
Кому интересно:

Т.к m и n взаимно просты, диогональ не будет проходить через узлы клеток, т.е общее число клеток, которые пересечёт диогональ равно - m+(n-1)
Клетки которые диогональ не пересечёт mn - m - n + 1 = (m-1)(n-1)
(m-1)(n-1) = 116
116 = 29*2*2;
116 = 116*1 => m = 117, n = 2
116 = 58*2 => m = 59, n = 3
116 = 29*4 => m = 30, n = 5; НОД(30, 5) = 5 > 1 - не взаимно просты.
Ответ: (117;2), (59;3)

3945. фаня, 23 октября 2011, 19:18:35
Задан закон s(t)=4х4-3х2-х+1 изменения пути движения материальной точки. Найти значения скорости и ускорения этой точки в момент времени t0=2.

3946. ГрафМонтеКристо, 23 октября 2011, 20:51:16
радиус основания первого конуса в 3 раза больше чем радиус основания второго. а образующая первого конуса в 2 раза меньше чем образующая второго. чему равна площадь боковой поверхности 1 конуса если площадь боковой поверхности второго равна 18 см

3947. 123, 23 октября 2011, 21:05:37
3945. фаня, 23 октября 2011, 19:18:35
Задан закон s(t)=4х4-3х2-х+1 изменения пути движения материальной точки. Найти значения скорости и ускорения этой точки в момент времени t0=2.


s(t)=4х4-3х2-х+1
u(t)=(s(t))'=16x^3-6x-1
a(t)=(u(t))'=48x^2-6

u(2) = 16*8-12-1=115
a(2) = 48*4-6=186

3948. Kattu22, 26 октября 2011, 01:02:33
Lexxus! Исправьте, пожалуйста, ответ к номеру 24655

3949. миланиум, 26 октября 2011, 16:33:51
АВСД прямоугольник АОВ 36 градусов найти САД ВДС если точка О точка пересечения диагонали прямоугольника

3950. Яночка, 27 октября 2011, 12:08:35
Ребят помогите решить срочно-по ребру а правильного тетраэдра найдите радиус вписанной сферы

3951. Ксения, 27 октября 2011, 17:16:16
Помогите пожалуйста!!!спорная оценка,нужно 1й пример упростить выражение sin2альфа - tgальфа. 2й пример тоже упростить sin(180градусов-альфа)+cos(90 градусов+альфа).

3952. Леонид, 27 октября 2011, 21:24:42
2) sina-sina=0 -поформулам приведения.
1) синус двойного угла, тангенс как отношение синуса к косинусу, синус за скобки,в скобках к общ.знам.,триг ед. ...

3953. 123, 28 октября 2011, 00:31:35
Ксения,
1)sin(2a)-tg(a)=2sin(a)cos(a)-sin(a)/cos(a)=(2sin(a)cos^2(a)-sin(a))/cos(a) = sin(a)(2cos^2(a) - 1)/cos(a) = tg(a)cos(2a)

3954. ONV, 28 октября 2011, 19:50:44
В случайном эксперименте бросают три игральные кости. Найдите вероятность того, что в сумме выпадет 9 очков. Результат округлите до сотых.

Помогите решить

3955. alfa20, 28 октября 2011, 22:42:34
В равнобедренном треугольнике АВС на боковой стороне ВС отмечена точка М так, что отрезок МС равен высоте треугольника, проведённой к этой стороне, а на боковой стороне АВ отмечена точка К так, что угол КМС - прямой. Найдите угол АСК. (Задача из прошедшей городской олимпиады для 9 класса. Что-то никак не получается...)

3956. 123, 29 октября 2011, 01:21:49
alfa20
Нарисуем равнобедренный треугольник, потом проведём высоту СН, она равна другой высоте. Т.е наша точка будет лежать на окружности радиусом которой является высота СН. Угол при вершине В (далее В). <HCB = 90 - B;
Вспомним пр точку К, и заметим что и неё мы провели прямую перпендикулярную радиусу, т.е касательную. Отрезки касательных из одной точки равны, СН=СМ, СК - биссектриса, т.е каждый из углов на которые она делит исходный угол равны 45 - B/2;
<A = <B = 90 - B/2, <HCA = 90 - B - 90 - B/2 = B/2;
Сложим 45 - В/2 + В/2 = 45

Ответ: 45

3957. 123, 29 октября 2011, 01:27:58
Упс опечатка =)
<HCA = 90-B/2-90+B = B/2

3958. Ксеня, 29 октября 2011, 09:40:52
надо расположить числа в порядке возрастания ctg100, ctg270 и ctg160
упростить sin32*sin148-cos32*sin302+ctg225

3959. Леонид, 29 октября 2011, 12:05:43
Уважаемый, LEXXUS! Прошу о помощи! 1) Из пункта А и В навстречу друг другу вышли одновременно два поезда. Каждый изнихдвигался сначала равноускоренно (начальные скорости поездов равны нулю,ускорения различны),а затем, достигнув некоторой скорости,-равномерно.Отношение скоростей равномерного движения поездов равно 5/4.В некоторый момент временискорости поездов оказались равными,а один изних прошёл к этому времени расстояние в 5/4 разабольше, чем другой.В пункты В и А поезда прибыли одновременно.Какую часть пути прошёл каждый из поездов к моменту, когдаих скорости оказались равными?
2) У восьми школьников в сумме имеется 7р 19к. Известно,что у любых двух изних различные суммы денег,ноу одного изних в целое число раз больше денег,чем у другого. Сколько денег у каждого школьника?

3960. Lexxus, 29 октября 2011, 18:35:35
3959. Леонид, 29 октября 2011, 12:05:43
Из пункта А и В навстречу друг другу вышли одновременно два поезда. Каждый из них двигался сначала равноускоренно (начальные скорости поездов равны нулю, ускорения различны), а затем, достигнув некоторой скорости, - равномерно.Отношение скоростей равномерного движения поездов равно 5/4. В некоторый момент времени скорости поездов оказались равными, а один из них прошёл к этому времени расстояние в 5/4 раза больше, чем другой. В пункты В и А поезда прибыли одновременно. Какую часть пути прошёл каждый из поездов к моменту, когда их скорости оказались равными?

Я немного подзабыл, есть ли это в школьной программе по математике, но по физике - точно должно быть.

Итак, строим графики зависимости скорости поездов от времени:



Площади получившихся трапеций (интеграл функций v1(t) и v2(t) от нуля до времени окончания пути T) - это путь, пройденный первым и вторым поездами соответственно. Мы знаем, что они равны.
А пути, пройденные до момента, когда их скорости сравнялись (T0), соответственно, - площади под графиками от нуля до T0. Мы знаем, что они относятся как 5/4.

Аккуратно расписываем всё, что мы знаем, скорости везде сокращаются, остаются только соотношения между временами. Выражаем всё через одну неизвестную величину.

У меня всё выразилось через T0 (время, когда скорости сравнялись. На графике - точка пересечения графиков):

T1 (время, когда первый - "красный" - поезд закончил ускоряться) = 3/2*T0;
T2 (время, когда второй - "синий" - поезд закончил ускоряться) = 3/4*T0;
T (время окончания маршрута) = 9/4*T0.

Теперь находим искомые части пройденного пути через отношение площадей. Тут у нас сократится и неизвестное T0.

В итоге будем иметь, что первый поезд к моменту равенства скоростей прошел 4/15 всего пути, а второй - 5/15.

3961. Lexxus, 29 октября 2011, 18:39:26
3959. Леонид, 29 октября 2011, 12:05:43
2) У восьми школьников в сумме имеется 7р 19к. Известно,что у любых двух изних различные суммы денег,но у одного изних в целое число раз больше денег, чем у другого.

Очевидно, задача неправильно сформулирована. Под это условие подойдет бесконечно много ответов. Например,
1р., 2р., 1к., 2к., 3к., 4к., 5к. и 4р4к.

3962. 123, 29 октября 2011, 19:54:03
3961. Lexxus, 29 октября 2011, 18:39:26
3959. Леонид, 29 октября 2011, 12:05:43

2) У восьми школьников в сумме имеется 7р 19к. Известно,что у любых двух изних различные суммы денег,но у одного изних в целое число раз больше денег, чем у другого.

Очевидно, задача неправильно сформулирована. Под это условие подойдет бесконечно много ответов. Например,

1р., 2р., 1к., 2к., 3к., 4к., 5к. и 4р4к.


Наверно имелось ввиду что у любых двух разные суммы денег отличающиеся в целое число раз

3963. 123, 29 октября 2011, 20:36:36
Ответ полученный глупым перебором (в копейках):
384; 192; 96; 32; 8; 4; 2; 1

Ну правда там были рассуждения что у последнего одна копейка (719 - простое число, за скобки мы можем вынести только 1)
У первого копеек 2^n * 3^p а потом и более точное 2^7 * 3 (иначе ниже будет 288 (никак не свести к необходимой сумме) выше 432 (при наиболее близком варианте даст 715))

Ну а дальше глупый и бессмысленный перебор

3964. alfa20, 29 октября 2011, 21:27:09
3957. 123, 29 октября 2011, 01:27:58
Упс опечатка =)

&lt;HCA = 90-B/2-90+B = B/2
СПАСИБО!!!

3965. Леонид, 30 октября 2011, 09:14:03
Спасибо, уважаемый LEXXUS, за задачи. Очень помогли! Но про копейки мною напечатано точно по тексту было. С Вашим замечанием к условию согласен. Ещё раз спасибо! "у любых из двух из них различные суммы денег, но у одного из них в целое число раз больше денег, чем у другого" - по тексту авторов. Ваше мнение - Наверно имелось ввиду что у любых двух разные суммы денег отличающиеся в целое число раз

3966. Ксеня, 30 октября 2011, 12:43:40
очень нужна помощь!!!надо расположить числа в порядке возрастания ctg100, ctg270 и ctg160
упростить sin32*sin148-cos32*sin302+ctg225. Помогите пожалуйста!!

3967. Wc7, 31 октября 2011, 15:31:34
Ребят,помогите пожалуйста;)))
заранее спасибо!))

1. Решите в натуральных числах уравнение ху – х – у = 2011 и найдите такое решение, при котором х2 + у минимально.
2. В треугольнике АВС точка К делит сторону ВС в отношении 2:1 (считая от точки В), точка L делит сторону AB в отношении 3:2 (считая от точки A), точка M делит сторону AС в отношении 4:3 (считая от точки А), а точка N делит отрезок АК в отношении 5:4 (считая от точки А). Найдите площадь четырехугольника ALNM, если площадь исходного треугольника АВС равна 1.
3. В пространстве произвольным образом расположены два правильных n-угольника, расстояние между центрами которых равно d. Найдите длину суммы всех векторов, начала которых лежат в вершинах первого, а концы – в вершинах второго n-угольника.
4. Найдите все рациональные решения уравнения sinx – siny = sin(x–y) (рациональным решением уравнения с двумя неизвестными называется пара рациональных чисел (x, y), удовлетворяющая уравнению).

3968. Kattu22, 31 октября 2011, 23:44:56
Lexxus! Исправьте, пожалуйста, ответ к номеру 24655. Я несколько дней назад этот номер, указал, но видно его не заметили.

3969. Даша, 1 ноября 2011, 11:27:37
надо немного подумать и всё получиться

3970. Lexxus, 1 ноября 2011, 14:12:59
3968. Kattu22, 31 октября 2011, 23:44:56
Lexxus! Исправьте, пожалуйста, ответ к номеру 24655. Я несколько дней назад этот номер, указал, но видно его не заметили.

Пардон. Теперь исправил.

3971. Арина, 2 ноября 2011, 10:15:55
Lexxus помоги мне пожалуйста решить з уровнения а) 87-х=39,б) z+24=43, в) (38+у)-18=31

3972. Kattu22, 2 ноября 2011, 14:52:09
Lexxus! Исправьте, пожалуйста, ответ к задаче 28115. Спасибо

3973. Лидия, 2 ноября 2011, 18:36:33
В треугольникеABC AC=BC=5, AB=8. Найдите tgA.

3974. кристина, 2 ноября 2011, 19:02:40
в трапеции АВСД основания АД и ВС относится как 3:1 Е середина АВ докажите ДЕ<2:3ДА+1:2ДС

3975. Lexxus, 2 ноября 2011, 22:16:14
3973. Лидия, 2 ноября 2011, 18:36:33
В треугольнике ABC AC=BC=5, AB=8. Найдите tgA.

Проведём высоту CH. Треугольник ABC равнобедренный, значит, CH будет и медианой, т.е. AH = AB/2.
Тангенс угла A равен отношению противолежащего и прилежащего катетов прямоугольного треугольника ACH, т.е. CH/AH.
А CH можно в свою очередь найти по теореме Пифагора.
Всё.

3976. Lexxus, 2 ноября 2011, 22:26:07
3974. кристина, 2 ноября 2011, 19:02:40
в трапеции АВСД основания АД и ВС относится как 3:1 Е середина АВ докажите ДЕ<2:3ДА+1:2ДС

Пусть F - середина DC, DF = (1/2)DC.
FE - средняя линия трапеции, и её длина равна (AD+BC)/2 = (AD+AD/3)/2 = (2/3)AD.

Сторона DE треугольника DEF всегда меньше суммы двух других его сторон DF+FE = (2/3)AD+(1/2)DC

3977. Екатерина, 3 ноября 2011, 07:42:31
Здравствуйте)Видимо я сошла с ума))У меня не сходятся моё решение с ответом этого уравнения)Найдите корень уравнения sqrt2x + 1 = 4.
У меня выходит так,что
2x+1=4;
2x=3;
x=1,5
в ответе же ответ 7,5.Пожалуйста,помогите разобраться))

3978. OS, 3 ноября 2011, 22:18:58
3977. Екатерина, 3 ноября 2011, 07:42:31
Здравствуйте)Видимо я сошла с ума))У меня не сходятся моё решение с ответом этого уравнения)Найдите корень уравнения sqrt2x + 1 = 4.
У меня выходит так,что
2x+1=4;
2x=3;
x=1,5
в ответе же ответ 7,5.Пожалуйста,помогите разобраться))

В квадрат надо возвести обе части ур-я.

3979. Маря, 4 ноября 2011, 18:02:06
1)Cередины сторон CD и AB параллелограмма ABCD лежат в плоскости а(альфа). а сторона BC не лежит в этой плоскости.докажите,что прямая AD и плоскость a параллельны.
2)точки A,B,C,D не лежат в одной плоскости.Докажите,что любые три из них являются вершинами треугольника.
3)прямая КМ параллельна стороне ВС треугольника АВС и не лежит в плоскости АВС. Выясните взаимное расположение прямых КМ и АВ и найдите угол между ними,если угол /_АВС=105 градусов
помогите ваще ничего не шарю
ПОЖАЛУЙСТА!!!!!

3980. Александр, 5 ноября 2011, 11:39:29
Помогите с задачами пожалуйста...
1) Диагональ прямоугольного параллелепипеда, в основании которого лежит квадрат, равна 8см, а диагональ боковой грани равны 7см. Найдите высоту параллелепипеда.
2) В прямой призме стороны основания равны 3, 4 и 5см, а полная поверхность равна 84см2. Определите боковую поверхность призмы и её высоту.

3981. Елена, 5 ноября 2011, 16:06:08
Помогите пожалуйста решить...
Диагонали трех граней прямоугольного параллелепипеда, сходящиеся в одной вершине, равны 8м, 10м и 12м. Найти линейные размеры этого параллелепипеда.

3982. Сергей, 5 ноября 2011, 18:26:07
Помогите, пожалуйста решить!!!
Найдите все двузначные числа, квадрат которых оканчивается теми же цифрами, что и само число.

3983. 123, 6 ноября 2011, 16:40:58
3982. Сергей, 5 ноября 2011, 18:26:07
Помогите, пожалуйста решить!!!

Найдите все двузначные числа, квадрат которых оканчивается теми же цифрами, что и само число.


Ответ:25, 76

Ну тупо перебором,чтобы последняя цифра совпадала нам нужны числа оканчивающиеся на 0,1,5,6.
0 не подходит. В случае с единицой, вторая цифра (с конца) будет равна удвоенной второй цифре (с конца) исходного числа((10а+1)^2 = 100a^2+20a+1). 5 - все числа будут заканчиваться на 25 (т.к (10а+5)^2 = 100a^2+100a+25) т.е подходит только 25, ну а с 6 перебором получаем только 76.

3984. Сергей, 6 ноября 2011, 16:58:32
Спасибо огромное!!! Всё понятно стало.

3985. aist, 6 ноября 2011, 20:33:24
задача 10357


x-15/x+5=5

в ответе -10

объсните плизз))))

3986. 123, 6 ноября 2011, 22:53:33
Ну во первых числитель и знаминатель надо в скобки заключать

ОДЗ x != 5 (!= неравно)
x-15=5x+25
4x = -40
x = -10

3987. Сашуля)), 8 ноября 2011, 11:09:07
Корень из 2х+1 если Х=-4;-9
Пожалуйста дайте ответ)))За ранее спасибо)))

3988. Юлия, 8 ноября 2011, 17:51:38
Помогите,пожалуйста,геометрию вообще не соображаю,а контрольная висит.
1.Найдите высоту правильной 3-угольной пирамиды,если сторона её основания равнв А, а апофема равнва Л.
2.Найдите величину двугранного угла при основании правильной 4-угольной пирамиды,если её боковые рёбра наклонены к плоскости основания под углом 30градусов.
3.Найти боковое ребро правильной 3-угольной пирамиды,у которой площадь основания равна 27корней из 3 (кв.см), а полная поверхность равна 72корня из 3(кв.см.)
Если можно,то с рисунками:)Помогите,пожалуйста,а то эта геометрия всю четверть испортит(((
За ранее огромное спасибо!!!!

3989. ксения, 8 ноября 2011, 21:58:00
Валя выбирает трехзначное число. Найти вероятность того, что оно делится на 51.

3990. Lexxus, 8 ноября 2011, 22:06:45
3989. ксения, 8 ноября 2011, 21:58:00
Валя выбирает трехзначное число. Найти вероятность того, что оно делится на 51.

Чтобы получить вероятность того, что случайно выбранное трехзначное число будет делиться на 51, нужно количество всех трёхзначных чисел, делящихся на 51, поделить на количество вообще всех трёхзначных чисел.

Всего трёхзначных чисел 900 (от 100 до 999).
На 51 делятся числа вида n*51, где n = 1,2,...
n=1 нам не подходит, потому что 1*51 = 51 - двухзначное.
Нам подходят: 2*51, 3*51, ... 19*51 - все трёхзначные.
Всего их получается 18.

18/900 = 0.02

3991. ксения, 8 ноября 2011, 22:10:20
помогите плиз . В каждой пятой банке кофе согласно условиям акции находится приз. Призы распределены по банкам случайно. Найти вероятность того, что Галя не найдет в своей банке приз.

3992. ксения, 8 ноября 2011, 22:11:51
Двое играют в кости. Они по разу бросают игральный кубик. Выигрывает тот, у кого больше очков. Если выпадет поровну, то ничья. Первый бросил кубик, и у него выпало 4. Найти вероятность того, что он выиграет.

3993. Шаман, 9 ноября 2011, 11:39:58
помогите пожалуйста.
решить неравенство log2-x(x+2)*logx+3(3-x)<=0


найдите все натуральные числа,последняя десятичная цифра которых 0 и которые имеют 15 различных натуральных делителей (включая единицу и само число)

3994. Т__Т, 11 ноября 2011, 14:43:46
2. В треугольнике АВС точка К делит сторону ВС в отношении 2:1 (считая от точки В), точка L делит сторону AB в отношении 3:2 (считая от точки A), точка M делит сторону AС в отношении 4:3 (считая от точки А), а точка N делит отрезок АК в отношении 5:4 (считая от точки А). Найдите площадь четырехугольника ALNM, если площадь исходного треугольника АВС равна 1.


3995. 123, 12 ноября 2011, 00:06:01
3994. Т__Т, 11 ноября 2011, 14:43:46
2. В треугольнике АВС точка К делит сторону ВС в отношении 2:1 (считая от точки В), точка L делит сторону AB в отношении 3:2 (считая от точки A), точка M делит сторону AС в отношении 4:3 (считая от точки А), а точка N делит отрезок АК в отношении 5:4 (считая от точки А). Найдите площадь четырехугольника ALNM, если площадь исходного треугольника АВС равна 1.


АК делит треугольник на два треугольника площадьми 2/3 и 1/3 (высота одна и та же, разные основания)
Потом по формуле S=ab*Sina/2 получаем отношения меньших треугольников и больших, а из него и площадь самих треугольников. Складывая их получим площадь.

3996. евгениЙ, 13 ноября 2011, 08:02:16
Диагональ осевого сечения цилиндра равна 8 см и наклонена к плоскости основания цилиндра под углом 30градусов. Найдите площадь полной поверхности цилиндра.

3997. Kattu22, 13 ноября 2011, 11:30:04
Lexxus! Исправьте, пожалуйста, ответ к задаче 283645.

3998. Lexxus, 13 ноября 2011, 17:02:33
3997. Kattu22, 13 ноября 2011, 11:30:04
Lexxus! Исправьте, пожалуйста, ответ к задаче 283645.

+

3999. Юлия, 14 ноября 2011, 15:11:53
Помоги ,никак не могу решить задачу ,вот(
Дан куб АВСДА1В1С1Д1,где АА1 равно 1 ,М-середина отрезка ВС1.Найти расстояния от точки М до плоскости АВ1Д1

4000. Lexxus, 14 ноября 2011, 15:16:35
3993. Шаман, 9 ноября 2011, 11:39:58
найдите все натуральные числа,последняя десятичная цифра которых 0 и которые имеют 15 различных натуральных делителей (включая единицу и само число)

Эта задача есть в базе решений части С.

4001. oksana, 14 ноября 2011, 16:50:09
помогите пожалуйста,оч надо Найти корень уравнения 54-х=25

4002. Игорь, 15 ноября 2011, 06:52:38
Помогите решить пжл, дискретная математика

1.Построить таблицы истинности для формул:

(x&#8594;y)~(x&#8594;(y&#8594;z)) &#773; над вторым компонентом полное отрицание

(x&#8594;y) &#773;&#8744;(x &#773;&#8594;y &#773;)

2.Упростить формулу: (x&#8764;y)&#8743;(y&#8764;z)&#8743;(z&#8764;x)

3.Доказать тождественную истинность формулы: (x&#8594;(y&#8594;z))&#8594;((x&#8743;y)&#8594;z)

4.Доказать соотношение: (x&#8744;y)&#8743;(x&#8744;y &#773;)&#8801;x

5.Записать двойственные формулы: x&#8743;z&#8744;x&#8743;z &#773;&#8744;y&#8743;z&#8744;x &#773;&#8743;y&#8743;z

4003. Игорь, 15 ноября 2011, 06:55:19
4001. oksana, 14 ноября 2011, 16:50:09
помогите пожалуйста,оч надо Найти корень уравнения 54-х=25


Корень 29

4004. Игорь, 15 ноября 2011, 11:32:13
http://upwap.ru/1832603

Вот ссылка на задачи, помогите решить пжл, и если можно с объяснениями(необязательно)

4005. Кто-то, 15 ноября 2011, 18:35:33
4004. Игорь, 15 ноября 2011, 11:32:13
http://upwap.ru/1832603
Вот ссылка на задачи, помогите решить пжл, и если можно с объяснениями(необязательно)


Это алгебра логики, а тут задачи ЕГЭ решают, ну или как минимум математику (школьный курс)

4006. Игорь, 15 ноября 2011, 21:01:58
4005. Кто-то, 15 ноября 2011, 18:35:33
4004. Игорь, 15 ноября 2011, 11:32:13

Это алгебра логики, а тут задачи ЕГЭ решают, ну или как минимум математику (школьный курс)



Я знаю, но написал в надежде что и с алгеброй логики помогут.

4007. Игорь, 16 ноября 2011, 15:08:34
http://upwap.ru/1832603

решите пжл, в качестве благодарности 50р на телефон.

4008. Марина, 17 ноября 2011, 15:53:46
найдите величину двугранного угла при основании правильной четырёхугольной пирамиды, если её боковые рёбра наклонены к плоскости основания под углом 60

вырй2муууучсыязфцАОШОЩДйуаомрсгорагшщсмрфоЩВМСЩЗУОышосшщзываом зрвчмшкапркапукц

4010. настена, 18 ноября 2011, 17:52:23
найдите площадь поверхности прямой призмы, в основании которой лежит ромб с диагоналями равными 3 и 4, и боковым ребром равным 5
ПОМОГИТЕ ПОЖАЛУЙСТА)*

4011. Цезарь, 18 ноября 2011, 18:51:29
1)Cередины сторон CD и AB параллелограмма ABCD лежат в плоскости а(альфа). а сторона BC не лежит в этой плоскости.докажите,что прямая AD и плоскость a параллельны.
2)точки A,B,C,D не лежат в одной плоскости.Докажите,что любые три из них являются вершинами треугольника.
3)прямая КМ параллельна стороне ВС треугольника АВС и не лежит в плоскости АВС. Выясните взаимное расположение прямых КМ и АВ и найдите угол между ними,если угол АВС=105 градусов
Очень прошу решите пожалуйста

4012. 123, 19 ноября 2011, 13:46:29
4011. Цезарь, 18 ноября 2011, 18:51:29
1)Cередины сторон CD и AB параллелограмма ABCD лежат в плоскости а(альфа). а сторона BC не лежит в этой плоскости.докажите,что прямая AD и плоскость a параллельны.

2)точки A,B,C,D не лежат в одной плоскости.Докажите,что любые три из них являются вершинами треугольника.

3)прямая КМ параллельна стороне ВС треугольника АВС и не лежит в плоскости АВС. Выясните взаимное расположение прямых КМ и АВ и найдите угол между ними,если угол АВС=105 градусов

Очень прошу решите пожалуйста

1. Проведём линию соединяющую середины сторон, она параллейна основаниям параллелограмма, т.к прямая параллельна прямой на плоскости, то она параллейна и самой плоскости.

2. Если есть три точки не являются вершинами треугольника, то они лежат на одной прямой, а через неё и оставшиюся точку можно провести плоскость, что неудовлетворяет условиям задачи

3.КМ || ВС значит КМ || ABC, т.к АВ не параллейна ВС, то она не параллейна и КМ => прямые скрещивающиеся. Угол равен 180-105 = 75 (рассмотреть параллейные прямые и секущую)

4013. Руслан, 20 ноября 2011, 18:10:48
Через терминал оплаты на мобильный телефон можно перевести деньги, при этом взимается комиссия – целое положительное число процентов. При переводе некоторой целочисленной суммы денег счет мобильного телефона пополнился на 847 рублей. Какая сумма была положена на счет, если известно, что комиссия менее 30%?

4014. Руслан, 20 ноября 2011, 18:12:12
Решите пожалуйста)

4015. саня, 20 ноября 2011, 18:42:57
помогите пожадуйста решить задачу с2. высота РО в правильной треугольной пирамиды РАВС равна корень из 108, а стороны ее основания АВС равны 6. Тчка N делит высоту РО в отношении 2:1 считая от точки Р. найдите угол ВNO

4016. Айгуль Хаматдинова, 20 ноября 2011, 18:44:21
пожалуйста,решете задачу:Вне плоскости альфа расположен треугольник ABC,у которого медианы АА1 и ВВ1 параллельны плоскости альфа.Через вершины В и С проведены параллельные прямые,пересекающие плоскость альфа в точках E и F.

Пожалуйста как можно побыстрее..

4017. Айгуль Хаматдинова, 20 ноября 2011, 18:48:44
пожалуйста...)))

4018. Айгуль Хаматдинова, 20 ноября 2011, 19:04:15
ну почему так долго(((

4019. 123, 20 ноября 2011, 20:26:55
4016. Айгуль Хаматдинова, 20 ноября 2011, 18:44:21
пожалуйста,решете задачу:Вне плоскости альфа расположен треугольник ABC,у которого медианы АА1 и ВВ1 параллельны плоскости альфа.Через вершины В и С проведены параллельные прямые,пересекающие плоскость альфа в точках E и F.

Пожалуйста как можно побыстрее..


Не оффтопь, и в чём собственно вопрос задачи?

4020. 123, 20 ноября 2011, 20:45:28
4013. Руслан, 20 ноября 2011, 18:10:48
Через терминал оплаты на мобильный телефон можно перевести деньги, при этом взимается комиссия – целое положительное число процентов. При переводе некоторой целочисленной суммы денег счет мобильного телефона пополнился на 847 рублей. Какая сумма была положена на счет, если известно, что комиссия менее 30%?


Разложим 847, для этого вспомним что на 7 оканчиваются числа кратные 3 и 7,
8+4+7=19 не делится на 3, проверим 7. В итоге 847 = 11*11*7. 77% от общей суммы составляют 847 рублей, значит 1% = 11 руб, откуда вся сумма 1100 рублей

4021. Лидия, 22 ноября 2011, 21:15:00
Огромное спасибо за сайт :)
Хотелось бы, чтобы при показе правильного ответа, указывалось и правильное решение этого примера, задания.

4022. шаша+, 23 ноября 2011, 09:35:26
Люди, помогите решить: Доказать, что при делении на 6 квадрата целого числа не может получиться 2 и 5. Пожалуйста, срочно.

4023. Lexxus, 23 ноября 2011, 09:57:58
4022. шаша+, 23 ноября 2011, 09:35:26
Доказать, что при делении на 6 квадрата целого числа не может получиться 2 и 5. Пожалуйста, срочно.

Э-э-э...
Ну,
n^2/6 = 2, n^2 = 12, 12 не является квадратом целого числа.
n^2/6 = 5. n^2 = 30, 30 тоже не является квадратом целого числа.

Собственно, всё.

4024. шаша+, 23 ноября 2011, 10:40:34
Cпасибо, а я искала подвох...

4025. Kattu22, 23 ноября 2011, 11:48:44
Lexxus! Исправьте, пожалуйста, ответы в заданиях 7471, 19403

4026. Lexxus, 23 ноября 2011, 12:54:07
4025. Kattu22, 23 ноября 2011, 11:48:44
Lexxus! Исправьте, пожалуйста, ответы в заданиях 7471, 19403

Спасибо.

4027. шаша+, 23 ноября 2011, 13:30:02
Lexxus, опять тупею. n^2/6,n^2=12,12 не является квадратом целого числа ( 5 тоже), но и то же с 3, с 4, и с 1 :

n^2/6=3, n^2=18, а 18 то же не является квадратом целого числа... и 4, и 1, и 3... Эти остатки повторяются, но как оформить решение?


4028. мАРИЯ, 24 ноября 2011, 12:27:16
помогите решить)
боковое ребро правильной треугольной пирамиды равно 5,а высота 3.найти объем пирамиды.

4029. Lexxus, 24 ноября 2011, 12:41:27
4027. шаша+, 23 ноября 2011, 13:30:02
Lexxus, опять тупею. n^2/6,n^2=12,12 не является квадратом целого числа ( 5 тоже), но и то же с 3, с 4, и с 1 :

n^2/6=3, n^2=18, а 18 то же не является квадратом целого числа... и 4, и 1, и 3... Эти остатки повторяются, но как оформить решение?

А теперь соберись и сформулируй условие задачи так, чтобы я его понял правильно.

4030. Ilia_, 24 ноября 2011, 14:43:39
>боковое ребро правильной треугольной пирамиды равно 5,а высота 3.найти объем пирамиды

Введем буковки:
ABCD - правильная треугольная пирамида с основанием ABC. DH - её высота.
AD=BD=CD=5, DH=3. По теореме Пифагора: AH=BH=CH=4.

Найдём площадь ABC: S_abc=3S_ahc=3*0.5*ah*bh*sin(ahb)=3*.5*4*4*sin(120)= 3/2*16*sqrt(3)/2=12*sqrt(3)

Объём ABCD = 1/3*S_abs*DH=12*sqrt(3)

4031. Kattu22, 25 ноября 2011, 14:42:32
Lexxus! Открытый банк пополнился заданиями В10. Добавьте их, пожалуйста.

4032. Леонид, 26 ноября 2011, 17:06:53
Здравствуйте!Посмотрите, пожалуйста № 129217 и №130617 - В14,что то не получается ответов целых или конечных десятичных дробей!

4033. надежда2010, 26 ноября 2011, 23:47:45
забыла пароль для входа на свою страничку, как можно восстановиться? Или все по новой начинать, место в рейтинге 2.

4034. ольга, 27 ноября 2011, 17:44:19
Здраствуйте! Помогите пожалуйста решить задачу
Даны вершин треугольника А(0;3);В(12;-6);С(10;8).Найти:
1)длину АВ
2)уравнение АВ и АС и их условные коэффициенты
3)величину угла А
4)уравнение высоты СD и ее длину
5)уравнение медианы СМ
6)уравнение биссектрисы СК
7) площадь треугольника
8) координаты вектора тяжести треугольника

4035. Lexxus, 28 ноября 2011, 00:00:20
4031. Kattu22, 25 ноября 2011, 14:42:32
Lexxus! Открытый банк пополнился заданиями В10. Добавьте их, пожалуйста.

Хорошо, в скором времени постараюсь обновить.

4036. Lexxus, 28 ноября 2011, 00:10:21
4033. надежда2010, 26 ноября 2011, 23:47:45
забыла пароль для входа на свою страничку, как можно восстановиться? Или все по новой начинать, место в рейтинге 2.

Ох, нет у меня процедуры восстановления пароля. Так что пока единственный способ - убедить меня, что вы и пользователь "надежда 2010" - это одно и то же лицо.

Для этого достаточно будет прислать мне на почту lexxus@mephist.ru правильные ответы на следующие вопросы:

1) Логин для входа в систему;
2) Дата регистрации (хотя бы примерная);
3) Максимально точный список пройденных тестов в системе "Тесты ЕГЭ Онлайн" и их результаты.

Короче, мне нужна та информация, которая однозначно бы определяла вас как владельца того самого аккаунта, пароль от которого вам хочется получить.

А вообще, процедуру восстановления пароля я, пожалуй, формализую в ближайшем будущем.

4037. Lexxus, 28 ноября 2011, 00:37:43
4033. надежда2010, 26 ноября 2011, 23:47:45
забыла пароль для входа на свою страничку, как можно восстановиться? Или все по новой начинать, место в рейтинге 2.


{внимательнее поглядел логи}

Хорошая попытка :)
И зачем тебе пароль от аккаунта "надежда 2010"? Просто чисто по-человечески интересно. Это как-то поможет в учёбе?

4038. саша, 28 ноября 2011, 18:10:36
Площадь трапеции равна 320 см2,а высота трапеции равна 8 см.Найдите основания трапеции,если длина одного из оснований составляет 60%длины другого.

4039. сергей))))), 28 ноября 2011, 18:21:26
высота пирамиды равна двенадцать сантиметров.Площадь Основания четыресто тридцать два метра в квадрате.На каком расстоянии от основания находится сечение,параллельное ему,если площадь сечения двадцать семь метров квадратных.

4040. оооо, 28 ноября 2011, 18:32:58
4039. сергей))))), 28 ноября 2011, 18:21:26
высота пирамиды равна двенадцать сантиметров.Площадь Основания четыресто тридцать два метра в квадрате.На каком расстоянии от основания находится сечение,параллельное ему,если площадь сечения двадцать семь метров квадратных.
помогие пятерка выходит в экзамене не могу решить...

4041. Влад, 28 ноября 2011, 19:42:09
В треугольнике АВС точка К делит сторону ВС в отношении 2:1 (считая от точки В), точка L делит сторону AB в отношении 3:2 (считая от точки A), точка M делит сторону AС в отношении 4:3 (считая от точки А), а точка N делит отрезок АК в отношении 5:4 (считая от точки А). Найдите площадь четырехугольника ALNM, если площадь исходного треугольника АВС равна1.

4042. Lexxus, 28 ноября 2011, 19:52:11
4039. сергей))))), 28 ноября 2011, 18:21:26
высота пирамиды равна двенадцать сантиметров.Площадь Основания четыресто тридцать два метра в квадрате.На каком расстоянии от основания находится сечение,параллельное ему,если площадь сечения двадцать семь метров квадратных.
4040. оооо, 28 ноября 2011, 18:32:58
помогие пятерка выходит в экзамене не могу решить...

Хреновая, однако, пятёрка.

Сечение, параллельное основанию, отсекает от пирамиды другую пирамиду, ей подобную. Площади оснований этих пирамид относятся как квадраты их высот.
Значит,
432/27 = 12^2/x^2, x = 3.
Но это высота отсеченной пирамиды. Значит, искомое расстояние равно
(12-x) = 9.

4043. соня, 28 ноября 2011, 20:25:59
В треугольнике АВС точка К делит сторону ВС в отношении 2:1 (считая от точки В), точка L делит сторону AB в отношении 3:2 (считая от точки A), точка M делит сторону AС в отношении 4:3 (считая от точки А), а точка N делит отрезок АК в отношении 5:4 (считая от точки А). Найдите площадь четырехугольника ALNM, если площадь исходного треугольника АВС равна 1.

4044. соня, 28 ноября 2011, 20:27:05
помогите пожалйста)

4045. 123, 28 ноября 2011, 21:00:09
4043. соня, 28 ноября 2011, 20:25:59
В треугольнике АВС точка К делит сторону ВС в отношении 2:1 (считая от точки В), точка L делит сторону AB в отношении 3:2 (считая от точки A), точка M делит сторону AС в отношении 4:3 (считая от точки А), а точка N делит отрезок АК в отношении 5:4 (считая от точки А). Найдите площадь четырехугольника ALNM, если площадь исходного треугольника АВС равна 1.


Уже говорил как решать, но если всё так далеко =)

На рисунке искомая фигура будет забита в уголке. Заметим что АК делит начальный треугольник на два (площадью 2/3 и 1/3)
Дальше воспользуемя формулой площади через синус и найдём нужные отношения
Sabk/Saln = 3
Sakc/Sanm = 63/20

Отсюда площади составляющих треугольников S1=2/9 S2=20/189 Складываем: 62/189

4046. Lexxus, 28 ноября 2011, 23:50:59
В базу данных добавлены новые задания и прототипы B9, B10 и B14
(всего 1663 новых задания).

4047. Kattu22, 29 ноября 2011, 11:06:36
Lexxus, спасибо!!!

4048. Vitya, 30 ноября 2011, 09:11:04
радиус в цилиндре равен 15 см. Сечение, параллельное оси цилиндра от неё на 10 см, имеет форму квадрата. Найти площадь полной поверхности цилиндра. помогите пожалуйста решить

4049. Vitya, 30 ноября 2011, 09:15:10
не много ошибся, найти нужно площадь сечения=)

4050. Lexxus, 30 ноября 2011, 11:39:53
4048. Vitya, 30 ноября 2011, 09:11:04
радиус в цилиндре равен 15 см. Сечение, параллельное оси цилиндра от неё на 10 см, имеет форму квадрата. Найти площадь полной поверхности цилиндра. помогите пожалуйста решить
4049. Vitya, 30 ноября 2011, 09:15:10
не много ошибся, найти нужно площадь сечения=)

По-моему, у тебя больше проблем с русским языком, чем с математикой :-

Длина хорда окружности радиуса 15 см, находящаяся в 10 см от её центра, равна 2*sqrt(15^2-10^2) = 10*sqrt(5).

Площадь сечения - квадрата, стороной которого является эта хорда, - равна (10*sqrt(5))^2 = 100*5 = 500.

4051. tu, 30 ноября 2011, 16:05:16
1.Вне плоскости альфа расположен треугольник ABC, у которого медианы АА1 и BB1 параллельны плоскости альфа. Через вершины В и С треугольника проведены параллельные прямые, которые пересекают плоскость альфа соответственно в точках E и F. Докажите, что ECBF - параллелограмм
2.Все грани параллелепипеда ABCDA1B1C1D1 - квадраты со стороной а. Через середину AD параллельно плоскости DA1B1 проведена плоскость. Найдите периметр сечения.


4052. шуша_вперед, 30 ноября 2011, 16:36:29
В задаче B10(283663) ответ должен быть (по правилам округления до сотых), все-таки, 0.95, а не 0.96, как указано в ответе, так как 0.953333...=0.95.

4053. Lexxus, 30 ноября 2011, 17:03:33
4052. шуша_вперед, 30 ноября 2011, 16:36:29
В задаче B10(283663) ответ должен быть (по правилам округления до сотых), все-таки, 0.95, а не 0.96, как указано в ответе, так как 0.953333...=0.95.

Читай комментарий №3940. Вслух. Пока не дойдёт.

4054. 123, 30 ноября 2011, 22:39:16
4051. tu, 30 ноября 2011, 16:05:16
1.Вне плоскости альфа расположен треугольник ABC, у которого медианы АА1 и BB1 параллельны плоскости альфа. Через вершины В и С треугольника проведены параллельные прямые, которые пересекают плоскость альфа соответственно в точках E и F. Докажите, что ECBF - параллелограмм

2.Все грани параллелепипеда ABCDA1B1C1D1 - квадраты со стороной а. Через середину AD параллельно плоскости DA1B1 проведена плоскость. Найдите периметр сечения.


1. Очевидно что медианы принадлежат плоскости треугольника, а т.к они параллейны плоскости альфа, то и треугольник паралелен ей (две пересекающиеся прямые одной плоскости || другой). Через параллейные прямые можно провести плоскость и по свойствам параллейных плоскостей BC || EF. ч.т.д
2. Параллейная A1D прямая - средняя линия AA1D Параллейная А1В1 прямая - прямая соед. середины сторон грани АА1В1В. Сечение является прямоугольник со сторонами sqtr(2)/2*a; a => S=(sqrt(2)*a^2)/2

4055. 123, 1 декабря 2011, 00:26:53
упс периметр нужен был =) Р=а(sqrt(2)+2)

4056. шуша_вперед, 1 декабря 2011, 09:24:19
4053. Lexxus, 30 ноября 2011, 17:03:33
4052. шуша_вперед, 30 ноября 2011, 16:36:29
В задаче B10(283663) ответ должен быть (по правилам округления до сотых), все-таки, 0.95, а не 0.96, как указано в ответе, так как 0.953333...=0.95.

Читай комментарий №3940. Вслух. Пока не дойдёт.

Прочитала. Дошло. Спасибо.

4057. ONV, 1 декабря 2011, 14:39:40
Найдите наибольшее значение функции y=2 в степени -37-12х-х^2.

4058. Zarina, 1 декабря 2011, 17:00:38
Окружность точками А,В,С,D разделена на четыре части. АВ=75градусов, ВС=48градусов, СD=145градусов, DA=92градуса. Хорды DB и AC пересекаются в точке Е№ Найдите углы АЕВ и ВЕС. Plizzzz pomogite

4059. 123, 1 декабря 2011, 18:02:16
4058. Zarina, 1 декабря 2011, 17:00:38
Окружность точками А,В,С,D разделена на четыре части. АВ=75градусов, ВС=48градусов, СD=145градусов, DA=92градуса. Хорды DB и AC пересекаются в точке Е№ Найдите углы АЕВ и ВЕС. Plizzzz pomogite


Используйте то, что вписанные углы равны половине градусной меры дуги на которую опираются, построим любой треугольник и из него найдём что углы - 110 и 70

4060. Регина05, 2 декабря 2011, 14:02:45
В сборнике билетов по биологии всего 25 билетов, в двух из них
встречается вопрос о грибах. На экзамене школьнику достаётся один
случайно выбранный билет из этого сборника. Найдите вероятность того,
что в этом билете не будет вопроса о грибах. ПОМОГИТЕ, ПОЖАЛУЙСТА РЕШИТЬ. СПАСИБО

4061. Регина05, 2 декабря 2011, 14:04:13

Треугольник ABC вписан в окружность с центром O . Найдите угол
BOC , если угол BAC равен 32° . никак не могу разобраться, все забыла(((

4062. Регина05, 2 декабря 2011, 14:05:45

Найдите корень уравнения

log3 ( х-3)=2

4063. Ilia_, 2 декабря 2011, 18:11:51
>Найдите корень уравнения

>log3 ( х-3)=2

x-3 = 3^2 = 9
x = 12

4064. надежда2010, 4 декабря 2011, 11:49:33
Lexxus, спасибо, у меня все уже позади и отлично, а рейтинг- просто ради спортивного интереса да для разминки умственной, а самое главное- помогаю знакомым лоботрясам преодолеть страх перед необратимостью наказания в форме ЕГЭ. помогает... А еще, огромное спасибо за сайт, лучшего не придумано, другие отпугивают . А сейчас я шаша+.

4065. Надежда!2011, 4 декабря 2011, 17:21:15
1.Найти расстояние между вершинами В и Д Прямоугольного параллелепипеда,
для которого АВ=4, АД=3, АА1=7.
2.Найдите площадь квадрата, вершины которого имеют координаты (4;1).(5;4),(2;5), (1;2).
3.В треугольнике АВС, АВ=ВС =10, tgA=4|3. Найти АВ


4066. 123, 4 декабря 2011, 23:28:05
4065. Надежда!2011, 4 декабря 2011, 17:21:15
3.В треугольнике АВС, АВ=ВС =10, tgA=4|3. Найти АВ


Ответ 10 (по условию) =)

А так предположим что спрашивалось АС, т.к мы знаем тангенс, то опустим высоту, получаем что высота равна 4х, а половина АС = 3х по т.Пифагора 10=5х,
х=2, отсюда АС=12

4067. alfa20, 6 декабря 2011, 22:25:09
Дорогой и любимый наш Lexxus, не планируете ли Вы создание аналогичного местного сайта для 9 класса по математике (ГИА-9)?

4068. Lexxus, 6 декабря 2011, 23:00:11
4067. alfa20, 6 декабря 2011, 22:25:09
Дорогой и любимый наш Lexxus, не планируете ли Вы создание аналогичного местного сайта для 9 класса по математике (ГИА-9)?

Я пока не знаю, планирую я или нет. Провернуть аналогичную процедуру с базой mathgia.ru - вопрос пары часов работы, не проблема.

Проблема в том, что мне уже не нравится контингент сайта - главным образом, девочки (70%) 16-17 лет (63%). Причём 10% аудитории не могут (не "не хотят" - именно не могут) правильно ответить на вопрос, какого они года рождения.

Понизив средний возраст пользователя ещё на год-два, я могу просто потерять интерес к проекту.

Так что пока в раздумьях.

4069. николай, 7 декабря 2011, 10:26:29
В треугольнике абс угол равен 90градусов,бс равен 6,ас равен 10.найдите син б

4070. КИРА, 9 декабря 2011, 20:05:21
СПАСИБО, ЧТО ВЫ ЕСТЬ!!!

4071. степаннн)), 11 декабря 2011, 15:59:34
Ольга Афанасьєва, Яків Бродський, Олександр Павлов, Анатолій Сліпенко
Математика. 11 класс. Уровень стандарта
помогите найти решебник(((
или ответ на 173,167 задание...
зарание огромное спасибо)

4072. VerveIn, 11 декабря 2011, 18:26:14
Пожалуйста, помогите вот с таким заданием :

Найдите точку минумума функции y = (x + 18) e^ x -18

4073. Lena, 15 декабря 2011, 21:22:40
Помогите, пожалуйста, решить задачу: Смешали 14 литров 30% водного р-ра, некоторого вещества с 10л 18% раствора этого же вещества. Сколько % составляет концентрация получившегося раствора? Задача из B, но ни одной похожей на сайте я не нашла((

4074. alfa20, 15 декабря 2011, 22:00:34
4073. Lena, 15 декабря 2011, 21:22:40
Помогите, пожалуйста, решить задачу: Смешали 14 литров 30% водного р-ра, некоторого вещества с 10л 18% раствора этого же вещества. Сколько % составляет концентрация получившегося раствора? Задача из B, но ни одной похожей на сайте я не нашла((

14*0.3+10*0.18=6 - литров чистого вещества в 14+10=24 литрах вместе взятых. Тогда 6:24=0,25=25%

4075. Юрий, 16 декабря 2011, 13:29:09
пожалуйста помогите решить задачу, очень нужно на зачет к понедельнику!!!!!!!!!!!!! Помогите пожалуйста!!!!!! всю голову изломал, ни чего не получается!!!


Основание пирамиды с вершиной S есть параллелограмм ABCD, в котором AC = 12. Известно, что пирамиду S ABCD можно вписать в сферу, причем центр описанной сферы лежит на отрезке SK, где K - точка пересечения диагоналей основания, и делит его в отношении 13 : 5, считая от вершины S. Объём V пирамиды S ABCD равен 108. Найдите синус двугранного угла между плоскостями ASC и BSD.

4076. Юрий, 16 декабря 2011, 13:37:13
Можно пожалуйста с решением!!!!!!!!!!!!!! за ранее огромное спасибо!!!!!!!!

4077. Alla, 16 декабря 2011, 19:51:42
Lexxus, пожалуйста, объясните как решить задачу.
Конкурс исполнителей проводится в 5 дней. Всего заявлено 80 выступлений — по одному от каждой страны. В первый день 8 выступлений, остальные распределены поровну между оставшимися днями. Порядок выступлений определяется жеребьёвкой. Какова вероятность, что выступление представителя России состоится в третий день конкурса?

4078. Alla, 18 декабря 2011, 21:24:26
Какие Вы "мобильные". Уже сама разобралась, помощь не требуется

4079. Наташа, 19 декабря 2011, 13:05:19
Пожалуйста помогите решить.
Найдите объем шара ,вписанного в усеченный конус, образующая которого равна 10 см и наклонена к плоскости основания под углом 45 градусов

4080. Виктория, 19 декабря 2011, 14:50:52
1)в правильной треугольной пирамиде боковая грань наклонена к плоскости основания под углом альфа. расстояние от основания высоты пирамиды до её апофемы равно l. Найдите: а) апофему пирамиды; б) боковую поверхность пирамиды

2) основание пирамиды - прямоугольный треугольник с катетами 6 и 8 см. все двугранные углы при основании пирамиды равны 60 градусов. найти полную поверхность пирамиды.

3) основание пирамиды - квадрат с периметром 16см. две смежные боковые грани пирамиды перпендикулярны плоскости основания. площадь меньшего диагонального сечения пирамиды вдвое меньше площади основания. найдите площадь большего диагонального сечения.

главное 3я задача) заранее благодарна)

4081. Lexxus, 19 декабря 2011, 20:12:47
4080. Виктория, 19 декабря 2011, 14:50:52
основание пирамиды - квадрат с периметром 16см. две смежные боковые грани пирамиды перпендикулярны плоскости основания. площадь меньшего диагонального сечения пирамиды вдвое меньше площади основания. найдите площадь большего диагонального сечения.

Для определённости, пирамида - ABCDE, ABCD - основание, BEC и DEC перпендикулярны ABCD.

Сторона основания равна 16/4 = 4.
Площадь основания равна 4^2 = 16.
Пусть EC = h - высота пирамиды.

Меньшее диагональное сечение - прямоугольный треугольник ACE, его площадь равна AC*EC/2 = h*2*sqrt(2).

По условию, она вдвое меньше площади основания, т.е.

2*sqrt(2)*h = 8,
h = 4/sqrt(2) = 2*sqrt(2)

Большее диагональное сечение - равнобедренный треугольник BED.
Его основание BD (диагональ квадрата) = 4*sqrt(2)
Его высота EO (O - точка пересечения диагоналей ABCD) из прямоугольного треугольника ECO по теореме Пифагора равна sqrt(h^2+OC^2) = sqrt((2*sqrt(2))^2+(4*sqrt(2)/2)^2) = sqrt(16) = 4.

Отсюда площадь BED = BD*EO/2 = 4*sqrt(2)*4/2 = 8*sqrt(2).

Ответ: 8*sqrt(2)



4082. Lexxus, 19 декабря 2011, 20:16:24
4077. Alla, 16 декабря 2011, 19:51:42
Lexxus, пожалуйста, объясните как решить задачу.
Конкурс исполнителей проводится в 5 дней. Всего заявлено 80 выступлений — по одному от каждой страны. В первый день 8 выступлений, остальные распределены поровну между оставшимися днями. Порядок выступлений определяется жеребьёвкой. Какова вероятность, что выступление представителя России состоится в третий день конкурса?
4078. Alla, 18 декабря 2011, 21:24:26
Какие Вы "мобильные". Уже сама разобралась, помощь не требуется

Ай-яй-яй, какое горе.

4083. Нинуся, 20 декабря 2011, 22:06:00
В правильной шестиугольной призме ABCDEFA1B1C1D1У1F1, все ребра которой равны 1, найдите расстояния от точки B до прямой EF1.

Попогите, пожалуйста, решить.
Нашла искомое расстояние (BF1), не могу найти BF, чтобы потом по теореме Пифагора вычислить BF1 (у одноклассника BF равно корню из 3). У нас в пробном ЕГЭ такая задачка попалась..

4084. Нинуся, 20 декабря 2011, 22:18:55
В правильной шестиугольной призме ABCDEFA1B1C1D1E1F1, все ребра которой равны 1, найдите расстояния от точки B до прямой A1D1 .

И если возможно, ещё решите и это. С ответом не сходится совершенно...

4085. 4050, 21 декабря 2011, 19:02:02
плоскости альфа и бета параллельны. прямая a пересекает плоскости альфа и бета соответственно в точках A и B, а прямая b в точках C и D. Найдите взаимное расположение прямых a и b

4086. Lexxus, 22 декабря 2011, 12:25:07
4083. Нинуся, 20 декабря 2011, 22:06:00
В правильной шестиугольной призме ABCDEFA1B1C1D1E1F1, все ребра которой равны 1, найдите расстояния от точки B до прямой EF1.

Рассмотрим треугольник BEF1.
EF1 = sqrt(2) - диагональ квадрата со стороной 1.
BE = 2 (это диаметр окружности, описанной вокруг правильного шестиугольника со стороной, равной 1).
BF1 тоже равно 2 (ну, например, из тех соображений, что прямоугольники BCEF и BFF1B1 равны, а BE и BF1 - это их диагонали).

Значит, треугольник BEF1 - равнобедренный, и расстояние между точкой B и прямой EF1 будет равно его высоте BH, которая равна
sqrt(BE^2-(EF1/2)^2) = sqrt(4-1/2) = sqrt(7/2)

4087. Елена, 22 декабря 2011, 14:20:37
1.высота конуса равна 4 корень из 3 см, а угол при вершине осевого сечения равен 120 градуссов. найдите площадь основания конуса.

4088. Sevilya, 22 декабря 2011, 21:40:28
Помогите пожалуйста решить. в прямом параллелепипеде стороны основания 2 корень из 2 см. и 5 см. образуют угол 45 градусов меньшая диагональ параллелепипеде равна 7 см. найдите его объем

4089. Айгуль, 23 декабря 2011, 11:44:57
Y=log2 x2 +2х+9 найти наименьшее значение функции

помогите, плиз!

4090. Айгуль, 23 декабря 2011, 11:46:26
4089. Айгуль, 23 декабря 2011, 11:44:57
Y=log2(2-основание) x^2 +2х+9 найти наименьшее значение функции

помогите, плиз!

4091. Lexxus, 23 декабря 2011, 13:11:30
4090. Айгуль, 23 декабря 2011, 11:46:26
Y=log2(x^2 +2х+9) найти наименьшее значение функции

А в чём проблема? Если просят наименьшее значение без указания интервала, значит, есть минимум. То есть, просто берешь производную, приравниваешь к нулю, находишь точки экстремума, вычисляешь значение функции в этих точках, берёшь меньшее.

4092. Костя, 23 декабря 2011, 17:26:10
Найдите площадь четырёхугольника ABCD,в котором AB=5 см,BC=13см,CD=9см,DA=15см,AC=12см

4093. Альберт, 23 декабря 2011, 17:38:03
AFперпендикулярно. (ABC) Найти расстояние от F до СВ. треугольник АВС равнобедренный

4094. юля, 23 декабря 2011, 19:47:37
вычисите cos 45 градусов минус sin в квадрате 150 градусов плюс cos 120 градусов?

4095. петя, 24 декабря 2011, 22:17:03
помогите решить задачу из ЕГЭ В10:

в кармане у Пети было 4 монеты по рублю и 2 монеты по 2 рубля Петя не глядя переложи какие то 3 монеты в другой карман. найдите вероятность того что обе двух рублёвые монеты лежат в одном кармане.

4096. сергей, 27 декабря 2011, 12:31:20
В равнобедренном треугольнике ABC провели биссектриссу NP. Какие значения может принимать угол BAD, если AP+PB=BC?

4097. Иван, 27 декабря 2011, 15:48:57
Основанием пирамиды SABC служит прямоугольный треугольник ABC, в котором гипотенуза АВ = 26 и катет АС = 24; ребро SA перпендикулярно к плоскости основания АВС и равно 18. Определить боковую поверхность этой пирамиды.

4098. Иван, 27 декабря 2011, 18:46:32
Основанием пирамиды служит прямоугольный треугольник с гипотенузой 26 см и катетом 24 см. Ребро, проходящее через их общую вершину, является высотой пирамиды и равно 18. Найдитебоковую поверхность пирамиды.

4099. Сергей, 29 декабря 2011, 13:20:08
Найдите cosa, если sina=(2*корень из 6)/ 5) и а пренадлежит (п/2; п)
Будте добры)

4100. Алена, 3 января 2012, 15:44:54
Высота конуса 20 см расстояние от центра основания до образующей равна 12см. Найдите объем конуса. помогите пожалуста))))))))))))))))))

4101. Слава, 6 января 2012, 13:59:27
Позвольте узнать что твориться с заданиями В8 я уже не знаю какие там могут быть ответы потому что они без логики, как будто наугад отвечали,(кроме типов экстремума и параллельности касательной к прямым) в особенности к заданиям типа количеств min и max точек.
или я такой идиот что не знаю как их делать или задачи такие, помогите пожалуйста.

4102. Катя, 6 января 2012, 15:53:34
в треугольнике abc угол a равен 45,а высота bd отсекает от основания ac отрезок dc=12см. найти площадь треугольника abc и высоту, проведенную к стороне bc. помогите решить)

4103. Леонид, 6 января 2012, 19:58:22
4101. Слава, 6 января 2012, 13:59:27
Всё там нормально и логично. Внимательно читай условие - график чего дан. От свойства фуекции к производной. Например: найти число целых точек, в которых производная положительна. Смотри промежутки возрастания по графику и на них (на промежутках) считай целые точки. Так все задания.

4104. Слава, 8 января 2012, 16:36:39
4103. Леонид, 6 января 2012, 19:58:22
Ок с этим понял а что же с количеством точек экстремума? у нас некоторые типы задач не прорабатывались в шк поэтому некоторые учу прям тут на ходу=(

4105. Антон, 8 января 2012, 22:21:21
В задаче 5565 неправильные исходные данные(расход топлива бензина 7 вместо 6)
http://live.mephist.ru/show/mathege2010/view/id/5565/
http://mathege.ru/or/ege/ShowProblems?offset=6&protoId=26676

4106. Антон, 9 января 2012, 16:41:44
А в 5441 ещё больше различий с оригиналом

4107. Жанна, 11 января 2012, 10:55:44
Рябята, не разобралась как представить решение задачи. Подскажите.

4108. Жанна, 11 января 2012, 14:40:16
4100. Алена, 3 января 2012, 15:44:54
Высота конуса 20 см расстояние от центра основания до образующей равна 12см. Найдите объем конуса. помогите пожалуста))))))))))))))))))


Алена,"расстояние от центра основания до образующей" - это радиус окружности, а дальше по формуле.

4109. BARMEN, 11 января 2012, 19:28:51
это мечта сдать егэ хреново хорошо...нет бы как раньше по билетам!!!!не представляю что будет....куда мы катимся...хотя при поступлении это хорошо не пересдаешь предметы...хть бы повезло...бывает попадается на егэ задания которые уже ты решал.у меня у знакомой выпал тест который она решала и сдала на 5 егэ

4110. BARMEN, 11 января 2012, 19:31:07
блин написал не понятно сдать егэ хорошо!херово что не все как раньше

4111. Lexxus, 11 января 2012, 19:57:24
4108. Жанна, 11 января 2012, 14:40:16
4100. Алена, 3 января 2012, 15:44:54
Высота конуса 20 см расстояние от центра основания до образующей равна 12см. Найдите объем конуса. помогите пожалуста))))))))))))))))))



Алена,"расстояние от центра основания до образующей" - это радиус окружности, а дальше по формуле.


Неправильно. Это именно расстояние - перпендикуляр, опущенный из центра основания к образующей.
Если вершина конуса O1, центр основания O, образующая O1A, а перпендикуляр - OB, то прямоугольные треугольники AO1O и OBO1 - подобны по двум углам. Значит, O1O/O1B = OA/OB

O1B = sqrt(O1O^2-OB^2) = sqrt(20^2-12^2) = 16
OA = O1O/O1B*OB = 20/16*12 = 15

Объём конуса
V = (1/3)*pi*(15^2)*20 = 1500*pi

4112. OS, 14 января 2012, 09:13:57
В задании 77382 указан наименьший из корней 12. Почему?

4113. Lexxus, 14 января 2012, 13:04:39
4112. OS, 14 января 2012, 09:13:57
В задании 77382 указан наименьший из корней 12. Почему?

Потому что он единственный. Основанием логарифма может быть только положительное число.

4114. ОльгаНик, 14 января 2012, 17:48:10
В задаче 16225 ошибка в ответе. Должен быть ответ -15, 36

4115. Lexxus, 14 января 2012, 17:52:23
4114. ОльгаНик, 14 января 2012, 17:48:10
В задаче 16225 ошибка в ответе. Должен быть ответ -15, 36

Принято, спасибо.

4116. Татьяна, 15 января 2012, 11:37:46
Помогите решить задачу. Всю ночь не спала.

Дана правильная прямая шестиугольная призма объема V.
Одна шестиугольная пирамида имеет основанием нижнее основание призмы, а вершина лежит в центре верхнего основания призмы.
Вторая шестиугольная пирамида имеет основанием шестиугольник с вершинами в серединах сторон верхнего основания призмы, а ее вершина лежит в центре нижнего основания призмы.
Найти объем общей части пирамид.

4117. Kattu22, 15 января 2012, 12:56:37
Lexxus! Исправьте ошибку в ответе номера 24745

4118. Lexxus, 15 января 2012, 13:14:00
4117. Kattu22, 15 января 2012, 12:56:37
Lexxus! Исправьте ошибку в ответе номера 24745

Принято, спасибо.

4119. tumor, 15 января 2012, 13:57:27
4116. Татьяна, 15 января 2012, 11:37:46
Помогите решить задачу. Всю ночь не спала.

Дана правильная прямая шестиугольная призма объема V.
Одна шестиугольная пирамида имеет основанием нижнее основание призмы, а вершина лежит в центре верхнего основания призмы.
Вторая шестиугольная пирамида имеет основанием шестиугольник с вершинами в серединах сторон верхнего основания призмы, а ее вершина лежит в центре нижнего основания призмы.
Найти объем общей части пирамид.


Решение можно попозже, я не участник. Мне просто интересно.

4120. рома., 15 января 2012, 18:23:55
помогите решить
надо найти cos,tg,ctg
sin альфа=-0,8 ,pi<альфа<3pi/2

4121. рома., 15 января 2012, 18:38:52
4120. рома., 15 января 2012, 18:23:55
помогите решить
надо найти cos,tg,ctg
sin альфа=-0,8 ,pi&lt;альфа&lt;3pi/2

мне очень надо в течении часа

4122. Lexxus, 15 января 2012, 19:18:46
4120. рома., 15 января 2012, 18:23:55
надо найти cos,tg,ctg
sin альфа=-0,8 ,pi<альфа<3pi/2


pi<альфа<3pi/2: синус и косинус отрицательны, тангенс и котангенс положительны.

Из основного тригонометрического тождества находишь модуль косинуса, тангенс и котангенс находишь по определению.

Кури учебник, иногда помогает.

4123. рома., 15 января 2012, 19:37:37
а решение написать можешь?

4124. Lexxus, 15 января 2012, 21:06:40
4116. Татьяна, 15 января 2012, 11:37:46
Помогите решить задачу. Всю ночь не спала.

Дана правильная прямая шестиугольная призма объема V.
Одна шестиугольная пирамида имеет основанием нижнее основание призмы, а вершина лежит в центре верхнего основания призмы.
Вторая шестиугольная пирамида имеет основанием шестиугольник с вершинами в серединах сторон верхнего основания призмы, а ее вершина лежит в центре нижнего основания призмы.
Найти объем общей части пирамид.

И где такое задают? Если я нигде не ошибся, то ответ - V/14.
Но один интеграл таки взять пришлось, вряд ли для школьной задачи подойдёт такое решение, а другого я не вижу.

4125. tumor, 15 января 2012, 23:11:49
4124. Lexxus, 15 января 2012, 21:06:40
4116. Татьяна, 15 января 2012, 11:37:46
Помогите решить задачу. Всю ночь не спала.

Дана правильная прямая шестиугольная призма объема V.
Одна шестиугольная пирамида имеет основанием нижнее основание призмы, а вершина лежит в центре верхнего основания призмы.
Вторая шестиугольная пирамида имеет основанием шестиугольник с вершинами в серединах сторон верхнего основания призмы, а ее вершина лежит в центре нижнего основания призмы.
Найти объем общей части пирамид.

И где такое задают? Если я нигде не ошибся, то ответ - V/14.
Но один интеграл таки взять пришлось, вряд ли для школьной задачи подойдёт такое решение, а другого я не вижу.


Это оказалась олимпиадная задача ( мне не сказали ), меня попросили решить, я не смогла. Задача для школьников. Потом узнала, что это олимпиада "Будущие исследователи.."
Спасибо, что не отругали за несвоевременное обращение, олимпиада оказывается еще идет.

4126. tumor, 15 января 2012, 23:17:06
4108. Жанна, 11 января 2012, 14:40:16
4100. Алена, 3 января 2012, 15:44:54
Высота конуса 20 см расстояние от центра основания до образующей равна 12см. Найдите объем конуса. помогите пожалуста))))))))))))))))))


Алена,"расстояние от центра основания до образующей" - это радиус окружности, а дальше по формуле.


расстояние - это длина перпендикуляра, из центра основания к образующей

4127. Антон, 16 января 2012, 10:48:09
24x во второй минус 2x минус 1 ровно ноль

4128. Олег, 16 января 2012, 21:02:16
Задание по математике № 24987.
В супермаркете проходит рекламная акция: покупая 2 шоколадки, покупатель ПОЛУЧАЕТ! еще одну шоколадку в подарок. Шоколадка стоит 35 рублей. Какое наибольшее число шоколадок ПОЛУЧИТ! покупатель за 310 рублей?
Вопрос. У Вас в ответах стоит получит 12 шт, а на деле 8 он КУПИТ!, а еще 4 ПОЛУЧИТ! т.е. если бы вопрос был сформулирован Сколько достанется шоколадок, то Ваш ответ правильный,

4129. Никита, 17 января 2012, 19:15:21
Через точку F биссектриссы угла BAC проведена прямая паралельная прямой AC и перессекающая луч AB в точке P.Вычислите градусныемеры углов треугольника APF,если уголFAC=20 градусов

4130. tumor, 18 января 2012, 13:31:59
4129. Никита, 17 января 2012, 19:15:21
Через точку F биссектриссы угла BAC проведена прямая паралельная прямой AC и перессекающая луч AB в точке P.Вычислите градусныемеры углов треугольника APF,если уголFAC=20 градусов


1) накрест лежащие углы
2) сумма углов треугольника

4131. Татьяна, 19 января 2012, 16:37:13
а как мне получить правильные ответы которые я сейчас решала..

4132. Kattu22, 19 января 2012, 17:13:15
Lexxus! Исправьте ошибку в ответе номера 8261

4133. Lexxus, 19 января 2012, 17:17:47
4132. Kattu22, 19 января 2012, 17:13:15
Lexxus! Исправьте ошибку в ответе номера 8261

Принято, спасибо.

4134. лена М, 20 января 2012, 08:55:27
Где найти задания С6 к ЕГЭ-2012 и решения к ним?

4135. 123, 21 января 2012, 20:46:35
4126. tumor, 15 января 2012, 23:17:06
4108. Жанна, 11 января 2012, 14:40:16
4100. Алена, 3 января 2012, 15:44:54
Высота конуса 20 см расстояние от центра основания до образующей равна 12см. Найдите объем конуса. помогите пожалуста))))))))))))))))))


Алена,"расстояние от центра основания до образующей" - это радиус окружности, а дальше по формуле.

расстояние - это длина перпендикуляра, из центра основания к образующей

Рассмотрите пары подобных треугольников (половинку осевого сечение и треугольник который отсекается перпендикуляром)

4136. Vlad, 23 января 2012, 14:27:01
помогите пожалуйста!!!!
Через вершину прямого угла С в равнобедренном треугольнике СДЕ проведена прямая САЮ,перпендикулярная плоскоститреугольника.Известно,что СА=35 дм,СД=12 корень из 2 дм.Найти расстояние от точки А до прямой ДЕ.

4137. Vlad, 23 января 2012, 14:28:34
ошибочка небольшая
Через вершину прямого угла С в равнобедренном треугольнике СДЕ проведена прямая СА,перпендикулярная плоскоститреугольника.Известно,что СА=35 дм,СД=12 корень из 2 дм.Найти расстояние от точки А до прямой ДЕ.Решение

4138. Таня, 23 января 2012, 16:13:02
:отрезок AD-биссектриса треугольника ABC.Через точку D проведена прямая, пересекающая сторону AB в точке E так,что AE=ED.Найдите углы треугольника AED,если угол BAC=64 градуса.

4139. Lexxus, 23 января 2012, 16:49:28
4137. Vlad, 23 января 2012, 14:28:34
Через вершину прямого угла С в равнобедренном треугольнике СДЕ проведена прямая СА,перпендикулярная плоскоститреугольника.Известно,что СА=35 дм,СД=12 корень из 2 дм.Найти расстояние от точки А до прямой ДЕ.

Проведём перпендикуляр CH к DE. Его длина будет равна CD/sqrt(2) = 12.
Так как CA перпендикулярна плоскости треугольника, то она перпендикулярна и CH.
Расстояние между точкой A и прямой DE будет равно AH, а AH - это гипотенуза прямоугольного треугольника ACH, значит,
AH = sqrt(AC^2+CH^2) = 37.
Ответ: 37 дм

4140. Lexxus, 23 января 2012, 16:53:56
4138. Таня, 23 января 2012, 16:13:02
отрезок AD-биссектриса треугольника ABC.Через точку D проведена прямая, пересекающая сторону AB в точке E так,что AE=ED.Найдите углы треугольника AED,если угол BAC=64 градуса

И чё, всё?
Треугольник AED по условию равнобедренный, угол EAD по условию равен половине угла BAC. Что непонятно?

4141. Таня, 23 января 2012, 16:59:51
Lexxus, ты каким местом задачу читал? В условии сказано:НАЙТИ УГОЛ AED

4142. Lexxus, 23 января 2012, 17:08:35
Хроническая тупость тут карается игнором.

4143. Neon, 23 января 2012, 22:24:09
1. Основание прямого параллелепипеда - ромб со стороной 8 см, острый угол которого равен 60 градусов. Найдите длину меньшей диагонали параллелепипеда, если его высота равна 15 см.

2. Диагональ прямоугольного параллелепипеда равна 7 см, а диагональ его боковой грани равна 5 см. Найти высоту параллелепипеда, если в его основании лежит квадрат.
please.....help me...

4144. Илья, 23 января 2012, 23:06:26
В равнобедренном треугольнике АВС угол при вершине В равен 120 градусов , длина медианы АТ равна 7см . Вычеслите радиус окружности , описанной около треугольника АВС

4145. Дмитрий, 25 января 2012, 12:24:13
http://live.mephist.ru/show/mathege2010/view/id/53255/
радиус равен корень из трех умножить на сторону разделить на шесть
R = 46*корень из 3*корень из 3 / 6 == 23
почему "Верный ответ пока не определен" когда "23" ответили уже несколько человек?

4146. DEN, 27 января 2012, 11:26:23
Lexxus! Помоги пожалуйста решить. Срочно!!!Дан выпуклый 6-угольник abcdef. Известно учто углы fae=bdc, а 4-ех угольники abdf и acde являются вписанными. Докажите что прямые bf и ce параллельны. Спасибо.

4147. Поддубный, 27 января 2012, 12:13:31
Народ подскажите пожалуйста кто-нибудь)Как мне найти решение ко всем заданиям В*?Именно Решение но не ответы?

4148. Надюха, 27 января 2012, 17:23:44
Люди как решать часть В ? Только все и точно . Но не ответы

4149. 123, 29 января 2012, 16:02:39
4146. DEN, 27 января 2012, 11:26:23
Lexxus! Помоги пожалуйста решить. Срочно!!!Дан выпуклый 6-угольник abcdef. Известно учто углы fae=bdc, а 4-ех угольники abdf и acde являются вписанными. Докажите что прямые bf и ce параллельны. Спасибо.


И как не стыдно, хоть бы конца олимпиады дождался

4150. Kattu22, 30 января 2012, 00:42:46
Lexxus! Исправьте, пожалуйста, ответ к заданию 6993. Спасибо

4151. Lexxus, 30 января 2012, 17:29:17
4150. Kattu22, 30 января 2012, 00:42:46
Lexxus! Исправьте, пожалуйста, ответ к заданию 6993. Спасибо

Принято, спасибо.

4152. ОльгаНик, 30 января 2012, 17:49:12
В задаче 7129 дан неверный ответ. Правильный ответ 8

4153. Lexxus, 30 января 2012, 17:53:19
4152. ОльгаНик, 30 января 2012, 17:49:12
В задаче 7129 дан неверный ответ. Правильный ответ 8

Принято, спасибо.

4154. Anjella, 31 января 2012, 18:25:44
Кубик весит 10 г. Сколько граммов будет весить кубик,ребро которого в 3 раза больше ,чем ребро первого кубика,если оба кубика изготовлены из одинакового материала?

4155. Алексей, 2 февраля 2012, 15:07:09
Помогите пожалуйста решить задачу. С5
Найдите все значения a, при каждом из которых наименьшее значение функции
f(x)=2ax+|x^2 - 8x +7| больше 1.
Не могу понять как решается эта система

4156. Алексей, 2 февраля 2012, 15:15:11
Anjella
масса равна плотность умнажить на объём m=p x V
значит m1/m2 = V1/V2 V1= 3a, а- сторона куба
V2=3(3а х 3а х 3а)= 3а х 27 значит, m1/m2=1/27; m2=10х27=270 грамм

4157. Алина, 2 февраля 2012, 19:37:30
Ребят, ну совсем просто, С1 помогите)
(6 cos^2 x + 5cosx - 4) sqrt(-4 sinx) = 0

Cпасибо, если отпишитесь)

4158. С2, 4 февраля 2012, 22:07:05
В прямоугольном параллелепипеде АВСDA1B1C1D1 ребро АВ=8, АD=6, СС1=7. Найти угол между плоскостями ADB1 и AD1B1

4159. Lexxus, 5 февраля 2012, 00:12:14
4157. Алина, 2 февраля 2012, 19:37:30
(6 cos^2 x + 5cosx - 4) sqrt(-4 sinx) = 0


Это уравнение эквивалентно совокупности:

1) Уравнение:
sqrt(-4sin(x)) = 0, откуда
sin(x) = 0
x = pi*n

2) Система:
2.1) 6 cos^2 x + 5 cosx - 4 = 0
2.2) -4sin(x) >= 0

В уравнении (2.1) заменим cos(x) на t и получим обычное квадратное уравнение с корнями -4/3 и 1/2.
-4/3 нам не подходит, поскольку косинус не может быть по модулю больше единицы.

Остаётся cos(x) = 1/2,
x = (+/-)pi/3 + 2*pi*n

Решая неравенство (2.2), получим:
pi+2*pi*n < x < 2*pi + 2*pi*n
(часть единичной окружности, лежащая в нижней полуплоскости).

Из решений уравнения (2.1) этому условию удовлетворяют только корни
x = -pi/3 + 2*pi*n.

Ответ: pi*n, -pi/3 + 2*pi*n

4160. Камилла, 5 февраля 2012, 20:08:35
Помогите пожалуйста решить задачу с3.

Log (осн. |х| )(&#8730;(9-х^2) -x -1)&#8805; 1

4161. Камилла, 5 февраля 2012, 20:14:22
Помогите пожалуйста решить задачу с3.

Log (осн. |х| )(корень(9-х^2) -x -1)>= 1

4162. Эльвина, 6 февраля 2012, 15:31:35
помогите решить задачу пожалуйста в6.

в треугольнике ABC угол C равен 90о,угол A равен 60 о,AB=8.найти AC

4163. Lexxus, 6 февраля 2012, 17:05:42
4161. Камилла, 5 февраля 2012, 20:14:22
Log (осн. |х| )(корень(9-х^2) -x -1)>= 1

Итак, поехали. Сперва найдём ОДЗ.

1.1) Условие на основание логарифма:
|x| > 0 => x <> 0 (здесь и дальше "<>" значит "не равно")
1.2) Условие на основание логарифма:
|x| <> 1 => x <> -1, x <> 1
1.3) Условие на выражение под знаком квадратного корня:
9-x^2 >= 0 => (3-x)(3+x) >= 0 => x принадлежит [-3;3]
1.4) Условие на выражение под знаком логарифма:
sqrt(9-x^2)-x-1 > 0
sqrt(9-x^2) > x+1 =>
1.4.1) либо (x+1) < 0 => x < -1
1.4.2) либо система
{9-x^2 > (x+1)^2, x >= -1}
Решая первое неравенство системы, получим
x принадлежит ( (-1-sqrt(17))/2; (-1+sqrt(17))/2 ).
Поскольку sqrt(17) - это примерно sqrt(16)=4, то
(-1-sqrt(17))/2 примерно равно -2.5
(-1+sqrt(17))/2 примерно равно 1.5
Итак, в (1.4.2) у нас получается:
x принадлежит [ -1;-1+sqrt(17))/2 )

Объединяя все условия из (1.3) и (1.4), имеем:
x принадлежит [ -3 ; (sqrt(17)-1)/2 )

Объединив это с (1.1) и (1.2), имеем полное ОДЗ:
[-3; -1) U (-1;0) U (0;1) U (1; (sqrt(17)-1)/2)

Теперь, собственно, само неравенство.
В зависимости от значения x у нас будет тут четыре случая:

2.1) x < -1 => abs(x) = -x > 1
log_(-x)(sqrt(9-x^2)-x-1) >= 1
Основание логарифма больше единицы => функция возрастающая => при потенциировании неравенство знак не меняет:

sqrt(9-x^2)-x-1 >= -x
9-x^2 >= 1
x^2 <= 8
x принадлежит [-2sqrt(2); 2sqrt(2)]
(sqrt(2) - это примерно 1.4, следовательно, 2sqrt(2) = примерно 2.8)
Итого в (2.1) имеем:
x принадлежит [-2sqrt(2);-1)

2.2) -1 < x < 0 => abs(x) = -x < 1
log_(-x)(sqrt(9-x^2)-x-1) >= 1
Основание логарифма меньше единицы => функция убывающая => при потенциировании неравенство меняет знак:
sqrt(9-x^2)-x-1 <= -x
x^2 >= 8
x принадлежит (-бесконечность; -2sqrt(2)] U [2sqrt(2); бесконечность).
С условием -1 < x < 0 это не пересекается, значит, в случае (2.2) решений нет.

2.3) 0 < x < 1 => abs(x) = x < 1
log_(x)(sqrt(9-x^2)-x-1) >= 1
Основание логарифма меньше единицы => функция убывающая => при потенциировании неравенство меняет знак:
sqrt(9-x^2)-x-1 <= x
В общем, решается это примерно также, как (1.4). Получится
2(sqrt(11)-1)/5 <= x <= 3

Тут нам важно понять, 2(sqrt(11)-1)/5 больше или меньше 1.
Решение в случае (2.3) будет только если
2(sqrt(11)-1)/5 < 1 => sqrt(11) < 3.5.
3.5^2 = 12.25 > 11, то есть в случае (2.3) мы всё-таки будем иметь решение
[2(sqrt(11)-1)/5; 1)

2.4. x > 1 => abs(x) = x > 1
log_(x)(sqrt(9-x^2)-x-1) >= 1
Основание логарифма больше единицы => функция возрастающая => при потенциировании неравенство знак не меняет:
sqrt(9-x^2)-x-1 >= x
Такое же неравенство, но с обратным знаком, мы уже только что решили, и выяснили, что больший корень меньше единицы. Следовательно, тут решений нет.

Итак, из (2.1) и (2.3) имеем:
x принадлежит [-2sqrt(2);-1) U [2(sqrt(11)-1)/5; 1)

А вот, для наглядности, как это всё выглядит:


4164. Мария, 6 февраля 2012, 19:07:06
Площади трех граней прямоугольного параллелепипеда 2,4,8метров в квадрате.Чему равен объем параллелепипеда?

4165. Lexxus, 6 февраля 2012, 19:33:05
4164. Мария, 6 февраля 2012, 19:07:06
Площади трех граней прямоугольного параллелепипеда 2,4,8метров в квадрате.Чему равен объем параллелепипеда?

Ну дык эта.
Обозначим длины граней - x, y, z
Из известных площадей граней составляем систему
{xy=2, yz=4, xz=8}
И решаем, решаем, думаем, соображаем.

4166. Анастасия , 6 февраля 2012, 19:46:57
наибольшая диагональ правильной шестиугольной призмы равна 8 и образует с боковым ребром угол в 30 градусов.найти объем призмы???

4167. Lexxus, 6 февраля 2012, 20:29:46
4166. Анастасия , 6 февраля 2012, 19:46:57
наибольшая диагональ правильной шестиугольной призмы равна 8 и образует с боковым ребром угол в 30 градусов.найти объем призмы???

Наибольшая диагональ, боковое ребро и большая диагональ шестиугольника в основании призмы составляют прямоугольный треугольник.
Дальше всё просто.
И решаем, решаем, думаем, соображаем.

4168. Камилла, 7 февраля 2012, 07:40:45
спасибо огромное.)

4169. маша, 8 февраля 2012, 20:42:55
дан прямоугольный параллелепипед ABCDA1B1C1D1. AC=13см, DC=5см, AA1= 12корень из 3 см. Вычислите градусную меру двугранного угла ADCA1.

помогите плиз.

4170. Дмитрий, 8 февраля 2012, 22:07:06
В задаче 283647
неправильный ответ,
правильный ответ 0.94
Приплюсуйте мне бал:):)

4171. Анастасия, 9 февраля 2012, 20:49:39
В треугольнике АВС угол С равен 90 градусов, АВ = 20, tg А = 3/4. Найдите ВС. Подскажите решение пожалуйста!!

4172. 123, 9 февраля 2012, 21:34:44
4171. Анастасия, 9 февраля 2012, 20:49:39
В треугольнике АВС угол С равен 90 градусов, АВ = 20, tg А = 3/4. Найдите ВС. Подскажите решение пожалуйста!!

Пусть катеты а и b равны 3х и 4х по т.Пифагора - 9х^2+16x^2=25x^2 => 20=5x
х=4 => ВС=4*3=12

4173. 123, 9 февраля 2012, 21:37:41
4170. Дмитрий, 8 февраля 2012, 22:07:06
В задаче 283647
неправильный ответ,
правильный ответ 0.94
Приплюсуйте мне бал:):)

(120-8)/120=112/120=0,93(3) всё правильно

4174. ОльгаНик, 11 февраля 2012, 18:29:12
Дмитрий прав. Решение120/(120+8)=0,9375, т.е. 0,94 после округления.

4175. 123, 12 февраля 2012, 14:34:48
4174. ОльгаНик, 11 февраля 2012, 18:29:12
Дмитрий прав. Решение120/(120+8)=0,9375, т.е. 0,94 после округления.


Обсуждалось выше

4176. Ди, 12 февраля 2012, 17:27:46
http://live.mephist.ru/show/mathege2010/view/id/23161/
объясните,пожалуйста,почему 25,а не 40?

4177. Lexxus, 12 февраля 2012, 17:38:19
4170. Дмитрий, 8 февраля 2012, 22:07:06
В задаче 283647
неправильный ответ,
правильный ответ 0.94
Приплюсуйте мне бал:):)

Спасибо, принято. Балл уже приплюсовался :)

4178. Lexxus, 12 февраля 2012, 17:39:33
4176. Ди, 12 февраля 2012, 17:27:46
http://live.mephist.ru/show/mathege2010/view/id/23161/
объясните,пожалуйста,почему 25,а не 40?

Потому что 40 - это явно перебор. А 25 - в самый раз.
Учим формулу площади трапеции.

4179. александр, 12 февраля 2012, 19:01:38
отмените егэ

4180. Сергей, 39 лет, 13 февраля 2012, 19:31:46
Прошу администрацию сайта поставить автобан на маты.......дети ж читают...Моя дочь (ей 11 лет) нашла ваш сайт.....я поставил родительский контроль на посещение вашего сайта. Заранее благодарен.

4181. Lexxus, 13 февраля 2012, 20:19:25
4180. Сергей, 39 лет, 13 февраля 2012, 19:31:46
Прошу администрацию сайта поставить автобан на маты.......дети ж читают...Моя дочь (ей 11 лет) нашла ваш сайт

Вот вам мой ответ двухлетней давности, с тех пор ничего не изменилось.

4180. Сергей, 39 лет, 13 февраля 2012, 19:31:46
я поставил родительский контроль на посещение вашего сайта.

А вот за это - спасибо. Теперь ваша дочь точно пополнит армию постоянных посетителей.

Да, ещё родительский контроль на посещение школы поставьте. Лично я все матерные лексические единицы выучил именно там.

4182. Ди, 14 февраля 2012, 13:35:53
4178. Lexxus, 12 февраля 2012, 17:39:33
4176. Ди, 12 февраля 2012, 17:27:46
http://live.mephist.ru/show/mathege2010/view/id/23161/
объясните,пожалуйста,почему 25,а не 40?

Потому что 40 - это явно перебор. А 25 - в самый раз.
Учим формулу площади трапеции.


Как логично то,что 40-перебор,а 25-нет...
Наверное,самое"понятное" объяснение.

4183. Mary, 14 февраля 2012, 18:33:49
В треугольнике ABC AC=BC,высота CH=0,5,sin A=корень из 17/17. Найдите AB. Помогите,очень срочно нужно. Пожалуйста!!!

4184. 123, 14 февраля 2012, 21:51:06
4182. Ди, 14 февраля 2012, 13:35:53
4178. Lexxus, 12 февраля 2012, 17:39:33
4176. Ди, 12 февраля 2012, 17:27:46
http://live.mephist.ru/show/mathege2010/view/id/23161/
объясните,пожалуйста,почему 25,а не 40?

Потому что 40 - это явно перебор. А 25 - в самый раз.
Учим формулу площади трапеции.

Как логично то,что 40-перебор,а 25-нет...
Наверное,самое"понятное" объяснение.

Площадь трапеции - полусумма оснований на высоту, вы наверно с параллелограммом спутали.
4183. Mary, 14 февраля 2012, 18:33:49
В треугольнике ABC AC=BC,высота CH=0,5,sin A=корень из 17/17. Найдите AB. Помогите,очень срочно нужно. Пожалуйста!!!

СН разобьёт исходный треугольник на два прямоугольных, а дальше по определению синуса (против.катет/гипот)

4185. ОльгаНик, 16 февраля 2012, 22:00:08
В задаче 23823 дан неверный ответ 12. Правильный ответ 10

4186. Lexxus, 16 февраля 2012, 22:06:19
4185. ОльгаНик, 16 февраля 2012, 22:00:08
В задаче 23823 дан неверный ответ 12. Правильный ответ 10

Принято, спасибо.

4187. JAGUAR, 21 февраля 2012, 12:44:26
Высота конуса=8см.Угол при вершине осевого сечения=120 градусам.Найти 1)S сечения конуса плоскостью проходящей через 2 образующие угол между которыми=30 градусам.2)S боковой поверхности ...Помогите решить прошу=(

4188. solo4848@mail.ru, 21 февраля 2012, 19:14:57
проведи высрту, половина осевого сечения - прямоугольный треугольник с катетом 8 (высота коеуса), углом 60, значит и углом 30 - образующая 16 - по катету против 30. Имеем сечение - по условию - треугольник со сторонами по 16 и углом между ними 30 - находи площадь , как полупроизведение сторон на синус угла между ними - это 1). радиус по т. Пифагора в половине осевого (того же) сечения и боковая пов piRL - всё известно. Успехов.

4189. Никита, 22 февраля 2012, 17:40:41
http://uztest.ru/Data/exercise/XIB4pramtri/XIB4pramtri_25c.gif
помогите решить !люди добрые!

4190. Kattu22, 22 февраля 2012, 23:23:24
Lexxus! Исправьте, пожалуйста ответ к заданию 245381. Спасибо!

4191. Lexxus, 23 февраля 2012, 11:50:37
4190. Kattu22, 22 февраля 2012, 23:23:24
Lexxus! Исправьте, пожалуйста ответ к заданию 245381. Спасибо!

Принято, спасибо.
Хотя рисунок такой, что так и тянет дать неправильный ответ. Обычно они более пропорциональны указанным длинам.

4192. Аленка, 23 февраля 2012, 23:38:14
Найдите объем многогранника, вершинами которого являются точки A, B, C, A1, B1, C1 правильной шестиугольной призмы , площадь основания которой равна 6, а боковое ребро равно 3.

знаю, что правильный ответ 3, но как к этому придти?? помогите, пожалуйста, решить.

4193. Михаил, 23 февраля 2012, 23:42:08
Найдите объем многогранника, вершинами которого являются точки B1, A, B, C, D прямоугольного параллелепипеда ABCDA1B1C1D1, у которого AB = 4, AD = 3, AA1 = 5.

4194. Татьяна, 24 февраля 2012, 14:14:40
1) Через вершину прямого угла С в равнобедренном треугольнике CDE проведена прямая СА=35 дм., CD=12 корей из 2. Найти расстояние от точки А до прямой DE?...

2). Дан прямоугольный параллелепипед ABCDA1B1C1D1, найдите друхгранный угол ADCA1, если АС=13 см. DC= 5 см. АА1= 12 корней из 3?

Выручайте!!!!!

4195. Просто Сережа, 24 февраля 2012, 21:08:01
[quote]4194. Татьяна, 24 февраля 2012, 14:14:40
1) Через вершину прямого угла С в равнобедренном треугольнике CDE проведена прямая СА=35 дм., CD=12 корей из 2. Найти расстояние от точки А до прямой DE?...
Татьяна! 12 корней из 2 (единица измерения?????) см ил дм?

4196. Просто Сережа, 24 февраля 2012, 21:18:04
[quote]4194. Татьяна, 24 февраля 2012, 14:14:40
1) Через вершину прямого угла С в равнобедренном треугольнике CDE проведена прямая СА=35 дм., CD=12 корей из 2. Найти расстояние от точки А до прямой DE?...
Если все-таки дм, а прямая является биссектрисой угла С, то ответ 23 дм

4197. Просто Сережа, 24 февраля 2012, 21:42:17
[quote]4194. Татьяна, 24 февраля 2012, 14:14:40

2). Дан прямоугольный параллелепипед ABCDA1B1C1D1, найдите друхгранный угол ADCA1, если АС=13 см. DC= 5 см. АА1= 12 корней из 3

угол 60 градусов. если нужно решение, пиши в личку

4198. Катерина, 25 февраля 2012, 18:05:10
В правильной шестиугольной призме ABCDEF...F1,все ребра которой равны 7,найдите расстояние от точки В до прямой D1C1.

4199. Lexxus, 26 февраля 2012, 23:11:39
4198. Катерина, 25 февраля 2012, 18:05:10
В правильной шестиугольной призме ABCDEF...F1,все ребра которой равны 7,найдите расстояние от точки В до прямой D1C1.

Прямая D1C1 параллельна прямой BE. Значит, расстояния от любой точки прямой BE до прямой D1C1 одинаковы.
Проведём перпендикуляр CH к прямой BE. Искомое расстояние будет равно длине гипотенузы C1H трямоугольного треугольника C1CH.

4200. Кристина, 27 февраля 2012, 21:05:26
Скажите а где найти тесты для 4 го класса

4201. Александр и Элла, 28 февраля 2012, 15:02:55
задание C5

4202. Tsokka, 1 марта 2012, 20:11:00
помогите решить задание C1. напишите в icq 642317829, я скину фото задания

4203. Nataly, 2 марта 2012, 19:00:53
Помогите решить задачу, пожалуйста:)

Расстояние между центрами двух окружностей, радиусы которых 17 см и 10 см, равно 21 см. Определить расстояние центров от точки, в которой прямая центров пресекается с общей касательной окружностей.

4204. Lexxus, 4 марта 2012, 15:20:56
4203. Nataly, 2 марта 2012, 19:00:53
Расстояние между центрами двух окружностей, радиусы которых 17 см и 10 см, равно 21 см. Определить расстояние центров от точки, в которой прямая центров пресекается с общей касательной окружностей.

Пусть O1 - радиус окружности с радиусом 17, O2 - центр окружности с радиусом 10. O1O2 = 21. AB - общая касательная, O1A = 17 и O2B = 10 - радиусы окружностей, опущенные к точкам касания. С - точка пересечения касательной с прямой O1O2



Проведём перпендикуляр O2H к O1A. AH = BO2 = 10.
Прямоугольные треугольники O1HO2 и O1AC имеют общий угол, следовательно, подобны.
Значит, O1C/O1O2 = O1A/O1H, откуда
O1C = 21*17/7 = 51
O2C = O1C-O1O2 = 51-21 = 30.

Ответ: 30 и 51

4205. Роман, 4 марта 2012, 20:11:21
В задании B4 (5387) ошибка!Задавайте корректные вопросы или пишите правильные ответы!А то вопрос - стоимость за один рейс, а в ответе - стоимость за всю перевозку!

4206. Kattu22, 4 марта 2012, 20:28:25
4205. Роман, 4 марта 2012, 20:11:21

цитата
В задании B4 (5387) ошибка!Задавайте корректные вопросы или пишите правильные ответы!А то вопрос - стоимость за один рейс, а в ответе - стоимость за всю перевозку!
Ответ в номере 5387 верный

4207. Ygrek, 5 марта 2012, 16:57:47
Добрый день!
Не поможете ли решить задачу? Заранее спасибо! Я ее решила, но с ответом не сходится. Ответ у меня - 0,225. А в презентации - 0,9.
Задача.
1. Конкурс исполнителей проводится в 5 дней. Всего заявлено 80 выступлений — по одному от каждой страны. В первый день 8 выступлений, остальные распределены поровну между оставшимися днями. Порядок выступлений определяется жеребьёвкой. Какова вероятность, что выступление представителя России состоится в третий день конкурса?
Мое решение: в третий день может быть 18 выступлений(( 80-8):4=18).
Число благоприятных событий-18.
Число всех событий- 80.
p= 18:80=0,225.
А откуда ответ 0,9? Да и в общем, интуиция подсказывает, что этого не может быть.

4208. Кристина, 5 марта 2012, 17:47:58
Добрый вечер . Помогите решить не получается .
После подорожания на 30% 1 кг персиков стал стоить 104 рубля. Сколько рублей стоил 1 кг персиков до подорожания .

пропорцию знаю как делать а дальше не получается ..ответ должен получиться 80

4209. артем, 7 марта 2012, 08:45:05
радиус основания цилиндра 1.5см, высота 4 см.чему равна диагональ осевого сечения?

4210. шаша+, 7 марта 2012, 23:52:33
кристина: После подорожания на 30% 1кг персиков стал стоить 104 руб. Сколько рублей стоил 1 кг до подорожания?

поле подорожания-130%----104руб
до подорожания- 100%----х руб, х=104умножаем на 100 и делим на 130, получаем 80......

4211. Александр, 8 марта 2012, 12:17:58
В задании В2 №212086, по моему, ошибка в формулировке вопроса задания.
Крутящий момент не может изменяться от 1000 Нм до 2500Нм, т.к. по графику
его максимальное значение 140 Нм. Ответ "120" применим к изменению Мкр.
при изменении оборотов от 1000 до 2500 об/мин.

4212. Александр, 8 марта 2012, 12:37:31
4209. артем, 7 марта 2012, 08:45:05
радиус основания цилиндра 1.5см, высота 4 см.чему равна диагональ осевого сечения?
Осевое сечение есть прямоугольник со сторонами 4 и3 см.
диагональ прям-ка есть гипотенуза тр-ка, катеты которого 4 и3см. Это - египетский тр-к, след-но, диагональ осевого сечения ( она же гипотенуза тр-ка)
равна ... правильно! 5-ти. Проверьте теоремой ПИФАГОРА.

4213. Ника, 11 марта 2012, 13:18:49
Треугольник АВС вписан в окружность с центром О. Найдите угол ВОС , если угол ВАС равен 28 градусов.

4214. леша, 11 марта 2012, 13:54:33
4213. Ника, 11 марта 2012, 13:18:49
Треугольник АВС вписан в окружность с центром О. Найдите угол ВОС , если угол ВАС равен 28 градусов.

решите плиз

4215. Lexxus, 11 марта 2012, 19:18:52
4213. Ника, 11 марта 2012, 13:18:49
Треугольник АВС вписан в окружность с центром О. Найдите угол ВОС , если угол ВАС равен 28 градусов.

Теорема о вписанном угле: "Вписанный угол равен половине центрального угла, опирающегося на ту же дугу".

BAC - вписанный угол, BOC - центральный, и оба они опираются на дугу BC.

4216. romka, 11 марта 2012, 20:27:45
помогите решить задачу:объем параллелепипеда АВСДА1В1С1Д1 равен 4,8. Найдите объем треугольной пирамиды АД1СВ1

4217. Diana, 12 марта 2012, 13:28:48
Пожалуйста помогите решить задачу B6. Острые углы прямоугольного треугольника равны 59 и 31 градусов. Найдите угол между биссектрисой и медианой, проведенными из вершины прямого угла. Ответ дайте в градусах.

4218. Ефремов Никита, 13 марта 2012, 10:28:46
даа вы лошары

4219. Евгений, 14 марта 2012, 16:51:13
Зачем так громко говорить о каком-то "банке задач ЕГЭ", если по сути то, что предлагаете вы --- это мусор, задачи части В вообще не нужно собирать, они однотипные и для решения их нужен самый минимум серого вещества, а вот того, что как раз и должно быть, то есть задач части С у вас нет. Почему?

4220. Евгений, 14 марта 2012, 16:51:47
Зачем так громко говорить о каком-то "банке задач ЕГЭ", если по сути то, что предлагаете вы --- это мусор, задачи части В вообще не нужно собирать, они однотипные и для решения их нужен самый минимум серого вещества, а вот того, что как раз и должно быть, то есть задач части С у вас нет. Почему это так?

4221. Денис, 15 марта 2012, 21:25:36
Помогите разобраться! Перепиши записи, исправляя ошибки. 1440-480:6=128; 1080:20:4+560

4222. Даниил, 16 марта 2012, 16:02:16
http://live.mephist.ru/show/mathege2010/view/id/283647/
здесь выбран не верный ответ: при округлении числа 0.9(3) до сотых будет 0.93, а не 0.94

4223. Lexxus, 17 марта 2012, 10:38:16
4222. Даниил, 16 марта 2012, 16:02:16
http://live.mephist.ru/show/mathege2010/view/id/283647/
здесь выбран не верный ответ: при округлении числа 0.9(3) до сотых будет 0.93, а не 0.94

Устал уже объяснять одно и то же.

4224. Даниил, 18 марта 2012, 10:46:39
4223. Lexxus, 17 марта 2012, 10:38:16
Устал уже объяснять одно и то же.


спасибо за разъяснение)

4225. Леонид, 18 марта 2012, 10:49:04
Увжаемый Lexxus, прошу дать консультацию. Лена четырежды бросает игральный кубик. В сумме у неё выпало 7 очков. Найдите вероятность того, что при втором броске выпало 4 очка.
Мои рассуждениия: 4 уже есть (не важно, от какого броска). Благоприятные: 4, 1, 1, 1; (один случай). Всего случаев - другие три кости бросаем все сразу, случаев 216. Вероятность 1:216 = Что не так в рассуждениях?

4226. Lexxus, 18 марта 2012, 15:20:13
4225. solo4848@mail.ru, 18 марта 2012, 10:49:04
Лена четырежды бросает игральный кубик. В сумме у неё выпало 7 очков. Найдите вероятность того, что при втором броске выпало 4 очка.
Мои рассуждениия: 4 уже есть (не важно, от какого броска). Благоприятные: 4, 1, 1, 1; (один случай). Всего случаев - другие три кости бросаем все сразу, случаев 216. Вероятность 1:216 = Что не так в рассуждениях?

Мы знаем, что при последовательном бросании 4 костей выпало 7 очков. Такое может быть всего в 3 случаях:
1) выпадает одна четвёрка и три единицы
2) выпадает тройка, двойка и две единицы
3) выпадает три двойки и одна единица

И в первом, и в третьем случае у нас по 4 варианта: берём цифру, которая выпадает один раз и располагаем её на каждой из возможных 4 позиций. Например, в 1 случае:
4111 1411 1141 1114

Во втором случае рассуждаем так. Берём одну из цифр, выпадающих один раз, и располагаем её на каждой из возможных 4 позиций. Например:
3XXX
X3XX
XX3X
XXX3
Теперь для двойки в каждом из этих вариантов осталось всего по три возможных позиции. Итого у нас 3*4 = 12 вариантов.

В итоге имеем 4+12+4 = 20 возможных вариантов, при которых выпадает 7.

А нас интересует только один из них: 1411
Вероятность - 1/20.

А вы наоборот - нашли вероятность того, что в сумме выпадет 7 при условии, что во втором броске выпало 4.

4227. Леонид, 18 марта 2012, 18:07:19
Спасибо - понятно. Ещё просьба - нельзя ли удалить эти записи или эл. адр - я его не туда по ошибке вляпал. Будьте добры! Ещё раз спасибо за обстоятельную консультацию!

4228. Даниил, 19 марта 2012, 23:44:30
у меня ещё один вопрос: в задании
http://live.mephist.ru/show/mathege2010/view/id/28363/
величина h, я думаю, как и величина R должна измеряться в километрах, тогда в ответе будет 1280, а не 1.28, как отмечено в решении

4229. Светлана, 21 марта 2012, 17:09:13
Уважаемые организаторы и составители задач, сегодня, решая очередную порцию заданий по математике части B-10 я натолкнулась на явную ошибку в задаче под номером 282857. Очень прошу переправить ответ, либо условия задачи, чтобы совпали с ответом, т.к. правильный ответ на это задание 0.92 а у Вас правильным ответом считается 0.93. Благодарю за внимание.

4230. Lexxus, 21 марта 2012, 18:13:05
4229. Светлана, 21 марта 2012, 17:09:13
Уважаемые организаторы и составители задач, сегодня, решая очередную порцию заданий по математике части B-10 я натолкнулась на явную ошибку в задаче под номером 282857.

Сегодня, просматривая очередную порцию комментариев, я натолкнулся на очевидное нежелание подумать или хотя бы почитать с десяток предыдущих комментариев, прежде чем очертя голову набивать свой.

4231. Павел, 21 марта 2012, 21:26:36
1. Основанием прямой призмы ABCDA1B1C1D1 является параллелограмм ABCD со сторонами 6 см и 12 см и углом 60&#61616;. Диагональ B1D призмы образует с плоскостью основания угол в 30&#61616;. Найдите площадь полной поверхности призмы.
2. Сторона основания правильной треугольной пирамиды равна 3 см, а угол между боковой гранью и основанием равен 45&#61616;. Найдите площадь полной поверхности пирамиды.
помогите решить

4232. Павел, 21 марта 2012, 21:28:38
1. Основанием прямой призмы ABCDA1B1C1D1 является параллелограмм ABCD со сторонами 6 см и 12 см и углом 60градусов. Диагональ B1D призмы образует с плоскостью основания угол в 30градусов. Найдите площадь полной поверхности призмы.
2. Сторона основания правильной треугольной пирамиды равна 3 см, а угол между боковой гранью и основанием равен 45градусов. Найдите площадь полной поверхности пирамиды.

4233. Светлана, 22 марта 2012, 06:57:14
[/quote]
Сегодня, просматривая очередную порцию комментариев, я натолкнулся на очевидное нежелание подумать или хотя бы почитать с десяток предыдущих комментариев, прежде чем очертя голову набивать свой.[/quote]
Во первых можно было ответить без сарказма, а во вторых у преподавателя математики возникает встречный вопрос,как при делении 92 на 100 у Вас получается 0.93?

4234. Lexxus, 22 марта 2012, 08:51:40
4233. Светлана, 22 марта 2012, 06:57:14
у преподавателя математики возникает встречный вопрос,как при делении 92 на 100 у Вас получается 0.93?

При делении 92 на 100 у меня получается 0.92.
А правильный ответ в задаче №282857, тем не менее, 0.93.

Объясняю популярно специально для преподавателя математики:

Фабрика выпускает сумки. В среднем на 1 (ОДНУ) качественную сумку приходится 1 (ОДНА) сумка со скрытыми дефектами. Найдите вероятность того, что купленная сумка окажется качественной.

Преподаватель математики авторитетно заявляет, что ответ в этой задаче будет (1-1)/1 = 0.

Но логика обычного здорового человека отчаянно кричит, что тут что-то не так. И рано или поздно он таки догадывается, что, несмотря на непререкаемый авторитет преподавателя математики, правильный ответ всё-таки будет 1/(1+1) = 0.5.

4235. Светлана, 30 марта 2012, 11:19:44
Благодарю за ответ, теперь всё встало на свои места, извините за предоставленные неудобства.

4236. Владислав, 30 марта 2012, 14:02:40
Мне кажется, или с ответами на часть В не все впорядке?
К примеру, Задание B10 (283645) : решая, делим 116/130=0,8923
Округляя до сотых, получаем 0.89, хотя правильный ответ 0,9
Возможно обьяснение связано с комментарием 4234. Lexxus'a, но я его не понял, прошу обьяснить доступней на более крупных числах.

4237. себастьян, 30 марта 2012, 16:58:40
все перепробовал, не могу решить систему из с3, помогите

система: (1) 4-х меньше *корень из*16+6х-(х в квадрате)
(2) 3-х/*корень из*х-1 больше или равно -1

4238. Светик, 31 марта 2012, 16:55:25
Ребята, помогите пожалуйста!Буду очень вам благодарна!
Из вершины G треугольника GHP проведен перпендикуляр GQ. Из точки Q опущен перпендикуляр на сторону HP. Найдите условие, при котором этот перпендикуляр пройдет через одну из вершин H или P треугольника.

Из вершины угла к его плоскости проведена наклонная, которая составляет со сторонами угла равные углы. Докажите, что ортогональной проекцией этой наклонной является биссектриса данного угла.

4239. евгений, 1 апреля 2012, 17:17:22
Стороны треугольника равны 25 29 36 из вершины большего угла проведен перпендикуляр к его плоскости равный 21. Вычислить расстояние от его концов до большой стороны

4240. Виктория, 2 апреля 2012, 16:32:36
6 класс №1197и 1200

4241. Ня, 3 апреля 2012, 19:29:39
помогите решить,пожалуйста!Срочно нужно!

1. Через вершину K треугольника MKP проведена прямая KN, перпендикулярная плоскости треугольника. Известно, что KN = 15 см, MK = KP = 10 см. Найдите расстояние от точки N до прямой MP.
2. Дан прямоугольный параллелепипед ABCDA1B1C1D1,квадрат ABCD. AC = 6&#8730;2 см, AB1 = 4&#8730;3 см. Вычислите градусную меру двугранного угла B1ADB.

3.Основание тетраэда DABC-треугольник со сторонами 13 см,14см,15 см.расстояние от точки D со сторон треугольника равны 5 см.Найти расстояние от точки D до плоскости АВС.



4242. Ирина , 8 апреля 2012, 18:16:27
Помогите решить задания части С

4243. Вера, 8 апреля 2012, 20:59:03
Прямая является касательной к графику функции . Найдите b, учитывая, что абсцисса точки касания больше 0.
Пожалуйста разъясните решение задачи

4244. Зульфия, 11 апреля 2012, 13:36:16
В розетку элетросети подключены приборы,общее сопротивление которых составляет R1=63Ом.Параллельно с ними в розетку предрологатся подклбчить обогеватель.Определите наименьшее возмодное сопротивление R2'этого электрообогреватля,если известно что про параллельном соединение двух проводников с сопротивлениями R1Ом и R2 Ом их общее сопротивление дается формулой Rобщ=R1R2/ R1+R2, а для нормального функционирования электросети общее сопротивление в ней должно быть не меньше 36 Ом.Ответ выразите в омах.

4245. Дмитрий, 16 апреля 2012, 21:23:18
Грузовой автомобиль перевозит технику из одного города в другой,каждый день проезжая на одно и то же расстояние больше,чем в предыдущий день.известно,что за первый день грузовой автомобиль прошёл 520км.Определите,сколько километров проехал грузовой автомобиль за третий день,если весь путь он проехал за 5 дней и расстояние между городами составляет 3270км.

4246. Полина, 18 апреля 2012, 22:42:35
Внутри двугранного угла из точки M, принадлежащей его ребру, проведен к нему перпендикуляр, на котором отложен отрезок MN, в два раза больший своей ортогональной проекции на одну из граней двугранного угла. Найдите угол, который образует MN с другой гранью, если двугранный угол равен 100.
Помогите пожалуйста

4247. Диана, 21 апреля 2012, 17:06:35
1)Точка M находится на расстоянии h от плоскости A проведены две наклонные MP и MQ где P и Q основания наклонных , соответственно под углами 45 и 60 градусов. Наити: PQ если угол POQ равен 150 градусов , где О-основание перпендикуляра MO, MO перпендикулярно альфе
2)Плоскость квадрата ABCD со стороной a перпендикулярна плоскости равнобедренного треугольника BCM с углом B - 120 градусов. Найдите S треугольника ADM.
Помогите пожалуйста решить

4248. гена, 26 апреля 2012, 15:59:54
Задание B8 (27496)
Не влезает график в окно....
Видно только три экстремума (нуля производной)

4249. Стелла, 27 апреля 2012, 08:45:43
В задании В3 (262205) просят найти площадь четырехугольника, а нарисован треугольник!!!??? Внимательнее надо быть господа преподаватели.

4250. Аринчик, 27 апреля 2012, 18:50:13
3764. ЮЛЯ, 6 июня 2011, 10:49:19
Из пункта А в пункт В расстояние между которыми 50 км одновременно выехали мотоциклист и велосепедист. Известно что за час мотоцикл проезжает на 30км больше чем вело. определите скорость вело если известно что он пробыл в пункт В на 1,5 часа позже мото

ПОЖ-ТА ПОЛМОГИТЕ РЕШИТЬ ЕГЭ
3764. ЮЛЯ, 6 июня 2011, 10:49:19
Из пункта А в пункт В расстояние между которыми 50 км одновременно выехали мотоциклист и велосепедист. Известно что за час мотоцикл проезжает на 30км больше чем вело. определите скорость вело если известно что он пробыл в пункт В на 1,5 часа позже мото

ПОЖ-ТА ПОЛМОГИТЕ РЕШИТЬ ЕГЭ
3764. ЮЛЯ, 6 июня 2011, 10:49:19
Из пункта А в пункт В расстояние между которыми 50 км одновременно выехали мотоциклист и велосепедист. Известно что за час мотоцикл проезжает на 30км больше чем вело. определите скорость вело если известно что он пробыл в пункт В на 1,5 часа позже мото

ПОЖ-ТА ПОЛМОГИТЕ РЕШИТЬ ЕГЭ
3759. Лиля, 6 июня 2011, 09:52:57
Помогите плиз решить задачу срочно с ЕГЭ!!!!! в правильной 6-тиугольной призме АВСДЕFА1В1С1Д1Е1F1 стороны по 5, боковые по 11. найти расстояние от точки А до Е1Д1
3759. Лиля, 6 июня 2011, 09:52:57
Помогите плиз решить задачу срочно с ЕГЭ!!!!! в правильной 6-тиугольной призме АВСДЕFА1В1С1Д1Е1F1 стороны по 5, боковые по 11. найти расстояние от точки А до Е1Д1
3759. Лиля, 6 июня 2011, 09:52:57
Помогите плиз решить задачу срочно с ЕГЭ!!!!! в правильной 6-тиугольной призме АВСДЕFА1В1С1Д1Е1F1 стороны по 5, боковые по 11. найти расстояние от точки А до Е1Д1
3759. Лиля, 6 июня 2011, 09:52:57
Помогите плиз решить задачу срочно с ЕГЭ!!!!! в правильной 6-тиугольной призме АВСДЕFА1В1С1Д1Е1F1 стороны по 5, боковые по 11. найти расстояние от точки А до Е1Д1
3759. Лиля, 6 июня 2011, 09:52:57
Помогите плиз решить задачу срочно с ЕГЭ!!!!! в правильной 6-тиугольной призме АВСДЕFА1В1С1Д1Е1F1 стороны по 5, боковые по 11. найти расстояние от точки А до Е1Д1
3759. Лиля, 6 июня 2011, 09:52:57
Помогите плиз решить задачу срочно с ЕГЭ!!!!! в правильной 6-тиугольной призме АВСДЕFА1В1С1Д1Е1F1 стороны по 5, боковые по 11. найти расстояние от точки А до Е1Д1

4251. Алена Ефремова, 27 апреля 2012, 22:48:49
основанием прямой призмы ABCDA1B1C1D1 является параллелограмм со сторонами 4 см и 4sqrt3( 4 корней из 3) и углом, равным 30 градусам. Диагональ AC1 призмы образует с плоскостью основания угол 60 градусов . Найдите площадь боковой поверхности призмы.

Помогите пожалуйста. Срочно!

4252. РамонЛоко, 1 мая 2012, 11:40:38
10/(sin(-33пи/4) * cos(21пи/4))

4253. Анжела, 1 мая 2012, 21:16:44
На счету Катиного мобильного телефона было 80 рублей, а после разговора с Андреем осталось 45 рублей. Сколько минут длился разговор с Андреем, если одна минута разговора стоит 2 рубля 50 копеек.

4254. Ленин, 2 мая 2012, 14:39:02
В параллелепипеде АВСТА1В1С1Т1 с рёбрами АВ = 10 , АТ = 24 и АА1 = найдите расстояние от вершины С до плоскости ВТА1 .
рисунок????

4255. Amarilllis, 7 мая 2012, 14:03:40
Доброго времени суток.
Если я ничего не путаю, то некоторые "решённые" задания в этой базе имеют неверные ответы. Вот B5: sqrt(4 + 3x) = x (http://live.mephist.ru/show/mathege2010/view/id/12789/), в качестве правильного ответа выбрано число -1. А между тем, в Викиучебнике (http://ru.wikibooks.org/wiki/%D0%98%D1%80%D1%80%D0%B0%D1%86%D0%B8%D0%BE%D0%BD%D0%B0%D0%BB%D1%8C%D0%BD%D1%8B%D0%B5_%D1%83%D1%80%D0%B0%D0%B2%D0%BD%D0%B5%D0%BD%D0%B8%D1%8F) написано: "Заметим, что уравнение sqrt(f(x)) = g(x) может иметь решение для g(x) >= 0, но не имеет решений, если g(x) < 0. " Да и в школьной программе что-то было про арифметический корень.
Если это действительно ошибка, то исправьте, пожалуйста.

И да, огромный респект авторам за эту базу. Очень полезная и приятная штуковина для подготовки к ЕГЭ. Спасибо! :)

4256. сашка, 9 мая 2012, 22:17:00
3992. ксения, 8 ноября 2011, 22:11:51
Двое играют в кости. Они по разу бросают игральный кубик. Выигрывает тот, у кого больше очков. Если выпадет поровну, то ничья. Первый бросил кубик, и у него выпало 4. Найти вероятность того, что он выиграет.

4257. Старцев Сергей, 10 мая 2012, 18:07:19
Здравствуйте! Проверьте правильный ответ в задаче В3 (22847)

4258. Дианочка, 10 мая 2012, 22:21:13
помогите решить задачу очень срочно. диагональ прямоугольного параллелепипеда равна 12 и наклонена к плоскости его грани под углом 30г. найдите ребро параллелепипеда, перпендикулярное плоскости этой грани.

4259. ЮРИЙ, 13 мая 2012, 12:55:01
Найти площадь прямоугольника, стороны которого равны ( под корнем 5 )+1 и ( под корнем 5 )-1

4260. Кристина, 18 мая 2012, 18:53:31
Подскажите пожалуйста как решить такую задачу: объем правильной треугольной призмы 250 кубических сантиметров. какова должна быть сторона основания чтобы полная поверхность призмы была наименьшей? Спасибо большое заранее за ответ!

4261. vladimir, 19 мая 2012, 16:03:39
Что значит: ... сумма точек экстремума функции f(x) ?

4262. Kattu22, 20 мая 2012, 17:47:17
Lexxus! Я понимаю, что это не по теме, но я поздравляю Вас с днем рождения. Желаю Вам здоровья, благополучия , а также процветания Вашему сайту, это самый удобный сайт для подготовки к ЕГЭ по математике из всех мною найденных. Продолжайте в том же духе.

4263. Студент, 24 мая 2012, 07:35:43
два велосепедиста одновременно отправляются в 96 километровый пробег первый едет со скоростью на 4кмч больше чем второй и приибывает к финишу вторым ответ дайте в кмч

4264. Студент, 24 мая 2012, 07:37:35
тетрадь стоит 20 рублей. какое наибольшее число таких тетрадей можно будет купить на 350 рублей после понижения цены на 15%

4265. Лера, 26 мая 2012, 11:16:19
найти синус если косинус равен корень из 19 делить на 10,промежуток 3пи на 2 и 2пи.

4266. Саня, 29 мая 2012, 10:37:51
Помогите решить задачу срочно! Основание пирамиды ABCD- равнобедренный треугольник ABC, в котором AB=BC=26? AC = 48. Ребро BD перпендикулярно плоскости основания и равно 40. Найдите тангенс двухгранного угла при ребре АС. Зарание СПС

4267. саша, 29 мая 2012, 10:38:57
4266. Саня, 29 мая 2012, 10:37:51
Помогите решить задачу срочно! Основание пирамиды ABCD- равнобедренный треугольник ABC, в котором AB=BC=26? AC = 48. Ребро BD перпендикулярно плоскости основания и равно 40. Найдите тангенс двухгранного угла при ребре АС. Зарание СПС

4268. Дурак, 1 июня 2012, 07:51:45
Радиус основания цилиндра равен 4см , высота в 2 раза больше длины окружности основания . Найдите объем


4269. Просто Юлька я, 1 июня 2012, 10:18:36
Скажите мне пожалуйста как отсюда удалится?????????????????????????????????????????????????????????????????????????????????????????77

4270. виктория, 1 июня 2012, 21:37:16
диагональ осевого сечения цилиндра равна 8 см и наклонена к основанию по углом 30 градусов. найти площадь полной поверхности цилиндра.
помогите решить пожайлуств=)

4271. виктория, 1 июня 2012, 21:43:39
равнобочная трапеция с основаниями 12 и 18 см и высотой 4 см вращается около большого основания. найти объем тела вращения. помогите решить пожайлуста =)

4272. виктория, 1 июня 2012, 21:49:52
в полусферу радиуса 5 см вписан цилиндр с диаметром основания 6см. найти высоту цилиндра. помогите решить пожайлуста=)

4273. виктория, 1 июня 2012, 21:55:20
сторона основания правильной шестиугольной призмы равна a, наибольшая диагональ призмы наклонена к плоскости основания под углом J. найдите полную поверхность и объем призмы. помогите решить пожайлуста =)

4274. Глупышка, 3 июня 2012, 14:29:34
основание тетраэдра DABC- треугольник со сторонами 13см,14см,15см. Расстояние от точки D со сторон треугольника равны 5см. Найти расстояние от точки D до плоскости ABC

4275. 123, 3 июня 2012, 17:09:27
Проекция вершины тетраэдра Н будет центром вписанной окружности т.к расстояния до сторон одинаковы. Найдём r(abc) через площадь

Sabc=sqrt(21*8*7*6)=sqrt(4^2*7^2*3^2)=84
Sabc=21*r=84 => r=4
DH(расстояние)=sqrt(25-16)=3

4276. насима, 3 июня 2012, 20:13:46
здравствуйте! скажите пожалуйста разрешается ли решать С2 используя матрицы ведь в школе не проходят?

4277. 123, 4 июня 2012, 14:42:02
4276. насима, 3 июня 2012, 20:13:46
здравствуйте! скажите пожалуйста разрешается ли решать С2 используя матрицы ведь в школе не проходят?


Задания ЕГЭ можно решать любыми способами, главное чтобы правильно было

4278. Алексей, 4 июня 2012, 19:44:22
Задание B10 (283647)
В нем ошибка.Неверный ответ

4279. Тимур, 4 июня 2012, 21:34:14
a b c d - прямоугольный параллелепипед, AB=4 см, AD=8 см, AA1=12см, найдите длину отрезка KM, где точки K и M принадлежат соответственно отрезку CC1 и BD,причем CK:KC=2:1 и BM:MD=3:1

4280. Аня, 5 июня 2012, 20:58:21
решите неравенство пожалуйста у=под корнем все х-6/х-1

4281. Иринка, 6 июня 2012, 20:12:15
Привет! Помогите,пожалуйста, решить С3:
9 в степени ( (х+5)^2 *(x-7) ) умножить на 1/2 в степени ( корень квадратный из (х^2 -2x-35)^3 ) <=1. Заранее спасибо!

4282. Вилена, 7 июня 2012, 08:46:28
C3. В правильной трехугольной призме ABCA1B1C1 стороны основания равны2,боковые ребра равны3,точка D середина ребра CC1. Найдите расстояние от вершины C плоскости ADB1.

4283. Павел, 7 июня 2012, 09:48:41
Задание B1 (26641)
В школьную библиотеку привезли новые учебники по русскому языку для 5 – 9 классов, по 80 штук для каждой параллели. Все книги одинаковы по размеру. В книжном шкафу 5 полок, на каждой полке помещается 25 учебников. Сколько шкафов можно полностью заполнить новыми учебниками?

Мне кажется, что ответ не верный, ведь справшивается: "Сколько шкафов можно полностью заполнить новыми учебниками", полностью можно заполнить лишь 2 шкафа и еще останутся учебники, а в ответе 3.

4284. настёна, 7 июня 2012, 10:06:35
есть кто онлайн??) помощь нужна

4285. лариса, 7 июня 2012, 11:15:42
10:31
заказ на 260 деталей первый рабочий выполняет на 7часов быстрее чем второй сколько деталей в час делает второй рабочий если известно что первый за час делает на 7деталей больше

4286. Рома, 7 июня 2012, 11:24:45
помогите пжлста

В цилиндрическом сосуде уровень жидкости достигает 216см. На какой высоте будет находиться уровень жидкости, если ее перелить во второй цилиндрический сосуд, диаметр которого в 6 раз больше диаметра первого?
ответ в сантиметрах

очень на вас надеюсь!!

4287. Леонид, 8 июня 2012, 21:58:59
Уважаемый LEXXUS! Как показать на чертеже требуемое расстояние? Задача С ЕГЭ этого года. ПОЖАЛУЙСТА!!! Не применяя векторов.
В правильной трехугольной призме ABCA1B1C1 стороны основания равны2,боковые ребра равны3,точка D середина ребра CC1. Найдите расстояние от вершины C плоскости ADB1.

4288. Lexxus, 8 июня 2012, 23:52:01
4287. Леонид, 8 июня 2012, 21:58:59
Уважаемый LEXXUS! Как показать на чертеже требуемое расстояние? Задача С ЕГЭ этого года. ПОЖАЛУЙСТА!!! Не применяя векторов.
В правильной трехугольной призме ABCA1B1C1 стороны основания равны2,боковые ребра равны3,точка D середина ребра CC1. Найдите расстояние от вершины C плоскости ADB1.

Не совсем понимаю, зачем это нужно, ибо через уравнение плоскости (думаю, именно это вы называете "с векторами") задача решается почти в одно действие, но извольте.

1. Продлеваем B1D и BC до пересечения в точке E. Как можно заметить, AC=EC, а значит, и DE=AD. Дальше всё совсем просто.
2. Из точки C проведём перпендикуляр CF к AE.
3. Из точки D проведём перпендикуляр DF к AE.
4. Высота CG прямоугольного треугольника DCE и будет искомым расстоянием от точки C до плоскости ADB1.

В ответе должно получиться 3/sqrt(13)

4289. Леонид, 9 июня 2012, 06:40:56
Спасибо большое ! (да через ур-е плоскости я назвал "с векторами"). Ваше решение (традиционное, скажем так) необходимо - не везде "активно" изучается материал "об уравнении". Ещё раз большое СПАСИБО!

4290. Витя, 10 июня 2012, 13:32:46
Уважаемый Лексус!
По поводу заметки 4288.
У вас, наверное ошибка, "Высота CG прямоугольного треугольника DCE" надо переправить на "Высота CG прямоугольного треугольника DCF."
И все равно мне непонятно почему СG является перпендикуляром к плоскости AB1D. Не могу никак определить две прмые в плоскости AB1D которым перпендикулярно СG.( я пока в 10 кл. Знаю теорему о перп. прямой и плоскости-если прямая перп.2-м пр. в плоскости , то она перп. плоск.).
Пожалуйста, объясните мне. Да уверен и не только мне.
А вот вопрос: я слышал про эти методы решения таких задач с помощью координат, векторов, и т.д. Но уравнения плоскости мы изучать не будем(так наша уч.матем.) сказала, это не программный материал. А если я сам изучу? При проверке не придерутся?

4291. юрий, 14 июня 2012, 08:47:01
помогите плиз )))
высота правильной пирамиды 20 см а боковое ребро 16 см.Найти площадь боковой поверхности

4292. Света, 3 августа 2012, 16:31:26
чему равен котангенс 2 альфа, если косинус альфа равен минус 3/5 альфа Е(пи/2 умножить на пи). Помогите пожалуйста! Почта моя: Soflizka@list.ru

4293. Света, 3 августа 2012, 16:33:53
сумма корней уравнения (4-х^2)корень из -3х-1=0

4294. маша, 27 августа 2012, 12:06:41
на экзамене 40 билетов, жима не выучил 6 из них. найдите вероятность того, что ему попадётся выученный билет... ( помогите пожалуйсто решить и покажите не только ответ но и решение))) ПОЖАЛУЙСТО!!!!!!!!!

4295. alfa20, 27 августа 2012, 16:18:48
4294. маша, 27 августа 2012, 12:06:41
на экзамене 40 билетов, жима не выучил 6 из них. найдите вероятность того, что ему попадётся выученный билет... ( помогите пожалуйсто решить и покажите не только ответ но и решение))) ПОЖАЛУЙСТО!!!!!!!!!


1)40-6=34 билета выучил
2)Р=34:40=0,85 искомая вероятность

4296. Эмма, 5 сентября 2012, 21:11:09
1.определите количество корней уравнения sin2x=sinx , принадлежащих интервалу (-3;3)
2.решите уравнение (tgx+1)(2sinx/2-корень из 2)=0.В ответ запишите отношение наименьшего положительного корня уравнения к числу пи.
3.найдите количество точек на отрезке [0;2pi], в которых функция y=1/tgx-1 не определена.
4.найдите отношение наименьшего по модулю корня уравнения sin^2 x=3sinxcosx-2cos^2 x к числу pi.


помогите решить пожалуйста это оочень срочно,я не понимаю как решать 3 и 4, а у 1 и 2 нужно узнать ответ,заранее спасибо!

4297. леонид, 20 сентября 2012, 18:33:40
4292 напечатано чёрти что! Откуда берёте задания - 4296? Никакие официальные материалы подобных заданий - с накрутками в условиях таких не содержат. Вам что, решать нечего к егэ?

4298. Витя, 24 сентября 2012, 19:35:47
Уважаемый Лексус!
Тут нашему классу недавно дали задачу из Типовых тестовых заданий авторов Семенов Ященко,стр. 19 №С2
Вот эту:основание прямой четырехугольной призмы АВСДА1В1С1Д1- прямоугольник АВСД, в котором АВ=5, АД=sqrt33. Найти tg угла между плоскостью грани АА1Д1Д призмы и плоскостью, проходящей через середину ребра СД перпендикулярно прямой В1Д, если расстояние между прямыми А1С1 и ВД равно sqrt 3.
Попробовал сам решить. Не получилось, стал искать в гугле и нашел Ваше решение.
Не сразу его понял, но разобрался.
А вопрос - предложение у меня такое: нельзя ли сделать поиск таких решенных задач С1,С2,.... на Вашем сайте? Чтоб можно было без гугля?
Если это не трудно конечно.
Спасибо.

4299. Lexxus, 24 сентября 2012, 19:56:09
4298. Витя, 24 сентября 2012, 19:35:47
А вопрос - предложение у меня такое: нельзя ли сделать поиск таких решенных задач С1,С2,.... на Вашем сайте? Чтоб можно было без гугля?
Если это не трудно конечно.
Спасибо.

Есть тут такой поиск:
Основание прямой четырехугольной призмы

4300. александра, 25 сентября 2012, 19:54:48
пожалуйста помогите решить уравнение
cos2x + 3sin(во 2 степени)x = 1,25
а потом найти корни этого уравнения, принадлежащие отрезку [п, 5п/2]
заранее спасибо !

4301. александра, 25 сентября 2012, 19:59:22
в правильной четырехугольной призме ABCDA1B1C1D1 стороны основания равны 1,а боковые ребра равны 3. На ребре AA1 отмечена точка Е так, что АЕ : ЕА = 2 : 1. Найти угол между плоскостями АВС и BED1

4302. Lexxus, 25 сентября 2012, 21:07:28
4300. александра, 25 сентября 2012, 19:54:48
cos(2x) + 3sin^2(x) = 1,25
а потом найти корни этого уравнения, принадлежащие отрезку [п, 5п/2]


cos(2x) + 3sin^2(x) = 1,25
По формуле косинуса двойного угла преобразуем первое слагаемое:
cos^2(x) - sin^2(x) + 3sin^2(x) = 1.25
cos^2(x) + 2sin^2(x) = 1.25
Используя основное тригонометрическое тождество, получим:
1 + sin^2(x) = 1.25
sin^2(x) = 0.25

sin(x) = (+/-)1/2

Корни уравнения sin(x) = 1/2: x = ((-1)^n)*pi/6 + pi*n
Корни уравнения sin(x) = -1/2: x = ((-1)^(n+1))*pi/6 + pi*n
А и то, и другое сразу можно записать проще:
x = (+/-)pi/6 + pi*n



Осталось только выяснить, какие из корней попадают в указанный промежуток. Если построить единичную окружность, всё хорошо видно.

Ответ: pi+pi/6, 2*pi-pi/6, 2*pi+pi/6
Ну, или: 7pi/6, 11pi/6, 13pi/6

4303. александра, 25 сентября 2012, 21:16:13
Спасибо !

4304. александра, 25 сентября 2012, 21:33:37
помогите пожалуйста еще в решении следующей задаче
в треугольнику АВС известны стороны АВ=14, ВС+18, АС=20. окружность, проходящая через точки А и С, пересекает прямые ВА и ВС соответственно в точках K и L, отличных от вершин треугольника.отрезок KL касается окружности, вписанной в треугольник АВС. Найти нужно длину отрезка KL.

4305. александра, 25 сентября 2012, 23:23:32
пожалуйста, рассмотрите 4301 и 4304

4306. Lexxus, 25 сентября 2012, 23:33:23
4305. александра, 25 сентября 2012, 23:23:32
пожалуйста, рассмотрите 4301 и 4304

Как только будет время. Увы, его всегда не хватает.

В С2 ответ arctg(sqrt(5)). Завтра, если не забуду, распишу решение.
С4 не смотрел ещё.

4307. александра, 26 сентября 2012, 00:33:10
и на том спасибо за помощь !

4308. erkenaz, 27 сентября 2012, 23:25:44
Гипотенуза прямоугольного треугольника лежит на
прямой 2х + у — 2 = 0, а точка C(3; —1) является вершиной пря-
мого угла. Площадь треугольника равна 9/4. Составить урав-
нения прямых, на которых лежат катеты.

4309. еркеназ, 27 сентября 2012, 23:28:26
привееееееееет друзья помогитеееееееееееееееееее

Гипотенуза прямоугольного треугольника лежит на
прямой 2х + у — 2 = 0, а точка C(3; —1) является вершиной пря-
мого угла. Площадь треугольника равна 9/4. Составить урав-
нения прямых, на которых лежат катеты.

4310. Витя, 29 сентября 2012, 20:10:44
Уважаемый Lexxus!
У меня к Вам очень важный вопрос по теории вероятности. Наша уч. матемтики решила ее с одним ответом а потом я нашел в инете другой ответ. Может Вы рассудите кто прав?
Задача из диагн. работы 11 кл от 25 сентября .
Стрелок стреляет по мишени 1 раз. В случае промаха стрелок делает 2-ой выстрел. Вероятность попадания в мишень при одном выстреле равна 0,7.Найти вероятность того, что мишень будет поражена(либо первым либо вторым выстрелом).
Учительница: используя формулу Бернулли п=ц из н по к умножить на п в степени к умножить на ку в степени 1-к получаем: 2*0,7*0,3=0,42
Сайт А. Ларина( там просто непонятные вычисления): 0,7+0,3*0,7=0,91
или еще : 1 - 0,3*0,3=0,91
И по моему второй ответ реальнее.
Как же тут считать?
Если нет времени, как Вы пишете, то просто напишите какой отв. правильнее.
Спасибо!

4311. Lexxus, 30 сентября 2012, 03:52:45
4310. Витя, 29 сентября 2012, 20:10:44
Стрелок стреляет по мишени 1 раз. В случае промаха стрелок делает 2-ой выстрел. Вероятность попадания в мишень при одном выстреле равна 0,7.Найти вероятность того, что мишень будет поражена(либо первым либо вторым выстрелом).

Не "либо первым, либо вторым выстрелом", а просто поражена. Не важно, первым, вторым или обоими.

4310. Витя, 29 сентября 2012, 20:10:44
Учительница: используя формулу Бернулли п=ц из н по к умножить на п в степени к умножить на ку в степени 1-к получаем: 2*0,7*0,3=0,42

О, господи... И откуда таких берут...
Ну хоть немножко-то соображать надо? Как может быть вероятность при двух попытках меньше, чем при одной?!

4310. Витя, 29 сентября 2012, 20:10:44
Сайт А. Ларина( там просто непонятные вычисления): 0,7+0,3*0,7=0,91
или еще : 1 - 0,3*0,3=0,91
И по моему второй ответ реальнее.

Правильно мыслишь.

Тут логика такая: посчитаем вероятность того, что мишень НЕ будет поражена, то есть стрелок оба раза промахнётся. Это просто: (1-0.7)*(1-0.7) = 0.09.
Соответственно, вероятность того, что этого не произойдёт, будет 1-0.09=0.91.

Можно и иначе: нам нужно вычислить вероятность того, что стрелок:
(поразит первую мишень И промахнётся по второй) ИЛИ (промахнётся по первой И поразит вторую) ИЛИ (поразит первую И поразит вторую)

Заменим в этой фразе события на соответствующие вероятности, "И" на "умножить", а "ИЛИ" на "сложить". Получим:

0.7*0.3 + 0.3*0.7 + 0.7*0.7 = 0.91

4312. Витя, 30 сентября 2012, 10:46:28
Спасибо,Lexxus!

4313. Кайрат, 2 октября 2012, 19:52:24
В прямоугольной призме стороны основания равны 3,4и5 см, а полная поверхость равна 84 квадратных см. Определите боковую поверхность призмы и ее высоту

4314. Кайрат, 2 октября 2012, 20:02:15
Сколько равных между собой диагоналей может иметь наклонный параллелепипед? ответ объясните. Кто нибудь решите эти 2 задачи пожалйста

4315. ДИМА, 7 октября 2012, 14:17:39
помогиите пару примеров решить, умные люди

sin(Pi*lgx) + cos (Pi+lgx) = 1

2^cos2x = 3 * 2^(cos^2x) - 4

4316. Ирина, 9 октября 2012, 15:09:52
Помогите решить:
между какими соседними целыми числами заключено значение выражения:
1/(&#8730;3 +1)+1/(&#8730;5+&#8730;3)+1/(&#8730;7+&#8730;5)+…+1/(&#8730;21+&#8730;19)

4317. Валерия, 18 октября 2012, 16:35:57
АВСДА1В1С1Д1-параллелепипед. М точка пересечения ДС1 и Д 1С. АВ=а,АД=в,АА1=с. Разложите вектор Ам по векторам а,в,с
Помогите с решением

4318. Дарья, 21 октября 2012, 13:07:04
В треугольнике ABC биссектриса AH делит сторону BC на отрезки, длины которых равны 28 и 12. Найдите периметр треугольника ABC, если AB – AC = 18.

4319. Анатолий, 21 октября 2012, 13:11:05
Часто особые проблемы доставляют отдельные разделы школьной математики. У одних это начала анализа, у других тригонометрия и т.д. Именно в этих разделах надо бы тренироваться в первую очередь. Хорошо бы, если бы был поиск и по разделам математики. По словам или номерам заданий это не то.
Это возможно

4320. яяяяяяяяяяяя, 29 октября 2012, 08:03:58
пожалуйста решите задачу 4143..............прошу вас срочно

4321. Lexxus, 29 октября 2012, 19:18:09
4320. яяяяяяяяяяяя, 29 октября 2012, 08:03:58
пожалуйста решите задачу 4143..............прошу вас срочно

Хм. Девять месяцев ожидания - это серьёзно.

4143. Neon, 23 января 2012, 22:24:09
1. Основание прямого параллелепипеда - ромб со стороной 8 см, острый угол которого равен 60 градусов. Найдите длину меньшей диагонали параллелепипеда, если его высота равна 15 см.

Если острый угол ромба равен 60 градусов, то его меньшая диагональ равна его стороне (8 см).
Меньшая диагональ параллелепипеда - это гипотенуза прямоугольного треугольника, один из катетов которого - меньшая диагональ основания (8 см), а другой - высота (15 см).
sqrt(8^2+15^2) = 17
Ответ: 17 см

4143. Neon, 23 января 2012, 22:24:09
2. Диагональ прямоугольного параллелепипеда равна 7 см, а диагональ его боковой грани равна 5 см. Найти высоту параллелепипеда, если в его основании лежит квадрат.

Сначала найдём сторону основания как катет прямоугольного треугольника, гипотенуза которого - диагональ параллелепипеда (7 см), а второй катет - диагональ его боковой грани (5 см).
Получится sqrt(7^2-5^2). Число некрасивое, но это только промежуточный результат, ничего с ним делать не будем.
Высоту параллелепипеда найдём как катет другого прямоугольного треугольника, гипотенуза которого - диагональ боковой грани параллелепипеда (5 см), а второй катет - найденная нами сторона основания.

В итоге имеем sqrt(5^2-(sqrt(7^2-5^2))^2) = sqrt(5^2-7^2+5^2) = sqrt(50-49) = 1
Ответ: 1 см

4322. марина, 30 октября 2012, 07:31:05
решите прямоугольный треугольник, если:
1. а=63; альфа=46градусов24минуты
2.с=100; бетта=16нрадусов16минут
3. а=72; в=30
4. в=55; альфа=41градусов7минут

4323. помогите, 30 октября 2012, 07:45:01
стороны основания параллелепипеда 6см и 8см,а высота 12 см. найдите полную поверхность параллелепипеда

4324. по, 30 октября 2012, 07:47:05
пожалуйста решите мне срочно очень надо

4325. Lexxus, 30 октября 2012, 17:18:41
4323. помогите, 30 октября 2012, 07:45:01
стороны основания параллелепипеда 6см и 8см,а высота 12 см. найдите полную поверхность параллелепипеда

4324. по, 30 октября 2012, 07:47:05
пожалуйста решите мне срочно очень надо

Да не вопрос.
От 336 до бесконечности. Зависит от угла между сторонами основания и угла наклона боковых рёбер к плоскости основания.

4326. Kattu22, 31 октября 2012, 14:27:54
Lexxus! Исправьте, пожалуйста, ответ к задаче 7733. Там в ответ неправильный. Спасибо.

4327. Lexxus, 31 октября 2012, 14:44:31
4326. Kattu22, 31 октября 2012, 14:27:54
Lexxus! Исправьте, пожалуйста, ответ к задаче 7733. Там в ответ неправильный. Спасибо.

Исправлено, спасибо.

4328. Mariya , 1 ноября 2012, 17:40:45
Дан треугольник АВС .плоскость параллельная прямой ВС , пересекают сторону АВ этого треугольника в точке В1 . а сторону АС - в точке С1 . Найдите длину отрезка В1С1 , если АВ1=8см, ВВ=4 см . Вс=9 см .

4329. Яр, 6 ноября 2012, 18:29:40
Привет добавьте задачи B14 на сайт

4330. Виктор, 8 ноября 2012, 13:49:56
точки а в с д не приндлежат плоскости. точка м-середина отрезка ад. через прямую вм и середину отрезка сд провели плоскость альфа.докажите,что плоскость параллельна прямой ас.ПОМОГИТЕ ПОЖАЛУЙСТА

4331. Kattu22, 11 ноября 2012, 18:20:36
Lexxus! Исправьте, пожалуйста, ответ к задаче 206196! Спасибо.

4332. Lexxus, 12 ноября 2012, 01:12:07
4331. Kattu22, 11 ноября 2012, 18:20:36
Lexxus! Исправьте, пожалуйста, ответ к задаче 206196! Спасибо.

Исправлено, спасибо.

4333. Милена, 15 ноября 2012, 19:05:50
Пожалуйста,это срочно,завтра утром сдавать нужно,помогите:
1.Найдите высоту правильной шестиугольной пирамиды, если сторона ее основания = а, а апофема = l( с чертежом).
2.Найдите величину двугранного угла при основании правильной четырёхугольной пирамиды,если её боковые рёбра наклонены к плоскости основания под углом 60 градусов.(с чертежом)
3.Найдите высоту правильной треугольной пирамиды ,у которой боковая поверхность равна 60^3,а полная поверхность равна 108^3( ^- корень)(с чертежом).
Заранее спасибо)

4334. Алёнка, 20 ноября 2012, 18:12:31
Помогите пожалуйста написать уравнение касательной к функции Х в кубе в точке минус 1

4335. Вера Алексеевна, 29 ноября 2012, 06:08:45
В тренировочной работе №1 МИОО задание В8 ориентировано на интеграл. В открытом банке я не нашла ни одного подобного задания. Кто может подсказать где найти такие задания?

4336. Проффессор, 12 декабря 2012, 22:26:40
Я проффессор спрашивайте меня в контакте завут Проффессор Алексеев.

4337. я робот, 12 декабря 2012, 22:28:24
позвольте сказать вы дураки на этом всё.

;6455555375476528769759746№"3237%444;42№2#244школа №17654;%;5№40904892849284928код школы 100102бnmnKJhT6%56%45343;№;"4в школе денег 100миллионоврубл.$$$$$

4339. Анастасия, 14 декабря 2012, 14:10:22
помогите пожалуйста решить 7sin(в квадрате)x=8sinx-cosx(в квадрате)x

4340. таня, 18 декабря 2012, 14:27:37
две стороны треугольника равны 7 корень из 2 см и 10 см а угол между ними равен 45 .Найдите площадь треугольника .

4341. Sladkoeshka), 18 декабря 2012, 15:07:39
найдите величину двугранного угла при основании правильной четырехугольной пирамиды,если ее боковые ребра наклонены к плоскости основания под углом 60 градусов

4342. Батабекжан, 21 декабря 2012, 23:38:15
АВСDА1В1С1D1-куб,Е-середина СС1. Определите число сторон сечения плоскостью ,которая проходит через точки А, В1 и Е

4343. Алена, 24 декабря 2012, 23:02:03
найдите высоту пирамиды,в основании которой лежит равнобедренный треугольник с основанием 6 см и высотой 9 см, если каждое боковое ребро пирамиды наклонено к плоскости основания под углом 30%

4344. Александра, 27 декабря 2012, 15:46:41
Помоги решить!Пожалуйста!!
Объём правильной треугольной призмы равен V. Какова должна быть сторона основания, чтобы полная поверхность была наименьшей?

4345. Гэрэлма, 2 января 2013, 08:14:14
1217. Игорь, 28 апреля 2010, 22:54:47
Леониду и Михаилу на сообщение 1210-

Очень замечательно , что попалась такая задача , так как я прорешал прототипы такого типа :

"Из А в В одновременно выехали два автомобиля. Первый проехал с постоянной скоростью весь путь. Второй проехал первую половину пути со скоростью, меньшей скорости первого на 17 км/ч, а вторую половину пути - со скоростью 102 км/ч, в результате чего прибыл в В одновременно с первым автомобилем. Найдите скорость первого автомобиля, если известно, что она больше 60 км/ч. Ответ дайте в км/ч."

Она решается без квадратного уравнения путем рассуждения - если вы отставали на х км/ч на первой половине пути , то чтобы догнать к финишу нужно вторую половину ехать со скоростью +2х от скорости движущегося с постоянной скоростью Итого имеем отставание было 17 и после увеличения на 2*17 стала 102 значит скорость постояннодвижущегося 102-34= 68

Если решать по схеме Леонида то выходим на уравнение -х^2+119х-3468=0

как его решить без калькулятора я не представляю хотя корни будут 51 и 68[/q[quote]1290. регишаааа), 6 мая 2010, 13:30:17
я ваще не понимаю зачем придумали ЕГЭ!!!!вот я возьму и не пойду его сдавать!!!и ВАМ советую!!!
uote]

4346. Kattu22, 7 января 2013, 21:48:53
Lexxus! Открытый банк пополнился новыми заданиями. Может ли Вы их добавить?

4347. Леонид, 8 января 2013, 17:13:09
Уважаемый, Lexxus! Помогите, пожалуйста, в решении этих задач! Заранее благодарю!
1) Две фабрики выпускают одинаковые стекла для автомобильных фар. Первая фабрика выпускает 45 этих стекол, вторая –– 55 . Первая фабрика выпускает 3 бракованных стекол, а вторая –– 1 . Найдите вероятность того, что случайно купленное в магазине стекло окажется бракованным.
2) Если гроссмейстер А. играет белыми, то он выигрывает у гроссмейстера Б. с вероятностью 0,52. Если А. играет черными, то А. выигрывает у Б. с вероятностью 0,3. Гроссмейстеры А. и Б. играют две партии, причем во второй партии меняют цвет фигур. Найдите вероятность того, что А. выиграет оба раза.
3)7) В чемпионате мира учавствуют 16 команд. С помощью жребия их нужно разделить на четыре группы по четыре команды в каждой. В ящике вперемешку лежат карточки с номерами групп:1, 1, 1, 1, 2, 2, 2, 2, 3, 3, 3, 3, 4, 4, 4, 4. Капитаны команд тянут по одной карточке. Какова вероятность того, что команда России окажется во второй группе?
4)На экзамене по геометрии школьнику достаётся один вопрос из списка экзаменационных вопросов. Вероятность того, что это вопрос на тему «Вписанная окружность», равна 0,2. Вероятность того, что это вопрос на тему «Параллелограмм», равна 0,15. Вопросов, которые одновременно относятся к этим двум темам, нет. Найдите вероятность того, что на экзамене школьнику достанется вопрос по одной из этих двух тем.

4348. Леонид, 9 января 2013, 17:11:44
Здравствуйте! Все ещё на каникулах?

4349. макс, 10 января 2013, 13:04:55
вычислить двести шеснадцать минус одна третия умножить скобках одна шистая минус в дроби 2минус пятьв дроби один умножить вскобках одна двадцать пятая минус один и двав дроби ПАМОГИТИ ПОЖАЛУЙСТА ПРОШУ

4350. Леонид, 21 января 2013, 11:26:56
Доброго времени! Что же Вы не реагируете на возвания о помощи? (4347). И ещё!
ТОЛИ СНИТСЯ МНЕ, ТОЛИ КАЖЕТСЯ, ТОЛИ АВТОР егэ КУРАЖИТСЯ...? ВАШИ ДОБАВКИ С ЯНВАРЯ, НАПРИМЕР В5: 3^((LOG С ОСН 27 ЧИСЛА 6Х)+9 = 3. ЭТО ШУТКА - ВСЕ ЗАДАНИЯ ТАКОВЫ, ПОЧТИ, В ПРОТОТИПАХ! ЗНАНИЯ ЧЕГО ПРОВЕРЯЕТЕ? ИЛИ ПОКАЗАНА АКТИВНОСТЬ АВТОРОВ "В СОВЕРШЕНСТВОВАНИИ" БАНКА ЗАДАНИЙ - РАБОТАЕМ, МОЛ...

4351. Зара, 29 января 2013, 09:48:48
Lim x->3 4x В квадрате +11х-3/х в квадрате -9х-36 Вычислить приделы функций

4352. Natali, 29 января 2013, 10:01:59
Здравствуйте! Помогите, пожалуйста, решить уравнение
2^3+x=0,4*5^3+x

4353. игорь, 14 февраля 2013, 20:50:19
я только что зарегался. Тут задания для 9 класса или для 11 просто нигде не написано!

4354. оля, 18 февраля 2013, 13:00:05
Здравствуйте ребята,помогите решить задание из части В5 "Корень из 15-2х=3

4355. JoMo16, 19 февраля 2013, 20:14:34
В треугольнике ABC угол A' = 45 градусов, AB = 12 см, AC = 6,5 см. Найдите его площадь.

Помогите пожалуйста.

4356. Алёна, 24 февраля 2013, 11:23:32
1.определите количество корней уравнения sin2x=sinx , принадлежащих интервалу (-3;3)
2.решите уравнение (tgx+1)(2sinx/2-корень из 2)=0.В ответ запишите отношение наименьшего положительного корня уравнения к числу пи.
3.найдите количество точек на отрезке [0;2pi], в которых функция y=1/tgx-1 не определена.
4.найдите отношение наименьшего по модулю корня уравнения sin^2 x=3sinxcosx-2cos^2 x к числу pi.

Пожалуйста,помогите.

4357. Маришка, 25 февраля 2013, 15:25:38
Сторона правильного треугольника равна 26 корней из 3-х.Найдите радиус окружности,описанной около этого треугольника.

4358. катя)), 25 февраля 2013, 18:54:50
Теплоход проходит по течению реки до пункта назначения 323 км и после стоянки возврощается в пункт отправления.Найдите скорость теплохода в неподвижной воде,если скорость течения равна 1км/ч стоянка длится 2 часа,а в пункт отправления теплохожд возврашается через 38 часов после отплытия из него.Ответ дать в км/ч»

4359. Марианна, 26 февраля 2013, 18:06:15
В правильной треугольной пирамиде SABC ребра BA и BC разделены точками K и L, соответственно, в отношении 2:1, считая от вершины B. Найдите угол между плоскостью основания ABC и плоскостью сечения SKL. Ответ выразите в градусах

4360. Борис, 27 февраля 2013, 05:06:40
В задаче номер 273981 (В9) в условии наверняка опечатка - найти угол ADE. Надо так: найти угол ADE1.

4361. Андрей, 3 марта 2013, 16:22:53
Катя)), подобные задачи были описаны выше, но я все же повторю. За х принимаем скорость теплохода в стоячей воде. Составляем уравнение: время туда +стоянка +обратно = 38, немного упрощая получим
323/(х+1) + 323/(х-1) = 36. Решая уравнение по всей науке натыкаемся на квадратное уравнение с дискриминантом 106921 (это 325^2, корни получатся рациональные). Жаль, что на ЕГЭ нет калькулятора.
НО! уравнение решается подбором, предполагая, что х - целое число, (х+1) и (х-1) - делители 323, а сумма двух дробей =36. Магия этого примера в том, что 36=17+19, а 323=17*19. Отсюда х=18 км/ч.
Такое решение не строго (был потерян один корень -1/18), но эффективно

4362. Александр, 3 марта 2013, 21:30:37
Помогитеееее

В11
Объем правильной четырехугольной пирамиды равен 81. Высота разделена на 3 равные части, и через точки деления проведены плоскости параллельные основанию пирамиды. Найдите объем средней отсеченной части.

4363. аделя, 5 марта 2013, 20:33:00
ПОМОГИ ПОЖАЛУЙСТА!!!!!!!!Основанием пирамиды MABCD является квадрат ABCD, ребро MD перпендикулярно к плоскости основания, AD = DM = a. Найдите площадь поверхности пирамиды.

4364. Анюта, 6 марта 2013, 22:12:24
Сколькими способами можно составить список из 25 студентов?

4365. Анюта, 6 марта 2013, 22:14:42
В пассажирском поезде 14 вагонов. Сколькими способами распределить 14 проводников, если за каждым вагоном закрепляется один проводник

4366. Анюта, 6 марта 2013, 22:16:44
5) Брошена игральная кость. Найти вероятность того, что выпадет нечетное число очков.

4367. AleX@nder, 7 марта 2013, 17:40:08
4364. Анюта, 6 марта 2013, 22:12:24
Сколькими способами можно составить список из 25 студентов?

Ну по-видимому 25-факториал-способами.

Сколько существует перестановок n предметов? n! (в смысле эн-факториал) :)

То есть надо посчитать 1 * 2 * 3 * ... * 20 * 21 * 22 * 23 * 24 * 25
Считать неохота :D

4365. Анюта, 6 марта 2013, 22:14:42
В пассажирском поезде 14 вагонов. Сколькими способами распределить 14 проводников, если за каждым вагоном закрепляется один проводник
Гмммм... Похоже то же самое - произведение всех чисел от одного до 14 - 14-факториал 14!

4366. Анюта, 6 марта 2013, 22:16:44
5) Брошена игральная кость. Найти вероятность того, что выпадет нечетное число очков.
Надо выписать так называемое пространство элементарных событий - что вообще может выпасть на игральной кости:
{1; 2; 3; 4; 5; 6}
В этом пространстве элементарных событий 6 элементов.

Теперь выпишем множество тех исходов, которые удовлетворяют условию задачи - нечётное количество очков:
{1; 3; 5}
В этом множестве 3 элемента.

Что значит "вероятность" в такого типа задачах? Это отношение того, что нужно, к тому, что может быть. Итак, искомая вероятность:

P{количество очков нечётное} = 3/6 = 1/2 = 0.5 = 50% = половина :)

4368. AleX@nder, 7 марта 2013, 18:03:04
4363. аделя, 5 марта 2013, 20:33:00
ПОМОГИ ПОЖАЛУЙСТА!!!!!!!!Основанием пирамиды MABCD является квадрат ABCD, ребро MD перпендикулярно к плоскости основания, AD = DM = a. Найдите площадь поверхности пирамиды.
Уууу... даже и не знаю- а можно ли тут делать чертежи? :-)

Если ребро MD перпендикулярно к плоскости основания, то обе прилегающих грани - прямоугольные треугольники. Это MCD и MAD, с прямым углом D. Эти треугольники будут равны (ибо их катеты равны a), площадь каждого из них равна a^2/2.

Остались две другие боковые грани - MBC и MBA. Блин, мне чего-то сильно кажется что они тоже прямоугольные. Почему? Да потому что MD был перпендикуляр к плоскости, и плоскости боковых граней MCD и MAD, через него проходящих, будут перпендикулярны к соответствующим сторонам квадрата. Грань MCD перпендикулярна AD (и соответственно BC) , а грань MAD перпендикулярна CD (и соответственно AB). Но боковое ребро MC лежит на грани MCD, значит оно перпендикулярно BC. Аналогично ребро MA лежит в плоскости грани MAD, и значит оно перпендикулярно AB. Тем самым треугольники MAB и MCB также прямоугольные и также равные.

Найдём MA (или MC - они равны). По теореме Пифагора это будет a корень из двух. Второй катет равен a (потому что это ребро основания). Площадь такого треугольника будет равна: S = (a * a * корень(2)) / 2. Таких треугольников тоже две штуки.

Итак, полная боковая поверхность:
S = a^2/2 + a^2/2 + (a^2 корень(2))/2 + (a^2 корень(2))/2 =
складывам дроби с одинаковым знаменателем и выносим а в квадрате за скобки
a^2 * (1 + корень(2))

А если вообще полная поверхность, то туда же ещё плюс основание: a^2, ведь в основании же квадрат, верно? :)
Тогда ответ будет: a^2 * (2 + корень(2))

Вот вроде бы и всё :-)
Могу ошибаться, если это так - прошу не серчать :-)

4369. AleX@nder, 7 марта 2013, 18:13:14
4362. Александр, 3 марта 2013, 21:30:37
Помогитеееее

В11
Объем правильной четырехугольной пирамиды равен 81. Высота разделена на 3 равные части, и через точки деления проведены плоскости параллельные основанию пирамиды. Найдите объем средней отсеченной части.
Ну тут, думаю, можно опереться на известный факт - линейные размеры пропорциональны коэффициенту подобия; площади пропорциональны квадрату коэффициента подобия, а объёмы - кубу коэффициента подобия.

Высота поделена на три равные части. Отношение высоты самой маленькой пирамиды к самой большой - 1:3. Тогда отношение их объёмов - 1:27. Если вся большая пирамида имеет площадь 81, то её маленькая пирамида-верхушечка имеет объём 81/27 = 3.

Теперь возьмём пирамиду побольше - с той же вершиной, но с основанием на отметке 2/3 от всей высоты. Её высота в два раза больше чем у пирамиды-верхушечки, и соответственно объём больше в 2 в кубе - в восемь раз. Объём этой "средней пирамиды" будет равен 3 * 8 = 24.

Ну вот и вся задача. Объём вредней части равен объём средней пирамиды минус объём пирамиды-верхушечки. Ответ: 24 - 3 = 21.

Вроде бы так :-)
Опять-таки - могу ошибаться - просьба не серчать, бумаги под рукой нет, решаю навскидку :D

4370. ксения, 12 марта 2013, 18:11:09
какой коринь с числа 17???

4371. AleX@nder, 12 марта 2013, 22:19:02
4370. ксения, 12 марта 2013, 18:11:09
какой коринь с числа 17???
Иррациональный! :)

Корень из 17 - это иррациональное число; оно непредставимо в виде конечной дроби. Это число чуть большее 4-х, и меньшее 5-и (потому что 4*4 = 16; и 5 * 5 = 25; а 17 лежит между 16 и 25). Корень из 17 - это четыре с чем-то...

Можно попытаться вычислить его более точно:
41 в квадрате = 1681 (почти 1700, разве нет?) :)
42 в квадрате = 1764 (заметный перебор)

4,1 ^ 2 = 16,81
4,2 ^ 2 = 17,64

Так что корень из 17 примерно равен 4,1схвостиком :)
(четыре целых, одна десятая и дальше сколько-то сотых, тысячных и т.д.)
Причём я думаю, что корень из 17 ближе к 4,1 ; нежели к 4,2 (то есть при округлении до десятых он тоже будет 4,1)

Точнее уже не могу - точнее уже может только калькулятор :)

4372. Алина, 14 марта 2013, 06:54:02
http://live.mephist.ru/tests/ege-math-2010-00/c05.gif

вопрос по этому заданию как его решить

4373. таня, 15 марта 2013, 12:56:24
дан параллепипедABCDA1B1C1D1.найти сумму векторов,а)AB+BC+CC1+C1D1

4374. Яна, 18 марта 2013, 20:19:30
дана правильная 4-х угольная пирамида,диагональ основания 8 см,высота 3 см.
Найти объем пирамиды?

4375. Яна, 18 марта 2013, 20:28:34
А..С1-прямоугольный параллелепипед,его диагональ 10 см,Угол наклона диагонали к плоскости основания = 30 градусов.
Найти диагональ основания.

4376. Вова, 19 марта 2013, 18:30:08
Концы отрезка AB лежат по разные стороны от прямой l. Расстояние от точни A до прямой l равно 24см , а расстояние от точки B до прямой l равно 30 см. Найдите расстояние от середины отрезка AB до прямой l. помогите пожалуйстса !

4377. Дмитрий, 19 марта 2013, 23:55:05
http://live.mephist.ru/show/mathege2010/view/id/263641/

"Правильный" ответ - 2, хотя я почему-то уверен, что 1.

4378. нюся, 28 марта 2013, 16:34:17
Основания пирамиды-ромб. Все ее боковые грани наклонены к плоскости основания под углом L. Найдите объем пирамиды, если сторона ромба равна B, а его острый угол равен L

4379. нюся, 28 марта 2013, 16:36:58
Диагональ боковой грани правильной четырёх угольной призмы равна 26см. Расстояние между непараллельными диагоналями противополоных боковых граней равна 10см. Вычислить объём призмы

4380. AleX@nder, 1 апреля 2013, 17:18:02
Сорри, и очень сильно извиняюсь, что с такой задержкой:

4373. таня, 15 марта 2013, 12:56:24
дан параллепипедABCDA1B1C1D1.найти сумму векторов,а)AB+BC+CC1+C1D1
Ну вообще-то сумма векторов - это новый вектор, который получится, если "начало" второго вектора "приставить" к "концу" первого. Попробуем проделать эту процедуру:

AB + BC => вышли из точки A в точку B, а потом вышли из точки B в точку C
в итоге получается что вышли из A и пришли в C =>
AB + BC = AC

AB + BC + CC1 = AC1
AB + BC + CC1 + C1D1 = AD1
Это будет диагональ боковой грани

4374. Яна, 18 марта 2013, 20:19:30
дана правильная 4-х угольная пирамида,диагональ основания 8 см,высота 3 см.
Найти объем пирамиды?
Тут надо воспользоваться формулой объёма пирамиды:

V = 1/3 * (площадь основания) * высоту
V = 1/3 Sh

А площадь основания можно найти исходя их формулы, что площадь четырёхугольника равна половине произведения диагоналей на синус угла между ними:
S = 1/2 d1 d2 * sin (d1 ^ d2)

Так как речь идёт о квадрате (пирамида правильная), то sin 90 град = 1

S = 1/2 d * d
V = 1/2 S * h
Из двух формул делаем одну:
V = 1/6 d * d * h = 1/6 * 8 * 8 * 3 = 32 см3

4375. Яна, 18 марта 2013, 20:28:34
А..С1-прямоугольный параллелепипед,его диагональ 10 см,Угол наклона диагонали к плоскости основания = 30 градусов.
Найти диагональ основания.
Гммм, ну наверное
AC : AC1 = cos 30 град

AC : AC1 = корень(3) : 2
AC : 10 = корень(3) : 2
Отсюда по пропорции:

AC = 10*корень(3) : 2
Похоже что
AC = 5 корень(3)

4376. Вова, 19 марта 2013, 18:30:08
Концы отрезка AB лежат по разные стороны от прямой l. Расстояние от точни A до прямой l равно 24см , а расстояние от точки B до прямой l равно 30 см. Найдите расстояние от середины отрезка AB до прямой l. помогите пожалуйстса !
Тут, честно говоря, очень трудно будет объяснить без рисунка. А можно ли вставлять рисунки - я не знаю (я не админ, а простой пользователь). Попробую пояснить задачу так, как сумею:

Если точки находились бы с одной стороны от прямой, то задача была бы на среднюю линию трапеции (расстояние от середины отрезка до прямой - это фактически средняя линия трапеции). И решалась бы она так:
(30 + 24) : 2 = 54 : 2 = 27 см

Так как точки с разных сторон от прямой, то надо брать не сумму, а разность:
(30 - 24) : 2 = 6 : 2 = 3 см.
Ответ: 3 см.
(без рисунка не смогу объяснить)

Можно остальное позже? :)))
Кстати всех с Первым Апреля (но решал я честно, без приколов) :))

4381. Тамара, 11 апреля 2013, 12:40:54
4347. Леонид, 8 января 2013, 17:13:09
Уважаемый, Lexxus! Помогите, пожалуйста, в решении этих задач! Заранее благодарю!
1) Две фабрики выпускают одинаковые стекла для автомобильных фар. Первая фабрика выпускает 45 этих стекол, вторая –– 55 . Первая фабрика выпускает 3 бракованных стекол, а вторая –– 1 . Найдите вероятность того, что случайно купленное в магазине стекло окажется бракованным.
2) Если гроссмейстер А. играет белыми, то он выигрывает у гроссмейстера Б. с вероятностью 0,52. Если А. играет черными, то А. выигрывает у Б. с вероятностью 0,3. Гроссмейстеры А. и Б. играют две партии, причем во второй партии меняют цвет фигур. Найдите вероятность того, что А. выиграет оба раза.
3)7) В чемпионате мира учавствуют 16 команд. С помощью жребия их нужно разделить на четыре группы по четыре команды в каждой. В ящике вперемешку лежат карточки с номерами групп:1, 1, 1, 1, 2, 2, 2, 2, 3, 3, 3, 3, 4, 4, 4, 4. Капитаны команд тянут по одной карточке. Какова вероятность того, что команда России окажется во второй группе?
4)На экзамене по геометрии школьнику достаётся один вопрос из списка экзаменационных вопросов. Вероятность того, что это вопрос на тему «Вписанная окружность», равна 0,2. Вероятность того, что это вопрос на тему «Параллелограмм», равна 0,15. Вопросов, которые одновременно относятся к этим двум темам, нет. Найдите вероятность того, что на экзамене школьнику достанется вопрос по одной из этих двух тем.
4347. Леонид, 8 января 2013, 17:13:09
Уважаемый, Lexxus! Помогите, пожалуйста, в решении этих задач! Заранее благодарю!
1) Две фабрики выпускают одинаковые стекла для автомобильных фар. Первая фабрика выпускает 45 этих стекол, вторая –– 55 . Первая фабрика выпускает 3 бракованных стекол, а вторая –– 1 . Найдите вероятность того, что случайно купленное в магазине стекло окажется бракованным.
2) Если гроссмейстер А. играет белыми, то он выигрывает у гроссмейстера Б. с вероятностью 0,52. Если А. играет черными, то А. выигрывает у Б. с вероятностью 0,3. Гроссмейстеры А. и Б. играют две партии, причем во второй партии меняют цвет фигур. Найдите вероятность того, что А. выиграет оба раза.
3)7) В чемпионате мира учавствуют 16 команд. С помощью жребия их нужно разделить на четыре группы по четыре команды в каждой. В ящике вперемешку лежат карточки с номерами групп:1, 1, 1, 1, 2, 2, 2, 2, 3, 3, 3, 3, 4, 4, 4, 4. Капитаны команд тянут по одной карточке. Какова вероятность того, что команда России окажется во второй группе?
4)На экзамене по геометрии школьнику достаётся один вопрос из списка экзаменационных вопросов. Вероятность того, что это вопрос на тему «Вписанная окружность», равна 0,2. Вероятность того, что это вопрос на тему «Параллелограмм», равна 0,15. Вопросов, которые одновременно относятся к этим двум темам, нет. Найдите вероятность того, что на экзамене школьнику достанется вопрос по одной из этих двух тем.

4382. Армин, 17 апреля 2013, 20:05:13
4377. Дмитрий, 19 марта 2013, 23:55:05
http://live.mephist.ru/show/mathege2010/view/id/263641/

"Правильный" ответ - 2, хотя я почему-то уверен, что 1.
4377. Дмитрий, 19 марта 2013, 23:55:05
http://live.mephist.ru/show/mathege2010/view/id/263641/

"Правильный" ответ - 2, хотя я почему-то уверен, что 1.
4377. Дмитрий, 19 марта 2013, 23:55:05
http://live.mephist.ru/show/mathege2010/view/id/263641/

"Правильный" ответ - 2, хотя я почему-то уверен, что 1.
4377. Дмитрий, 19 марта 2013, 23:55:05
http://live.mephist.ru/show/mathege2010/view/id/263641/

"Правильный" ответ - 2, хотя я почему-то уверен, что 1.
4359. Марианна, 26 февраля 2013, 18:06:15
В правильной треугольной пирамиде SABC ребра BA и BC разделены точками K и L, соответственно, в отношении 2:1, считая от вершины B. Найдите угол между плоскостью основания ABC и плоскостью сечения SKL. Ответ выразите в градусах
4359. Марианна, 26 февраля 2013, 18:06:15
В правильной треугольной пирамиде SABC ребра BA и BC разделены точками K и L, соответственно, в отношении 2:1, считая от вершины B. Найдите угол между плоскостью основания ABC и плоскостью сечения SKL. Ответ выразите в градусах

4383. алена, 19 апреля 2013, 10:00:15
Цена на электрический чайник была повышена на 23% и составила 1230 рублей. Сколько рублей стоил товар до повышения цены?


4384. Vikki, 20 апреля 2013, 19:24:19
У меня,например,не получается решать задачи на нахождение углов между плоскостями или прямой и плоскостью,хотя всю теорию знаю,а как до дела доходит,не получается ничего

4385. валерия, 21 апреля 2013, 12:42:54
В правильной треугольной пирамиде SABC ребра BA и BC разделены точками K и L, соответственно, в отношении 2:1, считая от вершины B. Найдите угол между плоскостью основания ABC и плоскостью сечения SKL. Ответ выразите в градусах

4386. 654, 21 апреля 2013, 17:08:56
прямая y=-4x-11 является касательной к графику функции 10/корень 116

4387. Наташа, 22 апреля 2013, 19:32:56
В правильной треугольной призме ABCA1B1C1 боковое ребро равно корень из 3 , а ребро основания равно 4. Точка-D середина ребра BB1 . Найдите объем пятигранника A1B1C1CD.

4388. Людмила, 29 мая 2013, 11:34:43
3448. Леонид , 10 апреля 2011, 15:52:16
производная у штрих = 3х^2 + 6x + 9 = 0. или х кв. + 2х + 3 = 0. Корней нет. Значения функции на концах отрезка У(-1) = - 34. У(4) = 64 + 48 + 36 -29 = 119. Что надо НАИБОЛЬШЕЕ или НАИМЕНЬШЕЕ, бери из 119 и -34.


неверное решение, так как не заметили в условии минус

4389. Ирина, 29 мая 2013, 13:43:48
Дан прямоугольный треугольник ABC с гипотенузой AC=16 см и катетом BC=12см .Отрезок SC=20 см,-перпендикуляр к плоскости ABC. а)Найдите /CS+CB+BA/(сумма векторов).б)Найдите угол между SA и плоскостью ABC.Помогите ребят

4390. 123, 1 июня 2013, 09:41:54
Помогите решить уравнение (38 +y)-18=31

4391. Светлыч, 2 июня 2013, 06:30:21
А почему бы не сделать в отрытом банке заданий егэ по математике задания части С?

4392. светлана, 3 июня 2013, 10:50:29
одна таблетка весит 20мги содержит 5% активного вещества.ребенку в возрасте до 6 месяцев врач выписывае 0,4 мг активного вещевства на каждый кг веса в сутки. сколько таблеток этого лекарства следует дать ребенку в возресте трех месяцев. и верно 5кг в течении суток

4393. Lexxus, 3 июня 2013, 10:58:59
4392. светлана, 3 июня 2013, 10:50:29
одна таблетка весит 20мги содержит 5% активного вещества.ребенку в возрасте до 6 месяцев врач выписывае 0,4 мг активного вещевства на каждый кг веса в сутки. сколько таблеток этого лекарства следует дать ребенку в возресте трех месяцев. и верно 5кг в течении суток

2 таблетки.
(5 кг * 0.4 мг/кг)/(20 мг * 0.05) = 2

4394. Уля, 6 июня 2013, 08:51:31
В основании пирамиды прямоугольник с диагональю 5 см, а одна сторона этого прямоугольника больше другой на 1 см. Высота пирамиды совпадает с меньшей стороны основания. Найдите объем пирамиды

4395. LLP, 14 июня 2013, 11:21:04
Lexxus,исправьте пожалуйста ответ к задаче 6989

4396. LLP, 14 июня 2013, 16:40:16
LEXXUS, почему в задаче 42575 ответ дан в км,а не в метрах, как спрашивается? надо бы устранить несоответствие...

4397. LLP, 16 июня 2013, 12:12:40
Lexxus, исправьте, пожалуйста, ответ к заданию 12737, - спрашивается меньший корень, а в ответе - больший!

4398. Lexxus, 16 июня 2013, 13:40:14
4396. LLP, 14 июня 2013, 16:40:16
LEXXUS, почему в задаче 42575 ответ дан в км,а не в метрах, как спрашивается? надо бы устранить несоответствие...

Ответ дан в метрах. Нужно внимательнее читать условие.

4397. LLP, 16 июня 2013, 12:12:40
Lexxus, исправьте, пожалуйста, ответ к заданию 12737, - спрашивается меньший корень, а в ответе - больший!

В ответе - единственный возможный корень.

4395. LLP, 14 июня 2013, 11:21:04
Lexxus,исправьте пожалуйста ответ к задаче 6989

Вот тут - да, ошибка была.

4399. LLP, 16 июня 2013, 14:56:59
Lexxus, в ответе к заданию 25569 ошибка!

4400. LLP, 16 июня 2013, 15:07:37
Lexxus, извините за мою невнимательность! -ответ к заданию 25569 ВЕРНЫЙ!

4401. LLP, 16 июня 2013, 17:59:04
Lexxus,в задании 245369 ответ 60 !?

4402. LLP, 19 июня 2013, 13:58:00
Lexxus, исправьте, пожалуйста, ответы к заданию 245369 и к заданию 19401.

4403. LLP, 19 июня 2013, 17:39:06
Lexxus, я тут пристаю к вам с исправлением ошибок, а вы, наверное, уже в отпуске? Проинформируйте, пожалуйста, к кому обращаться в таком случае?

4404. LLP, 20 июня 2013, 09:56:44
Когда задание из разряда "нерешённые" переходит в разряд "решённые" ?

4405. LLP, 20 июня 2013, 19:26:12
Lexxus, исправьте, пожалуйста, ошибку в ответе к заданию 14019.

4406. LLP, 20 июня 2013, 19:37:00
Lexxus, ответ к заданию 283699 до сотых НЕ ОКРУГЛЁН!

4407. 2ova, 20 июня 2013, 22:36:21
4406. LLP, 20 июня 2013, 19:37:00
Lexxus, ответ к заданию 283699 до сотых НЕ ОКРУГЛЁН!

Уважаемый LLP! Поскольку уважаемый Lexxus пока не счел возможным сразу ответить на Ваше обращение по задаче о сумках 283699, то отвечу я.

Русский язык крайне не логичен. Возьмем две близкие по структуре фразы: "На каждые 10 000 населения приходится 4 дальтоника" и "Очень жаль, что на десять девчонок по статистике девять ребят". В первом случае дальтоники затерялись среди населения, а во втором - ребята отдельно, девчата - отдельно.

Как и в задаче про сумки СО СКРЫТЫМИ (вот иезуиты!) дефектами (мухи-качественные сумки - отдельно, котлеты - сумки со СКРЫТЫМИ бля дефектами - отдельно).

Узнаю задачу: 2012 г., резерв. Сама на ней попалась. Но всё же правильный ответ на сайте: 0,9

Никому не говори, что я работала на проверке ЕГИ-2012 (((

4408. LLP, 21 июня 2013, 07:20:49
Zova,спасибо за внимание! И ВСЁ-ТАКИ, почему в 9 из 10 задачах с "сумками с дефектами" округление идёт по правилу? ТЕКСТЫ-ТО АБСОЛЮТНО АНАЛОГИЧНЫ!!!

4409. LLP, 21 июня 2013, 09:44:10
Lexxus, исправьте,пожалуйста, ошибку в ответе к заданию 7011. В ответе указана СУММА целых чисел, а запрашивается КОЛИЧЕСТВО!!!

4410. LLP, 23 июня 2013, 10:48:23
Lexxus,исправьте,пожалуйста,ошибку в ответе к заданию 6909.

4411. Lexxus, 23 июня 2013, 11:10:30
4409. LLP, 21 июня 2013, 09:44:10
Lexxus, исправьте,пожалуйста, ошибку в ответе к заданию 7011.

4410. LLP, 23 июня 2013, 10:48:23
Lexxus,исправьте,пожалуйста,ошибку в ответе к заданию 6909.

Спасибо, исправлено.

4412. LLP, 23 июня 2013, 14:16:20
Lexxus,в ответе к заданию 12785 указан несуществующий корень! Исправьте, пожалуйста, ошибку!

4413. LLP, 24 июня 2013, 11:31:48
Lexxus, при каких условиях задача из раздела "нерешённые" переходит в "решённые"?

4414. LLP, 27 июня 2013, 11:17:04
Lexxus,исправьте, пожалуйста, ошибку: ответ к заданию 28363 дан в км, а запрашавается в метрах!!!

4415. LLP, 28 июня 2013, 21:05:49
Lexxus, исправьте, пожалуйста, ошибку в ответе к заданию 32837.

4416. LLP, 29 июня 2013, 17:24:41
Lexxus, исправьте, пожалуйста, ошибку: в ответе к заданию 12799 указан несуществующий корень!!!

4417. LLP, 1 июля 2013, 14:11:21
КТО-НИБУДЬ КРОМЕ LEXXUSА ИСПРАВЛЯЕТ ОШИБКИ В ОТВЕТАХ???!!! АУ, МОДЕРАТОРЫ САЙТА, ПОЧЕМУ ТАКОЕ НЕВНИМАНИЕ К ПОСЕТИТЕЛЮ?

4418. Леонид, 30 июля 2013, 20:05:52
Народ! Переходите на сайт Дмитрия Гущина - молодцы! Здесь перестали реагировать на просьбы! А были в начале не заменимыми помощниками!!!

4419. Леонид, 17 августа 2013, 14:04:50
МИФИСТЫ - устраните безобразия в открытом банке заданий ЕГЭ - ужасная путаница В3,6,9,11. Пример, как не надо делать.

4420. Яков, 22 сентября 2013, 01:32:46
Задание B3 (23121)
(показов: 613, ответов: 34)

Найдите площадь трапеции, изображенной на рисунке.



Верный ответ пока не определен

ОН И НЕ МОЖЕТ БЫТЬ ОПРЕДЕЛЕН. СЕРЫМ ВЫДЕЛЕН ПАРАЛЛЕЛОГРАММ. А ТРАПЕЦИЙ ТАМ ДЕСЯТОК;)))

4421. Lexxus, 23 сентября 2013, 16:06:19
4412. LLP, 23 июня 2013, 14:16:20
Lexxus,в ответе к заданию 12785 указан несуществующий корень! Исправьте, пожалуйста, ошибку!
4414. LLP, 27 июня 2013, 11:17:04
Lexxus,исправьте, пожалуйста, ошибку: ответ к заданию 28363 дан в км, а запрашавается в метрах!!!
4415. LLP, 28 июня 2013, 21:05:49
Lexxus, исправьте, пожалуйста, ошибку в ответе к заданию 32837.
4416. LLP, 29 июня 2013, 17:24:41
Lexxus, исправьте, пожалуйста, ошибку: в ответе к заданию 12799 указан несуществующий корень!!!

Всё исправлено.
4420. Яков, 22 сентября 2013, 01:32:46
ОН И НЕ МОЖЕТ БЫТЬ ОПРЕДЕЛЕН. СЕРЫМ ВЫДЕЛЕН ПАРАЛЛЕЛОГРАММ

Иногра в определении трапеции не уточняется, что вторые две стороны не должны быть параллельны друг другу. В этом случае параллелограмм - частный случай трапеции.

4422. Кирилл Романов, 13 октября 2013, 19:00:48
помагите решить задачу за 5 клас номер 117

4423. Петя Соколов, 13 октября 2013, 19:03:06
помагите решить задачу за 5 класс номер 117

4424. ilyar, 15 октября 2013, 18:30:38
Когда вопросы переводятся из нерешенных в решенные?

4425. Lexxus, 18 октября 2013, 00:15:59
4424. ilyar, 15 октября 2013, 18:30:38
Когда вопросы переводятся из нерешенных в решенные?

Когда достаточное количество пользователей, правильно ответивших на уже решённые задания, дают один и тот же ответ. Это если в двух словах.

4426. Akzhan, 22 октября 2013, 14:10:24
Основанием пирамиды является прямоугольный треугольник с гипотенузой 26 см и катетом 24 см. Ребро, проходящее через их общую вершину, является высотой пирамиды и равно 18 см. Найдите боковую поверхность пирамиды

4427. ilyar, 23 октября 2013, 16:52:03
4426. Akzhan, 22 октября 2013, 14:10:24
Основанием пирамиды является прямоугольный треугольник с гипотенузой 26 см и катетом 24 см. Ребро, проходящее через их общую вершину, является высотой пирамиды и равно 18 см. Найдите боковую поверхность пирамиды

Может, вам надо найти общую площадь боковой поверхности? Тогда для начала найдем второй катет, равный 10 см. Первая боковая поверхность - прямоугольный треугольник с катетами 18 и 24. Площадь первого - 216 см^2. Вторая - тоже прямоугольный треугольник, но катеты у него - 10 и 24 и площадь у него - 120 см^2. Найдем гипотенузы этих двух треугольников, являющихся сторонами третьего. Они равны 30 и 26. После нахождения третьей площади найдите сумму площадей. Это и будет ответом.

4428. Alfiya, 23 октября 2013, 18:32:45
.Найти площадь полной поверхности правильной треугольной пирамиды,если её апофема равна 4см,а угол между апофемой и высотой пирамиды равен 30.

4429. Alfiya, 23 октября 2013, 18:35:31
НАйдите площадь полной поверхности куба, если расстояние от вершины верхнего основания куба до центра нижнего основания равно р

4430. Alfiya, 24 октября 2013, 19:22:16
найдите площадь полной поверхности правильной треуголной пирамиды,если двугранный угол при стороне основания равен 30, а радиус окружности, описанной около основания, равен 2

4431. Олеся, 27 октября 2013, 20:21:10
Куб АВСДА1В1С1Д1 ДМ=МД1. Найти угол между прямыми АД1 и ВМ

4432. ПАКК, 5 ноября 2013, 14:22:47
в трапеции abcd sina корень из 21/5 найдите косинус угла b

4433. Lev, 7 ноября 2013, 15:50:04
Добрый день! Пару дней назад возникла проблема по входу в систему Тесты Егэ, для решения задач по математике ЕГЭ 2014. С нескольких аккаунтов система пишет, что логин или пароль неверные. Что делать? Спасибо.

4434. Z.l.a.t.a, 8 ноября 2013, 20:09:26
В треугольнике ABC стороны AC и BC равны, AH - высота, cos угла BAC=26//5.Найдите cos угла BAH. Помогите пожалуйста:)

4435. Lexxus, 10 ноября 2013, 22:55:49
4433. Lev, 7 ноября 2013, 15:50:04
Добрый день! Пару дней назад возникла проблема по входу в систему Тесты Егэ, для решения задач по математике ЕГЭ 2014. С нескольких аккаунтов система пишет, что логин или пароль неверные. Что делать? Спасибо.

Это была одна из ошибок, вызванных обновлением программного обеспечения на сервере. Теперь всё должно работать.

4436. сява , 5 декабря 2013, 19:41:40
а умножить на 2 100 меньше да 54 минус равно икс нет плюс 18 равно икс

4437. Оксана, 8 декабря 2013, 07:45:56
Найдите диагональ куба ABCDA1B1C1D1 и двугранный угол A1CDB, если ребро куба равно 5см.

4438. Юлия, 9 декабря 2013, 19:48:05
Ребят, никак не могу решить задачу:Найдите величину двугранного угла при основании правильной четырехугольной пирамиды, если ее боковые ребра наклонены к плоскости основания под углом 60 градусов.

4439. Lexxus, 11 декабря 2013, 13:25:53
4438. Юлия, 9 декабря 2013, 19:48:05
Найдите величину двугранного угла при основании правильной четырехугольной пирамиды, если ее боковые ребра наклонены к плоскости основания под углом 60 градусов.

Допустим, пирамида - SABCD, ABCD - основание, O - точка пересечения диагоналей основания, M - середина ребра AB.
Нам известно, что угол SAO равен 60 градусов. То есть SO/AO = tg(60 градусов) = sqrt(3).
А нужно нам найти угол SMO, тангенс которого равен SO/MO.
MO = AO/sqrt(2) (как сторона и диагональ квадрата, соответственно).

Поэтому SO/MO = SO/AO*sqrt(2) = sqrt(3)*sqrt(2) = sqrt(6).
Ну, а искомый угол - соответственно, равен arctg(sqrt(6)) (примерно 67.8 градуса).

4440. Kattu22, 20 декабря 2013, 00:10:09
Lexxus, в открытом банке в этом году произошли изменения. Добавили еще одно задание, вернее В1 разбили на 2 задания и добавили туда заданий, многие задания поменялись местами и пополнились новыми заданиями. Возможно ли эти изменения внести на этот сайт

4441. Lexxus, 20 декабря 2013, 14:32:24
4440. Kattu22, 20 декабря 2013, 00:10:09
Lexxus, в открытом банке в этом году произошли изменения. Добавили еще одно задание, вернее В1 разбили на 2 задания и добавили туда заданий, многие задания поменялись местами и пополнились новыми заданиями. Возможно ли эти изменения внести на этот сайт

Да, я знаю об изменениях. Отчаянно пытаюсь выделить время, чтобы актуализировать данные на сайте.

4442. Kattu22, 22 декабря 2013, 09:55:01
Lexxus, опять перестали работать Тесты ЕГЭ. При вводе логина и пароля выбрасывает на регистрацию, но при регистрации нового аккаунт выбрасывает на начало регистрации.

4443. Lexxus, 22 декабря 2013, 21:34:32
4442. Kattu22, 22 декабря 2013, 09:55:01
Lexxus, опять перестали работать Тесты ЕГЭ. При вводе логина и пароля выбрасывает на регистрацию, но при регистрации нового аккаунт выбрасывает на начало регистрации.

Исправлено.

4444. Ananas, 24 декабря 2013, 22:20:17
В параллелепипеде ABCDA1B1C1D1 основание ABCD — квадрат со стороной, равной 8, остальные грани — прямоугольники. Боковое ребро равно 3, K — середина A1D1 Постройте сечение параллелепипеда плоскостью, проходящей через точки В, D, К, и найдите периметр сечения.
Выберите один ответ:

22

23
10+12&#8730;2

12+10&#8730;2

4445. Злой, 25 декабря 2013, 18:25:23
4444. Ananas, 24 декабря 2013, 22:20:17
В параллелепипеде ABCDA1B1C1D1 основание ABCD — квадрат со стороной, равной 8, остальные грани — прямоугольники. Боковое ребро равно 3, K — середина A1D1 Постройте сечение параллелепипеда плоскостью, проходящей через точки В, D, К, и найдите периметр сечения.
Выберите один ответ:

22

23
10+12&amp;#8730;2

12+10&amp;#8730;2

Эта задача решается в уме, ленивый ты кусок говна!

4446. гость, 25 декабря 2013, 19:58:35
Ответ к задаче 19401 указан неверно! Ведь вольшинство решило верно!

4447. гость, 27 декабря 2013, 17:48:05
В задаче 19389 исправьте. пожалуйста. ответ!

4448. ГОСТЬ, 27 декабря 2013, 18:57:33
Я так понимаю, Lexxus тут главный по исправлению ошибок? Пересмотрите, пожалуйста ответ к задаче 245369.

4449. ГОСТЬ, 6 января 2014, 18:35:53
Ответ к задаче 283651 указан неверно! Правильный ответ 0,93. Почему такое невнимание к посетителям сайта? И почему за три месяца вы так и не перешли к новой версии? Наверное, поэтому ( из-за вашего невнимания и некомпетентности) количество посетителей упало в разы по сравнению с прошлым годом. Очень жаль, ибо поначалу ваш сайт был одним из лучших по подготовке к ЕГЭ.

4450. ГОСТЬ, 9 января 2014, 15:57:40
Ответ к задаче 7127 указан неверно! верный ответ 8!

4451. ГОСТЬ, 15 января 2014, 20:03:16
А-А-А-У-У-У! Я ЧТО, ПОСЛЕДНИЙ ПОСЕТИТЕЛЬ НА ЭТОМ САЙТЕ? ВСЕ КАК-БУДТО ВЫМЕРЛИ... А МОЖЕТ ПРАВДА ВСЕ УЖЕ СЛИНЯЛИ С ЭТОГО САЙТА? ДА И МОДЕРАТОР ЧТО-ТО В ПОЛНОЙ ОТКЛЮЧКЕ...

4452. ДонКихот, 20 января 2014, 10:28:11
К задаче 282857 дан неверный ответ! Правильный ответ 0,92

4453. Lexxus, 26 января 2014, 21:57:13
4446. гость, 25 декабря 2013, 19:58:35
Ответ к задаче 19401 указан неверно! Ведь вольшинство решило верно!
4447. гость, 27 декабря 2013, 17:48:05
В задаче 19389 исправьте. пожалуйста. ответ!
4448. ГОСТЬ, 27 декабря 2013, 18:57:33
Я так понимаю, Lexxus тут главный по исправлению ошибок? Пересмотрите, пожалуйста ответ к задаче 245369.

Эти ответы исправлены.

4454. Lexxus, 26 января 2014, 22:03:04
4450. ГОСТЬ, 9 января 2014, 15:57:40
Ответ к задаче 7127 указан неверно! верный ответ 8!

Этот тоже исправлен.

4449. ГОСТЬ, 6 января 2014, 18:35:53
Ответ к задаче 283651 указан неверно! Правильный ответ 0,93. Почему такое невнимание к посетителям сайта? И почему за три месяца вы так и не перешли к новой версии? Наверное, поэтому ( из-за вашего невнимания и некомпетентности) количество посетителей упало в разы по сравнению с прошлым годом. Очень жаль, ибо поначалу ваш сайт был одним из лучших по подготовке к ЕГЭ.

А вот тут - в очередной раз убеждаюсь, что чем больше человек говорит, тем хуже умеет слушать. Я много раз объяснял, как решать подобные задачи.

И да, с сегодняшнего дня сервисы актуализированы.

4455. Kattu22, 27 января 2014, 00:03:25
Lexxus, задания переставили местами и теперь задания на вашем сайте не соответствуют нумерации открытого банка

4456. Lexxus, 27 января 2014, 10:12:51
4455. Kattu22, 27 января 2014, 00:03:25
Lexxus, задания переставили местами и теперь задания на вашем сайте не соответствуют нумерации открытого банка

Да, действительно, это я не заметил.

Тут мне требуется небольшая помощь. Насколько я понимаю (поправьте меня, если ошибаюсь), изменения выглядят так:
B1, B2, B3, B9, B14 и B15 - сейчас правильные.
(было)->(стало)
B4->B5
B5->B4
B6->B7
B7->B8
B8->B11
B10-> кажется, такие задания вообще исключены
B11->B6
B12->частично B10, частично B13
B13->B12

Правильно?

4457. Kattu22, 27 января 2014, 18:36:28
Не все все так, как Вы написали. Изменения выглядят так:
В1 ( было В1 )
В2 ( было В1 )
В3 ( было В2 )
В4 ( не изменилось )
В5 ( было В3 )
В6 ( было В10 )
В7 ( было В5 )
В8 ( было В6 )
В9 ( было В8 )
В10 ( было В11 )
В11 ( было В7 )
В12 ( не изменилось )
В13 ( было В9 )
В14 ( было В13 )
В15 ( было В14 )
Если в чем-то нужна помощь, могу помочь.

4458. Kattu22, 27 января 2014, 18:45:20
Lexxus, еще надо исправить, если это возможно, ответы к задачам: 283671, 283699, 283897. Спасибо

4459. Lexxus, 27 января 2014, 22:21:15
4457. Kattu22, 27 января 2014, 18:36:28
Изменения выглядят так:

Хорошо. По смыслу я написал почти то же самое, просто под "было" имел в виду состояние своей базы на тот момент.
Теперь, кажется, всё правильно.
В банке появилось какое-то количество новых заданий, но их я добавлю позже.

4458. Kattu22, 27 января 2014, 18:45:20
Lexxus, еще надо исправить, если это возможно, ответы к задачам: 283671, 283699, 283897. Спасибо

283671, 283699 - да.
А в последнем номере вы, видимо, ошиблись. К 283897 система ещё не определила верный ответ, при том что находится на правильном пути.

4460. Kattu22, 27 января 2014, 22:26:16
Ой, да опечаталась, 263897

4461. Lexxus, 27 января 2014, 22:58:06
4460. Kattu22, 27 января 2014, 22:26:16
Ой, да опечаталась, 263897

Понял, исправил.

4462. Kattu22, 27 января 2014, 23:06:33
Lexxus, спасибо большое, я сейчас просмотрела много задач, все теперь на своих местах.

4463. Жанара, 30 января 2014, 18:52:02
можно давать после окончания теста полное решение задания?

4464. ДонКихот, 31 января 2014, 21:08:58
Kattu22 объясните, пожалуйста, почему к задаче 282857 дан ответ 0,93, а не 0,92, ведь там же ответ даже округлять не надо!

4465. ДонКихот, 31 января 2014, 21:19:21
Kattu22, и к задаче 283663 почему ответ 0,96 а не 0,95?

4466. Lexxus, 1 февраля 2014, 21:25:34
4464. ДонКихот, 31 января 2014, 21:08:58
Kattu22 объясните, пожалуйста, почему к задаче 282857 дан ответ 0,93, а не 0,92, ведь там же ответ даже округлять не надо!
4465. ДонКихот, 31 января 2014, 21:19:21
Kattu22, и к задаче 283663 почему ответ 0,96 а не 0,95?

Читаем, думаем, анализируем

4467. Тедей, 4 февраля 2014, 20:58:43
Основание прямой призмы АВСDА1В1С1D1 – параллелограмм АВСD. АВ = 12см, АD = 15см, ВАD = . Найдите объём призмы, если диагональ DС1 боковой грани равна 13см.

4468. ДонКихот, 4 февраля 2014, 21:23:43
Lexxus, спасибо! Ну наконец-то до меня Д-О-Ш-Л-О !!!

4469. Владислава, 19 февраля 2014, 22:04:49
Задание B13 (282071)

Найдите угол AD_2E многогранника, изображенного на рисунке. Все двугранные углы многогранника прямые. Ответ дайте в градусах.

b9_236_204.eps

Не пойму почему 60 градусов. Подскажите, пожалуйста!

4470. ГОСТЬ, 25 февраля 2014, 19:16:09
Владислава, тр-к АД_2Е - РАВНОСТОРОННИЙ!

4471. Kattu22, 28 февраля 2014, 11:19:04
Lexxus!!!! К сожалению при входе на сайт с любого браузера вылазит таблица, что эта страница используется для атак на компьютеры пользователей. Это все началось еще во вторник. Войти можно только с техники Apple. Можно ли с этим что-то сделать? В любом случае, заранее спасибо

4472. Kattu22, 10 марта 2014, 10:35:17
Lexxus! Опять сайт перестал работать с любого браузера, работает только с техники Apple. Можно ли с этим, что-то сделать? Спасибо

4473. ГОСТЬ, 14 марта 2014, 22:57:12
Kattu22, зайдите на сайт с GOOGL: там безопасность лучше, чем в Яндекс.

4474. ГОСТЬ, 1 апреля 2014, 18:10:08
Lexxus, исправьте,пожалуйста, ответ к задаче 7075. Правильный ответ 4.

4475. Lexxus, 6 апреля 2014, 18:05:47
4474. ГОСТЬ, 1 апреля 2014, 18:10:08
Lexxus, исправьте,пожалуйста, ответ к задаче 7075. Правильный ответ 4.

Спасибо, исправлено.

4476. XIII, 17 апреля 2014, 18:32:26
основанием пирамиды служит прямоугольный треугольник с гипотенузой 26 и катетом 24 ребро проходящее через их общую вершину является высотой пирамиды и равно 18 определите полную поверхность пирамиды

4477. ГОСТЬ, 17 апреля 2014, 20:11:30
Lexxus, исправьте, пожалуйста, ответ к задаче 19657. Правильный ответ -0,25.

4478. ГОСТЬ, 18 апреля 2014, 20:54:03
Lexxus, исправьте, пожалуйста, ответ к задаче 12743. Правильный ответ 2. (значения переменной х на множестве ОДЗ положительны)

4479. ГОСТЬ, 18 апреля 2014, 20:54:14
Lexxus, исправьте, пожалуйста, ответ к задаче 12743. Правильный ответ 2. (значения переменной х на множестве ОДЗ положительны)

4480. ГОСТЬ, 18 апреля 2014, 20:54:48
Lexxus, исправьте, пожалуйста, ответ к задаче 12743. Правильный ответ 2. (значения переменной х на множестве ОДЗ положительны)

4481. ГОСТЬ, 18 апреля 2014, 20:55:09
Lexxus, исправьте, пожалуйста, ответ к задаче 12743. Правильный ответ 2. (значения переменной х на множестве ОДЗ положительны)

4482. ГОСТЬ, 18 апреля 2014, 20:55:49
Lexxus, исправьте, пожалуйста, ответ к задаче 12743. Правильный ответ 2. (значения переменной х на множестве ОДЗ положительны)

4483. ГОСТЬ, 19 апреля 2014, 13:56:40
Lexxus, исправьте, пожалуйста,ответ к заданию 9571. Правильный ответ -1,25

4484. obts, 22 апреля 2014, 10:52:00
Lexxus, подскажите, куда можно сообщить о некорректных (ошибочных) условиях задач.

4485. obts, 22 апреля 2014, 11:01:02
4482. ГОСТЬ, 18 апреля 2014, 20:55:49
12743

в ОДЗ слово "положительные" надо заменить на "неотрицательные", и ОДЗ имеет ещё одну границу <=4
Тогда ОДЗ [0;4]

4486. Литвиненко, 22 апреля 2014, 14:12:23
Какой минимальный порог будет в 2014 году?

4487. Оксаночка97, 24 мая 2014, 16:36:23
в задаче 276315 в ответе корень из 502

4488. Вера Васильевна, 25 мая 2014, 08:32:45
В задания В2 перенесены задачи на проценты из В1. Исправьте, пожалуйста.

4489. ИМЯН, 26 мая 2014, 17:09:39
Ребяяяят,а как это решить правильно?Я накидала пару решений,но боюсь они неправильные

4490. ИМЯН, 26 мая 2014, 17:11:23
Ребяяяят,а как это решить правильно?Я накидала пару решений,но боюсь они неправильныеНайдите корень уравнения: frac{x+23}{x-1}=-5.

4491. ЛЛП, 31 мая 2014, 14:18:52
Lexxus, к заданию 21193 дан неверный ответ, правильный ответ 1105000, исправьте, пожалуйста.

4492. ГОСТЬ, 21 июня 2014, 16:40:28
Lexxus, исправьте, пожалуйста, ответ к заданию 14019, правильный ответ: -2,5.

4493. ГОСТЬ, 21 июня 2014, 20:00:19
Lexxus, исправьте, пожалуйста, ответ к заданию 21193. Правильный ответ 1105000.

4494. ГОСТЬ, 25 июня 2014, 08:28:54
Lexxus, исправьте, пожалуйста, ответ к заданию 12743, правильный ответ 2.

4495. ГОСТЬ, 25 июня 2014, 13:41:14
Lexxus,исправьте, пожалуйста, ответ к заданию 7063.Правильный ответ 3.

4496. ОЛЬГА, 27 июня 2014, 05:13:52
ЧТО НЕ ВСЕ РОЖДЕНЫ БЫТЬ УМНЫМИ ДО ТАКОЙ СТЕПЕНИ

4497. ОЛЬГА, 27 июня 2014, 05:14:58
РЕШИТЕ ПОЖАЛУЙСТА треугольник со сторонами 5см,5см и 8см .НАЙТИ ОБЪЁМ КОНУСА

4498. ОЛЬГА, 27 июня 2014, 05:17:58
В коробке имеются 45карандашей,10 из которых сломаны.Художник наудачу извлекает 5 карандашей.Найти вероятность того,что извлечённые карандаши сломаны.

4499. Lexxus, 27 июня 2014, 12:04:33
4492. ГОСТЬ, 21 июня 2014, 16:40:28
Lexxus, исправьте, пожалуйста, ответ к заданию 14019, правильный ответ: -2,5.
4493. ГОСТЬ, 21 июня 2014, 20:00:19
Lexxus, исправьте, пожалуйста, ответ к заданию 21193. Правильный ответ 1105000.
4494. ГОСТЬ, 25 июня 2014, 08:28:54
Lexxus, исправьте, пожалуйста, ответ к заданию 12743, правильный ответ 2.
4495. ГОСТЬ, 25 июня 2014, 13:41:14
Lexxus,исправьте, пожалуйста, ответ к заданию 7063.Правильный ответ 3.

Всё исправлено, спасибо.


4500. Мария, 16 июля 2014, 15:57:53
Задание B3 (212087) привело в состояние ступора:

на сколько увеличится количество оборотов мотора, если крутящий момент увеличить с 1500 до 2500 Нм.

А на картинке момент от 0 до 400 Нм и кривулька возрастает, постоянная, убывает.

ответ: 80
Что имелось в виду? Может картинку перепутали?

4501. Lexxus, 17 июля 2014, 11:56:18
4500. Мария, 16 июля 2014, 15:57:53
Задание B3 (212087) привело в состояние ступора:

на сколько увеличится количество оборотов мотора, если крутящий момент увеличить с 1500 до 2500 Нм.

А на картинке момент от 0 до 400 Нм и кривулька возрастает, постоянная, убывает.

ответ: 80
Что имелось в виду? Может картинку перепутали?

Нормальная там картинка, от 0 до 2500 растёт до 140, дальше полочка. А при 1500 оборотах там как раз 60 Нм.

Там вопрос в задаче неправильный. Должно быть "на сколько увеличится крутящий момент при увеличении оборотов", а там наоборот.

4502. Леонид, 4 сентября 2014, 18:36:00
Помогите пожалуйста решить уравнение x^5 + x^3 + x = - 42 Очень срочно нужно. СПАСИБО за помощь!

4503. Алла, 13 октября 2014, 17:07:07
Леонид, 4 сентября 2014, 18:36:00 цитата

Помогите пожалуйста решить уравнение x^5 + x^3 + x = - 42 Очень срочно нужно. СПАСИБО за помощь!

x^5 =- x^3 -x - 42
у=x^5 (производная функции 5x^4 - не отрицательная, т. е функция возрастающая)
у=- x^3 -x - 42 (производная функции -3x^2-1 - отрицательная, т. е функция убывающая)
Т.о., уравнение x^5 =- x^3 -x - 42 имеет не более одного корня
корень уравнения (-2). Найти корень можно по схеме Горнера или подбором и выполнить проверку.

4504. ГОСТЬ, 27 октября 2014, 08:53:13
Lexxus, исправьте,пожалуйста,ответ к заданию 77253, правильный ответ 360000.

4505. Kattu22, 27 октября 2014, 18:55:15
4504. ГОСТЬ, 27 октября 2014, 08:53:13

цитата
Lexxus, исправьте,пожалуйста,ответ к заданию 77253, правильный ответ 360000.

Это задание решено верно! надо правильно читать вопрос к заданию, там сказано по сентябрь, а значение 360000 соответствует октябрю, который не входит в промежуток по сентябрь

4506. Kattu22, 28 октября 2014, 10:14:26
Спасибо, увидел!

4507. ГОСТЬ, 8 ноября 2014, 14:07:07
Lexxus, исправьте,пожалуйста,ответ к заданию 12813, правильный ответ 8.

4508. ГОСТЬ, 10 ноября 2014, 22:06:14
Lexxus, на вашем сайте номера заданий не соответствуют новой версии,на которую все сайты перешли аж с середины сентября!

4509. Владимир агрошкал, 13 ноября 2014, 19:51:51
На рисунке точки А и В лежат в плоскости альфа.а С - в плоскости бэтта.Постройте линии пересечения плоскости АВС с плоскостями альфы и бэтты. Поясните


4510. шарк, 16 ноября 2014, 11:05:45
Уважаемый Lexxus!
Не хотелось заниматься писаниной, но, видимо, придётся. Последнее время всё больше обнаруживаются ошибки в задачах. которые не имеют ответа или ещё не решены. Посмотрите, пожалуйста, задачу N 38557.
С уважением, ШАРК.

4511. Kattu22, 16 ноября 2014, 14:11:50
Уважаемый ШАРК, номер 38557 решен ВЕРНО

4512. "ЯНАО", 17 ноября 2014, 18:55:47
К сведению модератора данного сайта: к заданию 8587 правильный ответ 4, и он явно просматривается при увеличенном графике!

4513. даша, 18 ноября 2014, 17:34:59
матем

4514. Kattu22, 25 ноября 2014, 10:41:03
Lexxus, в этом году опять поменяли демоверсию. Теперь там опять 14 заданий. Объединили В1 и В2. И убрали букву В, теперь просто числовая нумерация. Не могли бы вы поменять все это на сайте. Если нужна помощь, готова помочь.

4515. "ЯНАО", 8 декабря 2014, 10:42:20
Lexxus,или кто за него,исправьте,пожалуйста,ответ к заданию 82479, правильный ответ 7.

4516. ГОСТЬ, 26 декабря 2014, 21:34:13
Исправьте,пожалуйста, ответ к заданию 119987, - правильный ответ 7.

4517. "ЯНАО", 2 января 2015, 19:54:30
Модератор этого очень полезного для выпускников сайта, исправьте,пожалуйста, ответ к заданию 9571, правильный ответ: -1,25.

4518. ГОСТЬ, 6 января 2015, 22:57:05
Исправьте,пожалуйста,ответ к заданию 82479, правильный ответ: 7

4519. шарк, 10 января 2015, 22:28:47
СОВЕТ: заведи новый логин, и заново набирай баллы,а не ползи с таким маленьким % доверия.(ВСЕ С ТЕБЯ УГОРАЮТ ОТ СМЕХА!)

4520. Алла, 14 января 2015, 17:04:51
Здравствуйте!
Подскажите, пожалуйста, как решить №26237

4521. ГОСТЬ, 29 января 2015, 21:29:20
КТО ТЕПЕРЬ за Lexxus ? Исправьте,пожалуйста,ответ к заданию 15467, правильный ответ 610.

4522. ГОСТЬ, 4 февраля 2015, 08:04:09
КТО ТЕПЕРЬ ЗА Lexxus? Исправьте,пожалуйста, ответ к заданию 83527, правильный ответ 1070.

4523. Kattu22, 4 марта 2015, 23:11:05
Жаль, что такой хороший сайт перестал работать и ни кому нет дела, заниматься его продвижением и модераторством

4524. шарк, 9 марта 2015, 19:55:35
Абитура этого сайта, прочтите! Господа, которые хотят показать своим работодателям, что они не зря едят хлеб! Зачем вы отключили опцию, которая предупреждала, что данная задача уже вами решалась? Почему вы за три года не удврили палец о палец, чтобы прорешать задачи, на которые "у вас пока??? нет ответа?" Потеря , видимо, создателя этой программы и по-настоящему знающего человека Lexxussa, привела к бардаку и беспомощному барахтанью в "собственном соку". У меня скопилось масса документов, из которых следует, что сайт, который маскируется под именем МИФИ - одного из лучших наших вузов, перешёл к руководству какими-то случайными людьми - с якобы высокой самооценкой и ложным самомнением. А на проверку получается - полный отстой. Полностью пропала реакция на неправильные ответы, никаких объяснений для обращающихся за помощью. Абитура! Посмотрите, как вас дурят. Обратите внимание, что задают вопросы и отвечают иногда одни и те же люди, инициируя якобы диалоги. Что только стоит так называемый % доверия, который, слава богу, исчез, так как он говорит ни о чём. А люди ведутся на это, желая заработать "на шару" ни о чём не говорящие цифры. Честь имею. Шарк

4525. ГОСТЬ, 23 марта 2015, 21:28:11
ШАРКУ: 99% из вами сказанного - ПОЛНЕЙШАЯ ЧУШЬ !!! А % ДОВЕРИЯ КАК РАЗ ГОВОРИЛ О ВАШИХ НЕДОСТАТОЧНЫХ ЗНАНИЯХ, ПОЭТОМУ ВЫ И РАДУЕТЕСЬ ЕГО ИСЧЕЗНОВЕНИЮ!!! Сами себе противоречите - что же вы сами-то упорно продолжаете набирать "НИ О ЧЁМ НЕ ГОВОРЯЩИЕ ЦИФРЫ" и никак не остановитесь,вовсю пользуетесь временными течническими затруднениями сайта??? Вам как человеку преклонного возраста (как бы вы ни шифровались) должно быть стыдно!!!

4526. Ray Ray, 25 марта 2015, 12:55:57
Сайту уже 4 года и при этом полно нерешенных задач. Закройте его уже раз не в силах поменять что-либо

4527. ГОСТЬ, 25 марта 2015, 20:22:02
ШАРКУ ( Ray Ray ) : ты достал уже всех своим брюзжанием !!! Не нравиться, не доволен, - сделай всем милость - НЕ ЗАХОДИ НА ЭТОТ САЙТ !!! ( считать научись - сайту уже 6 лет )

4528. ГОСТЬ, 11 апреля 2015, 22:24:40
ОЧЕНЬ РАДЫ ВОССТАНОВЛЕНИЮ ЛЮБИМОГО НАИПОЛЕЗНЕЙШЕГО САЙТА !!! СПАСИБО !!!

4529. ГОСТЬ, 19 апреля 2015, 21:00:50
ШАРКУ: с твоей стороны КРАЙНЕ НЕПОРЯДОЧНО возвращаться на всячески обруганный тобою сайт!!!

4530. Ирина, 19 мая 2015, 16:55:32
Найдите значение выражения
4sin(П/2+альфа)+соs(П-альфа).если соs альфа=-0.9

4531. ГОСТЬ, 5 июня 2015, 12:19:17
Исправьте,пожалуйста,ответ к заданию 212084, правильный ответ 12.

4532. ГОСТЬ, 23 июня 2015, 07:03:52
МОДЕРАТОРУ САЙТА: Исправьте,пожалуйста,ответ к заданию 55489, правильный ответ 242.

4533. ГОСТЬ, 24 июня 2015, 16:34:32
МОДЕРАТОРУ САЙТА: Исправьте,пожалуйста ответ к заданию 9571, правильный ответ: -1,25.

4534. ГОСТЬ, 26 июня 2015, 11:43:17
МОДЕРАТОРУ САЙТА: Исправьте, пожалуйста,ответ к заданию 82479, правильный ответ: 7.

4535. Валерия, 9 декабря 2015, 23:00:31
Ребят,кто может объяснить норм тему сложных производных,производных,и поиск точек минимума и максимума . Без тупого рассасывания не понимаю совсем. Спасибо)

4536. МАри, 18 февраля 2016, 22:10:08
Помогите пожалуйста (по теме комплексные числа)

изобразите на комплексной плоскости:
а) середину отрезка соединяющие точки 1+2i и 3+2i
б) множество точек Z удовлетворяющих условие argZ=П/4
в) множество точек Z , удовлетворябщее условие |Z|<=3

4537. Влад, 22 февраля 2016, 21:34:20
Есть кто живой?

4538. Анастасия, 10 апреля 2016, 14:23:38
Здравствуйте! Я пишу дипломную работу на тему"Исследование методов заполнения базы учебных заданий". Если это возможно, не могли бы Вы ответить, на основании каких методов Вы заполняете систему МИФИст заданиями: ручное заполнение, автоматизированное, частично автоматизированное (или другой вариант)?

С уважением, Анастасия Кузьмина

4539. anna, 19 апреля 2016, 16:03:13
Здравствуйте! кто может мне помочь геометрия по теме прямоугольный треугольник .
Найдите углы,которые образует высота прямоугольного треугольника,проведенная к гипотенузе,с катетами , если один из острых углов треугольника равен 37 градусов.

4540. Человек, 20 сентября 2016, 01:14:04
&#128521;&#128521;ээ есть кто?

4541. Соня, 22 сентября 2016, 21:56:58
В треугольнике АВС косинус угла С = 2/(корень из 11). Проведена окружность, которая касается стороны АС, пересекает сторону ВС в двух точках, которые лежат от точки С на расстоянии 9 и 11. Найти радиус окружности.

4542. Соня, 22 сентября 2016, 21:58:28
Помогите, пожалуйста

4543. Астрахань , 22 марта 2017, 14:41:08
Гипотенуза ab равнобедренного треугольника abc равна 12см. Точка n - середина катета ac.найдите расстояние от точки n до прямой ab.

4544. Владимир, 28 июня 2017, 22:36:50
4539. anna, 19 апреля 2016, 16:03:13
Здравствуйте! кто может мне помочь геометрия по теме прямоугольный треугольник .
Найдите углы,которые образует высота прямоугольного треугольника,проведенная к гипотенузе,с катетами , если один из острых углов треугольника равен 37 градусов.

Ну это же простейшая задача, если один из углов прямоугольного треугольника равен 37 град., то другой равен 180-90-37=53 град., то есть два острых угла прямоуг. треугольника имеют меру 53 и 27 градусов соответственно. Поскольку эти углы общие с двумя новыми прямоугольными треугольниками, которые получились при проведении высоты в исходном прямоугольном треугольнике, находим градусную меру их острых углов подобным образом, как и неизвестный острый угол исходного прямоугольного треугольника.

4545. Владимир, 28 июня 2017, 23:24:01
4543. Астрахань , 22 марта 2017, 14:41:08
Гипотенуза ab равнобедренного треугольника abc равна 12см. Точка n - середина катета ac.найдите расстояние от точки n до прямой ab.

Поскольку треугольник прямоугольный и равнобедренный: 1) мы можем узнать все его углы - один из них 90 град., два других в сумме тоже 90 град., то есть два по 45 град.
2) можно легко узнать длину неизвестных сторон: 12 в квадрате = сумме вадратов катетов, так как треугольник равнобедренный, то две его стороны (в данном случае катеты) равны, тогда 12 в квадрате (то есть 144) = 72 (144/2) + 72 (144/2), откуда 12= корень из 72 + корень из 72 . Всё, нашли длину катетов. Поскольку точка n - середина одного из катетов, делим корень из 72 пополам, это то же самое что и корнь из дроби 72/4, сокращаем подкоренную дробь и получаем корень из 18 - это расстояние от точки n до ближайшего острого угла. Проводим из основания треугольника перпендикуляр к точке n и выясняем его длину. Для этого у нас есть: длина отрезка от острого угла до точки n- корень из 18 ; острый угол в 45 град., перпендикуляр с углом у основания 90 град. --- а это значит что отсекаемый перпендикуляром прямоугольный треугольник тоже равнобедренный, как и исходно данный, следовательно мы можем легко узнать длину перпендикуляра, который является катетом в полученом равнобедренном прямоугольном треугольнике и равен второму катету, а сумма квадратов этих катетов равна 18 (квадрату гипотенузе, которая является половиной стороны-катета в исходно данном треугольнике)), таким образом каждый катет равен 3 (корень из 9).Это и есть расстояние от точки n до прямой

4546. Абдужалил, 10 января 2018, 22:10:12
основание пирамиды равнобедренный треугольник боковая сторона равна 4 а основание равно 2. Боковые грани пирамиды составляют с плоскостью основания угол в 60 градусов. Найдите грани. Как найти? подскажите пожалуйста

4547. Елизавета, 19 февраля 2018, 16:57:34
Какой из координатных площадей принадлежит точка А(3;0;-4)? Помогите пажалуйста!

4548. Вапрос, 15 января 2019, 21:52:33
Коментарий скрыт (грубость — 18%)показать


Рейтинг блогов



 

откуда • куда • где • eureka!
Бездарно потраченное время:
105800 дней